Vous êtes sur la page 1sur 465

downloaded from www.medicalbr.

com
Get more medical books and resources at

www.medicalbr.com

downloaded from www.medicalbr.com


e Davidson's
Self-assessment in
Medicine

downloaded from www.medicalbr.com


Edited by

Deborah Wake

CD
MB ChB (Hans), BSc, PhD, Diploma Clin Ed, MRCPE
Clinical Reader, University of Edinburgh; Honorary Consultant
Physician, NHS Lothian, Edinburgh, UK

Patricia Cantley
MB ChB, FRCP, BSc Hans (Med Sci)
Consultant Physician, Midlothian Enhanced Rapid Response and
Intervention Team, Midlothian Health and Social Care Partnership
and also Royal Infirmary of Edinburgh and Midlothian Community
Hospital, Edinburgh, UK

II

::s
Edinburgh London New York Oxford
CD
Philadelphia
ELSEVIER St Louis Sydney 2018

downloaded from www.medicalbr.com


ELSEVIER
© 2018, Elsevier Limited All rights reserved.

No part of this publication may be reproduced or transmitted in any form or


by any means, electronic or mechanical, including photocopying, recording,
or any information storage and retrieval system, without permission in writing
from the publisher. Details on how to seek permission, further information
about the Publisher's permissions policies and our arrangements with
organizations such as the Copyright Clearance Center and the Copyright
Licensing Agency, can be found at our website: www.elsevier.com/
permissions.
This book and the individual contributions contained in it are protected
under copyright by the Publisher (other than as may be noted herein).

Notices
Practitioners and researchers must always rely on their own experience and
knowledge in evaluating and using any information, methods, compounds or
experiments described herein. Because of rapid advances in the medical
sciences, in particular, independent verification of diagnoses and drug
dosages should be made. To the fullest extent of the law, no responsibility is
assumed by Elsevier, authors, editors or contributors for any injury and/or
damage to persons or property as a matter of products liability, negligence or
otherwise, or from any use or operation of any methods, products,
instructions, or ideas contained in the material herein.

ISBN: 978-0-7020-7151-5
International ISBN: 978-0-7020-7145-4

!J~
Working together
to grow libraries in
Book Aid
International developing countries
www.elsevter.com • www.bookaid.org

Printed in Poland
Last digit is the print number: 9 8 7 6 5 4 3 2 I

Executive Content Strategist: Laurence Hunter


Cont~nt Development Specialist: Carole McMurray
Project Manager: Louisa Talbott
Design: Miles Hitchen
Illustration Manager: Nichole Beard
Illustrator: MPS North America LLC
Marketing Manager: Deborah Watkins

downloaded from www.medicalbr.com


Contents
Preface vii
Introduction ix
Contributors xi
Abbreviations XV

1. Clinical decision-making 1
2. Clinical therapeutics and good prescribing 6
3. Clinical genetics 14
4. Clinical immunology 22
5. Population health and epidemiology 28
6. Principles of infectious disease 32
7. Poisoning 37
8. Envenomation 46
9. Environmental medicine 51
1

10. Acute medicine and critical illness 54


11. Infectious disease 73
12. HIV infection and AIDS 96
13. Sexually transmitted infections 103
14. Clinical biochemistry and metabolic medicine 107
15. Nephrology and urology 115
16. Cardiology 132
17. Respiratory medicine 154
18. Endocrinology 185
19. Nutritional factors in disease 203
20. Diabetes mellit.us 212
21. Gastroenterology 225

downloaded from www.medicalbr.com


downloaded from www.medicalbr.com
/
Preface
This is the first edition of Davidson's Self-assessment in Medicine, designed as an accompanying
volume to the internationally renowned textbook Davidson's Principles and Practice of Medicine.
Since the original Davidson's was first published in 1952, it has acquired a large following of medical
students, doctors and health professionals. Alongside the success of the main textbook, a demand
has emerged for a complementary self-assessment book covering a broad range of general medicine
topics. Our new book uses typical clinical scenarios to test the reader. Each chapter is written by a
specialty expert and the contents follow the style and chapter layout of Davidson's. This book can
be used either independently or in conjunction with the main book.
This book has been built around modern educational principles and utilises a contemporary assess-
ment style, in line with current undergraduate and postgraduate teaching. It is designed to help and
support students in their final undergraduate years and in the early years after qualification. The style
is compatible with that used in modern postgraduate examinations across the world.
The clinical scenarios have been chosen to be suitable for clinicians at any stage in their career,
supporting ongoing professional development. Clinical reasoning and judgement are encouraged,
with questions mirroring the situations and presentations that clinicians will meet in their everyday
practice. The content is applicable to a global audience and is based on current evidence-based best
practice.
The modern physician needs not only a sound knowledge base but also the ability to apply that
understanding appropriately to individual patients. The vision of the editors is to create a resource
that stimulates readers to build and apply their clinical knowledge to real-life scenarios, resulting in
1
excellent patient -centred care.
Deborah Wake and Patricia Cantley
Edinburgh, 2018

downloaded from www.medicalbr.com


Introduction
This book offers a broad education through formative self-assessment in general internal medicine.
The majority of the questions have been designed around clinical scenarios, with a number of optional
answers offered to the question posed. In general, the 'best fit' answer is sought unless otherwise
stated. Full explanations are given as appropriate to assist the reader in their learning.
The questions aim to cover a wide range of topics, divided into specialist chapters in line with
Davidson's Principles and Practice of Medicine. The questions have in general been based on UK
clinical practice and pharmacology, but where appropriate generic drug names are used and the
underlying principles are applicable internationally. Whilst the answers given are in line with best evidence-
based clinical practice, patient choice and cultural factors should always be considered when applying
the learning in individual patients and situations.

How to use this book


This self-assessment book can be used either independently or in conjunction with Davidson's.!
Readers may find it useful to read the relevant section of the main textbook in advance of tacklin~
the self-assessment; or they can use it subsequently to explore the topic in greater detail.
The questions, followed by their corresponding answers, have been arranged in the same chapter
order as Davidson's. The chapters are free-standing and can be read independently in any order.
Some of the questions are based on accompanying clinical images and radiology. Where it is
appropriate to see the image in colour, it has also been reproduced in a colour photographic SE;ction
at the back of the book.
Normal Reference Ranges for tests have not been used within the questions or explanations, but
can be found in the laboratory reference range chapter, at the end of the book.
Standard abbreviations are found within the text and are generally explained at first use. A full list
of abbreviations can be found at the front of the book.

downloaded from www.medicalbr.com


r
i

Contributors
Anna Anderson MBChB, MRCP, PhD Harry Campbell MD, FRCPE, FFPH, FRSE
Specialist Registrar Diabetes and Endocrinology, Professor of Genetic Epidemiology and Public
Western General Hospital, Edinburgh, UK Health, Centre for Global Health Research, Usher
Institute of Population Health Sciences
Brian J Angus BSc (Hons), DTM&H, FRCP, and Informatics, University of Edinburgh,
MD, FFTM(Gias) Edinburgh, UK
Associate Professor, Nuffield Department of
Medicine, University of Oxford, UK C Fiona Clegg BSc (MedSci), MBChB,
MRCP (UK)
Quentin M Anstee BSc (Hons), MBBS, PhD, Clinical Lecturer in Gastroenterology, School of
MRCP, FRCP Medicine, Medical Sciences and Nutrition,
Professor of Experimental Hepatology, Institute University of Aberdeen, Aberdeen, UK
of Cellular Medicine, Newcastle University,
Newcastle upon Tyne, UK; Honorary Consultant Gavin Clunie BSc, MBBS, MD, FRCP
Hepatologist, Freeman Hospital, Newcastle upon Consultant Rheumatologist and Metabolic Bone I
1
Tyne NHS Hospitals Foundation Trust, Newcastle Physician, Cambridge University Hospitals NHS
upon Tyne, UK Foundation Trust, Addenbrooke's Hospital,
Cambridge, UK
Jennifer Bain MBChB, MRCP, FRCA, FFICM
Fellow in Vascular Anaesthesia, Scottish Lesley A Colvin MBChB, BSc, FRCA, PhD,
Thoraco-abdorninal & Aortic Aneurysm Service, FRCP (Edin), FFPMRCA
Royal Infirmary of Edinburgh, Edinburgh, UK ConsultanVHonorary Professor in Anaesthesia
and Pain Medicine, Department of Anaesthesia,
Leslie Burnett MBBS, PhD, FRCPA . Critical Care and Pain Medicine, University
Chief Medical Officer, Genome.One, of Edinburgh, Western General Hospital,
Garvan Institute of Medical Research, Edinburgh, UK
Darlinghurst, Sydney; Honorary Professor,
University of Sydney, Sydney Medical School, Bryan Conway MB, MRCP, PhD
Sydney; Conjoint Professor, UNSW, St Senior Lecturer, Centre for Cardiovascular
Vincent's Medical School, Darlinghurst, Science, University of Edinburgh; Honorary
Sydney, Australia Consultant Nephrologist, Royal Infirmary
Edinburgh, Edinburgh, UK
Mark Byers OBE, FRCGP, FFSEM, FIMC,
MRCEM Nicola Cooper MBChB, FAcadMEd, FRCPE,
Consultant in Pre-Hospital Emergeney Medicine, FRACP
Institute of Pre-Hospital Care, London, UK Consultant Physician, Derby Teaching Hospitals
NHS Foundation Trust; Honorary Clinical
Associate Professor, Nottingham University,
Division of Medical Sciences and Graduate Entry
Medicine, Nottingham, UK

downloaded from www.medicalbr.com


---,
xii • CONTRIBUTORS Ii
i
Dominic J Culligan BSc, MBBS, MD, FRCP, Sally H Ibbotson BSc (Hons), MBChB (Hons),
FRCPath MD, FRCP (Edin)
Consultant Haematologist and Honorary Professor of Photodermatology, Photobiology
Senior Lecturer, Aberdeen Royal Infirmary, Unit, Dermatology Department, University of
Aberdeen, UK Dundee, Dundee, UK

Ruth Darbyshire MB BChir, MA(Cantab) Sara J Jenks Bsc (Hons), MRCP, FRCPath
Specialty Trainee in Ophthalmology, Yorkshire Consultant in Metabolic Medicine, Department of
and Humber Deanery, Yorkshire, UK Clinical Biochemistry, Royal Infirmary of
Edinburgh, UK
Graham Dark MBBS, FRCP, FHEA
Senior Lecturer in Medical Oncology and Cancer Sarah Louise Johnston MB ChB, FCRP,
Education, Newcastle University, Newcastle upon FRCPath
Tyne, UK Consultant in Immunology & HIV Medicine,
Department of Immunology and Immunogenetics,
Richard J Davenport DM, FRCP (Edin), North Bristol NHS Trust, Bristol, UK
BM BS, BMedSci
Consultant Neurologist and Honorary Senior David E J Jones MA, BM BCh, PhD, FRCP
Lecturer, University of Edinburgh, Edinburgh, UK Professor of Uver Immunology, Institute of Cellular
Medicine, Newcastle University; Consultant
David Dockrell MD, FRCPI, FRCP (Gias), Hepatologist, Freeman Hospital, Newcastle upon
FACP Tyne, UK
Professor of Infection Medicine, MAC/University of
Edinburgh Centre for Inflammation Research, Peter Langhorne MBChB, PhD, FRCP (Gias),
University of Edinburgh, Edinburgh, UK Hon FRCPI
Professor of Stroke Care, Institute of
Emad EI-Omar BSc (Hons), MBChB, Cardiovascular and Medical Sciences, University
MD (Hons), FRCP (Edin), FRSE of Glasgow, Glasgow, UK
Professor of Medicine, St George and Sutherland
Clinical School, University of New South Wales, Stephen Lawrie MD (Hons), FRCPsych,
Sydney, Australia Hon FRCP (Edin)
Professor of Psychiatry, University of Edinburgh,
Sarah Fadden BA, MB BChir, FRCA Edinburgh, UK
Senior Registrar in Anaesthesia, Royal Infirmary of
Edinburgh, Edinburgh, UK John Paul Leach MD, FRCP
Consultant Neurologist, Institute of Neurological
Catriona M Farrell MBChB, MRCP (UK) Sciences, Glasgow; Head of Undergraduate
Specialist Registrar Endocrinology and Diabetes, Medicine, University of Glasgow, Glasgow, UK
Ninewells Hospital, Dundee, UK
Andrew Leitch MBChB, BSc (Hons), PhD,
Amy Frost MA (Cantab), MBBS, MRCP MSc (Ciin Ed), FRCPE (Respiratory)
Clinical Genomics Educator, Affiliated to St Consultant Respiratory Physician, Western
George's University NHS Foundation Trust, General Hospital; Honorary Senior Lecturer,
London, UK University of Edinburgh, Edinburgh, UK

Neil Grubb MD, FRCP Gary Maartens MBChB, FCP(SA), MMed


Cardiology Consultant, Royal Infirmary of Professor of Medicine, University of Cape Town,
Edinburgh; Honorary Senior Lecturer, Cape Town, South Africa
Cardiovascular Sciences, University of Edinburgh,
Edinburgh, UK Lucy Mackillop BM BCh, MA (Oxon), FRCP
Consultant Obstetric Physician, Oxford University
Jyoti Hansi Hospitals NHS Found01tion Trust; Honorary Senior
Department of Gastroenterology, Royal Infirmary Clinical Lecturer, Nuffield Departmentof
of Edinburgh, Edinburgh, UK Obstetrics and Gynaecology, University of Oxford,
Oxford, UK

downloaded from www.medicalbr.com


l
CONTRIBUTORS • xiii

Michael MacMahon MBChB, FRCA, FICM, David E Newby BA, BSc (Hons), PhD, BM,
EDIC OM, DSc, FMedSci, FRSE, FESC, FACC
Consultant in Anaesthesia and Intensive Care, British Heart Foundation John Wheatley Chair of
Victoria Hospital, Kirkcaldy, Fife, UK Cardiology, British Heart Foundation Centre for
Cardiovascular Science, University of Edinburgh,
Rebecca Mann BMedSci, BMBS, MRCP, Edinburgh, UK
FRCPCh
Consultant Paediatrician, Taunton and Somerset John Olson MD, FRPCE, FRCOphth
NHS Foundation Trust, Taunton, UK Consultant Ophthalmic Physician, Aberdeen
Royal Infirmary; Honorary Reader, University of
Lynn Manson MBChB, MD, FRCP, FRCPath Aberdeen, UK
Consultant Haematologist, Scottish National
Blood Transfusion Service, Department of Paul J Phelan MBBCh, MD,
Transfusion Medicine, Royal Infirmary of FRCP (Edin)
Edinburgh, Edinburgh, UK Consultant Nephrologist and Renal Transplant
Physician, Honorary Senior Lecturer, University of
Amanda Mather MBBS, FRACP, PhD Edinburgh, Royal Infirmary of Edinburgh,
Consultant Nephrologist, Department of Renal Edinburgh, UK
Medicine, Royal North Shore Hospital; Conjoint
Senior Lecturer, Faculty of Medicine, University of Eric M Przybyszewski BS, MD
Sydney, Sydney, Australia Resident Physician, Department of Medicine,
Massachusetts General Hospital, Boston, USA
Simon R Maxwell BSc, MBChB, MD, PhD,
FRCP, FRCPE, FHEA Stuart H Ralston MBChB, MRCP, FMedSci,
Professor of Student Learning/Clinical FRSE
Pharmacology & Prescribing, Clinical Professor of Rheumatology, Rheumatic Diseases
Pharmacology Unit, University of Edinburgh, Unit, University of Edinburgh, Edinburgh, UK
Edinburgh, UK
Jonathan Sandoe MBChB, PhD, FRCPath
David McAllister MBChB, MD, MPH, MRCP, Associate Clinical Professor, University of Leeds,
MFPH UK
Wellcorne Trust Intermediate Clinical Fellow
and Beit Fellow, Senior Clinical Lecturer in Gordon Scott BSc, FRCP
Epidemiology and Honorary Consultant in Consultant in Genitourinary Medicine, Chalmers
Public Health Medicine, University of Glasgow, Sexual Health Centre, Edinburgh, UK
Glasgow, UK
Alan G Shand MD, FRCP (Ed)
Mairi H Mclean BSc (Hons), MBChB (Hons), Consultant Gastroenterologist, Gastrointestinal
PhD, MRCP Unit, Western General Hospital, Edinburgh, UK
Senior Clinical Lecturer in Gastroenterology,
School of Medicine, Medical Sciences and Robby Steel MA, MD, FRCPsych
Nutrition, University of Aberdeen; Honorary Department of Psychological Medicine, Royal
Consultant Gastroenterologist, Digestive Disorders Infirmary of Edinburgh; Honorary (Clinical) Senior
Department, Aberdeen Royal Infirmary, Aberdeen, Lecturer, Department of Psychiatry, University of
UK Edinburgh, Edinburgh, UK

Francesca E M Neuberger MBChB, Grant D Stewart BSc (Hons),


MRCP (UK) FRCSEd (Urol), MBChB, PhD
Consultant Physician in Acute Medicine and University Lecturer in Urological Surgery,
Obstetric Medicine, Southrnead Hospital, Department of Surgery, University of Cambridge;
Bristol, UK Honorary Consultant Urological Surgeon,
Department of Urology, .A,Ddenbrooke's Hospital,
Cambridge; Honorary Senior Clinical Lecturer,
University of Edinburgh, Edinburgh, UK

downloaded from www.medicalbr.com


xiv • CONTRIBUTORS

David R. Sullivan MBBS, FRACP, FRCPA Henry Watson MBChB, MD


Clinical Associate Professor, Clinical Biochemistry, Consultant Haematologist, Aberdeen Royal
Royal Prince Alfred Hospital, Camperdown, NSW, Infirmary, Aberdeen, UK
Australia
Julian White MBBS, MD
Victoria Ruth Tallentire BSc (Hons), MD, Professor and Department Head, Toxinology
FRCP (Edin) Department, Women's & Children's Hospital,
Consultant Physician, Western General Hospital; North Adelaide, Australia
Honorary Clinical Senior Lecturer, University of
Edinburgh, Edinburgh, UK Miles D Witham BM BCh, PhD, FRCP (Ed)
Clinical Reader in Ageing and Health, Department
Simon H Thomas MD, FRCP of Ageing and Health, University of Dundee,
Professor of Clinical Pharmacoloy and Dundee, UK
Therapeutics, Medical Toxicology Centre,
Newcastle University, Newcastle upon Tyne, UK

Craig Thurtell BMedSci (Hons), MBChB


MRCP
Specialty Registrar, Department of Diabetes &
Endocrinology, Ninewells Hospital, Dundee, UK

downloaded from www.medicalbr.com


/
Abbreviations
11~-HSD II ~-Hydroxysteroid APL Acute promyelocytic leukaemia
dehydrogenase APS Antiphospholipid syndrome
1311
Radioisotope iodine-131 APTT Activated partial thromboplastin
2,3-DPG 2,3-Diphosphoglycerate time
20WBCT 20-Minute whole-blood clotting ARDS Acute respiratory distress
test syndrome
5-ASA 5-Aminosalicylic acid ART Antiretroviral therapy
5-HIAA 5-Hydroxyindoleacetic acid AS Ankylosing spondylitis
AAV ANCA-associated vasculitis AST Aspartate aminotransferase
ACE Angiotensin-converting enzyme ATCG Adenine, thymine, cytosine,
AChR Acetylcholine receptor guanine
ACPA Anti-citrullinated peptide antibody ATG Anti-thymocyte globulin
ACR Albumin: creatinine ratio ATN Acute tubular necrosis
ACTH Adrenocorticotrophic hormone AVNRT Atrioventricular nodal re-entrant
ADH Antidiuretic hormone, vasopressin tachycardia
ADP Adenosine diphosphate AVP Arginine vasopressin
ADR Adverse drug reaction AVRT Atrioventricular re-entrant
AED Antiepileptic drug tachycardia
AFLP Acute fatty liver of pregnancy axSpA Axial spondyloarthritis
AFP Alpha-fetoprotein BAL Bronchoalveolar lavage
AICTD Autoimmune connective tissue BCC Basal cell carcinoma
disease BCG Bacille Calmette-Guerin
AIDS Acquired immune deficiency BD Behget's disease
syndrome BiPAP Bi-level positive airway pressure
AIH Autoimmune hepatitis BMD Bone mineral density
AK Actinic keratosis BMI Body mass index
AKI Acute kidney injury BNP Brain natriuretic peptide
ALL Acute lymphoblastic leukaemia BP Blood pressure
ALP Alkaline phosphatase BPH Benign prostatic hypertrophy
ALT Alanine transaminase BPPV Benign paroxysmal positional
AMA Antimitochondrial antibody vertigo
AMD Age-related macular degeneration BRCA1 BReast CAncer genes I
AML Acute myeloid leukaemia BRCA2 BReast CAncer genes 2
ANA Antinuclear antibody Ca2+ Calcium
ANCA Antineutrophil cytoplasmic CA-MRSA Community-acquired. meticillin-
antibody resistant Staphylococcus aureus
anti-EMA Anti-endomysia! antibody CAH Congenital adrenal hyperplasia
anti-tTG Anti-tissue transglutaminase cAMP Cyclic ade~:~osine monophosphate
APC Argon plasma coagulation CAP Community-acquired pneumonia
APKD Autosomal dominant polycystic CBT Cognitive behavioural therapy
kidney disease CCF Congestive cardiac failure

downloaded from www.medicalbr.com


xvi . ABBREVIATIONS

CD4 Cluster of differentiation 4 DIPJ Distal interphalangeal joints


CDC Centers for Disease Control and DIT Diiodotyrosine
Prevention DKA Diabetic ketoacidosis
CF Cystic fibrosis DLBL Diffuse large B-cell lymphoma
CFTR Cystic fibrosis transmembrane DLQI Dermatology Life Quality Index
conductance regulator DM1 Myotonic dystrophy type 1
CGA Comprehensive Geriatric DMARD Disease-modifying antirheumatic
Assessment drug
CGH Comparative genomic DMSA Dimercaptosuccinic acid
hybridisation DNA Deoxyribonucleic acid
CGRP Calcitonin gene-related peptide DOAC Direct oral anticoagulant
ClOP Chronic inflammatory DPP-4 Dipeptidyl peptidase 4
demyelinating polyneuropathy ORE Digital rectal examination
CIM Critical illness myopathy DRESS Drug reaction and eosinophilia
CJD Creutzfeldt-Jakob disease with systemic symptoms
CK Creatine kinase DVT Deep vein thrombosis
CKD Chronic kidney disease DXA Dual X-ray absorptiometry
CLL Chronic lymphocytic leukaemia E,V,M Eye, verbal, motor (in Glasgow
CML Chronic myeloid leukaemia Coma Scale)
CMV Cytomegalovirus EBUS-FNA Endobronchial ultrasound-guided
CN Cranial nerve fine needle aspiration
CNS Central nervous system EBV. Epstein-Barr virus
CNV Copy number variant ECF Extracellular fluid
C02 Carbon dioxide ECF Epirubicin, cisplatin and
COL4A5 Collagen type IV alpha 5 chain fluorouracil (cancer chemotherapy
COPD Chronic obstructive pulmonary combination)
disease ECG Electrocardiography
cox Cyclo-oxygenase ECMO Extracorporeal membrane
CPAP Continuous positive airway pressure oxygenation
CPE Carbapenemase-producing ECT Electroconvulsive therapy
Enterobacteriaceae ED Erectile dysfunction
CPPD Calcium pyrophosphate disease EDso Median effective dose: the dose'
CPR Cardiopulmonary resuscitation that produces a quanta! effect (all
CRP C-reactive protein or nothing) in 50% of the
CAPS Complex regional pain syndrome population that takes it
CSF Cerebrospinal fluid EEG Electroencephalography
CT Computed tomography eGFR Estimated glomerular filtration rate
CT-PET CT positron emission tomography EGFR Epidermal growth factor receptor
CTKUB CT scan of kidneys, ureters and EIA Enzyme immunoassay
bladder ELISA Enzyme-linked immunosorbent
CTPA CT pulmonary angiogram assay
CTS Carpal tunnel syndrome EMG Electromyography
eve Central venous catheter ENA Extractable nuclear antigens
CVD Cardiovascular disease ENT Ear, nose and throat
CVP Central venous pressure EPO Erythropoietin
CXR Chest X-ray ERCP Endoscopic retrograde
CYP Cytochrome P cholangiopancreatography
DBS Deep brain stimulation ESR Erythrocyte sedimentation rate
DDAVP Desmopressin ESRD End-stage renal disease
DGI Disseminated gonococcal ESWL Extracorporeal shockwave
infection lithotripsy
DILl Drug-induced liver injury ET Essential tremor
OILS Diffuse inflammatory EUS Endoscopic ultrasound
lymphocytosis syndrome FAP Familial adenomatous polyposis

downloaded from www.medicalbr.com


1_
-----------------

~-
ABBREVIATIONS • xvii

FAST HUG Feeding, analgesia, sedation, HBeAg Hepatitis B e antigen


thromboprophylaxis, head of bed HBsAg Hepatitis B surface antigen
elevation, ulcer prophylaxis, glucose HBV Hepatitis B virus
control (mnemonic to help prevent HCC Hepatocellular carcinoma
intensive care complications) hCG Human chorionic gonadotrophin
FOG Fludeoxyglucose HCo"- Bicarbonate
FEV1 Forced expiratory volume in 1 HCV Hepatitis C virus
second HDL High-density lipoprotein
FFP Fresh frozen plasma HDV Hepatitis D virus
FHH Familial hypocalciuric HELLP Haemolysis, elevated liver
hypercalcaemia enzymes, low platelet count
Fi02 Fraction of inspired oxygen HER Human epidermal growth factor
FODMAP Fermentable oligosaccharides, receptor
disaccharides, monosaccharides HEV Hepatitis E virus
and polyols HG Hyperemesis gravidarum
FSGS Focal segmental HHS Hyperosmolar hyperglycaemic
glomerulosclerosis state
FSH Follicle-stimulating hormone HIT Heparin-induced
FVC Forced vital capacity thrombocytopenia
FXR Farnesoid X receptor HIV Human immunodeficiency virus
G-CSF Granulocyte colony-stimulating HIVAN HIV-associated nephropathy
factor HL Hodgkin lymphoma
G6PD Glucose-6-phosphate HLA Human leucocyte antigen
dehydrogenase HLH Haernophagocytic
GABA y-Arninobutyric acid lyrnphohistiocytosis
GAD Glutamic acid decarboxylase HMS Hyperrnobility syndrome
GBD Global Burden of Disease HNF Hepatocyte nuclear factor
GBL Gamma butyrolactone HPOA Hypertrophic pulmonary
GBM Glomerular basement membrane osteoarthropathy
GBS Guillain-Barre syndrome HPV Human papilloma virus
GCA Giant cell arteritis HRCT High-resolution CT
GCS Glasgow Coma Scale HSV Herpes simplex virus
GFR Glomerular filtration rate HTLV Human T-cell lyrnphotropic virus
GGE Genetic generalised epilepsies HUS Haemolytic uraemic syndrome
GGT y-Giutamyl transferase IA-2 Islet antigen 2
GH Growth hormone IABP Intra-aortic balloon pump
GHB Gamma hydroxybutyrate IARC International Agency for Research
Gl Gastrointestinal on Cancer
GIP Gastric inhibitory polypeptide lBO Inflammatory bowel disease
GIST Gastrointestinal stromal cell IBS Irritable bowel syndrome
tumour lCD Implantable cardiac defibrillator
GLP-1 Glucagon-like peptide-1 lCD International Classification of
GLUTs Glucose transporters Diseases
GnRH Gonadotrophin-releasing hormone ICF Intracellular fluid
GOAD Gastro-oesophageal reflux disease ICP Intracranial pressure
GPA Granulomatosis with polyangiitis ICS Inhaled corticosteroid
GVHD Graft-versus-host disease ICU Intensive care unit
H+ Hydrogen ion IOU Intravenous dru,9 user
HACE High-altitude cerebral osderna lg Immunoglobulin
HAP Hospital-acquired pneumonia lgA Immunoglobulin A
HAPE High-altitude pulmonary oedema lgE Immunoglobulin E
HAV Hepatitis A virus IGF Insulin-like growth factor
HbA1c Glycated haemoglobin lgG Immunoglobulin G
HBc Hepatitis 8 core antigen lgM Immunoglobulin M

downloaded from www.medicalbr.com


xviii . ABBREVIATIONS

IGRA Interferon-gamma release assay MERS-CoV Middle East respiratory syndrome


IIH Idiopathic intracranial hypertension coronavirus
ILD Interstitial lung disease Mg2+ Magnesium
IM Intramuscular MGUS Monoclonal gammopathy of
INN International non-proprietary name uncertain significance
INR International normalised ratio MHC Major histocompatibility complex
IPF Idiopathic pulmonary fibrosis Ml Myocardial infarction
IPSS International Prostate Symptom MIT Monoiodotyrosine
Score MM Multiple myeloma
IRIS Immune reconstitution MMF Mycophenolate mofetil
inflammatory syndrome MODY Maturity-onset diabetes of the
ITP Immune thrombocytopenia young
IV Intravenous MPA Microscopic polyangiitis
IVIg Intravenous immunoglobulins MRCP Magnetic resonance
JC virus John Cunningham virus cholangiopancreatography
JIA Juvenile idiopathic arthritis MRD Minimal residual disease
JVP Jugular venous pressure MRI Magnetic resonance imaging
K+ Potassium mRNA Messenger ribonucleic acid
Kco Carbon monoxide transfer MRSA Meticillin-resistant Staphylococcus
coefficient aureus
LABA Long-acting j3 2-agonist MS Multiple sclerosis
LADA Latent autoimmune diabetes of MSE Mental state examination
adulthood MSM Man who has sex with men
LAMA Long-acting muscarinic antagonist MSU Mid-stream urine
LDH Lactate dehydrogenase MTP Metatarsophalangeal
LDL Low-density lipoprotein MuSK Muscle-specific kinase
LEMS Lambert-Eaton myasthenic MVA Mosaic variegated aneuploidy
syndrome Na+ Sodium
LFTs Liver function tests NAD Nicotinamide adenine dinuciE)6tide
LH Luteinising hormone NAFLD Non-alcoholic fatty liver dise11se
LMWH Low-molecular-weight heparin NASH Non-alcoholic steatohepatitis
LR Likelihood ratio NFFC Non-front-fanged colubrid (snake)
LSD Lysosomal storage disease NGS Next -generation sequencing
LUL Left upper lobe NHL Non-Hodgkin lymphoma
LUTS Lower urinary tract symptoms NICE National Institute for Health and
MALT Mucosa-associated lymphoid Care Excellence
tissue NIV Non-invasive ventilation
MAP Mean arterial pressure NMDA N-methyl-o-aspartate
MCI Minimal cognitive impairment NMO Neuromyelitis optica
MCP Metacarpophalangeal NNRTI Non-nucleoside reverse
MCPJ Metacarpophalangeal joint transcriptase inhibitor
MCTD Mixed connective tissue disease NNT Number needed to treat
MCV Mean corpuscular volume NR Normalised ratio
MOP Methylene diphosphonate NRTI Nucleoside reverse transcriptase
MDRD Modification of Diet in Renal inhibitor
Disease NSAID Non-steroidal anti-inflammatory
MDS Myelodysplastic syndromes drug
MEGX Monoethylglycinexylidide NSIP Non-specific interstitial pneumonia
ME LAS Mitochondrial encephalopathy, 02 Oxygen
lactic acidosis and stroke-like OA Osteoarthritis
episodes OBMT Omeprazole, bismuth subcitrate,
MELD Model for End-Stage Liver metrooidazole and tetracycline
Disease OCD Obsessive-compulsive disorder
MEN Multiple endocrine neoplasia OCP Oral contraceptive pill
MERS Middle East respiratory syndrome OGD Oesophago-gastroduodenoscopy

downloaded from www.medicalbr.com


~-
ABBREVIATIONS • xix

OGTT Oral glucose tolerance test PTE Pulmonary thromboembolism


OPIDN Organophosphate-induced PTH Parathyroid hormone
delayed polyneuropathy PTLD Post-transplant lymphoproliferative
OSA Obstructive sleep apnoea disorder
PaC02 Partial pressure of carbon dioxide PTSD Post -traumatic stress disorder
in arterial blood PUO Pyrexia of unknown origin
pANCA Perinuclear antineutrophil PVD Posterior vitreous detachment
cytoplasmic antibody RA Rheumatoid arthritis
Pa0 2 Partial pressure of oxygen in RAAS Renin-angiotensin-aldosterone
arterial blood system
PARP Poly-ADP ribose polymerase RAPD Relative afferent pupillary defect
PAS I Psoriasis Area and Severity RBILD Respiratory bronchiolitis-interstitial
Index lung disease
PBC Primary biliary cirrhosis RFA Radiofrequency ablation
PBI Pressure bandage and RIC Reduced-intensity conditioning
immobilisation RNA Ribonucleic acid
PCI Percutaneous coronary ROSC Return of spontaneous
intervention circulation
PCNL Percutaneous nephrolithotomy ROSIER Rule Out Stroke In the Emergency
PCOS Polycystic ovary syndrome Room (clinical stroke tool)
PCP Pneumocystis pneumonia RPR Rapid plasma reagin
PCR Polymerase chain reaction rt-PA Recombinant tissue plasminogen
PD Parkinson's disease activator
PDB Paget's disease of bone RTA Renal tubular acidosis
PDT Photodynamic therapy RV Residual volume
PEA Pulseless electrical activity SAAG Serum-ascites albumin gradient
PEEP Positive end-expiratory pressure SABA Short -acting ~ 2 -agonist
PEFR Peak expiratory flow rate Sa02 Arterial oxygen saturation
PEP Post-exposure prophylaxis SARS Severe acute respiratory
PET Positron emission tomography syndrome
PHT Pulmonary hypertension SBP Spontaneous bacterial peritonitis
PIP Proximal interphalangeal sec Squamous cell carcinoma
PIPJ Proximal interphalangeal joints SCLC Small cell lung cancer
PI Protease inhibitor SCRA Synthetic cannabinoid receptor
PKD Polycystic kidney disease agonist
75
PLE Polymorphic light eruption SeHCAT Se-homocholic acid taurine
PMF Progressive massive fibrosis SGLT2 Sodium and glucose
PMR Polymyalgia rheumatica co-transporter 2
P0 2 Partial pressure of oxygen SHBG Sex hormone-binding globulin
POCT Point -of-care test SIADH Syndrome of inappropriate
POEM Peroral endoscopic myotomy antidiuretic hormone (vasopressin)
POMC Pro-opiomelanocortin secretion
PPARy Peroxisome proliferator-activated SIJ Sacroiliac joint
receptor gamma SLE Systemic lupus erythematosus
PPCI Primary percutaneous coronary 502 Saturation of haemoglobin with
intervention oxygen
PPI Proton pump inhibitor SOFA Sequential Organ Failure
PRV Polycythaemia rubra vera Assessment
PSA Prostate-specific antigErn SpA Spondyloarthritis
PsA Psoriatic arthritis SPC Summary of product
PSC Primary sclerosing cholangitis characteristics
PSP Primary spontaneous SPECT Single-photQn emission computed
pneumothorax tomography
PSS Primary Sjogren's syndrome Sp0 2 Peripheral capillary oxygen
PT Prothrombin time saturation

downloaded from www.medicalbr.com


l
XX . ABBREVIATIONS

SScl Systemic sclerosis TPPA Treponema pa/lidum particle


SSRI Selective serotonin re-uptake agglutination assay
inhibitor TRAbs TSH receptor antibodies
STI Sexually transmitted infection TRM Treatment-related mortality
SVR Sustained viral response tRNA Transfer ribonucleic acid
Ta Triiodothyronine TSH Thyroid-stimulating hormone
T. Thyroxine TTP Thrombotic thrombocytopenic
TAe Trigeminal autonomic purpura
cephalalgia UDeA Ursodeoxycholic acid
TAeE Transarterial chemoembolisation UFH Unfractionated heparin
TB Tuberculosis UMOD Uromodulin
TBG Thyroxine-binding globulin uss Ultrasound scan
Teo Transfer factor for carbon UVB Ultraviolet B
monoxide 'iftO. Ventilation-perfusion
TEN Toxic epidermal necrolysis V2 Vasopressin 2
TFTs Thyroid function tests VAP Ventilator-associated pneumonia
TGA Transient global amnesia v" Volume of distribution
TGF Transforming growth factor VEGF Vascular endothelial growth factor
TIA Transient ischaemic attack VGee Voltage-gated calcium channel
TIPSS Transjugular intrahepatic VIP Vasoactive intestinal peptide
portosystemic stent shunt VLDL Very low-density lipoprotein
TKI Tyrosine kinase inhibitor VSD Ventricular septal defect
TKR Total knee replacement VTE Venous thromboembolism
TNF Tumour necrosis factor vWD von Willebrand disease
TNM System used in cancer staging: vWF von Willebrand factor
T = size and extent of the main/ vWF:Ag von Willebrand factor antigen
primary tumour; N = number vzv Varicella zoster virus
of nearby lymph nodes involved; wee White cell count
M = metastasis WHO World Health Organization
TPOs Thyroid peroxidise antibodies ZnT8 Zinc transporter 8

downloaded from www.medicalbr.com


---

N Cooper

Clinical decision-making
Multiple Choice Questions
1.1. In the specialty of internal medicine, 1.4. A test is performed to detect the presence
diagnostic error occurs in approximately what of a disease in a specific population. The
percentage of cases? results of the test can be summarised in the
A. 0-5% table below.
B. 6-10%
C.11-15% Disease No disease
D. 16-20% Positive test A B
E. 21-25% Negative test c D

1.2. A doctor is considering whether a patient Which of the following describes the positive
presenting with headache, fever and nuchal predictive value of the test?
rigidity may have meningitis. Regarding A. A/(A+B) X 100
likelihood ratios (LRs) for each clinical finding, B. A/(A+C) X 100
which of the following statements is true? c. A/(A+D) X 100
A. An LR greater than 1 decreases the D. D/(D+B) X 100
probability of disease E. D/(D+C) X 100
B. An LR greater than 1 increases the
probability of disease 1.5. An elderly woman fell and hurt her left hip.
C. An LR is the probability of the finding in On examination the left hip was extremely
patients with the disease painful to move and she was unable to stand.
D. An LR of 0 means the diagnosis is unlikely The pre-test probability of a hip fracture was
E. An LR of 1 means the diagnosis is certain deemed to be high. Plain X-rays of the pelvis
and left hip were requested.
1.3. A test is performed to detect the presence Which of the following statements best
of a disease. The results of the test can be describes 'post -test probability'?
summarised in the table below. A. The adjustment of probability after
Disease No disease I taking individual patient factors in
Positive test A B I to account
Negative test C D I B. The chance that a test will detect true
positives
Which of the following describes the C. The prevalence of disease in the population
sensitivity of the test? to which the patient belongs
A. A/(A+B) X 100 D. The probability of a disease after taking
B. A/(A+C) X 100 new information from a test result into
c. A/(A+D) X 100 account
D. D/(D+B) X 100 E. The proportion of patients with a test res,ult
E. D/(D+C) X 100 who have the disease

downloaded from www.medicalbr.com


T
2 • CLINICAL DECISION-MAKING
I
1.6. A doctor is considering whether to treat a examination carries the most diagnostic weight I
patient with antibiotics for a urinary tract in either a positive or negative direction? I
infection. The term 'treatment threshold' A. Blood pressure greater than 120/80 mmHg
describes a situation in which various factors B. Heart rate less than 90 beats/min
I
!
are evenly weighted. What is the best C. Oxygen saturations greater than 94% on air I
description of the factors involved? D. Respiratory rate less than 20breaths/min
A. The cost of the treatment, and whether the E. Temperature less than 37.5°C
treatment is likely to succeed
B. The quality of life of the patient, and risks 1.11. Which of the following statements best
and benefits of treatment describes 'patient-centred evidence-based
C. The risk and benefits of treatment medicine'?
D. The risks of the test, and risk and benefits of A. The application of best available evidence
treatment taking individual patient factors into account
E. The wishes of the patient, and whether the B. The application of best available evidence to
treatment is likely to succeed patient care
C. The application of clinical decision aids in
1.7. Dual process theory describes two distinct decision-making
processes of human decision-making. What is D. The implementation of a management plan
the accepted estimate of the proportion of time based on patient wishes
we spend engaged in type 2 (analytical) E. The use of evidence-based care bundles
thinking?
A.5% '1.12. According to research, under what
B. 25% circumstances are patients more likely to
c. 50% comply with recommended treatment and less
D. 75% likely to re-attend?
E. 95%
A. If relative risk instead of absolute risk is used
in explanations
1.8. In terms of human thinking and
B. If the consultation is longer
decision-making, what tendency does
C. If the patient is male
confirmation bias describe?
D. If they feel that they have been listened to
A. To look for supporting evidence to confirm and understand the treatment plan
a theory and ignore evidence that E. If visual aids have been used instead of text
contradicts it to explain the treatment plan
B. To rely too much on the first piece of
I
information offered
1.13. Which of the following statements best
C. To stop searching because we have found
describes what is meant by the term 'human
something that fits
factors'?
D. To subconsciously see what we expect to
see
A. An understanding of diagnostic error
E. To want to confirm our diagnoses with B. How equipment is designed to take human
others before making a decision behaviour into account
C. How fatigue affects human thinking and
1.9. Which of these factors is most likely to lead decision-making
to an increased incidence of errors in clinical D. How healthcare professionals communicate
decision-making? in a team
E. The science of the limitations of human
A. Age
performance
B. Fatigue
C. Gender
1.14. In terms of human thinking and
D. Use of checklists
decision-making, anchoring describes what
E. Working alone
tendency?
1.10. In a case of suspected pulmonary A. To look for supporting evidence to confirm
embolism in an ambulatory care setting, which a theory and ignore evidence that
of the following individual signs on physical contradicts it

downloaded from www.medicalbr.com


CLINICAL DECISION-MAKING • 3

B. To rely too much on the first piece of following statements is true regarding the
information offered interpretation of a 0-dimer result?
C. To stop searching because we have found A. A negative 0-dimer result in a high clinical
something that fits probability patient excludes acute VIE
B. To subconsciously see what we expect to B. A positive 0-dimer result means that acute
see VIE is present
E. To want to confirm our diagnoses with C. 0-dimer is a useful screening test in patients
others before making a decision presenting with breathlessness
D. 0-dimer testing in suspected acute VIE
1.15. The D-dimer test has a sensitivity of at results in lots of false negatives
least 95% in detecting acute venous E. 0-dimer testing in suspected acute VIE
thromboembolism (VTE). However, it has a low results in lots of false positives
specificity of around 40%. Which of the

Answers
1.1. Answer: C. exclude those without it. Even a very good test,
It is estimated that diagnosis is wrong 11-15% with 95% sensitivity, will miss 1 in 20 people
of the time in the undifferentiated specialties with the disease. Every test therefore has 'false
of internal medicine, emergency medicine positives' and 'false negatives'.
and general practice. Diagnostic error is A very sensitive test will detect most
associated with greater morbidity than other disease but may generate abnormal findings in
types of medical error, and the majority of healthy people. A negative result will therefore
diagnostic errors are considered to be reliably exclude the disease, but a positive test
preventable. is likely to require further evaluation. On the
other hand, a very specific test may miss
1.2. Answer: B. significant pathology but is likely to establish
Likelihood ratios (LRs) are clinical diagnostic the diagnosis beyond doubt when the result is
weights. positive.

probability of finding in patients


1.4. Answer: A.
LR = with disease
probability of finding in patients Positive predictive value= A/(A +B) x 100
without disease
Predictive values combine sensitivity,
An LR greater than 1 increases the specificity and prevalence. Sensitivity and
probability of disease (the greater the value, the specificity are characteristics of the test; the
greater the probability). An LR less than 1 population does not change this. However, as
decreases the probability of disease. LRs are doctors, we are interested in the question,
developed against a diagnostic standard (in the 'What is the probability that a person with a
case of meningitis, lumbar puncture results) so positive test actually has the disease?' The
do not exist for all clinical findings. LRs illustrate positive predictive value is the proportion of
how a probability changes - but do not patients with a test result who have the disease
determine the pnor probabl1ity of disease. If the and is calculated from a table of results in a
starting probability is high to begin with, an LR specific population. It is not possible to transfer
of around 1 does not affect this. this value to a different population.

1.3. Answer: B.
1.5. Answer: D.
Sensitivity= A/(A +C) x 100 Post-test probability is the probability of a
disease after taking new information from a test
Sensitivity is the ability to detect true result into account. The.pre-test probability of
positives; specificity is the ability to detect true disease is decided by the doctor - it is an
negatives. There is no test that can 100% of opinion based on gathered evidence prior to
the time detect people with a disease and ordering the test. Bayes' Theorem can be used

downloaded from www.medicalbr.com


4 • CLINICAL DECISION-MAKING

to calculate post-test probability for a patient in overload and time pressure. Poor team
any population. It is a mathematical way to communication and poorly designed equipment
describe the post-test probability of a disease or clinical processes also increase the likelihood
by incorporating pre-test probability, sensitivity of error. Age, gender and working alone are not
and specificity. factors that affect cognition. Use of checklists
has been shown to improve decision-making in
1.6. Answer: D. clinical settings.
The treatment threshold combines factors such
as the risks of the test, and the risks versus 1.1 0. Answer: B.
benefits of treatment. The point at which the Suspected pulmonary embolism is a
factors are all evenly weighted is the threshold. common problem referred to UK ambulatory
If a test or treatment for a disease is effective emergency care centres. Unexplained pleuritic
and low risk, then one would have a lower chest pain and/or a history of breathlessness
threshold for going ahead. On the other hand, are the most common symptoms. Vital signs at
if a test or treatment is less effective or high rest and the physical examination may be
risk, one requires greater confidence in the normal. The only feature presented with a
clinical diagnosis and potential benefits of negative likelihood ratio in the diagnosis of
treatment first. In principle, if a diagnostic test pulmonary embolism is a heart rate of less than
will not change the management of the patient, 90beats/min. In other words, the other normal
then it should not be requested, unless there physical examination findings (including normal
are other compelling reasons to do so. oxygen saturations) carry little diagnostic
weight.
1.7. Answer: A.
Psychologists believe we spend 95% of our 1.11. Answer: A.
daily lives engaged in type 1 thinking - the 'Patient-centred evidence-based medicine'
intuitive, fast, subconscious mode of refers to the application of best available
decision-making. In everyday life we spend little research evidence while taking individual patient
time (5%) engaged in type 2 thinking. Imagine factors into account - these include clinical
driving a car; it would be impossible to function factors (e.g. bleeding risk when consideri?g
efficiently if every decision and movement was anticoagulation) and non-clinical factors (e.g.
as deliberate, conscious, slow and effortful as the patient's inability to attend for regular' blood
in our first driving lesson. With experience, tests if started on warfarin).
complex procedures become automatic, fa:st
and effortless. The same applies to medical 1.12. Answer: D.
practice. Many studies demonstrate a correlation
between effective clinician-patient
1.8. Answer: A. communication and improved health outcomes.
Cognitive biases are subconscious errors that If patients feel they have been listened to and
lead to inaccurate judgement and illogical understand the problem and proposed
interpretation of information. In evolutionary treatment plan, they are more likely to adhere
terms, it is thought that cognitive biases to their medication and less likely to re-attend.
developed because speed was often more Whenever possible, doctors should quote
important than accuracy. This property of numerical information using consistent
human thinking is highly relevant to clinical denominators (e.g. '90 out of 100 patients who
decision-making. Confirmation bias is the have this operation feel much better, 1 will die
tendency to look for confirming evidence to during the operation and 2 will suffer a stroke').
support a theory rather than looking for Visual aids can be used to present complex
contradictory evidence to refute it, even if the statistical information.
latter is clearly present. Confirmation bias is Relative risk exaggerates small effects that
common when a patient has been seen first by distort people's understanding of true
another doctor. probability. Longer consultations and the use
of visual aids are tools to facilitate good
1.9. Answer: B. communication but in themselves do not
Cognition is affected by things like fatigue, guarantee this is the case. Gender by itself is
illness, emotions, interruptions, cognitive not a factor.

downloaded from www.medicalbr.com


CLINICAL DECISION-MAKING • 5

1.13. Answer: E. common human tendency to rely too heavily


Human factors is the science of the limitations on the first piece of information offered (the
of human performance and how technology, 'anchor') when making decisions.
our work environment and team communication
can adapt for this to reduce diagnostic and 1.15. Answer: E.
other types of error. Analysis of serious adverse A very sensitive test will detect most disease
events in health care show that human factors but generate abnormal findings in healthy
and poor team communication play a people. A negative result therefore means the
significant role when things go wrong. Human disease is unlikely, but a positive result is likely
factors training is being introduced into to require further evaluation. As with all
undergraduate and postgraduate medical diagnostic tests, a low pre-test probability plus
curricula and multi-professional team training in a negative 0-dimer virtually excludes acute
many countries. VTE. However, if the pre-test probability is
very high, a negative 0-dimer still leaves a small
1.14. Answer: B. but significant chance that acute VTE is
Cognitive biases are subconscious errors that present
lead to inaccurate judgement and illogical 0-dimer is commonly raised in conditions
interpretation of information. In evolutionary that have nothing to do with acute VTE: for
terms, it is thought that cognitive biases example, old age, pregnancy, heart failure,
developed because speed was often more sepsis and cancer. This is the reason for its low
important than accuracy. This property of specificity. It should be used only when the
human thinking is highly relevant to clinical history and physical examination are consistent
decision-making. Anchoring describes the with acute VTE.

downloaded from www.medicalbr.com


S Maxwell

Clinical therapeutics and


good prescribing
Multiple Choice Questions
2.1. Which of the following drugs exerts its E. Reacting chemically with the agonist to
action directly at an enzyme target? reduce the agonist concentration available to
A. Aspirin bind to receptors
B. Hydrocortisone
C. Insulin 2.4. Which of the following drugs induce the
D. Lidocaine hepatic cytochrome P450 enzymes that are
E. Morphine responsible for drug metabolism?
A. Cimetidine
2.2. Which of the following statements best B. Ciprofloxacin
describes the term 'potency'? C. Erythromycin
A. A less potent drug will always have a lower D. Rifampicin
efficacy than a more potent drug E. Valproate
B. More potent drugs have a lower ED 50
C. The potency of a drug has no bearing on 2.5. Which of the following drugs may exhibit
recommended dose ranges zero-order drug kinetics at therapeutic; drug
concentrations? '
D. The potency of a drug is the extent to which
the drug can produce a response when all A. Carbamazepine
of the available receptors are occupied B. Ciprofloxacin
E. The potency of a drug is unrelated to its C. Lamotrigine
affinity for a receptor D. Phenytoin
E. Vancomycin
2.3. Which of the following statements best
describes how a non-competitive antagonist 2.6. Which of the following statements about
drug affects the pharmacodynamic actions of the estimated volume of distribution 0/d) of a
an agonist? drug is true?
A. Binding irreversibly with the receptor to A. Drugs that are highly bound to albumin have
remove receptors as potential binding sites a lower vd
for the agonist B. Drugs with a large Vd are eliminated more
B. Binding to a different poptJiation of receptors rapidly after discontinuation
that produce a response antagonistic to that C. Larger Vd is associated with a shorter
of the agonist half-life
C. Causing cell death so that it cannot function D. Vd cannot be greater than the volume of the
D. Increasing the total number of receptors for body
the agonist, thereby reducing the proportion E. Vd of lipid-soluble drugs is larger i,n males
that it can occupy than females (of equivalent mass)

downloaded from www.medicalbr.com


CLINICAL THERAPEUTICS AND GOOD PRESCRIBING • 7

2.7. Which of the following factors might be A. Amoxicillin


expected to favour increased bioavailability of a B. Ciprofloxacin
drug that is given by mouth? C. Doxycycline
A. Enterohepatic circulation of the active drug D. Erythromycin
B. Gastroenteritis E. Rifampicin
C. Hypoalbuminaemia
D. Impaired renal function 2.12. A 71 year old woman with ischaemic
E. Solid rather than liquid formulations heart disease recently started taking
amiodarone 200 mg orally daily for control of
her atrial fibrillation. She has now been
2.8. For which of the following drugs do
admitted to hospital 3 months later with
pharmacogenetic differences commonly
episodes of dizziness and bradycardia (heart
influence the clinical effect in Western
rate 48beats/min). The electrocardiogram
populations?
shows a prolonged QT interval (530 ms).
A. Amlodipine Which of her current regular medicines below
B. Codeine is most likely to interact with amiodarone to
C. Gliclazide cause the QT prolongation?
D. Omeprazole
A. Clopidogrel
E. Simvastatin
B. Moxifloxacin
C. Nicorandil
2.9. Which of the following features is most D. Simvastatin
characteristic of hypersensitivity adverse drug E. Thyroxine
reactions?
A. They are associated with human leucocyte 2.13. Which of the following is the commonest
antigen (HLA) class haplotypes cause of prescribing errors in hospital
B. They are discovered early in the drug practice?
development process A. Calculation errors
C. They are dose related B. Duplicated prescribing I
D. They manifest several months after initial C. Failed medicines reconciliation I
exposure D. Prescribing without indication
E. They occur at the higher part of the E. Unintentional prescribing
therapeutic dose range
2.14. Which of the following is NOT information
2.10. Which of the following is an advantage of required as part of the regulatory process
the .spontaneous voluntary reporting methods leading to the granting of a marketing
of pharmacovigilance? authorisation ('license')?
A. It captures the majority of adverse drug A. Cost-effectiveness compared to standard
reactions treatment
B. It is able to quantify the risk of an adverse B. Efficacy in the licensed indication
drug reaction (ADR) after exposure to a drug C. Product information literature
C. It is specific for events that really are caused D. Quality of the manufacturing process
by the drug E. Toxicology studies
D. It provides early signal generation after
marketing of a new drug 2.15. A trial of 5000 hypertensive patients
E. Its information is generated by highly randomised them to treatment with a new oral
qualified professionals anticoagulant or a matched placebo. After a
follow-up period of 5 years, 150 patients in the
2.11. A 23 year old woman is takin~ a active treatment arm and 250 patients in the
combined oral contraceptive preparation. She placebo arm had suffered a stroke.
has developed an infection sensitive to a What is the number of patients that need to
number of common antibiotics. Which of the be treated (NNT) with th~ new treatment over 5
following antibiotic choices is most likely to years to prevent one stroke?
interact with the contraceptive preparation to A. 10
cause contraceptive failure? B. 15

downloaded from www.medicalbr.com


8 • CLINICAL THERAPEUTICS AND GOOD PRESCRIBING

c. 20 A. Codeine phosphate 60 mg orally 4 times


D.25 daily
E. 30 B. Lactulose 20 g 3 times daily
C. Pabrinex (vitamins B and C) intravenous
2.16. An 82 year old man has a routine high-potency solution for injection 2 pairs of
medication review with his family physician. 5 ml ampoules 3 times daily
He has a history of a transient ischaemic D. Spironolactone 100 mg orally daily
attack, hypertension and attacks of gout. E. Terlipressin acetate 1.5 mg intravenously
Which of the following prescriptions should 4 times daily
probably be discontinued?
2.20. The following dose expressions have been
A. Allopurinol 100 mg orally daily
found on a hospital inpatient chart.
B. Amlodipine 5 mg orally daily
Which dose expression violates acceptable
C. Aspirin 75 mg orally daily
prescribing practice?
D. Diclofenac 25 mg orally 3 times daily
E. Ramipril 5 mg orally daily A. 1 sachet
B. 1.4 g
C. 20 ml
2.17. Which of the following drugs would pose
D. 26 units
the greatest risk of teratogenic effects if
E. 100 11g
prescribed during the first trimester of
pregnancy?
2.21. Which of the following drugs should be
A. Amoxicillin prescribed by its proprietary (brand) name in
B. Mebeverine hydrochloride preference to the generic international
C. Rifampicin non-proprietary name (INN)?
D. Sodium valproate
A. Atorvastatin
E. Sulfasalazine
B. Ciclosporin
C. Ciprofloxacin
2.18. A 63 year old woman has progressively D. lrbesartan
deteriorating renal function presumed to be due E. Methyldopa
to the effects of renal scarring secondary to
chronic reflux nephropathy in childhood. Her 2.22. A 76 year old woman has been treated
most recent estimated glomerular filtration rate successfully with digoxin 187.5 11g orally daily
(eGFR) is 26 mUmin/1.73 m 2 . over a number of months to control the
Which of the patient's prescriptions below ventricular response rate to her atrial fibrillation.
would need to be amended? She has recently complained of som~ r\ausea
A. Clopidogrel 75 mg orally daily and so the plasma digoxin concentration has
B. Doxazosin 8 mg orally daily been measured to investigate the possibility
C. Metformin hydrochloride 1 g orally of digoxin toxicity as an explanation. On
twice daily examination, the radial pulse rate is irregularly
D. Pregabalin 50 mg orally twice daily irregular and 64beats/min. The plasma
E. Tamoxifen 20 mg orally daily digoxin concentration is 1.8 11g/L (target
0.8-2.0 11g/L).
2.19. A 44 year old man with alcoholic cirrhosis What is the most appropriate course of
of the liver is admitted to hospital with delirium, action with regard to her digoxin prescription?
irritability and painful distension of the abdomen A. Change digoxin dosage to 187.5 11g orally
as a result of ascites. His investigations show on alternate days
that he is anaemic (haemoglobin 82 g/L), B. Maintain the digoxin dosage at 187:5 11g
jaundiced (bilirubin 65 11moi/L "(3.8 mg/dl)), orally daily
hypoalbuminaemic (albumin 20 g/L) and has a C. Reduce the digoxin dosage to 62.5 11g orally
mild coagulopathy (international normalised daily
ratio (INR) 1 .6). D. Reduce the dig~in dosage to 125 11g orally
His initial prescription chart contains the five daily
prescriptions below. Which of the prescriptions E. Stop digoxin and start bisoprolol 2.5 mg
should be discontinued? orally daily

downloaded from www.medicalbr.com


CLINICAL THERAPEUTICS AND GOOD PRESCRIBING • 9

2.23. A 56 year old man is being treated with 2.24. A 78 year old wornan is reviewed in the
intravenous gentamicin for Gram-negative emergency department of a hospital with i
~·I
septicaemia that is presumed to be of urinary
tract origin. He is well hydrated and his renal
bruising. She is taking warfarin 3 rng and 4 rng
orally on alternate days as prophylaxis against
~ll
function is normal. He has had two previous recurrent pulmonary emboli. Her last 3-rnonthly
doses of gentamicin 360 mg as a 30-minute INR rneasurernent was 2.7. She has been
intravenous infusion at 1000 hrs on Wednesday otherwise well with no other new syrnptorns
and Thursday. Both previous plasma and she has not been put on any new
gentamicin concentrations have been checked medicines. Her investigations reveal a normal
by the senior doctor in charge of the ward and full blood count but an INR of 6. 7.
the third dose of gentamicin has been What is the appropriate course of action?
prescribed and is now due (Friday morning at A. Stop warfarin and give phytornenadione
1000 hrs). (vitamin K1) 1-3 rng by slow intravenous
When should the next plasma gentamicin injection
concentration be taken? B. Stop warfarin and give phytornenadione
A. 0400 hrs (Saturday) (vitamin K1) 1-5 rng by rnouth
B. 1400 hrs (Friday) C. Stop warfarin and start apixaban
C. 1800 hrs (Friday) D. Stop warfarin alid start low-molecular-weight
D. lrnrnediately after the infusion is completed heparin injections
E. lrnrnediately before the third dose E. Stop warfarin for 2 days only

Answers
2.1. Answer: A. the same active site as the agonist but does so
Aspirin acts on the enzyrne cyclo-oxygenase irreversibly, or (iii) the antagonist interferes with
and is a non-selective and irreversible inhibitor. the signal transduction rnechanisrn preventing 1
Hydrocortisone is a corticosteroid and acts on receptor-agonist binding resulting in a
a DNA-linked receptor. Insulin acts on a pharmacological effect
kinase-linked receptor. Lidocaine blocks a
voltage-sensitive Na+ channel. Morphine acts 2.4. Answer: D.
on a G-protein-coupled receptor. Rifampicin is a very potent enzyme inducer. ·
All of the other options are well recognisec:j EJ.S
2.2. Answer: B. enzyme inhibitors.
The potency of a drug is related to its affinity
for a receptor. Less potent drugs are given in 2.5. Answer: D.
higher doses. The lower potency of a drug can The clearance rate of rnost drugs increases
be overcome by increasing the dose. Option D progressively as their plasma concentration
refers to the 'efficacy' of a drug. increases ('first -order metabolism'). For a
srnall number of common medicines, their
2.3. Answer: A. rnetabolisrn is 'saturable', meaning that the
The terrn 'non-competitive antagonist' is used rate of clearance cannot increase further
to describe two distinct situations where an ('zero-order kinetics'). For those drugs, further
antagonist binds to a receptor, or its associated dose increases can cause disproportionate
signal transduction rnechanisrn, to prevent the increases in exposure and the likelihood of
agonist activating the receptor. The cornrnon toxicity.
feature is that increasing the concentration of
agonist cannot outcornpete the antagonist. 2.6. Answer: A.
The receptor is rendered inactive and so the The apparent volume of distribution 0/d) is the
rnaxirnal response of which the cell or tissue is volume into which a drug appears to have
capable is reduced. This can occur in three distributed following intravenows injection. It is
ways: (i) the antagonist binds to an allosteric calculated frorn the equation Vd = D/C 0 , where
site of the receptor, (ii) the antagonist binds to D is the amount of drug given and C0 is th'e

downloaded from www.medicalbr.com


1 0 • CLINICAL THERAPEUTICS AND GOOD PRESCRIBING

initial plasma concentration. Drugs that are CYP206 ('poor metabolisers'), and are less
highly bound to plasma proteins may have a vd able to deliver sufficient morphine levels. Some
below 10 L (e.g. warfarin, aspirin), while those individuals carry more than two functional
that diffuse into the interstitial fluid but do not copies of the CYP206 gene ('ultra-rapid
enter cells because they have low lipid solubility metabolisers') and are able to metabolise
may have a Vd between 10 and 30 L codeine to morphine more rapidly and
(e.g. gentamicin, amoxicillin). It is an 'apparent' completely. They may develop symptoms
volume because those drugs that are lipid of morphine toxicity (e.g. drowsiness,
soluble and highly tissue-bound may have delirium and shallow breathing) even at
a Vd of greater than 100 L (e.g. digoxin, low doses.
amitriptyline). Drugs with a larger Vd have longer
half-lives, take longer to reach steady state on 2.9. Answer: A.
repeated administration and are eliminated Drug hypersensitivity is typically immune
more slowly from the body following mediated. Some drugs (especially large
discontinuation. Females have a greater . molecules) may themselves stimulate immune
proportionate content of fat in their bodies and reaction but many others (or their metabolites)
so the volume of distribution of lipid-soluble act as 'haptens' that bind covalently to serum
drugs is increased. or cell-bound proteins, including peptides
embedded ·in major histocompatibility complex
2.7. Answer: A. (MHC) molecules. This makes the protein
Drugs that enter the enterohepatic circulation immunogenic, stimulating antibody production
are reabsorbed into the body after excretion in targeted at the drug or T-cell responses against
the bile. This occurs because intestinal flora the drug. The reaction can produce a variety of
split the water-soluble conjugated drug, reactions ranging from mild rashes through to
allowing the free drug to be reabsorbed into life-threatening anaphylaxis. These reactions are
the body and thus increasing its bioavailability. often rare and discovered later in the drug
Gastroenteritis favours more rapid transit development process. The susceptibility to
through the small intestinal absorptive region of hypersensitivity reactions is, in many cases,
the bowel and reduces oral bioavailability. strongly related to genetics. Those who are /
Hypoalbuminaemia may alter the proportion of susceptible will often react immediately to 1
the drug retained in plasma after absorption but minimal exposure to the drug, making it
does not alter the overall bioavailability in the very difficult to identify a dose-response
body. Impaired renal function may influence relationship.
clearance of a drug but does not influence
bioavailability. Aqueous solutions, syrups, elixirs, 2.10. Answer: D.
and emulsions do not present a dissolution Voluntary reporting is a continuously ope~ating
problem and generally result in fast and often and effective early warning system for
complete absorption as compared to solid previously unrecognised rare ADRs. It is better
dosage forms. Due to their generally good suited than most other methods to early
systemic availability, solutions are frequently detection of previously unknown reactions,
used as bioavailability standards against which especially for medicines that are prescribed in
other dosage forms are compared. high volume. Although doctors were initially the
main source of reporting, most other healthcare
2.8. Answer: B. professional groups, and patients, are now able
Codeine is an opioid analgesic drug that is to report in the UK. Their reports have been
licensed for the treatment of mild to moderately shown to be of equivalent value to those
severe pain, and it belongs to the drug class of produced by the medical reporters. Its
opioid analgesics. Codeine is metabolised by weaknesses include low reporting rates (only
the hepatic cytochrome P450 206 (CYP2D6) 3% of all ADRs and 10% of serious ADRs are
enzyme, which also metabolises many other ever reported), an inability to quantify risk
prescribed drugs. CYP2D6 converts codeine to (because the ratio of ADRs to prescriptions is
its active metabolite, morphine, which is unknown) and the influence of·prescriber
responsible for the analgesic effect. The awareness on likelihood of reporting {reporting
analgesic effect of codeine is attenuated in rates rise rapidly following publicity about
individuals who carry two inactive copies of potential ADRs).

downloaded from www.medicalbr.com


CLINICAL THERAPEUTICS AND GOOD PRESCRIBING • II

2.11. Answer: E. 2.14. Answer: A.


Although there have been past suggestions New drugs are given a 'market authorisation'
that broad-spectrum penicillins might interfere based on the evidence of quality, safety and
with gut flora to alter the enterohepatic efficacy presented by the manufacturer. The
recycling of oestrogens (reducing their regulator will not only approve the drug but will
bioavailability in the body), it is now thought also take great care to ensure that the
that the only types of antibiotic that interact accompanying information reflects the evidence
with hormonal contraception and make it less that has been presented. The summary of
effective are rifampicin-like antibiotics. The product characteristics (SPC), or 'label',
metabolism of oestrogens is accelerated by provides detailed information about
rifamycins, leading to a reduced contraceptive indications, dosage, adverse effects, warnings,
effect with combined oral contraceptives, monitoring, etc.
contraceptive patches and vaginal rings.
Erythromycin is a well-recognised inhibitor of 2.15. Answer: D.
the hepatic metabolism of many drugs The calculation of NNT can be undertaken in
(including oestrogens) but this will not result in two ways. First, the number of patients
contraceptive failure. prevented from suffering a stroke in the active
treatment compared to control arm was I 00
2.12. Answer: B. out of a total number at risk of 2500. Therefore,
Moxifloxacin is a quinolone antibiotic that the numbers treated for each one who
can be used to treat sinusitis, community- benefitted was 2500/1 00 = 25. An alternative
acquired pneumonia, exacerbations of chronic approach that works easily in less rounded
bronchitis, mild to moderate pelvic inflammatory numbers is to consider the difference in the
disease, or complicated skin and soft tissue percentage of patients in each group who had
infections. Along with other quinolones, it may a stroke, i.e. active treatment 150/2500 x I 00
block cardiac potassium channels and delay = 6% and placebo 250/2500 x I 00 = I 0%.
the repolarisation phase of the action potential The difference is 4%, meaning that if a single
to prolong OT interval. This may potentiate at-risk group of just 100 patients were
the similar actions of amiodarone. Patients considered, then 4 would benefit and so the
with a prolonged OT interval are at risk of NNT is I 00/4 = 25.
suffering episodes of torsades de pointes,
which may progress to cause cardiac 2.16. Answer: D.
arrest. Diclofenac sodium is a non-steroidal
anti-inflammatory drug (NSAID) that is indicated
2.13. Answer: C. for the treatment of inflammatory arthritis gild
Medication reconciliation is the process of other musculoskeletal conditions. NSAIDs are
creating the most appropriate list of contraindicated in elderly patients because of
medications for the patient - including drug their increased risk of adverse effects, notably
name, dosage, frequency and route - at a on the gastrointestinal mucosa and renal
transition of care from one provider to another. function. The likelihood of each of these
Failure to take an adequate medication history outcomes is increased by co-prescription of
from the patient (or relative), obtain information aspirin and rarnipril, respectively. All of the other
from another professional or another source medicines appear to have a clear indication for
increases the chance that important medicines use. Best practice will be to discuss the
will be inadvertently omitted. Medicines medications involved with the patient
reconciliation is also about considering that himself.
information in the light of the clinical
circumstances and altering or discontinuing 2.17. Answer: D.
prescriptions as necessary. The medicines Sodium valproate is associated with a risk of
reconciliation process is particularly important major and minor congenital malformations (in
at the admission, transfer and/or discharge particular neural tube defects) as well as
from hospital. Omission of medicines on long-term neurodeveloprnental effects. It should
admission or discharge from hospital may be avoided during pregnancy unless there is no
account for a third of all 'recorded errors in safer alternative and only after a carefully
some studies. discussing the risks with the patient.

downloaded from www.medicalbr.com


1 2 • CLINICAL THERAPEUTICS AND GOOD PRESCRIBING

2.18. Answer: C. such medicines include diltiazem, lithium,


The UK National Institute for Health and Care theophylline, phenytoin and insulin.
Excellence (NICE) recommends that the dose Non-proprietary names are also preferred in the
of metformin should be reviewed if the eGFR is case of many compound and modified-release
less than 45 mUmin/1 .73 m 2 and that it should preparations.
be avoided if the eGFR is less than 30 mU
min/1.73 m 2 . (Type 2 diabetes in adults: 2.22. Answer: D.
management. NICE guideline [NG28]. Published The patient has excellent control of her
December 2015.) ventricular rate and so digoxin appears to be
very effective. However, she is complaining of
2.19. Answer: A. nausea, which is a very common toxic effect of
This patient has severe liver disease digoxin although there could be numerous
demonstrated by the failure to synthesise other explanations. The plasma digoxin
clotting factors and albumin, and is showing concentration is at the top end of the normal
features of hepatic encephalopathy. In severe 'target' range. Although within that range it is
liver disease many drugs can further impair perfectly possible (and likely) that, because of
cerebral function and may precipitate hepatic natural inter-patient variation, this patient's
encephalopathy. These include all sedative nausea is'indeed caused by digoxin. Given that
drugs, opioid analgesics (e.g. codeine the rate control is so good, the optimal course
phosphate), those diuretics that produce of action is to keep this patient on digoxin but
hypokalaemia and drugs that cause reduce the dosage in the hope of relieving the
constipation (e.g. codeine phosphate). Patients symptoms but maintaining the therapeutic
with hepatic encephalopathy must avoid effect. In other words, be guided by the
constipation, and lactulose is a preferred beneficial and adverse effects of the medicine
laxative. Spironolactone is indicated in the for your specific patient rather than the
management of ascites. B vitamins are published reference ranges alone.
important in avoiding Wernicke's
encephalopathy in chronically malnourished 2.23. Answer: C.
patients. Terlipressin acetate is a Gentamicin can cause significant toxic effeits
vasoconstrictor that helps to reduce bleeding if it accumulates in the body (especially I
from oesophageal varices. nephrotoxicity and ototoxicity). It is almost
exclusively cleared by the kidney so the risk of
2.20. Answer: E. accumulation is increased in patients with
The only acceptable abbreviations of mass to impaired renal function. Whatever the baseline
be used on a written prescription chart are renal function, all patients should have, U\e
'rng' and 'g'. 'Micrograms' should be written serum gentamicin concentration monitored after
out in full to avoid the risk that the Greek each dose as a guide to the next dose and the
symbol rnu (!l) is mistaken for an 'm'. dose interval. This patient has had two doses
This would run the risk of a serious dosing administered already and each has been
error. followed by a serum concentration that has
indicated it is appropriate to maintain the same
2.21. Answer: B. dose and dose interval. The issue now is when
Where non-proprietary ('generic') titles are to take the next serum concentration. The
given, they should be used by prescribers. This normal recommended window is between 6
allows a pharmacist to dispense any suitable and 14 hours post -dose: measurements taken
product, which avoids delay to the patient and before or after this interval are less likely to
sometimes expense to the health service. The reflect the gentamicin exposure produced by
only exception to this preference for generic the previous dosage. Most hospitals have a
prescribing is where there is a 'demonstrable nomogram (based on the original Hartford
difference in clinical effect between each nomogram) that helps clinicians to respond
manufacturer's version of the formulation, appropriately to the serum concentration.
making it important that the patient should
always receive the same brand. Ciclosporin is 2.24. Answer: E.
available in the UK as Neoral, Capimune, The patient is taking warfarin as prophylaxis
Dexirnune and ciclosporin. Other examples of against future recurrent pulmonary emboli. The

downloaded from www.medicalbr.com


CLINICAL THERAPEUTICS AND GOOD PRESCRIBING • 13


target INR should be 2.5. She now presents for 2 days and then resume (at a lower dose)
with the INR out of control and this can be before re-measuring the INR. In the absence of
caused by several different factors (e.g. erratic bleeding or an INR greater than 8.0, there is no
tablet taking, altered liver function, dietary indication to give vitamin K, which will largely
change, interacting drug). The loss of control reverse the action of warfarin and put the I
puts her at increased risk of bleeding although patient at risk of thromboembolic events until it
there are no symptoms suggestive of a serious can be restarted or replaced with an alternative
bleeding episode. The appropriate course of anticoagulant.
action at this point is to withhold the warfarin

downloaded from www.medicalbr.com


A Frost

Clinical genetics
Multiple Choice Questions
3.1. Deoxyribonucleic acid (DNA) repair A. Acetylation of histone protein
mechanisms exist to repair damage that may B. Alternative splicing
arise spontaneously or as a result of C. Epigenetic modification
environmental exposures. Failure to repair DNA D. Gene silencing by microRNA species
damage prior to replication results in mutations. E. Post-translational glycosylation
Spontaneous deamination of a cytosine results
in its conversion to a uracil. If this were not 3.4. You receive a genetic test result for a 3
repaired prior to replication, what would be the year old boy with a history of Wilms' tumour
result? and microcephaly, confirming a diagnosis of
A. Conversion of a GA pair to a CT pair mosaic variegated aneuploidy (MVA), a rare
B. Conversion of a GC pair to an AT pair inherited predisposition to chromosomal
C. Conversion of a GT pair to an AC pair non-dysjunction. The genetic test has identified
D. Conversion of an AC pair to a GT pair a mutation in BUB1B, a key component of/.
E. Conversion of an AT pair to a GC pair the mitotic spindle checkpoint. You now need
to explain these results to his parents.
3.2. The central dogma of molecular biology Non-dysjunction occurs during cell division
describes the steps by which information when the sister chromatids attach to the mitotic
encoded by the DNA determines protein spindle and are pulled apart to separate poles
production. One of these steps is transcription. of the cell. What is this phase of the cell/ cycle
Which of the following elements are all essential called?
components in transcription? A. Anaphase
A. Promoter sequence, deoxynucleotides, DNA B. Interphase
polymerase C. Metaphase
B. Promoter sequence, DNA template, DNA D. Prophase
polymerase E. Telophase
C. Promoter sequence, DNA template,
ribonucleic acid (RNA) polymerase 3.5. You receive a referral to see a 32 year old
D. Ribosomes, DNA template, RNA polymerase woman who has recently been diagnosed with
E. Ribosomes, messenger RNA (mRNA) triple-negative breast cancer. Triple-negative
template, transfer RNAs (tRNAs) breast cancer is defined by the absence of
oestrogen receptors, wogesterone receptors
3.3. In thyroid C cells, the calcitonin gene and human epidermal growth factor receptor
encodes the osteoclast inhibitor calcitonin, 2 (HER2) expression, and this tumour type is
whereas in neurons, the same gene encodes particularly common in BRCA 1 mutation
calcitonin-gene-related peptide. Which of the carriers. Genetic testing of the BRCA 1 and
mechanisms of controlling gene expression BRCA2 genes reveals a heterozygous BRCA 1
listed below is responsible for this mutation (BRCA 1 c.37 48G>T). This mutation
multi-functionality? substitutes a G for a T, resulting in the creation

downloaded from www.medicalbr.com


.....______ _ _ --
CLINICAL GENETICS • 15

of a premature stop codon and a truncated she has microcephaly (occipitofrontal


protein, a so-called 'stop-gain mutation'. What circumference 0.4th centile), some subtle
other name is commonly used for this type of dysmorphic features and global developmental
mutation? delay. Which of the investigations listed
A. Deletion below is the most appropriate first-line
B. Frameshift mutation investigation?
C. Missense mutation A. Array comparative genomic hybridisation
D. Nonsense mutation (CGH)
E. Synonymous mutation B. Exome sequencing
C. Fragile X testing
3.6. A 37 year old woman with type I myotonic D. Karyotype
dystrophy (DMI) attends your clinic for E. Whole-genome sequencing
genetic counselling. She is 8 weeks pregnant.
Which of the following pieces of advice is 3.9. Random double-stranded breaks in
correct? DNA are a necessary feature of meiotic
A. A baby inheriting the condition is at risk of recombination. The frequency of these breaks
being more severely affected than her is dramatically increased by exposure to
B. Her chance of having a baby affected by this ionising radiation. These breaks are usually
condition is I in 4 repaired accurately by DNA repair mechanisms
C. Her partner should be referred for genetic within the cell; however, some will instead
testing undergo non-homologous end-joining. Which of
D. Only a male baby will be affected with this the following is a possible outcome of
condition non-homologous end-joining between
E. The mutation causing her condition is likely fragments from different chromosomes?
to have arisen post-zygotically A. Deletion
B. Duplication
3.7. A 16 year old girl is referred to your clinic C. Paracentric inversion
with primary amenorrhoea. On examination she D. Pericentric inversion
is on 0.4th centile for height. You request a E. Translocation
karyotype, the result of which is shown below.
What is your diagnosis? 3.10. Osteogenesis imperfecta type II is a lethal
condition causing severe bone deformity and
respiratory failure. It is caused by mutations in

(C' }< ,, )) /)
1 2 3 4 5
type I collagen genes, resulting in the
production of an abnormal protein that
interferes with the normal functioning of the

)( It lI >I >I U I~
wild-type protein. What is the name for this
type of mutation?
10 12
A. Dominant negative mutation
fl () nu JJ
14 15 16 17 18
B.
C.
Gain-of-function mutation
Loss-of-function mutation

19
II
20 ••
21
u
22
J D.
E.
Protein-truncating mutation
Stop-gain mutation

A. Edward's syndrome 3.11. You are asked to review a 17 year old boy
B. Klinefelter's syndrome with a diagnosis of Becker muscular dystrophy.
C. Lynch's syndrome He has two siblings, an unaffected brother and
D. Patau's syndrome a sister whose status is unknown. His parents
E. Turner's syndrome are fit and well; however, his maternal
grandfather also had Becker muscular
3.8. You receive a referral to review an 18 dystrophy. You need to.construct an
month old girl with developmental delay. She is appropriate pedigree for your notes. What
the first child of unrelated parents and there is symbol would you conventionally use to
no significant family history. On examination represent his mother in this case?

downloaded from www.medicalbr.com


16 • CLINICAL GENETICS

A. A diamond A. Affected males cannot transmit the condition


B. A half-shaded circle to their daughters
C. A shaded circle B. Affected males cannot transmit the condition
D. An open circle to their sons but all their daughters would be
E. An open circle with a central dot carriers
C. Female carriers may be variably affected due
3.12. You meet a family affected by Lynch to X-inactivation
syndrome, an autosomal dominant condition D. Females are affected more often than males
causing increased predisposition to cancer, E. The condition has arisen de novo and her
mainly of the colon and endometrium. You siblings do not require genetic testing
need to explain the concept of autosomal
dominant inheritance to the family. Which of the 3.14. You are asked to provide genetic
following is a typical feature of autosomal counselling for a couple who are expecting
dominant inheritance? their third child. They have two older children, a
A. 25% recurrence risk for a couple with an normally developing 9 year old daughter and a
affected child son who, at age 5, has significant learning
B. 50% chance of an unaffected child with an difficulties. There is a family history of learning
affected sibling being a carrier difficulties in the maternal grandfather and a
C. Affected individuals occurring in a single maternal uncle, and his daughter, in turn, has
generation a degree of developmental delay. You
D. Males more commonly affected than females construct a pedigree (Fig. 3.14) with the
E. Variable penetrance affected family members represented by
the filled symbols.
3.13. You receive a referral to review a 12 year The couple has just found out that they are
old girl with a 2-year history of worsening expecting a boy, and are concerned that, since
muscle weakness and pain, recurrent migraines in their family it is boys more than girls that
and vomiting. Her neurologist requested a seem to be affected, he may be at risk. They
genetic test, which confirmed the diagnosis of have heard that learning difficulties are ,
MELAS (mitochondrial encephalopathy, lactic commonly X-linked conditions, and want to/
acidosis and stroke-like episodes), a rare know whether you think this could be the c;;ase
mitochondrial disorder. She and her parents in their family and, if so, whether they cou!d
wish to discuss the inheritance of this condition have genetic testing of the X chromosome.
and its implications for their family. Which of the When reviewing a pedigree, which of these
following statements is true in relation to her features is NOT consistent with X-linked
condition? inheritance?

Fig. 3.14

downloaded from www.medicalbr.com


CLINICAL GENETICS • 17

A. Affected father and affected son 3.18. A 27 year old woman is referred to your
B. Affected members in each generation clinic by her family physician for advice. She
C. Affected son and affected maternal uncle was worried about her family history of breast
D. The presence of an affected fernale cancer and decided to undergo genetic testing
E. Variable expressivity through a private company offering a
next -generation sequencing (NGS) breast
cancer susceptibility gene panel test. They sent
3.15. You review a 39 year old wornan with
her the report but she is having trouble
advanced breast cancer. She has been referred
understanding some of the terminology used
to you for genetic testing because of her young
and needs some clarification. In NGS, what
age at diagnosis. You undertake diagnostic
does the term 'capture' refer to?
genetic testing but are unable to identify a
pathogenic mutation in either BRCA 1 or A. Binding of the library fragments as they are
BRCA2. Which of the following mechanisms washed over the flow cell
could be a contributing mechanism in her B. Downloading the relevant read data into the
tumour formation? analysis software
C. Identifying the differences between the reads
A. Apoptosis
and the reference genome
B. Autocrine stimulation
D. Pulling out the part of the genome to be
C. Gain-of-function mutation in a turnour
sequenced
suppressor gene
E. Successfully identifying a disease-causing
D. Loss-of-function mutation in an oncogene
variant
E. Passenger mutation
3.19. You review a family, several members of
3.16. You receive an array comparative genomic whorn have the same, rare condition, for which
hybridisation (array CGH) report for a patient no genetic cause has yet been identified. You
with developmental delay and autism. The are considering a clinical research project with
report is normal and has not identified a cause the aim of identifying the disease-causing
for the patient's difficulties. Which of the mutation in this family. You are trying to decide
following statements is true about what array whether whole-exome sequencing or
CGH is able to reliably detect? whole-genome sequencing would be a better
A. It will reliably detect aneuploidy approach. Which of the following is an
B. It will reliably detect balanced translocations advantage of whole-genome sequencing ovef
C. It will reliably detect intragenic deletions whole-exome sequencing? '
D. It will reliably detect mosaicism at the A. Increased detection of gene dosage
1% level abnormalities
E. It will reliably detect triploidy B. Increased detection of mosaicism
C. Increased likelihood that a variant detected
3.17. A 2 year old boy with global will be pathogenic
developmental delay and facial dysmorphism D. Less expensive
attends with his parents for the results of his E. Lower risk of identifying incidental findings
array CGH testing. His parents are healthy and
there is no family history of note. The test has 3.20. You are asked to review a 39 year old
identified a 446-kB deletion at 18p23, which woman who has had a positive result for
has been reported as a copy number variant trisomy 21 during non-invasive prenatal testing
(CNV) of uncertain significance. What would be for aneuploidy screening. She is very upset and
your next step in his management? is asking you if there is any chance that the
A. Exome sequencing of the boy and his test could be wrong. Which of the following is a
parents possible cause of a false-positive result in this
B. Intellectual disability gene panel testing circumstance?
C. Parental array CGH testing A. Confined placental mosaicism
D. Repeat the array using more closely spaced B. High maternal body IJlass index (BMI)
probes to give a higher resolution C. Maternal smoking
E. Request a karyotype to exclude a balanced D. Previous miscarriage of aneuploid fetus
translocation E. Test done too early in gestation

downloaded from www.medicalbr.com


1 8 • CLINICAL GENETICS

3.21. You are reviewing a 35 year old woman 3.22. You review a 42 year old woman who
with triple-negative breast cancer, in whom you developed breast cancer at the age of 27 that
have identified an underlying BRCA 1 mutation. was successfully treated, and has now
Her oncologisthas recommended that she developed an osteosarcoma in her right femur.
enters a trial of treatment with a poly ADP On discussion of her family history she tells you
ribose polymerase (PARP) inhibitor. She wants that her mother died when she was very young
to know more about how they work. Which of of brain cancer (glioblastoma) and that her
the following statements about the mechanism brother is currently receiving treatment for a
of PARP inhibitors is true? rhabdomyosarcoma. Apart from evidence of a
A. They block the double-stranded DNA previous mastectomy, there are no additional
break-repair pathway phenotypic features on physical examination.
B. They block the double-stranded DNA You suspect a familial cancer predisposition
break-repair pathway and up-regulate the syndrome. Which of the following cancer
single-stranded DNA break-repair pathway predisposition syndromes would be the best fit
C. They block the single-stranded DNA for this tumour spectrum?
break-repair pathway A. Birt-Hogg-Dube syndrome
D. They repair the double-stranded DNA B. Cowden's syndrome
break-repair pathway C. Gorlin's syndrome
E. They repair the single-stranded DNA D. Li-Fraumeni syndrome
break-repair pathway E. Lynch's syndrome

Answers
3.1. Answer: B. be joined together (alternative splicing) to
In DNA, bases are paired as follows: adenine produce more than one form of mRNA,
(A) with thymine (T) and guanine (G) with which may be tissue specific, as in this
cytosine (C). In RNA, the pairing is the same example.
except that adenine (A) pairs with uracil (U). If
unrepaired prior to replication, deamination of a 3.4. Answer: A.
cytosine (C) to a uracil (U) will result in pairing Whilst the other answers are all stages of the
with adenine (A), ultimately replacing the original cell cycle, it is during anaphase that the ~pindle
GC pair with an AT pair. fibres attach to the sister chromatids a~d pull
them apart.
3.2. Answer: C.
Transcription describes the production of RNA 3.5. Answer: D.
from the DNA template. RNA polymerase binds A stop-gain (or nonsense) mutation introduces
to the promoter sequence on the DNA a premature stop codon, resulting in a
template strand, then moves along the strand truncated protein. A synonymous mutation
producing a complementary mRNA molecule. is a base substitution that does not result in
DNA polymerase is not required for a change in the amino acid (because more
transcription but is an essential component of than one codon may encode a particular amino
DNA replication. Translation (production of the acid). A missense (or non-synonymous)
protein encoded by the mRNA) occurs on the mutation is a base substitution that results in a
ribosome, and requires an mRNA template and change in the encoded amino acid. A deletion
tRNAs. is the loss of one or more nucleotides. If the
number of nucleotides deleted from within a
3.3. Answer: B. coding region is not a multiple.of three, this
Transcription produces a nascent transcript, results in a frameshift mutation, with a typically
which then undergoes splicing to generate the severe effect.
shorter 'mature' mRNA molecule that provides
the template for protein production. Splicing 3.6. Answer: A.
removes the intronic regions and joins together Myotonic dystrophy type 1 (DM1) is a triplet-
the exons. Different combinations of exons may repeat disorder, caused by pathological

downloaded from www.medicalbr.com


CLINICAL GENETICS • 19

expansion of a run of CTG repeats within the produces a shorter, non-functional protein and
OMPK gene, located on chromosome 19. It is therefore an example of a loss-of-function

-'
shows autosomal dominant inheritance so mutation. A gain-of-function mutation results in
there is a 50% chance that the patient's baby activation or alteration of a protein's normal
will be affected, regardless of gender. function.
Expanded repeats are unstable and may
expand further during meiosis, so that offspring 3.11. Answer: E.
inheriting the condition are often more severely Becker muscular dystrophy is an X-linked ·
affected than the affected parent - a disorder. Since his grandfather was also
phenomenon known as anticipation. affected, the condition cannot have
Anticipation most commonly occurs during the arisen in your patient de novo and his mother is
transmission of the condition from mother to an obligate carrier. In genetic pedigrees,
child. The vast majority of individuals with females are represented by circles, and
DM1 have inherited their expanded CTG allele unaffected female carriers of X-linked
from a parent; new expansions of a normal conditions are represented by an open circle
allele are rare. with a central dot. Female carriers of autosomal
recessive conditions are represented by a
3.7. Answer: E. half-shaded circle. Fully shaded symbols
Turner's syndrome is a sex chromosome represent affected family members.
aneuploidy where there is rnonosomy of the X Diamonds are used to represent ongoing
chromosome (note the single X chromosome pregnancies.
and absence of Y chromosome in the
karyotype). Girls with Turner's syndrome are 3.12. ·Answer: E.
typically shorter than average and have Autosomal dominant conditions typically show
underdeveloped ovaries, resulting in delayed or variable penetrance - not all people who inherit
arrested development of secondary sexual a mutation will develop the disease. Affected
characteristics, delayed or absent menstruation individuals typically occur in each generation
and commonly infertility. (unless the mutation has arisen de novo in an
affected individual). Males and females are
3.8. Answer: A. equally affected. /
The initial management step here is to exclude The recurrence risk for a couple with an
a chromosomal cause for her difficulties. Array affected child will depend on whether the
CGH would be the most appropriate first-line mutation has arisen de novo in the affected
investigation as it provides a genome-wide child (in which case it is low, typically < 1%), or
screen for chromosomal abnormalities. It has has been inherited from a parent, in which,'c,ase
superseded the use of karyotyping in this it is 50%.
context as it provides a much higher-resolution
screen. Fragile X is a recognised cause of 3.13. Answer: A.
developmental delay but is unlikely here in the In mitochondrial inheritance, the mutation is in
context of the microcephaly. If the array CGH is the mitochondrial DNA and, since mitochondria
normal, then you may wish to proceed to are contributed by the oocyte and not by the
exorne sequencing, or a developmental delay sperm, inheritance is exclusively via the
gene panel. maternal line. Males and females are equally
affected. Variable penetrance and expressivity
3.9. Answer: E. is common in mitochondrial disorders due to
Translocation is the result of joiniog of two the degree of mitochondrial heteroplasmy (not
segments of DNA from different chromosomes. due to X-inactivation, as in X-linked disorders).
All the other answers describe structural Whilst it is possible that th,e condition has
rearrangements that may be founct within a arisen in the proband de •novo, it is more likely
single chromosome. that it was inherited from her mother. If her
mother is indeed a carrier, she will have
3.10. Answer: A. transmitted the conditio[l to all her offspring.
A dominant negative mutation interferes with Both the mother and siblings should therefore
the function of the wild-type protein. A be offered genetic testing, regardless of clinical
protein-truncating (or stop-gain) mutation symptoms.

downloaded from www.medicalbr.com


20 • CLINICAL GENETICS

3.14. Answer: A. either of them also carried the CNV it would be


X-linked conditions are not passed from father unlikely that it was contributing significantly to
to son, as the mutation is on the X his phenotype. It is not uncommon to identify
chromosome. Whilst X-linked conditions are benign inherited CNVs during array CGH
mostly restricted to males, occasionally female testing. If the CNV is not inherited from an
carriers may exhibit signs of an X-linked unaffected parent it is harder to assess its
disease due to skewed X-inactivation. Also, significance. You would need to carefully
when considering a pedigree, beware of the consider any genes that could be potentially
possible presence of phenocopies (i.e. disrupted. If you remain unconvinced that the
individuals with a similar phenotype who do not CNV provides an explanation for his difficulties
carry the mutation); with a phenotype as you may wish to proceed to further genetic
common and multifactorial as developmental testing, and consider an intellectual disability
delay/learning difficulties, this could be a gene panel or exome sequencing.
confounding factor in your analysis.
3.18. Answer: D.
3.15. Answer: B. In NGS, 'capture' refers to the 'pull-down' of a
If a mutation results in activation of a growth targeted region of the genome for sequencing.
factor gene or receptor, then that cell will This may constitute a single gene, a number of
replicate more frequently as a result of genes associated with a given phenotype or
autocrine stimulation. Tumour formation is condition (a gene panel), the exons of all
promoted by gain-of-function mutations in coding genes known to be associated with
oncogenes and loss-of-function mutations in disease (a clinical exome) or the exons of all
tumour suppressor genes, not the other way known coding genes (an exome).
around. Passenger mutations accumulate
within cancer cells but do not in themselves 3.19. Answer: A.
promote growth (unlike 'driver' mutations). Whole-genome sequencing enables more even
Apoptosis is programmed cell death and does coverage of genes, allowing better identification
not have a role in tumour formation. (The BRCA of gene dosage anomalies than whole-exome
test result is not relevant here - the question is sequencing. (Gene dosage refers to the /
simply testing knowledge of mechanisms number of copies of a gene that are presert in
promoting tumourigenesis.) a genome, and anomalies may be caused'by
CNVs such as deletions or duplications.) ,"
3.16. Answer: A. Whole-exome sequencing is, however, less
Array CGH provides a high-resolution expensive, and allows deeper sequencing and
genome-wide screen for chromosomal consequently better detection of mosaicism.
abnormalities. Mosaicism down to a 10% level Whole-genome sequencing will detect many
can often be detected. Since it relies on more variants, so it is associated with a greater
analysis of comparative dosage across the risk of incidental findings, and the likelihood of
genome, triploidy (all chromosomes present in any given variant detected being pathogenic is
an extra copy) may be missed. Similarly, reduced.
because with balanced translocations dosage
is unaffected, these may not be picked up and, 3.20. Answer: A.
if suspected, karyotyping should be Confined placental mosaicism (the aneuploidy
undertaken. Even with the most powerful being present in placental tissue but not in the
modern arrays, the resolution is limited to fetus) is the most well-recognised cause of
around 10 kB. This would therefore miss many false-positive results during non-invasive
smaller intragenic deletions. Larger deletions aneuploidy screening. Results should be viewed
and duplications (or indeed aneuploidy) would, in the context of ultrasound findings, and
however, be reliably detected. positive results need confirmation with invasive
testing. High maternal BMI and early gestation
3.17. Answer: C. are recognised causes of false-negative results.
The next step would be to test his parents to A previous pregnane){ will have no effect on
see whether either of them carried the same these results as cell-free fetal DNA is cleared
CNV. Since they are both phenotypically normal from the maternal circulation within 30 minutes
with no history of developmental problems, if of delivery.

downloaded from www.medicalbr.com


CLINICAL GENETICS • 21

3.21. Answer: C. sarcoma, breast carcinoma, brain cancer


PARP inhibitors work by blocking the (especially glioblastoma) and adrenocortical
single-stranded DNA break-repair pathway. In a carcinoma. There are no additional clinical
BRCA 1/2 mutation-positive tumour with an features other than the cancer susceptibility in
already compromised double-stranded DNA this syndrome. The other answers are also
break-repair pathway, the additional loss of the examples of other rare cancer predisposition
single-stranded break-repair pathway will drive syndromes, with different spectrums of tumour
the tumour cell towards apoptosis susceptibility, and in some cases additional
{programmed cell death). phenotypic clinical features.

3.22. Answer: D.
Mutations in the TP53 gene cause Li-Fraumeni
syndrome, a hereditary predisposition to

downloaded from www.medicalbr.com


SL Johnston

Clinical immunology
Multiple Choice Questions
4.1. Which of the following statements best A. Each component is able to function
describes a key feature of innate immunity? independently
A. It improves with repeated exposure to a B. It is ready to act immediately on pathogen
given antigen exposure
B. It includes interaction between pattern C. Primary lymphoid tissues include the spleen
recognition receptors on phagocytes and and mucosa-associated lymphoid tissue
pathogen-associated molecular patterns D. T- and B-cell receptors are antigen specific
C. It is not associated with primary immune E. Vaccination efficacy does not require
deficiency functional adaptive immunity
D. It requires antigen processing for activation
E. Memory and specificity are characteristic 4.5. Which of the following statements is cor1ect
features regarding primary immune deficiency? /
A. A number of X-linked conditions are ;'
4.2. Which of the following statements best recognised
describes a key feature of phagocytes? B. Bone marrow transplantation is required for
A. They are derived from thymic progenitors B-cell immune deficiency
B. They are involved in intra- and extracellular C. Gene therapy has not yet been applied to
killing of microorganisms primary immune deficiencies '
C. They do not damage host tissue D. Primary immune deficiency is invariably fatal
D. They have a long half-life without treatment
E. They include monocytes, macrophages, E. Primary immune deficiency only presents in
neutrophils and natural killer (NK) cells childhood

4.3. Which of the following statements 4.6. Which of the following statements is correct
describes a key function of cytokines? in relation to immunoglobulins?

A. They are routinely measured in clinical A. They are constructed of two identical heavy
practice chains and two identical light chains
B. They are small molecules that act as B. They are derived from thymic precursors
intercellular messengers C. They are limited to the intravascular
C. They do not require receptor interaction compartment
D. They have distinct and non-olierlapping D. They include six isotypes
biological functions E. They protect predominantly against
E. They have not been shown to have a role in intracellular infection
disease pathogenesis
4.7. Which of the following statements is
4.4. Which of the following statements is correct most consistent with immunoglobulin
with regard to adaptive immunity? deficiency?

downloaded from www.medicalbr.com


CLINICAL IMMUNOLOGY • 23

A. It commonly presents with opportunistic E. Type IV hypersensitivity is typically immediate


infection in onset
B. It has no association with autoimmune
disease 4.12. Which of the following clinic features
c. It is not associated with end-organ damage would be UNUSUAL in acute systemic type I
D. It is unlikely in myeloma hypersensitivity?
E. It may require immunoglobulin replacement A. Bronchospasm
therapy B. Eczematous rash
C. Hypotension.
4.8. Which of the following statements most D. Urticarial rash
accurately describes the complement system? E. Vomiting
A. Complement proteins are reduced as part of
the acute phase response 4.13. Which of the following statements is
B. It can be activated by one of two pathways correct regarding mast cell tryptase
- the classical or alternative pathway measurement?
C. It does not contribute to local inflammation A. It has a half-life of 24 hours
D. It ends with a final common pathway leading B. It is a less reliable marker of mast cell
to bacterial lysis activation than plasma histamine
E. It refers to a series of immune proteins C. It is elevated in all cases of anaphylaxis
produced by the primary lymphoid D. Mast cell tryptase is unstable in serum
tissues E. Serial measurement following appropriate
acute patient management can be helpful in
4.9. Which of the following statements best confirming a mast cell-activating event
describes complement deficiency?
A. C1 inhibitor deficiency leads to a low C3, 4.14. Which of these statements most
even between attacks of angioedema accurately describes anaphylaxis?
B. It can be routinely treated with complement A. Desensitisation therapy is recommended for
replacement therapy nut -induced anaphylaxis 1
C. It is associated with connective tissue B. It is rarely fatal I
disease C. It leads to increased vascular permeability /'
D. It is not associated with recurrent infection D. It results from cross-linking of pre-formed,
E. It is not influenced by complement control allergen-specific lgG on the surface of mast
proteins cells with subsequent mast cell activation .
E. Onset from allergen exposure is typically, 1
4.10. Which of the following statements best delayed by 24 hours '
describes secondary immune deficiency?
A. It can be drug induced 4.15. Which of these statements most
B. It is generally not associated with accurately describes autoimmune disease?
opportunistic infection A. It can affect multiple organ systems
C. It is less common than primary immune B. It is not influenced by environmental
deficiency factors
D. It is rarely life-threatening C. It is typically life-threatening
E. It is reversible with management of the D. It is typically monogenic
underlying cause E. It requires immunosuppressive therapy

4.11. Which of the following statements is 4.16. Which one of the following statements is
correct regarding hypersensitivity reactions? true regarding disease-modifying therapy in
A. The predominant cell type involved in type IV autoimmune disease?
hypersensitivity is the basophil A. Anti-tumour necrosis factor (TNF) therapy
B. Type I hypersensitivity is lgG mediated has been shown to alter the course of
C. Type II hypersensitivity results in circulating disease progression i11. rheumatoid arthritis
immune complexes B. Biological agents are generally now
D. Type Ill hypersensitivity" results in considered first -line therapy for inflammatory
complement activation bowel disease

downloaded from www.medicalbr.com


24 • CLINICAL IMMUNOLOGY

C. Inhibition of integrins has no proven efficacy diagnostic tests. Which of the following
D. Mononclonal antibodies used in autoimmune statements best fits the clinical scenario?
disease have not been associated with A. A defect in T-cell immunity is most likely
serious side-effects B. A periodic fever syndrome is most likely
E. Small-molecule inhibitors targeting C. An X-linked immune deficiency is most likely
intracellular signalling pathways have yet to D. Primary immune deficiency is ruled out by
be developed the patient's age
E. The diagnostic test would be lymphocyte
4.17. Which of the following statements is true immunophenotyping
regarding organ transplantation?
A. Acute rejection typically occurs within the 4.20. A 70 year old man presents to his family
first week post-transplant physician with recurrent lower respiratory tract
B. Chronic rejection is immune mediated infection. Sputum culture has confirmed
C. Co-stimulatory blockade has not been Streptococcus pneumoniae and Moraxel/a
shown to improve outcomes catarrhalis on multiple occasions. Which of the
D. Hyperacute rejection occurs as a result of following tests would have the lowest yield (i.e.
recipient pre-formed antibody would be LEAST helpful) in this context?
E. Post-transplant immunosuppression is only A. Full blood count with white cell differential
required for the first 6 months B. Lymphocyte immunophenotyping
C. Neutrophil function tests
4.18. A 57 year old woman with a 20-year D. Serum immunoglobulins and electrophoresis
history of rheumatoid arthritis presents to the E, Thoracic computed tomography (CT) imaging
emergency department with a right basal
pneumonia. She has received a number of 4.21. A 35 year old woman presents to the
disease-modifying drugs for the arthritis, allergy clinic for investigation of venom
including methotrexate, and has most recently hypersensitivity. She reports rapid onset of
been on rituximab, an anti-CD20 monoclonal localised swelling at the site of a wasp sting on
antibody targeting B cells. Which of the her forearm, with subsequent dyspnoea and;
following statements is correct? altered vision prior to collapsing. She was /
A. Immunoglobulin measurement is unlikely to treated at the scene by the paramedics pribr to
be informative transfer to her local hospital. She lives in q, rural
B. Immunoglobulin measurement should include area, is a keen cyclist and often cycles in·
paraprotein assessment for appropriate remote areas. Which of the following
interpretation statements is correct?
C. Methotrexate is not a risk factor for A. Component-resolved diagnostics sh~t1ld be
secondary immune deficiency the first -line test
D. Opportunistic infection does not need to be B. From the clinical history given, an adrenaline
considered (epinephrine) auto-injector is not indicated
E. The patient is at low risk of secondary C. The clinical history is not suggestive of
immune deficiency anaphylaxis
D. The patient's regular drug history is not
4.19. A 5 year old boy presents to the relevant
paediatric team with right upper quadrant pain E. Venom immunotherapy should be
and fever. He has a temperature of 38.5°C, considered for this patient
tenderness over a mildly enlarged liver and is
noted to have gingivitis. At the age of 3 years 4.22. Which of the following clinical scenarios is
he developed a cutaneous abscess following correctly paired with the underlying immune
minor trauma. His younger brother died at 2 deficiency?
years of age of sepsis; further details are not A. A 26 year old man presenting with
known. On imaging he is found to have a oesophageal candidiasis = primary antibody
5x6 em hepatic abscess, aspiration confirming deficiency
Staphylococcus aureus infection. On the B. A 40 year old woman presenting with
post-take ward round you are asked to increasing delirium; cerebral imaging ,and
consider the differential diagnosis and biopsy confirm central nervous system (CNS)

downloaded from www.medicalbr.com


CLINICAL IMMUNOLOGY • 25

lymphoma = human immunodeficiency virus D. Type II cryoglobulinaemia is most likely


(HIV) E. With a normal C3, genetic C4 deficiency is
c. A 54 year old man presenting with cerebral most likely
toxoplasmosis = complement deficiency
D. A 6 year old boy presenting with 4.24. A 35 year old woman presents with cough
osteomyelitis and fungal pneumonia = severe 3 months after a live related donor renal
combined immune deficiency transplantation for end-stage renal disease
E. A previously well 9 year old boy presenting secondary to lgA nephropathy. She is
with Haemophi/us influenzae otitis media = prescribed prednisolone, tacrolimus and
secondary immune deficiency mycophenolate immunosuppression. She also
received additional immunosuppression as an
4.23. A 54 year old man with chronic hepatitis early post-transplant biopsy confirmed acute
C presents with lower limb palpable purpura, rejection. Her chest X-ray confirms bilateral
arthralgia and neuropathy. His alanine airspace shadowing, predominantly in the
aminotransferase (ALT) is 80 U/L (1 0-60 U/L), mid-zones. Which of the following statements is
albumin 32 g/L (35-50 g/L), C-reactive protein correct?
18 mg/L (<6 mg/L), platelets 90x10 9/L A. At such an early stage, post-transplant
(150-400x10 9/L), complement C4 opportunistic infection is unlikely
0.08 g/L (0.15-0.5 g/L); urinalysis confirms B. Early bronchoscopy with bronchoalveolar
proteinuria and microscopic haematuria. Which lavage needs to be considered
one of the following statements is true? C. Immunosuppression needs to be immediately
A. Diagnostic serum analysis can be reliably withdrawn
undertaken on routine blood sampling D. Tuberculosis is the most likely diagnosis
B. Systemic lupus erythematosus (SLE) is most E. The white cell differential is unlikely to be
likely helpful
C. Type I cryoglobulinaemia is most likely

Answers
4.1. Answer: B. cytokines have a role in disease pathogenesis'.
Pathogen-associated molecular patterns, found They are not currently routinely measured in
on invading pathogens, are recognised by clinical practice.
pattern recognition receptors on phagocytic
cells, allowing phagocytosis and subsequent 4.4. Answer: D.
pathogen destruction. Memory, specificity and T and B lymphocytes carry unique
the need for antigen processing are features antigen-specific receptors, conferring specificity
of adaptive immunity. Primary immune of the adaptive immune response. Antigen
deficiencies affecting innate immunity are well processing is required by T cells, such that the
recognised. adaptive response requires time to develop.
Components of the adaptive immune response
4.2. Answer: B. work in concert rather than functioning
Phagocytic cells are derived from bone marrow independently. Primary lymphoid tissues are the
precursors. They include rnonocytes, bone marrow and thymus.
macrophages and neutrophils, and are involved
in intra- and extracellular killing of 4.5. Answer: A.
microorganisms. They may cause damage to Primary immune deficiency often presents in
host tissue and have a short half-lim. childhood but can present later. A number of
X-linked conditions are recognised.
4.3. Answer: B. Immunoglobulin replacement therapy is
Cytokines are small molecules that act as standard treatment for primary B-cell immune
intercellular messengers via interaction with deficiency. Gene therapy has been applied
specific cytokine receptors. They have to a number of specific primary immune
overlapping biological functions. Many deficiencies. There is a spectrum of immune

downloaded from www.medicalbr.com


26 • CLINICAL IMMUNOLOGY

deficiency disorders; not all are invariably fatal 4.11. Answer: D.


without treatment. Type Ill hypersensitivity results in circulating
immune complexes, immune complex
4.6. Answer: A. deposition and subsequent complement
Immunoglobulins (lg) are constructed of two activation. The basophil is not the predominant
identical heavy chains and two identical light cell type in type IV hypersensitivity. Type I
chains. lgG, lgA, lgM, lgE and lgD isotypes are hypersensitivity is lgE mediated.
recognised. lgM is the isotype limited to the
intravascular compartment. They are produced 4.12. Answer: B.
by B cells and predominantly protect against Bronchospasm, urticarial rash, vomiting and
extracellular infection. hypotension are well-recognised features of
type I hypersensitivity. Eczematous rash is
4.7. Answer: E. more consistent with type IV hypersensitivity.
Symptomatic immunoglobulin deficiency may
require immunoglobulin replacement therapy. 4.13. Answer: E.
Secondary immunoglobulin deficiency can Mast cell tryptase is stable in serum at room
occur with myeloma. Opportunistic infection is temperature and is a more reliable marker of
not commonly seen without additional T-cell mast cell activation than plasma histamine. It
deficiency. End-organ damage such as has a half-life of 2.5 hours. Serial measurement
bronchiectasis can occur, as can autoimmune can be helpful in confirming a mast cell-
disease. activating event but a negative result does not
e;<clude anaphylaxis.
4.8. Answer: D.
Complement proteins are produced by the liver 4.14. Answer: C.
as part of the acute phase response and can Anaphylaxis leads to increased vascular
be activated by three pathways, the classical, permeability, is typically rapid in onset following
alternate and lectin pathways. All three allergen exposure and can be fatal. It results
pathways end in the membrane attack from cross-linking of allergen-specific lgE on .
complex, leading to bacterial lysis. Local mast cells. Desensitisation therapy is curren ily
1
inflammation is induced by complement not routinely recommended for food allergy;1
breakdown products.
4.15. Answer: A.
4.9. Answer: C. Autoimmune disease is generally not
Complement deficiency is associated with monogenic and is influenced by environmental
connective tissue disease and recurrent factors. Many autoimmune conditions ah3 not
infection, late complement protein deficiency life-threatening, e.g. vitiligo or hypothyroidism.
being particularly associated with recurrent Not all conditions require immunosuppression:
neisserial infection. Complement deficiency is e.g. coeliac disease, where gluten withdrawal is
not routinely treated with complement required. It can affect multiple organs.
replacement therapy. C1 inhibitor deficiency
can, however, be treated with C1 inhibitor 4.16. Answer: A.
concentrate. Patients with this condition have a Anti-TNF therapy has been shown to alter the
low C4, even between attacks of angioedema. course of disease progression in rheumatoid
Complement control proteins have an important arthritis. Small-molecule inhibitors targeting
role in controlling complement activity. intracellular signalling pathways are in
development. Natalizumab, an integrin inhibitor,
4.10. Answer: A. has proven efficacy in multiple sclerosis.
Secondary immune deficiency can be drug Monoclonal therapy is not without potentially
induced, for example with immunosuppressive serious side-effects (e.g. natalizumab and
agents. It is much more common than primary progressive multifocal leucoencephalopathy).
immune deficiency, can be associated with Biological agents are generally second line.
opportunistic infection and can be life-
threatening. Secondary immune deficiency may 4.17. Answer: D.
not be reversible with· management of the Hyperacute rejection occurs as a result,of
underlying condition. recipient pre-formed antibody. Acute rejection

downloaded from www.medicalbr.com


CLINICAL IMMUNOLOGY • 27

typically occurs at 5-30 days post-transplant. considered. Component-resolved diagnostics is


Chronic rejection is multifactorial. Co-stimulatory not the first-line test. The patient may be on
blockade has been shown to improve outcome. medication that could influence future
Post-transplant immunosuppression is required anaphylaxis management, e.g.
for more than 6 months. angiotensin-converting enzyme inhibitor or
~-blocker.
4.18. Answer: B.
In this case, secondary immune deficiency has 4.22. Answer: B.
to be considered. Rituximab can lead to B-cell CNS lymphoma, driven by Epstein-Barr virus
depletion, usually transient, and can be infection, is seen in immune deficiency, typically
associated with antibody deficiency. late-stage HIV infection or following systemic
Paraprotein assessment should be undertaken immunosuppressive therapy. A previously well 9
for appropriate interpretation of immunoglobulin year old is unlikely to have a secondary
results. immune deficiency. Candida oesophagitis is
more consistent with T-cell immune deficiency,
4.19. Answer: C. as is toxoplasmosis. Severe combined immune
An immune deficiency is likely; the patient's age deficiency is likely to have presented before the
does not rule this out. With the family history, age of 6 years and the clinical findings
an X-linked condition should be considered. described are more consistent with a primary
With the organisms identified, X-linked chronic phagocyte deficiency.
granulomatous disease, a disorder of
phagocytes, rather than aT-cell defect, is most 4.23. Answer: D.
likely, in which case neutrophil function testing A type II cryoglobulinaemia in the context of
would be diagnostic. hepatitis C is most likely, from the clinical
history, raised ALT, elevated inflammatory
4.20. Answer: C. markers and low C4. SLE is on the differential
The clinical history is most consistent with diagnosis but crypglobulinaemia typically
symptomatic antibody deficiency. This may be presents with rash, arthralgia and neuropathy.
primary or secondary, for example with chronic Warm separated serum is required for /
lymphocytic leukaemia, and may be associated appropriate investigation. Genetic deficiency of ,j
with end-organ damage. Neutrophil function C4 is less likely in this context.
testing is indicated in the investigation of
chronic granulamotous disease rather than 4.24. Answer: B.
antibody deficiency. Chronic granulomatous Early bronchoscopy should be considered. ,"
disease presents at a much younger age Opportunistic infection could occur at 3
usually, with staphylococcal and fungal months. The white cell differential will help td
infection, abscess formation and granulomas; determine the risk of opportunistic infection.
the clinical history does not suggest this. Immunosuppression needs to be carefully
com;;idered rather than immediately withdrawn;
4.21. Answer: E. withdrawal would compromise the transplanted
The clinical history is suggestive of anaphylaxis, organ. Pneumocystis pneumonia is the most
in which case venom immunotherapy should be likely diagnosis.

downloaded from www.medicalbr.com


D McAllister, H Campbell

Population health and


epidemiology
Multiple Choice Questions
5.1. Which of the following diseases was among 5.4. Which of the following diseases is among
the top 15 ranked causes of global premature .the top I 0 ranked risk factors underlying GBD
mortality (measured by years of life lost) in 2013 in 2013 (ranked in terms of the proportion of
as part of the Global Burden of Disease (GBD) GBD that they cause)?
Exercise? A. Alcohol use
A. Alzheimer's disease B. Low-fibre diet
B. Asthma C. Low physical activity
C. Colorectal cancer D. Suboptimal breastfeeding
D. Congenital anomalies E. Vitamin A deficiency
E. Protein energy malnutrition
5.5. Patients who respond positively to a,n
5.2. Global premature mortality is measured by invitation for screening tend to be less
years of life lost, with higher ranking meaning socioeconomically deprived than those. who do
higher number of premature deaths. Between not. In view of this, what needs to be ·
1990 and 2013, which of the following rose in considered when evaluating such a '
the rankings of conditions causing global programme?
premature mortality? A. Incomplete follow-up
A. Human immunodeficiency virus (HIV)/ B. Lead-time bias
acquired immune deficiency syndrome C. Length bias
(AIDS) D. Recall bias
B. Iron deficiency anaemia E. Self-selection bias
C. Maternal disorders
D. Measles 5.6. Which of the following is one of Wilson and
E. Meningitis Jungner's criteria for the evaluation of a
national screening programme?
5.3. Which one of the following diseases is A. The case-finding should be a
among the top 15 ranked causes of global once-and-for -all project
disability (measured by years of life lived with B. The condition sought should be an important
disability; YLD) in 2013? health problem
A. Alcohol use disorders C. The condition sought should be self-limiting
B. Epilepsy D. The condition sought should have a low
C. Lens cataract mortality
D. Neural tube defects E. The condition sought should only be
E. Schizophrenia treatable if detected at screening

downloaded from www.medicalbr.com


POPULATION HEALTH AND EPIDEMIOLOGY • 29

5.7. In a clinical trial where participants are B. The NNT is one minus the absolute risk
randomly allocated to a treatment or control reduction
group, which one of the following statements C. The NNT is the number of patients who will
is true? benefit from a treatment if 100 typical
A. In randomisation, the doctor generally knows patients are treated
which treatment the patient will be allocated D. The NNT is the reciprocal (inverse) of the
to before they are enrolled in the trial difference in risk between different treatment
B. In randomisation, the patient generally knows groups
which treatment they will be allocated to E. The NNT is the reciprocal (inverse) of the risk
before they are enrolled in the trial ratio
c. Randomisation is performed so that the
number of patients in the treatment and 5.11. In a clinical trial, 2000 patients were
control groups are the same randomly allocated on a 1-to-1 basis to either a
D. Randomisation is primarily used to reduce placebo or 'Novotreat', a new drug. After
1-year of follow-up, 130 patients in the placebo
bias
E. Randomisation is primarily used to reduce group and 100 in the treatment group had
confounding died. What was the absolute risk reduction?
A.3%
B. 23%
5.8. In a case-control study examining the
effect of coffee consumption on lung cancer,
c. 0.77
D. 1.30
which one of the following might lead to
E. 33.33
confounding?
A. People with lung cancer are more likely to 5.12. A city has a population of 100000. Each
over-report smoking year 10 000 people are diagnosed with heart
B. People with lung cancer are more likely to disease for the first time after presenting to
under-report coffee consumption their doctor and 5000 people die of heart
C. Smokers with lung cancer drink more coffee disease. Of the latter, 1000 are not found to
than smokers without lung cancer have heart disease until after they died. /
D. Smoking is commoner in coffee drinkers Assuming that there are no other ways that /
E. There is a lot of variation in the amount of new cases are identified, which of the following
caffeine in different coffees is true of heart disease in this city?
A. The case fatality is 20%
5.9. Which of the following is a true description B. The incidence is 10 per 100 person-yea~s .
of a cohort study, as used in epidemiological C. The incidence is 11 per 100 person-yeafs,/
research? D. The incidence is 5 per 100 person-years
A. Cohort studies generally enrol people without E. The prevalence is 10%
disease
B. Odds ratios cannot be calculated 5.13. Which of the following is true of national
C. Participants are randomly assigned to health information systems?
different exposures A. Definitions of non-psychiatric illnesses are
D. People with the disease of interest are agreed nationally
selected along with similar people without B. Few countries produce national mortality
the disease of interest statistics
E. Risk ratios cannot be calculated C. Incidence rates can generally be easily
compared across countries
5.10. Which of the following is true of the D. Most countries record attendances at
number needed to treat (NNT), calc;:ulated from primary care facilities by cause
a randomised controlled trial? E. Most countries use an international standard
A. The more effective a treatment, the larger classification system for recording cause of
the NNT death

downloaded from www.medicalbr.com


30 • POPULATION HEALTH AND EPIDEMIOLOGY

Answers
5.1. Answer: D. a disease to be important globally, e.g. malaria
Congenital anomalies were ranked 10 in 2013 or primary liver cancer, but not important in a
in the GBD Exercise initiated by the World specific country. The test should be whether it
Bank. Asthma is a major cause of burden of is an important public health problem in the
disease but not of premature deaths. Protein specific country or region.
energy malnutrition has been declining as an
important cause of death due to economic 5.7. Answer: E.
development globally. Alzheimer's disease and Randomisation is performed primarily to ensure
colorectal cancer are both important causes of that, on average, different treatment (or
premature death but are both outside the top intervention) groups are similar, apart from the
20 (rank 29 and 27, respectively). intervention being studied. It is only true to say
that the groups are the same on average,
5.2. Answer: A. however, as differences can arise by chance.
HIV/AIDS rose from rank 27 in 1990 to rank 6 Randomisation only works if the treatment
in 2013 due to the global epidemic. Most other allocation is masked until after the decision is
infections, including meningitis, measles and taken to enrol a participant. In some trials, the
maternal infections, fell due to increased treatment allocation continues to be concealed
implementation of successful control measures. after enrolment, but this is done for a separate
Iron deficiency anaemia ranking fell from 35 . reason, to reduce bias.
to 45.
5.8. Answer: D.
5.3. Answer: E. Confounding is where a cause of the disease
Schizophrenia was ranked 11 as a cause of (or a marker of such a cause) is commoner in
YLD in 2013. Epilepsy is an important cause of the exposure group of interest, and is not itself
YLD but outside the top 20 (rank 23). Cataract, a consequence of that exposure. If coffee per
neural tube defects and alcohol use disorders se, caused smoking, it would not be j
are causes of disability but are all well below confounding but a causal chain. !'
the top 15 ranked causes.
5.9. Answer: A.
5.4. Answer: A. Cohort studies are observational studies where
Alcohol use was ranked 6 as a cause of GBD participants are selected to reflect sorrie
in 2013. Suboptimal breastfeeding is ranked 19 population, characterised according ti:Jitheir
in 2013, down from rank 11 in 1990, and baseline characteristics and followed dp over
vitamin A deficiency is ranked 23, down from time to observe occurrences of one or more
rank 36. Low-fibre diet (25) and low physical diseases of interest. The relationship between
activity (17) are also of lower rank. an exposure and outcome of interest can then
be studied. Risk ratios, odds ratios and rate
5.5. Answer: E. ratios can all be calculated in cohort studies.
Self-selection bias, lead-time bias and length Studies that enrol people with disease and then
bias are all classic sources of bias in evaluation follow up these people over time are better
of screening trials. Incomplete follow-up is also called case series with follow-up.
an important problem in all trials. Screening
evaluations normally are based on recorded 5.10. Answer: D.
events for their primary evaluations and do Developed by David Sackett, the number
not depend on recall of past events, so needed to treat (NNT) aims to provide doctors
recall bias is not one of the most important and patients with a more intuitive statistic for
problems. quantifying treatment effects than the standard
ratios and differences. It is calculated as the
5.6. Answer: B. inverse of the abwlute risk reduction (or risk
The important question is how common is the difference) between treatment groups. Like
condition in the specific population in whom the absolute risk reductions, NNTs are only
screening will be implemented. It is possible for comparable if the risks being reported are the

downloaded from www.medicalbr.com


---- ~----
POPULATION HEALTH AND EPIDEMIOLOGY • 31

same, including the time over which the risk know the proportion of the people with heart
applied, e.g. 1-year absolute risk reductions disease who die from any cause. Additional
and 5-year absolute risk reductions are not assumptions would also be needed to estimate
comparable. the prevalence of heart disease.

5.11. Answer: A. 5.13. Answer: E.


There are many ways to represent the effect of
drug treatments. The other measures are, in
order, (1) the number needed to treat, (2) the
Globally, the vast majority of countries use the
World Health Organization International
Classification of Diseases {lCD) system. For
a.
risk ratio of the placebo versus the treatment psychiatric illnesses, the disease is also I

group, (3) the risk ratio of the treatment versus defined. In most countries (the most notable
the placebo group and the (4) relative risk exception being the USA where the Diagnostic I
reduction. and Statistical Manual of Mental Disorders is
used), lCD criteria are also used to define
5.12. Answer: C. psychiatric illnesses. For non-psychiatric
The incidence rate is the number of new events illnesses, definitions are not included as part of
per unit of person-time. As 1000 of the people the lCD system, although there are some
who died with heart disease were not separate definitions of certain conditions such
previously diagnosed, these also represent new as diabetes which have been adopted widely
events. You do not have sufficient information {http://www.who.int/diabetes/publications/
to estimate the case-fatality as you do not diagnosis_diabetes2006/en/).

I
/
//

_________________________ downloaded from www.medicalbr.com ___: ____ ----


JAT Sandoe, DH Dockrell

Principles of
infectious disease
Multiple Choice Questions
6.1. A 53 year old lawyer from South Africa who She complains of severe headaches, slurred
is human immunodeficiency virus (HIV)- speech and right arm weakness, and a
seropositive has a medical review, which computed tomography (CT) head scan
reveals a positive interferon-gamma release shows multiple space-occupying lesions in
assay (IGRA), showing T cells reactive to her brain. What are her symptoms most likely
Mycobacterium tuberculosis antigens. He is due to?
asymptomatic and a chest X-ray is reported as A. Antiretroviral drug-related side-effect
negative. Which of these most accurately B. HIV-related damage to brain
describes his mycobacterial status? C. Immune reconstitution inflammatory
A. Active pulmonary disease syndrome (IRIS)
B. Commensal flora D. Metastatic carcinoma
C. Extrapulmonary infection E. Syphilitic gumma
D. Latent infection
E. Opportunistic infection 6.4. A 59 year old woman presents with a
pelvic tumour and is found to have cer\.tical
6.2. A 9 month old infant has a temperature of carcinoma. The use of which vaccine lfl'
39.5oC and is not feeding. The parents attend childhood would have reduced the chance of
the local clinic where the doctor can find no this cancer developing?
abnormalities on physical examination other A. Hepatitis B virus vaccine
than erythema of the right tympanic membrane. B. Human papilloma virus (HPV) vaccine
Treatment with which of the following is C. Measles vaccine
appropriate as a simple and safe intervention D. Pneumococcal conjugate vaccine
that may decrease the body temperature? E. Rubella vaccine
A. Anti-tumour necrosis factor (TNF) antibody
B. Aspirin 6.5. A 33 year old Nigerian man who has had a
C. Erythromycin haematopoeitic stem cell transplant for aplastic
D. Paracetamol anaemia six months previously returns to visit
E. Penicillin his family in Nigeria once a year. He attends
your vaccine clinic. which of the following
6.3. A 44 year old woman is diagnosed with vaccines should be avoided?
HIV infection and a low CD4 T-cell count. She A. Hepatitis B virus vaccine
starts antiretroviral therapy to treat her infection B. Influenza inactivated vaccine
and is seen at clinic 3 months later when she C. Pneumococcal protein conjugate vaccine
has an undetectable HIV viral load and a D. Tetanus toxoid
significant increase in her CD4 T-cell counts. E. Yellow fever virus vaccine

downloaded from www.medicalbr.com


....
PRINCIPLES OF INFECTIOUS DISEASE • 33

6.6. A 45 year old man is admitted to the there is purulent drainage from a wound in the
intensive care unit with a short history of right hip and aCT scan reveals a collection,
respiratory symptoms and shortness of breath. which is aspirated. The microbiologists identify
He arrived in the country 2 days ago. Initial Gram-positive cocci in clusters that they identify
polymerase chain reaction (PCR) for routine as Staphylococcus aureus. The presence of
respiratory viruses is negative but a sample which genetic element will influence therapy
sent to the national laboratory detects a decisions for this patient?

-
specific geographically restricted coronavirus. A. ampC extended-spectrum P-lactarnase ·
Travel to which of the following countries is B. mecA penicillin-binding protein
most likely to be associated with this virus? C. NDM-1 carbapenemase
A. Brazil D. TEM-12 P-lactamase
I
B. China E. vanA gene cluster
C. Saudi Arabia
D. United States of America 6.1 D. A 73 year old patient develops
E. Zambia Staphylococcus aureus bacteraernia with a
meticillin-sensitive strain that remains persistent
6.7. A 63 year old retired international aid despite flucloxacillin therapy. All prosthetic
worker who last worked abroad 15 months ago material and collections of infection have been
presents with fever and fatigue. He had worked removed or drained. Which intervention may
in many African countries but particularly in enhance the success of the therapy?
Sudan and West Africa. On examination he is A. Administering therapy as an infusion
found to have splenomegaly and his full blood B. High-dose once-a-day therapy
count reveals anaemia and thrombocytopenia. C. Increasing the dose frequency
Which of the following tropical infections is D. Prolonging treatment duration
most consistent with the clinical scenario? E. Switching to glycopeptide therapy
A. Dengue
B. Leishmaniasis 6.11. A 60 year old man with acute myeloid
C. Plasmodium falciparum leukaemia develops pulmonary aspergillosis. He l
D. Trypanosoma brucei gambiense is placed on treatment with voriconazole. Whic~'
E. Typhoid fever of the following is a recognised adverse effect )
of voriconazole therapy that the patient shoulq
6.8. Your hospital has had three cases of be counselled about?
severe community-acquired pneumonia in the A. Aplastic anaemia
last 3 weeks that have had positive tests for B. Dermatitis
urinary Legionel/a antigen. You typically have C. Oesophageal ulcer
one to two cases per year. The cases were D. Proximal renal tubular injury
cared for in different units and there was no E. Tendon rupture
direct contact between the cases while
hospitalised. All three cases have the same 6.12. A 33 year old man develops chicken pox
postal code. You contact the public health with pulmonary involvement. He is previously fit
doctors to discuss these cases. How would and well and has had no prior therapy for viral
you best describe the clustering of these infections. He is hypoxic and has tachypnoea.
cases? His chest X-ray reveals widespread nodules. In
A. Common source outbreak addition to oxygen, which antiviral agent should
B. Epidemic he receive?
C. Nosocomial linked cluster A. Aciclovir
D. Pandemic B. Amantadine
E. Person-to-person community spmad C. Cidofovir
D. Foscarnet
6.9. A 56 year old patient has had multiple E. Valaciclovir
orthopaedic operations to deal with
complications of a road traffic accident. These 6.13. A 35 year old wildlife photographer plans
have involved plates and screws for several a trip to Kenya and asks for advice about
fractures. Three weeks after the last operation antimalarial treatment. She has a long history of

downloaded from www.medicalbr.com


34 • PRINCIPLES OF INFECTIOUS DISEASE

depression but is currently not on any C. Paracetamol


treatment. The medical history is otherwise D. Ramipril
unremarkable. Which is the best choice of E. Simvastatin
malaria prophylaxis in this case?
I~
A. Atovaquone plus proguanil 6.15. A 63 year old patient develops bowel
B. Chloroquine perforation after abdominal surgery and is
C. Doxycycline admitted to the intensive care unit where he
D. lvermectin requires pressors and ventilation. Microbiology
E. Mefloquine shows a mixed infection including coliforms,
enterococci and Candida spp. He requires a
range of antimicrobials initially including
6.14. A 50 year old tourist returns from a trip in teiocoplanin, meropenem and caspofungin,
Kenya where she visited game parks and but subsequently develops a nosocomial
remembers receiving some tick bites. She has Acinetobacter baumannii infection and receives
a necrotic eschar at the site of one bite a colistin and tigecycline. He makes steady
maculopapular rash and fever plus headache. progress but when weaned from the ventilator
A diagnosis of boutonneuse fever due to is found to have encephalopathy. Which of the
Rickettsia conorii is made and she is prescribed antimicrobials he has received is associated
doxycycline. She has diabetes mellitus and with development of encephalopathy?
hypertension as well as osteoporosis. Which A. Caspofungin
medication that she currently takes should not B. Colistin
be taken at the same time as doxycycline? C. Meropenem
A. Calcium D. Teicoplanin
B. Metformin E. Tigecycline

Answers
,,l
6.1. Answer: D. suspicion of a drug-related side-effect, 'but the
The test result with a positive IGRA nature of the lesions on brain scan are not
but no symptoms or signs on chest X-ray consistent. They have occurred at a time when
suggesting infection and no evidence of active the immunodeficiency associated with HIV has
disease point to latent infection. Active been reversed, which raises suspicibri of IRIS.
pulmonary and extrapulmonary disease HIV itself can cause chronic loss of brain
would have signs and/or symptoms and volume but not space-occupying lesions, and
M. tuberculosis would not be found as syphilitic gumma or metastatic carcinoma
commensal flora. (although differential diagnoses) are not
associated with therapy and immune
6.2. Answer: D. reconstitution.
Temperature elevation involves generation of
cytokines and prostaglandins. Paracetamol is a 6.4. Answer: B.
simple intervention that can inhibit generation of The HPV vaccine is now part of the vaccine
prostaglandins and act as antipyretic. Although schedule for girls in many countries and
aspirin would do the same, it is not decreases the risk of cervical carcinoma. The
recommended routinely for children due to the hepatitis B vaccine also reduces the incidence
risk of Reye's syndrome. Anti-TNF therapy has of a cancer, in this case hepatocellular
other medical indications arid antimicrobials carcinoma, but the other vaccines do not have
would not be appropriate for what appears to an obvious link to reduction of cancer
be a viral illness. incidence.

6.3. Answer: C. 6.5. Answer: E.


The symptoms in this case have come on after Live vaccines should, in general, be avoided in
therapy was commenced, which raises immunocornprornised patients or only given

downloaded from www.medicalbr.com


PRINCIPLES OF INFECTIOUS DISEASE • 35

after careful risk-benefit analysis in exceptional 6.10. Answer: A.


cases. Yellow fever virus vaccine is a live virus Persistent bacteraemia increases the risk of
vaccine. The other vaccines are not live complications. When the strain is sensitive to
vaccines. antistaphylococcal penicillins (e.g. flucloxacillin)
these should be prescribed in preference
6.6. Answer: C. to other agents. Penicillins work by
The symptoms of severe respiratory syndrome time-dependent killing so the efficacy is, in

--
and presence of a geographically restricted theory, improved by continuous infusion.
coronavirus raise the possibility of Middle East Increasing the dose would be appropriate only
respiratory syndrome (MERS) coronavirus. for antimicrobials that kill by dose-dependent
Travel-related cases have been seen in many killing (e.g. aminoglycosides).
1
countries but the majority of cases have initially
been associated with travel to countries in the 6.11. Answer: B.
Middle East. Voriconazole can cause photosensitive
dermatitis and patients should be advised to
6.7. Answer: B. avoid sun exposure or to use a high-level
The clinical scenario with fever, splenomegaly sunblock on light-exposed areas. The other
and abnormalities on the full blood count could side-effects are associated with other
be seen with most of these travel-associated antimicrobials.
infections, although at this late stage
Trypanosoma brucei would generally present 6.12. Answer: A.
with encephalopathy. The key feature here is All the listed choices except amantadine are
the incubation period. Of these infections, only active against herpes virus infections but for
leishmaniasis and sleeping sickness would someone with an end-organ complication such
present with such a long incubation period. The as pneumonia, high-dose intravenous therapy
clinical scenario suggests visceral leishmaniasis aciclovir would be indicated. Valaciclovir would
not sleeping sickness. be appropriate for milder disease and foscarnet
for someone who has an infection with
6.8. Answer: A. resistance to aciclovir as may occur in an
The cases are most consistent with a common immunocompromised patient with frequent
source outbreak that is linked to some exposure.
common environmental source. Since the
patients were cared for in different units, there 6.13. Answer: A.
is no evidence of hospital transmission. Atovaquone when combined with proguanil is
Person-to-person transmission in the active against Plasmodium falciparum. This is a
community would be a less likely scenario for good choice as it is well tolerated. Mefloquine
this organism. The numbers are not consistent is associated with neurocognitive side-effects
with a pandemic or epidemic. and can worsen symptoms of pre-existing
psychiatric conditions. Doxycycline is another
6.9. Answer: B. option but can cause photosensitive dermatitis
The mecA gene encodes a penicillin-binding and may not be a good choice for someone
protein with low affinity for penicillins, including who will be working out of doors. Resistance
antistaphylococcal penicillins, and is the means chloroquine is no longer a suitable
usual basis of resistance in meticillin-resistant option in most malaria-endemic areas.
Staph. aureus (MRSA). Presence of this lvermectin is used against helminths not
genetic element, which is often screened malaria.
for in Staph. aureus strains, necessitates
use of an antimicrobial other than an 6.14. Answer: A.
antistaphylococcal penicillin (e.g. fl1:1cloxacillin). Tetracyclines absorption is limitedby cations
vanA encodes a penicillin-binding protein that such as calcium, iron and antacids that contain
has low affinity for glycopeptides and is found aluminium or magnesium. The other
in enterococci. The other options are medications will not alter absorption.
~-lactamase or carbapenemase enzymes Tetracyclines should be taken at a different
that mediate resistance in Gram-negative time, several hours separate from the
bacteria. cation-containing medicine.

downloaded from www.medicalbr.com


36 • PRINCIPLES OF INFECTIOUS DISEASE
T
I
6.15. Answer: B. should always be considered. Of the
Although there are a range of potential antimicrobials listed, colistin is associated, in
causes of encephalopathy in a patient particular, with neurotoxicity including
who has received ventilation for critical illness encephalopathy.
including infection, medication-related causes

downloaded from www.medicalbr.com


SHL Thomas

Poisoning
Multiple Choice Questions
7.1. Which one of the following is most likely to D. lsosorbide mononitrate
produce significant toxicity if ingested E. Paracetamol
accidentally by a child?
A. A 1 em length of pencil lead 7.4. A family of four people living in Jamaica
B. One combined oral contraceptive tablet develop vomiting, diarrhoea and abdominal
C. One liquid laundry detergent capsule pain a few hours after eating a well-cooked
D. One mouthful of emulsion paint meal of snapper fish in a seafood restaurant.
E. One prednisolone 5 mg tablet This subsequently progresses to unsteadiness
of gait, blurred vision and tingling in the hands
7.2. A patient develops prolonged and recurrent and feet. Which of the following is the most
episodes of torsades de pointes associated likely diagnosis?
with no palpable cardiac output after an A. Aconite poisoning
overdose of sotalol. All of these interventions B. Ciguatera poisoning
may be useful except one. Which one is NOT C. Paralytic shellfish poisoning
likely to be helpful in the management of this D. Salmonella poisoning
situation? E. Scombrotoxic fish poisoning
A. Cardiac pacing
B. Correction of hypokalaemia 7.5. Which of the following is the most likely
C. Intravenous bolus dose of magnesium explanation for the following clinical features
sulphate in an adult patient after drug overdose:
D. Intravenous infusion of isoproterenol tachycardia, delirium, hallucinations, fever,
E. Intravenous bolus dose of procainamide diarrhoea, shivering, inducible prolonged
clonus, seizures, raised creatine kinase?
7.3. A 33 year old male attends the emergency A. Anticholinergic toxidrome
department with breathlessness and chest pain B. Intercurrent infection
after using a recreational substance/street drug. C. Recent use of gamma hydroxybutyrate
On examination he looks cyanosed and has a D. Serotonin syndrome
tachycardia (120 beats/min). On supplemental E. Stimulant toxidrome
oxygen, his arterial blood gases show W
52.5 nmoi/L (pH 7.28), PaC0 2 2.7 kPa 7.6. A 54 year old man presents unconscious.
(20.3 mmHg), Pa0 2 17.3 kPa (129.8 mmHg) His pulse is 88 and blood pressure 142/78.
and 35% methaemoglobinaemia. Which Initial investigations reveal normal urea and
of the following is the most likely causative electrolytes, creatinine of 101 11moi/L (1.14 mg/
agent? dl) and glucose 7.3 m111oi/L (131.4 mg/dl).
A. Cocaine Arterial blood gases results include W
B. Gamma hydroxybutyn'ite 81.3 nmoi/L (pH 7.09), PaC02 1.8 kPa
C. Isopropyl nitrite (13.5 mmHg), base excess of -13 mmoi/L,

downloaded from www.medicalbr.com


38 • POISONING

carboxyhaemoglobin of 2% and lactate of intentional ingestion of an insecticide product


17.3 mmoi/L (155.9 mg/dl). Paracetamol and used on the farm where she lives. Her features
salicylate are not detected. A computed progress to flaccid paralysis of lower and
tomography (CT) scan of the head is normal. upper limbs with reduced tendon reflexes.
Which of the following is the most likely A diagnosis of organophosphate-induced
diagnosis? delayed polyneuropathy (OPIDN) is made.
A. Carbon monoxide poisoning Which of the following is the most likely
B. Cyanide poisoning causative agent?
C. Diabetic ketoacidosis A. Bendiocarb
D. Ethylene glycol poisoning B. Dichlorvos
E. Salicylate poisoning C. Methomyl
D. Sarin
7.7. A 34 year old painter/decorator presents E. Tabun
with fatigue and anaemia after spending several
weeks thermally stripping lead-based paint 7.11. A 21 year old man presents to hospital
without using adequate personal protective after snorting a white powder purchased as a
equipment. His blood lead concentration is 'research chemical' via the internet. He
substantially elevated. Which of the following develops tachycardia and hypertension. Which
chelating agents is the most appropriate of the following compounds is most likely to
antidote for treatment of chronic lead have been in the powder used?
poisoning? A. Cannabis
A. Desferrioxamine B. Gamma hydroxybutyrate
B. Dicobalt edetate C. Heroin
C. Dimercaprol D. Isobutyl nitrite
D. Dimercaptosuccinic acid (DMSA) E. Mephedrone
E. Hydroxocobalamin
7.12. Two adult males present to hospital after
7.8. Which of the following results indicates the they both used the same unidentified /
largest anion gap (all results are in mmoi/L)? recreational substance. Both have a Glasgow
A. Sodium 119, potassium 2.6, chloride 90, Coma Scale score <9 and bradycardia./Pupils
bicarbonate 27 are mid-sized. One has been incontine,nt for
B. Sodium 131, potassium 4.3, chloride 96, urine and both display myoclonus. Blobd gases
bicarbonate 18 show a mixed respiratory and metabolic
C. Sodium 135, potassium 5.6, chloride 101, acidosis in both. What substance is rn,ost likely
bicarbonate 24 to be responsible? /
D. Sodium 136, potassium 4.1, chloride 102, A. Diazepam
bicarbonate 25 B. Ethanol
E. Sodium 149, potassium 3.9, chloride 94, C. Gamma hydroxybutyrate
bicarbonate 21 D. Heroin
E. Ketarnine
7.9. A 24 year old man presents to hospital
after smoking a herbal mixture. Which of the 7.13. In relation to ingestion of a toxic
following features are characteristic of exposure substance, which of the following
to synthetic cannabinoid receptor agonists characteristics make a substance less
(SCRAs) but are not usually associated with appropriate for removal by haernodialysis?
use of herbal cannabis? A. Large volume of distribution
A. Ataxia B. Long half-life .·
B. Conjunctival irritation C. Low molecular w~ight
C. Psychosis D. Non-polar molecule
D. Seizures E. Not bound to activated charcoal
E. Tachycardia
7.14. An 18 year old male presented to hospital
7.10. A 23 year old female develops muscle after a deliberate overdose involving unknown
cramps and numbness about 3 weeks after an substances. At presentation there was a high

downloaded from www.medicalbr.com


POISONING • 39

concentration of paracetamol in the blood. A. Autumn crocus (Colchicum autumnale)


Over the subsequent 3 days he developed B. Deadly nightshade (Atropa belladonna)
worsening abnormalities of liver function C. Jequirity bean (Abrus precatorius)
(elevated bilirubin and alanine transaminase), D. Laburnum (Laburnum anagyroides)
clotting (raised international normalised ratio) E. Wolf's bane (Aconitum nape/Ius)
and renal function (elevated creatinine) and
episodes of hypoglycaernia. All the features 7.17. A 17 year old female patient presents
below are consistent with paracetamol unconscious having had a witnessed seizure
overdose, except one. Which feature is after an intentional drug overdose. Following
most likely to be caused by another the seizure she is drowsy but vital signs are
substance? otherwise satisfactory. Which of the following is
A. Acute kidney injury most likely to be responsible?
B. Early unconsciousness A. Amlodipine
C. Elevated international normalised ratio B. Diazepam
D. Hypoglycaemia C. Digoxin
E. Liver failure D. Mefenamic acid
E. Paracetamol
7.15. A 23 year old man develops toxicity after
deliberately ingesting a carbamate insecticide. 7.18. Deliberate release of a chemical warfare
Clinical features include headache, vomiting, agent is suspected when 12 people develop
diarrhoea, hypersalivation, abdominal pain, similar symptoms after an explosion on an
tachycardia, muscle weakness, fasciculation underground train. Initially these include anxiety,
and reduced ventilation due to respiratory breathlessness, vomiting and headache, but
muscle involvement. Which one of the following five cases have also developed convulsions.
is an example of a cholinergic (muscarinic) Three patients have become unconscious and
effect? two have died. Survivors have dilated pupils
A. Fasciculation and no respiratory tract or skin abnormalities
B. Mydriasis are found. Investigations show raised plasma
C. Respiratory muscle paralysis lactate but plasma acetylcholinesterase activity
D. Salivation is normal. This clinical picture is most
E. Tachycardia characteristic of exposure to which of the
following?
7.16. A 2 year old child develops tachycardia, A. Cyanide
delirium, fever associated with a flushed face, B. Lewisite
mydriasis and seizures after eating part of a C. Phosgene
plant. Which of the following is most likely to be D. Sarin
responsible? E. Sulphur mustard

Answers
7.1. Answer: C.
These are all substances of low toxicity (Box
7.1 Substances of very low toxicity
7.1) with the exception of liquid laundry
detergent capsules, which can cause CNS Writing/educational materials, e.g. pencil lead, crayons,
chalk
depression in children and are also corrosive,
Decorating products, e.g. emulsion paint, wallpaper paste
sometimes causing stridor, pulmonary Cleaning/bathroom products (except dishwasher tablets
aspiration and airway burns, as well as ocular and liquid laundry detergent capsules, which can be
damage if the liquid gets into the eyes. corrosive)
Pharmaceuticals: oral contraceptives, most antibiotics (but
not tetracyclines or antituberculous drugs), vitamins B, C
7.2. Answer: E. and E, prednisolone, emolliWlts and other skin creams,
These are all useful interventions for torsades baby lotion .
de pointes except procaihamide. Magnesium Miscellaneous: plasticine, silica gel, most household
plants, plant food, pet food, soil
sulphate reduces the risk of torsades without

downloaded from www.medicalbr.com


40 • POISONING

7.2 Complications of poisoning and their management


Complication Examples of causative agents Management
Coma Sedative agents Appropriate airway protection and ventilatory support·
Oxygen saturation and blood gas monitoring
Pressure area and bladder care
Identification and treatment of aspiration pneumonia
Seizures NSAIDs Appropriate airway and ventilatory support
Anticonvulsants IV benzodiazepine (e.g. diazepam 10-20 mg,
TCAs lorazepam 2-4 mg)
Theophylline Correction of hypoxia, acid-base and metabolic
abnormalities
Acute dystonias Typical antipsychotics Procyclidine, benzatropine or diazepam
Metoclopramide
Hypotension
Due to vasodilatation Vasodilator antihypertensives IV fluids
Anticholinergic agents Vasopressors (rarely indicated)
TCAs
Due to myocardial suppression ~-blockers Optimisation of volume status
Calcium channel blockers Inotropic agents
TCAs
Ventricular tachycardia
Monomorphic, associated with Sodium channel blockers Correction of electrolyte and acid-base abnormalities
QRS prolongation and hypoxia
Sodium bicarbonate (e.g. 50 ml 8.4% solution,
repeated if necessary)
Torsades de pointes, Anti-arrhythmic drugs (quinidine, Correction of electrolyte and acid-base abnormalities
associated with QT, amiodarone, sotalol) and hypoxia
prolongation Antimalarials Magnesium sulphate, 2 g IV over 1-2 mins, repeated
Organophosphate insecticides if necessary
Antipsychotic agents
Antidepressants
Antibiotics (erythromycin)
'
(NSA/0 = non-steroidal anti-inflammatory drug; TCA = tricyclic antidepressanlj /

affecting the QT interval and is first -line 7.4. Answer: B.


treatment (Box 7.2). Isoproterenol and pacing The clinical features could be consistent with
increase the underlying ventricular rate and options B or C but the association wit!l .eating
reduce the risk of recurrence, as torsades is a predator fish suggests ciguatera, which lis
bradycardia-dependent arrhythmia, although prevalent in the Caribbean. These neurological
they are infrequently needed. Hypokalaemia features are not found with Salmonella
worsens the risk of torsades, so correction poisoning. Scombrotoxic fish poisoning causes
is beneficial. Procainamide is a class Ia symptoms associated with histamine release,
anti-dysrhythmic drug that further prolongs which may include gastrointestinal disturbance,
ventricular repolarisation and would worsen the but neurological features are not characteristic.
risk of torsades. Aconite can also cause gastrointestinal
disturbances and paraesthesia but a plant is
7.3. Answer: C. less likely to be involved in this episode.
Methaemoglobinaemia is commonly caused by
organic nitrites, such as isopropyl nitrite, which 7.5. Answer: D
oxidise haemoglobin. Other causes are shown These are all characteristic features of serotonin
in Fig. 7.3. The other substances listed in syndrome (Box 7.5A). Inducible clonus and
the question do not have this effect directly. shivering are not characteristic of the
Note, however, that occasionally cocaine is anticholinergic or stimulant toxidromes or
contaminated with oxidising adulterants such gamma hydroxybut~rate (GHB) toxicity
as benzocaine or phenacetin, which may (although myoclonus may .be seen after GHB;
occasionally produce unexpected Box 7.5B). Clonus would not suggest
methaemoglobinaemia. intercurrent infection.

downloaded from www.medicalbr.com


POISONING • 41

Causes Consequences Treatment


Non-toxic • Haemoglobin-oxygen • Methylthioninium chloride
• Congenital methaemoglobinaemias dissociation curve is shifted ('methylene blue') 1-2 mg/kg
to the left (intravenous) is given
Toxic (Oxidising agents)
• Oxygen delivery to tissues • Reduces methaemoglobin
• Organic nitrites is reduced (see below)
• Nitrates • There is apparent 'cyanosis' • Used for symptomatic
• Benzocaine patients with severe
• Breathlessness, fatigue,
• Dapsone headache and chest pain methaemoglobinaemia
• Chloroquine occur (e.g. >30%)
• Aniline dyes • Delirium, impaired • Patients with anaemia or
• Chlorobenzene consciousness and seizures other comorbidities may

-
• Naphthalene may occur in severe cases need treatment at lower
• Copper sulphate concentrations

NADH Methylthioninium 8 NADP*


chloride (reduced} *·-

NAD Haemoglobin (Fe2+) Methylthioninium.43t NADPH


chloride (oxidised}

Fig. 7.3

7.6. Answer: B.
7.5A Anticholinergic and serotonergic
feature clusters This patient has marked metabolic acidosis
with greatly elevated lactic acid. While all of
Anticholinergic Serotonin syndrome
these diagnoses may cause metabolic acidosis,
Common Benzodiazepines SSRis
causes Anti psychotics MAOIs only cyanide and carbon monoxide are
TCAs TCAs associated with elevated lactic acid in the
Antihistamines Amphetamines absence of cardiovascular shock. Carbon
Scopolamine Tryptamines monoxide poisoning has probably been
Benzatropine Buspirone
Belladonna Bupropion excluded by the normal carboxyhaemoglobin
Some plants and (especially in result, unless exposure was many hours earlier.
mushrooms (see combination) Diabetic ketoacidosis and salicylate poisoning
Box 7.16B) are also excluded by the normal blood glucose
Clinical features and salicylate concentration, respectively.
Cardiovascular Tachycardia, Tachycardia,
hypertension hyper- or
hypotension 7.7. Answer: D.
Central Delirium, Delirium, Lead poisoning can be treated with oral DMSA
nervous hallucinations, hallucinations, (also called succimer) or parenteral sodium
system sedation sedation, coma
calcium edetate (Box 7.7). Indications for the
Muscle Myoclonus Shivering, tremor,
other listed chelating agents include poisoning
myoclonus, raised
creatine kinase with cyanide {hydroxocobalamin, dicobalt
Temperature Fever Fever edetate) or iron (desferrioxamine). Dimercaprol
Eyes Diplopia, mydriasis Normal pupil size has been used for heavy metal poisoning,
Abdomen Ileus, palpable Diarrhoea, vomiting including mercury, but has now been largely
bladder superseded by other chelating agents, because
Mouth Dry these are better tolerated.
Skin Flushing, hot, dry Flushing, sweating
Complications 'seizures Seizures
Rhabdomyolysis 7.8. Answer: E.
Renal failure The anion gap measures the difference
Metabolic acidosis between measured catiQns (sodium and
Coagulopathies potassium) and anions {chloride and
(MAO!= monoamine oxidase inhibitor,· SSRI = selective bicarbonate) and is usually calculated as [Na+ +
serotonin re-uptake inhibitor; TCA = tricyclic antidepressanV K+]- [cl- + HC03-]. The 'gap' reflects the

downloaded from www.medicalbr.com


42 • POISONING TI
~

"' ; 7.58 Stimulant, sedative and opioid feature clusters


Stimulant Sedative hypnotic Opioid
Common Amphetamines Benzodiazepines Heroin
causes MDMA ('ecstasy') Barbiturates Morphine
Ephedrine Ethanol Methadone
i Pseudoephedrine GHB Oxycodone
Cocaine Dihydrocodeine
Cannabis Codeine
Phencyclidine Pethidine
Cathinones (e.g. mephedrone) Dipipanone
Benzylpiperazine Buprenorphine
Dextropropoxyphene
Tramadol
Clinical features
Respiratory Tachypnoea Reduced respiratory rate and Reduced respiratory rate and
ventilation* ventilation
Cardiovascular Tachycardia, hypertension Hypotension* Hypotension, relative
bradycardia
Central nervous Restlessness, anxiety, anorexia, Delirium, hallucinations, Delirium, hallucinations,
system insomnia slurred speech slurred speech
Hallucinations Sedation, coma* Sedation, comat
Muscle Tremor Ataxia, reduced muscle tone Ataxia, reduced muscle tone
Temperature Fever Hypothermia Hypothermia
Eyes Mydriasis Diplopia, strabismus, Miosis
·nystagmus
Normal pupil size
Abdomen Abdominal pain, diarrhoea - Ileus
Mouth Dry - -
Skin Piloerection Blisters, pressure sores Needle tracks1
Complications Seizures Respiratory failure* Respiratory failuret
Myocardial infarction Aspiration Non~cardiogenic pulmonary
Dysrhythmias oedema I
I
Rhabdomyolysis Aspiration I
Renal failure
Intracerebral haemorrhage or infarction
'Especially barbiturates.
1/V use.

(GHB = gamma hydroxybutyrate; MDMA = 3,4-methy/ene-dioxymethamphetamine) ;

7.7 Specific antidotes used to treat poisoning


Mechanism of action Examples of antidote Poisoning treated
Glutathione repleters Acetylcysteine Paracetamol
Methionine
Receptor antagonists Naloxone Opioids
Flumazenil Benzodiazepines
Atropine Organophosphorus compounds
Carbamates
Alcohol dehydrogenase inhibitors Fomepizole Ethylene glycol
Ethanol Methanol
Chelating agents Desferrioxamine Iron
Hydroxocobalamin Cyanide
Dicobalt edetate
DMSA Lead
Sodium calcium edetate
Reducing agents Methylthioninium chloride Organic nitrites
Cholinesterase reactivators Pralidoxime Organophosphorus compounds
Antibody fragments Digoxin Fab fragments Digoxin
(DMSA = dimercaptosuccinic acid)

downloaded from www.medicalbr.com


POISONING • 43

tachycardia and hypertension would be typical


7.8 Anion and osmolar gaps in poisoning
stimulant effects. Cannabis is not a white
Anion gap Osmolar gap
powder. While pure heroin (diamorphine) is
Calculation [Na* + K*] [Measured
white, street heroin is usually discoloured
- [CI- + HCO,-] osmolality] - [(2
x Na) +Urea+ brown and smoked or injected rather than
Glucose]* snorted. GHB and isobutyl nitrite are liquids.
Reference range 12-16 mmoi/L <10 None of these others are likely to cause
Common toxic Ethanol Ethanol tachycardia and hypertension.
causes of Ethylene glycol Ethylene glycol
elevation Methanol Methanol
7.12. Answer: C.
Salicylates
Iron These features (especially myoclonus and
Cyanide incontinence) are characteristic of poisoning
*All units should be in mmo!IL. For non-S! units, the with GHB and the related substances gamma
corresponding formula is [Osm (measured)]- [(2 x Na butyrolactone (GBL) and 1 ,4 butanediol. The
(mEq/L)) + Urea/2.8 (mgldL) + G/ucose/18 (mgldL)]. other four substances can all cause coma but
bradycardia and incontinence are not typical.
Small pupils would be seen with heroin toxicity.
concentrations of unmeasured anions such as
albumin and phosphate and the normal range 7.13. Answer: A.
is 12-16 mmoi/L. Anion gap is increased by Characteristics associated with efficacy for
the presence of toxic anions such as the haemodialysis as a treatment for poisoning
metabolites of ethylene glycol or methanol. include small volume of distribution (high
In these examples, the calculated values are: proportion of body load in the blood/plasma),
(A) 4.6; (B) 21.3; (C) 15.6; (D) 13.1; and (E) low molecular weight, lipid soluble/non-polar at
37.9 mmoi/L. For more information see Box 7.8. physiological pH (can cross dialysis membrane)
N.B. for simplicity some recommend ignoring and a long half-life after overdose. Binding to
potassium in the calculation of anion gap (anion activated charcoal is not relevant for
gap= Na+- [CI- + HC03-]), as potassium haemodialysis (only haemoperfusion).
makes a limited contribution. Using this
calculation, the normal range is 8-12 mmoi/L. 7.14. Answer: B.
Use of this method gives the same answer for Paracetamol toxicity is characteristically
this question. associated with liver failure (including
!
I
hypoglycaemia and abnormal clotting) and rehal
7.9. Answer: D. failure. Paracetamol does not generally cause
Tachycardia, ataxia, conjunctival irritation and unconsciousness, although this can be a /' r
psychosis can all be characteristic of exposure feature of late-stage liver failure. It may also be
to both cannabis and synthetic cannabinoid caused by co-ingestants.
receptor agonists. Seizures, however, are often
reported after use of SCRAs but rarely after use 7.15. Answer: D.
of cannabis. Unlike cannabis, SCRAs have also These could all be cholinergic effects, but
been reported to cause hypokalaemia, acute tachycardia, reduced ventilation, mydriasis and
kidney injury and coma with respiratory acidosis. fasciculation are all cholinergic nicotinic (rather
than muscarinic) effects (Box 7 .15).
7.10. Answer: B.
Dichlorvos is an organophosphate insecticide 7.16. Answer: B.
that can sometimes cause OPIDN, but this The child has features consistent with an
complication is rare with nerve agents (e.g. anticholinergic toxidrome (Box 7.16A). Plants
sarin, tabun) and non-organophosphate with this effect include Atropa belladonna
carbamate insecticides (e.g. methornyl, (deadly nightshade), Datu~ stramonium Qimson
bendiocarb). Nerve agents would not be found weed, thorn apple) andBrugmansia spp.
on farms. (angel's trumpet). While ingestion of jequirity
beans, wolf's bane and laburnum could
7.11. Answer: E. potentially also cause convulsions,
Mephedrone is a white powder commonly anticholinergic effects would not be expected.
taken by nasal insufflation ('snorting'), and Autumn crocus causes gastrointestinal effects,

downloaded from www.medicalbr.com


i
44 • POISONING

7.15 Cholinergic features in poisoning*


Muscarinic Nicotinic
Respiratory Bronchorrhoea, bronchoconstriction Reduced ventilation
Circulation Bradycardia, hypotension Tachycardia, hypertension
Higher mental function Anxiety, delirium, psychosis
Muscle - Fasciculation, paralysis
Temperature Fever -
Eyes Diplopia, miosis, lacrimation Mydriasis
Abdomen Vomiting, profuse diarrhoea -
Mouth Salivation -
Skin Sweating -
Complications Coma, seizures, respiratory depression
'Both muscarinic and nicotinic features occur in organophosphorus poisoning. Nicotinic features occur in nicotine poisoning and
black widow spider bites. Cholinergic features are sometimes seen with some mushrooms.

7.16A Anticholinergic and serotonergic feature clusters


Anticholinergic Serotonin syndrome
Common causes Benzodiazepines SSRis
Antipsychotics MAOis
TCAs TCAs
Antihistamines Amphetamines
Scopolamine Tryptamines
Benzatropine Buspirone
Belladonna Bupropion (especially in combination)
Some plants and mushrooms (see Box 7.16B)
Clinical features
Cardiovascular Tachycardia, hypertension Tachycardia, hyper- or hypotension
Central nervous system Delirium, hallucinations, sedation Delirium, hallucinations, sedation, coma
Muscle Myoclonus Shivering, tremor, myoclonus, raised
creatine kinase I
Temperature Fever Fever (
Eyes Diplopia, mydriasis Normal pupil size
Abdomen Ileus, palpable bladder Diarrhoea, vomiting
'
Mouth Dry
Skin Flushing, hot, dry Flushing, sweating
Complications Seizures Seizures I I

Rhabdomyolysis '
Renal failure
Metabolic acidosis
Coagulopathies
(MAO/= monoamine oxidase inhibitor; SSRI = selective serotonin re-uptake inhibitor; TCA = tricyclic antidepressan~

hypotension and cardiogenic shock poisoning but are uncommon and likely to
(Box 7.16B). be associated with severe cardiovascular
effects.
7.17. Answer: D.
Mefenamic acid is a non-steroidal 7.18. Answer: A.
anti-inflammatory drug (NSAID) with a very high These are typical features of cyanide poisoning.
propensity to cause seizures. Other common Sarin can also cause vomiting, breathlessness
causes of seizures in the context of drug (associated with bronchospasm and
overdose include tricyclic antidepressants, bronchorrhoea) and convulsions, but small
antipsychotic drugs, antiepileptic drugs, other pupils, abnormalities on respiratory examination
NSAIDs (although much less commonly than and reduced plasma. or red cell
with mefenamic acid), anticonvulsants and acetylcholinesterase activity would be expected.
theophylline (Box 7.17). Paracetamol, diazepam Sulphur mustard, lewisite and phosgene can
and digoxin do not cause seizures. Seizures cause respiratory effects but convulsions and
have been reported with severe amlodipine coma would not be expected.

downloaded from www.medicalbr.com


POISONING • 45

7.16B Some poisonous plants and their clinical effects


Species (common name) Toxins Important features of toxicity
Plants
Abrus precatorius Oequirity bean) Abrin Gastrointestinal effects, drowsiness, delirium,
Ricinus communis (castor oil plant) Ricin convulsions, multi-organ failure
Aconitum nape/Ius (aconite, wolf's bane, Aconite Gastrointestinal effects, paraesthesiae, convulsions,
monkshood) ventricular tachycardia
Aconitum ferox (Indian aconite, bikh)
Atropa belladonna (deadly nightshade) Atropine, Anticholinergic toxidrome (see Box 7.16A)
Datura stramonium (Jimson weed, thorn apple) scopolamine,
Brugmansia spp. (angel's trumpet) hyocyamine
Colchicum autumnafe (autumn crocus) Colchicine Gastrointestinal effects, hypotension, cardiogenic
shock
Conium maculatum (hemlock) Toxic nicotinic Hypersalivation, gastrointestinal effects, followed by
alkaloids muscular paralysis
Digitalis purpurea (foxglove) Cardiac glycosides Cardiac glycoside toxicity
Nerium oleander (pink oleander)
Thevetia peruviana (yellow oleander)
Laburnum anagyroides (laburnum) Cytosine Gastrointestinal effects; convulsions in severe cases
Taxus baccata (yew) Taxane alkaloids Hypotension, bradycardia, respiratory depression,
convulsions, coma, arrhythmias
Fungi
Amanita phalloides (death cap mushroom) Amatoxins Gastrointestinal effects, progressing to liver failure
Cortinarius spp. Orellanine Gastrointestinal effects, fever, progressing to renal
failure
Psi/ocybe semilanceata ('magic mushrooms') Psilocybin, psilocin Hallucinations

7.17 Complications of poisoning and their management


Complication Examples of causative agents Management
Coma Sedative agents Appropriate airway protection and ventilatory support /

Oxygen saturation and blood gas monitoring


Pressure area and bladder care
~
Identification and treatment of aspiration pneumonia
Seizures NSAIDs Appropriate airway and ventilatory support
Anticonvulsants IV benzodiazepine (e.g. diazepam 10-20 mg, lorazepam 2-4/ng)
TCAs Correction of hypoxia, acid-base and metabolic abnormalities •
Theophylline
Acute dystonias Typical antipsychotics Procyclidine, benzatropine or diazepam
Metoclopramide
Hypotension
Due to Vasodilator antihypertensives IV fluids
vasodilatation Anticholinergic agents Vasopressors (rarely indicated)
TCAs
Due to myocardial J3-blockers Optimisation of volume status
suppression Calcium channel blockers Inotropic agents
TCAs
Ventricular tachycardia
Monomorphic, Sodium channel blockers Correction of electrolyte and acid-base abnormalities and hypoxia
associated with Sodium bicarbonate (e.g. 50 mL 8.4% solution, repeated if
QRS prolongation necessary)
Torsades de Anti-arrhythmic drugs (quinidine, Correction of electrolyte and acid-base' abnormalities and
pointes, amiodarone, sotalol) hypoxia ·
associated with Antimalarials Magnesium sulphate, 2 g IV over 1-2 mins, repeated if
QT, prolongation Organophosphate insecticides necessary
Antipsychotic agents
Antidepressants
.
Antibiotics (erythromycin)
(NSAID = non-steroidal anti-inflammatory drug; TCA = tricyclic antidepressanO

downloaded from www.medicalbr.com


J White

Envenomation

Multiple Choice Questions


8.1. A patient presenting with suspected 8.3. Considering the patient in Question 8.2,
envenoming requires a rapid but careful and above, if we knew he came from somewhere in
targeted assessment to determine if !ndia or Sri Lanka, given the particular
envenoming is present and what urgent circumstances of this bite, and that testing
responses are required. Which is the most showed a coagulopathy, which of the following
important first response in ensuring a good venomous animals would be most likely as a
outcome? cause for his bite?
A. Check for breathing and circulatory problems A. Cobra
and institute cardiopulmonary resuscitation B. Indian red scorpion
(CPR) if indicated C. Krait
B. Give antivenom D. Russell's viper
C. If the animal that caused the bite/sting is E. Saw-scaled viper
available, try and identify it, so that likely
risks can be determined and targeted 8.4. Our patient in Questions 8.2 and
treatment provided 8.3 is clearly envenomed and needs
D. Insert an intravenous (IV) line and consider antivenom urgently. In giving him antilie,nom,
an early IV fluid load which is the most important drug to have
E. Urgently examine for signs of developing available?
neurotoxic flaccid paralysis and bleeding A. Adrenaline (epinephrine)
tendency secondary to coagulopathy B. Antihistamine
C. Dopamine
8.2. A 32 year old man presents to your D. Hydrocortisone
hospital with a history of working in a rice E. Prazosin
paddy and being bitten by something, not
seen. He appears unwell and has swelling 8.5. The patient in Questions 8.2 to 8.4 has
around the bite on his foot, with two bite marks now developed a low urine output, despite
that are bleeding. What is a key test that might adequate IV fluids. In this small rural hospital
help you determine the type of snake and need setting, what test might best help in deciding
for antivenom? that acute kidney injury and renal failure is
A. 20-Minute whole-blood clotting test occurring?
(20WBC1) A. Dipstick test for proteinuria
B. Arterial blood gas B. Measure serum creatinine level
C. Blood pressure C. None of the above
D. Extended coagulation studies D. Renal biopsy
E. Serum electrolytes E. Renal ultrasound

downloaded from www.medicalbr.com


""'"-- ----
ENVENOMATION • 47

8.6. A patient presents at a small rural hospital data is considered, by experts, to be a reliable
you are working in, with a history of snakebite figure indicating the impact of snakebite?
and on investigation she has local bruising and A. Snakebite causes about 3000 deaths per
swelling around the bite site with oozing of year in India
blood, a coagulopathy, renal failure and B. Snakebite causes about 45 000 deaths per
developing flaccid neurotoxic paralysis. In which year in India
country is this hospital likely to be? C. Snakebite causes more than 200 000 deaths
A. Bangladesh per year worldwide
B. Burma D. Snakebite causes only about 20 000 deaths
C. India per year worldwide
D. South Africa E. Snakebite is less important than
E. Sri Lanka land mines in causing injuries requiring
an amputation
8.7. You are working in a hospital in Brazil and
you are asked to assess a young man with a 8.11. You are working in a hospital in northern
history of snakebite, occurring at around dusk. England, near areas of national parks, when a
He did not see the snake properly, so cannot 10 year old boy is presented with a history of
provide a description. The bite site is not stepping on and being bitten by a snake while
showing much swelling or local pain, but there playing in his garden, near natural parkland.
are obvious fang marks and he has bilateral The boy's father, who was not present when
ptosis and a positive 20WBCT. Which snake is the bite occurred, describes the snake as grey
the most likely cause? in colour, with indistinct darker markings along
A. Coral snake (Micrurus frontalis) the sides of the body and a pale narrow band,
B. Green racer (Philodryas o/fersit) like a collar, behind the head. What type of
C. Jararaca (Bothrops jararaca) snake might this be?
D. Neotropical rattlesnake or cascabel (Crotalus A. A European adder (Vipera berus)
durissus terrificus) B. A form of legless lizard ('slow worm', Anguis
E. Tiger snake (Notechis scutatus) tragi/is)
C. A grass snake (Natrix natrix) I

I
8.8. You are working in a hospital in rural D. A smooth snake (Carone/la austri<;J.ca)
Nigeria. A patient presents with a snakebite E. An escaped exotic snake, most likely a small
and already has local swelling and bruising mamba
around the bite site. A 20WBCT is positive
(blood not clotted at 20 minutes). Which of the 8.12. With regard to the patient in Question 1 ,
following snakes is most likely involved? 8.11 , this boy has now developed significant
A. Black-necked spitting cobra (Naja nigricollis) local swelling and bruising around the bite site,
B. Forest cobra (Naja melanoleuca) which is painful and swelling extends to much
C. Green mamba (Dendroaspis jamesom) of the bitten limb. He appears shocked, has
D. Puff adder (Bitis arietans) poor urine output and the preliminary report
E. Saw-scaled (carpet) viper (Echis ace/latus) from the laboratory indicates he may have a
coagulation abnormality. Questioning also
8.9. For a forest cobra bite (Naja melanoleuca), reveals that the father's description of the
which of the following is likely to be the most snake may not be accurate. What do you now
useful and effective first aid, if applied think is most likely to have bitten this boy?
correctly? A. A European adder (Vipera berus)
A. Electric shock B. A form of legless lizard ('slow worm', Anguis
B. Pressure bandage and immobilisation (PBI) tragi/is)
1
C. None of these listed C. A grass snake (Natrix 9atrix)
D. Scarification of the bite site D. A smooth snake (Carone/la austriaca)
E. Tourniquet E. An escaped exotic snake, most likely a small
mamba
8.1 0. Snakebite is variously claimed to be either
an important or quite unimportant medical 8.13. A young man presents to the emergency
problem. Which of the following epidemiological department of the London hospital you are

downloaded from www.medicalbr.com


48 • ENVENOMATION

working in, claiming he has been bitten spider it is. What sort of spider would you be
by a large spider that he was keeping concerned about?
as a pet. The bite occurred 6 hours ago, has A. Australian funnel web spider
been very painful locally and his attempts to B. Black widow spider
control the pain with oral analgesia have failed. C. Brazilian wandering spider
He was given the spider by a friend who D. Brown recluse spider
worked for an importer of fruit such as E. Mexican orange-kneed tarantula
bananas. He does not know what type of

Answers
8.1. Answer: A. for envenoming. Serum electrolytes may be
Options C, D and E are all necessary urgent useful, but not critical to initial assessment, and
requirements in managing a patient with may be hard to obtain in a rural hospital
suspected envenoming, but ensuring there is setting, as may extended coagulation studies.
no problem with the classic 'ABC' of airway, The latter will take far more time to provide an
breathing and circulation, and treating any answer than will the 20WBCT. Arterial blood
problems found, is the most urgent action. gas is not a critical test in initial assessment of
Snakebite patients, particularly in Asia, still die snakebite and if there is a coagulopathy,
unnecessarily because bystanders and health insertion of the needle or a line may pose
professionals forget about the ABC and fail to significant risks for the patient.
provide airway protection and external
respiratory support, when required, following 8.3. Answer: D.
envenoming by neurotoxic snakes such as The history tells us he was bitten in a paddy
kraits and some cobras. field, so likely a wetlands agricultural area, a
Option B might seem like an obvious answer, classic setting for a Russell's viper (Daboia
but not every patient bitten/stung by a russelii and Daboia siamensis) and this snal)e
venomous animal will develop medically causes coagulopathy, plus other effects. In I'
r!
significant envenoming, therefore not every some areas, such as Myanmar, Russell's 1ipers
patient needs antivenom. The other issue is also commonly cause acute renal failure. , ·
choosing which antivenom to use, and what Saw-scaled vipers (Echis spp.) also caJse
dose to administer, particularly in countries with coagulopathy, but tend to inhabit dry areas
several different antivenoms. In countries such rather than paddy fields. However, there are
as India, where only a polyvalent antivenom is sometimes exceptions, so knowing preci~ely
available, there is no requirement to delay while where the bite occurred and matching that to
choosing the right antivenom, but that does not the known local venomous fauna might assist
imply every patient should be given antivenom, in deciding if a saw-scaled viper might be the
so CPR, if indicated, takes precedence. cause. Cobras (Naja spp.) do not cause
coagulopathy, neither do kraits (Bungarus spp.)
8.2. Answer: A. nor Indian red scorpions (Hottentotta tamu/us).
A number of snakes may cause rapid
envenoming with development of a 8.4. Answer: A.
coagulopathy that can present as prolonged The greatest risk when giving antivenom is an
bleeding from the bite site, any other recent anaphylactic reaction, which, if not correctly
wound, or the gums. The 20WBCT is a rapid managed, may prove fatal. Managing
and simple test that can provide a useful guide anaphylaxis requires a multifaceted approach,
to the presence of snakebite coagulopathy. but the key drug is adr~naline (epinephrine), in
That, in turn, can help in determining what type most instances administered intramuscularly.
of snake may be involved and, if there are Adrenaline should always be immediately
several different antivenoms available, which available, in an appropriate dose, prior to giving
one to consider using. any antivenom.
Blood pressure is certainly an important test, Adrenaline, as a dilute subcutaneous
but an abnormal result is not a specific marker injection, has also been suggested as a

downloaded from www.medicalbr.com


ENVENOMATION • 49

pre-medication prior to giving antivenom, in classically cause paralysis, but not


certain situations and countries, a practice now coagulopathy. The green racer may, in some
supported by clinical trials in Sri Lanka, where cases, cause coagulopathy, but not paralysis,
its use was associated with a reduced though like most non-front-fanged colubrid
incidence of anaphylactic reactions to (NFFC) snakes, the clinical picture is not well
antivenom. defined.
Hydrocortisone and antihistamines have also
been suggested as pre-medication prior to 8.8. Answer: E.
giving antivenom, but clinical trial evidence The saw-scaled vipers routinely cause both
does not support their use. Prazosin has, in the significant tissue damage around the bite site
past, been the recommended treatment for and a severe coagulopathy.
envenoming by the Indian red scorpion, but Forest cobras cause only minor local bite site
more recent clinical trial evidence indicates that effects, major flaccid paralysis and do not
specific antivenom is more effective. Dopamine cause coagulopathy. Black-necked spitting
might be required in managing intractable cobras can cause severe local tissue injury, but
hypotension, but is not an alternative to not coagulopathy. Puff adders cause severe
adrenaline as the drug of first choice in treating local tissue injury, but not a clear coagulopathy,
anaphylaxis. although there may be bruising and oozing of
blood around the bite site. Green mambas
8.5. Answer: A. cause local and paralytic effects, not
In this setting it is likely that a simple urine coagulopathy.
dipstick test will be available and if this shows
proteinuria, in this clinical scenario, it is very 8.9. Answer: B.
likely the patient is developing renal failure. The forest cobra causes principally flaccid
A serum creatinine level may certainly help, neurotoxic paralysis, without major local tissue
but even if available, may take time to gain an damage around the bite site. Therefore, the
answer. Renal ultrasound is unlikely to be most important consideration is slowing venom
available and may not assist in diagnosis. Renal movement from the bite site to the rest of the
biopsy is inappropriate given the active body, a process that initially occurs particularly
coagulopathy. via the lymphatic system. The Australian-
developed PBI first aid is the most
8.6. Answer: E. effective safe way of achieving this.
The combination of coagulopathy, renal failure Because the pressure is applied to the
and neurotoxic flaccid paralysis is classic for Sri bite area and bitten limb, it has the
Lankan Russell's viper envenoming. It is rarely potential, for bites by snakes causing exter;Jsive
seen elsewhere within the range of this snake, bite site tissue injury, to worsen that injury. i
although there are a few reports from part of Therefore, the PBI method is not recommended
southern India. If the hospital was in south for bites by snakes likely to cause such local
eastern Australia (not an option listed), then a tissue injury; for these snakes, simple
tiger snake (Notechis scutatus) bite should be immobilisation of the bitten limb is
considered. recommended instead.
Tourniquets, while effective in the short term
8.7. Answer: D. in preventing venom movement, are painful and
Because of the likely snake fauna in Brazil, have a well-established reputation for
early development of paralysis (as evidenced by causing major ischaemic limb injury,
ptosis) plus coagulopathy is typical of a often necessitating amputation; therefore,
significant South American rattlesnake bite, tourniquets are not recommended first
quite different to rattlesnake bites in North aid for any snakebite. Electric shock treatment
America. for snakebite has been prbven to cause injury
The jararaca and related Bothrops spp. are and provides no benefit; it should never be
the most common cause of significant used as first aid for any envenoming.
snakebites in Brazil and often cause Scarification of the bite :;;ite, although
coagulopathy plus significant local bite site commonly used as first aid for. snakebite, has
injury, but not paralysis. Coral snakes mostly no benefit, causes significant harm and should
cause only minor local bite effects and never be used for snakebite.

downloaded from www.medicalbr.com


50 • ENVENOMATION

8.10. Answer: B. children, it can cause severe, even


The 'Million Death Study' in India provided the life-threatening envenoming, characterised by
best available data on the impact of snakebite local pain, swelling, bruising and sometimes .
in that country and estimated at least 45000 shock and a coagulopathy. Kidney injury can
Indians die from snakebite every year, far higher occur, although is rare.
than official Indian Government data, which Legless lizards are harmless, non-venomous,
uses only hospital data. as are grass snakes. Mambas (Dendroaspis
The precise number of snakebite deaths spp.) cause a quite different clinical picture of
each year worldwide is unknown, with envenoming.
estimates varying from a low of about 25 000
(almost certainly far too low), to> 100000, with 8.13. Answer: C.
some experts speculating that up to 200000 The severe local pain following a bite from a
deaths may occur, although this latter figure is large spider could be caused by a number of
not yet supported by definitive data. exotic species, but is a particular feature of
Snakebite causes nearly 100 times bites by the Brazilian wandering spider
more limb injuries requiring amputation, (Phoneutria nigriventer), otherwise known as the
each year, than land mines. Fortunately banana spider because it is sometimes
most of those injuries caused by accidentally imported in containers of fruit,
snakebites require only amputation of digits or notably bananas.
parts of a limb - less commonly whole-limb The Australian funnel web spiders (there are
amputation. a number of genera and species; the best
known is the Sydney funnel web spider, Atrax
8.11. Answer: C. 'robustus) can cause local severe pain, but the
The grass snake is a non-dangerous snake, major risk is systemic envenoming causing a
arguably the most common native snake in the catecholamine-storm-like clinical picture and
UK. The pale band or collar behind the head is this develops quickly within minutes to an hour
a useful diagnostic feature present in many or so after the bite. Your patient does not have
specimens, although not all. this clinical picture, so even if it was a funnel
There are legless lizards ('slow worms') in the web spider, he is out of the danger period!
UK, but they generally do not have this pattern Black widow spiders (Latrodectus spp.) a(e not
•I
of coloration. The European northern adder large and, while they can cause local pai[i, it
(Vipera berus) is the only venomous snake progresses to more regional, then gene\~lised
native to the UK. It is often grey in colour, with pain, plus features of neuroexcitation, so your
darker blotchy markings on the body. Mambas patient does not really fit this picture. E!rown
are tree-dwelling snakes and do not accord recluse spiders (Loxosceles spp.) genE3ri311Y
with the description given. Consider the cause little or no pain initially and it is dnly
possibility of an escaped exotic venomous many hours to days after people are bitten that
snake (depending on the setting) if the patient local necrosis and sometimes a significant
develops envenoming inconsistent with a systemic envenoming can develop; they are not
European adder bite. large spiders, so do not fit the picture
presented by your patient. The Mexican
8.12. Answer: A. orange-kneed tarantula (Brachypelma smith!)
The European northern adder does cause may cause locally painful bites, but more
snakebites in the UK, maybe as many as commonly causes skin or eye irritation from
70-100 cases per year and, particularly in shed abdominal hairs.

downloaded from www.medicalbr.com


M Byers

Environmental· medicine
Multiple Choice Questions
9.1. Research on individuals exposed to 9.3. A 67 year old patient is brought to the
radiation from the atomic bombs in Hiroshima emergency department having been found
and Nagasaki has shown an increased relative unwell in an unheated apartment during the
risk of developing malignancy (leukaemia, oral winter in Northern Europe. Severe reversible
cavity, oesophagus, stomach, colon, lung, hypothermia is best characterised by which of
breast, ovary, urinary bladder, thyroid, liver, the following?
non-melanoma skin and nervous system) A. A core temperature below 32°C
as a result of radiation exposure. Which of B. Bradycardia, a J wave on the
these statements best describes this electrocardiogram and loss of consciousness
observation? C. Chest and abdomen rigidity with a core
A. The Japanese have higher rates of cancer temperature below 13°C and serum I
than the world average potassium > 12 rnrnoi/L. ("
B. This is a deterministic effect of radiation D. Shivering, white peripheries and irritability
C. This is a stochastic (random) effect of E. Tachycardia, tachypnoea and slight delirium
!,'
radiation
D. This is an observational effect unrelated to 9.4. A family consult their family physician for.
the atomic bomb advice regarding a forthcoming holiday in thE:l
E. This is because of background radiation in tropics. Heat illness is a spectrum of disease
Japan affecting both the young and old. Which of
these statements is most correct?
9.2. Radiation can be divided into ionising and A. Complications of heat stroke include
non-ionising forms. Ionising radiation carries hypovolaemic shock, lactic acidosis,
enough energy to free electrons from atoms or rhabdomyolysis, hepatic failure and
molecules, thereby ionising them, and this can pulmonary oedema
damage tissues and cells. Which of these B. Exertional heat illness is more common in
forms of therapy is most likely to cause the elderly than in younger people
long-term radiation injury through high levels of C. Heat acclimatisation is characterised by
ionising radiation exposure to patients? decreased sweat volume, reduced sweat
A. Chest X-ray to diagnose a spontaneous sodium content and secondary
pneumothorax hyperaldosteronism to n;aintain body sodium
B. Radiofrequency ablation for cardiac balance
arrhythmias D. Heat stroke commonly occurs above 39°C
C. Serial whole-body computed tomography E. Heat syncope is another term for heat stroke
(CT) for cancer screening
D. Transurethral microwave therapy for prostatic 9.5. Acclimatisation is the proc(lss of the body
hypertrophy adjusting to the decreased availability of oxygen
E. Ultraviolet therapy for psoriasis at high altitudes. This becomes noticeable

downloaded from www.medicalbr.com


l
I
52 • ENVIRONMENTAL MEDICINE

above 2500m. Which of these changes occurs B. In about 10% of cases, no water enters the
in healthy individuals? lungs and death follows intense
A. A shift in the oxygen dissociation curve to laryngospasm ('dry' drowning)
the left after 2-3 days C. Long-term outcome depends on the severity
B. Deep prolonged sleep with vivid dreams of the cerebral hypoxic injury and is
C. Deep, slow breathing to maximise oxygen predicted by the duration of immersion and
uptake delay in resuscitation, but is independent of
D. Erythropoiesis and haemoconcentration the presence of cardiac arrest
mediated through the endocrine system D. Salt water is hypertonic and inhalation
E. Fluid retention to counteract the raised provokes alveolar oedema, producing a
haematocrit due to hypoxia distinct clinical picture from freshwater
drowning
E. Those rescued alive (near-drowning) are
9.6. A 36 year old mountaineer ascends to
often unconscious and not breathing.
3800m. He complains of feeling tired and
Hypoxaemia and metabolic alkalosis are
unwell. His companions notice that he is
common features during resuscitation
staggering and delirious. Which of the following
statements is true regarding illness at altitude?
9.8. A 48 year old woman is planning to do
A. Acetazolamide is the treatment of choice for some diving on her forthcoming holiday to the
high-altitude cerebral oedema (HACE) Caribbean. She is reading about the possible
B. Altitude sickness usually occurs between risks involved. Which of the following
1500 m and 2500 m, is characterised by statements is true?
vomiting and resolves spontaneously after a
A. Ambient pressure under water increases by
few days
101 kPa (1 atmosphere, 1 ata) for every 10m
C. High-altitude pulmonary oedema (HAPE) is a
of seawater depth, with the nitrogen in air
life-threatening condition that initially presents
causing narcosis below 30m of seawater
with symptoms of dry cough, exertional
and oxygen becoming toxic at inspired
dyspnoea and extreme fatigue
pressures above 40kPa (0.4ata) .·
D. Monge's disease (chronic mountain sickness)
B. As divers descend, the partial pressurfs of
is characterised by polycythemia and
the gases they are breathing decreas13 and
hypoxia that does not improve if the patient
the blood and tissue concentrations .bf
moves to lower altitudes to live
dissolved gases change accordinglyt
E. The cardinal signs of HACE are headache,
C. She can be confident that she will be able to
unilateral pupillary dilatation and dizziness
undertake a final dive on the morning of her
return flight home provided she h~s taken
1

9.7. A 5 year old boy is brought to the enough time on her final ascent '
emergency department following a drowning D. She should hold her breath on ascent to
incident. Which of the following statements is avoid arterial embolisation through a patent
true with regard to a drowned patient? foramen ovale
A. Fresh water is hypotonic and impairs E. The bends are caused by bubbles of carbon
surfactant function, causing alveolar collapse dioxide being released into the body tissues
and left-to-right shunting of unoxygenated whilst a diver ascends; this can be treated
blood with recompression therapy

Answers
I
9.1. Answer: C. interval of around ,2-5 years and solid tumours
Stochastic (chance) effects occur with after an interval of about 10-20 years.
increasing probability as the dose of radiation
increases. Carcinogenesis represents a 9.2. Answer: C.
stochastic effect, with not all exposed CT scans result in relatively high-radiation
individuals being affected. With acute exposure and whole-body CT screening has
exposures, leukaemias may arise after an not been demonstrated to meet generally

downloaded from www.medicalbr.com


ENVIRONMENTAL MEDICINE • 53

accepted criteria for screening. The risks The cardinal signs of HACE are ataxia and
associated are outweighed by the benefits of altered consciousness. It is rare, life-threatening
diagnostic CT and there is a small increase in and usually preceded by AMS. In addition to
lifetime risk of developing cancer. Options B, D features of AMS, the patient suffers confusion,
and E are non-ionising radiations and the disorientation, visual disturbance, lethargy and
radiation dose from one chest X-ray is ultimately loss of consciousness. Monge's
negligible. disease improves if the patient moves to lower
altitudes to live.
9.3. Answer: B.
Hypothermia occurs when the core 9.7. Answer: B.
temperature drops below 35°C. Severe Drowning remains a common cause of
hypothermia is characterised by a temperature accidental death throughout the world and is
below 28°C, bradycardia, bradypnoea, relatively common in young children. Fresh
arrhythmias and loss of consciousness. A rigid water causes alveolar collapse and right-to-left
chest and abdomen with a core temperature shunting of unoxygenated blood. Saltwater and
below 13°C and serum potassium > 12 mmoi/L freshwater drowning produce a similar clinical
is probably incompatible with life. picture. In near-drowning, metabolic acidosis is
Body temperature is controlled in the almost universal and cardiac
hypothalamus, which is directly sensitive to arrest is a poor prognostic indicator in recovery
changes in core temperature and indirectly from drowning. It is true that in about 10%
responds to temperature-sensitive neurons in of cases, no water enters the lungs and
the skin. The normal 'set-point' of core death follows intense laryngospasm ('dry'
temperature is tightly regulated within drowning).
37±0.5°C.
9.8. Answer: A.
9.4. Answer: A. The underwater environment is extremely
Exertional heat illness is more common in hostile. Other than drowning, most diving illness
athletes and sweat volumes increase with is related to changes in barometric pressure
acclimatisation. Heat stroke is rare below 40oC and its effect on gas behaviour. Partial
and heat syncope is a distinct condition and far pressures of gases increase with descent and
less serious than heat stroke. the bends are caused by the nitrogen bubbles
on ascending again. Arterial embolisation may
9.5. Answer: D. occur if the gas load in the venous system '!
Hyperventilation is caused by hypoxia sensed exceeds the lungs' abilities to excrete nitrogen,
through the carotid bodies and a diuresis or when bubbles pass through a patent ;
occurs secondary to haemoconcentration. After foramen ovale. A patent foramen ovale occ~rs
2-3 days the oxygen dissociation curve moves in 25-30% of asymptomatic individuals. A diver
to the right, making it easier for haemoglobin to must ascend slowly and breathe regularly
release oxygen to the tissues. Sleep and during ascent to avoid barotrauma.
nocturnal breathing patterns are frequently Recompression is the definitive therapy for
disturbed at altitude. decompression illness. Recompression reduces
the volume of gas within tissues (Boyle's law),
9.6. Answer: C. forces gas back into solution and is followed by
Above 2500m, high-altitude illnesses may slow decompression that allows the gas load to
occur in previously healthy people, and above be excreted.
3500m these become common. Acute Decompression illness can be provoked by
mountain sickness (AMS) symptoms develop a flying. Diving tables should be consulted to
few hours after ascent and include dizziness, p
leave a safe gap between final dive and a
fatigue and headache. subsequent plane journE:)Y·

downloaded from www.medicalbr.com


VR Tallentire,
MJ MacMahon, J Bain,
S Fadden

Acute medicine and


critical illness
Multiple Choice Questions
10.1. A 76 year old man develops an acute A. Low inspired oxygen concentration
kidney injury 2 days after an elective knee B. Shunt
replacement. His past medical history includes C. Ventilation-perfusion 0/IQ mismatch: alveolar
mild chronic obstructive pulmonary disease hypoventilation
and hypertension (controlled with an D. V/0 mismatch: perfusion defect
angiotensin-converting enzyme (ACE) inhibitor). E. V/0 mismatch: central hypoventilation
He is hypotensive (80/50 mmHg), oliguric and
has urea of 18 mmoi/L (50.4 mg/dl), creatinine 10.3. A 69 year old woman is admitted to the
of 165 J.Lmoi/L (1.87 mg/dl), potassium of intensive care unit (ICU) following a road t~ffic
5.1 mmoi/L and a normal bicarbonate level. He accident. She is known to have disseminatedIf
has received 30 mUkg of intravenous fluids pancreatic cancer with metastases in the1brain,
over the past 2 hours. liver and lung. She was intubated at the/scene
Which will be the most efficacious measure by the paramedics. She has sustained multiple
to improve his renal outcome? long bone fractures, a head injury and. is
A. Commence renal replacement therapy requiring a high inspired oxygen con~eljltration
B. Further fluid challenge to maintain her oxygen saturations. Wl:lich of
C. Intravenous dopamine infusion the following statements is MOST accurate?
D. Intravenous furosemide infusion A. All sedatives and analgesics must be
E. Intravenous noradrenaline (norepinephrine) stopped to allow an accurate assessment of
infusion conscious level
B. An advance directive stating that the patient
10.2. A 68 year old man presents with would not want to survive with severe
worsening exertional dyspnoea. He is known to disability should be taken into consideration
have non-alcoholic fatty liver disease (NAFLD), C. If a withdrawal decision has been made, it is
with cirrhosis on a recent ultrasound scan, mild unethical to extubate the patient as she may
jaundice but with no ascites or encephalopathy. die from airway obstruction
On examination he has normal breath sounds D. It is unjustifiable to withdraw active treatment
throughout both lung fields with no added while there is still a chance of recovery
sounds and his jugular venous pressure is not E. The family will need to be contacted to make
elevated. He has an oxygen saturation of 87% a decision about ongoing treatment
on air whilst standing (although this improves to
94% in the supine position). Multiple spider 10.4. A 22 year olcl.man is admitted to the ICU
naevi are noted over his thorax and abdomen. with an acute exacerbation of asthma. On
Which of the following processes are most high-flow oxygen he has a Pa02 of 3.1 kPa
likely to account for his hypoxaemia? (233 mmHg) and a PaC02 of 16 kPa

downloaded from www.medicalbr.com


ACUTE MEDICINE AND CRITICAL ILLNESS • 55

(120 mmHg). There is respiratory effort but no C. Positive end-expiratory pressure (PEEP)
audible air entry on auscultation. He is should be weaned to zero before extubation
unconscious and preparations are being is considered
made to intubate and ventilate him. Other than D. The fraction of inspired oxygen (Fi0 2) should
severe asthma, he has no other past medical be weaned to achieve a Pa02 of 15 kPa
history. (113 mmHg) and above
Which of the following statements is E. Tracheostomy should be immediately
CORRECT? performed if the patient fails a spontaneous
A. Large cannulae should be inserted bilaterally breathing trial
into the second intercostal space at the
mid-clavicular line 10.7. Which of the following findings suggests
B. Once intubated, a high respiratory rate and that ongoing intensive care treatment still has
large tidal volumes should be used initially to a realistic chance of a good neurological

--
clear the carbon dioxide outcome following cardiac arrest?
C. Once intubated, a respiratory rate of A. A computed tomography (CT) brain showing
12 breaths/min, a tidal volume of 6 mUkg preserved grey-white differentiation
and a prolonged expiratory time should be B. Absent corneal reflexes at 72 hours after
1
used initially return of spontaneous circulation (ROSC)
D. The blood gas is likely to be erroneous as C. Absent motor response to painful stimulus
his Pa02 is too high 72 hours after ROSC
E. The C0 2 will return to normal if the oxygen D. Bilaterally absent N20 spike on
mask is removed somatosensory evoked potentials
E. Myoclonic jerking within the first 24 hours
10.5. Which of the following statements best after ROSC
describes good practice when using sedation
and analgesia in intensive care? 10.8. A 37 year old previously healthy man is
A. A Richmond Agitation and Sedation Score brought into the emergency department by
(RASS) score of 0 suggests sedation is ambulance having stopped the car he was /
optimal driving. When the paramedics arrived he was .'
B. Etomidate is the most cardia-stable sedative conscious but appeared confused. This •/
and should be used as an infusion in progressed over the following 30 minutes to !,'
patients who are very unstable aphasia and the development of bilateral upp~r
C. Muscle relaxants can be used to reduce the and lower limb weakness with bilateral cranip.l
need for sedation nerve palsies. CT head scan on arrival is
D. Patients should be deeply sedated to reduce reported as normal.
the risk of delirium Which investigation is most likely to reveal
E. Sedation should not be stoppecj abruptly as the underlying pathology?
there is an unacceptable risk of A. CT angiogram of the circle of Willis
self-extubation B. CT scan of the cervical spine
C. Electrocardiogram (EGG)
10.6. A 45 year old man is being ventilated for D. Erythrocyte sedimentation rate (ESR)
acute respiratory distress syndrome (ARDS) E. Lumbar puncture
following a lobar pneumonia. The bedside
nurse has a number of questions regarding the 10.9. A 63 year old man is weaning from
strategy for ventilating this man as his lung mechanical ventilation. He has been ventilated
function improves. in the ICU for 3 months with Guillain-Barre
Which of the following points is correct? syndrome and profound ~eakness. A
A. During weaning, patients should"not be percutaneous tracheost9r'ny was placed 2
allowed to go for long periods with no months ago. He has suddenly developed
mechanical ventilator support if they are respiratory distress after a bout of coughing.
showing signs of respiratory distress He is now extremely tachypnoeic, sweating
B. If the patient is febrile, minute volumes will and in respiratory distress. He. has no
be lower and it is a good opportunity to previous history of any laryngeal or tracheal
wean the ventilatory support problems.

downloaded from www.medicalbr.com


l
I
56 • ACUTE MEDICINE AND CRITICAL ILLNESS

All of the actions below would be Which management plan is most likely to be
appropriate, except for one. Which of the successful?
following actions would NOT be considered A. Commencement of an adrenaline infusion.
best practice .in this situation? B. Immediate percutaneous coronary
A. Applying a facial high-flow oxygen mask intervention (PCI)
B. Applying a self-inflating bag to the C. Insertion of an intra-aortic balloon pump
tracheostomy and giving the patient several D. Intubation and ventilation with high levels of
large breaths PEEP
C. Calling for help E. Venous-arterial extracorporeal membrane
D. Passing a suction catheter through the oxygenation (ECMO)
tracheostomy to check patency
E. Removing the inner tube of the tracheostomy 10.12. Which of the following statements is
TRUE regarding gas carriage in the blood?
10.10. Which one of the following ventilated A. For a given PaC0 2, more carbon dioxide can
patients has ARDS according to the Berlin be carried by blood with haemoglobin that is
definition? 80% saturated with oxygen in comparison to
A. A man with a severe influenza pneumonia. 1 00% saturated
He has bilateral infiltrates on chest X-ray B. In capillaries with a high carbon dioxide
and a Pa02 of 10 kPa (75 mmHg) on an Fi0 2 content, e.g. exercising muscle, oxygen is
of 0.4 bound more tightly to haemoglobin, i.e. the
B. A man with left lower lobe pneumonia, a haemoglobin-oxygen dissociation curve is
normal echocardiogram and a Pa02 of shifted to the left
10 kPa (75 mmHg) on an Fi0 2 of 0.6 C. The majority of carbon dioxide is transported
C. A man with long-standing, progressive in the blood bound to haemoglobin
idiopathic pulmonary fibrosis. He has bilateral D. There is a greater oxygen than carbon
chest X-ray infiltrates, a normal dioxide content in arterial blood
echocardiogram and a Pa0 2 of 10 kPa E. When core temperature drops, for a given
(75 mmHg) on an Fi0 2 of 0.5 blood content of carbon dioxide, the P~C0 2
D. A woman with acute pancreatitis. She has will increase l
bilateral chest infiltrates, pleural effusions, a /
normal echocardiogram and a Pa02 of 10.13. A 45 year old man is admitted to 1the ICU
14 kPa (1 05 mmHg) on an Fi0 2 of 0.3 following coronary artery bypass surgery. He is
E. A woman with endocarditis and severe tachycardic, hypotensive and has a high
mitral regurgitation from a leaflet perforation. lactate. Clinical examination is unremarkable
She has bilateral chest X-ray infiltrates and and there is no bleeding apparent. Ah
,ECG
a Pa0 2 of 10 kPa (75 mmHg) on an Fi0 2 shows a sinus tachycardia with no other
of 0.6 abnormalities. His haemodynamic data (from
his pulmonary artery catheter) are as follows
(reference ranges are also given):
10.11. A 60 year old man becomes acutely
unwell on the medical ward. He was admitted
4 days prior with non-specific symptoms
(malaise, fever, coryza). On admission, his ECG Patient data Reference range
Cardiac output 10.2 Umin 4-8 Umin
showed sinus rhythm with no acute ST
Cardiac index 5.42 Umin/m' 2.5-4 Uminlm'
changes, but his serum troponin level taken
Pulmonary artery 15/9 mmHg 15-30/5-15 mmHg
24 hours post-admission was markedly raised.
pressures
A viral throat swab was positive for adenovirus. Pulmonary artery 7 mmHg 2-10 mmHg
He is tachycardic (180 beats/min), capillary wedge
hypotensive (65/30 mmHg), pale and clammy. pressure
On examination his chest is clear but he looks Central venous 6 mmHg 6-12 mmHg in
pressure ventilated patients
very unwell and a blood gas shows a lactate of
1 0 mmoi/L (90 mg/dl) with a haemoglobin of
120 g/L. His ECG now shows left bundle Which of the following statements is true?
branch block and a bedside echocardiogram A. An infusion of an inotrope such as.
confirms global left ventricular dysfunction. dobutamine is indicated

downloaded from www.medicalbr.com


ACUTE MEDICINE AND CRITICAL ILLNESS • 57

B. An intra-aortic balloon pump should be is considered appropriate it should be


inserted immediately performed awake
C. It is likely that one of the coronary grafts has E. Peripheral noradrenaline (norepinephrine) can
become occluded be used in this situation while intubation and
D. The data suggest he has a cardiac central venous access is established
tamponade
E. The data suggest that he is vasodilated 10.16. A 46 year old, previously healthy man is
in intensive care following an influenza
10.14. Which one of the following statements pneumonia with ARDS. He was on venous-
regarding daily management in intensive care is venous ECMO for 10 days and is now receiving
MOST accurate? ventilatory support via a tracheostomy. It is
A. Blood glucose 'levels should be maintained in noted today that he has bilateral weakness of
the range of 4-6 mmoi/L (72-1 08 mg/dL) his arms and legs. This has become apparent
B. Proton pump inhibitors and histamine-2 since his sedation has been lightened. On
antagonists reduce the incidence of
gastrointestinal bleeding
examination he has marked proximal muscle
weakness. Tone and sensation appear to be
normal, deep tendon reflexes are present but
m.
C. The haemoglobin should be kept
above 10 g/L in general intensive care reduced, and plantars are downgoing. Cranial I
patients nerves are intact and the patient is alert,
D. Unfractionated heparin (given twice daily by orientated and obeys commands.
subcutaneous injection) is more effective Which of the following diagnoses is most
than low-molecular-weight heparin at likely?
preventing pulmonary embolus in general A. A lesion of the brainstem
intensive care patients B. Critical illness myopathy
E. When pressure is being monitored, central C. Guillain-Barre syndrome
venous and arterial lines should be attached D. Rhabdomyolysis
to a pressurised bag of 5% glucose E. Spinal cord lesion

10.15. A 75 year old man who had been 10.17. A 65 year old woman is brought to /
/,'
previously fit and well was admitted to hospital intensive care after an acute deterioration on ,1
with shortness of breath. His chest X-ray the stroke ward. She had been recovering after
showed a left lower lobe pneumonia and he a left partial anterior circulation infarct when she
was commenced on intravenous antibiotics. His was noted to be acutely agitated and then
admission ECG showed a sinus tachycardia became drowsy. This progressed quickly t¢' 1
with left ventricular hypertrophy (by voltage unconsciousness and she was intubated t6;
criteria). Approximately 12 hours after facilitate a CT scan.
admission he acutely deteriorated with The CT head scan confirmed a large
tachypnoea, tachycardia, hypotension haemorrhagic transformation of the infarcted
(70/40 mmHg) and reduced oxygen area with 10 mm of midline shift, effacement of
saturations. On examination he is agitated, the ventricles and downward herniation of the
managing only incomprehensible sounds, with cerebellar tonsils. Twelve hours after arrival in
increased work of breathing and bilateral intensive care she is unresponsive (Glasgow
coarse crepitations. His ECG confirms atrial Coma Scale (GCS) score 3 despite no sedation
fibrillation with a ventricular rate of 120 beats/ for 4 hours), normothermic, and her pupils are
min. Which one of the following statements is fixed and dilated.
most accurate? Which of the following statements is true?
A. Antibiotics should be switched to a A. As she is not waking u8, a neurosurgeon
carbopenem should be asked to drain the haematoma
B. DC cardioversion is likely to resolve the B. Brain-death testing should be undertaken
clinical situation C. She has a good prognosis, provided she can
C. Intubation and ventilation is contraindicated survive the acute bleEl_d
due to advanced age D. She requires an electroencephalogram (EEG)
D. It is likely that this man is too unstable for and magnetic resonance imaging brain scan
anaesthetic agents to be used: if intubation to confirm brain death (under UK law)

downloaded from www.medicalbr.com


I .,
'
58 • ACUTE MEDICINE AND CRITICAL ILLNESS
T
r''!
E. She should be cooled to 28°C to reduce
intracranial pressure
10.21. A 25 year old man presents to the
emergency department with a 4-day history of
II
I chest pain and mild shortness of breath. He r
I 10.18. A 45 year old man arrives in the medical feels that the pain is worse on lying flat, ·
I !
receiving unit complaining of headache, blurred coughing and deep breathing; it is relieved by
.I vision, nausea and vomiting. His blood pressure
(BP) is 256/138 mmHg. A CT head scan is
sitting forwards and has improved with
ibuprofen. He has no significant past medical
normal. Which of the following is true? history but was recently unwell with a
A. Chest pain in this context is likely to be non-specific viral infection. An EGG shows
anxiety related widespread saddle ST elevation.
B. In this context, nausea and vomiting is What is the most likely cause of the chest
unlikely to represent raised intracranial pain?
pressure A. Aortic dissection
C. Shortness of breath in this context may be B. Musculoskeletal chest pain
the result of pulmonary oedema C. Pericarditis
D. The blood pressure should be reduced D. Pulmonary embolism
rapidly, aiming for a 50% reduction in the E. Type 1 myocardial infarction (MI}
mean arterial pressure in the first hour
E. The use of cannabis is a recognised 10.22. A 39 year old man is brought in by
precipitant ambulance to the emergency department. He
has a 3-day history of sore throat and feeling
10.19. An 87 year old woman presents to generally unwell. He is struggling to speak and
hospital with her daughter who found her in the swallow saliva and has marked stridor. He is
morning confused, incoherent and wearing distressed and does not want to lie down.
yesterday's clothes. Initial examination reveals Observations show: heart rate of 11 5 beats/
normal observations. Which of the following min, respiratory rate of 25breaths/min, BP
history and examination findings most likely 120/74 mmHg, Sp02 94% on 15 L oxygen,
suggest an alternative diagnosis other than temperature 39.4°C, blood glucose of .
delirium? 4.7 rnmoi/L (84.6 mg/dL), GCS score 15/The
A. A history of progressive deterioration over patient is normally fit and well. ;/
several months Which potential diagnosis requires mdst
B. A urine dipstick positive for leucocytes and urgent recognition and management? ,;'
nitrites A. Croup
C. Fluctuant course, with more florid confusion B. Epiglottitis /I
at night C. Lower respiratory tract infection
D. Periods of severe agitation requiring D. Nasal polyps
pharmacological management E. Tonsillitis
E. The inability to direct attention and sustain
conversation 10.23. Which one of these features is
associated with delirium but not dementia?
10.20. A woman is brought in the emergency A. Anxiety
department by the ambulance after being found B. Cognitive impairment
unconscious at home. Her pupils are equal and C. Disorientation
reactive. She does not open her eyes to a painful D. Reversibility
stimulus. She is making groaning noises but no E. Visual hallucinations
comprehensible words. She has no motor
response to supra-orbital pressure but she does 10.24. An 88 year old woman is referred to the
withdraw her arms briskly to mil-bed pressure. acute medical admissions ward by her family
How would you score her GCS? physician, who was asked to review her at
A. 5/14 home by a concerned carer. The patient has a
B. 6/15 medical history ot frequent falls and mild
c. 7/15 cognitive impairment, but has become
D. 8/15 increasingly confused since suffering a fall in
E. 11/15 her garden 5 weeks ago. At that time, the

downloaded from www.medicalbr.com


ACUTE MEDICINE AND CRITICAL ILLNESS • 59

patient sustained a few grazes and bruises to 64% when he was being moved into the
the right side of her body, including her head. ambulance, and symptoms improved slightly
The carer reports that the patient has also after administration of oxygen and morphine.
become increasingly unsteady on her feet and Clinical examination reveals respiratory rate of
has been complaining of having a headache 28 breaths/min, Sp0 2 90% on 8 L oxygen and
and blurred vision. She has been more sleepy reduced chest expansion on the right side,
than usual, but only intermittently. The patient's temperature 37.2°C, troponin and an ECG
cardiorespiratory observations are are unremarkable. A venous blood gas shows
unremarkable. From the options below, W is 51 nmoi/L (pH 7.29), PC0 2 7.0 kPa
what is the most likely cause of her current (52.5 mmHg), P02 3.1 kPa (23.3 mmHg),
symptoms? HC03- 25.5 mmoi/L, base excess -2.0 rnmoi/L
A. Chronic subdural haematoma and pulmonary vascular markings are absent
B. Dementia on the right on the chest X-ray. Which of the
C. Encephalopathy following is the most likely diagnosis?
D. Intracerebral haemorrhage A. Acute myocardial infarction
E. Subarachnoid haemorrhage B. Musculoskeletal chest pain
C. Pneumonia
10.25. A 24 year old woman with a body mass D. Primary spontaneous pneumothorax
index (BMI) of 38 kg/m 2 presents to the E. Secondary spontaneous pneumothorax
emergency department with a swollen and
tender right calf (3.5 em larger than the left). 10.28. A 20 year old man is brought to the
She has recently returned from a European emergency department after self-extricating
holiday resort and has sunburn, but is from a burning block of flats. He was trapped
otherwise well. She suffers from well-controlled inside the building for 15 minutes, in a flat on
asthma, for which uses inhalers, and is also the floor above the source of the fire. He has
taking the combined oral contraceptive pill. sustained no obvious injuries, but has a
Which of the following is the most likely persistent cough and feels 'dizzy'. Paramedics
diagnosis? have applied oxygen via a face mask, and the
A. Calf muscle tear patient's Sp0 2 is 99% on 2 L oxygen. An
B. Cellulitis secondary to an insect bite arterial blood gas sample is taken from the
C. Deep vein thrombosis patient. What additional test, not routinely
D. Dependent oedema requested on an arterial blood gas sample,
E. First -degree burn (sunburn) should be done?
A. Carboxyhaemoglobin
i''
10.26. Which of the following causes a leftward B. Fetal haemoglobin
shift of the haemoglobin-oxygen dissociation C. Haemoglobin A
curve? D. Nitrous oxide
A. Acidosis E. Superoxide
B. Decreased temperature
C. Increased 2,3-diphosphoglycerate (2,3-DPG) 10.29. Which of the following is a 'red flag'
D. Increased PC0 2 symptom in a person presenting with a
E. Increased temperature headache?
A. Associated with taking codeine tablets for a
10.27. Paramedics arrive in the emergency week
department resuscitation room with a 70 year B. Gradual onset (over an hour or more)
old man who has multiple comorbidities, C. Improved by lying down
including pulmonary fibrosis and chronic D. Right arm weakness
obstructive pulmonary disease (COPD; on E. Visual aura
home oxygen, with oxygen saturations normally
of 88-92% on air). He suddenly developed 10.30. A 60 year old woman with a previous
right-sided sharp chest pain and dypsnoea. head injury is admitted with collapse. Taken
The paramedics report that the patient initially from the history alone, which one feature may
had a respiratory rate of 34 breaths/min and point to a diagnosis of syncope rather than
Sp0 2 of 88% on 2 Umin oxygen, which fell to seizure?

downloaded from www.medicalbr.com


60 • ACUTE MEDICINE AND CRITICAL ILLNESS T
f
A. Amnesia A. Cardiac filling (preload) and contractility
B. Cyanosis B. Cardiac index
C. Olfactory aura C. Haemoglobin and oxygen saturation
D. Rapid recovery back to baseline state D. Heart rate and diastolic blood pressure
E. Tongue-biting E. Jugular venous pressure alone

10.31. Which one of the following does not 10.35. A 52 year old office worker is brought in
score points on routinely used early warning by ambulance to the emergency department.
systems in the context of medical observation She was found collapsed in the toilets at work,
monitoring? having earlier complained of a 4-day history of
A. Capillary blood glucose mild headache. She has a past medical history
B. Glasgow Coma Scale score of well-controlled hypertension, diet-controlled
C. Heart rate type 2 diabetes mellitus and she smokes 40
D. Sp0 2 cigarettes a day. Her partner reports that she
E. Temperature drinks three glasses of wine a week and has
never previously been admitted to hospital.
10.32. A 38 year old man is admitted for The paramedics have been supporting her
observation after falling over the handlebars of ventilation using an oropharyngeal airway and
his pushbike. He was wearing a helmet and bag-valve-mask. Examination reveals heart rate
had no loss of consciousness, although he of 50 beats/min, respiratory rate of 6breaths/
feels nauseated and 'faint' when standing up. min (without support), BP 190/105 mmHg,
You are asked to see him, as his heart rate has Sp02 90% on 15 L oxygen, temperature
increased from 95 to 150 beats/min over the 37.1°C, blood glucose 4.8 mmol/L (86.4 mg/
first hour of his admission. He is complaining of dl), GCS score 6 (E1, V2, M3) with no
upper abdominal pain and mild shortness of lateralising signs. There is no visible rash or
breath. Non-invasive BP is 100/60 mmHg and external evidence of injury.
he feels cool peripherally. His chest is clear, Which one of the following conditions is the
with no clinical evidence of pneumothorax or rib most likely cause of this patient's presentation?
fracture. His respiratory rate is 25 breaths/min, A. Alcohol withdrawal /
Sp0 2 99% on 4 Umin oxygen. His GCS score B. Bacterial meningitis ,/
is 15 with normal limb movements. You have C. First presentation of epilepsy 1
/

taken an arterial blood gas, full blood count, D. Hypoglycaemia


urea and electrolytes and a coagulation screen E. Subarachnoid haemorrhage
and are awaiting the results.
What is the next most useful investigation to 10.36. An 84 year old man is brought' to
elucidate the cause of this man's deterioration? hospital after a fall. He was found by his
A. CT abdomen daughter and had been lying on the floor for
B. CT head the previous 24 hours. He has a variety of
C. ECG soft tissue injuries but no serious head or
D. Renal ultrasound orthopaedic injuries. On catheterisation, his
E. V/6. scan urine is dark brown. Which one of these tests is
most suggestive of rhabdomyolysis?
10.33. In the general medical setting, what is A. 1+ blood on urine dipstick
the earliest and most sensitive sign of clinical B. Creatine kinase
deterioration? C. Haemoglobin
A. Blood pressure D. Renal ultrasound
B. Core temperature E. Urea
C. Heart rate
D. Respiratory rate 10.37. A 20 year old man is brought in by
E. Urine output ambulance to the emergency department,
complaining of moElerate central chest pain
10.34. In the context of cardiac physiology, and feeling lightheaded. These symptoms came
which of the following is stroke volume most on acutely whilst he was playing football.
dependent on? Observations show: heart rate of 180 beats/

downloaded from www.medicalbr.com


ACUTE MEDICINE AND CRITICAL ILLNESS • 61

min, respiratory rate of 18 breaths/min, BP 10.41. Hyperlactaemia is very rarely seen in


75/4 mmHg, Sp02 94% on 15 L oxygen, which of the following situations?
temperature 36.9°C, blood glucose 4.6 mmoi/L A. Cardiogenic shock
(82.8 mg/dl), GCS score 15. ECG reveals a B. Chronic renal failure
regular narrow complex tachycardia. C. Major haemorrhage
Which one of these treatments would be D. Metformin overdose
most appropriate as initial management? E. Sepsis
A. Adenosine 6 mg intravenous (IV) bolus
B. Amiodarone 300 mg IV bolus 10.42. A 49 year old woman is admitted to the
C. Digoxin 500 11g orally acute medical ward with a severe leg cellulitis.
D. Electrolyte replacement She has type 2 diabetes with a BMI of 41 kg/
E. Synchronised DC cardioversion m 2. Observations show: heart rate of 119beats/
min, respiratory rate of 20breaths/min, BP
10.38. The Third International Consensus 70/35 mmHg, Sp0 2 97% on air, temperature
Definitions for Sepsis and Septic Shock
(Sepsis-3) gives a clear definition of sepsis as
patients with suspected infection who have two
38SC, GCS score 15.
Which one of the following treatments would
you initiate first?
a.I
or more of three particular features. Which of A. Crystalloid IV bolus of up to 30 mUkg
these features form part of that definition? B. Hydrocortisone 100 mg IV
A. GCS score < 15 C. Peripheral IV adrenaline (epinephrine)
B. Increase of one point on the Sequential 80-240 11g/hr
Organ Failure Assessment (SOFA) score D. Peripheral IV noradrenaline (norepinephrine)
C. Respiratory rate >20breaths/min 480 11g/hr
D. Serum lactate > 1 .5 mmoi/L (> 13.5 mg/dl) E. Vasopressin (antidiuretic hormone, ADH)
E. Systolic blood pressure of < 110 mmHg 0.041U/min

10.43. For which of these conditions is


10.39. According to the Berlin definition of non-invasive ventilation (bi-level positive airway
ARDS, which set of variables would denote
pressure; BiPAP) the first-line respiratory
severe ARDS? treatment?
A. Pa0 2 of 9 kPa (67 .5 mmHg) and Fi0 2 A. Epiglottitis
0.8 = 11.25 B. H1 N1 influenza
B. Pa0 2 of 10 kPa (75.0 mmHg) and Fi02
C. Mild congestive cardiac failure
0.5 = 20 D. Severe pneumonia I ,
C. Pa0 2 of 12 kPa (90.0 mmHg) and Fi02 E. Type II respiratory failure with respiratory'
0.4 = 30
acidosis
D. Pa0 2 of 13 kPa (97 .5 mmHg) and Fi02
0.21 = 61 10.44. The following are all terms that are used
E. Pa0 2 of 16 kPa (120.0 mmHg) and Fi0 2
in relation to complications of invasive
0.5 = 32 ventilation for patients in an intensive care
setting, except for one. Which is the one that is
10.40. A 75 year old sustains a cardiac arrest not a real term used in this context?
secondary to myocardial ischaemia. Return of A. Atelectotrauma
spontaneous circulation is achieved after 15 B. Barotrauma
minutes of cardiopulmonary resuscitation (CPR) C. Biotrauma
and 2x DC shocks. Which one of these is a D. Pulmotrauma
physiological target after return of circulation E. Volutrauma I
post -cardiac arrest? /
A. A MAP of no greater than 45 mmHg 10.45. A patient with a known peanut allergy
B. Core temperature of 36°C and avoidance of presents to the emergency department with
pyrexia wheeze, tongue swelling. and difficulty talking.
C. Glucose 11-16 mmoi/L (198- 288 mg/dl) He does not routinely carry an. EpiPen. On
D. PaC02 <3.5 kPa (26.3 mmHg) examination he has an Sp0 2 of 98% on 15 U
E. Urine output of 0.1 mUkg/hr min oxygen via non-rebreathe mask, respiratory

downloaded from www.medicalbr.com


1
62 • ACUTE MEDICINE AND CRITICAL ILLNESS

rate of 25 breaths/min, heart rate of 120 beats/ clammy, but alert and has no chest pain. ECG
min, BP 130/80 mmHg. He finds talking demonstrates a sinus tachycardia with
difficult. He is otherwise fit and well. He does low-voltage QRS complexes. Which of the .
not yet have IV access. following is the most likely underlying
Which of the following is your first treatment diagnosis?
priority? A. Cardiac tamponade
A. IV access and crystalloid bolus B. Cardiogenic pulmonary oedema
B. I 0 11g adrenaline (epinephrine) IV C. Extension of the original infarction
C. 50 11g adrenaline intramuscularly (IM) D. Neurogenic pulmonary oedema
D. 0.5 ml I :I 000 adrenaline IM E. Pulmonary embolism
E. 5 ml I :10000 adrenaline sublingually
10.48. A 67 year old woman is admitted to the
10.46. A 75 year old man presents to the ICU after cardiac arrest. She received
hospital with a 4-day history of diarrhoea and immediate bystander CPR, and was found to
vomiting. He has a history of moderate left be in ventricular fibrillation when the ambulance
ventricular failure, prostate cancer and chronic crew arrived. She received 3x DC shocks
kidney disease stage 4. His regular medication before return of spontaneous circulation, and
includes aspirin 75 mg once daily, bisoprolol
5 mg once daily and ramipril 5 mg once daily.
had a total 'downtime' of 32 minutes. She was
intubated and ventilated on arrival in the I
On admission his observations are as follows:
heart rate of 60beats/min, BP 90/45 mmHg,
emergency department. Her best GCS prior to
that was El, VI, M2. Which of the following
I
respiratory rate of 16 breaths/min, Sp02 94% would suggest the potential for a good
on air. He is lethargic and slow to respond to neurological outcome?
questions. He is oliguric after catheterisation. A. A neuron-specific enolase >33 11g/L
ECG demonstrates peaked T waves. Blood B. Burst suppression on EEG
results include haemoglobin I 01 g/L, white cell C. CT head with poor grey-white matter
count 16 x 109/L, platelets 190 x 109/L, urea differentiation
20.2 mmoi/L (121 mg/dl), creatinine
367 11moi/L (4.15 mg/dl), sodium 134 mmoi/L,
potassium 8.3 mmoi/L. He receives two
D. Extensor motor response
E. Immediate bystander CPR I
/
1/
boluses of calcium gluconate and two infusions
of insulin/dextrose to manage his potassium. 10.49. Using checklists for interventionf in the
After this, a venous blood gas demonstrates W ICU is a key component of good patient care.
Which of the following forms part of the 'FAST
84 nmoi/L (pH 7.08) and potassium 1
HUG' checklist? ·
8.2 mmoi/L.
What would be your next step in managing A. Foot care
this man? B. Gowning and gloving
A. 40 mg IV furosemide bolus C. Spinal problems
B. Critical care referral for monitoring and D. Teeth
consideration of renal replacement therapy E. Ulcer prophylaxis
C. Further bolus calcium gluconate
D. Further fluid resuscitation 10.50. A 56 year old man sustains a significant
E. Further insulin/dextrose infusion lower limb injury after becoming trapped
between a wall and a car. He is admitted to an
10.47. A 63 year old man has been admitted to orthopaedic ward for observation and operative
the coronary care unit after percutaneous planning. You are asked to see him 12 hours
coronary intervention for ST elevation later with a swollen, painful calf and suspected
myocardial infarction (MI). Fdur hours after compartment syndrome. Which of the following
admission he develops acute respiratory is true regarding this condition?
distress. Observations are as follows: heart rate A. A serum D-dimer is both sensitive and
of 120 beats/min, BP I 00/75 mmHg, specific for compartment syndrome
respiratory rate of 28 breaths/min, Sp02 96% B. Absent peripheral pulses are an early sign
on 15 Umin oxygen. His jugular venous suggestive of developing compartment
pressure (JVP) is elevated. He is pale and syndrome

downloaded from www.medicalbr.com


ACUTE MEDICINE AND CRITICAL ILLNESS • 63

c. His leg should be reviewed by a consultant 10.54. A 68 year old woman, who has
surgeon on the morning ward round the previously been fit and well, required intubation
next day and ventilation in the ICU with severe
D. Pain is worse with passive stretching pneumonia. After 10 days, tracheostomy to aid
E. Sensation in the leg is likely to be normal ventilatory weaning proves difficult, secondary
to respiratory muscle weakness, and she
10.51. A 62 year old man is sedated and requires a further 19 days of ventilation before
ventilated in the ICU after a severe being weaned off her tracheostomy. On
subarachnoid haemorrhage. He has an neurological examination she has global
intracranial pressure (ICP) monitor in situ, which proximal muscle. weakness with no lateral ising
has been reading 15 mmHg consistently. He is signs. Sensory examination is normal. Reflexes
being sedated and analgesed with propofol and are generally decreased. Nerve conduction
alfentanil infusions. On a sedation break his ICP studies demonstrate reduced amplitude of

--
increases to 45 mmHg and his pupils increase transmitted voltage action potential with
in size bilaterally. His mean arterial pressure is preserved velocity. Muscle biopsy is normal
90 mmHg. Which would be your first action to and creatine kinase is unremarkable. What
manage his ICP? condition is most likely responsible for this
A. Administer mannitol bolus woman's difficulty in weaning from ventilation? II
B. Administer neuromuscular blockade A. Brainstem stroke
I
C. Increase propofol and alfentanil infusion rates B. Critical illness myopathy
D. Refer to neurosurgery for decompressive C. Critical illness polyneuropathy
craniectomy D. Guillain-Barre syndrome
E. Remove the intracranial pressure monitor E. Multiple sclerosis

10.52. A 45 year old man is admitted to the ICU 10.55. A 24 year old man is admitted to the ICU
after banding of oesophageal varices and after sustaining a severe head injury after a fall
significant upper gastrointestinal (GI) from a height at work. CT head on admission
haemorrhage. He has alcoholic liver disease demonstrates massive intracranial haemorrhage .
and continues to drink 1 L of vodka per day. with midline shift, andthe clinical opinion is thatj
He is haemodynamically stable with good gas of brain death. His family say that he previously1;
exchange and is extubated 12 hours expressed a wish to donate his organs if this /'
post-procedure. He is moved to a medical situation ever arose. Which of the following /'
ward for ongoing management. Seventy-two would prevent testing for brain death?
hours later he becomes confused and agitated, A. Administration of 10 mg morphine IV 2 hours
with evidence of tremor and paranoid ideation. previously ,' '
What is the best treatment for his current B. Administration of a bolus of atracurium 72
condition? hours previously
A. Benzodiazepines C. Core temperature of 36"C
B. IV haloperidol 2.5 mg D. Normal thyroid function tests
C. Oramorph E. Serum sodium of 133 mmoi!L
D. Quetiapine
E. Thiamine 10.56. Which of these is an early complication
of percutaneous tracheostomy carried out in
10.53. Which of these statements is true the ICU?
regarding the use of intra-aortic balloon A. Haemorrhage
pump (IABP)? B. Laryngeal stenosis
A. Carbon dioxide is used to inflate the balloon C. Tracheal stenosis
B. It is associated with improved sui'Vival in D. Tracheomalacia
cardiogenic shock E. Wound site infection
C. It is commonly inserted via the brachial artery
D. It is designed to improve diastolic pressure 10.57. A 67 year old man.presents to the
proximal to the balloon, emergency department with a 3-day history of
E. There is no risk of mesenteric ischaemia haematemesis and melaena. He has a past
when inserted correctly medical history of alcoholic liver disease and

downloaded from www.medicalbr.com


64 • ACUTE MEDICINE AND CRITICAL ILLNESS

mild asthma (well controlled). His observations count 5. 7 x 109/L, platelets 41 x 109/L, sodium
are: heart rate of 100beats/min, respiratory rate 132 mmoi/L, potassium 5.6 mmoi/L, urea
of 16 breaths/min, BP 85/40 mmHg 16 mmoi/L (96 mg/dl), creatinine 75 Jlmoi!L
(70/35 mmHg on standing), Sa0 2 95% on 6 L (0.85 mg/dl), lactate 4.5 mmoi/L (40.5 mg/dl).
oxygen, temperature 36.5°C, blood glucose Which of the following would be your
4.2 mmoi/L (75.7 mg/dl), GCS score 15. On immediate next step in managing this man?
examination, he is pale, cool peripherally, A. Arrange urgent upper Gl endoscopy
talking in full sentences, and his chest is clear. B. Critical care referral for monitoring ·
His abdomen is soft and non-tender but fresh C. Insertion of a Sengstaken-Biakemore tube
melaena is found on rectal examination. He is D. Large-bore IV access and red cell transfusion
not actively vomiting currently. Initial laboratory E. Terlipressin 2 mg IV
results show haemoglobin 42 g/L, white cell

Answers
10.1. Answer: E. framework of the country and the ethical values
This man has developed multi-organ of the patient, usually expressed through the
dysfunction following arthroplasty. The family. This woman has severe underlying
pathology is not well understood but may disease and a poor prognosis from the trauma,
involve an inflammatory response to the 'so although there is a theoretical chance of
cement. As the glomerular filtration rate (GFR) surviving the injuries, it may be ethically
falls, there may also be an accumulation of justifiable to withdraw active treatment
antihypertensive drugs, causing a cycle of (although a consensus must be reached with
organ dysfunction. If normal physiology is not the family). The decision regarding withdrawal
restored with 30 mUkg of fluid in a short of treatment should not be made by the family
period of time, it is unlikely that ongoing in isolation - the clinical team must guide t~e
intravenous fluid will be beneficial. Furosemide process. An advance directive provides u~eful,;
and dopamine may improve the urine output, information about the values of the patie~t.
but have no effect on the GFR in this context. Once a decision has been made to withctlraw
Renal replacement therapy is not required at treatment, it is an ethical obligation to pfovide
this stage and will not improve the renal palliative care (sedatives and analgesia)· and
outcome. A noradrenaline (norepinephrine) extubation is a reasonable course of ~ction. If
infusion is the best option as it will improve the the other injuries are deemed unsurvivable, it is
mean arterial pressure (MAP) and may improve not strictly necessary to stop all sedation/
the GFR by vasoconstriction of the efferent analgesia as this may cause a great deal of
arteriole (which is dilated by ACE inhibition). pain and suffering.

10.2. Answer: B. 10.4. Answer: C.


This man has orthodeoxia (arterial saturations This man has life-threatening asthma with type
reduce on standing). In the context of chronic II respiratory failure. The Pa0 2 is high as the
liver disease and a normal chest examination, it patient is receiving a high inspired oxygen
is likely that hepato-pulmonary syndrome is the concentration, and the pathology is air trapping
cause. The pathology involves inappropriate and alveolar hypoventilation. Even with very
dilation of the pulmonary capillaries so blood small amounts of ventilation, the Pa0 2 can be
flows through the lungs without becoming maintained if there is normal gas transfer
oxygenated. This is an example of shunt. The across the alveoli, but the PaC0 2 rises.
only treatment known to be effective is liver Intubation and ventilation is indicated as he has
transplantation. become unconscious. Initial ventilator settings
should prioritise adequate minute volume while
10.3. Answer: B. allowing sufficient expiratory time. High tidal
This is a very difficult situation and the right volumes and respiratory rates will lead to air
course of action will depend upon the legal trapping and a very high intrathoracic pressure:

downloaded from www.medicalbr.com


ACUTE MEDICINE AND CRITICAL ILLNESS • 65

this can cause cardiovascular collapse. Whilst infarction or bilateral cerebral infarcts are the
pneumothorax is a consideration, inserting most likely causes. ACT angiogram of the
cannulae without evidence of pneumothorax is circle of Willis has the highest diagnostic yield
likely to be harmful. in the acute setting. It is possible that this man
has locked-in syndrome.
10.5. Answer: A.
Patients should ideally be calm, able to follow 10.9. Answer: B.
commands and tolerate endotracheal intubation This scenario is likely to be an obstructed or
(a RASS score of 0). This is not always possible displaced tracheostomy tube. The most likely
as the tracheal tube is very stimulating and problem is that the inner tube has become
patients may need sedation to synchronise their blocked with secretions; this is easily remedied
breathing with the ventilator. Deep sedation by removing the inner tube and exchanging it
does not reduce the incidence of delirium or for a fresh one. A tracheostomy can become
post-traumatic stress disorder following displaced whereby it passes into a false tract
intensive care discharge: it probably increases alongside the trachea. Initial management
the risk of both complications. Daily sedation should focus on supplementing oxygen via the
breaks are used safely in many intensive care upper airway and establishing if the
units. There are many sedative agents that tracheostomy is patent (by attempting to pass
are used safely; however, etomidate is not a suction catheter). If it is not possible to pass
used by infusion as it causes adrenocortical a suction catheter, then the tracheostomy
suppression. should be removed and the patient re-intubated
via the oral route. It is dangerous to apply
10.6. Answer: A. positi~e pressure to a potentially dislodged
Weaning should occur when the primary tracheostomy. It can cause a large amount of
pathology has resolved. During a febrile surgical emphysema, which can prevent oral
episode, C0 2 production is increased and a intubation.
higher minute volume is required to maintain
normocapnia: not a good time to wean. During 10.10. Answer: A.

I
the weaning process, the PEEP is usually ARDS is a syndrome characterised by
maintained to keep the lung bases open, while infiltration of the lungs by an inflammatory
the pressure support and Fi0 2 are weaned exudate. This causes the features of
(although a Pa0 2 of > 10 kPa (75 mmHg) would hypoxaemia and reduced lung compliance. T~e
be a more usual target). A spontaneous Berlin definition of ARDS stipulates that the
breathing trial with minimal or no ventilator following must be present:
support is frequently used to assess if a patient (,
• The time of onset must be within 1 week ?f
is ready for extubation - if it is unsuccessful,
a known clinical insult, or new or worsening
the support should be reinstituted before any
respiratory symptoms, i.e. not long-standing
lung injury is incurred and further options
pulmonary disease
considered to optimise respiratory function. A
• Bilateral opacities present on chest X-ray,
tracheostomy is usually only considered when
not fully explained by effusions, lobar/lung
there is repeated failure to wean.
collapse or nodules, i.e. not lobar pneumonia
in isolation
10.7. Answer: A.
The loss of grey-white differentiation on a CT • Respiratory failure not fully explained by
scan is suggestive of a diffuse ischaemic injury cardiac failure or fluid overload. Objective
to the brain. All the other signs are strongly assessment (e.g. by echocardiography) must
predictive of a poor neurological outcome. exclude hydrostatic oedema if no risk factors
are present. (Therefore mitral regurgitation
10.8. Answer: A. causing pulmonary oecje"ma is this scenario
The differential diagnosis here is a cerebral would not classify) ·
vascular event, a rapidly progressive • Impaired oxygenation: Pa0,!Fi0 2 ratio of
encephalomyelitis or a form of seizure disorder. <40 kPa (300 mmHg)_
Given the speed of onset and the bilateral In this question ARDS in association with
neurological deficits, it is likely that there has influenza pneumonia is the only case that
been a vascular event. A vertebrobasilar meets these criteria

downloaded from www.medicalbr.com


~
66 • ACUTE MEDICINE AND CRITICAL ILLNESS
i
f

10.11. Answer: E. cardiac output, but have little or no role in this


The history and investigations are all suggestive scenario.
of viral myocarditis. This can cause hyperacute
cardiogenic shock. In this case, the use of 10.14. Answer: B.
inotropes is unlikely to modify the outcome as Controlling blood glucose levels at
the myocardium is profoundly damaged. An 6-10 mmoi/L (108-180 mg/dl) is
intra-aortic balloon pump is unlikely to provide recommended. Low-molecular-weight heparin
adequate organ perfusion. The global is more efficacious as thromboembolic
myocardial dysfunction and the presentation prophylaxis than unfractionated heparin.
more generally is not suggestive of coronary Five-percent glucose should not be used as the
disease, and ventilation with high PEEP will not 'flush' bag as it can lead to erroneous glucose
improve cardiac output. Venous-arterial ECMO measurements and a risk of inappropriate
has been used successfully in a number of insulin use. Several large studies have
severe myocarditis cases. Despite the severity suggested a transfusion threshold of 70 g/L
I of the cardiogenic shock, many patients with is as safe as 100 g/L and has a lower
l this condition will recover if they are adequately
supported.
requirement for transfusion. Proton pump
inhibitors or histamine-2 antagonists reduce the
incidence of gastrointestinal bleeding, but may
10.12. Answer: A be associated with increased nosocomial
There is a larger volume of carbon dioxide per infection.
100 ml of arterial blood than oxygen
(approximately 50 ml vs. 20 ml). The majority 10.15. Answer: E.
of blood carbon dioxide is carried as The clinical picture here may be due to
bicarbonate ions, but significant quantities are pulmonary oedema or secretion retention with
bound to haemoglobin (carbamino compounds) exhaustion and septic shock. It is unlikely that
and dissolved in the plasma. This is significant, DC cardioversion will correct the underlying
as desaturated haemoglobin has a higher pathology. Critical care is not contraindicated
affinity for carbon dioxide than fully saturated on the basis of age; frailty is a more important
haemoglobin; hence the PaC0 2 will be lower predictor of outcome. Intubation and ventilation
for a given carbon dioxide content if the oxygen can be performed with low doses of hYRhotic
saturations are low (the Haldane effect). This agents, and awake intubation would be/'
partially explains why patients with type II challenging in an agitated patient. Escqlation of
respiratory failure become more acidotic when antibiotics is not required unless there are
high levels of oxygen are administered. When specific risk factors for resistant orga,r:iisms.
temperature drops, carbon dioxide solubility Peripheral noradrenaline (norepinephfirie) via a
increases, so for a given content of carbon well-sited cannula may provide enough stability
dioxide, the partial pressure will be lower at to facilitate intubation and ventilation
lower temperatures. High capillary carbon (Box 10.15).
dioxide shifts the haemoglobin-oxygen
dissociation curve to the right, improving 10.15 Optimising safety during intubation
oxygen offloading at the tissues (Bohr
Intervene early in the disease process (once it has become
effect). clear that the disease trajectory is downward)
Use a stable anaesthetic technique: low doses of sedative
10.13. Answer: E. agents and rapidly acting paralytic agents
Remember that intubation should be performed by the
The data shows a high cardiac output with low
most experienced operator available
pulmonary arterial, left atrial (wedge pressure) Use techniques to optimise oxygenation and ventilation in
and central venous pressures. This pattern the period around intubation, e.g. keeping non-invasive
suggests vasodilatation, which is common ventilation in situ for pre-oxygenation, leaving high-flow
following cardiopulmonary bypass. Cardiac nasal cannulae on for the intubation process, and using
a video-laryngoscope in an anticipated difficult
tamponade typically causes a low cardiac intubation
output and increased intracardiac pressures.
Graft occlusion is likely to reduce the cardiac 10.16. Answer: B. •
output, and the pulmonary artery wedge The most likely diagnosis accounting for these
pressure is likely to increase. lnotropes and symptoms is critical illness myopathy (CIM).
intra-aortic balloon pumps may increase the There are no features of a central nervous

downloaded from www.medicalbr.com


ACUTE MEDICINE AND CRITICAL ILLNESS • 67

system lesion. Normal sensation and present withdrawal, or medication side-effect.


reflexes make a peripheral neuropathy unlikely. Hyperactive delirium is also characterised by
Rhabdomyolysis (and myositis) are a restlessness, agitation (which may be severe)
consideration and measuring a creatine kinase and impairment in sleep duration and
(CK) level is good practice, although this is far architecture. A longer disease trajectory is more
less common than CIM in this population. The suggestive of cognitive decline. A positive urine
prognosis of CIM is generally good provided dipstick is not uncommon in older patients and
the patient recovers from the primary is not diagnostic of a urinary tract infection but
pathology. may suggest a precipitant of delirium.

10.17. Answer: B. 10.19 Risk factors for delirium


It is likely that this patient is brain dead (given
Predisposing factors
the imaging and the dilated pupils). It is
Old age Sensory impairment
appropriate to undertake formal brain-death Dementia Polypharmacy
testing, which requires a strict set of clinical Frailty Renal impairment
criteria. If these are met, under UK law there Precipitating factors
is no requirement to undertake other Intercurrent illness Dehydration
investigations to demonstrate an absence Surgery Pain
Change of environment or ward Constipation
of perfusion to the brain. The evidence for
Sensory deprivation (e.g. Urinary catheterisatlon
cooling (to 32-33°C} and neurosurgery for darkness) or overload (e.g. Acute urinary retention
haemorrhagic transformation is uncertain: these noise) Hypoxia
treatments are not currently recommended. Medications (e.g. opioids, Fever
psychotropics) Alcohol withdrawal
10.18. Answer: C.
Hypertensive emergencies can develop in 10.20. Answer: C.
people with or without pre-existing The GCS is the composite score of the best
hypertension. Precipitants may include drugs eyes (out of 4), verbal score (out of 5) and
that produce a hyperadrenergic state, such as motor (out of 6). This woman will score 1 point
cocaine, amphetamines or monoamine oxidase for eyes, 2 points for verbal (incomprehensible
inhibitors. Nausea and vomiting may be a sign sounds) and 4 points for flexion/withdrawal,
of raised intracranial pressure, whilst dyspnoea totalling 7/15.
may indicate pulmonary oedema. Chest
discomfort may represent either myocardial 10.21. Answer: C.
ischaemia or aortic dissection. It is generally The pain associated with myocarditis or .
unwise to lower the blood pressure too quickly, pericarditis is often also described as 'sharp/ •
as ischaemic damage may occur in the and may 'catch' during inspiration, coughing br
vascular beds that have become accustomed lying flat. It typically varies in intensity with
to a higher perfusion pressure. A rule of thumb movement and the phase of respiration.
is that, in the absence of aortic dissection or Patients with myocarditis or pericarditis may
acute stroke, the mean arterial pressure should describe a prodromal viral illness. The pain
be reduced gradually by about 10 to 20% in associated with aortic dissection is typically
the first hour. The mean arterial pressure at the described as 'tearing' and may radiate to the
end of the first 24 hours of treatment should be intrascapular region. Pulmonary embolism may
no more than 25% lower than the initial mean demonstrate different ECG features, most
arterial pressure. commonly a sinus tachycardia, and rarely the
typically described 'S 10 3T3 ' pattern. A type 1
10.19. Answer: A. myocardial infarction would be unlikely in a 25
Delirium is generally characterised by several year old and musculoskeletal chest pain would
key features: disturbance of attentioFJ and not produce ECG changes(
awareness that fluctuates over the course of a
day, additional disturbance in cognition that is 10.22. Answer: B.
not explained by another pre-existing or Epiglottitis is the diagnosip most likely to lead to
evolving neurocognitive disorder, and airway obstruction and death. it. requires
identification of a plausible cause such as a prompt recognition and specialist airway
medical condition, substance intoxication or management. Traditionally considered a

downloaded from www.medicalbr.com


68 • ACUTE MEDICINE AND CRITICAL ILLNESS

paediatric disease, the incidence is increasing cancer, trauma, surgery, immobilisation,


in adults, whilst falling in the paediatric combined oral contraceptives, pregnancy
population after the introduction of the and genetic factors affecting coagulation
Haemophilus B influenza vaccine. processes.

10.23. Answer: D. 10.26. Answer: B.


Delirium manifests as a disturbance of arousal The haemoglobin-oxygen dissociation curve
with global impairment of mental function delineates the relationship between the
causing drowsiness with disorientation, percentage saturation of haemoglobin with
perceptual errors and muddled thinking. Unlike oxygen (S02) and the partial pressure (P0 2)
dementia, it is reversible when the underlying of oxygen in the blood. A shift in the curve
illness resolves. Cognitive impairment, visual will influence the uptake and release of
hallucination, anxiety and disorientation may oxygen by the haemoglobin molecule. The
occur in both delirium and dementia - it is curve moves to the right, increasing the
reversibility that is the hallmark of delirium. offloading of oxygen in the tissues, in response
to increases in temperature, 2,3-DPG, PC0 2
10.24. Answer: A. and acidosis. The curve moves to the left
Chronic subdural bleeds are common in the secondary to decreases in temperature,
elderly, and develop over a period of days to 2,3-DPG, PC0 2 and alkalosis (Fig. 10.26;
weeks, often after minor head trauma. The upper line). Clinically, if arterial P0 2 is critically
bleeding from a chronic bleed is slow, from low, oxygen uptake in the lungs may be
bridging vessels in the dura. Symptoms are impaired by a right shift, resulting in severe
manifold, but include headache, increasing ·tissue hypoxia and end-organ failure.
confusion, fluctuating consciousness and
ataxia. 10.27. Answer: E.
A pneumothorax is an abnormal collection
10.25. Answer: C. of air in the pleural space. A secondary
Deep vein thrombosis (DVT) is the formation of pneumothorax occurs as a complication o(
a blood clot within a deep vein, predominantly underlying lung disease, whereas a primat
in the legs. Signs include pain, swelling, spontaneous pneumothorax occurs !;'
redness, warmness, and engorged superficial spontaneously and in the absence of cli~fcal
veins. Risk factors for developing DVT include lung disease. Clinical signs may be 1y

100
93
87

6"' 75
~
c
0

~
~ 50
<f)
c '----+Normal
:.a0 P50 = 3.5 kPa
=26 mmHg
OJ
0
~ 25
"'
I

0~~~~~.-.-.-+-.-.-r-r-r-r
kPa 0 1 2 3 4 5 6 7 8 9 10 1112 13 14
mmHg 0 25 50 75 100
P0 2 (kPa or mmHg)

Fig. 10.26

downloaded from www.medicalbr.com


ACUTE MEDICINE AND CRITICAL ILLNESS • 69

non-specific. Chest X-ray signs include 10.32. Answer: A.


absence of pulmonary vascular markings This man most likely has concealed bleeding
peripherally and a rim of air between lung and from an abdominal injury, given his mechanism
chest wall may be visible. of injury. His observations suggest
hypovolaemia and a liver or splenic injury is
10.28. Answer: A. likely. This man requires fluid resuscitation, a
Carbon monoxide is a colourless, odourless, surgical opinion and appropriate imaging of his
tasteless, poisonous gas produced by abdomen. A chest X-ray to rule out thoracic
incomplete burning of carbon-based fuels. pathology and free air under the diaphragm
Carboxyhaemoglobin is a stable complex that would also be reasonable but would be highly
forms between haemoglobin and carbon unlikely to provide a definitive diagnosis.
monoxide molecules. This hinders the ability of
oxygen to bind to haemoglobin molecules, 10.33. Answer: D.
which will preferentially bind to carbon A raised respiratory rate (tachypnoea) is the
monoxide, resulting in tissue hypoxia even earliest and most sensitive sign of clinical
though the Sp02 reading may be normal. The
half-life of carboxyhaemoglobin can be reduced
deterioration. Tachypnoea may be primary (i.e.
a problem within the respiratory system) or
m.
by administration of I 00% oxygen or with secondary to pathology elsewhere in the body. .I
hyperbaric chamber therapy. All other parameters may also be affected but
tend to occur later as end-organ damage I
10.29. Answer: D. occurs.
Focal neurological symptoms, other than those
associated with migraine, may be indicative of 10.34. Answer: A.
an intracranial mass lesion. Other headache Stroke volume is dependent on cardiac filling
'red flags' include: sudden onset, constitutional (preload) and contractility. Stroke volume is the
symptoms (such as rash), signs of raised volume of blood ejected from the left ventricle
intracranial pressure (headache worse on lying per beat. An increased preload (i.e. increased
down), new onset in a person aged >60 years. venous return) and an increase in contractility
(as a result of increased sympathetic
10.30. Answer: D. stimulation, adrenaline infusion, increased
Premonitory symptoms associated with serum calcium, or glucagon infusion) will
syncope include nausea and lightheadedness, together increase stroke volume. Conversely,
whereas a seizure may present initially with an increase in afterload (as a result of increased
signs of confusion or symptoms of aura. The peripheral vascular resistance) will decrease
unconscious period of a seizure is also more stroke volume as the heart has to work har,der
obviously symptomatic, including motor seizure against higher resistance. Heart rate has some
activity, tongue-biting or urinary incontinence effect on stroke volume as the time for
(but these are not discriminatory), and is likely ventricular filling is decreased at fast rates,
to entail a more prolonged recovery (post-ictal) decreasing the stroke volume; however, it is
phase. not the main factor. Cardiac index is a measure
that relates cardiac output (stroke volume x
10.31. Answer: A. heart rate) to body surface area; the unit of
Abnormalities in respiratory rate, Sp0 2, measurement is Umin/m 2 . Haemoglobin and
temperature, blood pressure, heart rate, oxygen saturation do not directly affect stroke
neurological response and urine output form volume, except in very extreme situations (e.g.
core components in common early warning extreme anaemia or cellular hypoxia).
systems. Each abnormality may be allocated a
score from 0 to 3, with the composite score 10.35. Answer: E.
giving an indication of the severity of The combination of hyper;t~nsion, heavy
physiological derangement. Capillary blood smoking, sudden collaps'e and recent
glucose may be used as part of such a headache (possibly suggesting a herald bleed)
scoring system in specialist departments make the diagnosis of SL,!barachnoid
(e.g. endocrine or metabolic units) but is not haemorrhage most likely. She is not prescribed
included in routine early warning any drug for diabetes that would induce
systems. hypoglycaemia.

downloaded from www.medicalbr.com


70 • ACUTE MEDICINE AND CRITICAL ILLNESS

10.36. Answer: B. criteria are met (Box 10.39). All measurements


A creatine kinase level of greater than 1000 U/L should be taken on a minimum of 5 cmH 2 0 of
is highly suggestive, although it can rise to tens PEEP or continuous positive airway pressure
of thousands in severe cases. The other tests (CPAP):
would not help in the specific diagnosis. • Mild ARDS: 40-26.6 kPa (300-200 mmHg)
• Moderate ARDS: 26.6-13.3 kPa
10.37. Answer: E. (200-1 00 mmHg)
This patient has adverse signs relating to his
• Severe ARDS: ~13.3 kPa (~100'mmHg)
tachyarrhythmia (chest pain and hypotension).
Therefore DC cardioversion, rather than medical
management, is appropriate as per Adult Life 10.39 Berlin definition of ARDS
Support guidelines. Onset within 1 week of a known clinical insult, or new or
worsening respiratory symptoms
10.38. Answer: A. Bilateral opacities on chest X-ray, not fully explained by
effusions, lobar/lung collapse or nodules
To be diagnosed with sepsis, the patient must Respiratory failure not fully explained by cardiac failure or
have suspected infection and two or more of: fluid overload. Objective assessment (e.g. by
• Hypotension - systolic blood pressure echocardiography) must exclude hydrostatic oedema if
no risk faclor is present
<100 mmHg
Impaired oxygenation
• Altered mental status - GCS score ~ 14

• Tachypnoea- respiratory rate ;:::22 breaths/


10.40. Answer: B.
min
Temperature management, i.e. maintaining a
Sepsis can also be diagnosed by suspected
temperature of 36°C and avoidance of pyrexia,
infection and an increase of two or more points
should be facilitated by the use of a cooling
on the SOFA score (Box 10.38).
blanket. This should be continued for 72 hours.
Muscle relaxants may be required to prevent
10.38 Sequential Organ Failure Assessment shivering. Other physiological targets include:
(SOFA) MAP of > 70 mmHg and systolic BP
An assessment of admission characteristics (e.g. age and >120 mmHg, blood glucose of 6-10 mToi/L
pre-existing organ dysfunction) and the maximum/ (1 08-180 mg/dl) and oxygen saturatio~s of
minimum values of 12 routine physiological 94-98% (avoiding both hypoxaemia an<tl
measurements during the first 24 hours of admission hyperoxia). With respect to PaC02 in the
(e.g. temperature, blood pressure, GCS) that reflect the
physiological impact of the illness
ventilated patient, the aim should be fbr
Composite score out of 71 normocapnia (4.5-6 kPa; 33-45 mm~g).
Higher scores are given to patients with more serious I
underlying diagnoses, medical history or physiological 10.41. Answer: B.
instability; higher mortality correlates with higher scores
All other options other than chronic renal failure
SOFA score
are likely to result in tissue hypoperfusion or
A score of 1-4 is allocated to six organ systems
(respiratory, cardiovascular, liver, renal, coagulation and poor oxygen utilisation and thus anaerobic
neurological) to represent the degree of organ metabolism. This leads to a rise in lactate and
dysfunction, e.g. platelet count > 150 x 109/L scores 1 is frequently used both to diagnose
point, < 25 x 109/L scores 4 points hypoperfusion or poor oxygen utilisation and to
Composite score out of 24
Higher scores are associated with increased mortality measure response to treatment.

10.42. Answer: A.
10.39. Answer: A. This woman has sepsis, the initial treatment of
Severity of hypoxaemia is calculated using a which includes early antibiotics and IV fluid. The
Pa/Fi0 2 ratio. This is a number calculated by other options may ~II be used to manage
the Pa0 2 from an arterial blood gas divided by sepsis; however, Jhey are not first line and
the fraction of inspired oxygen (Fi0 2 , expressed should only be used with critical care
as a fraction). For example, a patient with a oversight.
Pa0 2 of 10 kPa (75 mmHg) on 50% oxygen,
i.e. Fi0 2 of 0.5, would have a Pa/Fi0 2 ratio of 10.43. Answer: E.
20 kPa (150 mmHg). This would be defined as Non-invasive ventilation is the first -lif!e therapy
moderately severe ARDS, if the other Berlin in patients with type II respiratory failure usually

downloaded from www.medicalbr.com


ACUTE MEDICINE AND CRITICAL ILLNESS • 71

secondary to an acute exacerbation of COPD, 10.50. Answer: D.


because it reduces the work of breathing and Compartment syndrome classically occurs
offloads the diaphragm, allowing it to recover following extrinsic compression of a limb due to
strength. trauma or reduced conscious level (especially
when caused by drugs or alcohol). It usually
10.44. Answer: D. presents with a tense, firm and exquisitely
Volutrauma is the result of distending forces painful limb. The pain is characteristically
from the tidal volume; barotrauma results from exacerbated by passive muscle stretching .or
excessive pressures required to inflate the lung; squeezing the compartment and altered
atelectotrauma results from alveolar collapse sensation is common. Absent peripheral pulses
and re-opening throughout the respiratory are a late sign, and their presence does not
cycle: the release of inflammatory cytokines in exclude the diagnosis. Clinical suspicion of
response to cyclical distension is termed compartment syndrome should prompt CK
biotrauma. Pulmotrauma is not used in invasive measurement and urgent surgical review - this
ventilation nomenclature currently. cannot wait until the next day. D-dimer
measurement is not indicated in suspected
10.45. Answer: D. compartment syndrome.
A dose of 0.5 ml 1 : 1000 adrenaline
(epinephrine) IM forms part of the current Adult 10.51. Answer: C.
Life Support guidelines for anaphylaxis. IV Initial strategies to manage raised ICP include
adrenaline may be given by an experienced ensuring adequate sedation, sitting the patient
operator in anaesthesia or critical care but is up to ensure adequate venous drainage from
not a first-line management technique. the head, and ventilating to low-normal
end-tidal C0 2 . If this fails, further strategies
10.46. Answer: B. include instituting neuromuscular blockade, and
This patient has hyperkalaemia refractory to administering mannitol or hypertonic saline. A
medical management and therefore the next neurosurgical opinion may be appropriate if the
step is renal replacement therapy. In view of his above strategies are unsuccessful.
ECG changes, and potential for significant
deterioration, this should take place in a critical 10.52. Answer: A.
care area. Given the timescale and his past medical
history, this man is most likely experiencing ,
10.47. Answer: A. acute alcohol withdrawal. He is at risk of both
Whilst most cases of cardiogenic shock delirium and hepatic encephalopathy (especially
following Ml are due to left ventricular in view of his recent Gl bleed), and these rJ~ed
dysfunction, the clinical features of a raised JVP to be considered in the differential diagnosis1•
and narrow pulse pressure, alongside Whilst benzodiazepines are still considered
low-voltage QRS complexes, suggest the most appropriate treatment for
tamponade secondary to infarction and free management of alcohol withdrawal, careful
wall rupture post-MI. An urgent dosing must be used in the context of
echocardiogram should be performed, followed advanced liver disease.
by pericardiocentesis.
10.53. Answer: D.
10.48. Answer: E. An IABP is commonly inserted via the femoral
Early bystander CPR and early defibrillation are artery. Despite its effectiveness in achieving
the two interventions most likely to improve predetermined physiological goals, there is no
survival after cardiac arrest. All other answers convincing evidence for increased survival in
are markers suggestive of poor prognosis. cardiogenic shock. The b~loon is inflated with
helium and, even when ~nserted correctly under
10.49. Answer: E. radiological screening, mesenteric ischaemia
The FAST HUG checklist for daily ward rounds remains a risk. The balloon is inflated in
includes: feeding, analgesia, sedation, diastole, augmenting thEl. forward flow of blood
thromboprophylaxis, head of bed elevation (to to the abdominal organs and improving
reduce the incidence of passive aspiration), diastolic pressure proximal to the balloon, thus
ulcer prophylaxis and glucose control. optimising coronary perfusion.

downloaded from www.medicalbr.com


1
72 • ACUTE MEDICINE AND CRITICAL ILLNESS
i
10.54. Answer: C. of IV morphine 2 hours previously raises the
Critical illness polyneuropathy is due to
peripheral nerve axonal loss and presents as
proximal muscle weakness with preserved
possibility that the coma is secondary to a
narcotic drug and, therefore, brain-death
testing cannot be carried out reliably.
I
sensation. Nerve conduction study results are
as described for this patient. The clinical history 10.56. Answer: A.
is not in keeping with multiple sclerosis or Wound site infection, tracheal stenosis and
brainstem stroke and, by definition, muscle tracheomalacia are late complications of
biopsy must be abnormal in critical illness percutaneous tracheostomy. Performed
myopathy. correctly, laryngeal damage should not be a
complication of percutaneous tracheostomy.
10.55. Answer: A. In those patients where percutaneous
Preconditions for testing for brain death tracheostomy is anticipated to be difficult, ear,
include: the patient is deeply comatose, is not nose, and throat (ENl) referral should be made
hypothermic, has no profound electrolyte or for consideration of surgical tracheostomy.
metabolic disturbance and that the patient
is maintained on a ventilator because 10.57. Answer: D.
spontaneous respiration is inadequate or has Whilst all of the options may be applicable to a
ceased. Whilst atracurium is a muscle relaxant patient with upper Gl haemorrhage, the first
that would lead to cessation of respiration, it is priority must be resuscitation and maintenance
short acting and would not cause effects 72 of adequate haemoglobin and thus
hours later. The administration of a large dose oxygen-carrying capacity.

I
/

downloaded from www.medicalbr.com


DH Dockrell, BJ Angus

Infectious disease
Multiple Choice Questions
11.1. A 29 year old woman returns from a trip C. Hepatitis A serology
to Vietnam. She ignored pre-travel advice and D. Hepatitis B serology
vaccinations. She ate local foods, including E. Leptospirosis serology
several freshwater fish dishes. One month after
her return she starts to note migratory nodules 11.4. A 12 month old child presents to casualty
over her abdomen, which are itchy. Her with his father. He has been eating poorly and
eosinophil count is mildly elevated. What is the running a fever for the last 36 hours, after which
most likely cause of this clinical picture? he developed a widespread maculopapular rash
A. Ascaris lumbricoides on the trunk. There are no localising findings on
B. Clonorchis sinensis physical examination. The father tells you that
C. Fasciola hepatica his son has had all his vaccinations, including
D. Gnathostoma spinigerum measles, mumps and rubella (MMR). What is
E. Wuchereria bancrofti the potential cause of this infection?
A. Coxsackie virus
11.2. A 34 year old man who works as an army B. Enterovirus 71
reservist presents with bilateral facial nerve C. Human herpesvirus 6
palsy coming on over a period of a few days. D. Parvovirus B19
Otherwise, neurological examination of cranial E. Rubella
nerves is normal. He has been on regular army
exercises in rural Wales. He does not 11.5. A 26 year old pregnant woman, in the
remember any tick bites or a typical rash for seventh month of pregnancy, presents
Lyme disease. What is the likeliest diagnosis? concerned that she was visited 5 days ago by
A. Botulism her niece who the next day developed an itchy
B. Cerebovascular infarction vesicular rash. The niece stayed in her house
C. Complex migraine for 3 days. The niece saw her family physician
D. Neuroborreliosis on her return home and has been diagnosed
E. Tetanus with chickenpox. The woman is concerned
because she does not remember ever having
11.3. A 42 year old businessman presents chickenpox as a child, a fact confirmed by her
with fever and back pain. He had visited family mother. You arrange to check a varicella zoster
in Pakistan 8 months previously. He has a serology, which is negative. Which of the
temperature of 38.6°C. Urine dipstick'is negative following should you offer to prescribe?
as is his chest X-ray. Blood tests show a mild A. Aciclovir orally for 7 days
hepatitis and mild thrombocytopenia. What test B. Intravenous immunoglobulin
will be most likely to establish the diagnosis? C. Vaccination against varicella zoster virus
A. Blood film D. Valaciclovir orally ·
B. Dengue serology E. Varicella zoster immunoglobulin

downloaded from www.medicalbr.com


7 4 • INFECTIOUS DISEASE

11.6. A 54 year old man receives a cadaveric referred to an outpatient clinic and initial history
renal transplant. Before transplantation he is and physical examination have revealed no
found to be cytomegalovirus (CMV) obvious abnormalities. The travel history is
immunoglobulin G (lgG) negative and he unremarkable and she has never lived in
receives a transplant from a person who is countries with risk of tropical infections or
CMV lgG positive. Administration of which drug tuberculosis. Routine bloods show normal full
lessens his chance of developing CMV and its blood count but C-reactive protein (CRP) and
associated complications post-transplantation? erythrocyte sedimentation rate (ESR) that are
A. Brincidofovir elevated. Liver function tests show minor
B. Cidofovir abnormalities and the urinalysis shows some
C. Foscarnet protein and red blood cells. Human
D. Valganciclovir immunodeficiency virus (HIV) serology is
E. Zanamivir negative. Routine blood cultures are negative
and a chest X-ray, computed tomography (CT)
11.7. A 28 year old man returns from a holiday abdomen and echocardiogram are all reported
to Brazil. After a short febrile illness he is as normal. What would be an appropriate next
diagnosed with Zika virus. What practical step in investigation?
advice should he be given? A. Bone marrow aspirate for culture
A. Avoid alcohol for 2 months B. Cerebrospinal fluid examination
B. Avoid sharing towels for 1 week C. Liver biopsy
C. Avoid strenuous exercise for 2 weeks D. Mammogram
D. Condom usage for 6 months .E. Positron emission tomography (PET) scan
E. Sexual abstinence for 2 weeks
11.11. A 29 year old man is referred to clinic
11.8. A survivor from the West African Ebola virus because of 4 weeks' symptoms of fevers,
disease outbreak presents for routine medical arthralgia and sore throat. On examination he
check-up. Which of the following is a late has enlarged cervical lymph nodes but the
complication, frequently described in survivors, pharynx shows no erythema or purulence ..
which it may be appropriate to assess for? There is hepatosplenomegaly and you no~ a
pale pink macular rash over the abdornery.
A. Anterior uveitis
Initial blood tests show an increase in ,,
B. Diabetes mellitus
polymorphonuclear leucocytes and a m#rkedly
C. Hypothyroidism
elevated ferritin. Routine cultures and
D. Immune thrombocytopenic purpura
autoantibodies are negative and an HIV test is
E. Ulcerative colitis I,
pending. What would be an initial empiric
treatment? ·
11.9. A 23 year old nurse, previously fit and
well, presents with fever, persistent sore throat A. Antiretroviral therapy
and stridor. He is unable to eat or drink. B. Erythromycin
On examination he has tonsillar enlargement C. Non-steroidal anti-inflammatory drugs
and anterior and posterior cervical D. Penicillin
lymphadenopathy. A spleen tip is palpable E. Prednisolone
in the abdomen. Blood tests reveal a
lymphocytosis and borderline elevation of the 11.12. A 50 year old man is being treated for
transarninases. A blood film shows frequent acute myelogenous leukaemia with
atypical lymphocytes. Which of the following chemotherapy. He develops neutropenic fever.
should be used to treat his condition? Physical examination is unremarkable and the
central venous catheter '(CVC) tunnel site
A. Aciclovir
B. Cytotoxic T lymphocytes demonstrates no erythema or pus. Which of
C. Prednisolone the following would/be most helpful in
D. Rituximab establishing a diagnosis of a eve line
E. Valaciclovir infection?
A. Differential time to positivity of CVC versus
11.10. A 61 year old woman presents with 3 peripheral blood culture
weeks' unexplained fever. She has been B. Negative peripheral blood cultures

downloaded from www.medicalbr.com


INFECTIOUS DISEASE • 75

c. Positive peripheral blood cultures after < 12 A. Atypical lymphocytes


hours of culture B. Low haemoglobin with increased
D. Urine cultures demonstrating Pseudomonas reticulocytes
aeruginosa C. Low haemoglobin with low mean corpuscular
E. Vegetations on the aortic valve on volume
echocardiogram D. Monocytopenia
E. Thrombocytopenia
11.13. A 25 year old man presents with fever,
pharyngitis, oral ulceration and a severe
11.16. A 13 year old Peruvian boy is evaluated
headache. History reveals he spent the last 3
by a neurologist for new-onset seizures. As
weeks working in South-east Asia and returned
part of the work-up a magnetic resonance
3 days ago. Physical examination reveals signs
imaging (MRI) scan of the head is performed,
of meningism. He is noted to have a solitary
which shows multiple small lesions, several of
ulcer at his anus. Blood tests reveal
which appear calcified. After establishing a
lymphopenia, thrombocytopenia and abnormal
diagnosis, what would appropriate therapy for
liver function tests. A malaria filrn and rapid
this condition include?
diagnostic test, dengue antigen and
polymerase chain reaction (PCR), and A. Albendazole
monospot are all negative. Which of the B. Atovaquone
following is most likely to establish a definitive C. Mebendazole
diagnosis? D. Nifurtirnox
E. Piperazine
A. Cytomegalovirus serology
B. HIV-1 combined antigen and antibody test
11.17. A 6 year old child is bought into hospital
C. Lumbar puncture and bacterial culture for
by his father with fever and a petechial rash.
syphilis
You perform a physical examination and some
D. Pathergy test for Behget's disease
initial blood tests that show evidence of
E. Rectal biopsy and PCR for lymphogranuloma
leucopenia and thrombocytopenia but no
venereum
evidence of malaria. A dengue PCR is positive.
11.14. A 45 year old woman returns from a Which of the following signs on medical
2-week vacation in Mexico. She has developed examination suggests the need for
diarrhoea with abdominal bloating and nausea. hospitalisation and intensive monitoring for
complications?
She has had 6-8 bowel motions a day without
blood or mucus, which have persisted now for A. Ankle swelling
16 days. Blood tests show normal full blood B. Lymphadenopathy / •
count, CRP and blood biochemistry. C. Petechial rash
Which of the following tests is most likely to D. Shifting dullness on abdominal examination
establish the diagnosis? E. Temperature elevation for rnore than
A. Anti-transglutarninase antibodies 48 hours
B. Cyclospora cayetanensis PCR
C. Entamoeba histolytica serology 11.18. An 18 year old student with known
D. Shigella flexneri cultures frorn the stool beta-thalassemia major that is stable with
E. Yersinia enterco!itica serology regular transfusions presents with increased
fatigue and lethargy. Full blood count, blood
11.15. You are working as part of a medical aid film and a bone marrow aspirate are suggestive
organisation's team providing care in a refugee of aplastic crisis. He lives with his family and
carnp in sub-Saharan Africa. A 5 year old child has five siblings, ranging in age from 3 to 16
is bought by her mother for a medical years. During follow-up the aplasia gradually
check-up. She appears small for her age and resolves without further rned(cal intervention.
you examine a stool sample by microscopy, Which of the following infebtions is the rnost
which shows multiple ova consistent with a likely cause of the aplasia?
diagnosis of hookworm. If a full blood count A. Cytomegalovirus
was analysed on this child, which of the B. Dengue
following abnormalities might be apparent with C. Human T-celllymphotrophic virus type 1
a heavy burden of this parasite? (HTLV-1)

downloaded from www.medicalbr.com


76 • INFECTIOUS DISEASE

D. Mumps
E. Parvovirus Bl 9

11.19. A 7 year old child presents with a


short history of fever, tender cervical
lymphadenopathy and pus on the tonsils. He is
D. Necator americanus
E. Strongyloides stercora/is

11.22. A 40 year old man from Turkey presents


with a history of chronic back pain and fever. On
examination an MRI scan shows sacroiliitis. He
I
i

treated with penicillin and his symptoms has a long history of consuming unpasteurised
recover. One week later his mother presents milk and the initial work-up includes testing
with a similar history and is also treated with with a serum agglutination test, which comes
penicillin. Six weeks later the child is bought back positive at high titre. What would be an
back by his mother with acute pharyngitis and appropriate initial antimicrobial regimen?
a throat swab confirms group A streptococcal A. Doxycycline, rifampicin and gentamicin
infection. His medical history is otherwise B. Flucloxacillin with rifampicin
unremarkable. In addition to prescribing C. Fluconazole with flucytosine
penicillin what additional steps would be D. lmipenem followed by doxycycline and
appropriate? co-trimoxazole
A. Aspirin prescription for 6 months E. Streptomycin with chloramphenicol
B. Blood tests for immunodeficiency
C. Clindamycin 11.23. A 40 year old with HIV presents with a
D. Erythromycin treatment 3-week history of headache. He is an
E. Throat swabs on all the family and treatment intravenous drug user and has not engaged
of all carriers of group A streptococci . with care or antiretroviral therapy. His last
recorded CD4 T-cell count was 48cells/mm 3
11.20. A 32 year old teacher presents with 18 months ago. His neurological examination
severe pain in her left leg, specifically and a CT scan of his head are all normal. A
excruciating pain in the calf. On examination lumbar puncture is performed. Which essential
there is an area of purplish discoloration but diagnostic test would help establish the
otherwise little to see. Temperature is 39.5°C, diagnosis? .
pulse rate 122 beats/min and blood pressure
(BP) 90/60 mmHg. Which of the following is the
A. ~-o-glucan
assay in serum
B. Cryptococcal antibody measurement )n
I
most appropriate initial investigation to promptly ~rum v
!/
establish a diagnosis? C. Cryptococcal antigen test on cerebrospinal
A. CT scan leg fluid (CSF)
B. Doppler leg D. Cryptococcal PCR on CSF I
C. Inspection of muscles in theatre by a E. Galactomannan enzyme-linked
surgeon immunosorbent assay (ELISA) on CSF
D. MRIIeg
E. Ultrasound leg 11.24. An 84 year old nursing home resident is
re-admitted with Clostridium difficile infection.
11.21. A 25 year old man from Somalia She has been on a prolonged course of
presents to the hepatologist because of antimicrobials to treat an intra-abdominal
derangements in his liver function tests. Blood infection that arose as a complication of a
tests reveal an elevated alkaline phosphatase ruptured diverticular abscess but these have
and bilirubin as well as a blood eosinophilia. now stopped. Her first bout of C. difficile
Abdominal ultrasound shows a mass in the left infection was severe and treated with
lobe of the liver and some lymph node vancomycin. She then relapsed and was
enlargement around the porta hepatis. He has treated in a clinical trial with fidaxomicin. This is
been previously well and takes no regular her second relapse over a 3~month period.
medications and drinks no alcohol but does Prior to her diverticular abscess she had
chew khat leaves. Serology for which parasite been well and was only on treatment for
may be positive in this case? hypertension. WhC!,_t is a potential therapeutic
A. Enterobius vermicularis option to manage her/elapsing infection?
B. Fasciola hepatica A. Ciprofloxacin
C. Gnathostoma spinigerum B. Glucocorticoids

downloaded from www.medicalbr.com


INFECTIOUS DISEASE • 77

c. Faecal transplantation which chronic medical condition and its


D. Life-long fidaxomicin treatment would most likely prevent her from
E. Life-long metronidazole having the yellow fever vaccine, and might
prompt her to reconsider whether she should
11.25. A 17 year old student presents with a undertake this trip?
short history of severe headache and A. Depression, stable on a selective serotonin
photophobia. A physical examination notes re-uptake inhibitor (SSRI)
neck stiffness but nothing else. A lumbar B. Diabetes mellitus, on insulin
puncture is performed and shows an elevation C. Epilepsy, controlled on chronic seizure
of protein and an increase in white cells - all medication
lymphocytes. Initial PCR for enterovirus and D. HIV on antiretroviral therapy with a high CD4
herpes simplex virus (HSV) are negative and T-cell count in the normal range·
blood HIV antigen and antibody are also E. Liver transplantation 18 months previously,
negative. He has not lived in an area that puts receiving tacrolimus and mycophenolate
him at risk of any geographically restricted or immunosuppression
arthropod-transmitted central nervous system
infections. As a child the patient moved about 11.28. A 27 year old female engineer is
constantly with his parents who were musicians admitted with fever, BP 80/40 mmHg and
and does not remember having very many pulse rate of 120 beats/min. There is a
immunisations. He has no known risks for widespread faint erythematous rash over her
tuberculosis (TB), does not take any medicines body and a history of a sore throat 5 days ago.
or recreational drugs and has no other medical Initial blood tests show a marked elevation of
problems. Which important additional the neutrophil count, evidence of disseminated
diagnostic cause of meningitis should be intravascular coagulopathy and acute kidney
considered in this case? injury. Your region has a very low level of
A. Adenovirus community-acquired meticillin-resistant
B. Chickenpox Staphylococcus aureus (CA-MRSA). In addition
C. Measles to intravenous fluids, high-dose penicillin and
D. Mumps flucloxacillin, which other antimicrobial is most
E. Rubella often added to the initial regimen?
A. Chloramphenicol
11.26. A 7 year old male heart transplant B. Ciprofloxacin
recipient is referred to your clinic for advice. He C. Clindamycin
receives a annual influenza ('flu') vaccine but D. Doxycycline
this 'flu' season there have been a large E. Vancomycin
number of 'flu' cases in individuals despite
immunisation, suggesting the circulation of a 11.29. A 25 year old maize farmer from
strain not covered by this year's vaccine. 'Flu' Tanzania develops high fever and lymph node
cases have started to increase in your enlargement in the groin. He is bought into
community and several children in the boy's your medical facility and noted to have
school have started to develop 'flu'. What tachycardia and hypotension. You perform an
additional agent could be used as prophylaxis aspirate of the collection in the groin and then
to decrease the chance of influenza infection in a Gram stain on this material. What are you
this boy at high risk of influenza morbidity? most likely to see when you look at the slide
A. Amantidine down the microscope?
B. Oseltamivir A. Bipolar staining Gram-negative coccobacilli
C. Ribavirin B. Chinese letter appeara~ce of Gram-positive
D. Rimantadine rods /
E. T enofovir C. Clusters of Gram-positive cocci
D. Drumstick appearance of Gram-positive rods
11.27. A 26 year old female botanist attends the E. Filamentous pseudor-oycelium
travel clinic for advice prior to deciding whether
to undertake a 6-week trip to the Amazon 11.30. A 40 year old man presents to his local
rainforest to study the local flora. On review, clinic in India with widespread cutaneous

downloaded from www.medicalbr.com


78 • INFECTIOUS DISEASE 1
I
I
!
lesions and a diagnosis of leprosy (Hansen's A. Blood film
disease) is established by slit skin smears. The B. Elevated lgE
presence of which of the following clinical C. Protrusion of a large worm from a skin
I findings suggests that he may have a high nodule
I
I
bacterial index? D. Skin snips
A. Clearly demarcated skin lesions E. Slit-lamp eye examination
B. 'Glove and stocking' sensory disturbance
C. Hypopigmented skin lesions 11 .34. A 33 year old Spanish female veterinarian
D. Loss of sensation over lesions presents with fever, headache and dry cough
E. Loss of sweating over lesions of 5 days' duration. She has been working long
hours helping local farmers deliver lambs and
11 .31. A 35 year old doctor returns to visit his calves. On examination she has crackles and
family in rural India. Two weeks later he decreased breath sounds in the left lower lobe.
develops a fever, which progresses over a Blood tests are relatively unremarkable other
number of days to high-grade fever, in than some elevation of the transaminases.
association with profound malaise. He develops Blood cultures are negative but a serological
a dry cough and subsequently diarrhoea. On test is positive. What is the likely causative
examination a spleen tip is palpable. Blood organism based on the occupational setting?
cultures identify a Gram-negative rod, which is A. Bacillus anthracis
still being identified to the species level. While B. Ch/amydophi/a psittaci
antimicrobial sensitivities are being determined, C. Coxiella bumetii
which one of the following antimicrobial agents · D. Legionel/a pneumophila
should form initial empirical treatment? E. Yersinia pestis
A. Ceftriaxone
B. Chloramphenicol 11.35. A 60 year old businessman from Mumbai
C. Ciprofloxacin went on vacation to South Africa. While there
D. Co-amoxiclav he went on safari and remembers noting a tick
E. Co-trimoxazole bite. He used insect repellent and took mp.laria
prophylaxis. On returning home he devel6ps
11.32. A 43 year old Egyptian engineer attends fever and a severe retro-orbital headach.~. On
for routine medical examination. He is noted to examination he has no signs of meningi.sm but
have haematuria and repeated urinalysis appears flushed. You also note several) black
confirms this observation. He is referred for a eschars on both lower extremities but. rio rash.
cystoscopy and found to have squamous Initial blood work shows lymphopenia', ,but is
carcinoma of the bladder. What infection is otherwise unremarkable. Blood cultures are
most likely to be associated with this finding? negative, three malaria films show no parasites
A. Bacille Calmette-Guerin (BCG) and HIV tests are negative. Infection with which
B. Escherichia coli (enterohaemorrhagic) of the following pathogens best explains the
C. Sa/monel/a Typhi constellation of findings and likely cause of this
D. Schistosoma haematobium man's syndrome?
E. Treponema pal!idum A. Borrelia duttonii
B. Coxiella bumetii
11 .33. A 50 year old former aid worker returns C. Plasmodium knowlesi
after working in several South Pacific islands D. Rickettsia africae
over the last 15 years. She gave up trying to E. Salmonella Paratyphi
prevent mosquito bites and did not sleep under
mosquito nets. She subsequently experienced 11.36. A 41 year old man from a rural community
several episodes a year of pain and linear in the Democratic Republic of Congo presents
streaks of erythema on her legs, and also to his local clinic complaining of irregular
noted the swelling of glands in her groin. She bouts of fever and painless enlargement of
has now started to notice swelling in both legs. lymph nodes, particularly in the neck. He has
A full blood count reveals eosinophilia. Which of developed a headache and his daughter says
the following will be most helpful to support the villagers have noted a personality ch?nge.
diagnosis? Physical examination also detects t! spleen

downloaded from www.medicalbr.com


INFECTIOUS DISEASE • 79

tip but no other abnormalities. The full blood and meningism. He recently returned back to
count identifies a relative lymphocytosis and his horne in a rural area of Vietnam for a
mild elevation of transaminases. HIV tests and 3-week visit. A lumbar puncture is performed,
malaria films are negative. What would be the which shows a marked increase in white cells
best test, if available, to establish a diagnosis? and protein, and he commences treatment with
A. Blood film ceftriaxone. Later that evening the laboratory
B. Bone marrow aspirate contact you to say they have reviewed the
c. Liver biopsy white blood cells and performed sorne
D. Lymph node aspirate additional stains, which confirm there are
E. Splenic aspirate significant numbers of eosinophils, in this case
reported as 20% of the total white blood cells.
11.37. A 26 year old woman receiving total Which of the following is a potential cause of
parenteral nutrition for management of short this man's eosinophilic meningitis?
bowel syndrome, caused as a complication of A. Angiostrongylus cantonensis
Crohn's disease, is admitted because of fever B. Japanese encephalitis virus
and fatigue. Blood cultures grow Candida C. Non-prescription analgesics
tropicalis both from her peripheral blood
cultures and from the lumen of her tunnelled
central venous catheter, with the line cultures
D. Schistosoma japonicum
E. Taenia solium a.
turning positive 4 hours before the peripheral I
11.40. A 32 year old female anthropologist was
cultures. The central venous catheter is
living in remote regions of the Brazilian
removed, temporary venous access is
rainfqrest, studying the indigenous population.
established and treatment with anidulafungin is
While there, she lived in local dwellings.
commenced. In addition, which of the following
Approximately 3 months frorn the end of her
should be performed?
trip she developed an illness with an indistinct
A. CT abdomen rash and noted some enlarged lymph nodes.
B. Lumbar puncture Before returning home she went to a large
C. MRI head clinic in Brazil where she was noted to have
D. Oesophagogastroduodenoscopy lymphadenopathy and splenomegaly. They I·
E. Ophthalmological review performed sorne additional tests, including .1
xenodiagnostics with a triatomine bug, which 1;'
11.38. A 47 year old man with acute resulted in a diagnosis. She was advised she(1
myelogenous leukaemia is admitted with
needed treatment but she preferred to defer '
neutropenic fever. There are no localising signs
treatment until she was back home. Which i
or symptoms. Cultures through the central
medication is most likely to treat this / '
venous catheter and the peripheral cultures are
condition?
negative. A CT chest scan is negative, as is a
galactomannan assay. Despite treatment with A. Nelfinavir
piperacillin-tazobactam, and subsequent B. Niclosamide
addition of caspofungin and teicoplanin, he C. Nifurtimox
remains febrile but there are still no localising D. Nitazoxanide
signs. His other medications include allopurinol, E. Nystatin
omeprazole and alendronic acid. Increasing
lymphadenopathy is noted and there are 11.41. A 23 year old woman attends her family
abnormal liver function tests but no other physician having noticed a 'bull's eye' rash on
abnormalities. Which of the following is a likely her thigh and developing flu-like symptoms.
cause of this syndrome? She walks her dog regularly through local
woodland in southern England. What action
A. Allopurinol hypersensitivity reaction
should be taken? ,/
B. Cytomegalovirus infection
C. Epstein-Barr virus infection A. Ensure tetanus vaccination is up to date
D. Invasive fungal infection B. No action required
E. Penicillin allergy C. Prescribe intravenous. ceftriaxone for
2 weeks
11.39. A 25 year old man from South-east Asia D. Prescribe oral doxycycline for 2 weeks
presents with severe headache, photophobia E. Test for antinuclear antibodies

downloaded from www.medicalbr.com


80 • INFECTIOUS DISEASE

11.42. A 55 year old man returns from Medina 11.45. A 42 year old businessman presents with
in the Kingdom of Saudi Arabia. He developed a history of seizures over the last 3 weeks. He
a coryzal illness, which progressed rapidly to has been previously fit and well. He lives in a
severe dyspnoea 4 days ago and he was large house with domestic servants. A CT scan
hospitalised for 3 days in Medina before he of his head shows a number of small cystic
took his own discharge and flew home. On space-occupying lesions with a characteristic
reaching horne his family were concerned he appearance, some of which demonstrate an
was increasingly short of breath and took him opacified area protruding into the cyst. What is
to hospital. He is known to have diabetes the likely organism causing this presentation?
mellitus and chronic lymphocytic leukaemia. A. Angiostrongyloides cantonensis
On examination he is febrile; his pulse rate is B. Taenia solium
106 beats/min, respiratory rate is 20 breaths/ C. Gnathostoma spinigerum
min and oxygen saturation is 90% on air. His D. Toxocara spp.
BP is 116/78 mmHg. The examination shows E. Trichinella spiralis
bilateral crackles through both lung fields and
the chest X-ray shows bilateral infiltrates. Which 11.46. A 34 year old native Australian man is
of the following illnesses should first be admitted to a hospital in Darwin, Australia, with
excluded in this case? a widespread itchy rash with crusting lesions all
A. Acute respiratory distress syndrome (ARDS) over his body. Some have secondary infection
complicating pneumonia and he has a heart murmur. What is the
B. Avian influenza likeliest diagnosis?
C. Meningococcal sepsis . A. Impetigo
D. Middle East respiratory syndrome B. Melioidosis
coronavirus (MERS-CoV) C. Pustular psoriasis
E. Severe acute respiratory syndrome (SARS) D. Scabies
E. Varicella zoster
11.43. A 44 year old intravenous drug user is
admitted with fever, tachycardia and low blood 11.47. A 68 year old man is admitted to h9spital
pressure. Chest X-ray shows multiple nodules complaining ofabdorninal pain radiating t6 his
in the lungs. After initial blood cultures are back, following a bout of food poisoning/ Blood
performed, which intravenous antimicrobial cultures are recurrently positive with twc/ out of
should be included in initial empirical therapy? two bottles growing Salmonella Enteritidis.
(Local antimicrobial-resistance patterns suggest What is the investigation most likely to. reveal
good activity can be expected.) the diagnosis? /' ,
A. Flucloxacillin A. CT scan abdomen
B. Meropenem B. HIV serology
C. Moxifloxacin C. Serum electrophoresis
D. Piperacillin-tazobactam D. Transoesophageal echocardiograrn
E. Tigecycline E. Transthoracic echocardiogram

11.44. A 42 year old woman presents with fever. 11.48. A 24 year old female student returned
She returned from a holiday in India 2 months from a trekking holiday in Nepal 25 days ago
previously having spent 8 months travelling in with fever and diffuse abdominal pain. She has
rural areas. She has a temperature of 38.2°C. not had diarrhoea. On examination, pulse is
Urine dipstick is negative. Blood tests show 56 beats/min, BP 97/54 mmHg and
a mild hepatitis and thrombocytopenia. A temperature 39.4°C. She has a tender right
diagnosis of vivax malaria is made on blood iliac fossa and small faint spots on her
film. What test will help with f\]rther treatment? abdomen but no other skin lesions. What is the
A. Antiplatelet antibodies likeliest diagnosis?
B. Haemoglobin electrophoresis A. Appendicitis
C. Hepatitis B serology B. Cyclosporiasis •
D. Test for glucose-6-phosphate C. Dengue
dehydrogenase (G6PD) deficiency D. Scrub typhus
E. Ultrasound of spleen E. Typhoid

downloaded from www.medicalbr.com


INFECTIOUS DISEASE • 81

11.49. A 21 year old man presents to an with pain and tenderness on the left side of his
emergency department in the UK with a 3-day neck. On examination he is fevered and
history of bloody diarrhoea and right iliac fossa shocked with low oxygen saturations on room
abdominal pain. He had eaten takeaway food 2 air. Chest X-ray shows a blood-borne
days previously but other members of his family pneumonia and ultrasound shows left internal
had also eaten the meal and were well. He has jugular vein thrombosis. What is the diagnosis?
a family history of ulcerative colitis. What is the A. Adult Still's disease
likeliest diagnosis? B. Haemophagocytic lymphohistiocytosis (HLH)
A. Amoebiasis C. Kikuchi's disease
B. Bacillus cereus toxin food poisoning D. Lemierre's syndrome
C. Campylobacter infection E. Streptococcal toxic shock syndrome
D. Crohn's disease
E. Ulcerative colitis 11.54. A 44 year old truck driver was involved in
a road traffic collision; this resulted in a
11.50. A 21 year old student returns from a trip traumatic injury to his pelvis, which was
to Belize in Central America with a non-healing contaminated with soil from a ditch. He
ulcer on his face. A biopsy and PCR confirm develops a brain abscess, which is drained,
a clinical diagnosis of leishmaniasis. The and on microscopy shows long, filamentous,
organism is identified a L. braziliensis. What is branching Gram-positive rods that are weakly
the most appropriate treatment? acid-fast. What is the likeliest organism
A. Cryotherapy involved?
B. lntralesional stibogluconate A. Actinomyces israelii
C. Liposomal amphotericin B. Clostridium perfringens
D. No treatment indicated C. Mycobacterium chelonae
E. Paromomycin D. Nocardia asteroides
E. Sporothrix schenckii
11.51. A 34 year old man is admitted to
intensive care with a diagnosis of Pneumocystis 11.55. A 35 year old anthropology researcher ,.
pneumonia. He is noted to have widespread returned from a trip to Sarawak studying ·
violaceous papules on his skin and hard palate. primate behaviour 7 days previously. He had Qb
1
On biopsy these are Warthin-Starry silver stain history of monkey bites but had been workind
positive. What is the likeliest diagnosis? close to primates. He complains of fever, 1
A. Bacillary angiomatosis headache and diarrhoea. Examination reveals
B. Kaposi's sarcoma hepatosplenomegaly and his full blood cou.nt
C. Malignant melanoma shows mild anaemia, a mildly elevated white
D. Sporotrichosis cell count and a platelet count of 76x109/L.
E. Stevens-Johnson syndrome Malaria rapid diagnostic test is negative. What
is the likeliest diagnosis?
11.52. A 4 7 year old man is admitted to A. Chesson variant Plasmodium vivax infection
intensive care with a diagnosis of Pneumocystis B. Herpes B infection
pneumonia and HIV. He is noted to have C. Monkeypox
widespread purple papules on his skin and D. Plasmodium knowlesi infection
hard palate. On biopsy these are human E. Rabies
herpesvirus 8 (HHV-8) DNA positive. What is
the likeliest diagnosis? 11.56. An 18 year old female presents unwell
A. Bacillary angiomatosis with sudden onset of bloody diarrhoea with
B. Kaposi's sarcoma fever and abdominal pain. Temperature is
C. Malignant melanoma 38.9°C, pulse 110 beats/min and BP
D. Sporotrichosis 93/56 mmHg. She looks jaundiced and pale
E. Stevens-Johnson syndrome with diffuse abdominal pain. Blood tests show
a haemoglobin of 67 g/L, white cell count
11.53. A 19 year old man develops a sore 18.6x109/L, platelets 110x10~/L; bilirubin
throat and fever; 2 days after the onset, he 98 f.Lmoi/L (5.73 mg/dl), aspartate
develops left-sided chest pain and haemoptysis aminotransferase (AST) 21 U/L, creatinine

downloaded from www.medicalbr.com


82 • INFECTIOUS DISEASE

345 Jlmoi/L (3.90 mg/dL). What is the likeliest A. Contact dermatitis


diagnosis? B. Listeriosis

I A. Haemolytic uraemic syndrome C. Monkeypox


D. Palmoplantar pustulosis
I B.
C.
Hantavirus infection
Leptospirosis E. Smallpox
I D. Toxic shock syndrome
11.61. A 54 year old woman presents with
E. Typhoid fever
sudden onset of severe left -sided abdominal
11.57. A 24 year old woman is investigated for pain; she scores this at i 0 out of i 0. There is
chronic bloody diarrhoea with left iliac pain by no history of wounds or trauma. The pain is
colonoscopy. Biopsy shows granulomas in the constant in nature and not associated with
terminal ileum and CT scan shows mesenteric food. On examination she has a temperature of
lymphadenopathy. A presumptive diagnosis of 38.6°C and BP of 92/54 mmHg. Her left
Crohn's disease is made; however, stool hypogastrium is extremely tender with a small
culture grows a Gram-negative organism at bruise on the skin but she has no peritonism.
low temperatures. What is this likely to be Venous blood gas analysis shows an W ion
identified as? concentration of 76 nrnoi/L (pH 7. i 2) and a
A. Campylobacter jejuni lactate of 4.3 mrnoi/L (38. 7 mg/dL). What is the
B. Mycobacterium pseudotuberculosis likeliest diagnosis?
C. Necator americanus A. Gas gangrene
D. Salmonella Enteritidis B. Lemierre's syndrome
E. Yersinia enterocolitica C. Necrotising fasciitis
D. Perforated descending colon
11.58. A 44 year old farmer presents with fever E. Splenic rupture
and increasing shortness of breath after
clearing out one of his barns. His symptoms 11.62. A 23 year old intravenous drug user
progressed over the 48 hours following presents to the emergency department. He had
exposure, despite avoiding the barn. He has recently been injecting heroin into his s~ln. He
not had any similar symptoms previously. He is complains of double vision and has prg'gressive
found to have widespread respiratory fine difficulty in swallowing. On examinatiod he has
crackles and his chest X-ray reveals pulmonary ptosis and divergent eye movements/What is
oedema. Blood tests show evidence of acute the likeliest diagnosis? 'I
kidney injury. What is the likeliest diagnosis?
A. Guillain-Barre syndrome
A. Acute allergic bronchiolitis B. Myasthenia gravis /,
B. Allergic bronchopulmonary aspergillosis C. Staphylococcal brain abscess
C. Goodpasture's syndrome D. Tetanus
D. Granulomatous polyangiitis E. Wound botulism
E. Hantavirus infection

11.63. A 17 year old intravenous drug user


11.59. A 27 year old zoology student is on a
presents with abdominal pain, vomiting and
field trip examining bats. She receives a bite.
fever. On examination there is a painless black
Vaccination against which of the following
eschar on her left arm. What is the likeliest
should be recommended?
diagnosis?
A. Diphtheria
A. Cutaneous anthrax
B. Hepatitis A
B. Gas gangrene
C. Mumps
C. Lyme disease
D. Rabies
D. Necrotising fasciitis
E. Smallpox
E. Staphylococcal bacteraemia
11.60. A i 9 year old pet shop owner presents
with a pustular rash on her hands and arms. 11.64. A 45 year.old laboratory technician
She had recently received a shipment of pet originally from India presents with a fever and
Gambian pouch rats. What is the likeliest unilateral swelling in his neck. He also
diagnosis? complains of mild testicular pain. He is

downloaded from www.medicalbr.com


INFECTIOUS DISEASE • 83

otherwise well and blood tests are normal. A. Adenovirus


What is the likeliest diagnosis? B. Human herpesvirus 7
A. Cervical tuberculosis C. HIV-2
B. Kikuchi's disease D. Human T-celllymphotropic virus type 1
(HTLV-1)
c. Lymphoma
E. West Nile virus
D. Mumps
E. Teratoma
11 .69. A 44 year old Chinese businessman.
11.65. A 34 year old intravenous drug user returns from a trip to Hong Kong with a cough,
presents with painful swelling of his left arm. On fever and myalgia. He is diagnosed as having a
examination his arm is swollen, oedematous lower respiratory tract infection. He is worried
and dusky, with apparent crepitus. A diagnosis about 'bird flu' since he visited his relatives who
of gas gangrene is made. What is the causative are farmers and keep poultry in a rural area.
organism? Which of the following viruses might be
responsible for his syndrome if there is an
A. Clostridium difficile
epidemiological link to visiting the farm?
B. Clostridium perfringens
C. Clostridium septicum A. Bocavirus
D. Clostridium sordellii B. HI Nl pmd2009 influenza A
E. Clostridium tetani C. H5NI influenza A
D. Parainfluenza
11.66. A 24 year old waitress complains of E. SARS coronavirus
flushing and tingling around her mouth after
eating some left-over canned sardines for lunch 11 .70. A 14 year old refugee recently arrived
and has no known occupational or recreational from South Sudan is found to have ulcerated,
exposures to toxins. The symptoms resolve pustular nodules on his elbow. He also has
over an hour. What is the likeliest diagnosis? underlying deformity of the humerus. His blood
tests show him to be HIV negative but the
A. Ciguatera poisoning
venereal disease research laboratory (VORL)
B. Monosodium glutamate poisoning
test positive. What is the likeliest diagnosis?
C. Paralytic shellfish poisoning
D. Scombroid poisoning A. Bejel
E. Thallium poisoning B. Congenital syphilis
C. Pinta
11.67. An 84 year old man, previously fit and D. Podoconiosis
well, presents with a pustular painful rash in the E. Yaws (
I
left ophthalmic nerve distribution. He was
started on intravenous aciclovir but deteriorated 11.71. A 34 year old woman who takes
with headache, fever and delirium followed by prednisolone and azathioprine for control of her
sudden onset of a right hemiparesis. What is Grahn's disease presents with a 10-day history
the likeliest diagnosis? of headache and double vision. On examination
she has meningism and a 3rd ·nerve palsy. CSF
A. Acute diffuse encephalomyelitis
B. Aciclovir-induced encephalopathy shows a white cell count of 500 cells/mm 3 ,
C. Cavernous sinus thrombosis which are predominately lymphocytes, an
elevated protein and low glucose. Special
D. Granulomatous cerebral angiitis
E. Sagittal sinus thrombosis stains and antigen tests are negative. What is
the likeliest infecting organism?
11 .68. A 54 year old Jamaican man presents A. Cryptococcus neoformans
with fatigue and a widespread rash. On B. Listeria monocytogenes
examination he has widespread . C. Mycobacterium tubercul9sis
lymphadenopathy. A blood coLnt shows a D. Neisseria meningitidis '
significant increase in lymphocytes and on flow E. Streptococcus pneumoniae
cytometry these are found to be T cells. He is
also found to have an elevated blood calcium. 11.72. A 34 year old man presents with
Which of the following is a potential viral cause painful ulceration on his hand after catching
of this syndrome? and skinning wild rabbits on his farm.

downloaded from www.medicalbr.com


l
I
84 • INFECTIOUS DISEASE

I
On examination he has a area of ulceration on D. Necator americanus j
his right hand with axillary lymphadenopathy.
What is the likeliest infecting organism?
E. Taenia solium
I
I
I•

A. Bartonella henselae 11.76. A 16 year old presents unwell with I!


I
B. Francisel/a tularensis sudden onset of bloody diarrhoea with fever I
!
C. Mycobacterium marinum and abdominal pain. Her temperature is
D. Orientia tsutsugamushi 38.4 oc, pulse 112 beats/min and BP
E. Sporothrix schenckii 89/54 mmHg. She appears jaundiced and pale
with diffuse abdominal pain. Blood tests show
11.73. A 73 year old man with a history of a haemoglobin of 57 g/L, white cell count
benign prostatic hypertrophy, mitral 19.3x109/L, platelets 106x109/L, bilirubin
regurgitation, prior cholecystectomy and an 95 11moi/L (5.55 mg/dl), AST 25 U/L, creatinine
indwelling catheter presents with a 10-day 467 11rnoi/L (5.28 mg/dl). Which
history of fever and malaise. On examination he of the following is the most appropriate
I is febrile, tachycardic and has a mitral treatment?

I regurgitant murmur. Blood cultures are A. Azithromycin


repeatedly and rapidly positive for a B. Ciprofloxacin
Gram-positive coccus, identified as C. Co-trimoxazole
Enterococcus faeca/is. Which of the following is D. No antimicrobial indicated
most likely to establish the source of this man's E. Vancomycin
enterococcal infection?
A. Catheter urine sample . 11.77. A 24 year old student has returned from
B. CT abdomen a holiday in the Canary Islands. She had
C. Echocardiogram noticed a black scar where she had been bitten
D. MRI cholangiogram by a tick. Since then she has felt increasingly
E. Ultrasound of biliary tree unwell with fatigue and shortness of breath on
exertion. Her blood tests show a haemolytic
11.74. A 44 year old truck driver was involved in anaemia. What is the most likely causativE)
a road traffic collision; this resulted in a
traumatic injury to his pelvis, which was
contaminated with soil from a ditch. He
organism?
A.
B.
Babesia microti
Mycoplasma pneumoniae
I
1
ii
/1
develops a brain abscess, which is drained, C. Plasmodium falciparum ·;I

and on microscopy shows long, filamentous, D. Plasmodium vivax


branching Gram-positive rods that are weakly E. Rickettsia prowazekii
acid-fast. Which of the following is the most
appropriate treatment? 11.78. A 47 year old man originally from Zambia
A. Co-trimoxazole plus imipenem presents with lymphadenopathy and lethargy.
B. Doxycycline plus gentamicin His CSF reveals trypanosomiasis. Which of the
C. Rifabutin and clarithromycin following is the most appropriate treatment?
D. Rifampicin, isoniazid, ethambutol and A. Benznidazole
pyrazinamide B. Melarsoprol
E. Vancomycin plus rifampicin C. Miltefosine
D. Nifurtimox
11.75. You are working in a hospital in Darwin, E. Sodium stibogluconate
Northern Australia, and are asked to see a
surgical emergency admission of a 13 year old 11.79. A 31 year old man presents with a 3-day
refugee from New Guinea who has severe history of bloody diarrhoea and cramping
abdominal pain and on CT scan has necrotising abdominal pain. He had eaten a takeaway meal
colitis. He is obviously malnourished and very 2 days previously but other members of his
unwell. What is the likeliest causative primary family had also eaten the meal and were well.
organism leading to this syndrome? Which of the following is the most appropriate
A. Ascaris lumbricoides treatment? •
B. Clostridium difficile A. Azithromycin
C. Clostridium perfringens B. Ciprofloxacin

downloaded from www.medicalbr.com


INFECTIOUS DISEASE • 85

C. Co-amoxiclav concerned about herpes B virus infection. What


D. Erythromycin is the correct treatment for this infection?
E. No antibiotics A. Aciclovir
B. Foscarnet
11.80. A 21 year old woman presents having C. Ganciclovir
been bitten by a rhesus macaque monkey D. Human normal immunoglobulin
while working in a research laboratory. She is E. No treatment available

Answers
11.1. Answer: D. 11.5. Answer: E.
The presentation suggests exposure to a The patient is non-immune to varicella zoster
zoonotic parasite. The clinical scenario with virus 0/ZV) and has had a significant exposure
itchy migratory nodules emerging after eating during pregnancy within the last 7 days so
local freshwater fish dishes suggests
gnathostomiasis. Ascariasis and filariasis, the
should receive passive immunisation with
varicella zoster immunoglobulin.
0.
latter caused by Wuchereria bancrofti, are not Immunoglobulin would not have as high levels I
caused by zoonotic parasites while both of antibodies. Vaccination would take too long
fascioliasis and Clonorchis sinensis infections to generate immunity. Treatment with aciclovir
primarily involve the hepatobiliary system. or valaciclovir is not indicated (Box 11.5).

11.2. Answer: D. 11.6. Answer: D.


Valganciclovir is indicated because he is high
Neurological Lyme disease can present as
facial nerve palsy and not all patients remember risk for CMV due to his lack of pre-existing
a bite. Treatment is usually with at least I immunity and receipt of an organ from a
month of intravenous ceftriaxone. Botulism is CMV-seropositive donor. Foscarnet and /
usually a descending paralysis and tetanus cidofovir are used to treat CMV but only when , ·
1
resistance arises to ganciclovir or valganciclovii';
causes trismus with spasm.
their side-effect profile makes them less §
suitable for use in prophylaxis. Brincidofovir i~
11.3. Answer: A. being developed as an alternative agent to :
The blood film is essential in returning travellers. cidofovir while zanamivir is used to treat I•
Falciparum malaria usually present up to a influenza A virus.
month after travel, but when patients have
travelled to areas where vivax malaria is found, 11.7. Answer: D.
they may have delayed symptoms. This Zika virus may be transmitted in semen for
patient's blood film showed typical trophozoites prolonged periods after recovery. Condom use
of Plasmodium vivax. The incubation period is advised for those infected for at least 6
here would exclude the other infections. months and for those who have returned from
an endemic area for at least 2 months if they
11.4. Answer: C. do not develop signs of infection. Alcohol and
The history of rash after fever resolution and dietary interventions have not been found to
the age of the child make roseala infantum the alter Zika virus infection to date. Strenuous
likely exanthem in this case. This is caused by exercise is not contraindicated, although in the
human herpesvirus 6 or 7. Parvovirus B 19 also early recovery period of any serious virus
causes an exanthem, but typically with infection it is prudent to di~continue it. Infection
alternative features like a 'slapped cheek' rash. is not spread by body cbntact, so avoiding
Rubella is not likely as the efficacy of live rubella sharing towels is not necessary.
vaccine is very high. Coxsackie and enterovirus
infections can cause rashes, although 11.8. Answer: A.
enterovirus 71 has been primarily associated Late complications of Ebola virus disease
with neurological syndromes. include uveitis, sensineural deafness and

downloaded from www.medicalbr.com


86 • INFECTIOUS DISEASE

11.5 Infections in pregnancy


Infection Consequence Prevention and management
Rubella Congenital malformation Childhood vaccination and vaccination of
non-immune mothers post-delivery
Cytomegalovirus Neonatal infection, congenital malformation Limited prevention strategies
Zika virus Congenital malformation Avoidance of travel, delay in pregnancy if infected
Varicella zoster virus Neonatal infection, congenital VZ immunoglobulin
malformation, severe infection in mother
Herpes simplex virus Congenital or neonatal infection Aciclovir and consideration of caesarean section for
(HSV) mothers who shed HSV from genital tract at time of
delivery. Aciclovir for infected neonates
Hepatitis B virus Chronic infection of neonate Hepatitis B immunoglobulin and active vaccination
of newborn
Hepatitis E virus Fulminant hepatitis, pre-term delivery, fetal Maintenance of standard food hygiene practices
loss
HIV-1 Chronic infection of neonate Antiretroviral drugs for mother and infant and
consideration of caesarean section if HIV-1 viral
load detectable. Avoidance of breastfeeding
Parvovirus 819 Congenital infection Avoidance of individuals with acute infection if
pregnant
Measles More severe infection in mother and Childhood vaccination, human normal
neonate, fetal loss immunoglobulin in non-immune pregnant contacts
and vaccination post-delivery
Dengue Neonatal dengue if mother has infection. <5 Vector (mosquito) control
weeks prior to delivery
Syphilis Congenital malformation Serological testing in pregnancy with prompt
treatment of infected mothers
Neisseria gonorrhoeae Neonatal conjunctivitis (ophthalmia Treatment of infection in mother and neonate
and Chlamydia neonatorum)
trachomatis
Listeriosis Neonatal meningitis or bacteraemia, Avoidance of unpasteurised cheeses and other ·
bacteraemia or pyrexia of unknown origin
in mother
dietary sources j
Brucellosis Possibly increased incidence of fetal loss Avoidance of unpasteurised dairy products r/
Group B streptococcal Neonatal meningitis and sepsis. Sepsis in Risk- or screening-based antimicrobial prophylaxis
infection mother after delivery in labour (recommendations vary between countries)
Toxoplasmosis Congenital malformation Diagnosis and prompt treatment of cases, avoidance
of under-cooked meat while pregnant
,.
I
Malaria Fetal loss, intrauterine growth retardation, Avoidance of insect bites. Intermittent preventative
severe malaria in mother treatment during pregnancy to decrease incidence
in high-risk countries

arthritis. The other conditions listed have not monoclonal antibody) or cytotoxic
been reported as common late sequelae. T lymphocytes, but these do not have a role in
treatment for immunocompetent individuals.
11.9. Answer: C.
Glucocorticoids are sometimes used to treat 11.1 0. Answer: E.
complications of Epstein-Barr virus (EBV) The patient has features of pyrexia of unknown
infection. Potential indications include massive origin and evidence of raised inflammatory
tonsillar enlargement causing airway markers with some a9normalities in the liver
compromise, haemolytic anaemia or tests and urinalysis:1 Potential concerns, in
thrombocytopenia, and sometimes neurological addition to infection, include connective tissue
complications. Antivirals such as aciclovir and disorders and malignancy. A PET scan may aid
valaciclovir have no role. Patients who are identification of sites of inflammation and
immunosuppressed may develop selection of potential sites for biopsy to
lymphoproliferative disorders with EBV infection, establish a diagnosis. Investigation for
which may be treated with rituximab (anti-CD20 malignancy rnay be undertaken, but its yield is

downloaded from www.medicalbr.com


w__
INFECTIOUS DISEASE • 87

low if there are no clues to a potential source. test should combine detection of antigen with
Bone marrow aspirate for culture, lumbar antibody since the patient may not yet have
puncture and liver biopsy may be part of the developed an antibody to HIV. Although there
work-up but the diagnostic yield is low if there are several considerations in the diagnosis, the
are no signs localising to these sites, as in this clinical features and laboratory features are
case. compatible with acute retroviral syndrome
(primary infection) from recently acquired HIV
11.11. Answer: E. infection. All of the other diagnoses mentioned
The patient has presented with features of may be considered, but Behget's is a
pyrexia of unknown origin (PUO). Although comparatively rare cause of oral and genital
there is a history of sore throat there is no sign ulceration, syphilis is not diagnosed by bacterial
of pharyngitis and cultures are negative. The culture on cerebrospinal fluid but rather by
rash and markedly elevated ferritin, along with serology, and detection of a sexually
the other clinical features, make adult-onset transmitted infection such as lymphogranuloma
Still's disease a consideration, which is a venereum would not explain all the features in
clinical diagnosis that requires treatment with this case but would indicate the need for
prednisolone or alternative anti-inflammatory further tests to exclude sexually acquired HIV
therapy. There is no indication that this is infection.
streptococcal pharyngitis, so antibiotics are
not indicated. HIV-induced acute retroviral 11.14. Answer: B.
syndrome should always be considered There are multiple potential causes of diarrhoea
with presentation with PUO and rash but in travellers. In this case there is a relatively
should only be treated after diagnostic long history but an absence of acute
confirmation. inflammatory markers or evidence of dysentery.
This would fit best with a parasitic cause such
11.12. Answer: A. as cyclosporiasis, cryptosporidiosis or with
Central venous catheter infections are giardiasis. Shigella spp. and Entamoeba
suggested by detecting positive cultures in the histolytica cause a dysenteric illness and
sample from the eve at least 2 hours prior to Yersinia enterocolitica often presents with /
the peripheral blood sample, detecting 5- to abdominal pain mimicking an acute abdomen./
10-fold greater colony counts in the eve In chronic diarrhoea - usually defined as /
sample versus the peripheral blood sample or diarrhoea lasting at least 2-4 weeks - when jt
detecting at least 15 colony-forming units in infective causes have been excluded, other '
culture of the CVC tip. Peripheral blood cultures causes such as coeliac disease, inflammatoly
are frequently positive with eve line infections bowel disease and malignancy should alwdys
and a short time to culture positivity would be be considered.
suggestive of endovascular infection but not
specifically eve line infection. Although 11.15. Answer: C.
right-sided endocarditis may complicate eve Hookworm infection can cause iron deficiency
line infection, there is usually no reason for anaemia, which would be indicated by a low
left -sided endocarditis as evidenced by mean corpuscular volume. Haemolysis,
vegetations on the aortic valve. A positive urine indicated by raised reticulocytes, and
culture for P. aeruginosa would be more thrombocytopenia are associated with other
suggestive of a urinary catheter-related parasitic infections, notably malaria. Atypical
infection. lymphocytes are typically associated with viral
infections but occasionally are seen with
11.13. Answer: B. malaria and trypanosomiasis.
Any traveller with an unexplained illness should
have HIV infection excluded. In this case the 11.16. Answer: A.
presence of an anal lesion suggests the The scenario is suggestive of
possibility of unprotected anal intercourse. In neurocysticercosis for which albendazole is
addition to performing a genitourinary medicine most often prescribed. P.raziquantel is the
screen to establish the source of the lesion, an preferred alternative and the other anti-parasitic
HIV test should be performed. In this case, agents are not recommended treatment
since recent acquisition is a consideration, the options for this infection.

downloaded from www.medicalbr.com


88 • INFECTIOUS DISEASE T
I

11.17. Answer: D. may have been contaminated with the parasite,


The World Health Organization has issued such as watercress and also the khat plant,
criteria for the identification of dengue with which is commonly chewed by people who live
warning signs. These mandate intensive in regions around the Horn of Africa or Arabian
medical management and monitoring. The Peninsula.
development of ascites, suggested by shifting
dullness or other signs of fluid accumulation, 11.22. Answer: A.
are one of these warning signs. The other signs The history of living in an endemic area and of
listed here are not regarded as warning signs. consuming unpasteurised milk makes ·
brucellosis a potential cause; it is an agent
11.18. Answer: E. whose diagnosis involves confirmation by a
Parvovirus B19 is associated with transient serological test such as a serum agglutination
aplastic crisis in patients with test. The regime in option A would be
haemoglobinopathy or haemolytic anaemia. appropriate for this infection; the regimens in
Dengue and CMV cause anaemia but typically options B, C and D might be considered for
not aplastic crisis, while HTLV-1 tends to cause bone infection due to Staphylococcus aureus,
acute T-cell leukaernia/lyrnphorna. Candida spp. or in melioidosis, respectively;
streptomycin is usually used in infections like
11.19. Answer: E. plague and tularaemia where bone infection is
Streptococcal infections are usually sensitive to less likely.
penicillin, which is the treatment of choice in
pharyngitis. Although clindamycin and 11.23. Answer: C.
erythromycin are alternatives and may be used The individual has a low CD4 T-cell count and is
in penicillin allergy, some strains show at risk of meningitis caused by an opportunistic
resistance. A work-up for immunodeficiency is infection. Cryptococcal meningitis is a leading
not indicated since there have been no prior consideration and should be excluded. The test
features of infection; however, since several of choice is a cryptococcal antigen test on the
family members appear to have developed CSF, which is highly sensitive and specific. PCR
pharyngitis, it would be appropriate to screen is not routinely performed and detection oft
all family members at the same time, since blood antibody against cryptococci is not ~·sed
streptococci can be nasopharyngeal in the diagnosis of cryptococcal meningiti$.
commensals and can be passed from one ~-o-glucan and galactomannan assays a,fe used
family member to another. Aspirin may be used in the diagnosis of Aspergillus spp. and in the
for rheumatic fever but there is no indication case of ~-o-glucan some other fungi. Although
there are signs of rheumatic fever. cryptococcal infection can be detected at low
levels with the ~-o-glucan, this test is not
11.20. Answer: C. routinely used to diagnose cryptococcal
The scenario is suggestive of necrotising infection.
fasciitis, which is a medical emergency. In this
case there are signs suggestive of shock, and 11.24. Answer: C.
urgent surgical intervention and debridement of Faecal transplantation has emerged as a
involved muscles is essential. Imaging potential therapy for relapsing C. difficile
modalities will only delay definitive treatment infection in those without other
and surgical inspection will provide the most contraindications. Although fidaxomicin
prompt diagnosis and management. decreases the risk of relapse compared to
vancomycin, it is not used long term.
11.21. Answer: B. Metronidazole is used in short courses and
Although there are several causes of deranged vancomycin is usuall)yused in prolonged
liver function tests with eosinophilia, including tapering courses to;~revent relapse, although, if
drug-related causes, the scenario in this case there is no other option, it might be considered
and the obstructive features make infection for an indefinite period. Glucocorticoids may be
with a fluke that can infect the bile duct a used in severe disoose but are not used to
consideration. Of the causes listed, only manage relapse and ciprofloxacin has been
Fasciola hepatica is a fluke that infects the bile implicated in the development rather than the
duct and it is associated with eating plants that prevention of C. difficile infection.

downloaded from www.medicalbr.com


INFECTIOUS DISEASE • 89

11.25. Answer: D. addition to ~-lactam antibiotics, clindamycin is


An incomplete vaccination history or waning usually added to decrease toxin production.
immunity can put young adults at risk of There is a low likelihood of CA-MRSA, so this
mumps, which can cause an aseptic or probably does not need to be added initially,
lymphocytic meningitis. Although multiple other but the decision on this would be governed by
viruses can cause aseptic meningitis, mumps is the risk of MRSA. The other antimicrobials
a potential consideration, especially when would be added in other settings, although
common causes such as enteroviruses, doxycycline is an alternative agent that may
HSV, HIV and geographically restricted also decrease protein synthesis.
mosquito-mediated viral infections are not
present and when there is no history of TB 11.29. Answer: A.
exposure, chronic medication use or medical The epidemiological setting and the appearance
comorbidity. of a severe illness with a groin swelling
suggestive of a bubo make plague a likely
11.26. Answer: B. diagnosis. The causative agent Yersinia pestis
When vaccination has not been performed or gives the appearance of safety pins or bipolar
may have been unsuccessful due to drift in the staining with Gram stain. Staph. aureus
circulating strain from the vaccine strain, appears as a cluster on Gram stain and would
prophylaxis is indicated for high-risk patients be in the differential of any abscess. Other
such as heart and lung transplant recipients. characteristic appearances on Gram staining
Prophylaxis uses the neuraminidase inhibitors include Corynebacterium diphtheriae, which
oseltamivir or zanamivir rather than the M2 appear as rods at acute angles suggestive of
proton channel inhibitors amantadine and Chinese letters. A drumstick appearance due to
rimantidine, which were used in the past. the presence of a terminal spore is seen with
Ribavirin and tenofovir are not used in the Clostridium tetani. Candida albicans can appear
prophylaxis of influenza. as a filamentous form suggestive of a mould,
but this, in fact, is a pseudomycelium formed
11.27. Answer: E. by the yeast.
The yellow fever vaccine is a live vaccine that I
can cause viscerotropic disease in those with 11.30. Answer: B. /
immunosuppression. The decision to give the A high bacterial index is associated with
11
vaccine is based on assessing the relative lepromatous leprosy. The presence of jl
balance of risks and benefits. In this case, widespread lesions and a 'glove and stocking'
although both HIV and organ transplantation distribution sensory disturbance both sugge~t
are associated with immunosuppression, the lepromatous leprosy. The other findings of/ ·
relative risk is much greater in the scenario with clearly demarcated hypopigrnented lesions with
liver transplantation only 18 months previously, early loss of sensation and sweating are all
due to its associated more significant degree of more in keeping with tuberculoid leprosy, where
immunosuppression due to tacrolimus (a a lower bacterial index would be expected.
calcineurin inhibitor) and mycophenolate. In
contrast, controlled HIV with a normal CD4 11.31. Answer: A.
count is not an absolute contraindication and The likely diagnosis is typhoid fever.
the vaccine may be considered if it is essential. Antimicrobial resistance is increasing, meaning
The other conditions are not associated with many agents formerly used are no longer active
significantly increased risk of side-effects from such as chloramphenicol, amoxicillin and
this vaccine, although they may influence other co-trimoxazole. Ciprofloxacin resistance has
medical considerations. For example also emerged as major problem, particularly in
mefloquine malaria prophylaxis would not be Asia, so ceftriaxone is the most reasonable
I
used with a seizure history and the· initial choice. /
antimicrobial linezolid should be avoided for
those on an SSRI. 11.32. Answer: D.
Schistosoma haematobiym is associated with
11.28. Answer: C. squamous cell bladder carcinoma, which is a
The scenario is suggestive of streptococcal or less common histological type of bladder
staphylococcal toxic shock syndrome. In carcinoma in the developed world. BCG is

downloaded from www.medicalbr.com


90 • INFECTIOUS DISEASE

sometimes used in the treatment of bladder films may be positive, they are less likely to be
carcinoma and Salmonella Typhi carriage is positive than in T. brucei rhodesiense infection,
associated with Schistosoma mansoni infection except early on in the infection. Since this
in particular but not bladder carcinoma. infection has likely been present for some time,
Enterohaemorrhagic E. coli is associated with aspiration of the lymph nodes is more likely to
haemolytic uraemic syndrome but not bladder make a diagnosis. In addition, nothing has
carcinoma, and syphilis, caused by Treponema been noted on blood films sent for malaria
pallidum, is not associated with bladder testing, which should also reveal trypanosomes.
carcinoma. Serologic responses against T. brucei and a
lumbar puncture should also be performed. A
11 .33. Answer: A. bone marrow aspirate and splenic aspiration
The story is most suggestive of lymphatic are tests employed in the diagnosis of
filariasis, which would best be diagnosed by leishmaniasis and a liver biopsy would not be
looking for microfilaria on a blood film or by helpful in this case to determine the cause of
serology. An elevation of lgE is often seen in the liver function test abnormalities. In addition,
lymphatic filariasis but is not of itself diagnostic. these tests need specialist facilities, particularly
A slit-lamp examination or skin snip is used to splenic aspiration.
diagnose onchocerciasis, while a protruding
worm would be more suggestive of 11.37. Answer: E.
dracunculiasis. Candidaemia has a propensity to lead to
intraocular infection. This has resulted in the
11 .34. Answer: C. recommendation that all patients with
The occupation as a veterinarian and, in candidaemia should be assessed by an
particular, the recent contact with animals that ophthalmologist with dilated fundoscopy.
have been giving birth, puts this veterinarian at Oesophagogastroduodenoscopy is used to
risk of Coxiella burnetii infection. Although the assess Candida oesophagitis and the other
other microorganisms listed can all cause investigations are indicated if signs or
pneumonic illness they have distinct symptoms suggest infection at these sites.
epidemiological settings. Chlarnydophila psittaci
is associated with exposure to sick birds such 11 .38. Answer: A. (
as parrots and Legionella pneurnophlia with Drug fever is an important consideration in
11
contaminated water-cooling towers and other patients with pyrexia of unknown origin,/;
water systems. Bacillus anthracis has been· particularly in those who are on multipl~1
associated with bioterrorism and Yersinia medications. Allopurinol is a potential cause of
pestis, the causative agent of plague, with drug-related hypersensitivity and is aq~ociated
hunters and others exposed to endemic plaque with abnormal liver function tests and
in rural settings. lymphadenopathy. Penicillins and other
antimicrobials can cause fever but in this case
11.35. Answer: D. the fever predated use of piperacillin-
Despite the precautions this patient took, tick tazobactam. The failure to respond to
bites can be hard to avoid. The appearance of broad-spectrum antimicrobial therapy and the
eschars with no findings on blood film are absence of any localising features or positive
suggestive of African tick bite fever, and the microbiological tests mean that a
features of multiple eschars without rash are microbiological cause remains unproven and
more suggestive of Rickettsia africae than of other possibilities need to be excluded. Herpes
Rickettsia conorii. Borrelia duttonii is also virus aetiologies are important, particularly in
transmitted by ticks but results in relapsing solid organ transpl9-nt recipients or those who
fever. The other infections are less likely in this have received a haematopoietic stem cell
scenario as they do not link to the history of transplant, but these are not found in this case
tick bites, explain the eschars and would be and a patient with acute leukaemia on
expected to provide other laboratory findings. chemotherapy is not at particular risk of these
infections. lnvasive.fungal infection remains a
11 .36. Answer: D. diagnostic considera\ion, particularly for
The patient is likely to have Trypanosoma patients with acute myelogenous leukaemia,
brucei garnbiense infection. Although blood but the fever has remained despite addition of

downloaded from www.medicalbr.com


INFECTIOUS DISEASE • 91

caspofungin, and the galactomannan and chest the possibility of haematogenous spread of
CT scan has been negative. septic emboli from infected thrombophlebitis or
right-sided endocarditis. In an area with low
11.39. Answer: A. rates of MRSA, flucloxacillin is a good choice
Eosinophilic meningitis is seen with for empiric coverage of a potential
Angiostrongy/us spp. infections and also with endovascular Staph. aureus infection.
gnathostomiasis or coccidioidomycosis. The Meropenem and piperacillin-tazobactam might
other infections listed may be found in be considered in cases of sepsis but would
South-east Asia but cause alternative clinical not be first choice where Staph. aureus needs
neurological syndromes. to be treated. Tigecycline might be used
against MRSA in certain settings, such as skin
11.40. Answer: C. and soft tissue infection, but would not be first
The epidemiological setting, clinical scenario choice when potential bloodstream infection
and use of xenodiagnoses are consistent with a needs to be treated. Moxifloxacin is not usually
diagnosis of Chagas' disease (American used in Staph. aureus infection.

--
trypanosomiasis), which is treated with
nifurtimox. Niclosamide and nitazoxanide are 11.44. Answer: D.
used to treat other parasites. Radical cure of vivax malaria requires the use of
the 8-aminoquinoline drug primaquine. This
1
11.41. Answer: D. causes oxidative stress, which can result in
The likeliest diagnosis is acute Lyme borreliosis. massive haemolysis in patients who have low
This is increasing in frequency in the UK. The G6PD activity due to various inherited traits.
commonest organism responsible in Europe The other tests will not impact on the
is Borrelia burgdorferi. The recommendation treatment.
is for family physicians to treat with oral
doxycycline or amoxicillin for uncomplicated 11.45. Answer: B.
acute disease. Cysticercosis is caused by the pork tapeworm
Taenia solium and results when humans ingest
11.42. Answer: D. tapeworm ova, often from an infected
Any patient who has a history of recent travel household contact. The disease leads to cysts
to the Middle East along with fever and severe that can involve the subcutaneous tissue,
respiratory symptoms should have infection muscle and brain. The lesions are visible on CT
with MERS-CoV excluded before infection or MRI scan of the head and can have a
control measures can be relaxed. SARS is characteristic appearance. They can result in a
another coronavirus infection that leads to variety of neurological features, including ,
severe respiratory symptoms but circulated in new-onset seizures. Treatment is most often
2003. There are no risk factors for avian with albendazole.
influenza, which would require a history of
contact with chickens. ARDS complicating 11.46. Answer: D.
pneumonia would be in the differential but does Scabies with increasing drug resistance is a
not have the same influence on infection huge problem in indigenous populations in
control policy and therefore is not the first Australia. This is associated with
diagnosis to be excluded. Meningococcal post -streptococcal rheumatic fever with
sepsis has been reported after pilgrimages to rheumatic heart disease as a sequela.
the Middle East and sepsis can present with lverrnectin is used for large infestations. The
respiratory symptoms but would be expected other answers would not explain both the skin
to present with additional signs of sepsis. lesions and the heart murmur.

I
11.43. Answer: A. 11.47. Answer: A. /
The empiric therapy of fever in an intravenous Salmonellosis can invade and colonise aortic
drug user should include coverage of Staph. arteriosclerotic plaques and result in a mycotic
aureus. The specific agents will be influenced aortic aneurysm in older.patients. Endocarditis
by local antimicrobial resistance patterns and is uncommon with salmonellae.. Persistently
rates of meticillin-resistant Staph. aureus positive blood cultures raise the possibility of
(MRSA). In this scenario the chest X-ray raises endovascular infection and while this is most

downloaded from www.medicalbr.com


92 • INFECTIOUS DISEASE

often associated with a central venous catheter


infection or endocarditis, certain organisms are
lesions on the palate usually indicates that
there is deep-organ involvement, such as gut
T
associated with other foci and require specific and lung, which may lead to life-threatening
investigations, as in this case. bleeding. Treatment is with doxorubicin and
immune reconstitution.
11.48. Answer: E.
This is a typical presentation of enteric fever 11.53. Answer: D.
caused by Salmonella Typhi. Nepal is a Lemierre's syndrome is due to suppurative
high-risk country for typhoid, which is jugular vein. thrombophlebitis with bacteraemia
contracted from contaminated water. She has usually secondary to a bacterial sore throat.
signs of sepsis, which would make Treatment is usually supportive with penicillin.
cyclosporiasis less likely, and the rash would It is often complicated by disseminated
not be typical for appendicitis. The incubation intravascular coagulopathy.
period is too long for dengue. Although scrub
typhus causes a rash, the history with 11.54. Answer: D.
prominent abdominal symptoms and absence Nocardia are filamentous Gram-positive
of an eschar and regional lymphadenopathy is branching rods that stain positive with a
less typical. modified acid-fast stain. They are found in
the soil and may lead to brain abscesses.
11.49. Answer: C. Actinomyces israelii and Clostridium perfringens
Campylobacter is the commonest cause of are also Gram-positive, but C. perfringens
infective colitis in the UK. Although inflammatory would not appear as branching rods and
bowel disease should be considered, it is less although A. israe/ii would have a similar
common than Campylobacter. microbiological appearance, Nocardia would be
more likely to cause a brain abscess in this
11.50. Answer: C. scenario. Mycobacterium chelonae would stain
Mucocutaneous leishmaniasis due to with an acid-fast stain and Sporothrix is a
L. braziliensis should always be treated
aggressively with systemic therapy due to the
risk of dissemination. The other treatments
form. I
dimorphic fungus and would appear as a yeast

1
would not prevent dissemination. 11.55. Answer: D. /'
1
Plasmodium knowlesi is the sixth human(
11.51. Answer: A. malaria now that Plasmodium ovate has 'been
Bacillary angiomatosis is caused by infection classified as two subspecies on the basis of
with Bartonella species, a slow-growing genetic homology. P. know/esi is asso~iated
Gram-negative bacillus that causes problems in with close contact with non-human primates
immunocompromised hosts. These include and is usually a mild infection, which does not
endocarditis, trench fever and bacillary peliosis relapse. Its life cycle is 24 hours (as opposed to
(widespread blood-filled cavities in major 48 hours for fa/ciparum), giving rapid changes
organs). Treatment is with doxycycline. in fever. Chesson strain vivax is relatively
Kaposi's sarcoma also can cause violaceous resistant to primaquine and is found in
papules but histology should show Indonesia.
characteristic spindle cell formations and would
not be positive with Warthin-Starry silver stain, 11.56. Answer: A.
which detects the Bartonella species. Similarly, Haemolytic uraemic syndrome is usually
the histology does not show features of associated with infection with E. coli 0157.
melanoma and this also would not have a Treatment is supportive. The other options
positive Warthin-Starry stain; nor would the would not give this combination of anaemia,
other conditions listed, which also would have jaundice and renal failure.
alternative dermatological appearances.
11.57. Answer: E.
11.52. Answer: B. Yersinia enteroco/itica is commonly found in
Kaposi's sarcoma is an angioproliferative pork, causes mild to moderate gastroenteritis
tumour related to HHV-8 infection in and can produce significant mesenteric adenitis
irnrnunocompromised hosts. The finding of after an incubation period of 3-7 days. It

downloaded from www.medicalbr.com


INFECTIOUS DISEASE • 93

resolves slowly. The other options would not 11.62. Answer: E.


give this clinical picture. Wound botulism results from the contamination
of a wound with the bacterial species
11.58. Answer: E. Clostridium botulinum, which then secretes
Hantavirus is a ribonucleic acid (RNA) virus botulinum toxin into the bloodstream. It has
associated with pulmonary and renal failure become increasingly common in people who
mimicking pulmonary syndrome in leptospirosis, take drugs intradermally or subcutaneously
and is associated with a history of contact with (skin popping). Botulism is typically described
mice within a wide geographical area. Some as a 'triad of bulbar palsy and descending
strains of hantaviruses cause the potentially paralysis, lack of fever, and clear senses'.
fatal diseases - hantavirus haemorrhagic fever Staphylococcal brain abscesses and tetanus
with renal syndrome and hantavirus pulmonary are also associated with drug injection but the
syndrome - while others have not been patient's neurological symptoms with prominent
associated with known human disease. bulbar signs and ocular involvement make
Treatment is supportive. The other syndromes these less likely. Staphylococcal brain abscess

--
are non-infectious and while all enter the would be more associated with injection into
differential, the scenario means one should the vein than skin popping. The other
consider hantavirus. conditions may cause ocular signs but are not
particularly associated with drug use.
1
11.59. Answer: D.
Bat bites have been associated with 11.63. Answer: A.
transmission of rabies and individuals who are Anthrax amongst drug users is related to heroin
bitten or likely to be exposed to bats should contaminated with anthrax spores. Urgent
have a rabies vaccine. The other infections are surgical debridement (to remove dead or
not associated with bats. devitalised tissue and drain any abscess/
collection) is most important. This should be
11.60. Answer: C. performed alongside empiric antibiotic
Monkeypox is a relatively harmless infection treatment to cover Bacillus anthracis as well as
with a poxvirus. Although similar in appearance other more common causes of soft tissue
to smallpox, it is a mild infection. The most infe.ction. Antibiotic treatment may involve
recent outbreak in the USA was related to ciprofloxacin and clindamycin intravenously in
importation of pet rats from The Gambia in combination with penicillin, flucloxacillin and
Africa. metronidazole (i.e. a five-drug combination).
Gas gangrene is not particularly associated \('iith
11.61. Answer: C. an eschar and is more associated with dusii'Y'
The cardinal feature of necrotising fasciitis is skin discoloration and crepitus. Neither Lyme
pain out of keeping with clinical signs. The disease not staphylococcal bacteraemia are
diagnosis is by surgical exploration with associated with an eschar, and necrotising
necrotic deep tissue being seen. It is usually fasciitis would typically be associated with more
due to a polymicrobial infection and treatment pain at the site and other skin features.
is with broad-spectrum antibiotics including a
macrolide to reduce toxin production plus 11.64. Answer: D.
surgical debridement. Gas gangrene is Some adults may not have been immunised
associated with wounds, and crepitus may be with MMR. The combination of neck swelling
detected in the skin. Splenic rupture would be due to parotitis and orchitis is highly suggestive
associated with trauma and there is no history of mumps. The other causes listed might
of this. Lemierre's syndrome presents as explain cervical swelling or the combination of
a pain in the neck, which results from a cervical and testicular prob~ms but are less
thrombophlebitis, typically of. the internal jugular consistent from the relatiyely healthy nature of
vein, complicating a sore throat. A colonic this patient and the short history of symptoms.
perforation would be considered with the
location of the abdomen pain but would not be 11.65. Answer: B.
expected to cause the skin lesion visible over Gas gangrene is due to infectiw with C.
the hip, which is a key feature that raises the perfringens. Treatment involves surgical
possibility of necrotising fasciitis. debridement and penicillin with clindamycin.

downloaded from www.medicalbr.com


r
,,
'I
I
94 • INFECTIOUS DISEASE

Tetanus would present with an eschar, and the presents with ulcerated skin lesions followed
other species are either not associated with skin by bone and joint deformity caused by
lesions or have alternative skin presentations. Treponema pal/idum pertenue. Treponema!
serology cross-reacts and so is unreliable.
11.66. Answer: D. Pinta is found mainly in South America and
Scornbroid fish can produce histamines, which bejel is mainly in the Middle East and West
increase when bacteria rnetabolise histidine. Africa.
This rapidly causes symptoms. Uncooked tuna
steaks are a common cause. Ciguatera 11.71. Answer: B.
poisoning presents with prominent abdominal Listeria should be considered in
symptoms and a range of neurological immunocompromised hosts as well as in
symptoms, and the neurological symptoms pregnancy and in people over 55 years of age.
may be prolonged. The dietary history, CSF is usually lymphocytic. Treatment is with
exposure history and lack of paralysis make the ampicillin as it is inherently resistant to
various other options unlikely. cephalosporins. TB is also a consideration,
especially if the patient has had anti-tumour
11.67. Answer: D. necrosis factor (TNF) therapy, but the short
Granulomatous cerebral angiitis is a rare history is against this. Cryptococcal disease is
complication of shingles, especially when in the often associated with lower cell numbers and
ophthalmic branch of the trigeminal nerve. It is would be associated with detection of
often fatal and only diagnosed post-mortem. cryptococcal antigen or identification of yeast
Aciclovir-induced encephalopathy is unusual on Indian ink testing. The other causes are also
with modern formulations and in the absence of possible but less likely in this scenario and
renal impairment. The combination of confusion would more usually produce neutrophils in
with more focal neurology makes the other the CSF.
possibilities unlikely.
11.72. Answer: B.
11.68. Answer: D. Francisella tularensis is a zoonotic infection
HTLV-1 is found in Japan, the Caribbean and associated with contact with wild rodents(
Central and South America and can cause including rabbits. It is spread by ticks in Jhe
T-cell lymphoproliferation. It may be wild and treatment is with aminoglycosi~es or
smouldering or aggressive but should be ciprofloxacin or doxycycline. The other ,~ptions
suspected in patients with T-cell are not associated with wild rodent contact
lymphoproliferation, particularly when from an leading to skin ulceration. I'
I
area where the virus is found. It is also I

associated with hypercalcaemia. The other 11.73. Answer C.


viruses are not specifically associated with Enterococcal infection may occur in the urinary
T-celllymphoproliferative disorders, although tract and in the biliary tract, which is particularly
HIV should always be excluded in the presence relevant in someone with a history suggestive
of lymphoproliferation. HIV-2, however, is not of gallstone disease. Thus imaging these sites
associated with the Caribbean and is mainly would be reasonable as part of the
found in West Africa. investigation. However, the presence of multiple
positive blood cultures and the rapid time to
11.69. Answer: C. positivity suggests an endovascular focus;
There are a variety of influenza strains echocardiography is essential in this case and
associated with avian influenza but of those the test most likely to establish the source as
listed only H5NI is an avian strain and has infective endocarditis. This man has a prior
caused outbreaks in the region around Hong history of a regurgitajlt murmur, which may or
Kong, while H7N9 has been implicated in cases may not be relatecY'I:o the current presentation
around Shanghai. HI Nl is a human strain and but this also increases his risk of having
the other viruses are not influenza A viruses. enterococcal endocarditis.

11.70. Answer: E. 11.74. Answer: A.


Non-venereal treponema! infections are Co-trimoxazole plus imipenem would, work well
endemic in many tropical countries. Yaws for the treatment of Nocardia infection, but

downloaded from www.medicalbr.com


INFECTIOUS DISEASE • 95

treatment should also involve surgical 11.77. Answer: A.


drainage. The other options would be more Babesiosis is caused by a tick-borne
appropriate for other infections: for example, intra-erythrocytic protozoon parasite and is
options C and D list agents used in atypical found throughout the world. It can lead to
mycobacterial infection and in tuberculosis, these clinical signs and, in the context of the
respectively. Several options contain agents tick bite, is more likely than the alternatives.
with limited brain penetration such as Treatment is with quinine and clindamycin.
vancomycin or gentamicin, which make
them less ideal or necessitate intrathecal 11.78. Answer: B.
dosing. Benznidazole and nifurtimox are for American
trypanosomiasis while stibogluconate and
11.75. Answer: C. miltefosine are for leishmaniasis. Melarsoprol is
Clostridial necrotising enteritis or pigbel is an the treatment of choice.
often-fatal type of food poisoning caused by a
~-toxin of C. perfringens, type C. The toxin is 11.79. Answer: A.
normally inactivated by certain proteases or by Azithromycin is the treatment of choice for
normal cooking. Pigbel is more likely in protein
malnutrition or in the presence of trypsin
inhibitors, either in foods such as sweet
Campylobacter (which is the most likely
organism) due to increasing quinolone
resistance. It is preferred to erythromycin due
a.
potatoes or during infection with Ascaris sp. to fewer side-effects. I
roundworms.
11.80. Answer: A.
11. 76. Answer: D. Herpes B is an endemic DNA virus of macaque
No antimicrobial indicated; there is monkeys, which can rarely cause encephalitis
some evidence that antibiotics may prolong in humans exposed through bites. Treatment
illness with E. coli 0157. Treatment is with aciclovir is recommended as soon as
supportive. possible and may be life-saving.

I
!'

downloaded from www.medicalbr.com


G Maartens

HIV infection and AIDS


Multiple Choice Questions
12.1. What is human immunodeficiency virus A. HIV antibodies detected by enzyme-linked
(HIV) protease enzyme responsible for? immunosorbent assay
A. Budding of HIV from the cell B. HIV antibodies detected by western blot
B. Cleavage of post-translational regulatory C. p24 antigen detection
proteins D. Polymerase chain reaction (PCR) to detect
C. Fusion of HIV with the host cell surface .HIV RNA
D. Integration of viral deoxyribonucleic acid E. Rapid point -of-care antibody test
(DNA) into the host genome
E. Reverse transcription of viral ribonucleic acid 12.5. A 35 year old HIV-positive man presents
(RNA) to DNA with diarrhoea of 4 weeks' duration
accompanied by tenesmus. Blood and mucus
12.2. What is the risk of acquiring HIV during a is present in the stool. He is not on antiretro/iral
single act of unprotected vaginal sexual therapy. His CD4 count is 17 cells/mm 3 • WH,ich
intercourse when the male partner is HIV of the following is the most likely diagnosis?
l/
infected and not on antiretroviral therapy, and A. Cryptosporidiosis
the female partner is HIV uninfected? B. Cystoisosporiasis
A. 0.001% C. Cytomegalovirus (CMV) colitis
B. 0.01% D. Giardiasis
c. 0.1% E. Microsporidiosis
D.1%
E. 10% 12.6. Which one of the following non-acquired
immunodeficiency syndrome (non-AIDS)
12.3. Which of the following is a correct cancers has been shown to have an increased
statement regarding the features of primary HIV incidence in HIV-infected patients?
infection? A. Anal cancer
A. A maculopapular rash is a common B. Breast cancer
feature C. Melanoma
B. Atypical lymphocytosis occurs more D. Ovarian cancer
frequently than in Epstein-Barr virus (EBV) E. Prostate cancer
infection
C. Primary infection is asymptomatic in most 12.7. A 45 year old man with a CD4 count
people of 23 cells/mm 3 presents with gradually
D. The incubation period is 5-7 days progressive spastic paraplegia and urinary
E. Transient CD4 lymphocytosis is usual incontinence. There i~ impaired short-term
memory. Plain X-ray of the spine is normal.
12.4. What is the most sensitive blood test for What is the most likely cause of the
diagnosing HIV during primary infection? paraplegia?

downloaded from www.medicalbr.com


HIV INFECTION AND AIDS • 97

A. Cytomegalovirus polyradiculopathy
B. Human T-cell lymphotropic virus type 1
(HTLV-1) myelopathy
C. Multiple sclerosis
D. Tuberculosis of the spine
E. Vacuolar myelopathy

12.8. An HIV-infected woman with a C04 count


of 36cells/mm 3 presents with symptomatic
anaemia. She is not on antiretroviral therapy.
Full blood count shows: haemoglobin 26 g/L,
normochromic and normocytic, with a low
reticulocyte index; white cell count and platelets
are normal. Bone marrow biopsy shows pure
red cell aplasia. Which one of the following viral
infections is likely to be responsible?
A. Cytomegalovirus
B. Epstein-Barr virus
C. Herpes simplex virus Fig. 12.11
D. JC virus (John Cunningham virus)
E. Parvovirus B19
saturation is 90% on room air. White blood cell
12.9. Which of the following is a clinical count and haemoglobin are normal. What is the
indication for starting prophylactic most likely diagnosis?
co-trimoxazole in HIV-infected adults in A. Bacterial pneumonia
middle- to high-income settings? B. Lymphoid interstitial pneumonitis
A. Enlarged parotid glands C. Pneumocystis jirovecii pneumonia
B. Fungal nail infections D. Pulmonary Kaposi's sarcoma
C. Generalised lymphadenopathy E. Pulmonary tuberculosis
D. Herpes zoster infection
E. Oral hairy leucoplakia 12.12. Which of the following statements is true
about the diffuse inflammatory lymphocytosis
12.10. A 24 year old HIV-infected man presents syndrome (OILS)?
with a 5-day history of fever, cough and right A. OILS is a recognised complication of
pleuritic chest pain. There are crackles and co-infection with human herpesvirus 8
signs of consolidation in the right lung B. OILS is a risk factor for parotid gland
posteriorly and chest radiograph shows dense lymphoma
consolidation of the right lower zone. What is C. OILS is associated with polymyositis
the most likely pathogen? D. OILS is characterised by infiltration of the
A. Mycobacterium tuberculosis parotids with B lymphocytes
B. Pneumocystis jirovecii E. OILS is usually associated with a marked
C. Pseudomonas aeruginosa C04 lymphocytosis
D. Streptococcus pneumoniae
E. The 'atypical bacteria' (e.g. Mycoplasma 12.13. A nurse administers an intramuscular
pneumoniae, Chlamydophila pneumoniae, injection to an HIV-infected woman who is not
and Legionella species) yet on antiretroviral therapy. After the injection
she pricks her finger on the needle. What is the
12.11. A 34 year old man with a C04 count of approximate risk of acqujing HIV following this
116 cells/mm 3 presents with a •4-week history of occupational exposure?·
dry cough and progressively worsening A. 0.003%
dyspnoea. On examination he has a fever of B. 0.03%
38°C, respiratory rate of 26breaths/min, and no c. 0.3%
focal chest signs. Chest radiograph shows a D. 3%
bilateral interstitial infiltrate (Fig. 12.11 ). Oxygen E. 30%

downloaded from www.medicalbr.com


98 • HIV INFECTION AND AIDS

12.14. A 37 year old man with a CD4 count of prescribe a course of fluconazole for possible
24cells/mm 3 presents with painless, Candida oesophagitis. Two weeks later she
progressive visual loss. On fundoscopy the returns with no improvement. What is the most
vitreous is clear, and haemorrhages and likely cause of her dysphagia?
exudates are seen on the retina. What is the A. Cytomegalovirus oesophageal ulceration
most likely diagnosis? B. Herpes simplex virus oesophageal ulceration
A. Cytomegalovirus retinitis C. Kaposi's sarcoma of the oesophagus
B. HIV retinopathy D. Major aphthous ulceration of the ·oesophagus
C. Ocular syphilis E. Oesophagitis to azole-resistant Candida
D. Ocular toxoplasmosis species (e.g. C. kruse!)
E. Progressive outer retinal necrosis due to
varicella zoster virus 12.19. A 39 year old man presents with
asymmetric cervical lymphadenitis for 2
12.15. What is the mechanism of action of the months. His CD4 count is 234cells/mm 3 .
antiretroviral drugs raltegravir, dolutegravir and The largest node is 4x3 em and is fluctuant.
elvitegravir? Several nodes are matted together. What is the
A. Chemokine receptor CCR5 antagonist most likely diagnosis?
B. Fusion inhibitor A. HIV lymphadenopathy
C. lntegrase inhibitor B. Kaposi's sarcoma
D. Protease inhibitor C. Non-Hodgkin lymphoma
E. Reverse transcriptase inhibitor D. Pyogenic lymphadenitis
E. Tuberculosis
12.16. A 44 year old woman with a CD4 count
of 73cells/mm3 presents with a progressive left 12.20. Which of the following statements is
hemiplegia and headache over a week. Her correct about AIDS-associated Kaposi's
magnetic resonance imaging scan shows sarcoma?
multiple ring-enhancing mass lesions with A. It is a spindle-cell tumour of
surrounding cerebral oedema. What is the most lymphoendothelial origin /
likely diagnosis? B. It is associated with infection by hum:tn
A. Brain abscess herpesvirus 6 /
B. Cerebral toxoplasmosis C. Multiple skin lesions indicate a poor;/
C. Cryptococcoma prognosis
D. Primary central nervous system (CNS) D. The commonest site of visceral spread is the
lymphoma brain /'
I
E. Tuberculoma E. Women are more likely than men to develop
Kaposi's sarcoma
12.17. What is the correct statement regarding
the immune reconstitution inflammatory 12.21. Which of the following statements on viral
syndrome (IRIS)? load in HIV infection is correct?
A. Antiretroviral therapy (ART) should be A. A viral load change of 15 848 to
stopped if IRIS is suspected 10 000 copies/ml (difference of 0.21og 10) is
B. It is more common in patients responding regarded as a significant reduction 4 weeks
poorly to ART after starting antiretroviral therapy
C. It is more common when ART is initiated B. The viral load should be suppressed after 6
with higher baseline CD4 counts months of effective antiretroviral therapy
(> 200 cells/mm 3) C. Vaccination transiently decreases the viral
D. It usually presents within the first 3 months load
of initiating ART D. Viral load meas)Jres intracellular viruses
E. The mortality is high (approximately 25%) E. Viral load reaches a relatively stable plateau
2 weeks after seroconversion
12.18. A 26 year old woman with newly
diagnosed HIV infection and a CD4+ 12.22. A 42 year old man presents with severe
lymphocyte count of 34cells/mm 3 presents with headache and vomiting of 3 weeks' duration.
dysphagia. There is no oral candidiasis. You His CD4 count is 62cells/mm 3 . Computed

downloaded from www.medicalbr.com


HIV INFECTION AND AIDS • 99

tomography (CT) scan of the brain is normal. 12.23. Which of the following features is
Lumbar puncture shows mild pleocytosis with characteristic of HIV-associated nephropathy
positive cryptococcal antigen test and elevated (HIVAN)?
opening pressure of 34cmH 2 0. You commence A. Heavy proteinuria (> 1 .5 g/24 hrs) is a usual
therapy with intravenous amphotericin B and finding
flucytosine for the cryptococcal meningitis. B. People of European descent are more likely
What is the most appropriate management for to develop HIVAN
the raised intracranial pressure? C. Severe hypertension is a characteristic
A. Acetazolamide feature
B. Dexamethasone D. Small kidneys on ultrasound are typically
C. Insert a ventriculo-peritoneal shunt seen when the creatinine clearance
D. Mannitol decreases to 30 mUmin or less
E. Therapeutic lumbar puncture, removing E. The course of the disease is relatively benign
enough cerebrospinal fluid to reduce with few progressing to end-stage renal
pressure to < 20 cmH 20 failure

Answers
12.1. Answer: B. Budding occurs after cleavage of proteins by
This is the main function of protease protease.
(Fig. 12.1 ). Fusion is mediated after binding to
CD4 and the chemokine receptor, reverse 12.2. Answer: C.
transcriptase mediates reverse transcription, Several factors increase this risk: sexually
and integrase mediates integration of viral DNA. transmitted infection (especially genital ulcers),

Attachment
to CD4 receptor

Fusion


inhibitors

Reverse
transcription
of viral RNA
genome
r
......0

. - ~'" 1\1\
Genom1c J\J v v -
Reverse
transcriptase
inhibitors

Dou. ble-stranded
DNA
Chemokine M.:<~r~ntm
(CCR5 or

t
Chemokine
receptor
antagonist

Protease
inhibitors
RNA ~-·~JM~Ji\. 1 Viral mRNA
Reverse
transcription
I
Cell nucleus

=-><#
~4-$,~
____
Transcription
Proviral DNA --"~~~Sffi

Fig. 12.1 Life cycle of HIV. Arrows indicate sites of action of antiretroviral drugs.

downloaded from www.medicalbr.com


1 00 • HIV INFECTION AND AIDS

Acute HIV syndrome


Wide dissemination of virus Opportunistic
T
Seeding of lymphoid organs diseases

Prima~ I
;nrefFlrl _____ Clinical
la_t..Je,...
Constitutional
n_cY_ _ _s_y_m.,ptoms

'f 1200
I
· E 1100
]11 000 <
10 6 ~
~ 900 )>
+-'
c 800 ()
::J
10s.g
u 700
0
Q)
600
gf
>. -o
u 500 104 !!l
0
..c 400 3
Q_
r
E 300 -o
_;::,
1o3 PI
1- 200
+ 3
'<t 100 Ill
0
0 0 L-~~~~~~7~~~~~~~~=-~~~~~10 2
Weeks Years
Fig. 12.3 Virological and immunological progression of untreated HIV infection.

cervical ectopy, uncircumcised male partner papilloma virus infection; the other cancers
and menstruation. listed are not caused by viruses.

12.3. Answer: A. 12.7. Answer: E.


Primary HIV is a glandular fever-type illness with This is a typical presentation of this HIV j
an incubation period of 2-4 weeks. It differs disorder, which is usually accompanied b i/
from EBV infection with less prominent atypical features of AIDS dementia. Tuberculosis rould
lymphocytosis and a rash is common (with EBV, usually cause abnormalities on spine X-rJys.
/.
rashes usually only occur when aminopenicillins CMV polyradiculopathy causes lower motor
are given). Typically CD4 lymphocytes are neuron signs and pain. Myelopathy from
transiently decreased (Fig. 12.3). HTLV-1 and multiple sclerosis are not;HIV
associated and do not cause memory loss.
12.4. Answer: D.
Nucleic acid amplification tests are the most 12.8. Answer: E.
sensitive, followed by p24 antigenaemia. This is the commonest infectious cause of red
Antibodies typically are detectable 2-6 weeks cell aplasia in HIV infection. The antiretroviral
later. drug lamivudine is another rare cause. CMV
and some other viruses may occasionally
12.5. Answer: C. cause pancytopenia, but not pure red cell
The other diseases all present with small-bowel aplasia.
diarrhoea. The presence of blood and mucus in
the stool together with tenesmus is typical of 12.9. Answer: E.
large-bowel diarrhoea. CMV involvement of the This is a World Health Organization (WHO)
gastrointestinal tract typically causes ulceration stage 3/Centers for Disease Control and
and occurs mainly in the oesophagus and 1
Prevention (CDC) B ~anifestation. The others
colon, but any part of the gastrointestinal tract are all WHO stage 2, which is not an indication
can be involved. to start co-trimoxazole. Note that in low-income
countries co-trimOXI;lZole is given to all,
12.6. Answer: A. irrespective of CD4 count or clinical stage, as it
HIV increases the incidence of virus-related is of benefit in high infectious disease~ burden
cancers. Anal cancer is linked to human settings (including areas with malaria risk).

downloaded from www.medicalbr.com


HIV INFECTION AND AIDS • 101

12.10. Answer: D. 12.15 Commonly used antiretroviral drugs


Pneumococcal pneumonia incidence is
Classes Drugs
markedly increased in HIV. Atypical bacteria
Nucleoside reverse Abacavir, emtricitabine,
can present in this way, but are less common
transcriptase inhibitors lamivudine, tenofovir,
causes. Pseudomonas pneumonia is rare. (NRTis) zidovudine*
Tuberculosis can present acutely, but is usually Non-nucleoside reverse Efavirenz*, rilpivirine
a more subacute illness and the chest transcriptase inhibitors (only if viral load
radiograph is seldom that of dense lobar (NNRTis) <100000)
consolidation with no other features. Protease inhibitors (Pis) Atazanavir, darunavir,
lopinavir*
lntegrase inhibitors Raltegravir, dolutegravir,
12.11. Answer: C. elvitegravir
The duration of symptoms is too long for
Chemokine receptor inhibitor Maraviroc
bacterial pneumonia. The prominent dyspnoea
*These drugs are no longer recommended as first-line
and chest radiograph appearance is typical of options in high-income countries due to their toxicity.
Pneumocystis jirovecii pneumonia. Lymphoid
interstitial pneumonitis can have a similar
radiographic appearance but is a more chronic
illness and fever is uncommon. Pulmonary
tuberculosis with adult respiratory distress
syndrome is possible, but this is a rare
complication in tuberculosis.

12.12. Answer: C.
OILS is a benign polyclonal CDS infiltration of
tissues, especially the parotids, and is
associated with human leucocyte antigen
(HLA)-DRB1. A number of autoimmune
disorders are seen in OILS, including
polymyositis.

12.13. Answer: C.
Several factors increase this risk: high viral load
in source patient, hollow-bore needle that was
in source patient's vessel, visible blood on the
needle.

12.14. Answer: A.
CMV retinitis is the commonest cause of visual Fig. 12.16 Cerebral toxoplasmosis. Multiple ring-enhancing
loss in AIDS. Toxoplasmosis often causes a lesions with surrounding oedema are characteristic.
concomitant vitritis and HIV retinopathy does
not cause visual loss. 12.16. Answer: B.
This is the commonest cause of
12.15. Answer: C. space-occupying lesions in patients with
The standard combination antiretroviral HIV infection- CD4 counts are usually
regimens are two nucleoside reverse < 100 cells/mm 3 and multiple lesions are typical
transcriptase inhibitors (NRTis) together with a (Fig. 12.16).
non-nucleoside reverse transcriptase inhibitor
(NNRTI), protease inhibitor (PI) or integrase 12.17. Answer: D.
inhibitor (Box 12.15). Dual NRTI cor-nbinations IRIS is an exaggerated intl£mmatory response
are usually emtricitabine or lamivudine (they seen in the first 3 months after starting ART. It
have the same mechanism of action and so are is most common in patients starting ART with
never combined) together with one of abacavir, low CD4 counts. It is us~jal to continue ART
tenofovir or zidovudine. See Fig. 12.1, above, and provide symptomatic relief for IRIS
for mechanisms of action •of the different manifestations; steroids may be useful with
antiretroviral drugs. life-threatening manifestations.

downloaded from www.medicalbr.com


I 02 • HIV INFECTION AND AIDS

12.18. Answer: D.
Candida oesophagitis is the commonest cause,
12.21. Answer: B.
The level of viraemia is measured by
T
but failure to respond in this case virtually rules quantitative PCR of HIV RNA, known as the
this out (azole-resistant Candida species are viral load. Determining the viral load is crucial
usually only seen in patients with prolonged for monitoring responses to antiretroviral
azole use). The next commonest cause is major therapy. People with high viral loads (e.g.
aphthous ulceration, which responds well to > I 00 000 copies/ml) experience more rapid
steroids and ART. declines in CD4 count, while those with low
viral loads(< IOOOcopies/mL) usually have slow
12.19. Answer: E. or even no decline in CD4 counts. Viraemia
Tuberculosis nodes are often matted and may peaks during primary infection and then drops
fluctuate. Malignant nodes rarely fluctuate, as the immune response develops, to reach a
unless there is central necrosis. Persistent plateau about 3 months later (see Fig. 12.3,
generalised lymphadenopathy of HIV is typically above).
symmetrical and does not fluctuate. Transient increases in viral load occur with
intercurrent infections and immunisations, so
12.20. Answer: A. the test should be done at least 2 weeks
Kaposi's sarcoma (KS) is a spindle-cell tumour afterwards. Viral loads are variable; only
of lymphoendothelial origin. All forms of KS are changes in viral load of more than
due to sexually transmitted human herpesvirus 0.51og 10 copies/ml are considered clinically
8, also known as KS-associated herpesvirus. significant.
In Africa, the male: female ratio of
AIDS-associated KS is much lower than is seen 12.22. Answer: E.
with endemic KS, but men are still more The raised intracranial pressure is best
affected than women, despite the fact that the managed by therapeutic lumbar puncture as
seroprevalence of human herpesvirus 8 is the this is a communicating hydrocephalus.
same in both sexes. Steroids and acetazolamide have been shown
AIDS-associated KS is always a multicentric to be harmful. Shunting is seldom necessary.
disease. Early mucocutaneous lesions are Mannitol is irrational, as the primary proble/r.h is
macular and may be difficult to diagnose. As not cerebral oedema. ,"
'!
the disease progresses, the skin lesions
l
become more numerous and larger. 12.23. Answer: A. 11

Lymphoedema is common, as lymphatic HIVAN typically presents with nephrotic '


vessels are infiltrated. KS also commonly syndrome and/or chronic kidney disease (CKD).
spreads to lymph nodes and viscerally, Progression to CKD is rapid with pres¢~ed
especially to the lungs and gastrointestinal kidney size. People of African descent are more
tract. likely to develop HIVAN.

downloaded from www.medicalbr.com


GR Scott

Sexually transmitted
infections
Multiple Choice Questions
13.1. Most women with genital Chlamydia C. Gonorrhoea
trachomatis are symptomless. In women who D. Lymphogranuloma venereum
do develop symptoms, which of the following is E. Secondary syphilis
the most common?
A. Deep dyspareunia 13.4. Most of the following are complications of
B. Dysuria disseminated gonococcal infection (DGI). Which
C. Increased vaginal discharge would you be UNLIKELY to see as a
D. Lower abdominal pain recognised complication of DGI?
E. Unexpected vaginal bleeding A. Arthritis
B. Endocarditis
13.2. A patient with confirmed infection with C. Pustular rash
herpes simplex virus type 2 (HSV-2) has had D. Tenosynovitis
symptomatic recurrences approximately every E. Uveitis
/'
month for the last year. What is the standard /
first-line antiviral regime to be prescribed in this 13.5. What is the most likely diagnosis in this 17
case? year old young man who has become aware of
these painless "lumps"?
A. Aciclovir 200 mg four times daily
B. Aciclovir 400 mg twice daily
C. Famciclovir 250 mg once daily
D. Famciclovir 750mg once daily
E. Valaciclovir 500 mg once daily

13.3. A 34 year old human immunodeficiency


virus (HIV)-positive man who has sex with men
(MSM) complains of severe rectal pain,
blood-stained discharge and tenesmus. He is
taking antiretroviral therapy, has a CD4 count
of 636 and an undetectable HIV viral load.
Proctoscopy reveals rectal inflammation and A. Coronal papillae
visible mucopus. Which is the most likely cause B. Genital warts
of this presentation? C. Lichen planus
A. Campylobacter infection D. Molluscum contagiosum
B. Cytomegalovirus colitis E. Sebaceous glands

downloaded from www.medicalbr.com


1 04 • SEXUALLY TRANSMITTED INFECTIONS

13.6. Which of the following antimicrobial drugs 13.10. A 22 year old MSM presents for an
is unlikely to be effective against genital STI screen. His only complaint is of pain on
Chlamydia trachomatis infection? defecation. Examination reveals an anal
A. Amoxicillin fissure. Serological tests for syphilis are as
B. Ciprofloxacin follows:
C. Erythromycin
Test Result
D. Ofloxacin
Antitreponemal lgG EIA Negative
E. Oxytetracycline Negative
APR
TPPA Positive - titre 160
13.7. Which of the following statements is true Antitreponemal lgM EIA Positive - optical density 3.4
of infection with human papilloma virus (HPV)
types 6/11? Which of the following is the most likely
A. More cases of perianal warts are seen in explanation of the serology?
homosexual men than in heterosexual men A. Early latent syphilis
B. This is associated with penile cancer B. False-positive syphilis serology
C. This is cleared by treatment with topical C. Primary syphilis
liquid nitrogen D. Secondary syphilis
D. This is prevented by vaccination with Cervarix E. Treated syphilis
E. This will result in visible genital warts in less
than 50% of cases 13.11. A 38 year old married man tells you that
he had unprotected sex exactly 1 week ago
13.8. A symptomless 43 year old MSM with a woman who he thinks may be an
undergoes a routine STI screen for the first time intravenous drug user (IOU). Which of the
in 3 years. All tests for STis were previously following statements is true?
negative. His serological tests for syphilis are
A. He can safely have sex with his wife
now as follows:
if all tests for STis taken today are
negative
f---0-Te::.:sc=-t_ _ _ _ _ _ _ _ ____cR:..:.:esult
Antitreponemal immunoglobulin G Positive - optical B. He can safely have sex with his wife if/
f---0-(lg"-G-'--),---en-:-z'-ym_e_i_m_m-:-un_o_as_s_,ay---'(~EI--'A)_ _--cd_ensity 20.3 given treatment with a single dose of .;
f---'-'R2ap:c:id'-'p:c:lac:.smc=a"'re.:"ag""inc__('-'R'-'PR:!._)------'P-'ositive - titre 4 azithromycin 1 g ,,·
Treponema pal/idum particle Positive - titre C. He is at significant risk of acquiring h,~patitis
1--ag,_,g'---lu_tin_a_tio_n_a_ss_a-'--y-'--[T_PP_A-'--)_ _ _ _>_5120 C (HCV) '
Antitreponemal immunoglobulin M Negative D. He should be offered post-exposur~
(lgM) EIA
prophylaxis (PEP) against HIV /
E. He should be offered vaccination against
Which of the following is most compatible
hepatitis B (HBV)
with these results?
A. Early latent syphilis 13.12. A symptomless 29 year old MSM
B. Partially treated late syphilis presents for an STI screen. Serological tests for
C. Primary syphilis syphilis are as follows:
D. Secondary syphilis
E. Untreated late syphilis Test Result
Antitreponemal lgG EIA Positive - optical density 33
13.9. A 19 year old woman complains of APR Negative
moderate lower abdominal pain that has been TPPA Positive - titre > 5120
present for 2 weeks, and is particularly Antitreponemal lgM EIA Positive - optical density 11.4
noticeable during sex. Which of the following
actively supports a diagnosis-of chlamydial Which of the foii9I!Ving is the most likely
1
salpingitis? explanation of the serology?
A. A dipstick urine test showing haematuria +++ A. Early latent syphilis
B. A positive pregnancy test B. False-positive syphilis serology
C. A temperature of 36.3"C C. Fully treated late latent syphilis
D. Diarrhoea D. Partially treated late latent syphilis
E. Right upper quadrant tenderness E. Untreated late latent syphilis

downloaded from www.medicalbr.com


SEXUALLY TRANSMITTED INFECTIONS • I 05

13.13. The following infections are not thought the last year. Which of the following statements
of as being STis, but which is the only one that is most appropriate?
cannot be sexually transmitted? A. As her partner has been symptom-free for a
A. Cytomegalovirus (CMV) year, she can be reassured that there is no
B. Hepatitis A (HAV) risk of transmission to her
G. Plasmodium vivax B. Primary genital herpes is more likely to lead
D. Shigella sonnei to disseminated infection if it is caused by
E. Zika virus HSV-2
G. She should avoid unprotected sex for the
13.14. A 27 year old woman is 24 weeks duration of the pregnancy
pregnant. She mentions to you that her current D. She should be commenced on
male partner has a previous history of genital valaciclovir 500 mg once daily to prevent
herpes caused by herpes simplex virus type I transmission
(HSV-1). Although he has had few recurrences E. Her baby should be delivered by caesarean
in the past, he has had no symptoms at all in section

Answers
13.1. Answer: E. MSM but diarrhoea would be a more prominent
Chlamydia can cause a cervicitis, and the symptom. Cytomegalovirus (CMV) colitis is only
resulting friability may present as unexpected seen in end-stage HIV infection, which is clearly
bleeding, especially after sexual intercourse. not the case here.
Urethritis resulting in dysuria is less common,
but may be mistaken for eubacterial cystitis. 13.4. Answer: E.
Deep dyspareunia and lower abdominal pain Typical manifestations of DGI include
are symptoms of ascending infection monoarthropathy, vasculitic rash and
(salpingitis/pelvic inflammatory disease), which tenosynovitis. Endocarditis is seen rarely. The
occurs less frequently than was believed sexually transmitted infection {STI) associated
previously. Increased vaginal discharge is with uveitis is secondary syphilis.
possible, but in most cases is probably due to
an unrelated condition like bacterial vaginosis or 13.5. Answer: A.
candidiasis. Coronal papillae are a normal anatomical
feature, which become more prominent in
13.2. Answer: B. adolescence, and young men can mistake
Early studies using aciclovir to suppress these normal skin appendages for an infection,
recurrences found that 200 mg four times daily especially genital warts. Warts would not be
was more effective than 400 mg twice daily, but limited to the corona, and are usually either
the difference is small enough to recommend more papular or keratotic. Molluscum lesions
the less frequent dosing regime, which is going are umbilicated. Lichen planus typically presents
to make adherence easier. Valaciclovir and as violaceous flat-topped papules. Sebaceous
famciclovir are more expensive and so reserved glands, also known as Fordyce spots, are seen
for cases where aciclovir is ineffective. The on the shaft and base of the penis.
recommended dose of valaciclovir is 500 mg
once daily as per option E, but the correct 13.6. Answer: B.
starting dose of famciclovir is 250 mg twice daily. Erythromycin and oxytetracycline were used
before the advent of azithr9mycin and
13.3. Answer: D. doxycycline, respectively/Ofloxacin is a
Lymphogranuloma venereum is the likeliest quinolone with antichlarnydial efficacy, but this
cause of severe proctitis and is most often is not the case for ciprofloxacin. Somewhat
diagnosed in HIV-positive MSM in the UK. surprisingly, amoxicillin w.,as found to be
Gonococcal proctitis is usually less severe than effective in the treatment of Chlamydia in
in this case, as are the rare cases of syphilitic pregnancy, although azithromycin is much
proctitis. Campylobacter infection is seen in preferred now.

downloaded from www.medicalbr.com


, 1 06 • SEXUALLY TRANSMITTED INFECTIONS

13.7. Answer: E. 13.11. Answer: E.


The percentage of infected patients who A rapid course of vaccination against hepatitis
develop visible warts is unknown, but is B - with inoculations today, and in 1 and 3
definitely a small minority. Although homosexual weeks' time - would give good protection
men (MSM) are relatively more likely to get against this infection that is more common in
perianal warts, the majority of cases present in IDUs. PEP for HIV is only effective if given up to
heterosexual men. The mode of inoculation is 72 hours following risk. Female to male sexual
unclear. Liquid nitrogen destroys infected tissue transmission of HCV is extremely rare. One
but does not clear HPV infection. HPV types week after exposure is too soon to rely upon
6/11 are not associated with genital cancer negative tests for any STI. Negative tests for
- HPV types 16/18 are the most common Chlamydia and gonorrhoea become reliable at
oncogenic types. Cervarix vaccine only protects 2 weeks, negative fourth-generation HIV tests·
against HPV types 16/18; Gardasil also become reliable at 4 weeks, and negative tests
protects against types 6/11 . for HBV and HCV are reliable at 3 months.
Azithromycin is only reliably curative for
13.8. Answer: B. chlamydia! infection, less so for syphilis and
A diagnosis of primary or secondary syphilis is gonorrhoea, and would have no effect upon
based on typical clinical findings so cannot be viruses such as HIV or HBV.
applied to a symptomless individual. The
negative lgM makes early latent syphilis unlikely, 13.12. Answer: A.
and the RPR titre in untreated early or late Three of the four tests are positive, so this is
latent syphilis would be expected to be much not a false-positive scenario. The negative RPR
higher - at the very least 32. The titre here of 4 is almost certainly false and represents a
is more likely to represent accidental treatment prozone phenomenon where the very high
- in this case, antibiotics for a dental infection. antibody titre prevents formation of the
It would still be prudent to offer definitive antibody-antigen lattice necessary to observe
treatment, e.g. with a course of three injections flocculation in the test. Diluting the serum will
of benzathine penicillin at weekly intervals. allow this to be observed. The strongly positive
lgM test makeslate infection extremely uflkely.
13.9. Answer: E.
Right upper quadrant tenderness is a feature of 13.13. Answer: C. /
perihepatitis, a rare complication of ascending Male to female transmission of Zika viruJ !r
is
chlamydia! infection. Diarrhoea is not a described. Outbreaks of shigellosis and 'HAV
symptom suggestive of salpingitis. Haematuria are seen in MSM. CMV is shed in genital
would be more suggestive of a urinary tract secretions. Plasmodium vivax (malaria;}' is not
infection. A normal temperature neither known to be sexually transmitted.
supports nor refutes the diagnosis. A positive
pregnancy test would raise concerns about a 13.14. Answer: C.
possible ectopic pregnancy. Both HSV-1 and HSV-2 have a greater risk of
causing disseminated disease in pregnancy, so
13.10. Answer: C. it is important that she is counselled effectively
The positive lgM is suggestive of early infection. to prevent acquisition. Symptomless shedding
False-positive lgM tests are possible, but a of virus can continue in the absence of clinical
second positive test, in this case the TPPA, episodes, so there is a risk of transmission in
makes that unlikely. The low TPPA titre is this scenario. Valaciclovir has been shown to
compatible with very early infection. Although reduce HSV transmission in sera-discordant
the chancre of primary syphilis is usually couples, is probably safe to take in pregnancy,
painless, this is not necessarily so for an anal but would be a suboptimal strategy. Caesarean
chancre, so primary infection is most likely. The section would only ,tieconsidered if she
RPR would be strongly positive in secondary or developed primar/infection around the time of
early latent syphilis. In treated infection, the lgG delivery. Avoidance of sex or consistent
EIA would remain positive, but the lgM would condom use repre(lents the safest strategy in
become negative. this case.

downloaded from www.medicalbr.com


A Mather, D Burnett,
DR Sullivan

Clinical biochemistry and


metabolic medicine
Multiple Choice Questions
14.1. What is a particular advantage of programmes. What is the most common form
obtaining a test analysis and result using a of inheritance of these LSDs?
point-of-care test (POCT) system rather than A. Autosomal dominant
using a traditional central laboratory? B. Autosomal recessive
A. POCT analysers often have a wider menu of G. Multifactorial
available tests than central laboratories D. X-linked dominant
B. POCTs avoid the need to use the laboratory E. X-linked recessive
or the medical records
C. POCTs provide test results at the time of 14.4. In the investigation of glycogen storage /
seeing the patient diseases (glycogeneses), which of the following
D. POCTs are generally cheaper than traditional is a commonly used non-invasive test or finding
testing that may be useful in diagnosing this condition?
E. POCTs use newer technology and are A. Cataract in the lens of the eye
generally more accurate and precise B. 'Cherry-red spot' in the fundus of the ey,k
G. Dislocated lens (ectopia lentis) in the eye
14.2. Which of the following is an autosomal D. Exercise-induced fatigue or pain in muscles
recessive inherited disorder, often diagnosed E. Hypopigmentation of the skin
through newborn screening programmes
and treated with dietary modification, which 14.5. A 59 year old man presents for
can present with wide-ranging clinical cardiovascular risk assessment, but he has not
manifestations, including vascular disorders, fasted for the blood collection that was to be
skin hypopigmentation, ectopia lentis, and performed during his appointment. Which of
disorders of the central nervous or skeletal the following plasma lipid or lipoprotein levels is
systems? most likely to be affected by his recent
A. Cystathionuria consumption of food?
B. Cystinosis A. Calculated low-density lipoprotein (LDL)
C. Cystinuria cholesterol
1
D. Homocystinosis B. High-density lipoprotein (HDL) cholesterol
E. Homocystinuria G. Lipoprotein (a)
D. Non-HDL cholesterol
14.3. There have been many different lysosomal E. Total cholesterol
storage diseases (LSDs) discovered, and some
of these have been included in successful 14.6. The same 59 year old man returns with
population-wide community genetic screening a set of fasting results that include: total

downloaded from www.medicalbr.com


1 08 • CLINICAL BIOCHEMISTRY AND METABOLIC MEDICINE
l
cholesterol 6. 7 mmoi/L (259 mg/dL), fasting blood. When she receives the results she is
triglyceride 3.3 mmoi/L (292 mg/dL), HDL worried that the test is inaccurate due to
cholesterol 0.9 mmoi/L (35 mg/dL), calculated haemolysis of the cells whilst performing
LDL cholesterol 4.3 mmoi/L (166 mg/dL), venepuncture. What is the dominant
non-HDL cholesterol 5.8 mmoi/L (224 mg/dL) intracellular cation that may be inaccurately
and fasting serum glucose 6.9 mmoi/L (124 mg/ reported in this situation?
dL). What is the best indicator of the metabolic A. Bicarbonate
component of his cardiovascular risk? B. Calcium
A. Calculated LDL cholesterol C. Magnesium
B. Fasting plasma glucose D. Potassium
C. HDL cholesterol E. Sodium
D. Non-HDL cholesterol
E. Total cholesterol 14.10. One litre of normal saline is given to a
patient in the emergency department. How is
14.7. The same 59 year old man fails to this fluid likely to be distributed between the
improve his lipid profile following diet and fluid compartments?
exercise advice, and pharmacological treatment A. Intracellular fluid 0 mL, extracellular fluid
is deemed necessary. Which of the following 1000 mL, plasma volume 1000 mL
medications may have a detrimental effect on B. Intracellular fluid 0 mL, extracellular fluid
the triglyceride component of his lipid profile? 1000 mL, plasma volume 200 mL
A. An anti-PCSK9 monoclonal antibody C. Intracellular fluid 1000 mL, extracellular fluid
B. Cholestyramine 0 mL, plasma volume 0 mL
C. Ezetimibe D. Intracellular fluid 500 mL, extracellular fluid
D. Niacin 500 mL, plasma volume 500 mL
E. Rosuvastatin E. Intracellular fluid 666 mL, extracellular fluid
334 mL, plasma volume 68 mL
14.8. The same 59 year old man commences
atorvastatin 20mg every evening. His follow-up 14.11. In comparison to the ultrafiltrate foLf.ld in
lipid profile and glucose reveals: total Bowman's capsule, which of these term~ best
cholesterol 3. 7 mmoi/L (143 mg/dL), fasting describes the filtrate that leaves the pro~(mal
triglyceride 1.1 mmoi/L (97 mg/dL), HDL tubule? p
cholesterol 1 .1 mmoi/L (42 mg/dL), calculated A. Hyperosmolar
LDL cholesterol 2.1 mmoi/L (81 mg/dL), B. Hypertonic
non-HDL cholesterol 2.6 mmoi/L (1 00 mg/dL) C. Hypo-osmolar (
I
and fasting serum glucose 8.9 mmoi/L (160 mg/ D. Hypotonic
dL). A subsequent glucose tolerance test is E. Isotonic
diagnostic of new-onset type 2 diabetes. What
best describes the relationship between the 14.12. Amino acids are almost entirely
onset of diabetes and the use of statins? reabsorbed from the glomerular filtrate via
A. Diabetes development is more likely in those active transport in which section of the
with pre-existing impaired fasting glucose nephron?
B. The development of diabetes is inconsistent A. Collecting duct
with the fact that fasting triglyceride has B. Early distal tubule
improved C. Late distal tubule
C. The development of diabetes is unrelated to D. Loop of Henle
the dose or potency of the statin E. Proximal tubule
D. The development of diabetes means that
I
statins are now contraindicated in that 14.13. A 35 year o)d man has been hiking in hot
individual weather. He collapses and is brought into the
E. The onset of diabetes and the use of statins emergency department. He is found to have a
are completely unrelated blood pressure of .95/62 mmHg with a postural
drop of 15 mmHg. His1 pulse rate is 112 beats/
14.9. A 57 year old man is having a blood test min, his jugular venous pressure is npt visible
and the resident doctor finds it difficult to take and he has a dry tongue. Which statement

downloaded from www.medicalbr.com


CLINICAL BIOCHEMISTRY AND METABOLIC MEDICINE • 109

describes an element of his physiological C. Excess aldosterone release into the


response to this clinical scenario? circulation
A. Increased atrial natiuretic peptide release D. Inadequate solute delivery to the early distal
B. Increased catecholamine release tubule
C. Increased glomerular filtration rate E. Inadequate solute delivery to the loop of
D. Reduced renin release Henle
E. Vasoconstriction of renal efferent arterioles
14.17. A 17 year old male presents to the .
emergency department with poorly controlled
14.14. Which statement best explains why loop type I diabetes. He is found to be
diuretics are the most effective at promoting hyponatraemic with the following results:
salt and water excretion? sodium 123 mmoi/L and plasma glucose
A. Vasopressin (antidiuretic hormone, ADH) acts 35 mmoi/L (630 mg/dL). Which one of the
on the ascending limb of the loop of Henle following is the most likely cause of the
to increase water permeability abnormal sodium reading?
B. Loop diuretics block the triple co-transporter A. Autoimmune hypothyroid disease
that prevent the reabsorption of potassium B. Hyperosmotic hyponatraemia secondary to
C. The ascending limb of the loop of Henle is hyperglycaemia
permeable to both water and sodium C. Hypoglycaemic agent-induced
D. The ascending limb of the loop of Henle is hyponatraemia

-
the last segment to reabsorb sodium D. Loss of water in excess of sodium
E. The sodium reabsorptive capacity of the E. Osmotic diuresis-induced hypovolaemic
segments distal to the ascending limb of the hyponatraemia
loop of Henle is limited

14.15. An 83 year old woman presents to the


14.18. A 32 year old man who has been I
diagnosed with chronic schizophrenia lives with
emergency department delirious and his mother and has been managing well in the
disorientated. She has a history of community on stable medications for some
hypertension, treated with a thiazide diuretic. time. An ambulance was called to the house 1
Her blood tests reveal the following: serum when he started having seizures and his blood~
sodium 116 mmoi!L; serum osmolality on presentation to the emergency department/
239 mmol/kg; urinary osmolality 385 mmol/kg. are as follows: sodium 116 mmoi!L; potassiu\f\
I
Which of the following abnormalities is 4.0mmoi/L; chloride 88mmoi/L; bicarbonate''
responsible for her inability to maximally dilute 20mmoi/L; urea 9mmoi/L (54mg/dL);
her urine? creatinine 661J-moi/L (0. 75 mg/dL}. His mot~er
A. Abnormal function of the early distal cannot recall any changes to his medications or
tubule in his behaviour but does comment that he has
B. Inadequate vasopressin in the circulation been drinking up to 8 L of water per day.
C. Inadequate response of the collecting duct Which of the following results are most likely
to vasopressin to be found on further investigation?
D. Inadequate solute delivery to the early distal A. Serum osmolality 235mmol/kg; urine
tubule osmolality 74mmol/kg; urine sodium
E. Inadequate solute delivery to the loop 24mmoi!L
of Henle B. Serum osmolality 262mmol/kg; urine
osmolality 112 mmol/kg; urine sodium
14.16. A 67 year old man with hypertension and 5mmoi/L
diabetes has chronic kidney disease with a C. Serum osmolality 270mmol/kg; urine
stable serum creatinine of 2671J-moi!L (3.02 mg/ osmolality 135mmol/kgj urine sodium
dL). Amongst other symp}oms, he .complains of 42mmoi/L /
nocturia. Which of the following abnormalities is D. Serum osmolality 290 mmol/kg; urine
responsible for his inabilit¥ to maximally osmolality 84mmol/kg; urine sodium
concentrate his urine? 34mmoi/L
A. Abnormal function of the early distal tubule E. Serum osmolality 280mmol/kg; urine
B. Excess vasopressin release into the osmolality 64mmol!kg; urine sodium
circulation 44mmoi/L

downloaded from www.medicalbr.com


4
110 • CLINICAL BIOCHEMISTRY AND METABOLIC MEDICINE
I

14.19. In which one of the following clinical 14.24. The amount of potassium excreted by
scenarios is urine sodium excretion likely to be the kidneys will decrease in which of the
less than 20 mmol/24 hrs? following situations?
A. Acute diarrhoea A. When dietary intake of potassium increases
B. Adrenal insufficiency B. When distal tubule sodium delivery increases
C. Hypothyroidism C. When plasma aldosterone concentration
D. Renal disease increases
E. Syndrome of inappropriate antidiuretic D. When the patient has acute metabolic
hormone (vasopressin) secretion (SIADH) acidosis
E. When the patient has respiratory alkalosis
14.20. A 57 year old man with hypertension is
found to have a tumour arising in the zona 14.25. A 42 year old patient has the following
glomerulosa of the adrenal gland that leads to bloods. Arterial blood gases: W 57.5nmoi/L (pH
uncontrolled secretion of a hormone that is 7.24); Pa0 2 11.1 kPa (83 mmHg); PaC02 4.3 kPa
responsible for his hypertension. (32 mmHg); bicarbonate 15 mmoi!L. Serum
Which of the following would you expect to biochemistry: sodium 134 mmoi/L; potassium
decrease in this scenario? 2.4mmoi/L; chloride 109mmoi/L. Urine pH 5.2;
following administration of intravenous sodium
A. Extracellular fluid volume
bicarbonate, urine pH is 5.8.
B. Plasma concentration of bicarbonate
What is the likely underlying cause of these
C. Plasma concentration of potassium
abnormalities?
D. Thyroid-stimulating hormone
E. Tubular reabsorption of sodium A. Loop diuretic abuse
B. Thiazide diuretic abuse
14.21. A 12 year old boy is being investigated C. Type 1 (distal) renal tubular acidosis
for fatigue. A physical examination, including D. Type 2 (proximal) renal tubular acidosis
blood pressure, is normal. Blood results show: E. Type 4 renal tubular acidosis
sodium 135 mmoi/L, potassium 3.1 mmoi/L,
bicarbonate 35 mmoi/L; 24-hour urine results: 14.26. A 38 year old man presents with a;
potassium 245 mmol/24 hrs, calcium 1-week history of arthralgia, rash, haem9,turia
12mmol/24hrs (N < 7.5). and mild peripheral oedema. Blood test~ taken
What is the most likely diagnosis? in the emergency department show thdt his
serum creatinine is 6201-!moi/L (7.01 m~/dL).
A. Bartter's syndrome
What pattern of acid-base disorder is most
B. Gitelman's syndrome
likely to occur in this clinical scenario'?
C. Laxative abuse I

D. Primary hyperaldosteronism A. Metabolic acidosis with no respiratory


E. Type 1 renal tubular acidosis (RTA) compensation
B. Metabolic acidosis with respiratory
14.22. Metabolic acidosis is seen in conjunction compensation
with which cause of hypokalaemia? C. Metabolic alkalosis with respiratory
compensation
A. Diarrhoea
D. Respiratory acidosis with metabolic
B. Gitelman's syndrome
compensation
C. Loop diuretics
E. Respiratory alkalosis with metabolic
D. Primary hyperaldosteronism
compensation
E. Vomiting

14.27. A 42-year-old homeless man is brought


14.23. Hypokalaemia may be seen in into the emergency pepartment. He is known
association with normal blood pressure in to have a history 9f' alcohol abuse and presents
which of the following conditions? on this occasion with delirium, shortness of
A. Bartter's syndrome breath and blurred vision. Initial investigations
B. Cushing's syndrome show the following. Arterial blood gases (ABG):
C. Gordon's syndrome H+ 58.9nmoi/L (pH 7.23); PaC0 2 3.6kPa
D. Liddle's syndrome (27 mmHg); bicarbonate 12 mmoi/L. Blood
E. Primary hyperaldosteronism results: sodium 130 rnrnoi/L; potassl~m

downloaded from www.medicalbr.com


CLINICAL BIOCHEMISTRY AND METABOLIC MEDICINE • 111

4.3mmoi/L; urea 7.2mmoi/L (43mg/dL); C. Haemoglobin


creatinine 113!lmoi/L (1 .28 mg/dL); chloride D. Hydrogen phosphate
97 mmoi/L; glucose 4.2 mmoi/L (76 mg/dL). E. Proteins
What is the most likely diagnosis?
A. Acute kidney injury 14.29. A high school student is nervous about
B. Diabetic ketoacidosis an upcoming exam and breathes rapidly with
c. Ethylene glycol ingestion anxiety before fainting. If you were to take an
D. Methanol ingestion ABG at this point, what would you most likely
E. Severe ethanol intoxication find?
A. High pH; high HC03-, high PC0 2
14.28. Which of the following is the most B. High pH; normal HC03-, low PC02
important buffer in the blood? C. High pH; low HCQ3-, low PC0 2
A. Ammonia D. Low pH; high HC0 3-, high PC02
B. Bicarbonate E. Low pH; low HC03-, high PC02

Answers
14.1. Answer: C. recessive pattern. It is characterised by high
The key advantage of POCT testing over concentrations of cysteine in the urine, leading
central laboratory testing is that rapid availability to cysteine stone formation in the urinary tract.
of the result enables immediate medical Cystinosis is a lysosomal storage disease and
decisions and actions. POCTs are generally is also inherited in an autosomal recessive
more expensive than the equivalent test manner. There is accumulation of cystine within
performed in a central laboratory. While POCT tissues. It is one of the causes of Fanconi's
instruments often use new technology, the
requirement for portability or miniaturisation
may involve design or engineering compromises
syndrome, in which there is abnormal renal
tubular function. Cystathionuria (also called
cystathionase deficiency) is also an autosomal .·
I
that result in less accuracy or precision than recessive disorder, in which there is abnormal {
the equivalent standard laboratory test. Most accumulation of plasma cystathionine, leading,#
POCT instruments are designed for a specific to increased urinary excretion. It is often ~c
environment or group of tests, and so their considered to be a benign biochemical
menu is usually more restrictive than standard anomaly. ;'
I
laboratory analysers. All laboratory and
pathology results, including POCT, should 14.3. Answer: B.
always be recorded in the medical records. Most lysosomal storage diseases exhibit an
autosomal recessive pattern of inheritance,
14.2. Answer: E. although a few can be X-linked recessive (e.g.
Homocystinuria is inherited in an autosomal Fabry's disease).
recessive manner. It is most commonly caused
by loss of function of the cystathionine 14.4. Answer: D.
~-synthase (CBS) gene. This affects the Exercise-induced fatigue or pain in muscles is
metabolism of the amino acid methionine and associated with several of the glycogenoses.
causes accumulation of the related amino acids An ischaernic lactate forearm test can be used
homocysteine and methionine. It is often as a clinical diagnostic test for some forms of
diagnosed through newborn screening glycogen storage disease. The cherry-red spot
programs. Dietary treatment is avaiiEJble, in the fundus is typically ays'ociated with
designed to correct the imbalance in the amino Tay-Sachs disease, one/of the inherited GM2
acids caused by the missing enzyme function. gangliosidoses. Hypopigrnentation, ectopia
There is no condition called homocystinosis. lentis and cataracts can be associated with
This should not be confused with many conditions, some of which are inherited,
hornocystinuria (see option E). Cystinuria is an but the glycogenoses are not typically part of
aminoaciduria, inherited in an autosomal this group.

downloaded from www.medicalbr.com


112 • CLINICAL BIOCHEMISTRY AND METABOLIC MEDICINE

14.5. Answer: A.
Calculated LDL cholesterol is correct because
body water, as in option E. Fluids that are rich
in proteins (such as concentrated albumin) will
T
i

the calculation includes the triglyceride level, remain in the plasma volume, as in option A.
which increases following food consumption. Normal saline distributes within only the
The effect of food consumption on the other extracellular compartment as in option B.
measurements is small by comparison,
especially in relative terms. 14.11. Answer: E.
In the proximal tubule, water reabsorption
14.6. Answer: D. closely matches sodium reabsorption, meaning
Non-HDL cholesterol is correct because it that the fluid that enters the loop of Henle is
allows for the presence of small dense LDL and isotonic with the fluid that leaves the Bowman's
other atherogenic lipoproteins. This is capsule.
particularly relevant in hypertriglyceridaemia,
with or without accompanying elevation of 14.12. Answer: E.
fasting plasma glucose. It is more strongly The proximal tubule reabsorbs filtered sodium
associated with cardiovascular disease (CVD) in by coupling re-entry of sodium into the
studies where comparison has been made with proximal tubular cell with amino acids as well
the other alternatives. as glucose, phosphate and other organic
molecules.
14.7. Answer: B.
Cholestyramine reduces recirculation of bile 14.13. Answer: B.
acids, down-regulates the farnesoid X receptor This man has hypovolaemia and sodium
(FXR) and stimulates the replacement of the bile depletion as evidenced by his symptoms and
acids by conversion of cholesterol via 7 signs on presentation. The kidneys respond to
alpha-hydroxylase. The response to the this scenario by activating mechanisms that will
down-regulation of FXR includes increased increase sodium reabsorption, thereby restoring
synthesis and secretion of triglyceride and very sodium and fluid balance. Mechanisms that will
low-density lipoproteins (VLDLs). The other increase sodium reabsorption include increased
agents have neutral or favourable effects on catecholamine release and increased renin/
triglyceride levels. release. In order to restrict fluid loses the /
kidneys will reduce glomerular filtration ra,fe in
14.8. Answer: A. part by vasoconstriction of renal afferen~/
Type 2 diabetes following statin therapy is likely arterioles.
in those with pre-existing impaired fasting
glucose. It is proportional to the dose and 14.14. Answer: E. I
/'
potency of the statin, but the CVD benefit of Loop diuretics inhibit the Na,K,2CI triple
the response clearly outweighs the CVD risk of co-transporter in the ascending limb of the loop
the diabetes. Statins modestly improve of Henle and are the most effective diuretics as
triglyceride, even in the presence of diabetes. this transporter reabsorbs about 25% of the
sodium load. More distal reabsorption by the
14.9. Answer: D. sodium-chloride transporter in the distal tubule
The dominant intracellular cation is potassium. only accounts for about 5% of sodium
If cells haemolyse during venepuncture, reabsorption and increased delivery to this
increased potassium will be released from the segment when using a loop diuretic
cells and a patient may be erroneously overwhelms the reabsorptive capacity of that
diagnosed with hyperkalaemia. transporter. Option C is incorrect as the
ascending limb is permeable only to sodium;
14.10. Answer: B. the triple co-transpor)er does transport
Total body water is about one-third extracellular potassium as in opJi6n B, but this is not
fluid (ECF) and two-thirds intracellular fluid. ECF relevant to the diuretic effect; in option D, the
is about one-fifth plasma and four-fifths ascending limi;J of the loop of Henle is not the
interstitial fluid. Fluids that contain neither last segment to reabsorb sodium, as outlined
sodium nor protein (such as 5% dextrose) will above; and in option A, vasopressin acts on
distribute in all the body fluid compartments in the collecting ducts to increase wate~
proportion to the normal distribution of total permeability.

downloaded from www.medicalbr.com


CLINICAL BIOCHEMISTRY AND METABOLIC MEDICINE • 113

14.15. Answer: A. 14.19. Answer: A.


In order to maximally dilute urine, there needs Acute diarrhoea would result in extrarenal
to be normal function of both the loop of Henle sodium and water loss and the normal renal
and the early distal tubule. Thiazide diuretics response of sodium conservation. In the other
inhibit the normal function of the early distal scenarios urine sodium would be high or
tubule by blocking the sodium-chloride normal due to effects of limited sodium
co-transporter. An inability to maximally dilute reabsorption in the nephron secondary to
urine can also result from options D and E but vasopressin, or lack of cortisol/thyroxine
this is not the mechanism of thiazide diuretics. response.
Absence of vasopressin is required for maximal
dilution of the urine. 14.20. Answer: C.
Aldosterone is produced in the zona
14.16. Answer: E. glomerulosa of the adrenal gland and acts on
Chronic kidney disease results in poor solute the mineralocorticoid receptors in the distal
delivery to the loop of Henle causing a failure tubules and collecting ducts of the nephron.
to generate the medullary concentration It acts to reabsorb sodium and excrete
gradient. Adequate solute delivery to and potassium. In excessive quantities, such as in
function of the early distal tubule is required for Conn's syndrome as described here, it causes
maximal dilution of urine but not concentration. hypertension and hypokalaemia.
Failure of vasopressin effect, either through
inadequate release or blunted action at the 14.21. Answer: A.
level of the collecting duct (rather than excess Metabolic alkalosis associated with
vasopressin), contributes to poor urinary hypokalaemia and urinary potassium wasting is
concentration. typical of diuretic use, or in this case Bartter's
syndrome, which mimics loop diuretic use.
14.17. Answer: B. Gitelman's or thiazide diuretics would also
Hyperglycaemia causes osmotic shifts of present like this, but are associated with low,
water from the intracellular to the extracellular not high, urinary calcium. Laxative abuse would
space, causing a relative dilutional be associated with renal conservation of /
hyponatraemia. The serum sodium corrects potassium and therefore low urinary potassium,'
to 131 mmoi/L when using the correction level. Primary hyperaldosteronism is associated
factor of 1.6mmoi/L for every 5.5mmoi/L with hypertension and RTA with acidosis. /1
increase in serum glucose. The other causes '
of hyponatraemia are possible but would 14.22. Answer: A.
result in a genuine reduction in sodium Loop and thiazide diuretics, Bartter's syndr,bme
concentration and option D would cause and Gitelman's syndrome, and primary
hypernatraemia. hyperaldosteronism are all associated with
metabolic alkalosis. As outlined in Fig. 14.22,
14.18. Answer: A. vomiting is also associated with metabolic
These results are consistent with primary alkalosis while diarrhoea causes loss of
polydipsia which is the likely diagnosis here. bicarbonate thereby resulting in a normal anion
The serum osmolality is low,. confirming gap acidosis.
hypotonic hyponatraemia and the urinary
osmolality is also low suggesting relative 14.23. Answer: A.
excess water intake. Option B, which Answers B-E are associated with hypertension
demonstrates low urinary sodium, is seen in while Bartter's syndrome is associated with low
patients with low effective ar:terial volume due or normal blood pressure readings.
to either extrarenal losses or hypervolaemic
I
states. The high urinary sodium and osmolality 14.24. Answer: D. /
seen in option C is consistent with SIADH or A number of factors alter potassium secretion
renal sodium loss. Option D is consistent with in the distal nephron segments. Increased distal
hyperosmotic hyponatraemia, as seen in sodium delivery and increased plasma
hyperglycaemia, and option E suggests aldosterone concentration will result in greater
isosmotic hyponatraemia such as with luminal sodium entry through epithelial sodium
hyperlipidaemia. channels, thereby increasing potassium

downloaded from www.medicalbr.com


114 • CLINICAL BIOCHEMISTRY AND METABOLIC MEDICINE

bicarbonate reabsorption, which makes type 2


Vomiting
(proximal) RTA the most likely diagnosis.

Gastric loss of 14.26. Answer: B.


Acute kidney injury as seen in this patient with
nephritic syndrome is associated with metabolic
acidosis with respiratory compensation.

Hypovolaemia I 14.27. Answer: D.


Looking first at the ABG, there is a low pH
t Proximal suggesting an acidosis. Bicarbonate and
Na+Hcoa- carbon dioxide are both low, consistent with a·
reabsorption
metabolic acidosis with respiratory
compensation. The anion gap = (Na+ + K+)
Hypokalaemia I - (CI--HC03-) is 23, which is high. This
indicates the presence of unmeasured anions.
A high anion gap acidosis is commonly caused
t Distal W by an endogenous acid load as seen in
secretion diabetic ketoacidosis or kidney injury, but in
this patient the absence of hyperglycaemia
or significant renal impairment make these
Metabolic
alkalosis t H+ excretion I diagnoses unlikely. Other causes of an
increased anion gap acidosis are related
to exogenous acid loads from poisoning.
Fig. 14.22 Generation and maintenance of metabolic
alkalosis during prolonged vomiting. Loss of HCI- generates
Methanol poisoning typically presents with
metabolic alkalosis, which is maintained by renal changes. visual impairment and in severe cases results
in permanent blindness.

secretion. Acid-base disturbances have 14.28. Answer: B. L


complex effects on renal potassium excretion. As outlined in the above questions, the m9;st
Alkalosis is generally associated with increased important buffer system in blood and tissues
potassium secretion while acute metabolic involves the reaction of hydrogen ions (Hif with
acidosis is associated with reduced renal bicarbonate (HC03 -) to form carbonic acib
potassium excretion. However, over time, (H 2C03) and ultimately C0 2 and H20. Hydrogen
acidosis will cause an increase in distal sodium phosphate (HP04) and ammonia are in;/portant
delivery and an increase in aldosterone urinary buffers that associate with W ions
production that will result in an increase in secreted into the luminal space, thereby
potassium secretion. reducing luminal W concentration and allowing
for continued acid secretion.
14.25. Answer: D.
The patient has a metabolic acidosis with a low 14.29. Answer: C.
pH, low bicarbonate and compensatory low The student would have a respiratory alkalosis,
PaC0 2 • Acidosis with a low potassium, makes best represented by option C. Given the acute
type I or 2 renal tubular acidosis (RTA) the only nature of the respiratory alkalosis, a small
possible answers. Type 4 renal tubular acidosis change in bicarbonate concentration occurs,
would be associated with hyperkalaemia and but if respiratory alkalosis persists over days to
diuretic abuse is associated with metabolic weeks, the kidneys would have time to make
alkalosis. The urine pH is initially normal, but adjustments to acid secretion and produce
becomes alkalotic when bicarbonate is further compensation and reduction in
administered. This is consistent with poor HC0 3- concentration.

downloaded from www.medicalbr.com


8 Conway, P Phelan,
GD Stewart

Nephrology and urology


Multiple Choice Questions
15.1. A 45 year old man presents with a 6-week formula-derived estimated glomerular filtration
history of bilateral ankle swelling. On rate (eGFR) of 40 mUmin/1. 73 m 2 . Which
examination his pulse was 72 beats/min, blood person below is likely to have the lowest
pressure (BP) 126/68 mmHg, jugular venous measured (true) glomerular filtration rate (i.e. the
pressure (JVP) was not elevated and eGFR is falsely reassuring)?
auscultation of heart and lungs was A. A 25 year old male body builder
unremarkable. He had no stigmata of chronic B. A 40 year old African American man with II
liver disease. Which of the following is the most hypertension
appropriate initial investigation? G. A 45 year old woman currently taking I
A. Abdominal ultrasound scan trimethoprim for a urinary tract infection
B. D-dimer D. A 56 year old man with type 2 diabetes and
G. Echocardiogram an above-knee amputation j
D. Urinalysis E. An 85 year old woman with hypertension ,1
E. Urinary sodium and type 2 diabetes
!'
f
·;I
15.2. A 72 year old man is found to have acute 15.5. A 46 year old man with a 10-year history
kidney injury (AKI). Urine microscopy reveals the of type 2 diabetes presents with a 6-week ;'
presence of red cell casts. What is the most history of bilateral leg swelling. He reports 1
likely aetiology of his renal failure? that he had been taking non-steroidal
A. Acute tubular necrosis anti-inflammatory drugs (NSAIDs) for
B. Haemolytic uraemic syndrome osteoarthritis regularly for the past 3 months.
G. Microscopic polyangiitis Investigations reveal: eGFR >60 mUmin/
D. Sclerodermic renal crisis 1. 73 m2 ; urinalysis: protein 4+, blood negative;
E. Tubulointerstitial nephritis protein: creatinine ratio 1680 mg/mmol; and a
serum albumin of 14 g/L. Serum albumin and
15.3. Which of the following is maintained in the urinary albumin: creatinine ratios 4 months
circulation when transiting through the kidney previously were 36 g/L and 25 mg/mmol,
and not freely filtered acro~s the normal respectively. What is the most likely diagnosis?
glomerular filtration barrier? A. Amyloidosis
A. Free light chains B. Diabetic nephropathy
B. Glucose G. lgA nephropathy /
G. Glutamine D. Minimal change disease
D. Immunoglobulin A (lgA) E. Tubulointerstitial nephritis
E. Lithium
15.6. A 25 year old man presents with visible
15.4. The following subjects all have a haematuria. He reports that he had a very sore
Modification of Diet in Renal Disease (MDRD) throat 2 weeks previously, but is otherwise well.

downloaded from www.medicalbr.com


11 6 • NEPHROLOGY AND UROLOGY

His blood pressure and renal function are both


normal. Protein: creatinine ratio was elevated
of hydronephrosis. No urine is available for
urinalysis. What is the most appropriate initial
T
(100 mg/mmol). What is the most likely investigation from the list below?
diagnosis? A. Anti-glomerular basement membrane
A. Bladder cancer (GBM)/antineutrophil cytoplasmic antibody
B. lgA nephropathy (ANCA)/antinuclear antibody (ANA) serology
C. Polycystic kidney disease (PKD) B. Computed tomography (CT) pulmonary
D. Post-infectious glomerulonephritis angiography
E. Renal calculus C. Genetic testing for Alport's disease
D. Plasma protein electrophoresis
15.7. A 69 year old man is diagnosed with E. Renal biopsy
streptococcal endocarditis and commenced on
benzylpenicillin and gentamicin. His renal 15.1 0. A 32 year old man is referred to the
function is normal on admission, but 1 week nephrology clinic for investigation of persistent
later it has deteriorated (eGFR 28 mU non-visible haernaturia initially detected at an
min/1.73 m 2). Investigations reveal: urinalysis: insurance medical examination. He is otherwise
blood 3+, protein 3+; ultrasound scan: well, with no personal or family history of renal
normal-sized kidneys with no hydronephrosis; disease. His BP is 126/68 rnmHg. Preliminary
serum complement level (C3 and C4) is low. investigations reveal: urinalysis: blood 3+,
What is the most likely diagnosis? protein negative; creatinine 100 11moi/L
A. Acute interstitial nephritis (1.13 mg/dl); eGFR > 60 rnUmin/1. 73 m 2.
B. Acute tubular necrosis What is the most likely diagnosis?
C. Infection-related glomerulonephritis A. Alport's disease
D. Microscopic polyangiitis B. Bladder tumour
E. Pre-renal failure C. lgA nephropathy
D. Membranous nephropathy
15.8. A 76 year old woman attends her family E. Vesica-ureteric reflux
physician complaining of bilateral leg swelling
and vague aches and pains. Initial investigations 15.11. A 75 year old woman has peripheral/
reveal: urinalysis: protein 4+, trace blood; vascular disease and stage 3 CKD with ,
haemoglobin 79 g/L; white cell count 1.9x109/L; proteinuria due to lgA nephropathy. Her Bf is
platelet count 46 x 109/L; sodium 131 mmoi/L; 136/80 mrnHg on lisinopril 40 mg, amlodj.~ine
potassium 4.6 mmoi/L; urea 15 mmoi/L 10 rng and bendroflurnethiazide 2.5 mg (all
(90.1 mg/dl, BUN 42.0 mg/dl); creatinine once daily). Her renal function has been ·
176 11moi/L (1.99 mg/dl); albumin 23 g/L. relatively stable over the past 2 years w'ith
What is the most likely finding on renal biopsy? current eGFR 39 mUmin/1.73 m 2 . Ultrasound
A. Amyloidosis scan revealed that her left kidney length at
B. Cast nephropathy 9 em was smaller than the right kidney at
C. Interstitial nephritis 11.5 em. Magnetic resonance angiography
D. Minimal change disease confirmed a 90% stenosis at the ostium of the
E. Thrombotic thrombocytopenic purpura (ITP) left renal artery. What is the most appropriate
management from the list below?
15.9. A 49 year old male presents with A. Check plasma renin activity
deafness, shortness of breath, haemoptysis, B. Commence a statin
reduced urinary output and ankle swelling. On C. Discontinue lisinopril
examination: BP is 170/100 mmHg; JVP is D. Perform angiography and stenting to her left
4 em above the sternal angle, there are bibasal renal artery
crepitations in the lungs and he has bilateral leg E. Start warfarin
swelling to the mid-calves. Initial investigations
reveal: haemoglobin 92 g/L, white cell count 15.12. A 62 year old man presents with a large
9x 109/L; platelet count 460x 109/L; sodium myocardial infarction and undergoes primary
142 mmoi/L; potassium 6.8 mmoi/L; urea coronary angiograplly and stenting. Two days
45 mmoi!L (270 mg/dl); creatinine later he develops a low~grade fever and dusky
1260 11moi!L (14.25 rng/dl); albumin 32 g/L. discolouration of the toes on both feet,
Chest X-ray: bi-basal air space shadowing; although peripheral pulses are palpable. eGFR
ultrasound: normal-sized kidneys, no evidence was 52 mUmin/1. 73 rn 2 pre-procedure and

downloaded from www.medicalbr.com


NEPHROLOGY AND UROLOGY • 117

falls to 25 mUmin/1 .73 m 2 2 days later. Other D. Pre-renal failure


investigations reveal: urinalysis: blood 1+, E. Thrombotic thrombocytopenic purpura
protein 1+; haemoglobin 12 g/L; white cell
count 10.6x109/L with eosinophilia; platelet 15.14. A 60 year old man with long-standing
count ?Ox 109 /L. Creatine kinase is elevated at stage 4 chronic kidney disease presents with
640 U/L. What is the most likely cause of his vague bony pain. Blood tests reveal eGFR
acute kidney injury? 17 mUmin/1.73 m 2 ; calcium 2.92 mrnoi/L
A. Cholesterol embolisation (11.70 mg/dl); phosphate 1.82 mmoi/L
B. Contrast nephropathy (5.64 rng/dl), parathyroid hormone (PTH) is
C. Haemolytic uraemic syndrome elevated at 156 pmoi/L (14 71 pg/ml), alkaline
D. Renal artery thrombosis phosphatase 470 U/L. What is this picture
E. Rhabdomyolysis consistent with?
A. Excess vitamin D consumption
15.13. A 17 year old male returns from an B. Milk alkali syndrome
Outward Bound centre holiday and falls ill with C. Primary hyperparathyroidism
vomiting and bloody diarrhoea. His acute illness D. Secondary hyperparathyroidism
subsides, but 3 days later he notices that his E. Tertiary hyperparathyroidism
urinary output has declined and his ankles begin
to swell. He attends his family physician where 15.15. A 62 year old man with stage 3 CKD
his temperature is 38.2°C, BP is 164/92 mmHg (eGFR 39 mUmin/1 .73 rn 2) is noted to have
and he has bilateral ankle oedema, but no other haemoglobin of 79 g/L, white cell count
clinical signs. The following investigation results 8.9x109/L; platelet count 146x109/L; mean
are obtained: urea 36 rnmoi/L (216 mg/dl); corpuscular volume (MCV) 76 fl. What is the
creatinine 640 J.!moi/L (7.24 mg/dl); sodium most appropriate investigation?
129 mmoi/L; potassium 6.4 mmoi/L; A. Bone marrow biopsy
haemoglobin 64 g/L; white cell count B. Serum erythropoeitin level
9.6x 109/L; platelet count 36x 109/L; blood film C. Serum folate studies
shows schistocytes; urinalysis: blood 1+, protein D. Serum iron studies
negative; stool cultures negative for Escherichia E. Ultrasound scan of-abdomen /
coli 0157. What is the most likely 11
diagnosis? 15.16. The Reciprocal creatinine plot shown of b
A. Haemolytic uraemic syndrome 48 year old man would be consistent with the\
B. Lupus nephritis natural history of progression of which of the
C. Malignant hypertension following causes of kidney failure? I'
I

71.----------------------------------------------------.

Q)
'iii
()
(/)

'iii
()
e
Q. 100
'(3
~
c
0
.: 125
ro
0
E 150
e
()
.E 175
Q) 200
c 1: I
·c:
250
11:

~ 300
~
()
400
500
700
1000
16.05.1991 11.05.1995 06.05.1999 01.05.2003 26.04.2007 21.04.2011

downloaded from www.medicalbr.com


I I 8 • NEPHROLOGY AND UROLOGY

A. Adult polycystic kidney disease dl), increased from 126 iJ.mOI/L (1.43 mg/dl) 3
T
B. Microscopic polyangiitis months previously). Urinalysis: blood 1+, protein
C. Multiple myeloma 2+, no leucocytes; ultrasound scan of graft
D. Post -infectious glomerulonephritis revealed no hydronephrosis. What is the most
E. Renovascular disease likely explanation for the deterioration in renal
function?
15.17. A 42 year old woman with lgA A. Acute pyelonephritis
nephropathy and stage 3 CKD (eGFR 45 mU B. Acute rejection due to non-adheren'ce with
min/ I. 73 m 2 ) is developing proteinuria immunosuppression
(protein: creatinine ratio is I 20 mg/mmol). BP is C. Anti-glomerular basement membrane disease
I 58/86 mmHg and she is commenced on an D. Chronic allograft injury
ACE inhibitor (lisinopril I 0 mg daily). Two weeks E. Thrombosis in the artery to the graft
later her eGFR has fallen to 37 mUmin/1 .73 m2
and her potassium has risen from 5.2 to 15.21. A previously fit 17 year old male presents
5.9 mmoi/L, although BP and protein: creatinine with a 2- to 3-week history of arthralgia and
ratio have fallen to 146/82 mmHg and 30 mg/ more recently has developed a skin rash on the
mmol, respectively. She is already on a lower legs. Just prior to admission to hospital
low-potassium diet. What is the most he developed abdominal discomfort with
appropriate management? blood-stained stool. On examination, he has a
A. Add a thiazide diuretic widespread non-blanching rash over his limbs.
B. Add a ~-adrenoceptor antagonist (~-blocker) Initial investigations reveal: urinalysis: blood 3+;
C. Commence calcium resonium protein 3+; eGFR 46 mUmin/1 ,73 m 2 ;
D. Increase the lisinopril dose protein: creatinine ratio 220 mg/mmol;
E. Stop the lisinopril haemoglobin 120 g/L, white cell count
12.9x 109 /L; platelet count 259x 109 /L;
15.18. Which of the following is true regarding C-reactive protein 62 mg/L. What is the most
peritoneal dialysis? likely diagnosis?
A. Fluid removal is achieved by increasing the A. Anti-glomerular basement membrane di sease
concentration of sodium in the dialysate
B. Hyperkalaemia is less common than for
B.
C.
Haemolytic uraemic syndrome
Henoch-Schonlein purpura
7
;!
haemodialysis D. Post -streptococcal glomerulonephritis f
C. It is associated with improved patient survival E. Systemic lupus erythematosus ;,
compared with haemodialysis
D. It is unsuitable for elderly patients 15.22. A 62 year old man presents witlil' sudden
E. Peritonitis is usually caused by gut bacteria anuria on a background history of sev~ral
traversing the bowel wall weeks of 'not passing much urine'. He denies
dysuria or haernaturia but admits to having
15.19. Which of the following is typical of the a poor stream for many years. He is
development of pre-eclampsia? normotensive and otherwise looks well and has
A. Low serum urate level no systemic symptoms. What is the best initial
B. Maternal history of cigarette smoking diagnostic investigation?
C. Occurrence in the mother's first pregnancy A. Blood test for electrolytes and renal function
D. Onset of hypertension in the second B. CT of kidneys and urinary tract with contrast
trimester C. Renal biopsy
E. Prolonged prothrombin time D. Renal ultrasound scan
E. Urinalysis for red cell casts
15.20. A 14 year old boy with end-stage renal
disease due to reflux nephropathy received a 15.23. An 18 year old male presents with
renal transplant from his mother. Aged 17 he haematuria and proteinuria. He undergoes renal
transferred to the adult renal service and he left biopsy which shows a mesangiocapillary
home to go to university the following year. Six glomerulonephritis !Olattern of injury.
months later he attends the transplant clinic. Immunofluorescence shows complement C3
He is asymptomatic, but his graft function has staining with no immunoglobulin depqsition.
deteriorated (creatinine 297 llmoi!L (3.36 mg/ Electron microscopy, demonstrates

downloaded from www.medicalbr.com


NEPHROLOGY AND UROLOGY • 119

electron-dense deposits in a ribbon-like pattern dL), potassium 6.8 mmoi/L, corrected calcium
in the glomerular basement membrane (so 1.97 mmoi/L (7.90 mg/dL), international
called 'dense deposits'). What is the most likely normalised ratio (INR) 2.0. Urine dipstick shows
underlying cause of his mesangiocapillary haematuria but no proteinuria. Direct urinalysis
glomerulonephritis? revealed no cells or casts. What is the likely
A. Autoimmune disease cause of his kidney injury?
B. Genetic defect of alternative complement A. Acute interstitial nephritis
pathway B. ATN due to viral infection
C. Hepatitis B infection C. Haemorrhage into the kidneys
D. Hepatitis C infection D. Pre-renal injury due to dehydration from
E. Monoclonal gammopathy alcohol
E. Rhabdomyolysis
15.24. A 49 year old woman presents with
acute kidney injury after an acute illness 15.27. A patient with acute kidney injury has
manifested by myalgia, diarrhoea and vomiting. been anuric for 12 hours despite fluid
Her BP is 84/50 mmHg and she has dry challenges. Potassium is 5.2 mmoi/L, urea is
mucous membranes. She was taking ibuprofen, very high and a pericardia! rub is audible. The
paracetamol and domperidone during the patient appears euvolaemic. A decision is made
illness. Her renal function improves rapidly with to commence haemodialysis due to concerns
intravenous (IV) fluids. Which one of the regarding uraemia and specifically uraemic
following findings are likely to be present? pericarditis. What will the first dialysis session
A. Dense granular ('muddy brown') casts on involve?
urinalysis A. A large surface area dialyser
B. Hypercalcaemia B. A short 2-hour session initially
C. Hyponatraemia C. Heparin anticoagulation
D. Low (< 1 %) fractional excretion of sodium D. High blood flow rate of 400 mUmin
E. Low urine specific gravity E. Ultrafiltration of 2 L (fluid removal)

15.25. A 68 year woman develops malaise and 15.28. In a patient presenting with renal
a low-grade fever. She has no rash and impairment, which of the following is most
appears euvolaemic. She takes atorvastatin, helpful in discriminating between AKI and a lat.e
omeprazole, amlodipine and digoxin regularly presentation of CKD? )
and takes ibuprofen intermittently. Urinalysis A. Anaemia
shows some leucocytes but no casts, B. Hyperphosphataemia ;'
I
haematuria or proteinuria. She has a creatinine C. Hyponatraemia
of 320 11moi/L (3.62 mg/dL), which has been D. Renal biopsy showing interstitial fibrosis and
68 11moi/L (0.77 mg/dL) 1 year previously. tubular atrophy
What is the likely cause of renal injury? E. Small echogenic kidneys on ultrasound
A. Acute interstitial nephritis
B. ATN due to rhabdomyolysis 15.29. A 32 year old man with lgA nephropathy
C. Glomerulonephritis since the age of 18 received a well human
D. Pre-renal injury due to NSAIDs leucocyte antigen (HLA)-matched kidney
E. Urinary obstruction transplant from his older brother. He had no
pre-formed anti-HLA antibodies and the kidney
15.26. A 55 year old man with significant functioned immediately. One week later his
cardiovascular disease and diabetes has acute urine output is noted to be lower than the
kidney injury in the context of a viral illness. He previous days and his. creatinine is increased,
was at a social gathering where he consumed having previously droppetto normal in the
alcohol and woke the next morning unwell. He first few days post-transplant. His BP is
had fever, aches and pains, headache and felt 180/90 mmHg, he has dipstick-positive blood
thirsty. He takes atorvastatin, lansoprazole, on urinalysis and he look§ euvolaemic. What is
amlodipine, bisoprolol, warfarin, digoxin the likely diagnosis?
regularly. He passed a small amount of dark A. Acute cellular rejection
urine. His creatinine is 190 11moi/L (2.15 mg/ B. BK polyomavirus nephropathy

downloaded from www.medicalbr.com


120 • NEPHROLOGY AND UROLOGY

G. Hyperacute rejection
D. Recurrent lgA nephropathy
A. A granulomatous interstitial nephritis
B. Calcium deposition in the tubules
T
E. Renal artery stenosis G. Focal segmental glomerulosclerosis (FSGS)
D. Necrotising cresentic glomerulonephritis
15.30. A 56 year old woman with polycystic E. Widespread interstitial fibrosis and tubular
kidney disease received her second kidney atrophy
transplant. She had pre-formed anti-HLA
antibodies (from the first transplant) but the 15.33. Patients with advanced liver disease are
cross-match was negative so she proceeded at risk of developing AKI, termed hepatorenal
to transplant using induction therapy syndrome. Which of the following is true of this
(anti-thymocyte globulin; ATG). She had syndrome?
immediate function of the transplant but A. Aggressive dialysis may prevent hepatic
suffered an acute rejection after 2 months, encephalopathy
which was successfully treated with IV B. lgA deposition is a common cause
glucocorticoids. She developed a urinary tract G. Kidney biopsy should be performed for an
infection (UTI) in the week after the steroids accurate diagnosis
were administered, which cleared with D. Outcomes are good with haemodialysis
oral antibiotics. Her renal function has E. The aetiology is haernodynamically mediated,
deteriorated again at 4 months and her so urine sodium will be reduced
serum shows BK polyomavirus on polymerase
chain reaction testing. A biopsy reveals 15.34. Which of the following is true in diabetic
BK polyomavirus nephropathy. Risk factors nephropathy?
for BK polyomavirus include which of the A. ACE inhibitors generally cause resolution of
following? proteinuria and stabilisation of renal function
A. Augmented immunosuppression (ATG and B. Biopsy is generally needed to confirm the
high-dose steroids) diagnosis
B. Polycystic kidney disease G. It is an uncommon cause of end-stage renal
G. Presence of anti-HLA antibodies disease (ESRD) outside of North America
D. Previous UTI D. Sodium-glucose co-transporter-2 (SGLTi)
E. Second transplant inhibitors, such as ernpagliflozin, may bEll
associated with improved cardiovasculcit /,
and
15.31. What is the pathogenesis of 'myeloma renal outcomes and work by improving'·
kidney' (cast nephropathy)? insulin sensitivity
A. Glomerular light chain deposition due to light E. The natural history is of slow development of
chains, and rarely heavy chains, often giving rnicroalbuminuria over years, with ov~rt
a nodular pattern of injury proteinuria and renal impairment at a late
B. Light chain misfolding, creating glomerular stage
deposits that are Congo red positive
G. Light chains precipitating with Tamm-Horsfall 15.35. A 23 year woman presents with a facial
protein in the tubular lumen rash and arthralgia soon after getting married.
D. Proximal tubular injury and dysfunction due She is found to have an eGFR of 106 mU
to light chain deposition in tubular epithelial min/1.73 m 2 , red cell casts in her urine and
cells 5.5 g/24 hrs of proteinuria. Renal biopsy
E. Tubular damage due to hypercalcaemia confirms lupus nephritis. Which of the following
is true in this patient?
15.32. A patient with known sarcoidosis has A. Best treatment for this patient is with
developed renal impairment over the past 2 cyclophosphamide gnd glucocorticoids
months. Corrected serum calciom is slightly B. Mycophenolate mofetil would be the
high (2.7 rnrnoi/L; 10.82 mg/dl). A renal biopsy induction agent of choice, along with
is performed and glucocorticoids are glucocorticoids
commenced. Renal function gradually G. She probably has.mild lupus nephritis that
normalised over a period of several weeks. can be managed with an ACE inhibitor alone
What would the likely initial renal biopsy findings D. She should be referred immediately to the
show? transplant team

downloaded from www.medicalbr.com


NEPHROLOGY AND UROLOGY • 121

E. She would be at high risk for recurrence D. Ruptured berry aneurysm


after a renal transplant causing allograft E. Ruptured hepatic cyst
loss
15.40. A 65 year man presents with flank
15.36. A 42 year old woman from China discomfort and haematuria. He is found to have
presents with slowly progressive renal failure in bilateral polycystic kidneys and some liver cysts
the context of taking herbal remedies for years on ultrasound scan. His renal function is
containing aristolochic acid. Which of the normal, with an eGFR of 90 mUrnin/1.73 m 2 •
following clinical characteristics is likely in this He gives a family history on his mother's side
patient? of 'cysts in the kidney' but no family member
A. Biopsy demonstrating focal segmental ever needed dialysis or a transplant. Which of
glomerulosclerosis the following is true about his disease?
B. Bland urine sediment with interstitial fibrosis A. Any offspring have a 50% chance of
on biopsy developing the condition
c. Heavy proteinuria in the nephrotic range B. He is a good candidate for tolvaptan to slow
D. High anion gap metabolic acidosis cyst growth
E. Large kidneys on ultrasound scanning C. He is at high risk of liver failure due to a
polycystic liver
15.37. A 22 year old man develops diabetes D. He probably has a mutation in PKD1
and is found to have renal impairment with E. He will probably develop end-stage renal
small kidneys on scanning, in addition to noted disease within 5 years
pancreatic atrophy. He has a history of gout for
2 years and his father developed ESRD aged 15.41. A 75 year old man with hypertension,
38. His father also had 'renal failure' due to heart failure, peptic ulcer disease and
cystic disease and has diabetes. A mutation in osteoarthritis presents with acute kidney injury
which gene is likely? after being prescribed ibuprofen. His usual
A. COL4A5 medicines are lisinopril, furosemide, omeprazole
B. HNF1-beta and atorvastatin. He has 1+ proteinuria and no
C. PKD1 haematuria on dipstick. What is the mechanism
D.PKD2 underlying his renal failure?
E. UMOD A. Afferent arteriolar vasoconstriction with
ibuprofen (in context of efferent vasodilatation
15.38. In patients with Alport's syndrome with lisinopril)
(hereditary nephritis), which of the following B. Afferent arteriolar vasoconstriction with 1
statements is true? lisinopril (in context of efferent vasodilatation
A. After kidney transplant, patients may develop with ibuprofen)
anti-GBM disease C. Afferent arteriolar vasodilatation with
B. All patients harbouring pathogenic COL4A ibuprofen (in context of efferent
mutations develop progressive chronic vasoconstriction with lisinopril)
kidney disease D. Afferent arteriolar vasodilatation with lisinopril
C. Deafness may occur due to otosclerosis (in context of efferent vasoconstriction with
D. Female carriers of X-linked disease (COL4A5 ibuprofen)
mutations) do not manifest disease E. Rhabdomyolysis from the statin
E. It is always an X-linked condition
15.42. A 64 year old man has osteoarthritis in his
15.39. A 20 year old woman has a history of knees and is prescribed ibuprofen regularly for 3
autosomal dominant polycystic kidney disease months. He notices some swelling in his ankles
(APKD) on her father's side. Her paternal and his family physician fil)ds his urine dipstick
grandmother and her father's brother both died reveals 4+ protein with r16 blood or white cells.
suddenly in their 50s. What was the likely His creatinine is normal and his serum albumin
cause of death? is 190 g/L. A renal biopsY.. is performed. What is
A. Mitral valve prolapse the light microscopy likely to reveal?
8. Myocardial infarction A. Fibrin microthrombi in glomerular capillary
C. Pulmonary embolism loops

downloaded from www.medicalbr.com


122 • NEPHROLOGY AND UROLOGY

B. Intense interstitial inflammation, with


infiltration of the tubules by neutrophils,
lymphocytes and some eosinophils
C. Necrotising cresentic glomerulonephritis
D. Normal glomeruli
15.47. A 27 year old woman presents as an
emergency with rigors, flank pain and fever.
Non-contrast CTKUB reveals an 8-mm stone in
the left mid-ureter. Which is the optimal
management option?
T
E. Tubular dilatation, breaks in the tubular A. Extracorporeal shockwave lithotripsy (ESWL)
basement membrane, interstitial oedema and B. Percutaneous nephrolithotomy (PCNL)
sloughing of necrotic tubular cells into the C. Ureteric stent insertion
tubular lumen D. Ureterolysis
E. Ureteroscopy and laser fragmentation of
15.43. Regarding micturition, which of these stone
statements is correct?
A. A low-compliance bladder is required for 15.48. In which of the following situations would
voiding to be initiated you consider treating an asymptomatic patient
B. Contraction of the pelvic floor commences identified to have > 10 5 E. co/i/ml urine?
micturition A. Healthy 14 year old girl
C. Micturition is initiated when the compliance B. 24 year old woman, normal ultrasound and
limit of the bladder is reached flexible cystoscopy in the past
D. Voiding is controlled by the cerebellum C. 32 year old pregnant woman
E. Voiding is coordinated by the pontine D. 67 year old man with a urethral catheter in
micturition centre situ
·E. 78 year old woman with a ureteric stent in
15.44. What is the optimal imaging to rule out place for retroperitoneal fibrosis
bone metastases in a man with prostate
cancer? 15.49. Following a trial of treatment with
A. Contrast-enhanced CT urogram a-adrenoceptor antagonist (a-blocker)
B. Dimercaptosuccinic acid (DMSA) static medication, a 65 year old man is referred by
radionuclide scan his family physician to urology with poor f~9w.
C. Non-contrast CT of kidneys, ureters and terminal dribbling and hesitancy. Which o~;he
bladder (CTKUB) following is the most relevant investigatior?
D. Pelvic magnetic resonance imaging (MRI) A. Cystoscopy /!
scan B. MRI pelvis
E. Technetium-labelled methylene C. Prostate biopsy
diphosphonate (99"'Tc-MDP) radionuclide D. Ultrasound prostate
scan E. Urinary flow test

15.45. A 25 year old woman from Uganda who 15.50. A 49 year old woman presents with
has recently delivered a baby presents with visible haematuria. A cystoscopy is normal, but
new continuous incontinence. What is she likely a contract-enhanced CT scan of chest,
to be suffering with? abdomen, pelvis reveals a 17 -em left renal
A. Duplex kidney with insertion of upper pole mass, consistent with a renal cell cancer. What
moiety into the vagina is the best treatment option for this woman?
B. Overflow incontinence A. Cryotherapy
C. Stress urinary incontinence B. External beam radiotherapy
D. Urge incontinence C. Open radical nephrectomy
E. Vesicovaginal fistula D. Robotic partial nephrectomy
E. Tyrosine kinase in~ibitor (TKI)
15.46. What is the most likely Gause of painless,
visible haematuria in a 60 year old man?
/
15.51. A 72 year old fit ex-smoking man is
A. Ureteric stone identified on flexible cystoscopy to have a 4-cm
B. Bladder cancer bladder tumour. C)'Stoscopy and transurethral
C. lgA nephropathy resection of bladder tumour provides tissue
D. Systemic lupus erythematosus that on pathological examination shovys a
E. Upper urinary tract urothelial cancer G3pT2 urothelial cell cancer. What is the
I

downloaded from www.medicalbr.com


NEPHROLOGY AND UROLOGY • 123

optimal management for this muscle-invasive 15.55. In a 67 year old man with benign
cancer? prostatic hypertrophy (BPH) who has a large
prostate (70 cc) and is already treated with an
A. Brachytherapy
B. Chemotherapy (gemcitabine and cisplatin) a-blocker but with ongoing bothersome
c. Observation with regular flexible cystoscopy symptoms of hesitancy and poor flow, which of
D. Partial cystectomy the following options is most appropriate?
E. Radical cystectomy A. 5a-reductase inhibitor such as finasteride
B. High-intensity focused ultrasound therapy
15.52. A healthy 81 year old man presents C. Open prostatectomy
with back pain to his family physician. A PSA D. Robot-assisted laparoscopic radical
is undertaken, which measures 2350 ng/ml. prostatectomy
The patient is referred to a urologist who E. Transurethral resection of the prostate
identifies a craggy, hard prostate gland and
undertakes a bone scan, which shows multiple 15.56. A 81 year old man attends as an
bone metastases. What is the best treatment emergency having passed nothing more than
option for this man? 50 ml of urine for 2 days. He has nocturnal
A. Active surveillance enuresis, a palpable bladder and a creatinine of
B. External beam radiotherapy to pelvis 378 [!moi/L (4.28 mg/dl). What is the most
C. Gonadotrophin-releasing hormone (GnRH) appropriate initial management?
agonist therapy A. Bilateral ureteric stent insertion
D. High-frequency focused ultrasound B. Haemodialysis
E. Radical prostatectomy C. Start an a-blocker, i.e. tamsulosin
D. Transurethral resection of the prostate
15.53. What is the most appropriate set of E. Urethral catheterisation
investigations for a 71 year old male smoker who
presents with dysuria and the family physician 15.57. A 54 year old female has stress
identifies persistent non-visible haematuria? incontinence proven by urodynamics. What is
A. DMSA static scan, mid-stream urine (MSU) the most appropriate initial management?
for microbiology culture, renal tract A. Anticholinergic medication
ultrasound B. Botulinum neurotoxin type A
B. MRI pelvis and MSU C. Pelvic floor exercises
C. MSU, flexible cystoscopy, renal tract D. Sacral nerve stimulation
ultrasound E. Tension-free vaginal tape
D. Nil, only investigate when visible haematuria {
E. Non-contrast CTKUB, transrectal ultrasound 15.58. Which of the following statements is 'true
scan and biopsy regarding erectile dysfunction (ED)?
A. lntracavernosal alprostadil should be
15.54. An 18 year old male presents with considered as a first-line treatment option
long-standing mild left testicular pain, with a B. Perineal trauma is the most common cause
hard 1-cm lump in the testicle. What is the C. PSA should be checked in all men
most appropriate course of action? D. Pudendal artery angiography is useful in early
A. Analgesia and observation assessment
B. CT scan E. Risk factors for cardiac disease should be
C. Intravenous antibiotics and observation assessed
D. Nuclear medicine scan
E. Scrotal ultrasound

Answers
15.1. Answer: D. heart sound) or chronic liver disease and there
He has no clinical evidence of heart failure (JVP is no history of obstructive urinary symptoms;
not elevated, no basal crepitations, no third therefore there is no indication to perform

downloaded from www.medicalbr.com


124 • NEPHROLOGY AND UROLOGY

echocardiogram/abdominal ultrasound at this


point. Bilateral leg swelling is unlikely to be due
implies a glomerular rather than a
tubulo-interstitial disease process and the
T
to a deep vein thrombosis (DVT), unless it absence of haematuria renders a
affects the inferior vena cava. This raises the glomerulonephritis such as lgA nephropathy
possibility of either renal failure or nephrotic unlikely. The rapid rise in proteinuria is too
syndrome and urinalysis may be helpful in sudden to be accounted for by diabetic
either circumstance. nephropathy alone. Amyloid is less likely, as it
is associated with rheumatoid arthritis and not
15.2. Answer: C. osteoarthritis, and the rapid onset of nephrosis
Red cell casts indicate the presence of a in a relatively young man would be atypical for
glomerulonephritis and are not observed in amyloid.
tubulo-interstitial disease or haemolytic uraemic
syndrome/sclerodermic renal crisis. 15.6. Answer: B.
The proteinuria renders bladder cancer,
15.3. Answer: D. polycystic kidney disease and renal calculi less
Small molecules such as glucose, amino acids likely. Furthermore, bladder cancer would be
(glutamine) and lithium are freely filtered. Most rare in this age group. Post-infectious
free light chains are also filtered and may be glomerulonephritis typically presents with
taken up by tubules, causing tubular damage. non-visible haematuria after the infection is
Immunoglobulins are too big to cross the resolved. In addition, hypertension and renal
normal glomerular barrier, but may do so in failure are common. Visible haematuria is a
nephrotic syndrome, leading to increased risk common presentation of lgA nephropathy,
of infection. typically during an upper respiratory tract
infection. The haematuria settles spontaneously
15.4. Answer: D. and the renal prognosis is typically good.
The MDRD equation estimates GFR
based on the serum creatinine level, and hence 15.7. Answer: C.
it will be inaccurate in patients whose muscle
bulk is atypical for someone of that sex and
age. The body builder and African American
I
Acute tubular necrosis due to gentamicin and
interstitial nephritis due to amoxicillin and
pre-renal failure related to sepsis are all 1
male will have greater muscle bulk and hence common in this scenario; however, the 3+/'
higher creatinine for a given level of renal blood and 3+ protein on urinalysis would 16oint
function compared to what would be expected towards a glomerulonephritis. Microscopic
for a sedentary Caucasian male; hence the polyangiitis is a possibility, but this is not
MDRD eGFR will underestimate the true GFR associated with low complement levels(
(for this reason a correction factor of 1.21 which are observed in infection-related
should be applied to the eGFR in those of glomerulonephritis.
African American descent). Trimethoprim
competes with creatinine for excretion in the 15.8. Answer: A.
distal tubule and hence will increase serum This is a presentation of nephrotic syndrome,
creatinine; thus the MDRD eGFR will which is consistent with amyloid or minimal
underestimate true GFR. Loss of muscle bulk change disease. The pancytopenia could not
following amputation will lead to a lower be explained by minimal change disease, but
creatinine and hence the MDRD equation raises suspicion of a bone marrow disorder
will overestimate the true GFR. The MDRD such as myeloma. While myeloma could cause
eGFR should approximate to true eGFR in the cast nephropathy, this would present with AKI
elderly woman with chronic kidney disease rather than nephrotic syndrome. While
(CKD). haemolytic uraemic SYJldrome (HUS)ffiP may
cause low platelets)hey do not cause
15.5. Answer: D. pancytopenia and do not present with
Minimal change disease classically presents nephrotic syndrome'
with sudden onset of nephrotic syndrome and
is associated with consumption of NSAIDs. 15.9. Answer: A.
Although NSAIDs may also cause The presence of haemoptysis and kid~ey injury
tubulo-interstitial nephritis, the heavy proteinuria indicates a pulmonary renal syndrome, most

downloaded from www.medicalbr.com


NEPHROLOGY AND UROLOGY • 125

commonly due to granulomatosis with 15.12. Answer: A.


polyangiitis (previously known as Wegener's The dusky toes (sometimes called trash
granulomatosis), anti-glomerular basement foot) raise clinical suspicion of cholesterol
membrane disease or lupus. Pulmonary emboli in the microvasculature (especially
embolus may cause haemoptysis, but it would if peripheral pulses are intact) and this
not explain the renal failure in the context of diagnosis is supported by the low-grade
hypertension. While Alport's disease can cause fever and eosinophilia. Contrast nephropathy
deafness, it does not account for the is the other main differential diagnosis;
haemoptysis, nor the acute nature of the however, it would not account for the trash
process. Renal biopsy is likely to be required, foot or eosinophilia, nor would renal artery
but the risk of bleeding is very high at this point thrombosis. Although the creatine kinase
due to hypertension and uraemia. Serological is elevated, either due to myocardial ischaemia
testing should be performed urgently given the or mild leg muscle damage, at this level
high risk of one of the above causes of there is likely to be insufficient myoglobinuria
pulmonary renal syndrome (most likely to cause AKI. While low platelets and
granulomatosis with polyangiitis given the AKI are consistent with haemolytic uraemic
deafness). syndrome, the haemoglobin is only mildly
reduced and this does not fit the clinical
15.10. Answer: C. picture.
Asymptomatic non-visible haematuria is a
common presentation of lgA nephropathy. 15.13. Answer: A.

--
Alport's disease is a possibility, although the The combination of low haemoglobin, low
absence of deafness and a family history of platelets and schistocytes on blood film
renal disease renders this less likely. suggest microangiopathic haemolytic anaemia,
Membranous nephropathy presents with which may be due to a number of conditions,
nephrotic syndrome and vesica-ureteric reflux including haemolytic uraemic syndrome or l
would rarely cause isolated haematuria with no thrombotic thrombocytopenic purpura. The
evidence of proteinuria or CKD. Ultrasound antecedent bloody diarrhoea and predominant
scan and cystoscopy to exclude uroepithelial renal versus neurological complications are I
tumour would need to be considered if he were
over 40 years old.
consistent with HUS rather than TIP. The
negative E. coli 0157 stool cultures do not rul~''
I
out HUS as they have been taken after the [
15.11. Answer: B. diarrhoeal phase of the illness. Malignant
The Study of Heart and Renal Protection hypertension may also cause microangiopathic
(SHARP) provides evidence for reduced haemolytic anaemia; however, the blood /
cardiovascular events with statins in patients pressure is typically much higher than observed
with CKD with or without renal artery here. Scleroderma renal crisis, but not lupus,
disease. The patient's renal function is stable may cause microangiopathic haemolytic
and blood pressure is well controlled and anaemia and AKI. While vomiting and diarrhoea
she has proteinuria, and therefore her lisinopril predispose to pre-renal failure, his high blood
should be continued; however, she should pressure and leg swelling would indicate that
be informed to discontinue lisinopril he is hypervolaemic, not hypovolaemic.
transiently should she develop vomiting,
diarrhoea or fever. The Angioplasty and 15.14. Answer: E.
Stenting for Renal Artery Lesions (ASTRAL) High serum calcium due to excess calcium or
and Cardiovascular Outcomes in Renal vitamin 0 consumption should suppress the
Atherosclerotic Lesions (CORAL) trials have PTH level. The PTH level here is inappropriately
not found any benefit from renal artery elevated, indicating hyperp9rathyroidism. Serum
revascularisation in this context and·similarly phosphate should be low.in primary
there is no evidence for the use of warfarin. hyperparathyroidism. In patients with CKD,
Plasma renin activity does not help discriminate calcium is initially maintained in the normal
those who might benefit from angioplasty range by elevated PTH (q_econdary
and will be difficult to interpret in the hyperparathyroidism); however,. as here,
context of angiotensin-converting enzyme (ACE) eventually the gland may become autonomous
inhibition. and the PTH level will be very high, resulting in

downloaded from www.medicalbr.com


1 26 • NEPHROLOGY AND UROLOGY

an elevated serum calcium concentration


(tertiary hyperparathyroidism).
post-dialysis. Fluid removal is achieved by
altering the glucose concentration in the
T
dialysate. Peritoneal dialysis may be the most
15.15. Answer: D. appropriate modality for renal replacement
While erythropoietin (EPO) deficiency is therapy for elderly patients who may not
common in patients with chronic kidney tolerate the fluid and electrolyte shifts
disease, the haemoglobin level here is associated with haemodialysis. There is no
disproportionately low for this level of renal evidence of a survival benefit when
failure. Haemoglobin < 10 g/L is not usually haemodialysis and peritoneal dialysis have been
observed until stage 4 CKD. Serum EPO levels compared, although transplantation does
may be difficult to interpret in this context, confer improved survival. Peritonitis is typically
although if they are low or indeed normal, this caused by skin contaminants translocating
is inappropriate in the context of anaemia and through the lumen or along the tract of the
makes renal EPO insufficiency more likely. The peritoneal catheter.
MCV is low, indicating potential iron rather than
folate deficiency, and white cell and platelet 15.19. Answer: C.
counts are normal, rendering a bone marrow Pre-eclampsia is more common in first
problem or hypersplenism less likely. pregnancies or first pregnancy with a new
partner. Serum urate level may be elevated,
15.16. Answer: A. which may be helpful in diagnosis.
The slow and very consistent rate of decline in Pre-eclampsia typically presents in the third
renal function illustrated here is consistent with trimester, and onset of hypertension prior to
polycystic kidney disease. Post-infectious this raises the possibility of pre-existing renal
glomerulonephritis is rapidly progressive, and disease. Maternal history of smoking may
microscopic polyangiitis is also typically more actually reduce the risk of pre-eclampsia.
rapidly progressive than here and may be Prolonged prothrombin time suggests the
associated with remissions and relapses. development of disseminated intravascular
Progression of renovascular disease typically coagulation.
occurs in a step-wise manner. Myeloma
typically affects an older age group and does
not explain slow progression over 20 years.
15.20. Answer: B.
I
Unfortunately, the age group with the lovyest
graft survival includes adolescence. The/
15.17. Answer: A. transition from paediatric to adult care and to
There is good evidence that ACE inhibitors more independent living away from the parental
are the drug of choice to treat hypertension home is a high-risk period for non-adherence.
and reduce proteinuria in patients with The rate of decline in renal function here is too
CKD and protein: creatinine ratio > 100 mg/mmol, rapid to be explained by chronic allograft
and initiation of lisinopril has been partially nephropathy. The absence of symptoms or
effective in this patient. The fall in eGFR of leucocytes in the urine makes acute
<20% is acceptable and all alternative pyelonephritis in the graft unlikely. Graft
measures should be taken to reduce potassium thrombosis is rare outside of the early
before stopping the ACE inhibitor. Calcium transplant phase or during very severe
resonium is only suitable for short-term dehydration. Anti-GBM disease may
management of hyperkalaemia due to risk of occur in patients with Alport's disease who
bowel perforation. While BP is suboptimal, receive a kidney with a normal collagen IV
increasing the lisinopril or adding a 13-blocker isoform.
are not recommended at this level of
potassium. A thiazide would be more 15.21. Answer: C. I
appropriate as this will have the combined A purpuric rash wi!Ji renal impairment,
benefit of reducing BP and lowering potassium. abdominal and joint pain is typical of Henoch-
Schonlein purpura. Haemoglobin and platelets
15.18. Answer: B. are normal; therefare haemolytic uraemic
Hyperkalaemia is less common than for syndrome is unlikely. Anti-glomerular basement
haemodialysis where potassium oscillates from membrane disease, post-streptococc:al
high values pre-dialysis to low values glomerulonephritis and systemic lupus

downloaded from www.medicalbr.com


NEPHROLOGY AND UROLOGY • 127

erythematosus could account for the renal 15.26. Answer: E.


failure and urinary findings, but not the Rhabdomyolysis is the most likely diagnosis
purpura. given the dark urine and risk factors, including
a statin, viral illness and alcohol use. Many
15.22. Answer: D. cases will have several risk factors and
Anuria in this setting is probably caused by rhabdomyolysis may occur after being on a
bladder outflow obstruction: hence an statin for some time. The dipstick 'haematuria'
ultrasound is the correct answer. A CT scan without red cells being visible is due to urine
would likely diagnose this too, but ultrasound is myoglobin. Intracellular ions (potassium,
the best, quickest and cheapest test. A phosphate) tend to be particularly high with
catastrophic vascular event is a less common rhabdomyolysis and calcium may be low
cause in which a contrast CT may be helpful. (precipitates with phosphate).
Red cell casts could indicate a rapidly
progressive glomerulonephritis, although this is 15.27. Answer: B.
much less common. Bloods for urea and The first dialysis is designed to be a short,
electrolytes will not be helpful in diagnosis, incomplete treatment due to the risks of dialysis
although they should obviously be performed, disequilibrium syndrome if the uraemia is
and a biopsy should not be needed if the corrected too quickly. Therefore a short session
cause is obstruction. is performed, using a small surface area
dialyser with low blood and dialysate flows.
15.23. Answer: B. Anticoagulation is generally not used for the
A mesangiocapillary glomerulonephritis pattern first session, as a dialysis catheter will recently
of injury has two broad causes based on the have been placed, and in this case also due to
immunofluorescence findings: complement concerns regarding uraemic pericarditis, which
deposition, which is caused by inherited may be haemorrhagic precipitating tamponade.
alternative pathway complement gene
mutations with unregulated complement 15.28. Answer: E.
activation; and immunoglobulin deposition,
which may be caused by chronic infections
(frequently viral hepatitis), autoimmune diseases
Hyperphosphataemia and anaemia are
frequently considered signs of likely CKD but
they may occur early in the course of AKI (and ,
I
and monoclonal gammopathy. 'Dense deposit are common in certain causes of AKI such as ;'
disease', with a mesangiocapillary pattern of haemolytic uraemic syndrome) so are not very[
injury, is due to inherited complement helpful discriminators. Hyponatraemia has no ·
mutations. value in this setting. Small kidneys are a
non-invasive, helpful indication of chronic rEJhal
15.24. Answer: D. injury. It is rarely necessary to perform a renal
This is pre-renal injury, without evidence of biopsy for the presence of interstitial fibrosis
acute tubular necrosis (ATN), as renal function and tubular atrophy, which will be a universal
improved fully with fluids. Therefore she would finding in small atrophic kidneys.
likely manifest low urine sodium, low urine
fractional excretion of sodium and concentrated 15.29. Answer: A.
urine (high specific gravity). Dense granular Acute cellular rejection is commonest from day
casts would probably be present in ATN. 6-7 to week 12 post-transplant. Hyperacute
There is no particular reason she should be rejection is rare with modern cross-matching
hyponatraemic or hypercalcaemic. techniques and occurs immediately
post-transplant and this patient had no
15.25. Answer: A. preformed anti-HLA antibodies. Renal artery
This patient likely has allergic acute interstitial stenosis manifests after several months with
I
nephritis due to her proton pump intlibitor hypertension and slowly,98teriorating transplant
(omeprazole) as she has a mild fever and function. BK polyomaviriJs nephropathy may
her urine has some white cells but nothing occur as early as 1-2 months post-transplant
else to suggest glomerulonephritis. Pre-renal but not this early. Recurri)nt lgA nephropathy
injury/ATN is a possibility but, given happens often but is often not clinically
euvolaemia and lack of an apparent insult, it is significant and would perhaps be a late cause
unlikely. of transplant dysfunction. Causes not listed that

downloaded from www.medicalbr.com


128 • NEPHROLOGY AND UROLOGY

would need to be ruled out include a urine leak


causing obstruction and a vascular thrombosis
continuous treatments are better tolerated with
a reduced risk of precipitating encephalopathy.
T
(transplant artery or vein). lgA nephropathy is associated with chronic liver
disease but is not the cause of hepatorenal
15.30. Answer: A. syndrome.
BK polyomavirus causes an interstitial nephritis
in renal transplant patients. It appears to be 15.34. Answer: E.
much less common in non-renal solid organ SGLT2 inhibitors work by inducing glycosuria
recipients. It appeared as an entity in the era of via impaired glucose reabsorption at the
modern immunosuppression with tacrolimus proximal tubule. Diabetic nephropathy is the
and mycophenolate. Risk factors are commonest cause of ESRD in the developed
augmented immunosuppression such as ATG world and likely worldwide. Biopsy is generally
or high-dose glucocorticoids given for acute not performed when the diagnosis is clear from
rejection. the patient history and the patient has overt
proteinuria, but may be performed if atypical
15.31. Answer: C. features present (e.g. short history of
Cast nephropathy is a tubular injury as well-controlled diabetes, haematuria). ACE
described in option C and presenting with renal inhibitors generally decrease proteinuria and
impairment. Option A refers to monoclonal slow, but do not halt, progression of the
immunoglobulin deposition disease (usually light disease.
chain deposition disease). Option B refers to
amyloidosis, which may occur with multiple 15.35. Answer: B.
myeloma and presents with proteinuria or The presence of red cell casts and heavy
nephritic syndrome. Option D refers to proteinuria indicates severe glomerular injury
Fanconi's syndrome, a proximal tubulopathy. and a likely proliferative lupus nephritis that
needs immunosuppression. Mycophenolate
15.32. Answer: A. mofetil and glucocorticoids have been shown
Sarcoidosis typically causes a granulomatous to be as effective as the traditional treatment of
interstitial ·nephritis. It is not associated with the cyclophosphamide and steroids for both I
FSGS lesion. Option D refers to a rapidly induction and maintenance treatment. As t~ls
progressive glomerulonephritis such as ANCA woman likely wants to preserve her fertility/
vasculitis or anti-GBM disease. A chronic mycophenolate mofetil is a better choice f,br
interstitial nephritis may manifest as widespread her. Most patients who develop ESRD go into
'scarring' (interstitial fibrosis and tubular remission. If transplanted, recurrence rnay
atrophy) but the process described above is occur post -transplant, but usually does /not
relatively acute and resolved with treatment. cause significant nephritis, possibly due to
While sarcoidosis frequently causes post-transplant immunosuppression.
hypercalcaemia, calcium does not deposit in
tubules, but larger-scale nephrocalcinosis may 15.36. Answer: B.
be seen on an ultrasound scan. This patient has chronic interstitial nephritis,
which manifests as progressive renal failure,
15.33. Answer: E. small kidneys and urine showing no blood and
Hepatorenal syndrome should be considered a minimal to no proteinuria. Biopsy will often have
pure form of pre-renal injury, mediated by no glomerular changes but will demonstrate
reduced renal perfusion, due to splanchnic interstitial fibrosis and tubular atrophy. Patients
vasodilatation and up-regulation of the with chronic interstitial nephritis may have
renin-angiotensin system among others. a renal tubular acidosis, which is a
Therefore urine sodium is classically low. It is a hyperchloraemic (i.e. nqn-anion gap) acidosis.
diagnosis of exclusion and renal· biopsy is /
generally not performed, and may be 15.37. Answer: B. 1 '
dangerous in a coagulopathic liver patient. HNF1-beta mutations may cause several renal
Hepatorenal syndrome portends a dismal phenotypes, which may differ within the same
prognosis and dialysis is only performed if the family (including interstitial nephritis, cystic
liver disorder is remediable or a liver transplant kidneys, vesica-ureteric reflux), maturity~onset
is likely. If dialysis is performed, slow diabetes of the young, pancreatic atrophy,

downloaded from www.medicalbr.com


NEPHROLOGY AND UROLOGY • 129

gout, hypomagnesaemia and abnormal liver glomerular perfusion. ACE inhibitors cause
function tests. COL4A5 mutations cause efferent arteriolar vasodilatation, further
X-linked Alpert's syndrome, which would not fit dropping intra-glomerular pressure and hence
here (male-to-male transmission, cystic disease, GFR. The diuretic may cause volume depletion,
other features). He likely has an autosomal adding to the insult. Rhabdomyolysis is not the
dominant condition but he does not have cause, as the atorvastatin is not a recent
polycystic kidney disease as no cysts are medicine, and urine myoglobin causes a
evident on scanning. UMOD mutations may false-positive dipstick for blood.
cause a chronic interstitial nephritis and gout,
but would not explain the other features 15.42. Answer: D.
(diabetes, pancreatic atrophy). This patient has developed the nephrotic
syndrome after taking NSAIDs so the
15.38. Answer: A. possibilities are minimal change disease,
Anti-GBM antibodies may develop due to characterised by normal light microscopy, or
normal type IV collagen subunits expressed on membranous nephropathy. Option D refers to
the donor kidney. Female carriers of X-linked minimal change disease and none of the
disease may be symptomatic, although milder answers describe membranous nephropathy.
than males, due to random inactivation of the X Option A refers to thrombotic microangiopathy,
chromosome. Deafness may occur due to the which NSAIDs do not cause. Options B and E
presence of abnormal cochlear type IV collagen. refer to acute interstitial nephritis and acute
Some patients with type IV collagen mutations tubular necrosis. Both may be caused by
develop subtle abnormalities manifested NSAIDs, but do not cause the nephrotic
clinically by haematuria only (thin basement syndrome. Option C refers to rapidly
membrane disease). Alport's syndrome is
usually X-linked (COL4A5 mutations) but
progressive glomerulonephritis such as ANCA
vasculitis.
~
autosomal recessive and dominant disease
may occur (COL4A3 and COL4A4 mutations).

15.39. Answer: D.
15.43. Answer: E.
The micturition cycle has a storage (filling)
phase and a voiding (micturition) phase. During
I
Patients with APKD are at risk of liver cysts, the filling phase, the high compliance of the
cerebral berry aneurysms and mitral valve detrusor muscle allows the bladder to fill
prolapse. A ruptured liver cyst would not cause steadily without a rise in intravesical pressure.
sudden death but a berry aneurysm certainly As bladder volume increases, stretch receptors
would. Mitral valve prolapse is usually in its wall cause reflex bladder relaxation and
asymptomatic but may lead to mitral increased sphincter tone. At approximately
regurgitation; however, it would be a rare cause 75% bladder capacity, there is a desire to void ..
of sudden death. Voluntary control is now exerted over the desire
to void, which disappears temporarily.
15.40. Answer: A. Compliance of the detrusor allows further
The patient has APKD. It is an autosomal increase in capacity until the next desire to
dominant condition, so offspring have a 50% void. Just how often this desire needs to be
chance of inheriting it. Given his preserved inhibited depends on many factors, not the
renal function and good prognosis in affected least of which is finding a suitable place in
family members, the mutation is likely located in which to void.
the PKD2 gene. While liver cysts are common, The act of micturition is initiated first by
liver failure is very rare in APKD, particularly in voluntary and then by reflex relaxation of the
men. Tolvaptan is indicated for patients pelvic floor and distal sphincter mechanism,
deemed to be high risk for progression, which followed by reflex detrusor ;;ontraction. These
this man is not, given his preserved·renal actions are coordinated lj¥' the pontine
function well into his 60s and good prognosis micturition centre. ·
in affected family members.
15.44. Answer: E.
15.41. Answer: A. Although MRI and CT scanning may identify
NSAIDs cause prostaglandin-induced afferent some large bone metastases in the area
arteriolar vasoconstriction, which drops scanned, they will not identify smaller deposits

downloaded from www.medicalbr.com


130 • NEPHROLOGY AND UROLOGY

(which may be multiple). The injection of examination (ORE), International Prostate


Tc-labelled methylene diphosphonate Symptom Score (IPSS) questionnaire and flow
9
(" mTc-MDP) to undertake a whole-body bone test with residual volume of urine assessment
scan is needed to definitively identify bone by ultrasound. Prostate biopsy would be
metastases. undertaken if prostate cancer was suspected
by ORE and/or raised prostate-specific antigen
15.45. Answer: E. (PSA). MRI pelvis is mainly used to assess the
This description is pathognomonic of a prostate for presence of cancer, not
vesicovaginal fistula secondary to a prolonged assessment of lower urinary tract symptoms
obstructed labour. Similar symptoms may (LUTS). Cystoscopy is not an initial investigation
occur in an infant with congenital ectopic ureter for voiding LUTS. Prostate ultrasound is useful
inserting into the vagina but this would not to assess the exact size, presence of
present for the first time in a woman in her 20s. calcification or abscess of the prostate, but is
not routinely used to assess LUTS.
15.46. Answer: B.
The commonest causes of visible haematuria 15.50. Answer: C.
are: urinary tract infection, bladder cancer and This woman may be cured by a total
urinary tract stones. Ureteric stones are usually nephrectomy. In a tumour of this size, an open
painful rather than painless; nephrological approach is likely to be undertaken by most
causes of visible haematuria are less common surgeons. The lesion is too large for a partial
than urological causes. Upper urinary tract nephrectomy or ablative approach such as
urothelial cell cancer is rare relative to bladder cryotherapy. Radiotherapy is not a treatment
cancer. Of these choices, bladder cancer is the option for renal cancer. TKis are used in
most likely pathology. metastatic disease.

15.47. Answer: C. 15.51. Answer: E.


This patient has an infected, obstructed left This fit patient is best managed with radical
kidney secondary to an obstructing ureteric cystectomy to try and cure the high-grade
stone. The critical step here is to unobstruct (G3) muscle-invasive (T2) bladder cancer. j
the kidney and allow recovery of the sepsis The other options are not appropriate in thip
with antibiotics and resuscitation. The key setting. /
urological interventions to unobstruct the kidney /
are a ureteric stent or percutaneous 15.52. Answer: C.
nephrostomy tube insertion. Definitive treatment This man's grossly elevated PSA, ORE findings
options (ESWL, PCNL or ureteroscopy) are not and bone scan result indicate he has /
appropriate at this time and should be deferred metastatic prostate cancer. This is incurable
to a later date when the patient has recovered but controllable with GnRH agonist injections
from sepsis. (androgen flare covered by initial androgen
receptor blocker treatment for 3-4 weeks).
15.48. Answer: C. The other therapies are not appropriate for
Asymptomatic bacteriuria is defined as metastatic disease, nor is observation in a man
> 105 organisms/ml urine in healthy, with symptoms of metastatic disease.
asymptomatic patients. It is commonly
identified in patients with indwelling catheters 15.53. Answer: C.
and stents. This condition should be treated Persistent non-visible haematuria is 2 of 3 urine
with antibiotics in infants, pregnant women and dipstick tests positive for at least 1+ blood.
those with urinary tract abnormalities. Investigations should be undertaken in patients
who have associated symptoms (such as
15.49. Answer: E. dysuria) that would il)dicate a possible
This man has lower urinary tract symptoms, intravesical lesion. Additionally, this man is a
most likely secondary to bladder outlet smoker, putting him at higher risk for bladder
qbstruction. His family physician has correctly cancer. The most appropriate initial
trialled him on treatment with an a-blocker. On investigations are MSU to ~ule out infection,
attending the urology department he should cystoscopy and upper tract imaging toyisualise
initially be assessed by digital rectal the urinary tract.

downloaded from www.medicalbr.com


NEPHROLOGY AND UROLOGY • 131

15.54. Answer: E. self-catheterisation. Haemodialysis is not a


This man has a testicular cancer until proven curative treatment option in the setting of
otherwise. He should be seen urgently and, high-pressure urinary retention. Medical
following examination, undergo an urgent management, such as an a-blocker, is
ultrasound, which is the gold standard contraindicated.
investigation to rule out a testicular cancer.
Testicular cancer is almost always treated with 15.57. Answer: C.
an initial inguinal orchidectomy. The first -line treatment of stress incontinence
is pelvic floor exercises taught by a
15.55. Answer: A. urophysiotherapist. If unsuccessful, further
This man should initially be escalated to management options include tension-free
combination medical therapy for BPH with a vaginal tape. Anticholinergic medication,
Sa-reductase inhibitor. Further suitable botulinum toxin injection and sacral nerve
treatments for symptoms that are refractory to stimulation are all treatment options for urge
medical therapy include: transurethral resection incontinence.
of the prostate, laser prostatectomy or open
prostatectomy (Millen's procedure). 15.58. Answer: E.
High-intensity focused ultrasound therapy or In men presenting with new-onset erectile
robot-assisted laparoscopic radical dysfunction it is vital to ensure that they do not
prostatectomy are treatments used for prostate have previously undiagnosed coronary artery
cancer. disease that has manifest as ED. Risk factors
for vascular disease such as hypertension and
15.56. Answer: E. hyperlipidaemia should be evaluated. Perineal
These symptoms indicate high-pressure chronic trauma is a rare cause of ED. Pudendal
urinary retention for which the initial artery angiography is rarely performed.
management is insertion of a urinary catheter; Phosphodiesterase type 5 inhibitors are the
this will result in improvement in renal function. first-line treatment options for ED, not
Bilateral ureteric stent insertion will not relieve intracavemosal alprostadil. Depending on the
the more distal prostatic obstruction of the characteristics of the patient, the consultation
urinary tract. The patient may be managed for a man with ED may be a good opportunity
thereafter with bladder outlet surgery such to discuss lower urinary tract symptoms and a
as a transurethral resection of the prostate, PSA test; however, this is not essential to the
long-term urethral catheterisation or intermittent assessment of the ED component.

downloaded from www.medicalbr.com


I,

DE Newby, NR Grubb

Cardiology
Multiple Choice Questions
16.1. A 55 year old man with a history of poorly elevated jugular venous pressure (JVP). Which
controlled hypertension presents with a history of the following conditions is most likely to
of sudden-onset central chest pain. There are explain this physical finding?
no diagnostic electrocardiogram (EGG) A. Aortic stenosis
abnormalities, and an interval troponin B. Dehydration
concentration is not diagnostic of myocardial C. Exacerbation of asthma
infarction. What diagnosis should be confirmed D. Increased left atrial pressure
or excluded next? E. Recurrent pulmonary embolism
A. Anxiety
B. Aortic dissection 16.5. A 56 year old man presents with a history
C. Myocarditis of headache. He is noted to have a loud
D. Pericarditis second heart sound on auscultation. Whic~ of
E. Pneumothorax the following pathologies could explain thi~
. d'1ng.?
f1n /I
16.2. The term 'orthopnoea' refers to A. Aortic incompetence
breathlessness (dyspnoea) in a particular B. Essential hypertension
situation. Which answer below describes that C. Mechanical mitral valve replacement
situation? D. Mitral incompetence
A. After several hours of sleep E. Postural hypotension
B. Due to asthma
C. Immediately on lying flat 16.6. Which of the following pathologies can be
D. On exertion associated with an early diastolic murmur?
E. On sitting upright A. Long QT syndrome type 1
B. Marfan's syndrome
16.3. A 75 year old woman presents to her C. Mitral valve prolapse
family physician with a 24-hour history of rapid, D. Myotonic dystrophy
irregular palpitations accompanied by fatigue. In E. Wolff-Parkinson-White syndrome
an elderly patient, what is the most likely cause
of palpitations? 16.7. An 80 year old woman with a history of
A. Atrial ectopic (premature) beats palpitation presents with a painful left leg. On
B. Atrial fibrillation examination, pulse rate is 80 beats/min and
C. Supraventricular tachycardia irregular, blood pressure (BP) 170/96 mmHg.
D. Ventricular ectopic (premature) beats The left leg is pale, cold, and sensation is
E. Ventricular tachycardia reduced. The popliteal, dorsalis pedis and
posterior tibial pulses cannot be felt. Her only
16.4. A 74 year old woman presents with regular medications are aspirin and digoxin.
breathlessness. She is found to have an What is the most likely diagnosis?

downloaded from www.medicalbr.com


CARDIOLOGY • 133

A. Acute arterial plaque rupture with lower limb A. A normal baseline troponin and elevated
ischaemia 6-hour troponin level is suspicious of
B. Deep venous thrombosis with secondary myocardial infarction
reduction of arterial blood flow B. A normal EGG excludes myocardial infarction
G. Dissection of the femoral artery due to G. A normal initial troponin level excludes
uncontrolled hypertension myocardial infarction
D. Peripheral embolism with lower limb ischaemia D. Failure of chest pain to resolve with nitrates
E. Reduced lower limb perfusion due to cardiac confirms myocardial infarction
failure E. T-wave inversion on the EGG confirms
myocardial infarction
16.8. A 50 year old man is assessed because
of 3 weeks of fever and influenza-like 16.12. A 72 year old hypertensive woman
symptoms. Examination findings are presents with a history of sudden-onset, rapid,
tachycardia (heart rate 105 beats/min), and a irregular palpitation. She has had several
large pulse pressure, BP 140/45 mmHg. Initially episodes over the previous 3 months, which
it was thought a murmur was present but have resolved within 1 hour. She feels tired and
repeat examination reveals no murmur. slightly lightheaded during episodes. From this
Investigations reveal no evidence of chest or history, which of the following most likely
urinary infection. What are these findings most explains her symptoms?
compatible with? A. Atrial fibrillation
A. Acute myocarditis B. Sinus arrhythmia
B. Acute viral pericarditis G. Supraventricular tachycardia
G. Infective endocarditis affecting the aortic valve D. Ventricular ectopic beats (extrasystoles)
D. Infective endocarditis affecting the tricuspid E. Ventricular tachycardia

~.
valve
E. Influenza 16.13. In the management of cardiac arrest, ~
which of the following most accurately
16.9. You assess a 62 year old woman 2 days describes basic life support (BLS)?
after treatment for anterior myocardial
infarction. On examination she is tachycardic
A. Administration of intravenous drugs and j
external defibrillation (the two 'D's)
and tachypnoeic, and has a harsh systolic B. External cardiac massage only
murmur radiating to the right side of the chest. G. Support of airway, breathing and circulation
There are fine inspiratory crepitations audible at (ABC)
the lung bases. What is the most likely D. Support of airway, breathing and circulation,
explanation for these findings? and assessment of disability and exposure
A. Acute aortic incompetence (ABC DE)
B. Left ventricular free wall rupture E. Support of airway, breathing and circulation,
G. Papillary muscle rupture and mitral and assessment of disability and exposure,
incompetence treatment of fibrillation (ABCDEF)
D. Post -infarction pericarditis with pericardia! rub
E. Rupture of the interventricular septum 16.14. Which of the following statements is true
of a pulseless electrical activity (PEA) cardiac
16.10. Which of the following physical signs is arrest?
associated with left ventricular failure?
A. Cardiopulmonary resuscitation (CPR) should
A. A gallop rhythm with a fourth heart sound be carried out for 1 minute before the rhythm
B. A gallop rhythm with a third heart sound is reassessed
G. A loud second heart sound B. Intravenous amiodarone will restore cardiac
I
D. A quiet first heart sound output /
E. Fixed splitting of the second heart sound G. It is initially managed with immediate
defibrillation
16.11. A 55 year old man with type 2 diabetes D. Reversible causes incLude hyperthyroidism
presents with a 1-hour history of severe central and hypercalcaemia
chest pain. Which of the following statements E. Reversible causes include hypothermiaand
is true? hypoxia

downloaded from www.medicalbr.com


1 34 • CARDIOLOGY

16.15. A 65 year old female presents with chest


pain, and the 12-lead EGG shows evidence of
acute inferior myocardial infarction complicated
16.19.~-Adrenoceptor antagonists (~-blockers)
are used in which of the following situations?
A. Acute left ventricular failure
1
by hypotension. An echocardiogram is B. Cardiac failure associated with bradycardia
performed and shows markedly reduced G. Cardiogenic shock
movement of the right ventricular walls, D. Chronic left ventricular systolic dysfunction
indicating that right ventricular infarction has E. High-output cardiac failure
occurred. Left ventricular function is only mildly
impaired. Which of the following physical signs 16.20. A 71 year old woman with a history of
would be expected in this situation? hypertension presents with fatigue and rapid,
A. Tachycardia, a late systolic murmur and irregular palpitations. She normally takes
ascites enalapril for blood pressure control. Clinical
B. Tachycardia, and absent jugular venous examination reveals an irregularly irregular
pulse because of inability to develop right pulse, rate 125 beats/min, and BP
heart pressure 128/86 mmHg. Cardiovascular examination is
G. Tachycardia, acute development of otherwise normal. A 12-lead EGG is performed,
peripheral oedema and acute ascites which shows atrial fibrillation with poor
D. Tachycardia, basal crepitations and a third ventricular rate control, but no other
heart sound abnormality. Which of the following drugs is the
E. Tachycardia, elevated jugular venous pulse most suitable agent to control heart rate in this
due to failure of right ventricular pump patient?
function, and hepatomegaly A. Adenosine
B. Amiodarone
16.16. What relationship does Starling's Law of G. ~-blocker
the heart describe? D. Flecainide
A. Between blood pressure and cardiac output E. Lidocaine
B. Between cardiac filling and blood pressure
G.
D.
E.
Between
Between
Between
cardiac filling and cardiac output
heart rate and blood pressure
heart rate and cardiac output
16.21. An 85 year old man presents with a
6-month history of sudden episodes of l
lightheadedness, which last up to 15 seconds.
He is admitted to hospital with an episbde of
16.17. What underlying pathophysiological syncope resulting in facial injury. Exan-fine the
changes is chronic cardiac failure associated rhythm strip below. Which conduction
with? abnormality does this show? 1
I
A. Activation of the renin-angiotensin- A. Complete (third-degree) AV block
aldosterone system (RAAS) B. Left bundle branch block
B. Inhibition of the RAAS G. Mobitz type II second-degree AV block
G. Inhibition of the sympathetic nervous system D. Sinus bradycardia
D. Reduced production of brain natriuretic E. Wenckebach (Mobitz type I) second-degree
peptide (BNP) AV block
E. Systemic vasodilatation
16.22. Which of the following rhythms is NOT
16.18. Loop diuretics such as furosemide and commonly associated with sick sinus syndrome
bumetanide have which of the following (sinoatrial disease)?
effects? A. Atrial fibrillation
A. Diuresis due to inhibition of potassium and B. Atrial tachycardia
water reabsorption G. Sinus bradycardia
B. Diuresis due to inhibition of sodium and D. Sinus pauses
water reabsorption E. Ventricular tachycardia
G. Diuresis due to inhibition of water
reabsorption only
D. Increased serum potassium levels due to
enhanced distal tubule function
E. Osmotic diuresis Fig. 16.21

downloaded from www.medicalbr.com


CARDIOLOGY • 135

16.23. A 75 year old woman has a history of A. A 26 year old man with polymorphic
hypertension and diabetes. She presents with ventricular tachycardia (torsades de pointes)
atrial fibrillation. What is her CHA2 DS 2 -VASc occurring after cocaine use
score? B. A 48 year old man who presents with acute
inferior myocardial infarction complicated
A. 2 within the first 6 hours by ventricular
B.3
fibrillation
c. 4 C. A 55 year old woman with syncope; EGG
D.5
monitoring shows sinus rhythm with
E. 6
third-degree atrioventricular block
D. A 75 year old man with syncope; ambulatory
16.24. Which of the following drugs is known to
EGG shows sinus bradycardia and daytime
be effective in preventing stroke in patients with
sinus pauses of up to 5 seconds
atrial fibrillation?
E. An 80 year old man with a history of anterior
A. Amiodarone myocardial infarction 6 months previously; he
B. Apixaban is fit, has never experienced arrhythmia, and
C. Aspirin a cardiac magnetic resonance scan shows
D. ~-blocker poor left ventricular function (left ventricular
E. Clopidogrel ejection fraction 28%)

16.25. The EGG below shows a regular, narrow


16.28. A 17 year old male presents to the
complex tachycardia in a patient presenting
emergency department with an episode of
with sudden-onset, rapid palpitation. Which of
collapse. Witnesses report he became extremely
the following should be used first in attempting
blue at the time of collapse, which occurred on
to terminate this rhythm?
walking. The patient tells you he has a history of
A. Direct current cardioversion congenital heart disease. On examination you
B. Intravenous adenosine note he is centrally cyanosed. Which of the
C. Intravenous ~-blocker following congenital conditions is the most likely
D. Oral ~-blocker explanation for this presentation?
E. Vagal manoeuvres, e.g. Valsalva manoeuvre
A. Coarctation of the aorta
B. Congenital heart block
16.26. For which of the following scenarios
C. Patent foramen ovate
would a permanent pacemaker be an
D. Tetralogy of Fallot
appropriate treatment?
E. Wolff-Parkinson-White syndrome
A. Paroxysmal atrial fibrillation
B. Prevention of sudden death due to 16.29. Which of the following is true of
ventricular fibrillation
Eisenmenger's syndrome?
C. Sick sinus syndrome associated with syncope
D. Sinus bradycardia in an athlete A. Breathlessness and fatigue are uncommon
E. Supraventricular tachycardia symptoms
B. It occurs in patients with patent foramen ovate
16.27. Which of the following patients is a C. Left to right shunting occurs because of
suitable candidate for an implantable cardiac pulmonary hypertension
defibrillator? D. Life expectancy is markedly reduced
E. Patients are peripherally but not centrally
cyanosed

16.30. A 48 year old woma~ registers with a


new family physician. She'tells the .doctor she
had a small hole in her heart from birth but
that it did not require. any treatment. On
examination, pulse is 70peats/min and regular;
BP 122/76 mmHg. You detect a loud,
high-pitched systolic murmur at the left sternal
Fig. 16.25 border, accompanied by a thrill. Which of the

downloaded from www.medicalbr.com


136 • CARDIOLOGY T
I
following conditions would explain the history D. Obesity
and physical findings? E. Recreational cannabis use
A. Anterior mitral leaflet prolapse
B. Atrial septal defect
16.35. By which of the following features is
hypertrophic cardiomyopathy usually
G. Patent foramen ovale
characterised?
D. Persistent ductus arteriosus
E. Ventricular septal defect A. Asymmetric left ventricular hypertrophy with
marked thickening of the interventricular
16.31. A 21 year old man presents with a recent septum
history of an influenza-like illness initially B. Asymmetric left ventricular hypertrophy with
characterised by fever, myalgia and headache. marked thickening of the anterior left
He develops pleuritic-type chest discomfort and ventricular wall
breathlessness. On examination, pulse is G. Hypertrophy of both atria and both
105 beats/min and regular; BP 105/60 mmHg. ventricles
The JVP is not elevated. Heart sounds 1 and 2 D. Hypertrophy of the left ventricle and atrophy
are present with a loud to-and-fro harsh sound of the right ventricle
present in systole and diastole. Which of the E. Symmetrical left ventricular hypertrophy
following conditions explains this clinical
presentation? 16.36. Cardiac transplantation is considered in
A. Acute viral pericarditis which group of patients with cardiomyopathy?
B. Aortic valve endocarditis A. Asymptomatic patients
G. Mitral valve endocarditis B. Frail elderly patients with end-stage heart
D. Persistent ductus arteriosus failure
E. Pulmonary embolism G. Patients who do not wish to take life-long
medication
16.32. What is the appropriate initial treatment D. Patients who have poor quality of life despite
for the symptoms of acute pericarditis? optimal drug therapy
A. Intravenous glucocorticoids E. Patients who have symptoms but good
B. Intravenous morphine quality of life on optimal drug therapy /
G. Oral amiodarone
D. Oral aspirin 16.37. A 48 year old woman with no sig~l~icant
E. Rectal diclofenac previous medical history collapses while running
a marathon. Despite attempts at resuscitation,
16.33. Which of the following best describes she does not survive. Postmortem exq.mination
dilated cardiomyopathy? reveals asymmetric left ventricular hypertrophy
A. A disease of the myocardium characterised with disproportionate thickening of the
by chamber enlargement and thinning of the interventricular septum. A postmortem
left and right ventricular walls diagnosis of hypertrophic cardiomyopathy is
B. A disease of the myocardium characterised made. What is the most likely cause of this
by disproportionate thickening of the patient's sudden collapse?
interventricular septum A. Atrial fibrillation
G. A disease of the myocardium characterised B. Left ventricular failure
by infiltration of myocardial tissue resulting in G. Pulmonary embolism
restricted contraction and relaxation D. Right ventricular failure
D. Isolated dilatation of the atria, causing atrial E. Ventricular arrhythmia
fibrillation
E. Isolated dilatation of the right ventricle, 16.38. A 30 year old ':'>'oman has recently been
causing ventricular tachycardia diagnosed with dilaJed cardiomyopathy. Her
diagnosis was maae with echocardiography,
16.34. Which of the following is a cause of which showed moderate left ventricular
dilated cardiomyopathy? dilatation and impalrment. She has noticed
A. A high-cholesterol diet herself becoming increasingly fatigued on
B. Heavy alcohol consumption moderate exertion. Her younger sister. died
G. Mutation in cardiac sodium channel gene suddenly the previous year and she is very

downloaded from www.medicalbr.com


CARDIOLOGY • 137

worried about the risk of sudden death. Which 16.43. A 75 year old male smoker presents
of the following treatments is known to reduce with a 6-week history of progressive exertional
her risk of sudden death? breathlessness and fatigue. Latterly he has
A. Aspirin noticed his ankles swelling in the afternoon.
B. ~-blocker (e.g. metoprolol) On examination, pulse is I 00 beats/min and
c. Calcium channel blocker (e.g. verapamil) regular; BP 92/60 mmHg. The JVP is elevated
D. Loop diuretic (e.g. furosemide) and rises on inspiration. Heart sounds are quiet
E. Percutaneous coronary intervention (PCI) and there are no added sounds. There is
bilateral pitting oedema to the knees. A chest
16.39. A 55 year old woman presents with a X-ray is requested, which shows apparent
history of acute, severe, constricting central cardiomegaly with a globular cardiac
chest pain associated with anterior ST segment silhouette. You suspect a possible pericardia!
elevation on the 12-lead EGG. She immediately effusion. Which of the following statements
undergoes coronary angiography, which shows is true?
no evidence of coronary artery disease and no A. A large effusion can be a sign of
coronary occlusion. An echocardiogram shows malignancy
left ventricular apical dilatation, with normal left B. A pericardia! rub is always heard if the
ventricular basal contraction. Which of the effusion is large
following factors is most likely to have C. An EGG is the best investigation to confirm
precipitated this illness? the diagnosis
A. Acute emotional stress D. High-dose diuretic therapy will resolve the
B. Cigarette smoking pericardia! effusion
C. Excessive alcohol consumption E. In symptomatic patients, cardiac surgery is
D. Genetic factors required to remove the pericardia! fluid
E. Viral infection
16.44. An 18 year old man presents with
16.40. Which of the following is associated with sudden onset of sharp chest pain.
excessive alcohol consumption? The pain is made worse by deep inspiration
A. Atrial fibrillation or lying down flat. It is relieved by sitting
B. Diverticulitis forward and taking shallow breaths. He
C. Hypertrophic cardiomyopathy presents to the emergency department and
D. Hypotension an EGG is recorded because the attending
E. Supraventricular tachycardia doctor suspects acute pericarditis. What
is the most specific EGG change in
16.41. Atrial myxoma is the most common pericarditis?
primary cardiac tumour. Which of the following A. PR interval prolongation
is true of atrial myxoma? B. PR segment depression
A. Atrial myxomas are usually malignant C. ST depression
B. It occurs more commonly in the right atrium D. ST elevation
than in the left atrium E. T-wave inversion
C. Surgery is not indicated because atrial
myxomas are benign 16.45. A 46 year old man has recently fractured
D. Surgery is usually indicated to prevent his leg, which is in a plaster cast. He suddenly
embolic complications such as stroke becomes very breathless, unwell and collapses.
E. The tumour commonly obstructs the aortic The attending doctor suspects a pulmonary
valve embolus from a deep vein thrombosis. The
doctor performs an EGG. What is the most
16.42. Which of the following conditions may common EGG change in patients with
result in chronic pericardia! constriction? pulmonary embolism? /
A. Acute myocardial infarction A. Anterior T-wave inversion
B. Dilated cardiomyopathy B. Atrial fibrillation
C. Excessive alcohol consumption C. 'SIQ3T3'
D. Osteoarthritis D. Sinus tachycardia
E. Tuberculosis E. ST elevation

downloaded from www.medicalbr.com


138 • CARDIOLOGY

16.46. In patients with a pericardia! effusion,


what is the most important clinical sign to
16.51. An SO year old woman presents with
shortness of breath and swollen ankles. Her
T
determine whether there is cardiac tamponade? ECG showed some poor R-wave progression.
She was referred for an echocardiogram and
A. Cyanosis
B. Haematuria was found to have a high ejection fraction.
Which of these conditions is the most likely
G. Peripheral oedema
D. Pulsus paradoxus cause of her presentation?
E. Raised JVP A. Acute myocarditis
B. Aortic stenosis
16.47. The following medical treatments are all G. Dilated cardiomyopathy
associated with improved symptoms in patients D. lschaemic cardiomyopathy with extensive
with heart failure due to left ventricular systolic infarction
dysfunction. However, which of the treatments E. Restrictive cardiomyopathy
has NOT been shown to also improve survival?
A. Bisoprolol 16.52. Neuroendocrine system activation is a
B. Enalapril feature of heart failure. Abnormalities of which
G. Furosemide hormone can cause heart failure rather than
D. Sacubitril-valsartan result from heart failure?
E. Spironolactone A. Aldosterone
B. Angiotensin II
16.48. Which of the following antiplatelet drugs G. Catecholamines
is a phosphodiesterase inhibitor? D, Thyroxine
A. Cangrelor E. Vasopressin (antidiuretic hormone, ADH)
B. Clopidogrel
G. Dipyridamole 16.53. Which of the following biomarkers is a
D. Prasugrel structural protein rather than a cardiac
E. Ticagrelor enzyme?
A. Aspartate aminotransferase
16.49. A 54 year old security guard who is B. Creatine kinase
obese and enjoys drinking alcohol and cigarette G. Creatine kinase MB
smoking with his friends has a diet high in D. Lactate dehydrogenase
saturated fats. He has an acute myocardial E. Troponin I
infarction. Which lifestyle risk factor has the
strongest association with myocardial 16.54. A patient has a stent placed in hi,s right
infarction? coronary artery. On return to the ward, he gets
A. Excess alcohol severe chest pain and becomes very unwell.
B. High-saturated fat diet The nurse undertakes an ECG and calls the
G. Obesity interventional cardiologist to review the patient
D. Sedentary activity because she is concerned that he has a
E. Smoking thrombosed stent. What ECG features would
suggest the stent has become occluded?
16.50. A 36 year old smoker has sudden onset A. Anterior T-wave inversion
of chest pain whilst out walking in a remote B. Atrial fibrillation
island of Scotland. He attends the local hospital G. Atrioventricular block
and is found to have ST segment elevation D. ST elevation in I, aVL and V6
myocardial infarction. Which treatment has the E. ST elevation in V2-V5
strongest time-dependent benefit (i.e. the
quicker received, the better the. outcome) for 16.55. A 72 year old woman has had
ST segment elevation myocardial infarction? 'indigestion' for 4 Clays with vomiting and
A. Aspirin sweating. She presents to the emergency
B. ~-blocker department where a delayed presentation inferior
G. Heparin ST segment elevation myocardial infarction is
D. Percutaneous coronary intervention diagnosed. She has already developed, Q
E. Tissue plasminogen activator waves in leads II, Ill and aVF. One day after

downloaded from www.medicalbr.com


CARDIOLOGY • 139

admission to hospital, she suddenly deteriorates C. Smoking cessation


with severe breathlessness, low blood pressure D. Statin
and sudden onset of pulmonary oedema. What E. Warfarin
is the most likely cause?
A. Acute papillary muscle rupture 16.60. Limb ischaemia can take many forms
B. Acute pericarditis and has varied causes. This may result in
C. Atrial septal defect sudden acute vessel occlusion from arterial
D. Free wall rupture spasm or thrombosis, or more chronic
E. Mural thrombus processes. What is the most likely underlying
cause of severe limb ischaemia in an otherwise
16.56. A patient admitted to the emergency well, thin 30 year old heavy smoker?
department with severe chest pain and ST A. Atherosclerosis
segment deviation suddenly collapses and is B. Atrial fibrillation
found not to be breathing or have a pulse. C. Buerger's disease
A cardiac arrest call is made. What is the most D. Diabetes mellitus
likely cause of his collapse? E. Raynaud's disease
A. Asystole
B. Complete heart block 16.61. A 65 year old man with known
C. Free wall rupture hypertension presents with severe central chest
D. Pulseless electrical activity pain that radiates between his shoulder blades.
E. Ventricular fibrillation He is sweaty with a BP of 200/1 00 mmHg in
his right arm, a pale left arm and an ECG
16.57. A 75 year old man is incidentally found showing sinus tachycardia. His chest X-ray
to have a pulsatile swelling in his abdomen on shows mediastinal widening and a computed
a routine health check. He is sent for an tomography scan shows a type A aortic
abdominal ultrasound scan, which confirms the dissection. Which of the following is known to
presence of an abdominal aortic aneurysm. reduce mortality?
Which risk factor is protective against the
A. Anticoagulation
formation and expansion of an abdominal aortic
B. Control of the blood pressure
aneurysm?
C. Emergency repair of the ascending aorta
A. Diabetes mellitus D. Intravenous p-blockade
B. Family history of aneurysm disease E. Prevention of limb or renal ischaemia
C. Hypercholesterolaemia
D. Hypertension 16.62. A short young woman presents with
E. Smoking severe chest pain, vomiting and a sinus
tachycardia. She is in the last trimester of
16.58. A 39 year old heavy smoker presents pregnancy and has had normal blood pressure
with calf pain on walking and is referred to a and observations at antenatal care. She is
vascular surgeon for assessment. Which clinical admitted for observation but is later found
feature would be most reassuring? collapsed and in cardiac arrest. Despite
A. Capillary refill <2 seconds attempts at resuscitation, mother and child die.
B. Cold temperature Postmortem reveals an aortic dissection. What
C. Hair loss is the most likely underlying cause for the
D. Pallor dissection?
E. Pulselessness A. Coarctation of the aorta
B. Intramural haematoma
16.59. A 65 year old smoker with hypertension C. Marfan's syndrome
1
is found to have an abdominal aortic aneurysm D. Pregnancy /
on population screening with ultrasound. Which E. Undiagnosed hypertension
intervention will most reduc~ his future risk of
aortic aneurysm rupture? 16.63. A 50 year old WOIJlan with diabetes, who
A. Angiotensin-converting ernzyme (ACE) smokes, presents with jaw pai,n, severe nausea,
inhibitor autonomic arousal and vomiting. An ECG is
B. P-blocker performed in the emergency department and

downloaded from www.medicalbr.com


140 • CARDIOLOGY
--,
r
!
!

shows anterior ST segment elevation. What is


the best immediate reperfusion therapy?
D. Poor adherence with medication
E. Renal failure I
A. Coronary artery bypass graft surgery
B. Morphine 16.67. An 18 year old woman presents with a II
C. Primary percutaneous coronary intervention sore throat and suspected acute rheumatic
D. Streptokinase fever. Which of the following is a minor
E. Tissue plasminogen activator manifestation of acute rheumatic fever?
A. Carditis
16.64. An 81 year old non-smoker presents with B. Chorea
chest pain and an ECG with ST segment C. Erythema marginatum
depression. His troponin concentration is D. Raised C-reactive protein
456 ng/L (reference range <34 ng/L). He is E. Subcutaneous nodules
treated with an angioplasty and stent 2 days
later. At the same time, a 60 year old smoker 16.68. An 18 year old woman has a raised
with diabetes has a large anterior ST segment C-reactive protein, a rash consistent with
elevation myocardial infarction, has ventricular erythema marginatum and pyrexia. However,
fibrillation in the ambulance and has immediate the clinician remains uncertain about the
defibrillation. He undergoes immediate diagnosis of acute rheumatic fever. Rapid
percutaneous coronary intervention on arrival at response to which treatment will help to
hospital and has a troponin concentration of confirm the diagnosis?
>50000 ng/L. A medical student asks who has A.Aspirin
the better prognosis. What is the biggest . B.Bed rest
predictor of mortality following acute myocardial C.Diuretics
infarction? D.Glucocorticoids
A. Age E. High-dose antibiotics
B. Cardiac arrest
C. ECG changes 16.69. A 34 year old woman presents with
D. Smoking symptoms of breathlessness on exertion, a
E. Troponin concentration malar flush and has a past history of rhe;,lnatic
fever. She is in sinus rhythm and has an/
16.65. A 35 year old executive has a echocardiogram that confirms mitral steposis.
reproducible BP of 180/1 00 mmHg. She is Which physical sign is she likely to have?
referred for assessment in the clinic. You A. Ejection systolic murmur
perform a range of tests to determine B. Mid-systolic click
whether there is an underlying cause for her C. Pre-systolic accentuation
hypertension. What is the commonest cause of D. Quiet second heart sound
secondary hypertension? E. Thrusting apex beat
A. Congenital adrenal hyperplasia
B. Conn's syndrome 16.70. An 80 year old man presents with an
C. Phaeochromocytoma incidental ejection systolic murmur. His family
D. Renal disease physician notices a parasternal thrill. What is
E. Thyrotoxicosis the likely underlying reason for the thrill?
A. Aortic stenosis
16.66. A 60 year old man is referred by his
B. Large atrial septal defect
family physician because despite four drugs he
C. Mitral stenosis
continues to have uncontrolled blood pressure.
D. Pulmonary hypertension
The doctor feels that the patient needs further
E. Right ventricular hypertrophy
investigation for a potential se<;:ondary cause of
hypertension. What is the commonest cause of
16.71. A 43 year old man undergoes a routine
poorly controlled hypertension?
health check with his employers. He is found to
A. Conn's syndrome have a murmur, isolated systolic hypertension
B. Glucocorticoid-suppressible (180/60 mmHg) and left ventricular hypertrophy
hyperaldosteronism on his ECG. A significant regurgitant qlood
C. Hyper-reninaemia flow is noticed across the aortic valve' on

downloaded from www.medicalbr.com


CARDIOLOGY • 141

echocardiogram. Which of the following clinical In what manner is hypertrophic cardiomyopathy


signs is likely to be observed? commonly inherited?
A. Crescendo-decrescendo murmur A. Autosomal dominant
B. Palpable thrill in the aortic area B. Autosomal recessive
c. Prominent pulsation in the neck (de Musset's C. Never inherited
sign) D. X-linked dominant
D. Quiet second heart sound E. X-linked recessive
E. Slow rising pulse
16.76. A 23 year old woman is referred for
16.72. A 65 year old man presents with a evaluation because she is very tall, has
4-week history of general malaise and lethargy. problems with her vision and has a heart
He has had two courses of antibiotics that murmur. Her family physician is concerned that
have temporarily improved his symptoms but she may have Marfan's syndrome. What
he continues to feel worse over time. His family structural gene is associated with an
physician notices he has become anaemic. He abnormality in Marfan's syndrome?
attends the emergency department and he is A. Fibrillin
admitted to hospital with a fever. He has some B. Myosin heavy chain
blood cultures taken and he undergoes an C. Myosin-binding protein
echocardiogram, which shows a mass on his D. Titan
mitral valve. What is the most likely organism E. Troponin
that will be grown from his blood cultures?
A. Staphylococcus aureus 16.77. A patient presents with an incidental
B. Staphylococcus epidermidis finding of a mass in the left atrium whilst
C. Streptococcus faecalis undergoing an echocardiogram for
D. Streptococcus gallolyticus hypertension. What is the most likely cardiac
E. Viridans streptococci tumour in this situation?
A. Angiosarcoma
16.73. Considering the patient in Question B. Atrial myxoma
16.72, before the blood culture results are C. Fibroelastoma
known, the junior doctor reviews the 65 year D. Fibroma
old man and examines him for evidence of E. Lipoma
endocarditis. What is the commonest sign that
the doctor is likely to find? 16.78. A 43 year old man has an extensive
A. Haematuria anterior myocardial infarction and has receiyed
B. Osier's nodes antiplatelet, anticoagulant and statin therapy.
C. Roth's spots He is referred for an echocardiogram. What will
D. Splinter haemorrhages transthoracic echocardiography most usefully
E. Subconjunctival haemorrhages assess in this setting?
A. Cardiac arrhythmia
16.74. Considering the patient in Questions B. Future prognosis
16.72 and 16.73, blood cultures demonstrate C. Left ventricular function and the presence of
viridans streptococci. What is the most mural thrombus
· appropriate antibiotic regime to commence the D. Myocardial scar formation
patient on? E. Thrombus in the left atrium
A. Intravenous ampicillin and gentamicin
B. Intravenous benzylpenicillin and gentamicin 16.79. The man with an extensive anterior
C. Intravenous flucloxacillin myocardial infarction in Question 16.78
D. Intravenous vancomycin and gentamicin undergoes coronary angipgraphy and is found
E. Oral benzylpenicillin to have coronary artery 'disease. Which features
on angiography predict the best outcome/
16.75. An army recruit is referred for improvements with coro!_lary artery bypass graft
assessment because there is a family history of surgery?
sudden cardiac death and an, uncle was A. Diabetes mellitus and diffuse three-vessel
diagnosed with hypertrophic cardiomyopathy. coronary heart disease

downloaded from www.medicalbr.com


1 42 • CARDIOLOGY

B. Left main stem stenosis and significant left


ventricular systolic dysfunction
C. Severe proximal disease of the left anterior
(reference range <34 ng/L). Which of the
following treatments is likely to worsen his
prognosis?
1
descending coronary artery A. Aspirin
D. Three-vessel coronary heart disease with B. Fondaparinux
good left ventricular function C. Intravenous tissue plasminogen activator
E. Two-vessel coronary heart disease (tPA)
D. Metoprolol
16.80. The man with an extensive anterior E. Ticagrelor
myocardial infarction in Questions 16.78 and
16.79 has left main stem and triple-vessel 16.84. An anaesthetist is seeking advice
disease and is referred for coronary artery regarding a patient with coronary heart disease,
bypass graft surgery. However, the surgeon is diabetes mellitus and a murmur. Which of the
concerned that the anterior wall is completely following is NOT a significant risk factor for
infarcted and is no longer viable. The surgeon perioperative myocardial infarction during
wants to know if the anterior wall has significant non-cardiac surgery?
amounts of scar tissue. Which imaging modality A. Aortic stenosis with a peak gradient of
is best to identify the scar of acute myocardial 25 mrnHg
infarction? B. Diabetes mellitus treated with insulin and
A. Computed tomography associated with renal failure
B. Coronary angiography C. Recent (within 4 weeks) stenting of a severe
C. Echocardiography proximal stenosis in the left anterior
D. MRI descending coronary artery
E. Stress echocardiography D. Recent acute coronary syndrome
E. Severe left ventricular dysfunction
16.81. An 83 year old woman presents
with acute pulmonary oedema, BP of 16.85. A 67 year old woman presents with
180/100 rnrnHg and a Sa0 2 of 85%. Which predictable exertional angina pectoris when
treatment is UNLIKELY to be helpful in this climbing steep-inclines. She has been /
setting? commenced on aspirin, statin and a P-b,lbcker.
She attends your clinic for assessment/Which
A. Furosemide
of the following suggests the patient is at low
B. Intravenous dobutarnine
risk of future events?
C. Intravenous nitrates
D. Non-invasive ventilation A. Poor exercise tolerance
E. Supplementary oxygen therapy B. Poor left ventricular function
C. Post-infarct angina
16.82. A 43 year old woman with a past history D. Recent onset of symptoms
of breast cancer is referred with a gradual E. ST segment depression during stage 3 of
onset of breathlessness. An echocardiograrn the Bruce Protocol
demonstrates a dilated poorly contracting left
ventricle. You wish to investigate potential 16.86. You review a 50 year old smoker 2
causes of her dilated cardiomyopathy. Which of months after successful treatment for a
the following would be an irreversible cause of myocardial infarction. Which intervention has
her dilated cardiomyopathy? the greatest benefit to prevent a recurrence of
A. Alcohol excess myocardial infarction?
B. Anthracycline chemotherapy A. ACE inhibitor therapy
C. Haernochrornatosis B. Aspirin
D. Hypothyroidism C. Regular and frequent aerobic exercise
E. Thyrotoxicosis D. Smoking cessation
E. Statin therapy
16.83. A 56 year old man presents with sudden
onset of chest pain radiating down his left arm,
ST segment depression of the EGG and a
plasma troponin concentration of 4365 ng/L

downloaded from www.medicalbr.com


CARDIOLOGY • 143

Answers
16.1. Answer: B. hypertension), the second heart sound may
In a patient with poorly controlled hypertension, be loud. Postural hypotension will have little
aortic dissection should be considered as a effect on the intensity of heart sounds at rest.
potential cause of acute chest pain. While Aortic incompetence is often associated
interscapular pain is a common feature of acute with a quiet second heart sound, and mitral
aortic dissection, the presentation is highly incompetence with a quiet or absent first heart
variable and central chest pain commonly sound. A mechanical mitral valve replacement
occurs. If antiplatelet or antithrombotic drugs will produce a loud mechanical first heart
are given before excluding this diagnosis, fatal sound.
bleeding may occur.
16.6. Answer: B.
16.2. Answer: C. Marfan's syndrome is a connective tissue
Orthopnoea refers to breathlessness occurring disorder that is associated with abnormal
immediately on lying flat, whereas the term production of elastic tissues. This can affect the
'paroxysmal nocturnal dyspnoea' refers to aorta, aortic root and aortic valve. Aortic root
sudden episodes of breathlessness occurring dilatation can lead to aortic regurgitation and is
at night -time. It can occur with respiratory also associated with increased risk of aortic
pathologies such as chronic obstructive dissection. Aortic regurgitation occurs with
pulmonary disease but is most often associated onset at the beginning of diastole, as soon as
with heart failure. It is caused by the aortic valve closes, and produces an early
gravity-dependent changes in pulmonary diastolic murmur. Myotonic dystrophy is
capillary hydraulic pressure leading to alveolar associated with dilated cardiomyopathy and
oedema. conducting system problems, which can lead
to atrioventricular block and ventricular
arrhythmias. Long OT syndrome is an inherited
16.3. Answer: B.
arrhythmia syndrome that is not usually
The most common cause of a rapid, irregular
associated with any structural cardiac
rhythm in the elderly is atrial fibrillation. In
abnormality. Mitral valve prolapse produces a
patients with very frequent atrial or ventricular
late systolic murmur. Wolff-Parkinson-White
ectopic beats, the pulse is also very irregular
syndrome is rarely associated with structural
but a regular pattern can usually be perceived
cardiac abnormalities (which are Ebstein's
within it.
anomaly and rarely hypertrophic
cardiomyopathy) and is not associated with
16.4. Answer: E. aortic incompetence.
The internal jugular vein is in direct continuity
with the right atrium, and there is no venous 16.7. Answer: D.
valve between the two. The JVP therefore is a Clinical features of acute limb ischaemia include
reflection of right atrial pressure, which pallor, pain, pulselessness, paraesthesia and
becomes elevated in conditions where either 'perishing-with-cold' -the five 'P's. Deep
there is increased resistance to right ventricular venous thrombosis would cause limb swelling,
ejection (e.g. pulmonary hypertension due to venous engorgement, and a dusky blue
chronic lung disease, or recurrent pulmonary discoloration, and this does not affect arterial
embolism) or mechanical dysfunction of the flow. In cardiac failure, peripheral blood flow is
right heart (e.g. right ventricular infarction, not sufficiently reduced to cause limb ischaemia
right-sided valve disease). except in cardiogenic shock. In a patient with a
history of atrial fibrillation.(ernbolisation from
16.5. Answer: B. the left atrial appendage is the most likely
The second heart sound, which occurs at the cause of limb ischaemia. Aspirin does not
beginning of ventricular diastole, occurs when provide effective prophylaxis against this and
the aortic and pulmonary valves close. When current guidelines recommend the use of
either aortic or pulmonary artery diastolic warfarin or a direct oral anticoagulant such as
pressure is high (e.g. in essential or pulmonary apixaban.

downloaded from www.medicalbr.com


144 • CARDIOLOGY

16.8. Answer: C. plasma troponin concentration takes time to


Infective endocarditis is often diagnosed become detectable. The admission troponin
relatively late in its clinical course. It may initially level may be normal if the patients attends
present with non-specific symptoms that lead soon after the onset of symptoms. If the
to a diagnosis of influenza or viral infection. Any 6-hour troponin level is normal then acute
patient with unexplained fever and a cardiac coronary syndrome is not likely to explain the
murmur, especially if changing, should be patient's chest pain and other causes should
assessed for possible endocarditis, with then be considered. An elevated troponin level
urinalysis, an ECG, echocardiogram, blood is suspicious of myocardial infarction
cultures, and blood testing for white cell count but should be interpreted in the context of the
and C-reactive protein concentration. In this clinical presentation. Some non-cardiac
case the wide pulse pressure is suggestive of pathologies (e.g. sepsis, pulmonary embolism)
aortic incompetence which, if severe, may are also commonly associated with minimal
occur without a murmur. myocardial injury and therefore troponin
release.
16.9. Answer: E.
After myocardial infarction, haemodynamic 16.12. Answer: A.
compromise associated with a new murmur Atrial fibrillation is the most common
may be caused by either papillary muscle tachyarrhythmia encountered in older patients
rupture, or rupture of the interventricular and is seen in approximately 2% of patients
septum (acquired ventricular septal defect; aged over 70 years, and in some studies up to
VSD). With acquired VSD the murmur often 10% of those aged over 80 years. Ventricular
radiates to the right sternal border because of ectopic beats would not produce episodic
left-to-right shunting across the interventricular symptoms of this type and sinus arrhythmia is
septum, whereas the murmur of acute mitral a normal variant and would not cause any
incompetence would be more likely to radiate symptoms. Supraventricular tachycardia
to the axilla or the back. Acute left ventricular normally causes regular palpitation.
free wall rupture is almost always fatal and
would not cause a murmur. While pericarditis 16.13. Answer: D. I
may cause a sound that could be confused Basic life support describes the intervedtions
for a murmur, serious haemodynamic that can be carried out with minimal e~uipment
compromise is rare, as the associated in the event of a cardiac arrest. It does not
pericardia! effusion is usually small. Aortic include defibrillation or administration of
incompetence is not a complication of intravenous drugs. It does include chest
myocardial infarction. compression and mouth-to-mouth !
resuscitation, but the ABCDE mnemonic is a
16.10. Answer: B. helpful aide memoire for these and the other
Clinical signs of left ventricular failure are components of basic life support.
tachycardia, a gallop rhythm with a third heart
sound (which is the sound of abrupt left 16.14. Answer: E.
ventricular filling due to high left atrial pressure), Pulseless electrical activity means that
and bi-basal inspiratory fine crepitations at the there is an organised cardiac rhythm seen
lung bases. A fourth heart sound occurs on the ECG, but no discernible cardiac output.
during atrial systole because of increased left Defibrillation is not appropriate, as this is a
ventricular stiffness in patients with left treatment for ventricular fibrillation.
ventricular hypertrophy. A loud second heart Amiodarone can cause hypotension and is
sound is usually caused by systemic or not an appropriate treatment. In current
pulmonary hypertension. A quiet first heart resuscitation protocols, CPR should be carried
sound may accompany mitral out for 2 minutes before the rhythm is
regurgitation. reassessed. Reversible causes of PEA include
hypothermia, hypoxia, hypovolaemia, hypo-/
16.11. Answer: A. hyperkalaemia (the four 'H's), and
Troponin testing is an important component in thrombosis (coronary or pulmonary), tension
the assessment of patients with chest pain. pneumothorax, tamponade and toxins
In patients with acute myocardial infarction, (the four 'T's). ·

downloaded from www.medicalbr.com


CARDIOLOGY • 145

16.15. Answer: E. 16.20. Answer: C.


While peripheral oedema and ascites are First -line therapy for rate control in atrial
signs of right-sided cardiac failure, they typically fibrillation consists of ~-blockade (or, if
take days or weeks to develop. Acute contraindicated, a rate-limiting calcium channel
right ventricular failure is characterised by blocker such as verapamil can be used). In this
hypotension, a compensatory sinus case, the ~-blocker could be prescribed in
tachycardia, elevation of the jugular venous place of enalapril, as it may provide quite
pulse because of ineffective right ventricular effective blood pressure control, as well as
ejection, and hepatomegaly can develop quite limiting the heart rate. None of the other agents
quickly because of hepatic venous are appropriate for rate control in atrial
congestion. fibrillation. Lidocaine is used to treat ventricular
arrhythmias. Flecainide and amiodarone are
16.16. Answer: C. used for rhythm control (i.e. maintenance of
Starling's Law describes the relationship sinus rhythm) and not rate control, in atrial
between cardiac filling (preload) and cardiac fibrillation. Adenosine is an ultra-short -acting
output. Low preload causes inadequate atrioventricular (AV) node blocker and is not
ventricular filling and low output. Moderate used to treat atrial fibrillation.
preload causes optimal cardiac filling and
cardiac output. Very high preload causes 16.21. Answer: C.
ventricular stretch and reduces the efficiency of In Mobitz type II second-degree AV block, most
contraction, resulting in reduced cardiac output. P waves conduct normally to the ventricles and
Patients with decompensated cardiac failure are associated with a QRS complex. Some P
have high preload pressure, and diuretics and waves do not conduct and there is no

--
vasodilator medication can reduce this and preceding increase in the P-R interval before
improve cardiac function. the blocked P wave. This reflects block in the
His-Purkinje system where conduction is
16.17. Answer: A. 'ali-or-nothing'. In contrast, Mobitz type I
Cardiac failure is associated with activation of second-degree AV block is characterised by 1
the sympathetic nervous system and RAAS. progressive lengthening of the P-R interval
The resulting production of noradrenaline block. This reflects block in the AV node itself, .
(norepinephrine) and angiotensin II cause where conduction is 'decremental', i.e. the AV1
peripheral vasoconstriction. BNP production node exhibits signs of 'fatigue' with each /
increases in cardiac failure in response to successive beat.
ventricular stretch.
16.22. Answer: E.
16.18. Answer: B. Sinoatrial disease is characterised by
Loop diuretics interfere with the countercurrent abnormalities of sinus rate, and atrial
sodium exchanger in the loop of the nephron. arrhythmias such as atrial flutter, atrial
This prevents water reabsorption and results in tachycardia and atrial fibrillation. Ventricular
loss of sodium and water (natriuresis). arrhythmias are not commonly associated with
this condition.
16.19. Answer: D.
~-Blockers have several beneficial effects in 16.23. Answer: D.
chronic cardiac failure - improvement of The CHA2 DS 2 -VASc score is used to assess
diastolic filling, reduction of myocardial stroke risk in patients with atrial fibrillation (and
ischaemia, and prevention of ventricular atrial flutter). The mnemonic takes account of
arrhythmias and atrial fibrillation, ~-Blockers clinical risk factors for stroke (C, congestive
reduce heart rate so should not be heart failure = 1 point; H, hypertension = 1
used if the patient is already bradycardic. In point; A2 , age 2 75 years/2 points; D,
acute cardiac failure (e.g. acute left ventricular diabetes mellitus = 1 po1nt; S2 , previous
failure or cardiogenic shock),. in which left stroke or transient ischaemic attack = 2 points;
ventricular systolic function i(l acutely V, vascular disease= 1 point; A, age 65-74
compromised, ~-blockers should not be used years = 1 point; Sc, sex category female = 1
as they may further impair systolic point). In this case, the score is 5 points (2
function. points for age 275 years, 1 point each for

downloaded from www.medicalbr.com


146 • CARDIOLOGY

female gender, diabetes and hypertension). comorbidities, age is not a barrier to


This is associated with quite a high risk of implantation. ICDs are not indicated for patients
stroke (approximately 5% annual risk if who have experienced ventricular arrhythmias
untreated) and this patient should be due to reversible factors (e.g. drug misuse) or
considered for oral anticoagulation. in the acute phase of myocardial infarction, as
subsequent risk of similar arrhythmias is
16.24. Answer: B. generally low. Patients with sinoatrial disease or
Antiplatelet drugs are no longer recommended AV nodal block without ventricular arrhythmia
for stroke prevention in atrial fibrillation, are treated with a permanent pacemaker, not
although they are effective at preventing stroke an lCD.
due to carotid vascular disease. Amiodarone
and ~-blockers can help prevent atrial fibrillation 16.28. Answer: D.
episodes but are not known to reduce stroke The only one of these conditions associated
risk. Apixaban is an oral factor Xa inhibitor, with a significant intracardiac shunt is tetralogy
which has been shown in large-scale clinical of Fallot. Central cyanosis occurs because of
trials to be effective at preventing stroke in shunting of blood through a ventricular septal
patients with atrial fibrillation and moderate-to- defect, and this is exacerbated by the
high stroke risk. over-riding aorta (i.e. the aorta over-rides the
defect, causing blood from the right ventricle to
16.25. Answer: E. be ejected directly into the aorta) and by
This EGG shows a narrow, complex tachycardia muscular right ventricular outflow obstruction.
with no obvious P waves. The P waves may be Cyanotic episodes may be precipitated by fever
concealed in the QRS complex or ST segment or by dehydration. In most cases the condition
The term 'supraventricular tachycardia' is used is recognised and corrected in infancy.
to describe this rhythm. The two most likely
mechanisms are atrioventricular nodal 16.29. Answer: D.
re-entrant tachycardia (AVNRT) or Eisenmenger's syndrome occurs in patients
atrioventricular re-entrant tachycardia (AVRT). with untreated intracardiac shunts such as atrial
The key to terminating these tachycardias is to or ventricular septal defects. Initially shuntil')g is
from the left to the right side of the heart, ~nd
cause transient block in the AV node and the
1
quickest and least invasive way of doing this is central cyanosis does not occur. The resl)ionse
by using vagal manoeuvres such as carotid to increased pulmonary blood flow is pul~onary
sinus pressure or the Valsalva manoeuvre. vasoconstriction, which leads to permanent
sclerotic changes in the pulmonary
16.26. Answer: G. microvasculature. This causes right heart
Pacemakers are used to treat or prevent pressure to increase to the point it exceeds left
bradycardia and the main indications are heart pressure. Shunt reversal and central (and
symptomatic sinoatrial disease and AV nodal peripheral) cyanosis then occur. Breathlessness
disease. Pacemakers are not effective at and fatigue are common symptoms. Patients
preventing atrial fibrillation or supraventricular with Eisenmenger's syndrome have markedly
tachycardia. Sinus bradycardia in an athlete is a reduced life expectancy because of cardiac
normal, physiological finding that requires no failure and cardiac arrhythmias. Patent foramen
treatment An implantable cardiac defibrillator ovale is not a cause of Eisenmenger's
(lCD), not a permanent pacemaker, is used to syndrome and it does not cause significant
prevent sudden death due to ventricular intracardiac shunting.
arrhythmias in vulnerable patients.
16.30. Answer: E.
16.27. Answer: E. Ventricular septal defect (VSD) causes a harsh
ICDs are indicated for primary prevention in systolic murmur that may radiate to the right
patients with previous myocardial infarction who side of the sternum. Small VSDs do not cause
have chronically impaired left ventricular significant shunting but can produce a loud
function. It is thought that the scar burden in murmur. Atrial septE,~I defect might cause a
these patients predisposes them to ventricular quiet systolic flow murmur. Persistent ductus
arrhythmias, which, when they occur, are arteriosus causes a continuous murmur
poorly tolerated. As long as there are no throughout systole and diastole. Patent

downloaded from www.medicalbr.com


CARDIOLOGY • 147

foramen ovale produces no abnormal proteins such as troponins, tropomyosin,


auscultatory findings. Mitral valve prolapsed myosin heavy chain, actin and actin-binding
causes a late systolic murmur and is not proteins, among many, but cardiac sodium
referred to as a 'hole' in the heart. channel gene mutations predispose to cardiac
arrhythmias by causing long OT syndrome or
16.31. Answer: A. Brugada syndrome.
Pericarditis is associated with friction between
the epicardial surface of the heart and the 16.35. Answer: A.
pericardia! sac. This causes a scratchy Hypertrophic cardiomyopathy is characterised
to-and-fro sound in time with the cardiac cycle, by left ventricular hypertrophy. This is often
which is distinct from a murmur. It is associated asymmetric with the interventricular septum
with pleuritic chest pain, which may be affected classically affected. There are other variants,
by sitting forward or backward. Heart sounds such as apical hypertrophic cardiomyopathy.
are either normal or, if there is a large
pericardia! effusion, diminished. It may occur in 16.36. Answer: D.
the context of flu-like illness and a viral Cardiac transplantation is limited by the
aetiology is common. Endocarditis is not availability of donor organs, the need for
associated with pleuritic chest pain. Persistent life-long immunosuppressive therapy to prevent
ductus arteriosus is a congenital (rather than rejection, and the risks of surgery and the
acute) condition, which is associated with a drugs used afterwards. Therefore it is only
continuous murmur. offered to patients with cardiac failure who
remain symptomatic despite adherence with
16.32. Answer: D. optimal pharmacological therapy and, where
Aspirin, through its anti-inflammatory effects, is appropriate, cardiac resynchronisation therapy.
a very effective symptomatic treatment for
pericarditis. Non-steroidal anti-inflammatory 16.37. Answer: E.
drugs such as diclofenac can also be used Hypertrophic cardiomyopathy is associated with
orally. Steroids are rarely required. Amiodarone disorganisation and fibrosis of left ventricular
is an anti-arrhythmic drug and has no role in myocardial tissue. This can predispose patients 1
the management of acute pericarditis. to sudden ventricular arrhythmias. and these 1
may occur without warning during intense 1
16.33. Answer: A. exercise. The risk is highest in patients with /
Dilated cardiomyopathy is characterised by gross hypertrophy or left ventricular outflow
dilatation of the atria and ventricles, and tract obstruction. Sorne genetic variants are
thinning of ventricular walls. Hypertrophic also associated with high risk, such as trop6nin
cardiomyopathy causes disproportionate T mutations. Right ventricular failure and
thickening of myocardium, particularly the pulmonary ernbolisrn are not common in
interventricular septum. Myocardial infiltration patients with hypertrophic cardiomyopathy.
(e.g. with amyloid protein) can cause restrictive Atrial fibrillation occurs and rnay cause
cardiomyopathy, which does not cause cardiac syrnptorns but is rarely life-threatening.
dilatation but does restrict myocardial
contraction and relaxation. 16.38. Answer: B.
Loop diuretics have no effect on mortality in
16.34. Answer: B. patients with cardiac failure. Rate-limiting
Cigarette smoking is a leading' cause of calcium channel blockers such as diltiazern and
cardiovascular disease but its main influence is veraparnil are usually avoided, as they have a
on the genesis of atherosclerosis and coronary negative inotropic effect, which rnay aggravate
artery disease. Likewise, obesity is associated cardiac failure. Aspirin and percutaneous
with risk of hypertension and type 2· diabetes coronary intervention are)featrnents for
mellitus, but is not a risk factor for coronary artery disease,. not cardiomyopathy.
cardiomyopathy. Hypercholesterolaemia may
have dietary and genetic components and is a 16.39. Answer: A.
risk factor for coronary artery disease, not Takotsubo (stress) cardiomyopathy occurs
cardiomyopathy. Dilated cardiomyopathy can most often in females and is associated with
be caused by genetic defects of sarcomeric emotional stress. It can occur due to

downloaded from www.medicalbr.com


148 • CARDIOLOGY

bereavement, acute non-cardiac illness, natural


disasters and other major life events. It is
characterised by chest pain and ECG changes
relieve symptoms and to obtain fluid for
laboratory analysis. Patients with pericardia!
effusion are very dependent on high preload
I
i

that mimic myocardial infarction. Troponin pressure to maintain cardiac output, so


elevation is common but coronary angiography diuretics may cause significant hypotension.
does not show occlusive coronary artery disease Large effusions may occur because of
or intracoronary thrombus. Echocardiography malignancy, usually metastatic disease from
shows a characteristic left ventricular lung or breast cancer.
appearance of apical dilatation, giving the
appearance of an octopus trap or takotsubo! 16.44. Answer: B.
'Saddle' ST segment elevation is a common
16.40. Answer: A. feature of acute pericarditis, but it can be
Alcohol has many negative effects on health. confused with an ST segment elevation
These include liver disease, pancreatitis, myocardial infarction, Brugada syndrome, and
hypertension and cognitive dysfunction. It also a normal variant in some ethnic groups such as
causes many behavioural and social problems, those of African or Caribbean descent. In
particularly if alcohol dependency occurs. contrast, PR interval depression is very specific
Cardiac effects include atrial fibrillation and to pericarditis and, when seen, is usually
dilated cardiomyopathy, both of which diagnostic.
may be reversible if the patient abstains
early enough. 16.45. Answer: D.
Sinus tachycardia is the most common ECG
16.41. Answer: D. 'abnormality in pulmonary embolism, although
Atrial myxoma is the most common cardiac atrial fibrillation may also occur. The next
tumour and 75% or more occur in the left commonest ECG change is anterior T-wave
atrium. Large tumours may partially obstruct inversion due to right ventricular wall stress.
the mitral valve, affecting cardiac output and The S103T3 (large S wave in lead I, 0-wave
causing a tumour 'plop' on auscultation. and T-wave inversion in lead Ill) pattern is
Tumours are benign but can be associated commonly absent but, when present, is m.ore
with cerebral and peripheral embolism (which is specific to massive pulmonary embolism./
how they often first present), so surgery is I
usually indicated to prevent this. 16.46. Answer: D. I
Although elevation of the JVP and peripheral
16.42. Answer: E. oedema often occur with chronic pericardia!
Chronic pericardia! constriction is a late effusion, they are not specific signs of cardiac
complication of tuberculous and viral tamponade. Here, pulsus paradoxus and
pericarditis and is caused by pericardia! fibrosis, Kussmaul's sign (the JVP falling on inspiration)
contraction and adhesion to the epicardium. are specific signs. Pulsus paradoxus is an
It can also complicate chronic inflammatory exaggeration of physiological variation in blood
disorders such as rheumatoid disease. Acute pressure caused by compression of the heart
myocardial infarction can lead to acute in the pericardia! sac, and is characterised by a
post -infarct pericarditis, but this almost never large fall in blood pressure during inspiration.
leads to pericardia! constriction.
16.47. Answer: C.
16.43. Answer: A. All of the agents listed except furosemide have
Large pericardia! effusions are normally not been shown to improve survival in patients with
associated with a pericardia! rub as the heart failure due to left ventricular systolic
pericardium and epicardium are well separated dysfunction. Loop <diuretics such as furosemide
by pericardia! fluid and friction does not occur. are important for symptom control, but so far,
The ECG may show small complexes but is not no large-scale randomised trial has shown
a sensitive test, and an echocardiogram is survival benefit.
required to make the diagnosis. The chest
X-ray may show a spherical or globular 16.48. Answer: C.1

cardiac silhouette. In symptomatic patients, P2Y12 receptor antagonists inhibit adenosine


percutaneous pericardia! drainage is used to diphosphate (ADR')-dependent platel~t

downloaded from www.medicalbr.com


CARDIOLOGY • 149

activation and all of the agents listed except by diastolic dysfunction - the inability of the left
dipyridamole act via this receptor. Dipyridamole ventricle to fill properly in diastole.
is a phosphodiesterase inhibitor, which blocks
the response to ADP by inhibiting breakdown 16.52. Answer: D.
of cyclic adenosine monophosphate (cAMP) Both the sympathetic nervous system and the
and inhibits the re-uptake of adenosine into RAAS systems are activated in heart failure.
platelets. Vasopressin may also be released from the
posterior pituitary in response to reduced ·
16.49. Answer: E. cardiac output. Thyroid hormone levels are
smoking is by far the strongest. modifiable risk generally unaffected in cardiac failure but
factor for coronary artery disease. Obesity is profound hypo- or hyperthyroidism can cause
associated with hypertension, type 2 diabetes heart failure.
and unfavourable lipid profile, and is thus
associated with risk of myocardial infarction. 16.53. Answer: E.
High levels of dietary saturated fat (e.g. from Troponin I is a structural myocardial protein
red meat and processed meat products) are subunit, and not an enzyme. Along with the
also known to be associated with increased other markers listed, it is released into the
cardiovascular risk. blood stream after acute myocardial infarction
from injured myocardial tissue.
16.50. Answer: E.
Both percutaneous coronary intervention and 16.54. Answer: C.
fibrinolytic drug therapy are treatment If the patient has occluded his stent, then
modalities for acute ST elevation myocardial the EGG will show an acute inferior ST
infarction. Both treatments aim to re-open the segment elevation myocardial infarction.
culprit coronary vessel to restore perfusion to Electrocardiographic features of acute inferior
the infarct territory. In randomised studies, myocardial infarction include ST segment
administration of tPA or other fibronolytic drugs elevation in the inferior leads (II, Ill and aVF) and
had a strongly time-dependent beneficial effect. sometimes atrioventricular block.
If administered more than 8-10 hours after the
onset of symptoms, risk of treatment begins to 16.55. Answer: A.
outweigh benefit. As fibrinolytic drugs take Sudden, severe pulmonary oedema after
time to work, and may not completely restore myocardial infarction may be a sign of a
flow in the culprit vessel, they are best mechanical complication. Acute papillary
administered early. Percutaneous coronary muscle rupture causes sudden and very severe
intervention and the other therapies described mitral regurgitation, which, in turn, is
do not have such a time-dependent effect on complicated by pulmonary oedema. Acute
outcome. When primary percutaneous coronary pericarditis causes sharp chest pain but does
intervention cannot be provided within 2 hours, not cause pulmonary oedema. Free wall rupture
fibrinolytic therapy should be administered usually causes pulseless electrical activity (PEA)
immediately. cardiac arrest and is almost always fatal. Atrial
septal defect is not a complication of
16.51. Answer: E. myocardial infarction. Left ventricular mural
Dilated cardiomyopathy, myocarditis and thrombus is usually asymptomatic, and is
myocardial infarction all reduce left ventricular detected on echocardiography. It can lead to
systolic function and are associated with low stroke and peripheral embolism.
left ventricular ejection fraction (LVEF), a
measure of the percentage of left ventricular 16.56. Answer: E.
blood ejected in systole. Aortic stenosis is Ventricular fibrillation is an, early complication
associated with either normal LVEF, or if of acute myocardial info/6tion and is the
severe, sometimes low LVEF. Restrictive leading preventable cause of death. Early
cardiomyopathy is associated with myocardial recognition of myocardial infarction is therefore
infiltration and sometimes reduction in left important. Sudden death rates rnay be reduced
ventricular cavity size, but normal systolic by education of the public about symptoms
function. LVEF is high but stmke volume low of myocardial infarction and the need to
due to small cavity size. Heart failure is caused seek immediate medical help, and by the

downloaded from www.medicalbr.com


,
'"I 150 • CARDIOLOGY

now-ubiquitous placement of external can cause limb ischaemia because of its


defibrillators in emergency ambulances. association with stroke and peripheral
Community first responder programmes and embolism. Diabetes mellitus is associated with
public access defibrillation are other strategies atherosclerotic and microvascular disease
that allow a more rapid response to myocardial and is strongly linked with limb ischaemia;
infarction and cardiac arrest in rural areas. however, it would be unusual in a
normal-weight individual of this age without
16.57. Answer: A. symptoms.
Diabetes mellitus has been shown in large
cohort studies to be protective against the risk 16.61. Answer: C.
of development of abdominal aortic aneurysm, While control of blood pressure is important in
and where aneurysm is present, the rate of type A aortic dissection, through use of
enlargement is slower than in non-diabetics. ~-blockers or other antihypertensive agents, it
The reason for this negative association is is early surgery that has the greatest effect on
unclear. mortality. Type A aortic dissection involves the
ascending aorta and patients may die because
16.58. Answer: A. of cardiac tamponade, aortic rupture, or
Acute limb ischaemia leads to pallor, pain, dissection into downstream arteries resulting in
pulselessness, paraesthesia and ischaemia of limbs or organs. The most
'perishing-with-cold' -the five 'P's. Chronic effective way of preventing this is to repair the
limb ischaemia is associated with hair loss in entry point of the dissection in the ascending
the affected limb. Capillary refill time is a aorta. Anticoagulation is contraindicated in
measure of peripheral perfusion and is tested ·acute aortic dissection as it may cause fatal
by squeezing the skin over the fingers or toes bleeding.
until it blanches, then assessing the time taken
for colour to fully return. A capillary refill time of 16.62. Answer: D.
<2 seconds is a sign of good peripheral Hypertension, because of its population
perfusion and if >3 seconds is a sign of prevalence, is the leading cause of aortic
reduced peripheral perfusion. dissection; however, this would have been
picked up on antenatal checks in this ca~e.
16.59. Answer: C. Marfan's syndrome (usually associated with tall
~-Blockers and ACE inhibitors help reduce stature) and coarctation of the aorta arJ'
arterial wall stress and, through their role in relatively uncommon conditions, but both have
controlling hypertension, may help reduce risk a strong association with aortic dissection.
of aortic aneurysm expansion and rupture. Intramural haematoma refers to spontaneous
Statins reduce the rate of progression of bleeding into the aortic wall and may be the
atherosclerosis and may help reduce risk of precursor to aortic dissection. Pregnancy-
rupture through cholesterol-dependent and associated dissection is rare, but when it
cholesterol-independent effects. However, of all occurs it is usually in the third trimester or
interventions, smoking cessation has the postpartum period, and is more likely to occur
greatest effect in reducing the risk of aneurysm in patients with predisposing conditions such
rupture. as Marfan's syndrome.

16.60. Answer: C. 16.63. Answer: C.


Atherosclerotic peripheral vascular disease is Primary percutaneous coronary intervention
the most common cause of limb ischaemia. (PPCI) is more effective at reperfusing the
Buerger's disease is a form of obliterative infarct -related territory than fibrinolysis with
arteritis affecting small and medium-sized streptokinase or tPAy Fibrinolytic drugs may not
vessels, strongly associated With cigarette reach the site of v~sel occlusion if there is no
smoking. It causes limb ischaemia and flow, and will do nothing to treat the culprit
gangrene, and presents at a relatively young occlusive atherosclerotic lesion. PPCI usually
age. Raynaud's disease is a vasospastic completely restore~ blood flow by fragmenting
condition associated with some connective the clot and by opening up the site of stenosis.
tissue disorders. It can cause digital ischaemia It is associated with lower mortality apd lower
and in some cases infarction. Atrial fibrillation rates of subsequent angina and re-infarction.

downloaded from www.medicalbr.com


CARDIOLOGY • 151

Coronary artery bypass surgery is not used to 16.69. Answer: C.


treat acute myocardial infarction but is an Mitral stenosis is characterised by the presence
effective treatment for some patients with of a tapping apex beat, reflecting a palpable
chronic coronary artery disease. opening snap, accompanied by a low-pitched
mid-diastolic murmur. If the patient is in sinus
16.64. Answer: A. rhythm, pre-systolic accentuation of the
There is a strong association between age and murmur may occur because of atrial
risk of death after myocardial infarction. contraction. A loud second heart sound may
In-hospital mortality is three times greater in be heard due to secondary pulmonary
individuals aged over 80 years than it is in hypertension, which often accompanies mitral
those aged 60-65 years. While risk of stenosis.
myocardial infarction is much higher in smokers
than in non-smokers to start with, the risk of 16.70. Answer: A.
death in smokers after myocardial infarction is A thrill is indicative of aortic stenosis or
lower than in non-smokers, probably because hypertrophic obstructive cardiomyopathy, both
their main risk factor is modifiable. EGG of which are not associated with a parasternal
changes and troponin concentration are not heave. A parasternal heave occurs because of
good predictors of mortality risk. Cardiac arrest right ventricular hypertrophy and does not
within 24 hours of myocardial infarction is an cause a thrill. Conditions which lead to
effect of acute ischaemia and does not predict pulmonary hypertension (e.g. mitral stenosis,
risk of sudden death after discharge from chronic lung disease and atrial septal defect)
hospital. may therefore cause right ventricular
hypertrophy and a parasternal heave.
16.65. Answer: D.
There are many uncommon endocrine causes 16.71. Answer: C.
of hypertension, including those listed, but renal Aortic regurgitation is associated with a
disease is the most common cause. large-volume, collapsing pulse and an early
diastolic murmur associated with a systolic
16.66. Answer: D. 'flow' murmur. In severe aortic regurgitation,
All other options given apart from D describe the pulse pressure may be so large as to cause
recognised causes of secondary hypertension. prominent neck pulsation. A slow rising pulse,
Antihypertensive drug therapy, along with crescendo-decrescendo murmur, quiet second
lifestyle changes, effectively controls blood heart sound and palpable thrill in the aortic area
pressure in most patients with hypertension. are signs of aortic stenosis.
The most common cause of poor blood
pressure control is therefore poor adherence 16.72. Answer: E.
with antihypertensive therapy. This may be Viridans streptococci are the most common
because of side-effects, and also because of cause of endocarditis on a native heart valve.
the asymptomatic nature of the condition. Staphylococcus aureus is the most common
organism to infect prosthetic valves.
16.67. Answer: D.
The revised Jones criteria are used to diagnose 16.73. Answer: A.
rheumatic fever. The condition is diagnosed Cutaneous signs of endocarditis (options B, D
if two major criteria are met, or one major and E) are not seen in most patients with the
and two minor criteria are met. Carditis, condition, but when present, are highly
subcutaneous nodules, erythema marginatum diagnostic of it. Roth's spots (seen on
and chorea are all major criteria, whereas fundoscopy) are also relatively uncommon.
elevation of C-reactive protein (or erythrocyte Haematuria (often microscopic) is a common
sedimentation rate) is a minor criteFion. manifestation of endoca;d~is.

16.68. Answer: A. 16.74. Answer: B.


Aspirin is the drug of choice in rheumatic fever Viridans streptococci an~ usually very sensitive
and is used in high doses compared with those to benzylpenicillin, and this agent works
used in common analgesia, Glucocorticoids are synergistically with gentamicin. Bactericidal
not used in this condition. blood concentrations can only be achieved with

downloaded from www.medicalbr.com


~
I II

!I
I
152 • CARDIOLOGY

frequent intravenous dosing. Ampicillin is not as


effective, and flucloxacillin and vancomycin are
of myocardial fibrosis. In addition to assessing
scar burden and distribution after myocardial
principally used to treat staphylococcal infarction, it is also helpful in the diagnosis of
infections. and risk stratification in cardiomyopathies,
because of the association between myocardial
16.75. Answer: A. fibrosis, these conditions, and risk of ventricular
Hypertrophic cardiomyopathy is often familial, arrhythmias. It is also useful to help guide the
and the most common mode of inheritance is likelihood of success from coronary artery
autosomal dominant. bypass graft surgery.

16.76. Answer: A. 16.81. Answer: B.


Mutations in myosin heavy chain, troponin and The main components in the management of
myosin-binding protein most often lead to acute pulmonary oedema are bed rest, oxygen
hypertrophic cardiomyopathy. Titan mutations therapy, intravenous nitrates and intravenous
(and some myosin-binding protein mutations) diuretics. Non-invasive continuous positive
may cause dilated cardiomyopathy. It is airway pressure (CPAP) ventilation is helpful in
mutations in fibrillin, a glycoprotein critical to resistant cases. Dobutamine is an inotrope that
production of elastic tissue, that most often increases cardiac work; it is sometimes used in
leads to Marfan's syndrome. the management of cardiogenic shock, but is
not appropriate in a patient with high blood
16.77. Answer: B. pressure and cardiac failure.
Atrial myxoma is the most common cardiac
tumour. It is a benign tumour that usually 16.82. Answer: B.
occurs in the left atrium and is associated with Endocrine causes of dilated cardiomyopathy,
increased risk of stroke and peripheral and alcohol-related cardiomyopathy, are often
embolism. reversible as long as the underlying problem
is treated early enough. Anthracycline
16.78. Answer: C. chemotherapy can cause acute or late-onset
Transthoracic echocardiography is a form of dilated cardiomyopathy that responds Of11Y in a
ultrasound imaging that has limitations. It is limited manner to J3-blockers and ACE lhhibitors
good for assessing heart valve and myocardial and which may cause permanent cardi'ac
function but has limited value in characterising dysfunction. /
tissues (e.g. for fibrosis). The left atrial
appendage is the most common site for 16.83. Answer: C. I
thrombus formation in atrial fibrillation and this Non-ST segment myocardial infarctibn is I
structure is not visible during transthoracic normally initially managed with dual antiplatelet i
echocardiography. The electrocardiogram, not therapy (e.g. aspirin and ticagrelor), and an
echocardiogram, is used to assess cardiac antithrombotic agent (e.g. fondaparinux or
arrhythmias. Whilst poor left ventricular function enoxaparin). J3-Biockade is often used as
is associated with a poor future prognosis, in prophylaxis against angina and arrhythmias.
isolation, echocardiography gives limited Intravenous tPA is a treatment for acuteST
information about prognosis. elevation myocardial infarction and has not
been shown to improve outcome in patients
16.79. Answer: B. with non-ST segment elevation myocardial
The decision between percutaneous coronary infarction. Indeed, patients with ST segment
intervention (PCI) and coronary artery bypass depression have a worse outcome with
graft surgery is an important one in patients thrombolytic therapy.
with angina or after myocardial infarction. The
patients who have the most·to gain from 16.84. Answer: A.
surgery are those with left main stem disease Surgery is associated with activation of platelets
and left ventricular impairment. and coagulation pathways, so patients who
have had recent Qlyocardial infarction or recent
16.80. Answer: D. percutaneous coronary .intervention are at
Gadolinium-enhanced MRI is currently the most increased risk of thrombosis in the affected
sensitive imaging modality for the identification vessel, resulting in myocardial infaretion.

downloaded from www.medicalbr.com


CARDIOLOGY • 153

Patients with left ventricular impairment are at abnormalities develop are likely to have a high
increased risk of acute cardiac failure and ischaemic threshold and are not at high risk of
haemodynamic problems in the perioperative major cardiovascular events. Conversely,
phase. Insulin-treated diabetic patients and patients with new-onset, rapidly progressive, or
those with renal failure may have occult limiting symptoms may have critical coronary
coronary artery disease and are at increased artery disease. Patients with poor left ventricular
risk of perioperative myocardial infarction. Aortic function have poor cardiac reserve and carry
stenosis with a relatively small peak pressure higher than average risk because they tolerate
gradient is not likely to cause haemodynamic myocardial ischaemia poorly.
problems during or after surgery.
16.86. Answer: D.
16.85. Answer: E. Smoking is the strongest risk factor for the
Exercise tolerance testing can be used to development of coronary artery disease.
identify patients with coronary artery disease More than any other lifestyle modification, or
who have a low threshold for myocardial any other preventative therapy, smoking
ischaemia. Patients who can exercise into cessation makes the largest difference to
stage 3 of the Bruce Protocol before ECG cardiovascular risk.

I'

downloaded from www.medicalbr.com


A Leitch

Respiratory medicine
Multiple Choice Questions
17.1. A 46 year old woman has a recent diagnosis small vessel disease. Arterial blood gas: W
of adenocarcinoma of the lung made at 60 nmoi/L (pH 7.22), Pa0 2 8.7 kPa
bronchoscopy 1 week ago. She presents to the (65 mmHg), PaC02 10 kPa (75 mmHg),
emergency department acutely short of breath HC0 3- 26 mmoi/L. What is the most likely
with a non-productive cough. She has an ache in .cause of her deteriorating conscious level?
the centre of her chest that is made worse by A. Cholesterol embolism - ventilation/perfusion
breathing in. She is apyrexial. Oxygen saturations (i/16.) mismatch
are 91% on 40% oxygen. Respiratory rate is B. Chronic obstructive pulmonary disease
30 breaths/min. Blood pressure (BP) is (COPD) with oxygen toxicity - loss of hypoxic
100/65 mmHg and pulse is 110 beats/min. drive
Examination reveals decreased expansion of C. Flail segment due to rib fracture - loss of
I
the right side with dullness to percussion elastic recoil j
throughout the right side. Her trachea is D. Opi~te toxicity - suppression of the
1
deviated to the right and the apex beat is not respiratory centre j
palpable. Breath sounds are reduced on the E. Undetected fracture of C3 - diaphragmatic
right. What is the most likely diagnosis? failure
A. Collapse of the right lung
B. Pericardia! effusion 17.3. A 55 year old man has smoked 30
C. Right-sided pleural effusion cigarettes per day since he was 15 years old.
D. Right-sided pneumonia He is a taxi driver. He finds he is increasingly
E. Right-sided pneumothorax breathless on exertion. Oxygen saturations are
98% on room air. Examination reveals tracheal
17.2. An 83 year old woman was passenger in tug, reduced cricostemal distance and a barrel
a car that collided with a lamppost in the city chest. He has reduced cardiac dullness
centre. She was initially complaining of pain in and symmetrically reduced air entry. CXR
her right hip and ribs but has become reveals hyperinflation and spirometry reveals
increasingly drowsy since the paramedics moderate airways obstruction. The patient
administered 2 mg of morphine. She is brought walks 300 m on an incremental walk test
to the emergency department by ambulance. before becoming breathless; oxygen saturations
Urgent X-rays reveal a pelvic fracture, and a are maintained.
single right -sided rib fracture. · What pathologic~;~hange best explains why
Having, initially been drowsy but responsive he is breathless on exertion?
she is now unresponsive. Oxygen saturations A. Activation of central chemoreceptors
are 87% on 2 Umin oxygen via nasal cannulae. B. Exercise-induced bronchospasm
She is apyrexial. BP is 110/66 mmHg, pulse is C. Loss of elastic recoil
65 beats/min. There are no new findings on D. Paradoxical diaphragm movement
examination. An urgent CT brain reveals only E. Pulmonary hypertension

downloaded from www.medicalbr.com


RESPIRATORY MEDICINE • 155

17.4. A 52 year old man presents with


4 days of haemoptysis. Over the last
2 months he has lost weight without
experiencing night sweats. He has
smoked 30 cigarettes per day for 40 years
and worked in construction. He thinks it is likely
that he encountered asbestos at work but
cannot recall any specific exposure. He
returned home from holiday in Guyana 1 week
ago. What is most likely to cause the CXR
findings shown below.

A. Bronchoscopy
B. CT chest, abdomen, pelvis
C. D-dimer
D. Echocardiogram
E. Positron emission tomography
(PET) scan

17.6. A 52 year old man presents with a 1-week


history of fever, left -sided pleuritic chest pain
and increasing shortness of breath. He has a

-
past medical history of alcohol dependency,
alcoholic liver disease and chronic pancreatitis.
He has smoked 10 cigarettes per day for 30
A. Bronchial carcinoma years and still drinks 28 units a week. He
B. Granulomatous polyangiitis worked as a casual labourer on building sites
C. Lung abscess where asbestos was removed when he was in
D. Pulmonary and pleural tuberculosis (fB) his 20s.
E. Pulmonary infarct What is most likely to be causing the CXR
shown below?
17.5. A 26 year old woman presents with 24
hours of central chest pain and progressive
shortness of breath over a month. Examination
reveals pulse 105 beats/min, BP
105/65 mmHg, oxygen saturations 96% on
room air, apyrexial, respiratory rate 16 breaths/
min. Chest clear; heart sounds dual, no
murmurs. Electrocardiogram, (ECG): sinus
tachycardia; no other abnormalities.
Blood tests: haemoglobin 11 0 g/L, white cell
count 0NCC) 9 x 109/L, platelets 340 x 109/L,
urea and electrolytes normal range, C-reactive
protein (CRP) 67 mg/L.
Given the CXR below, what should the next
test be?

A. Bronchial carcinoma·
B. Empyema
C. Hepatic hydrothorax
D. Mesothelioma
E. Pleural effusion secondary to pancreatitis

downloaded from www.medicalbr.com


156 • RESPIRATORY MEDICINE

17.7. A 63 year old woman had a CT pulmonary


angiogram (CTPA) when she presented with
left-sided pleuritic chest pain to the medical
What should be the next step?
A. Antituberculous chemotherapy
B. Further follow-up scan in 3 months
T
I
I
!
assessment unit. There was no pulmonary C. Palliative chemotherapy
thromboembolism but the appearance below D. Palliative radiotherapy
was noted on CT scan. E. Right upper lobectomy

17.9. A 51 year old woman presents to the


emergency department with sudden-onset
breathlessness and right -sided pleuritic chest
pain. She is a non-smoker with no significant
occupational exposures and no significant past
medical history. She requires 40% oxygen to
maintain her saturations at 92%, apyrexial, BP
101/62 mmHg, pulse 112 beats/min,
respiration rate 30 breaths/min.
What is shown on the CXR?

The patient recovered spontaneously over 24


hours and had no other symptoms or past
medical history. She has smoked 10 cigarettes
per day for 40 years. What is the best next
step?
A. Bronchoscopy
B. Commence antibiotics
C. CT-guided biopsy
D. Interferon-gamma release assay
E. PET scan
A. Dextrocardia
17.8. A 63 year old woman had a pulmonary B. Left-sided tension pneumothorax
nodule identified incidentally during a CTPA C. Right-sided lung collapse
examination. She has a 20 pack-year smoking D. Right-sided pleural effusion
history but no past medical history and is E. Right-sided pneumothorax
asymptomatic with performance status 0. She
has had pulmonary function testing, including 17.10. A 53 year old woman has had asthma
lung volumes and gas transfer, with no for 30 years. It had been well controlled until
abnormalities identified. She went on to have the last year. Despite escalation of her inhaled
the test below and the only abnormality therapies and intermittent oral corticosteroids
identified is shown. she has an ongoing cough productive of
copious volumes of green sputum,
breathlessness and wheeze. After review in the
respiratory clinic she is sent for a CT scan.
What is shown on the image here?

downloaded from www.medicalbr.com


RESPIRATORY MEDICINE • 157

17.12. For the patient described in Question


17.11 , what would be the best next step in his
management?
A. Bedside thoracic ultrasound and pleural
aspiration
B. Diuretic therapy
C. Repeat echocardiogram
D. Stop apixaban
E. Thoracic ultrasound scan-guided intercostal
chest drain

17.13. A 35 year old man has been unwell for a


week. He has experienced fever, productive
cough, rhinitis and aching muscles. He now
A. Bronchiectasis presents with left -sided pleuritic chest pain to
B. Chronic bronchitis and emphysema the emergency department. Observations
C. Cystic fibrosis include temperature 39"C, oxygen saturations
D. Honeycombing and interstitial fibrosis 94% on room air, BP 134/76 mmHg, pulse
E. Typical changes of asthma 101 beats/min, respiratory rate 24 breaths/min.
His CXR is below.

11.11. A 71 year old man presents with


increasing shortness of breath with no
cough or systemic symptoms. He has
atrial fibrillation for which he takes apixaban.
A recent echocardiogram suggests
preserved left ventricular systolic function and
no valvular abnormalities. He has no significant
respiratory exposures and has always worked
in an office. He has only ever holidayed in
North America.
What does his CXR show?

What is shown on the CXR?


A. Bilateral hilar lymphadenopathy
B. Collapse/consolidation of the lingula
C. Diffuse right -sided infiltrates
D. Left hilar consolidation
E. Left -sided pneumothorax

A. Right middle and lower lobe collapse


B. Right-sided bronchial carcinoma
C. Right-sided consolidation
D. Right -sided mesothelioma
E. Right-sided pleural effusion

downloaded from www.medicalbr.com


158 • RESPIRATORY MEDICINE

17.14. A 29 year old man presents with


progressive shortness of breath over a week
and a new central chest discomfort. He has a
1
I
productive cough. He works in telemarketing
and smokes cannabis recreationally. He has no II
other past medical history of note.
Observations include temperature 38.5°C,
I
oxygen saturations 93% on room air, BP
I
I
I
112/87 mmHg, pulse 106 beats/min,
!
respiratory rate 24 breaths/min. His CXR is
shown.
I
What would be the next step in his
management? i
A. Admit for observation
B. CT scan
C. Human immunodeficiency virus (HIV) test
D. Intercostal chest drain
E. Therapeutic aspiration

17.15. A 54 year old woman attends the


rapid-access lung cancer clinic with
haemoptysis and an abnormal CXR. CT
scanning confirms a large cavitating right upper
lobe mass and bronchoscopic biopsy confirms
a squamous lung cancer. The PET scan below
was arranged.
What does the CT-PET scan show?
A. Uptake confined to the right upper lobe lesion
B. Uptake in the right upper lobe lesion and a
spinal metastasis
C. Uptake in the right upper lobe lesion and
physiological uptake in large vessels
D. Uptake in the right upper lobe lesion and
right hilar lymph node
E. Uptake in the right upper lobe
lesion, a spinal metastasis and a sternal
metastasis

downloaded from www.medicalbr.com


RESPIRATORY MEDICINE • 159

11.16. A 73 year old woman has struggled Blood tests reveal: haemoglobin 143 g/L,
with increasing shortness of breath on WCC 12 x 109 /L {neutrophilia), platelets
exertion over the last year. In addition, she 435 x I 09/L, urea 9 mmoi/L (54 mg/dL),
has a dry cough. She worked in an office until creatinine I 02 J.l.moi!L (1.15 mg/dL), sodium
she retired. She has a pet dog. She has 128 mmoi/L, bilirubin 12 J.l.rnoi/L (0. 70 mg/dL),
osteoarthritis, osteoporosis and hypothyroidism. alanine transaminase (ALT) 243 U/L, y-glutamyl
She takes regular paracetamol, a transferase (GGT) 354 U/L, alkaline
bisphosphonate and calcium/vitamin D phosphatase 250 U/L, CRP 334 mg/L. His ·
supplementation. Her sister was treated CXR is below.
for TB when they were children and
she had X-ray screening that she thinks
was clear. Examination reveals finger
clubbing and bi-basal crackles. Her CT scan is
shown below.

What abnormality is seen on his CXR?


A. Extensive left -sided consolidation
B. Large left-sided pleural effusion
C. Left -sided bronchial carcinoma
D. Left -sided empyema
E. Left -sided pulmonary infarct
What is the likely cause of the CT
appearance? 17.18. For the same patient described in
A. Bronchiectasis Question 17.17, what is his CURB-65 score?
B. Emphysema A. 0
C. Idiopathic pulmonary fibrosis B. I
D. Lymphangitis carcinomatosa c. 2
E. Scarring related to pulmonary tuberculosis D. 3
E. 4
17.17. A 45 year old man has returned home
from a holiday in Spain with a dry cough, 17.19. For the same patient described in
left-sided pleuritic chest pain and fever. He had Question 17.17, which of the following is the
started some amoxicillin he bought whilst in causative organism likely to be?
Spain. He has been sent to the medical A. Chlamydia pneumoniae
assessment unit after a family physician visit at B. Haemophilus influenzae
home where he was found to, be quite C. Legionel!a pneumophila
muddled. He has a fever of 39.5°C and oxygen D. Mycoplasma pneumoniae
saturations of 85% on ait. Respiratory rate is E. Streptococcus pneumphiae
26 breaths/min, BP I 03(63 rnrnHg, pulse
112 beats/min. Examin*ion reveals left-sided
bronchial breathing with/ increased vocal
resonance. 1

downloaded from www.medicalbr.com


I 60 • RESPIRATORY MEDICINE

17.20. A 38 year old man presents with cough 17.22. A 75 year old man with no past medical
l I
productive of blood-streaked sputum, fever and history presents with increasing shortness of
left-sided pleuritic chest pain. In addition he has breath over 6 months. He previously worked at
developed troublesome cold sores. His past a shipyard where he had significant exposure
medical history includes appendicectomy. He to asbestos. He has a large right-sided pleural
works in a bank. His CXR is below. effusion. Pleural aspiration is performed and
reveals an exudate but cytopathological
examination identifies no malignant cells. CT
scanning reveals circumferential pleural
thickening but no other abnormalities.
Which test is most likely to give a diagnosis?
A. Abrams needle biopsy
B. Bronchoscopy
C. Echocardiogram
D. Repeat pleural aspiration
E. Thoracoscopy

17.23. A 59 year old smoker presents with a


new cough. CXR is abnormal and subsequent
CT scanning identifies a 4-cm tumour
peripherally in the left lower lobe and an
enlarged hilar lymph node on the ipsilateral
side. The tumour is surrounded by
What does his CXR show? emphysematous change and the radiologist
feels CT-guided biopsy would have a high risk
A. Left -sided apical pneumothorax
of pneumothorax. PET scanning reveals high
B. Left -sided basal consolidation
FOG avidity in the presumed tumour and
C. Left -sided collapse
indeterminate uptake in the hilar lymph node
D. Left -sided pleural effusion
but no other significant uptake. The patifnt has
E. Left -sided reticulonodular opacification
a forced expiratory volume in I second ;(FEV1)
17.21. An 82 year old man presents with of 1.5 L (predicted 3.2) and a forced vral
increasing shortness of breath on exertion over capacity (FVC) of 3.0 L (predicted 3.4).
the last 2 years. He has finger clubbing and What should be the next investigation?
bi-basal inspiratory crackles within the chest. A. CT -guided biopsy
His CT chest reveals bilateral, diffuse fibrotic B. Endobronchial ultrasound-guided fineneedle
change and basal honeycombing. The aspiration (EBUS-FNA)
radiologist feels the appearance is in keeping C. Flexible bronchoscopy
with usual interstitial pneumonia. The patient is D. Repeat CT scanning
keen that everything be done to ensure the E. Thoracoscopy
diagnosis is accurate. Which one of the
following statements is true? 17.24. A 37 year old asthmatic woman presents
A. A typical CT appearance is sufficient for with difficult asthma control despite maximal
diagnosis inhaled therapy and montelukast. Her asthma
B. Blind transbronchial needle aspiration would control consistently deteriorates in April and
likely give the diagnosis because of diffuse May. During this time the patient experiences
nodal involvement streaming eyes'and nasal secretions. The
C. Bronchoscopy with cytopathological analysis patient is an enthusiastic runner but finds it
of washings would help characterise the difficult to compleJ~ a run during these months.
nature of his pulmonary fibrosis The patient has no exposure to animals or
D. Endobronchial ultrasound of his subcarinal birds and wqrks in an office. Serum allergy
node would exclude more diffuse disease testing reveals ths following results:
E. Transbronchial lung biopsy would give a TotallgE 1241 kU/L
tissue diagnosis because idiopathic Specific lgE to dog 24 kU/L
pulmonary fibrosis is diffuse Specific lgE to cat 3.4 kU/L

downloaded from www.medicalbr.com


RESPIRATORY MEDICINE • 161

Specific lgE to tree pollen 0.2 kU/L FEV1 3.2 L


Specific lgE to oilseed rape 34 kU/L FVC 4.4 L
Specific lgE to grass pollen 25 kU/L FEV/FVC 0. 73
What is the likely explanation for the patient's What does the respiratory function testing
presentation? reveal?
A. The patient has exercise-induced asthma A. A mixture of obstruction and restriction
(lgE testing irrelevant) B. A restrictive defect
B. The patient has lied about exposure to C. A reversible obstructive defect
animals (specific lgE to cats and dogs D. An irreversible obstructive defect
elevated) E. Inhaled salbutamol has confounded testing
c. The patient is non-adherent with medication
(total lgE remains high) 17.27. A 62 year old woman attends the family
D. The patient is sensitised to montelukast (lgE practice surgery with increasing shortness of
results represent cross-reactivity) breath on exertion and is sent for full pulmonary
E. The patient's asthma is triggered by function testing. She is an ex-smoker of 25
seasonal pollens (specific lgE to oilseed rape pack years and works caring for horses at a
and grass positive) riding stable. Respiratory function testing
reveals:
17.25. A 49 year old patient returns from a FEV1 3.2 L
holiday in Portugal with fever, a dry cough, FVC 3.8 ~
increasing shortness of breath. His wife has FEV/FVC 0.84
brought him to hospital because he had an Tot<:llung capacity (TLC) 4.1 L
episode where he became very muddled and Residual volume (RV) 0.6 L
seemed not to recognise his family members. Transfer factor for carbon monoxide (Tlc0 )
In Spain he stayed in a hotel and on his return 1.5 (44%)
he was exposed to a nephew who has since Transfer coefficient (Kcol 1 (56%)
been diagnosed with influenza and has been How should the respiratory function testing
admitted to the local paediatric hospital with be interpreted?
breathing difficulties. A. Gas transfer reduction indicates
Observations on arrival include: temperature emphysema
39.3°C, BP 110/55 mmHg, pulse 102 beats/ B. Gas transfer reduction indicates
min, oxygen saturations 89% on room air. hypersensitivity pneumonitis
Blood tests reveal an inflammatory response C. No evidence of airways obstruction
and a CXR shows bilateral infiltrates. What is D. There is a restrictive picture
the most useful diagnostic test likely to be? E. There is an obstructive picture
A. Differential cell count in bronchial lavage
B. High-resolution CT chest (HRCT) 17.28. A 53 year old man presents with a
C. Nucleic acid amplification testing for 3-month history of non-productive cough.
respiratory pathogens He has no other symptoms but is finding the
D. Paired serology for mycoplasma and cough tiresome and frustrating. He is a
legion ella non-smoker and works in an office. He is on
E. Sputum cytology omeprazole 20 mg once a day, which has
controlled his acid reflux for the last 2 years.
17.26. A 45 year old smoker is sent for He has trialled a nasal spray and cough linctus
pulmonary function testing because of without improvement. Spirometry is normal.
breathlessness, wheeze and productive cough. Which of the following statements is true?
There are no other significant respiratory A. A CXR should be performed
exposures. The family physicial') has.trialled a B. A trial of an inhaled cortitosteroid (ICS) is
salbutamol inhaler, which has been somewhat warranted /
helpful. Spirometry reveals: C. He should see an ear, nose and throat (ENT)
FEV1 2.6 L surgeon •
FVC 4.4 L D. His cough has become a habit and he must
FEV1/FVC 0.59 get used to it
Following nebulisation of 2.5 mg salbutamol: E. His reflux medication should be increased

downloaded from www.medicalbr.com


162 • RESPIRATORY MEDICINE

17.29. A 75 year old woman has been referred She has never been an active person but now
with a daily, chronic non-productive cough struggles with breathlessness on exertion,
that has been present for at least I 0 years. especially walking uphill or when carrying
She has no nocturnal and no nasal symptoms. shopping bags. She stopped smoking
Her only other symptom is of back pain 15 years ago when her husband had a heart
following a further vertebral fracture in the last attack. She had smoked 20 cigarettes per day
month. before that.
Her past medical history includes: Her spirometry is within normal limits but her
osteoporosis (multiple vertebral fractures and CXR suggests a hilar abnormality. A
kyphosis), previous duodenal ulcer, TB subsequent CT scan demonstrates that this
meningitis as a child. She is a life-long was a projectional anomaly and excludes a
non-smoke~. Her medication includes: sinister cause. The image below is from the CT
orneprazole 20 rng once daily, alendronic acid scan performed 2 months previously.
once a week, calcium and vitamin D, and cod
liver oil capsules. CXR reveals a large hiatus
hernia and significant kyphosis.
What is the most likely cause of her cough?
A. Asthma
B. Gastro-oesophageal reflux
C. Hypercalcaemia
D. Lung cancer
E. Tuberculosis

17.30. A 42 year old woman presents with a


4-month history of breathlessness that occurs
at rest and on exertion, exclusively in the
daytime. She has no other symptoms and is a
non-smoker. The breathlessness started
following an episode of upper respiratory tract What is the most likely cause of this
infection where she had significant nasal woman's breathlessness?
blockage. She has no nasal symptoms now but
A. Asthma
often has the feeling that she cannot get a
B. Cardiac failure
satisfying breath, like she cannot extract
C. COPD
enough oxygen from the air. Sometimes this
D. Interstitial lung disease
becomes very frightening and on one occasion
E. Pulmonary thromboembolism
she was admitted to the emergency
department after nearly passing out. Her
17.32. A 64 year old woman with COPD
D-dimer was positive but a CTPA was normal
presents with right-sided pleuritic chest pain
and she was discharged home with a
that carne on suddenly and left her feeling very
salbutarnol inhaler, which she sometimes finds
breathless. She had felt very well until that
helpful. Her father was a smoker and died of
point, having been away on holiday for 3 weeks
lung cancer in his 60s.
in North America. She was well on the flight,
Examination is unremarkable, spirometry is
which lasted 5 hours. Her last exacerbation of
normal, oxygen saturations are 98% on room
COPD was 4 months ago, when she
air. What should be the next investigation?
was admitted to hospital and received
A. Arterial blood gas oxygen, nebulised bronchodilators and
B. Bronchoscopy prednisolone.
C. Dysfunctional breathing studies and a Observations: oxygen saturations 84% on
Nijmegen questionnaire room air, BP 151/81 mmHg, pulse 110 beats/
D. Echocardiogram min; apyrexial. Examination reveals globally
E. Lung volume and gas transfer studies decreased air entry and mild bipedal
oedema.
17.31. A 67 year old woman has been Investigations: CXR hyperinflation, ~il focal.
progressively breathless over the last year. CRP 15 mg/L, D-dirner 865 ng/mL, WCC

downloaded from www.medicalbr.com


--------~--~---~--~~~~~~~~~-

RESPIRATORY MEDICINE • 163

11 x 109/L; ECG: sinus tachycardia with a respiratory department because of recurrent


rightward axis. exacerbations of bronchiectasis. The cause of
What is the most likely cause of her the bronchiectasis was tuberculosis treated 20
chest pain? years ago. The patient has had a large apical
A. Bronchospasm cavity on the right side of the CXR for at least
B. Malignant pleural disease 10 years.
c. Pneumonia CXR today appears to show that this cavity
D. Pneumothorax has partially filled in. What is the likely cause of
E. Pulmonary thromboembolism his haemoptysis?
A. Carcinoma
B. Mycetoma
17.33. An 84 year old man has had a nagging
C. Pulmonary infarction
ache in his left chest that has slowly built up
D. Suppurative pneumonia
over 3 months. The pain is worst in his axilla
E. Tuberculosis
but radiates posteriorly towards the spine. The
patient smoked a pipe for 50 years and worked
initially in the merchant navy before taking up a 17.36. A 32 year old cystic fibrosis patient
job in a brewery. presents with massive haemoptysis in the form
Examination reveals: dullness to percussion of 250 ml of fresh red blood. The patient is
from the left lung base to the mid-zone and actively coughing more blood but is
erythema ab igne of the posterior chest wall. haernodynamically stable (BP 145/72 mmHg,
Which of the following is the most likely cause pulse 98 beats/min). The most useful
of this chest pain? inves~igation is likely to be:

A. Chronic thromboembolic disease A. Bronchial artery angiography


B. Herpes zoster B. Bronchoscopy
C. Mesothelioma C. Coagulation studies
D. Pneumonia D.CTPA
E. Tietze's syndrome E. Sputum culture

17.34. A 45 year old man attends his family 17.37. A 67 year old man has a CT
physician with a sprained wrist following a colonogram as a screening test for /
mistimed punch at his karate class. The doctor iron-deficiency anaemia. No colonic abnormalj{y
notices that he has clubbed fingers. The patient is identified but a 6-mm nodule is identified in
has no past medical history of note, is a the right lower lobe of the lung. The radiologist
non-smoker and, apart from his painful wrist, is suggests referral to the respiratory team for/
asymptomatic. He says people have always ongoing follow-up.
commented on his fingers and that his father's With regard to pulmonary nodules, the risk of
fingers are similar in appearance. On checking malignancy increases with which of the
the patient's record the doctor notes finger following?
clubbing was first recorded in his teenage A. A smooth margin
years. B. Central deposition of calcification
What is the next step the family physician C. Lack of smoking history
should take? D. Size< 4 mm
A. Check bloods (including ~FTs, thyroid E. Upper lobe distribution
function tests and erythrocyte sedimentation
rate) 17.38. A 65 year old woman with rheumatoid
B. CXR to exclude cancer arthritis has a CT scan to determine whether
C. Reassurance I she has an associated int(:lrstitial lung disease.
D. Referral to respiratory clinic She has mild basal interlobular septal
E. Sweat test to exclude c~stic fibrosis thickening in keeping with early interstitial lung
disease (ILD}. The radiologist also identifies a
17.35. A patient presents acutely having speculated, 1 .5 cm-diarn"eter right upper lobe
coughed up 50 ml of fresh red blood suddenly nodule that he suggests may require further
that morning. He is well known to the investigation.

downloaded from www.medicalbr.com


l
164 • RESPIRATORY MEDICINE

Which of the following statements is Pleural fluid aspiration is performed under


true with regard to PET scanning for pulmonary ultrasound guidance. Which one of the
nodules? following values might be expected?
A. It always detects bronchoalveolar A. Pleural fluid glucose 5.6 mmoi/L (101 mg/dL)
carcinoma B. Pleural fluid LDH 150 U/L
B. Inflammatory nodules are reliably C. Pleural fluid pH 6.9 (H+ 126 nmoi/L)
excluded D. Pleural fluid protein 27 g/L
C. It has replaced tissue biopsy E. Pleural fluid triglycerides 5.4 mmcii/L
D. It is not useful in nodules < 1 em in diameter (478 mg/dL)
E. It offers no more information than a
standard CT 17.42. In which of the following conditions might
acute type I respiratory failure be expected?
17.39. Which of the following is a cause of a
A. Flail chest injury
transudative pleural effusion?
B. Lobar collapse
A. Drug-induced pleural effusion C. Lymphangitis carcinomatosa
B. Hypothyroidism D. Obstructive sleep apnoea (OSA)
C. Pulmonary infarction E. Opioid toxicity
D. Rheumatoid arthritis
E. Tuberculosis 17.43. A 27 year old woman with severe atopic
asthma is 26 weeks pregnant. She has been
17.40. Which one of the following statements is awake at night coughing and short of breath
true? Light's criteria includes: for 48 hours. She has been trying to avoid
A. Pleural fluid: serum protein using her salbutamol inhaler but nonetheless
ratio> 0.6 has had to take it 6 to 8 times in the last 24
B. Pleural fluid lactate dehydrogenase (LDH) > hours. She has no nasal symptoms and no
two-thirds of the upper limit of normal serum reflux.
LDH Observations: oxygen saturations 94% on
C. Pleural fluid LDH <two-thirds of the upper room air, respiratory rate 22 breaths/min 1 BP
limit of normal serum LDH 110/65 mmHg, pulse 96 beats/min, ap~rexial;
D. Pleural fluid LDH: serum LDH peak expiratory flow rate (PEFR) 240 ymin
ratio> 0.5 (best 450 Umin). /
E. Pleural fluid: serum protein ratio < 0.5 Examination of the chest reveals bilateral
wheeze, mild bilateral pedal oedema, heart
17.41. A 64 year old man presents sounds dual, no murmur.
with fever, sweats, right-sided pleuritic The patient's current medications include:
chest pain and breathlessness. He is an regular high-dose ICS/Iong-acting ~ 2 -agonist
ex-smoker and drinks 2 L of cider a day. (LABA) and oral montelukast; inhaled
The CXR (A) and thoracic ultrasound (B) are short-acting ~ 2 -agonist (SABA) as required. She
shown below. insists she has been adherent with medication,

downloaded from www.medicalbr.com


RESPIRATORY MEDICINE • 165

although she is very worried about taking any 17.46. A 57 year old woman has been coughing
medications whilst pregnant. for 3 years. She always carries tissues with her
What should be the next step in her to collect the phlegm she coughs up
management? throughout the day. Sometimes her phlegm
A. Low-molecular-weight heparin (LMWH) can be green and she feels run-down and
unwell. Antibiotics seem to help but she only
B. Oral amoxicillin
feels better for 2-3 weeks.
C. Oral prednisolone
D. Reduce high-dose leS/LABA The patient is a non-smoker who works ·in an
office. She finds her cough embarrassing and
E. Stop montelukast
work colleagues have been giving her a hard
time. What would be the best investigation for
17.44. A 48 year old asthmatic is referred to
this patient?
clinic because of increased frequency of
asthma exacerbations. He has been waking at A. a,-Antitrypsin levels
night with cough and breathlessness that B. Bronchoscopy
require extra doses of inhaled salbutamol. C.eXR
Spirometry reveals an obstructive defect and D. High-resolution eT chest
blood tests reveal an eosinophilia of E. Immunoglobulin levels
0.67 x 109/L. eXR is clear. The patient's
current therapy includes Flixotide 500 11g/ 17.47. A patient who attends the asthma clinic
salmeterol 25 11g 1 puff twice a day; salbutamol has been experiencing significant deterioration
2 puffs, as required. The patient has started in control. The main problem is a cough
prednisolone 40 mg for 5 days as prescribed productive of green sputum that is difficult to
by his family physician today. expectorate and a right -sided pleuritic chest
In line with British Thoracic Society pain that has developed in the last 48 hours.
guidelines, what should be suggested in order Since then breathing has been more difficult.
to step-up therapy? Blood tests reveal: haemoglobin 136 g/L, wee
A.
B.
C.
Add amoxicillin
Add montelukast
Double prednisolone dose
14 x 109/L (neutrophils 7 x 109/L, lymphocytes
1.46 x 109/L, monocytes 0.8 x 109/L, eosinophils
4.7 x 109/L), platelets 340 x 109/L, eRP
~I
D. Provide home nebuliser 120 mg/L. eXR is shown below. I
E. Start omalizumab

17.45. A 65 year old man with known eOPD


and mild airways obstruction on spirometry
presents with increasing shortness of breath on
exertion. He mainly struggles walking on
inclines or climbing stairs. He does not
experience exacerbations of eOPD and he has
had no courses of oral prednisolone or
antibiotics in the last 2 years.
His current inhaled therapy includes a
long-acting muscarinic antagonist (LAMA) and
an SABA. Examination reveals decreased air
entry bilaterally, with loss of cardiac dullness on
percussion. The patient dOes not desaturate on
an incremental walk test.
What would be a reasowl.ble addition for this
patient to escalate therapy?: What is the likely cause. of the deterioration
A. Ambulatory oxygen I in control?
B. Inhaled corticosteroid (ldS)/LABA A. Bacterial infection
combination inhaler B. Fungal infection
C. LAMNLABA combination inhaler C. Mycobacterial infection
D. Oral prednisolone I D. Parasitic infection
E. Salbutamol nebuliser E. Viral infection

downloaded from www.medicalbr.com


166 • RESPIRATORY MEDICINE

17.48. A 24 year old man with cystic 17.51. A 32 year old man presents with a 5-day
fibrosis has recently moved into the history of left -sided pleuritic chest pain, fever
area. He keeps relatively well and and cough productive of rusty sputum.
missed some appointments at his previous Observations include: BP 100/60 mmHg, pulse
service. He has had two exacerbations of 105 beats/min, temperature 38.2°C, respiratory
bronchiectasis in the last year and is rate 21 breaths/min, oxygen saturations 87%
disappointed with his most recent lung on room air. Examination reveals dullness to
function measures. He has heard about a new percussion and bronchial breathing on the left.
medicine called ivacaftor that is only beneficial Nasolabial cold sores are noted.
to some patients with cystic fibrosis and Which organism is likely to be responsible for
wonders if he qualifies. this presentation?
lvacaftor is a small-molecule drug that A. Aspergillus fumigatus
corrects the function of which of the following B. Herpes simplex virus (HSV)
cystic fibrosis transmembrane regulator (CFTR) C. Mycobacterium tuberculosis
gene defects? D. Pneumocystis jirovecii
A.l1F508 E. Streptococcus pneumoniae
B. G542X
C. G551D 17.52. A 53 year old businessman presents with
D.R117H fever, chills, cough and shortness of breath. He
E. W1282X has recently returned from a trip to the Middle
East where he visited a number of countries
17.49. A 24 year old cystic fibrosis patient has .and spent time in the city as well as visiting
failed to recover from a recent exacerbation of more rural areas.
her bronchiectasis. Previously Haemophilus Examination reveals temperature of 40°C,
influenzae and Staphylococcus aureus have pulse 115 beats/min, BP 100/50 mmHg,
been isolated from her sputum. A 2-week oxygen saturations 80% on room air. CXR
course of co-amoxiclav followed by 2 weeks of shows diffuse infiltrates.
doxycycline have failed to improve spirometry Which of the following statements is most
or reduce sputum load. CXR is unchanged and accurate? -
blood tests are unrevealing, apart from CRP of A. Burkholderia pseudomallei needs to bf
134 mg/L. covered /
The microbiology team have isolated B. He should be isolated and tested for
Pseudomonas aeruginosa in the most recent carbapenemase-producing
sample provided. Which one of the following Enterobacteriaceae (CPE) 1
statements is true of P. aeruginosa in cystic C. He should be isolated and tested for Middle
fibrosis? East respiratory syndrome (MERS)
A. Intravenous antibiotic therapy is rarely D. He should be isolated and tested for severe
required acute respiratory syndrome (SARS)
B. It is a benign coloniser of the bronchiectatic E. Local antibiotic protocol for
airways community-acquired pneumonia (CAP)
C. It is one of the earliest bacteria isolated in should be followed ·
sputum from CF patients
D. Nebulised azithromycin 3 times a week 17.53. A 73 year old man has been in hospital
suppresses infection for 3 days having undergone elective hip
E. Nebulised tobramycin is an effective surgery. He is acutely c;onfused in the middle of
treatment in chronic colonisation the night with a temperature of 38.3°C.
Urinalysis is negative but blood testing reveals
17.50. Acute coryza is most commonly caused raised inflammatoryr-harkers. A CXR clearly
by which of the following? shows a new right-sided infiltrate.
A. Bordetel/a pertussis Which of the following approaches is
B. Haemophilus inf/uenzae appropriate? '
C. Mycoplasma pneumoniae A. A CTPA should be ordered
D. Rhinovirus B. Blood cultures should be taken and a
E. Streptococcus pneumoniae watch-and-wait policy favoured /

downloaded from www.medicalbr.com


RESPIRATORY MEDICINE • 167

c. Local antibiotic guidelines for CAP should be rifampicin. He is receiving the standard
followed treatment regimen. Two weeks into therapy, he
D. Local antibiotic policy for hospital-acquired phones the specialist nursing team as he has
pneumonia (HAP) should be followed painful eyes and is worried that the therapy is
E. Local antibiotic policy for not working.
ventilator-associated pneumonia (VAP) What is the likely cause of this presentation?
should be followed A. Drug resistance to ethambutol and
pyrazinamide
17.54. A 72 year old man initially improves B. Ethambutol
following treatment for an exacerbation of C. Immune reconstitution
COPD. He has been in hospital for I 0 days D. Intercurrent viral infection
when he spikes a temperature of 39°C, his E. Non-tuberculous Mycobacterium
oxygen saturations drop and he starts to
expectorate green sputum with blood-streaking. 17.58. A 54 year old man is due to start a
A CXR reveals dense left-sided consolidation. monoclonal antibody-based therapy for active
Late-onset HAP is often attributable to which Crohn's disease but the radiologist has noted a
of the following microorganisms? minor abnormality on the patient's recent CXR.
A. Acinetobacter The patient had a bacille Calmette-Guerin
B. Chlamydia (BCG) vaccine in childhood and has no known
C. Haemophilus TB contacts. He has no respiratory symptoms.
D. Legionella Local guidance suggests checking an
E. Streptococcus interferon-gamma release assay (IGRA) on a
peripheral blood sample.
17.55. The mortality from HAP is approximately Which one of the following statements is
which of the following? true with regard to the IGRA?
A.IO% A. A positive result should prompt the clinician
B. 20%
c. 30%
to start antituberculous chemotherapy
B. It is more specific than tuberculin skin testing
~
D.40% C. It is now the first-line test for diagnosis of / 1 I
E. 50% active TB
D. It is only positive where there is systemic I
17.56. A 34 year old woman has been unwell mycobacterial infection I
with high fever, pleuritic chest pain and cough E. It measures the release of interferon-alpha
productive of foul sputum. She is an from sensitised T cells
intravenous drug-user and has noted that her
usual injection site in the groin has developed a 17.59. A 64 year old woman presents with back
fluctuant swelling. pain, weight loss and a palpable mass in her
CXR shows multiple nodules and a CT loin that extends into the buttock. She is
shows a predominantly basal distribution and reviewed by the orthopaedic team and imaging
notes that some of the nodules are cavitating. suggests the mass is of fluid consistency.
What is the likely explanation for these findings? They aspirate pus easily and send it to the
A. Aspiration pneumonia laboratory for culture and cytopathological
B. Infective endocarditis examination. They ask for advice about further
C. Metastatic cancer testing.
D. Pulmonary thromboembolism Which of the following would be an important
E. Tuberculosis additional test?
A. Bronchoscopy
17.57. A 28 year old student of Chinese origin B. Echocardiogram to exclude septic embolus
has begun treatment for pulmonary tuberculosis C. Flow cytometry
that presented with a typical clinical picture and D. Fluid biochemistry
CXR. Sputum was positive for acid- and E. Mycobacterial testing.
alcohol-fast bacilli, and polymerase chain
reaction (PCR) for Mycobacterium tuberculosis 17.60. A 34 year old haematology patient has
has detected no resistance to isoniazid or been receiving cytotoxic chemotherapy for

downloaded from www.medicalbr.com


168 • RESPIRATORY MEDICINE

acute myeloid leukaemia. He is isolated with


neutropenic sepsis thought to be secondary to
fluid aspiration reveals an adenocarcinoma.
What is the correct staging of this lung cancer?
1
pneumonia. He has fever, pleuritic chest pain A. I
and haemoptysis. CT scanning has revealed B. II
multiple nodules with what is described as G. Ill
surrounding inflammatory change. The D. IV
radiological differential includes infection, E. Staging not yet complete
inflammation and malignancy. The patient has
made no response to standard antibiotics for 17.64. A 36. year old non-smoker presents with
neutropenic sepsis. isolated haemoptysis. She has no systemic
What should be the next step in symptoms but has been expectorating small
management? amounts of fresh blood most mornings in the
A. Antituberculous chemotherapy last 2 weeks. She denies epistaxis and is quite
B. Bronchoscopy clear she has been coughing blood rather than
G. CTPA vomiting.
D. Respiratory virus throat swab A CT scan identifies an area of collapse in
E. Voricdnazole the left upper lobe (LUL) and a possible small
tumour in the main bronchus. Bronchoscopy
17.61. A 63 year old female non-smoker reveals a vascular-looking tumour emerging
presents to her family physician with a new from the LUL orifice.
cough. Her CXR is abnormal with a 5-cm lesion What is the most likely tumour type for this
in the right mid-zone. She has since undergone 1\)sion?
bronchoscopy at which an endobronchial A. Adenocarcinoma
tumour was biopsied. Which is the most B. Breast
common histological type of lung cancer? G. Carcinoid
A. Adenocarcinoma D. Small cell
B. Large cell carcinoma E. Squamous
G. Mesothelioma
D. Small cell carcinoma 17.65. A 56 year old, non-smoking i
E. Squamous carcinoma man presents with a 3-month history of arl
irritating non-productive cough. CXR is J
17.62. A 73 year old woman with metastatic abnormal and subsequent CT scanning
non-small cell lung cancer is considering reveals metastatic disease in the bones
palliative chemotherapy. Although she has lost and a 4-cm left-sided peripheral mass.,
weight and been more tired she has not Bronchoscopy achieves biopsy and ·
experienced any other symptoms. Her cancer cytopathological examination identifies
was discovered when she had a CT scan adenocarcinoma, which proves to be EGFR
because of a combination of weight loss, (epidermal growth factor receptor) positive on
smoking habit and finger clubbing. molecular analysis.
She has developed increasingly painful What would be the treatment of choice for
wrists. What is the most likely cause for this this man?
problem? A. Adjuvant chemotherapy and lobectomy
A. Bone metastases B. Best supportive care
B. Finger clubbing G. Cisplatin and pemetrexed
G. Horner's syndrome D. Erlotinib
D. Hypercalcaemia E. Radiotherapy
E. Hypertrophic pulmonary osteoarthropathy ;

(HPOA) 17.66. A 64 year old/Vi/oman with breast cancer


metastatic to liver, lungs and bone has been
17.63. A 56 year old smoker presents with receiving second-line chemotherapy but has
increasing shortness of breath. A CXR shows a become increasingly. breathless over 3 months.
left -sided pleural effusion and CT scanning has She received antibiotics for a presumed
demonstrated a 3-cm tumour in the left lung pneumonia after CXR identified bilateraJ
but no evidence of lymphadenopathy. Pleural infiltrates but made no improvement. "

downloaded from www.medicalbr.com


RESPIRATORY MEDICINE • 169

subsequent CT scanning has identified diffuse physician arranges a CXR, below, and refers to
interstitial thickening radiating from the hilar the respiratory clinic.
regions.
What is the likely diagnosis?
A. Drug-induced pneumonitis
B. Lymphangitis carcinomatosa
c. Pneumocystis pneumonia
D. Pulmonary oedema
E. Venous thromboembolism

17.67. A 28 year old woman presents to the


acute receiving unit with a vague chest
discomfort that seems to move through into
her back. CXR appears to show mediastinal
widening. A CT angiogram is requested to rule
out aortic dissection but it identifies a large
mass in the anterior mediastinum and some
cervical, supraclavicular and axillary At review 4 weeks later she feels back to her
lymphadenopathy. usual self apart from a persistent mild lethargy.
What is the most likely diagnosis? What is the most likely cause for this
presentation?
A. Bronchogenic cyst
B. Lymphoma A. HIV
C. Neurogenic tumour B. Lymphoma
D. Oesophageal tumour C. Pneumonia
E. Retrostemal goitre D. Sarcoidosis
E. Tuberculosis
17.68. A 79 year old woman has been
increasingly short of breath over 6 months. 17.71. A 55 year old man presents with
She has no significant past medical history increasing breathlessness over 2 years. He has
and is on no medication. She has no a dry cough. Examination of the chest is
significant respiratory exposures. Examination unremarkable. Pulmonary function testing
reveals bilateral, fine basal crackles and finger reveals a restrictive defect. HRCT identifies mild
clubbing. An HRCT demonstrates bilateral bilateral hilar lymph node enlargement, nodules
basal, peripheral reticulation with honeycomb in a bronchovascular distribution and extensive
cysts. fibrotic change.
What is the most likely diagnosis? What is the most likely diagnosis?

A. Acute interstitial pneumonia A. Sarcoidosis stage 0


B. Idiopathic pulmonary fibrosis B. Sarcoidosis stage I
C. Lymphocytic interstitial pneumonia C. Sarcoidosis stage II
D. Non-specific interstitial pneumonia D. Sarcoidosis stage Ill
E. Sarcoidosis E. Sarcoidosis stage IV

17.69. Respiratory bronchiGiitis-interstitial lung 17.12. A 72 year old woman has had
disease (RBILD) is more common in which of rheumatoid arthritis for 20 years. She recently
the following groups? had a chest infection but fully recovered. A
CXR was ordered to exclude pneumonia but
A. Non-smokers
revealed multiple smooth nodules. CT scanning
B. Patients > 65 years of age
also identifies four smoo;th nodules of varying
C. Patients with connective tissue diseases
size. A CXR recovered ·from storage shows that
D. Smokers
these nodules have been present for at least 5
E. Women
years. •
What is the likely cause of these nodules?
11.70. A 28 year old woman develops a painful
rash on her lower limbs, arthralgia and fever. A. Bronchiectasis
She feels run-down and unwell. Her family B. Metastatic cancer

downloaded from www.medicalbr.com


170 • RESPIRATORY MEDICINE

C. Pulmonary embolism
D. Pulmonary fibrosis
CXR bilateral peripheral, especially upper-zone
consolidation.
What should the treatment be for this
T
E. Rheumatoid nodules
patient?
17.73. A 64 year old man presents with high A. Antibiotics to cover hospital-acquired
fever, left -sided pleuritic chest pain and pneumonia
shortness of breath. His CXR is below. B. Antituberculosis chemotherapy
C. Continue current therapy
D. Intravenous furosemide
E. Stop daptomycin and give prednisolone

17.75. A 45 year old non-smoking man presents


with fever, weight loss and breathlessness. On
the night of his admission he has developed
stridor. CXR reveals multiple, bilateral
pulmonary nodules. His past medical history
includes sinusitis and conductive deafness.
Blood tests reveal: urea 7 mmoi/L (42 mg/
dl), creatinine 165 ~moi/L (1.87 mg/dl),
haemoglobin 102 g/L, wee 15 X 109/L
(neutrophils 12 x 109/L), CRP 145 mg/L .
.What is the most likely diagnosis?
A. Acute eosinophilic pneumonia
A chest drain is sited and excellent resolution B. Bronchial carcinoma
is achieved. The pleural fluid characteristics are C. Chronic eosinophilic pneumonia
as follows: pH 6.9 (H+ 126 nmoi/L); protein D. Eosinophilic granulomatosis with polyangiitis
38 g/L; LDH 800 U/L; glucose 0.3 mmoi/L E. Granulomatous polyangiitis
(5.4 mg/dL). Two weeks later he re-presents
with CXR changes on the other side, with 17.76. A 70 year old woman has been J
right-sided pleuritic chest pain and fever. He is coughing, short of breath, intermittently fevyrish
still taking antibiotics. In addition he has and run-down over 6 weeks. Her past me~ical
bilateral, symmetrical arthritis with joint swelling. history includes transient ischaemic attack and
What is the likely diagnosis? recurrent urinary tract infections. Her
medications include aspirin, simvastatin,,
A. Empyema
amlodipine and nitrofurantoin. She is a
B. Lung cancer
non-smoker and has a pet cat.
C. Lymphoma
Examination reveals oxygen saturations of
D. Mesothelioma
91% on room air, BP 121/76 mmHg, pulse
E. Rheumatoid arthritis
89 beats/min, apyrexial, respiratory rate
18 breaths/min. Her CXR shows bi-basal
17.74. A 61 year old woman is receiving
infiltrates.
intravenous daptomycin for staphylococcal
What is the most likely diagnosis?
discitis. She has required intravenous fluids for
a mild acute kidney injury (AKI) secondary to A. Atypical pneumonia
sepsis and dehydration. Her clinical condition B. Cryptogenic organising pneumonia
and inflammatory markers had been improving C. Idiopathic pulmonary fibrosis
with this therapy. After 2 weeks of antibiotics D. Nitrofurantoin lung disease
she spikes a fever to 39°C, her wee rises and E. Non-specific interstitial pneumonia (NSIP)
she develops an oxygen requirement (oxygen /I
saturations 94% on 35% oxygen). 17.77. A 26 year old. non-smoking woman
Blood tests: haemoglobin 102 g/L, WCC presents for the .second time with
16 x 109/L (neutrophils 9 x 109/L, eosinophils pneumothorax. She has no other past medical
5 x 109/L, lymphocytes 1 x 109/L, monocytes history. Good resolution is achieved with chest
0.8 x 109/L), CRP 150 mg/L, urea 8 mmoi/L drainage but a CT scan identifies multiple cysts
(48 mg/dL), creatinine 89 ~moi/L (1.01 mg/dL). throughout the lung parenchyma. The CT chest

downloaded from www.medicalbr.com


RESPIRATORY MEDICINE • 171

catches an abnormality on the upper pole of 17.80. Which of the following statements is true
the right kidney, which is incompletely imaged with regard to progressive massive fibrosis
and the radiologist suggests an MRI for better (PMF)?
characterisation of the lesion. A. Characterised by small radiographic nodules
What diagnosis should be considered and B. Chyloptysis is associated
further investigated? C. Finger clubbing and basal crackles are
A. Alveolar microlithiasis characteristic
B. Alveolar proteinosis D. It has no impact on lung function
c. Lymphangioleiomyomatosis E. It may progress even after exposure ceases
D. Lymphocytic interstitial pneumonia
E. Pulmonary Langerhans cell histiocytosis 17.81. A 41 year old stonemason admits to
(histiocytosis X) shortness of breath on exertion. A screening
CXR and pulmonary function testing are both
17.78. A 43 year old woman presents abnormal so he has been referred to the
with cough, shortness of breath and wheeze. respiratory clinic. Silicosis results from the
She is a smoker, has no past medical history inhalation of which of the following?
and no exposure to birds or animals. She had A. Coal
been off work for 2 weeks but made an B. Cotton
improvement after starting inhaled C. Quartz
beclometasone and a short -acting D. Silicone
bronchodilator as required. On return to work, E. Tin
things seemed to be fine but deteriorated after
about 3 weeks. She works behind the counter 17.82. A 72 year old man presents with
in a local bakery. progressive breathlessness over 6 months;

-
The patient would like to know if more recently he has had a vague ache in the
she has occupational asthma. Which right side of his chest that has kept him awake
test would be most helpful in making the at night.
diagnosis?
A. Histamine challenge test
B. Peak expiratory flow rate diary
His past medical history is significant for two
separate episodes of 'benign asbestos pleurisy'•
in his 50s. He has pleural plaques and receiVfld
I
C. Specifc lgE to flour compensation. He had worked in the J
D. Spirometry with reversibility construction industry and had frequent, heavy
E. Sputum eosinophils exposure to asbestos. He stopped smoking 20
years ago during a bout of pleurisy, having
17.79. A 55 year old geologist has been started age 12 years and smoked an average
coughing, breathless and experiencing of 20 cigarettes per day.
arthralgia since renovation started at her home. CXR reveals a right -sided pleural effusion
The work was started because of damp and and pleural plaques. It is not possible to see
has involved some structural work. Her home the right-sided costophrenic angle.
always seems to be dusty currently. Her CT scanning reveals pleural plaques,
husband is a stonemason and has been right-sided pleural effusion and mild thickening
working on a new piece at home in their of the pleura that extends onto the
garage. At work she has been preparing mediastinum anteriorly.
beryllium samples for a PhD project, which has What is the most likely reason for his
been quite stressful as deadlines for submission presentation?
approach. A. Asbestos pleural plaques
CXR shows bilateral hilar lymphadenopathy B. Asbestosis
with some soft nodularity in the mid-zones. C. Benign asbestos pleurisy
What is the most likely diagnosis? D. Diffuse pleural thickening
A. Berylliosis E. Mesothelioma
B. Dysfunctional breathing
C. Hypersensitivity pneumonitis 17.83. A 69 year old wo~an has progressive
D. Sarcoidosis breathlessness on exertion. Because she
E. Silicosis described a vague feeling of her chest

downloaded from www.medicalbr.com


172 • RESPIRATORY MEDICINE

tightening during one of these episodes, her spontaneous VTE (treated with 6 months of
family physician started aspirin and glyceryl warfarin) and a family history of VTE (mother
trinitrate spray and referred to cardiology. She and uncle).
underwent CT coronary angiogram. This Observations: oxygen saturations 88%
identified no coronary artery disease but diffuse on room air, respiratory rate 22 breaths/min,
ground glass and some centrilobular nodules pulse 110 beats/min, BP 110/65 mmHg. Chest
were picked up incidentally in the lungs. is clear. Right calf is greater in circumference
The patient has no past medical history, than left by 3 em. Heart sounds dual, no
worked as a secretary and was a non-smoker. murmurs.
She has kept a pet parrot at home for the last Investigations: CXR reveals marginally
year. elevated right hemidiaphragm; EGG: sinus
What is the likely diagnosis? tachycardia; CRP 35 mg/L, 0-dimer 200 ng/
A. Aspirin sensitivity ml.
B. Breathing artefact What should the next test be for this
C. Hypersensitivity pneumonitis patient?
D. Idiopathic pulmonary fibrosis A. CT pulmonary angiogram
E. Sarcoidosis B. Echocardiogram
C. Fluoroscopy of the diaphragm
17.84. A 72 year old man presents with cough D. Respiratory virus throat swab
and weight loss. He smoked 20 cigarettes per E. Sputum microscopy culture and sensitivity
day until 5 years ago. In addition, he worked
lagging pipes with 'monkey dung' during his 1.7.87. A 28 year old woman has an anterior
apprenticeship. cruciate ligament repair and is recovering at
Examination reveals a supraclavicular lymph home in an above-knee cast when she starts
node but no significant chest findings. His CT to feel like she will pass out every time she
scan notes the supraclavicular lymph node but stands up. She attends the emergency
also suggests there is a peripheral 5 em department.
peripheral mass in the left lung and an enlarged Examination reveals BP 80/45 mmHg, pulse
left-sided hilar lymph node. There is an 11 0 beats/min, oxygen saturations 92% on/.' air,
indeterminate lesion in the liver and MRI is respiratory rate 22 breaths/min, apyrexial.
suggested for clarification. Chest is clear. /
What should the next diagnostic test be? Investigations: CXR is clear. Bloods arrl as
A. CT-guided biopsy follows: haemoglobin 100 g/L, WCC
B. Endobronchial ultrasound 11 x 109/L, platelets 200 x 109/L, 0-dif)ler
C. Flexible bronchoscopy 1200 ng/ml, urea 10 mmoi/L (60 mg/dl),
D. Liver biopsy creatinine 92 11moi/L (1.04 mg/dl). EGG reveals
E. Supraclavicular lymph node biopsy sinus tachycardia. As the investigations are
being reviewed the patient has a cardiac arrest.
17.85. Which of the following associations in What should the immediate management
relation to lung disease is correct? include?
A. Anthrax and inadequately pasteurised milk A. Apixaban
B. Chlamydia psittaci and hide factory workers B. Intravenous heparin infusion
C. Coxiella burnetii and sewage workers C. LMWH
D. Francisella tularensis and muskrat contact D. Thrombolysis
E. Leptospiral pneumonia and welding E. Warfarin

17.86. A 45 year old man presents to the acute 17.88. A 26 year old w9man has been
receiving unit with sudden-onset right-sided increasingly short of _9feath over 2 years. She
pleuritic chest pain, shortness of breath and a attends the emergency department and is
swollen, painful right calf. He recently had noted to be hypoxaemic with swollen ankles.
right-sided anterior cruciate ligament Her EGG shows right bundle branch block. An
reconstruction abroad (following a skiing echocardiogram is arranged.
accident) and flew home from Canada in the Pulmonary hypertension is defined a~ a
last week. He has a past medical history of mean pulmonary artery pressure measured at

downloaded from www.medicalbr.com


RESPIRATORY MEDICINE • ~ 73

right heart catheterisation of at least which of 17.92. Which of the following statements is true
the following? with regard to breathing during sleep?

A. ~5 mmHg A. Abnormal ventilatory drive is present in


B. 25 mmHg obstructive sleep apnoea
c. 35 mmHg B. During sleep rnuscle tone increases
D. 45 mmHg C. Forty per cent of middle-aged wornen snore
E. 55 mmHg D. Hypoventilation accompanies norrnal sleep
E. Palatoglossus and genioglossus contract
17.89. A 24 year old woman presents with actively during expiration
breathlessness and palpitations that has
become worse over the preceding year. She 17.93. A 55 year old man has been increasingly
has had significant social stress because of the sleepy during the daytime. He is having trouble
end of a relationship and the death of her at work as he has been found asleep at his
mother. Examination reveals an elevated jugular desk and has taken to napping during his
venous pressure but no other abnormalities. An breaks. He had a near-rniss in his car. His
ECG has a rate of 76 beats/min and a Epworth sleepiness score is ~ 8. His BMI is 36.
rightward axis. CXR reveals a paucity of What is the result of his sleep study likely to
peripheral vasculature. be?
What would be the most appropriate next A. 5 apnoealhypopneas per hour of sleep
Investigation? B. ~ 0 apnoealhypopneas per hour of sleep
A. D-dimer C. 20 apnoealhypopneas per hour of sleep
B. Dysfunctional breathing studies and D. Central sleep apnoea
Nijmegen questionnaire E. Narcolepsy
C. HRCT
D. Spirometry 17.94. A 55 year old wornan has an apical lung
E. Transthoracic echocardiography cancer on the left side with lyrnph node

~I
involvement. She receives radiotherapy
17.90. A 23 year old woman has been following a mediastinoscopy. She has a hoarse
diagnosed with primary pulmonary voice and is worried this is because of her lung
hypertension following right heart turnour. Bronchoscopy reveals no vocal cord
catheterisation and extensive investigation to paralysis.
exclude alternative causes of her presentation What is the likely cause?
at a specialist unit. A. Chronic laryngitis
Which of the following therapies rnay B. Endotracheal intubation during
be indicated in primary pulmonary mediastinoscopy
hypertension? C. Laryngeal tuberculosis
A. Bosentan D. Left recurrent laryngeal nerve involvement by
B. Cyclizine the turnour
C. Etanercept E. Psychogenic aphonia
D. lnflixirnab
E. lsosorbide rnononitrate 17.95. A 34 year old man presents with
acute-onset shortness of breath and left -sided
17.91. A 45 year old wornan presents pleuritic chest pain. Examination reveals oxygen
with cough that appears in May and saturations of 94% breathing roorn air,
is gone by auturnn. In addition, she decreased air entry on the left side of the chest
experiences nasal discharge and watering with hyper-resonant percussion note. CXR
eyes. Examination is unremarkable and reveals large left-sided pneumothorax. A
spirometry is normal. therapeutic aspiration is performed and 2.5 L of
What is the likely diagnosis? air is aspirated with no change in the X-ray
A. Allergic asthrna appearance.
B. Allergic rhinitis What should be the n_ext step?
C. Bordete/la pertussis A. Adrnit for observation and oxygen
D. Perennial rhinitis B. Bronchoscopy
E. Viral upper respiratory tract infection C. Cardiothoracic surgery

downloaded from www.medicalbr.com


17 4 • RESPIRATORY MEDICINE

D. Continue therapeutic aspiration


E. Intercostal chest drain
C. Hernia through foramen of Bochdalek
D. Idiopathic diaphragmatic paralysis
1
I

E. Post -polio syndrome


17.96. A 72 year old man is increasingly
breathless and has woken in the night 17.97. Which of these associations between
breathless. He can no longer go for his regular pathophysiological features and conditions is
weekly swim. His past medical history includes correct?
ischaemic heart disease, hypertension and A. Diaphragmatic rupture and ankylosing
polio in childhood. spondylitis
His CXR is under-inspired despite three B. Eventration of diaphragm and foramen of
attempts. A sleep study reveals significant Bochdalek
hypoventilation and a morning arterial blood C. Pectus carinatum and poliomyelitis
gas identifies decompensated type II respiratory D. Pectus excavaturn and asthma
failure. What is the likely diagnosis? E. Thoracic kyphoscoliosis and tuberculosis
A. Bronchial carcinoma
B. Eventration of the diaphragm

Answers
17.1. Answer: A. mismatch and would be more likely to cause
The examination findings point towards type I respiratory failure. There are no
collapse of the right lung because the trachea examination findings in keeping with COPD and
is pulled to that side (the opposite would be the the oxygen involved (although delivered in an
case with pleural effusion). The collapse must uncontrolled fashion) is low flow.
be significant because the apex beat is not
palpable, suggesting the heart is pulled towards 17.3. Answer: C.
the right side by mediastinal shift. The patient's In COPD, loss of elastin fibres results in small
diagnosis was made at bronchoscopy, airway collapse and air trapping during /
suggesting a central tumour. Pneumothorax is expiration. This dynamic hyperinflation is ,Initially
less likely than if the patient had a peripheral noticed on exertion because expiration ti'me is
tumour that had been biopsied using computed shortened during exercise. Exercise-induced
tomography (CT) guidance. Pericardia! effusion bronchospasm would be more likely in asthma.
would not explain the respiratory examination There are no examination findings that suggest
findings. The presentation is too acute for sufficient pulmonary hypertension to cause
pneumonia and the patient is apyrexial. An breathlessness. There is no reason for the
urgent chest X-ray (CXR) would be an diaphragm to move paradoxically in this case.
important test and the patient may require Central chemoreceptors are stimulated by a
urgent radiotherapy or interventional rise in C0 2 , which might be expected in more
bronchoscopy to re-inflate the lung. advanced disease.

17.2. Answer: E. 17.4. Answer: A.


The patient has type II respiratory failure The CXR shows a large central cavitating mass
following a road traffic accident that caused with a smaller nodule immediately superior to
multiple fractures. It seems very likely that she the mass and a•further nodule at the lower pole
would have sustained a whiplash-type injury to of the right hilum. There is a small left-sided
the neck. C3, 4 and 5 innervate the diaphragm pleural effusion. The,possible answers given
via the phrenic nerve and a fracture at this level represent a reasol)cibte differential diagnosis for
can cause respiratory failure. The patient has this appearance. The history strongly favours a
type II respiratory failure with a normal bronchial carcinoma.
alveolar-arterial gradient. Opiate toxicity is
unlikely given the dose involved. A flail segment 17.5. Answer: B.
requires multiple rib fractures. A normal The CXR shows significant widening,of the
alveolar-arterial gradient is against V/6. mediastinum and a small right-sided pleural

downloaded from www.medicalbr.com


·-·--------~~~~-

RESPIRATORY MEDICINE • 175

effusion. Given the age of the patient, the function testing, she should be referred to the
imaging and the subacute presentation, the cardiothoracic surgery team.
likely diagnosis is lymphoma and CT scanning
is required to identify the extent of disease and 17.9. Answer: C.
identify a possible site for obtaining a tissue The CXR shows total collapse of the right lung
diagnosis. Echocardiogram might be helpful if with the heart and mediastinum shifted to the
aortic dissection or pericardia! effusion were right and tracheal deviation. This appearance is
suspected. PET scan might be useful in staging likely to have been caused by a proximal
or to assess treatment response. D-dimer is obstructing lesion such as a tumour.
less relevant with the obvious CXR abnormality.
Bronchoscopy is unlikely to be a useful test, 17.10. Answer: A.
although endoscopic ultrasound/endobronchial The CT scan shows evidence of bilateral,
ultrasound would allow nodal sampling. proximal bronchiectasis and an area of varicose
bronchiectasis in the right upper lobe and
17.6. Answer: B. non-specific inflammatory change. The clinical
The CXR shows the classic D-shaped picture and radiology point towards allergic
appearance of an empyema. Thoracic bronchopulmonary aspergillosis, although
ultrasound and diagnostic aspiration should be further investigation (e.g. peripheral blood
undertaken as an emergency. Insertion of an eosinophilia, total immunoglobulin E (lgE),
intercostal chest drain is a priority and CT Aspergillus precipitins - lgE specific to
scanning to plan an insertion site may be Aspergillus) would be required to confirm this.
required if the ultrasound scan appearance is
very complicated. Mesothelioma and bronchial 17.11. Answer: E.
carcinoma are less likely because of the acute The CXR shows right-sided pleural effusion
presentation. There is no abdominal element to with a meniscus appreciable. The trachea
the presentation on this occasion, although is relatively central as the right lung is
pancreatitis can cause large (usually left-sided) compressed by the pleural effusion. The
pleural effusion. Hepatic hydrothorax usually meniscus and homogenous opacification make
causes a simple, right-sided effusion, so the consolidation unlikely. This is not collapse
D-shape configuration would be very unusual. because the trachea is not pulled towards the
opacification and there is a meniscus. ;
17.7. Answer: E. Right -sided bronchial carcinoma might cause /
We do not have measurements for the small pleural effusion but the CXR does not give us
pulmonary nodule at the apex of the right lung this diagnosis. Right-sided mesothelioma
but repeat CT scanning is not an option, so we cannot be diagnosed based on CXR but is a•
must presume it is > 8 mm in maximum cause of pleural effusion.
diameter, and we note the patient is an
asymptomatic smoker, so a PET scan to 17.12. Answer: D.
assess nodule activity is the best answer. The The CXR shows an isolated right-sided pleural
patient is asymptomatic (the initial pain was on effusion. It seems unlikely that this relates to
the left and the lesion is on the right), so cardiac failure given the normal
commencing antibiotics and interferon-gamma echocardiogram. It will be important to further
release assay (IGRA) is irrelevant. Further risk investigate this with a diagnostic aspiration but
assessment prior to an invasive test is required this cannot be performed safely given the
as CT-guided biopsy is difficult in this area and apixaban therapy.
the nodule is likely to be too small to allow this.
Standard flexible bronchoscopy would not 17.13. Answer: E.
allow access to the nodule. The CXR shows a left-sided pneumothorax.
It seems likely the patient h<;ts a concurrent
17.8. Answer: E. respiratory tract infection that may be viral but I,

I I
The PET scan image reveals a fludeoxyglucose requires further investigation.
(FOG) avid right upper lobe pulmonary nodule i
''
that must be presumed to be an early-stage 17.14. Answer: E.
bronchial carcinoma. Given the patient's The patient has a large left -sided pneumothorax
performance status and normal pulmonary (> 2 em depth measured at hilum) and is

downloaded from www.medicalbr.com


176 • RESPIRATORY MEDICINE

symptomatic. He probably has a concurrent might have been expected to improve his
1
I

respiratory infection that may require symptoms if pneumonia was related to a more
investigation. The first step in management of a common pneumonia-causing organism (e.g. S.
primary spontaneous pneumothorax would be pneumoniae).
therapeutic aspiration.
17.20. Answer: B.
17.15. Answer: D. The patient presents with classic symptoms of
The CT-PET shows significant uptake in the pneumonia and might be expected to· isolate S.
right upper lobe cancer and in an ipsilateral pneumoniae on sputum examination given the
right hilar node, suggesting T4N2MO disease. rusty sputum, pleuritic chest pain and cold
There is apparent uptake in the marrow of the sores. The CXR shows left-sided basal
spine and sternum but this is physiological and consolidation.
not typical of metastatic deposit. The CT-PET
has upstaged the patient as the hilar lymph 17.21. Answer: A.
node was not obviously pathologically enlarged A typical clinical presentation and CT
on standard CT scanning. appearance should allow an interstitial lung
disease multidisciplinary meeting to reach a
17.16. Answer: C. diagnosis of idiopathic pulmonary fibrosis (IPF)
The patient has finger clubbing and bi-basal without a tissue sample. Bronchoscopy has no
crackles and presents with shortness of breath specific diagnostic features but may be useful
and dry cough late in life. The CT scan shows in the setting of intercurrent infection or atypical
bilateral peripheral lung cysts in a honeycomb CT scans/presentations. Transbronchiallung
pattern with some traction bronchiectasis. This biopsy would be risky and likely to be
clinical presentation and CT pattern is typical of non-diagnostic. There are no diagnostic lymph
idiopathic pulmonary fibrosis but should be node features in IPF.
confirmed by the assessment of an interstitial
lung disease multidisciplinary team. 17.22. Answer: E.
Direct visualisation and targeted biopsy of any
17.17. Answer: A. pleural lesion provides the best chance of a
The CXR shows extensive left-sided tissue diagnosis. Mesothelioma cannot b~
consolidation. The left costophrenic angle is diagnosed on cytopathological examinatipn of
clear so pleural effusion is unlikely. Loculated pleural fluid. Bronchoscopy would not help with
pleural fluid could give this appearance but the pleural disease but may be helpful if there were
likeliest cause is a pneumonia given the an endobronchial lesion with pleural
examination findings. metastases. Echocardiogram is not c6nsidered
helpful because there is pleural thickening as
17.18. Answer: C. well as a right-sided pleural effusion, so heart
The CURB-65 score was originally developed failure is an unlikely cause.
to predict mortality in community-acquired
pneumonia and is now widely used for the 17.23. Answer: B.
assessment of disease severity. The The patient requires a diagnostic test to
components are C = confusion, U = urea confirm lung cancer, determine histological type
> 7 mmoi/L (42 mg/dl), R = respiratory rate and complete staging. It seems likely he has
2 30 breaths/min, B = blood pressure systolic T2a, N1, MO disease. EBUS-FNA will meet all
< 90 mmHg or diastolic s 60 mmHg, 65 = age these requirements. Flexible bronchoscopy
2 65 years. His CURB-65 score is 2. He scores might give histological type but would not
points for confusion and urea. confirm disease in the hilar lymph node.
CT -guided biopsy is perhaps more likely than
17.19. Answer: C. flexible bronchosc9py to confirm histological
The patient has pneumonia with delirium, type but would have an attendant risk of
hyponatraemia, deranged liver function tests pneumothorax. Thoracoscopy has no role as
(LFTs) and high fever. He requires investigation there is no evidence of pleural disease. Repeat
for legionella and antibiotics to cover this CT scanning would delay diagnosis in a
organism. It may be that his LFT derangement scenario where curative treatment m<:J.Y be
relates to amoxicillin therapy but this antibiotic possible.

downloaded from www.medicalbr.com


RESPIRATORY MEDICINE • 177

17.24. Answer: E. very careful about attributing chronic cough to


seasonal deterioration in asthma control is a 'habit'. Reflux may be at the root of this cough
common presentation and it seems likely that but at this stage it is important to rule out
the patient encounters these allergens when parenchymal disease/bronchial carcinoma. A
she is out running. It will be important in future trial of steroid medication would usually be
to monitor the pollen counts and to avoid areas given if there was a suspicion of asthma or
of high antigen density (e.g. near oilseed rape eosinophilic bronchitis, but further testing would
fields). The patient may be allergic to cats and be useful prior to this. ENT surgeons may be
dogs but it would not explain this presentation. consulted when there is chronic rhinitis/sinusitis
It is possible to have positive specific lgEs but as part of a cough presentation, which does
no clinical phenotype on antigen exposure. The not respond to standard therapies.
history is as important as the serological
testing. Total lgE cannot be used to monitor 17.29. Answer: B.
medication adherence. Exercise-induced Given the CXR findings and history, reflux is the
asthma would not have such strong most likely cause even though the patient is
seasonality. asymptomatic (proton pump inhibitor started
following a duodenal ulcer, so no ongoing
17.25. Answer: C. heartburn). TB is unlikely as the cough is
Nucleic acid amplification testing of sputum or non-productive and there are no systemic
throat swab would differentiate between symptoms. Asthma is less likely because there
legionella and influenza in this case. The test is no nocturnal element. Hypercalcaemia does
result would be available quickly. Sputum not cause cough, although if it were present
cytology has a limited role in respiratory alongside cough it should raise the suspicion of
diagnostics. HRCT would be unlikely to offer malignancy. Lung cancer is unlikely because of
more information than the CXR. Paired serology the duration of the cough.
would determine whether the patient had been

~I
infected by legionella but would not be available 17.30. Answer: C.
in a timely fashion. Differential cell count would This is a very typical presentation of
not give a specific answer about the causative dysfunctional breathing. The patient had an
pathogen. initial illness, which may have altered the j
breathing pattern by preventing nasal breathin~,
17.26. Answer: C. and has a reasonable fear of lung cancer I
The patient has an obstructive defect that because a close relative died of it. A CTPA has
entirely resolves following nebulised ruled out an acute venous thromboembolism
bronchodilator; this is in keeping with asthma and parenchymal lung disease (including lung
despite the patient's smoking history. Inhaled cancer) as a cause. Echocardiogram might be
therapy taken on the morning of reversibility useful if there was a murmur, background of
testing can confound the test but in this congenital heart disease or if there were signs
instance there is a clear result with strong of right heart dysfunction (CTPA would not
evidence of reversibility. necessarily exclude chronic venous
thromboembolism (VTE) or pulmonary
17.27. Answer: C. hypertension). Normal CT scan makes ILD or
The gas transfer is reduced and partially emphysema unlikely. Arterial blood gas would
corrects for lung volume. There is no evidence not necessarily be abnormal in dysfunctional
of airways obstruction. The respiratory function breathing (although almost certainly would have
tests are non-diagnostic and further clinical shown respiratory alkalosis during the
details and, potentially, imaging studies are emergency department presentation).
likely to be required. FEV1 , FVG and FEV/FVC
are within normal limits. 17.31. Answer: C.
Although this woman has stopped smoking,
17.28. Answer: A. she accumulated a significant total number of
Chronic cough has many causes but it is pack years. Her history is of chronic
important not to miss parenchymal lung breathlessness, typical of COPD, and her CT
disease or bronchial carcinoma and a CXR scan shows emphysema. It is often difficult for
ought to be performed. Physicians should be patients to accept that although they have

downloaded from www.medicalbr.com


178 • RESPIRATORY MEDICINE

stopped smoking, their lung function continues regularly attending with exacerbations of
to decline. Spirometry is within normal limits bronchiectasis is likely to have had sputum
because she has emphysema-dominant screened for mycobacteria intermittently. There
disease with no evidence of airways obstruction are no systemic symptoms to suggest a
on testing. She would have an abnormal gas pneumonia and the haemoptysis is solely fresh
transfer. blood (not mixed in with purulent sputum) and
of sudden onset. Pulmonary infarction is less
17.32. Answer: E. likely because of the lack of other symptoms
The most likely cause of this pain is pulmonary (pleuritic chest pain and shortness of breath)
thromboembolism because she has recently and the CXR changes.
been on a long-haul flight, is hypoxaemic,
tachycardic and has a positive D-dimer. 17.36. Answer: A.
Malignant pleural disease would be unlikely to Bronchial artery angiography will demonstrate
have such a sudden onset. Pneumothorax and abnormally dilated areas of bronchial
pneumonia are not supported by the X-ray vasculature and with active bleeding can isolate
findings. Bronchospasm can give a central the leaking point. This can be difficult to
chest tightness but not a peripheral pleuritic interpret in chronic suppurative lung disease as
chest pain. Central chest tightness is a the bronchial vasculature is often diffusely
common finding in e«acerbations of COPD abnormal. Sputum culture is unlikely to be
because of coughing and strain on costal helpful in isolated massive haemoptysis
cartilages and intercostal muscles. although infection may be a precipitant in
chronic lung disease (such as cystic fibrosis
17.33. Answer: C. ·(CF) bronchiectasis). Bronchoscopy may be
The most likely cause is mesothelioma because helpful in the presence of a central lung tumour
~··
of the patient's employment history (asbestos and can sometimes determine whether the
was often found in the boiler rooms of older bleeding point is in the right or left lung (CXR or
ships), the insidious nature of the pain and its CT can be helpful here too), but bronchial
nagging quality. It seems likely the patient has artery angiography is increasingly favoured
a pleural effusion or perhaps pleural thickening. because of the potential to perform
Chronic thromboembolic disease might present embolisation of the aberrant artery. A CTfA
with recurrent pleuritic chest pain but more may determine the source of bleeding (e)g.
commonly is associated with progressive tumour or pulmonary arteriovenous I
breathlessness. Pneumonia might be expected malformation) in some cases but is unlikely to
to present with more systemic symptoms over be helpful here. Coagulation studies should be
a shorter time period. Tietze's syndrome performed as CF patients may be deficient in
presents more acutely and is usually vitamin K and can have liver disease, but the
self-terminating with supportive measures. more likely cause is abnormal bronchial
vasculature.
17.34. Answer: C.
It seems very likely that this man has familial 17.37. Answer: E.
clubbing. There is documented evidence of the Nodules greater than 4 mm require careful
presence of finger clubbing for approximately follow-up unless they have benign
30 years and the patient is asymptomatic. characteristics such as central deposition of
Interestingly, his father probably has finger calcification (which may suggest hamartoma).
clubbing too. There is no need for further Spiculated margins are more typical of
investigation or onwards referral. malignant lesions. Malignant nodules are more
common in upper lobes; benign nodules
17.35. Answer: B. distribute evenly. Sm9king is a very strong risk
The likely cause of haemoptysis here is a factor for lung car1_9er.
mycetoma developing in an old cavity caused
by tuberculosis. Carcinoma is less likely than 17.38. Answer: D.
mycetoma in this scenario but scar carcinomas PET scanning is u~eful for nodules > I em in
can develop in areas of the lungs previously diameter. It detects metabolic activity, which is
damaged by infection. Tuberculosis is likely to usually higher in malignant disease. However,
have been adequately treated and a patient metabolic activity can be high in inflammatory

downloaded from www.medicalbr.com


RESPIRATORY MEDICINE • 179

nodules, which can lead to false positives. nebulised bronchodilator and be admitted for
Tissue diagnosis must still be pursued even observation. Her therapy should not be
with a positive PET scan to identify the best reduced because the greater risk to patient and
treatment option. False-negative PET scans can fetus is uncontrolled asthma. Antibiotics would
occur in very slow-growing cancers or in only be considered where there was strong
neuroendocrine cancers. Metabolic activity is objective evidence of infection (fever, sputum
assessed by PET scan, which is not detected culture positive, CXR infiltrate). The presentation
by a single CT scan, although could be inferred is not suggestive of pulmonary
by nodule growth on serial CT scans. thromboembolism.

17.39. Answer: B. 17.44. Answer: B.


Transudative effusions include organ failure: The scenario does not provide enough
cardiac, renal, liver and thyroid. Hypothyroidism information to advocate initiation of omalizumab
is therefore the correct answer, although it is (we do not know patient's body mass
relatively rare. index (BMI), total lgE, sensitisation to
allergens). Montelukast and oral theophylline
17.40. Answer: B. preparations are recommended as
Light's criteria suggest an exudate additional therapy at this stage. Doubling the
where two of the three criteria are met. prednisolone would increase side-effects
It is nonetheless important to take a without increasing efficacy. There is no
holistic overview of the case as the criteria indication for an antibiotic here. Home
misclassify transudates as exudates 25% of nebulisers are not recommended in asthma
the time. because of the risk of late presentation with
significant exacerbation.
17.41. Answer: C.
The presentation and imaging are highly 17.45. Answer: C.
suggestive of empyema. As such, we would The patient's main complaint is of increased
expect a high pleural fluid protein and LDH and shortness of breath on exertion. ICS would be
a low pleural fluid glucose and pH. Elevated indicated if there was an increase in
pleural fluid triglycerides would be expected in exacerbation frequency. Oral prednisolone
chylothorax. would be useful in the context of a current
exacerbation of COPD. Ambulatory oxygen is
17.42. Answer: B. not indicated without desaturation on exertion.
Acute type I respiratory failure might be Whilst a nebuliser might well improve the
expected in lobar collapse where the collapsed patient's symptoms, an escalation of inhaled
lobe is underventilated but perfused and C0 2 is therapy would be a better approach. LABN
cleared in the neighbouring functional lung units LAMA combination inhalers offer enhanced
but haemoglobin saturation does not allow bronchodilatation and improvements in exercise
augmentation of oxygen uptake. If lobar tolerance.
collapse was associated with more widespread
mucus plugging and bronchospasm (e.g. in 17.46. Answer: D.
asthma or allergic bronchopulmonary A chronic productive cough should raise a
aspergillosis), type II respiratory failure might suspicion of bronchiectasis. HRCT would show
occur because of generalised V/6. mismatch. the thickened, dilated airways characteristic of
OSA usually causes chronic type II respiratory the disease. CXR might show bronchiectatic
failure. Flail chest injury might cause acute type changes but would not be as sensitive.
II respiratory failure, as might opioid toxicity. Bronchoscopy may be useful if sputum
Lymphangitis carcinomatosa would cause samples are not definitive (e.g. intermittent
chronic type I respiratory failure. non-tuberculous Mycobife':terium isolation).
a 1-Antitrypsin deficiency is a rare cause of
17.43. Answer: C. bronchiectasis but this test would follow CT
This patient is on maximal therapy for atopic diagnosis. lmmunoglob4)in levels can be low
asthma but has features of poor control where an immunodeficiency is.a cause of
suggesting a significant exacerbation. She bronchiectasis {lgA, lgM, lgE and lgG with
needs to have oral corticosteroid therapy, subclasses should be measured).

downloaded from www.medicalbr.com


180 • RESPIRATORY MEDICINE 1
I
17.47. Answer: B. 17.52. Answer: C.
The diagnosis is likely to be allergic The patient has recently returned from
bronchopulmonary aspergillosis. This is an the Middle East and has a serious
allergic reaction to Aspergillus, which can drive respiratory infection. Isolation and
excess sputum production, bronchiectasis and exclusion (or diagnosis) of MERS are priorities.
lobar collapse (this would explain the pleuritic Burkholderia pseudomallei is endemic to
chest pain and CXR findings). The key tests South-east Asia and Northern Australia. Local
would be Aspergillus serology, sputum antibiotic protocol is irrelevant here because of
mycology and HRCT. The management would the patient's travel history. CPE testing is
include oral corticosteroids, an antifungal agent, important when patients have been hospitalised
nebulised bronchodilators and chest in countries with a high prevalence of this
physiotherapy. Bronchoscopy to remove thick organism.
secretions may be required to promote lung
re-inflation. 17.53. Answer: D.
The patient has been in hospital for > 2 days
17.48. Answer: C. and has a clinical and radiological presentation
lvacaftor corrects the G551 D-mutant consistent with pneumonia. The priority is to
CFTR. G551D is present in 4% of the start treatment for HAP.
CF population, whereas Ll.F508 is present
in 70%. 17.54. Answer: A.
Late-onset HAP is most often attributable to
17.49. Answer: E. Gram-negative bacteria (e.g. Escherichia,
Pseudomonas aeruginosa is a significant Pseudomonas, Klebsiella spp. and
pathogen in CF bronchiectasis. It tends to Acinetobacter baumannit), Staphylococcus
emerge as patients move towards adulthood. aureus (including rneticillin-resistant Staph.
Intravenous antibiotics are often administered in aureus, MRSA) and anaerobes.
exacerbations caused by P. aeruginosa,
frequently by implanted subcutaneous venous
17.55. Answer: C.
access ports. Oral azithromycin therapy 3 times
The mortality from HAP is high, at
a week is an effective therapy but is not
approximately 30%. I
nebulised.
I
I
17.50. Answer: D. 17.56. Answer: B.
The common cold is most frequently caused by The patient has developed haematogenous
the rhinovirus. Bordetella pertussis causes lung abscesses from an infected injection site.
whooping cough, which can be very prolonged It is likely that the right side of the heart
in adults. (pulmonary and tricuspid valves) is affected and
the patient has infective endocarditis.
17.51. Answer: E.
The above presentation is typical of a 17.57. Answer: B.
lobar pneumonia caused by Streptococcus This presentation is most likely to be to be
pneumoniae. HSV is likely to be the optic neuritis secondary to ethambutol
cause of the cold sores that frequently (although isoniazid can also cause this).
accompany this presentation but would Patients starting this drug should be warned to
be a rare cause of a lobar pneumonia. report all eye-related symptoms to their clinical
Pneumocystis jirovecii is an important team and to stop ethambutol until advised
cause of pneumonia in immunocompromised otherwise.
individuals. Mycobacterium tuberculosis rarely
presents so acutely but should be 17.58. Answer: B.
considered in upper zone pneumonias, IGRA is less likely to give a false-positive
in the immunocompromised and in patients response in patients who have had the BCG or
with relevant travel or exposure histories. have opportunistic Qlycobacterial infection. It
Aspergillus can cause a lobar pneumonia but may be positive in active TB but also in latent
usually in the context of underlying lung TB and should not be used as a first-line
disease. diagnostic test or as a guide to therapy. The

downloaded from www.medicalbr.com


RESPIRATORY MEDICINE • 181

test relies on the release of interferon-gamma not associated with the usual features of the
from sensitised T cells. carcinoid syndrome.

17.59. Answer: E. 17.65. Answer: D.


This presentation is consistent with psoas Significant survival benefits are conferred by
abscess and, whilst there may be a treatment with tyrosine kinase inhibitor drugs in
bacteriological cause, this would be a typical patients with EGFR mutation, even in the
extrapulmonary presentation of TB. Malignancy presence of metastatic disease. The presence
is a major differential diagnosis. The type of of these mutations is more common in
imaging utilised above is not identified, but a non-smoking patients with adenocarcinoma.
CT/magnetic resonance imaging (MRI} would
be invaluable to assess for spinal disease and 17.66. Answer: B.
potentially cord involvement. The possible answers reflect a sensible
differential diagnosis for this presentation. The
17.60. Answer: E. most likely cause, however, is lymphangitis
The clinical presentation and radiology findings given the insidious onset, the presence of
are suggestive of invasive pulmonary metastatic disease in the lungs and the CT
aspergillosis and treatment with an antifungal findings.
agent should not be delayed for further testing
(although mycological culture would be 17.67. Answer: B.
indicated - induced sputum/bronchoscopy). An The most likely diagnosis is lymphoma because
intercurrent VTE is thought less likely given the of the tumour location and the
scenario, but pleuritic chest pain and lymphadenopathy.
haemoptysis should bring this to mind in other
contexts. Voriconazole is currently first-line 17.68. Answer: B.
therapy. Insidious breathlessness in a woman in her
eighth decade with typical examination
17.61. Answer: A. and CT findings mean that this woman
Adenocarcinoma has recently become the is likely to receive a diagnosis of idiopathic
commonest cause of lung cancer. pulmonary fibrosis without further invasive 1
investigation after discussion at an interstitial )
17.62. Answer: E. lung disease multidisciplinary meeting. Lung I
The most likely cause is HPOA, a painful biopsy adds little information and may be
periostitis of the distal radius and ulna. Finger associated with significant morbidity and even
clubbing is not painful. Bone metastases are mortality.
possible but less likely. Hypercalcaemia can
certainly cause bone pain and measuring 17.69. Answer: D.
calcium levels is always indicated when there is RBILD is a smoking-associated idiopathic
bone pain. Horner's syndrome can cause pain interstitial pneumonia that is more common in
in the inner aspect of the arm and small muscle men and presents between the ages of 40 and
wasting in the hand. 60 years.

17.63. Answer: D. 17.70. Answer: D.


The presence of a cytology-positive pleural The symptom complex, combined with a
effusion automatically makes this lung cancer typical CXR appearance (bilateral hilar lymph
stage IV. If the pleural effusion had an node and paratracheal lymph node
alternative cause such as intercurrent infection enlargement) is sufficient for a diagnosis of
and was cytology-negative, the tumour could sarcoidosis and this presentation usually has an
be stage lb. excellent prognosis.

17.64. Answer: C. 17.71. Answer: E.


A young non-smoker with isolated haemoptysis The presence of extensive. fibrosis even in the
and a localised, vascular tumour is likely to presence of enlarged lymph nodes is
have a bronchial carcinoid. These tumours are suggestive of sarcoidosis stage IV. The
rare but have an excellent outcome. They are patient's breathlessness and abnormal lung

downloaded from www.medicalbr.com


182 • RESPIRATORY MEDICINE

function suggest the patient has respiratory 17.76. Answer: D.


failure from a silent progression of pulmonary The answers above are a reasonable differential
sarcoidosis. The absence of inspiratory crackles diagnosis for the presentation. A progressive
on examination suggests the fibrotic change is pneumonitis is a side-effect of nitrofurantoin
due to sarcoidosis. taken long term for prevention of urinary tract
infections. The subacute onset with systemic
17.72. Answer: E. symptoms mitigates against IPF or NSIP, whilst
Rheumatoid arthritis has many respiratory atypical pneumonia would be expected to be
complications including nodules, pleural more acute. A cryptogenic organising
effusion, bronchiectasis and interstitial lung pneumonia should not be assumed whilst a
disease. In addition, medications given for potential cause is evident.
rheumatoid arthritis also have respiratory
complications and leave patients prone to a 17.77. Answer: C.
range of respiratory infections. The recurrence of pneumothorax in the
presence of a multicystic lung disease and the
possibility of a renal tumour in a young woman
17.73. Answer: E.
suggests lyphangioleiomyomatosis may be the
The patient's CXR appearance and pleural fluid
underlying diagnosis. Pulmonary Langerhans
characteristics are typical of empyema.
cell histiocytosis is strongly associated with
However, the recurrence on the opposite side
smoking. Lymphocytic interstitial pneumonia is
in the presence of antibiotic therapy and
usually associated with connective tissue
development of arthritis make rheumatoid
disease or HIV. The other conditions do not
arthritis more likely. Lung cancer can produce
cause lung cysts and are not associated with
pleural effusions but the joint findings are not
pneumothorax.
typical of HPOA. Mesothelioma does not tend
to present so acutely and is usually unilateral at
17.78. Answer: B.
presentation. Lymphoma often presents with
All of these tests might be useful in
pleural effusions but the pH and glucose levels
investigating this patient's case. However, a
would not be typical.
detailed PEFR diary that measures a minimum
of 4 times per day for at least 3 weeks an~
17.74. Answer: E. during a period away from work would bEj most
The high peripheral blood eosinophil count, informative about whether the patient is I
peripheral X-ray shadowing and potential experiencing occupational asthma.
causative agent (daptomycin) make
drug-induced chronic eosinophilic pneumonia 17.79. Answer: A.
the likely diagnosis. As the patient has become A patient who works with beryllium and
systemically unwell with an oxygen requirement presents with a sarcoid-like clinical picture and
it is reasonable to start prednisolone aswell as imaging should have berylliosis excluded as a
stopping the causative agent. The high priority. Her husband's work should not
eosinophil count makes HAP unlikely. TB influence her respiratory status unless she is
should be considered (it can also cause a spending significant amounts of time assisting
discitis) but does not cause a peripheral blood him. It is possible that where there is damp,
eosinophilia. Fluid overload is considered less mould is also present, but hilar adenopathy
likely because of the presence of fever and would be uncommon in hypersensitivity
atypical radiology. pneumonitis. Although she has some work
stress, dysfunctional breathing would not
17.75. Answer: E. explain her arthralgia, cough or X-ray
The presentation here is of multisystem appearances.
granulomatous polyangiitis. The patient has
multiple nodules on CXR and the presentation 17.80. Answer: E.
with stridor suggests subglottic stenosis. It Finger clubbing and basal crackles are
seems likely the patient also has renal characteristic of idiopathic pulmonary fibrosis.
involvement. Sinusitis and conductive deafness Melanoptysis (black sputum) is associated.
are also suggestive of granulomatous Simple coal worker,'s pneumoconiosis has no
polyangiitis. impact on lung func;tion but PMF may lead to

downloaded from www.medicalbr.com


RESPIRATORY MEDICINE • 183

respiratory failure. PMF is characterised by 17.85. Answer: D.


large conglomerate masses. Chlamydia psittaci infects birds
(e.g. parrots and budgerigars).
17.81. Answer: C. Coxiella burnetii is the causative agent of Q
Silicosis results from the inhalation of crystalline fever, and farm workers, abattoir workers and
silica usually in the form of quartz. hide factory workers may be exposed. Anthrax
may occur in workers exposed to infected
17.82. Answer: E. hides, hair, bristle, bone meal and animal
The diagnosis of exclusion here is carcasses.
mesothelioma. The patient has a pleural
effusion, extension of pleural thickening onto 17.86. Answer: A.
the mediastinum, chest wall pain that keeps This is a clear case of pulmonary
him awake at night and significant asbestos thromboembolism (PTE) in a high-risk patient.
exposure. Although he has had benign D-dimer has been inappropriately checked as
asbestos pleurisy in the past, the presentation the patient is high risk and this test only safely
here is more sinister. Asbestosis is a fibrosing excludes VfE in low-to-moderate risk patients.
lung condition that would be picked up on CT. Echocardiogram may show right heart strain
Although the patient has pleural plaques, these but cannot diagnose PTE. Sputum microscopy,
should not affect his respiratory function or culture and sensitivity, and respiratory virus
clinical condition. throat swab would be appropriate if the history
and CXR were in keeping with respiratory
17.83. Answer: C. infecti.on. The elevated diaphragm here is likely
The patient has exposure to a parrot and to reflect pleuritic pain or potentially atelectasis
a CT in keeping with hypersensitivity in keeping with PTE. Diaphragmatic studies are
pneumonitis. It is very likely she has not indicated here.
bird fancier's lung. Although the CT was
focused on the coronary arteries, the image 17.87. Answer: D.
quality is not likely to have created these
findings by artefact. Aspirin sensitivity presents
The patient's presentation is typical of a large,
central pulmonary thromboembolism and she . .I
differently and her shortness of breath was has suffered a cardiac arrest. The immediate
present prior to starting aspirin. Sarcoidosis is
possible (it is nearly always in the differential for
abnormal CT chest) but the history of a parrot
at home needs to be explored first. Idiopathic
management should include resuscitation and /
thrombolysis. Thrombolysis is indicated in
hypotension unresponsive to fluid resuscitation
and in cardiac arrest.
I
I
pulmonary fibrosis has a different CT
appearance. It would be important to ensure a 17.88. Answer: B.
CT scan focused on the lungs was arranged to Pulmonary hypertension is defined as mean
ensure optimal imaging available at baseline. pulmonary artery pressure of at least 25 mmHg
Removal of the parrot and deep cleaning of the at rest, as measured at right heart
room it resided in are likely to lead to complete catheterisation.
resolution of the clinical and radiological
findings. 17.89. Answer: E.
This presentation could be compatible with
17.84. Answer: E. primary pulmonary hypertension, although this
The patient is at significantly increased risk of would require further investigation. A
lung cancer because of his smoking and his transthoracic echocardiogram is a good initial
occupation ('monkey dung' is a form of test. Dysfunctional breathing should not be
asbestos). The least invasive test that will give a assumed because of social 1 stress, especially in
histological diagnosis is peripheral lymph node the presence of physical fil<amination findings
biopsy. Endobronchial ultrasound would be a and abnormal investigations. HRCT might
reasonable test but is more invasive. Liver identify enlarged pulmonary arteries but would
biopsy may not give an answer as the lesion not be the ideal study. D·-c!imer would not be
may not be related to the primary cancer. appropriate as it would not rule out chronic
Flexible bronchoscopy is less likely to give an thromboembolic disease as the cause of
answer in a peripheral tumour. pulmonary hypertension. Spirometry would not

downloaded from www.medicalbr.com


184 • RESPIRATORY MEDICINE

necessarily show an abnormality, although gas 17.95. Answer: E.


transfer is often reduced. This is a primary spontaneous pneumothorax
(PSP). Therapeutic aspiration was the
17.90. Answer: A. appropriate first step but has failed. An
Endothelin inhibitors such as ambrisentan and intercostal chest drain would be the
bosentan are therapies used in primary next step as most PSPs will resolve
pulmonary hypertension. Cyclizine and nitrates without the need for referral to cardiothoracic
are contraindicated. There is no role for surgery. An intervention is required to
anti-tumour necrosis factor agents. treat the pneumothorax, although
observation and oxygen would be
17.91. Answer: B. useful whilst awaiting a skilled intercostal chest
The seasonal presentation with prominent nasal drain practitioner. Bronchoscopy has no role
symptoms and watering eyes suggests allergic here but may be useful in lung/lobar collapse,
rhinitis. which should not be confused with
pneumothorax.
17.92. Answer: D.
Palatoglossus and genioglossus contract 17.96. Answer: E.
actively during inspiration. Abnormal ventilatory This man has a past history of polio and he
drive is present in central sleep apnoea. Forty seems to have bilateral diaphragmatic
per cent of middle-aged men snore. During weakness that is significantly impairing his
sleep muscle tone decreases. respiratory function. A diaphragmatic defect or
e~entration would be unlikely to cause
17.93. Answer: C. respiratory failure. Bronchial carcinoma would
The patient's presentation is typical of be more likely to cause unilateral diaphragmatic
obstructive sleep apnoea and 15 or more paralysis. A history of polio means that this is
apnoealhypopnoeas per hour of sleep is unlikely to be idiopathic.
diagnostic.
17.97. Answer: E.
17.94. Answer: B. Thoracic kyphoscoliosis is caused by vertebral
There is no vocal cord paralysis, so local disease, trauma, neuromuscular disease or ;can
trauma during intubation is the likely cause of be a congenital abnormality. Asthma is /
the acute-onset hoarseness. associated with pectus carinatum. I

downloaded from www.medicalbr.com


AJ Anderson

Endocrinology
Multiple Choice Questions
18.1. A 22 year old woman presents with a D. Increased growth hormone (GH)
few weeks' history of malaise and weight loss. E. Increased transforming growth factor-alpha
On clinical examination she has palmar (TGF-a)
hyperpigmentation. With which investigation
should she be followed up? 18.3. A 28 year old woman presents with
A. Dexamethasone suppression test secondary amenorrhoea and galactorrhoea.
B. Magnetic resonance imaging (MRI) abdomen An MRI scan of her brain is likely to show a
C. MRI pituitary lesion in which area?
D. Synacthen test A. Anterior pituitary
E. Thyroid function tests B. Hypothalamus
C. Lactiferous ducts
18.2. A 52 year old South Asian man is found D. Pars intermedia ,Ir
E. Posterior pituitary
to have thickened pigmented skin at the back
of his neck and in the axillae. His body mass I
index (BMI) is elevated at 38 kg/m 2 . Acanthosis 18.4. A 38 year old man is referred with a
nigricans in this setting is due to which of the history of polydipsia and polyuria passing over
following pathology? 3 L of urine in 24 hours. He undergoes a water
A. Axillary perspiration and friction deprivation test, which shows the following
B. Hyperinsulinaemia results:
C. Increased fibroblast growth factor activation

Plasma Plasma Glucose Urine


sodium osmolality (mmoi/L) (mg/dl) osmolality Urine volume
Time/result (mmoi/L) (mOsm/kg) (mOsm/kg) (L)
0 hours 144 303 5.1 92 223 0.04
2 hours 144 301 5.2 94 346 0.08
Desmopressin (DDAVP) 2 ll9 intramuscular
1 hour 138 292 4.8 86 528 0.02
post-DDAVP

What is the underlying cause? 18.5. Where does arginine/asopressin (AVP)


A. Cranial diabetes insipidus exert its maximum effect1 in the kidney?
B. Diabetes mellitus A. Collecting ducts
C. Nephrogenic diabetes insipidus B. Distal convoluted tubule
D. Normal response to water deprivation C. Glomerulus
E. Psychogenic polydipsia D. Loop of Henle
E. Proximal tubule

downloaded from www.medicalbr.com


186 • ENDOCRINOLOGY

18.6. A 21 year old student is found to have (0.46 ng/dL) and TSH <0.01 miU/L. TSH
hyperthyroidism. She is counselled on receptor antibody (TRAb) levels are not
treatment options including radioactive iodine elevated. What is the most appropriate
and antithyroid medications. Carbimazole acts management?
on which part of the thyroid hormone synthesis A. Commence propranolol
pathway? B. Consent for radioactive iodine
A. Cleavage of thyroglobulin by proteolysis C. Perform ultrasound scan
B. Coupling of monoiodotyrosine (MIT) and D. Screen the infant for hyperthyroidism
diiodotyrosine (DIT) forming triiodothyronine E. Treat with selenium
CTsl and thyroxine (T4 ) 18.10. A 40 year old male smoker presents
C. Dehalogenation of iodinated tyrosine to
with weight loss and blood tests suggesting
recycle iodide
biochemical primary hyperthyroidism. Which of
D. Organification of iodide by thyroid peroxidase
the following features would suggest that the
incorporating tyrosine forming MIT and DIT
hyperthyroidism is due to Graves' disease?
E. Thyroglobulin synthesis
A. Eyelid retraction
18.7. A 56 year old woman is reviewed in clinic. B. Gynaecomastia
She was diagnosed with hypothyroidism 15 C. Lack of orbitopathy
years previously and has been on levothyroxine D. Male gender
100 ~g once daily ever since. Recent thyroid E. Palpable smooth goitre with bruit
function tests have shown thyrotrophin 18.11. A 74 year old woman is admitted to
(thyroid-stimulating hormone; TSH) 8.2 miU/L .hospital with a 3-month history of lethargy,
and free thyroxine (free T4) of 15.6 pmoi/L weight gain and increasing shortness of breath.
(1.21 ng/dL). TSH secretion by the Hypothyroidism can result in which of the
hypothalamus is increased by which of the following cardiovascular effects?
following?
A. Diastolic hypertension
A. A decrease in thyroxine-binding globulin B. High cardiac output
levels C. Low cholesterol
B. A large increase in free T4 beyond the normal D. Reduced peripheral vascular resistance
reference range E. Systolic hypertension /
C. During early hours of the morning
D. A fall in free T4 of 5 pmoi/L (0.39 ng/dL) 18.12. A 34 year old woman presents tolher
E. An increase in circulating free T3 family physician with weight loss, palpitations
and amenorrhoea. Thyroid function tests
18.8. A 23 year old asymptomatic woman confirm thyrotoxicosis with free T4 30.2 pmoi/L
attends her family physician for thyroid function (2.35 ng/dL) and TSH <0.01 miU/L. TRAb
testing as her mother has recently been levels are not elevated. A thyroid scintigraphy
commenced on levothyroxine. Thyroid function scan is performed revealing the following
tests (TFTs) show TSH 6miU/L, and free pattern of uptake.
T4 of 12.4 pmoi/L (0.96 ng/dL). Her serum
thyroid peroxidase antibodies are strongly
positive. What is the most appropriate
management plan?
A. Arrange a scintigraphy scan
B. Check thyroglobulin antibodies
C. Reassure and discharge
D. Repeat TFTs in 4-6 months
E. Start levothyroxine and recheck TFTs in
6 weeks

18.9. A 26 year old woman presents 12 weeks


post-partum with symptoms of weight loss,
palpitations and troublesome tremor. Her
thyroid function is checked, revealing free T4
24.2 pmoi/L (1.88 ng/dL), free T3 7.1 pmoi/L

downloaded from www.medicalbr.com


ENDOCRINOLOGY • 187

What is the most likely diagnosis? 18.15. A 23 year old white woman presents
A. Exogenous thyroxine intake with a 6-month history of increasing neck
B. Graves' disease swelling and discomfort on swallowing.
C. Iodine deficiency On examination she has a smooth diffuse
D. Toxic multinodular goitre symmetrical goitre. Thyroid function tests
E. Transient thyroiditis are normal and thyroid antibody levels are
undetectable. Ultrasound shows a diffuse
18.13. A 28 year old man presents to his family and symmetrical echogenic pattern,
physician with a 6-month history of neck with no significant nodularity. Which
swelling. On examination he has a 2x3 em of the follow statements is correct in this
palpable lump on the left side of his neck, which scenario?
moves with swallowing. He has no associated
A. Associated lymphadenopathy is normal
clinical symptoms. He undergoes a scintigraphy
B. Radioactive iodine treatment should be used
scan that reveals the following image.
to shrink the gland
C. She is likely to experience symptoms of
lethargy and weight gain
D. The goitre may enlarge during pregnancy
E. There is a high risk of malignancy

18.16. A 53 year old woman whose thyroid


function tests have been stable and within the
normal range on 200 J.Lg of levothyroxine for
the last 6 months is found to have a TSH
<0.01 miU/L. You ask her about her current
medications including those bought over the
counter. Which of the following is associated
with increased bioavailability of levothyroxine?
A. Calcium
B. Colestyramine
C. Iron
D. Phenytoin
E. Vitamin C
What is the most likely underlying diagnosis?
A. Hashimoto's thyroiditis 18.17. A 25 year old woman was commenced
B. Thyroglossal duct cyst on levothyroxine 3 weeks ago for primary
C. Thyroid carcinoma hypothyroidism. She is concerned about
D. Toxic adenoma ongoing symptoms of dry skin and dry hair.
E. Toxic multinodular goitre She has been otherwise well with nil else to
report on systemic enquiry. What is the most
18.14. A 28 year old man is referred to appropriate management plan?
the endocrinology clinic with a relapse of A. Add in liothyronine
Graves' disease. Thyroid function tests reveal B. Check her anti-tissue transglutaminase
free T4 28.4 pmoi/L (2.21 ng/dl), TSH antibody levels
<0.01 miU/L and TRAb 7.6iU/L. He is C. Check her TRAbs
considering radioactive iodine C31 1) treatment. D. Increase levothyroxine dose now
Which of the following statemynts is TRUE E. Reassure and organise for repeat thyroid
about this treatment? function test in a further 3, weeks
A. 131 1 is given as an intravenoJJs infusion I
B. He has a 10% risk of devel~ping 18.18. Thyroid function tests are performed
hypothyroidism in the first 12 months on a patient who has been on levothyroxine
C. He is likely to lose weight f~llowing 131 1 replacement for several years. The following
D. He is safe to father a child within 6 months results are observed. TSH 8.2 miU/L, free T4
of treatment 21.6 pmoi/L (1.68 ng/dl) and free T3
E. It may cause deterioration of active Graves' 4.2 pmoi/L (0.27 ng/dl). What is the most likely
ophthalmopathy underlying diagnosis?

downloaded from www.medicalbr.com


I 88 • ENDOCRINOLOGY

A.
B.
C.
Non-thyroidal illness
Non-functioning pituitary tumour
Thyroiditis
18.21. A 72 year old man was commenced on
amiodarone for atrial fibrillation 6 months
previously. He is referred with a history of
r
D. Treatment with amiodarone weight loss, tremor and sweating. Blood tests
E. Variable treatment adherence reveal TSH <0.01 miU/L, free T4 26.1 pmoi/L
(2.03 ng/dl) and free T3 4.1 pmoi/L (0.27 ng/
18.19. A 32 year old woman attends her family dl). What is a scintigraphy scan likely
physician with symptoms of tremor and vague to show?
anterior neck discomfort. Initial blood tests A. Diffuse increased uptake
reveal a free T4 14 pmoi/L (1.09 ng/dl) and B. Diffuse low uptake
TSH 0.12miU/L. Subsequent TFTs 4 weeks C. Multinodular goitre
later show TSH 89.8miU/L and free D. Normal uptake
T4 <5 pmoi!L (0.39 ng/dl). Scintigraphy scan E. Solidary toxic nodule
shows the following:
18.22. What is the correct treatment for
type II thyrotoxicosis secondary to
amiodarone?
A. Carbirnazole
B. Discontinue amiodarone only and allow
resolution of thyroid function
C. Glucocorticoids
D. Levothyroxine
E. Radioactive iodine

18.23. A 62 year old woman is referred with a


3-month history of diffuse swelling in her neck
and weight loss. On palpation she has a large,
firm nodular goitre and non-tender cervical
lymphadenopathy She complains of ,
hoarseness of her voice. What is her likely
diagnosis? 1

A. Anaplastic carcinoma
What is the most likely underlying cause? B. Follicular cell carcinoma
A. Graves' disease C. Medullary carcinoma
B. Primary hypothyroidism D. Papillary cell carcinoma
C. Subacute thyroiditis E. Toxic adenoma
D. Toxic adenoma
E. Toxic multinodular goitre 18.24. A 34 year old woman is reviewed
in the thyroid cancer clinic following total
18.20. A 75 year old man with a past medical thyroidectomy and neck dissection with lymph
history of ischaemic heart disease is admitted node clearance for a confirmed (pT 4, N I , MO)
to hospital following a fall at home. He papillary thyroid cancer. Follow-up management
becomes increasingly delirious on the ward. post-surgery should include which of the
As part of his work-up, thyroid function is following?
checked showing TSH 0.2miU/L, free T4 A. Levothyroxine replacement aiming to keep
26.2 pmoi/L (2.04 ng/dl) and free T3 TSH within the normal range
3.3 pmoi/L (0.21 ng/dl). What is the most B. Radioactive iodine treatment when TSH is
appropriate management plan? fully suppressed
A. Check for TRAbs C. Regular computed tomography (CT) scans of
B. Commence carbimazole treatment the neck
C. Consent for radioactive iodine treatment D. Screening of family members for thyroid
D. Discontinue cardiac medications cancer
E. Repeat thyroid function tests when the E. Thyroglobulin measurement at regular
patient has fully recovered intervals ·

downloaded from www.medicalbr.com


ENDOCRINOLOGY • 189

18.25. A 32 year old woman attends her family total testosterone 5.6 nmoi/L (162 ng/dl),
physician with a 12-month history of inability to SHBG 42.1 nmoi/L (4.00 Jlg/ml), FSH 2.1 IU/L
conceive. She has been having regular periods (0.5 Jlg/L) and LH 1.76 IU/L (0.2 Jlg/L). He has
every 28 days. Ovulation can be confirmed by no other past medical history of note and has
which of the following tests? not fathered any children. On examination
A. Day I 0 rise in follicle-stimulating hormone visual fields are normal, testes are 5 ml volume
(FSH) and soft on palpation. He has little in the way
B. Day 13 surge in oestradiol of pubic or axillary hair. He has not noticed· any
G. Day 14 surgein progesterone problem with his sense of smell. Which of
D. Day 14 surge in luteinising hormone (LH) the following is the most likely underlying
E. Regular menses diagnosis?
A. Kallmann's syndrome
18.26. You review a 21 year old woman in the B. Klinefelter's syndrome
reproductive endocrinology clinic. She has a G. Previous trauma to the testes
history of secondary amenorrhoea and anorexia D. Reduced testosterone secondary to obesity
since she was 18 years old. She is keen to E. Reduced testosterone with age
know why she has stopped having periods.
Functional hypothalamic amenorrhoea is 18.30. An 18 year old woman with a BM I of
underpinned by which process? 31 kg/m 2 attends her family physician with
A. A high LH-to-FSH ratio troublesome hirsutism, acne and irregular
B. Gonadotrophin-releasing hormone (GnRH) periods. Hyperandrogenism as a sequela of
resistance polycystic ovary syndrome (PCOS) may result
G. High circulating leptin from which of the following?
D. Hyperprolactinaemia A. Genetic mutation in 3~-hydroxysteroid
E. Reduced pulsatility and secretion of GnRH dehydrogenase
B. Higher FSH compared with LH synthesis by
18.27. A 16 year old girl presents to her the pituitary gland
family physician having never had a period. G. Increased pulsatility of GnRH
She is noted to be of short stature. Blood D. Reduced aromatisation of androgens by
tests reveal FSH 26.2 IU/L (5.9Jlg/L), LH theca cells
18.5 IU/L (2.0 Jlg/L) and oestradiol <50 pmoi/L E. Reduction in circulating SHBG
(13.6 pg/ml). What is the next most
appropriate investigation? 18.31. A 23 year old student with known
A. CT scan ovary and adrenal glands karyotype 45X attends the clinic seeking
B. Karyotype advice as she is wanting to achieve pregnancy.
G. MRI pituitary Which of the following should she be
D. Synacthen test counselled about?
E. Ultrasound scan ovaries A. Child is more likely to have low 10
B. Increased risk of ovarian cancer
18.28. A 12 year old boy attends his family G. She will require anti-androgen therapy
physician as it has been noticed that he is D. She will require screening for aortic
falling behind in school and is considerably dissection
shorter than his classmates. Which of the E. There is a high chance of her becoming
following is consistent with the diagnosis of pregnant spontaneously
constitutional delay?
A. Bone age consistent with chronological age 18.32. A 17 year old boy is referred with
B. More common in females ' delayed puberty. He has nQticed that he is
G. Occurs as young as 3-6 months of age taller than his classmate7!0n clinical
D. Smaller adult height than predicted examination he is found to have sparse facial
E. Upper-to-lower body ratio< 1 and body hair as well as small pre-pubertal-
sized testes and penis. Blood tests reveal
18.29. A 56 year old man is referred to the testosterone 7.5 nmoi/L (216 ng/dl), FSH
endocrinology clinic with a history of poor libido 12 IU/L (2.8 [Jg/L), LH II IU/L (1.2 11g/L). What
and erectile dysfunction. Blood tests reveal is the most appropriate next test?

downloaded from www.medicalbr.com


T
190 • ENDOCRINOLOGY

A. Chromosomal analysis A. Dehydration secondary to


B. GH measurement gastroenteritis
C. Karyotype B. Malignancy
D. Prolactin level c. Mutation in calcium-sensing receptor gene
E. Thyroid function test D. Mutation in the MENIN gene (MEN 1)
E. RET oncogene mutation (MEN 2/3, also
18.33. What is the principal pathological known as MEN 2a/2b}
abnormality in Klinefelter's syndrome?
18.37. A 36 year old woman is admitted to
A. Defect in transforming growth factor-~ hospital with abdominal pain. Blood tests reveal
(TGF-~)
signalling pathway calcium 3.01 mmoi/L (12.1 mg/dl}, phosphate
B. Disordered migration of GnRH-producing 0.5 mmoi/L (1.55 mg/dl}, magnesium
neurons 0.8 mmoi!L (1 .94 mg/dl), urinary calcium
c. Dysgenesis of the seminiferous tubules 12.8 mmol/24 hrs (512 mg/24 hrs;
D. Dysregulated testosterone synthesis 32.0 mmoi/L} (elevated), PTH 12.3 pmoi!L
E. LH resistance (116 pg/ml}. What is the most appropriate
follow-up management plan?
18.34. A 62 year old man has noticed that his
A. Commence loop diuretic
breasts have been swollen and tender to touch.
B. Commence thiazide diuretic
He is on a long list of medications. Which of
C. Monitor calcium levels
the following medications can cause
D. Refer for parathyroid surgery
gynaecomastia?
E. Vitamin D replacement
A. Bendroflumethiazide
B. Dihydrotestosterone 18.38. A 52 year old woman is referred to the
C. Finasteride clinic with a 4-month history of lethargy,
D. Metolazone generalised weakness and pains in her hands
E. Tamoxifen and feet. Blood tests reveal TSH 1.3miU/L
(normal), total free T4 18.4 pmoi/L (1.43 ng/dl),
18.35. A 72 year old man with a history of calcium 2.2 mmoi/L (8.81 mg/dl}, phosphate
chronic kidney disease stage 4 is admitted with 0.6 mmoi/L (1.86 mg/dl), PTH 6.4 pmol/~
perioral paraesthesia and cramping in his (60 pg/ml), alkaline phosphatase 158 IU1L,
hands. Blood tests reveal calcium 1.7 mmoi!L creatinine 73 J.lmoi/L (0.83 mg/dl).
(6.81 mg/dl) and parathyroid hormone (PTH} What is the likely underlying diagnosis?
level 8.7 pmoi!L (82 pg/ml}. PTH has which of
A. Autoimmune polyendocrine syndrome
the following effects?
B. Haemochromatosis
A. Absorption of calcium from the gut C. Magnesium deficiency
B. Absorption of phosphate from the gut D. Parathyroid adenoma
c. Conversion of 25-hydroxyvitamin D to E. Vitamin D deficiency
1 ,25-dihydroxyvitamin D
D. Excretion of magnesium from the distal 18.39. A 33 year old man is found to have a
convoluted tubule significant postural hypotension with a drop in
E. Increases phosphate reabsorption from systolic blood pressure from 130/75 to
proximal tubule 100/60 mmHg. Further investigations reveal
elevated renin levels. but low aldosterone levels
18.36. A 32 year old man is admitted to hospital on sitting. Where in the adrenal gland is the
with gastroenteritis. Blood tests reveal total enzyme aldosterone synthase located?
calcium 2.72 mmoi/L (10.9 mg/dl), potassium
A. Chromaffin cells.
3.7 mmoi/L, magnesium 1.2 mmoi/L (2.92 mg/
B. Medulla
dl}, urea 4.7 mmoi/L (28.2 mg/dl), creatinine
C. Zona fasciculata
76 l..tmoi/L (0.86 mg/dl} and PTH 6.0 pmoi!L
D. Zona glomerulosa
(57 pg/ml}. He states that his mother is
E. Zona reticularis •
currently being investigated for hypercalcaemia,
which was picked up on a routine blood test.
What is the most likely underlying cause of his 18.40. A mother seeks advice for her .7 year old
blood results? daughter who has started developi~g pubic hair

downloaded from www.medicalbr.com


ENDOCRINOLOGY • 191

and body odour, and her voice has become A. Bilateral lesions
deeper. A diagnosis of congenital adrenal B. Hounsfield units <I 0 HU
hyperplasia (CAH) is being considered. C. Retention of contrast
What is the commonest enzyme deficiency D. Size < 4 ern
in CAH? E. Smooth surface
A. II ~-hydroxysteroid dehydrogenase
B. ~?a-hydroxylase 18.45. A 24 year old man is admitted to the
c. 17~-hydroxysteroid dehydrogenase emergency department having collapsed at the
D. IS-hydroxylase gym. He describes symptoms of headache,
E. 21-hydroxylase feeling flushed with associated palpitations and
sweating. He was observed by his friends to
18.41. A 63 year old man presents to his family become pale before he collapsed. He is
physician with a 1-month history of weight gain persistently hypertensive and a 24-hour urine
and difficulty climbing stairs. On clinical collection shows elevated metadrenalines
examination he is found to have a blood (metanephrines). First-line treatment for this
pressure of 182/85 mmHg, abdominal striae condition should be with which of the
and bruising on his arms. An overnight following?
dexamethasone test reveals a morning serum A. Bisoprolol
cortisol level of 153 nmoi/L (5.55 f.!g/dl). Which B. Dexamethasone
of the following would be an appropriate next C. Fludrocortisone
investigation? D. Ketoconazole
A. 24-hour urine free cortisol E. Pheroxybenzamine
B. Adrenal vein sampling
C. Bilateral inferior petrosal sinus sampling 18.46. A 28 year old man is referred having
D. CT adrenals been found, on home blood pressure
E. High-dose dexamethasone suppression monitoring, to have hypertension. His serum
test potassium at diagnosis was 2.9 mmoi/L. He
has been commenced on antihypertensive
18.42. A 56 year old man with recently therapy and is referred in for investigation of
diagnosed lung cancer has noticed weight gain, mineralocorticoid excess. Which of the
easy bruising of his skin, increased thirst and following antihypertensive therapies may .
difficulty climbing stairs. Which type of lung interfere with these investigations by increasing.·
cancer is associated with this endocrinological plasma renin concentrations?
picture? A. Amlodipine
A. Adenocarcinoma B. Bendroflumethiazide
B. Carcinoid tumour C. Bisoprolol
C. Large cell carcinoma D. Diltiazem
D. Mesothelioma E. Doxazosin
E. Squamous cell carcinoma
18.47. Glucose-stimulated insulin secretion by
18.43. Hypokalaemia associated with Cushing's the pancreas is augmented by which of the
syndrome is due to which underlying following?
mechanism? A. Dipeptidyl peptidase-4
A. Activation of mineralocorticoid receptors B. Glucagon-like peptide- I
B. Adipocyte proliferation C. Insulin-like growth factor-1
C. Increased glycogen synthesis D. Leptin
D. Increased protein breakdown E. Somatostatin
E. Insulin resistance /
18.48. An 18 year old woman with no past
18.44. An incidental finding of an adrenal mass medical history and on no regular medications
is discovered when a 72 year woman has a CT is admitted to hospital following a collapse at
scan. Which of the following parameters is home. She describes prodromal symptoms of
associated with an increased likelihood of palpitations, weakness and diplopia. Plasma
malignancy? glucose concentration is measured at

downloaded from www.medicalbr.com


1 92 • ENDOCRINOLOGY

2.3 mmoi/L (41 mg/dl). Plasma samples are A. Adenohypophysis


taken on admission, showing C-peptide B. Diaphragrna sellae
<0.66 nmoi/L (<2 ng/ml) and insulin C. Infundibulum
104 pmoi/L (151-liU/mL). What is the most likely D. Neurohypophysis
diagnosis? E. Sella turcica
A. Alcohol
B. Critical illness 18.52. A 62 year old man attends the diabetes
C. Exogenous insulin clinic and is noted to have large hands .and
D. lnsulinoma prognathism. He complains of headaches of
E. Sulphonylurea ingestion increasing frequency and severity over the
preceding 6 months since his last visit to the
18.49. A 56 year old man presents with a clinic. Which endocrine investigation should be
5-year history of chronic right-sided lower performed to investigate the cause of his
abdominal pain and altered bowel habit. More symptoms?
recently he has developed symptoms of A. 24-hour urinary steroid profile
increased shortness of breath, diarrhoea and B. Dexamethasone suppression test
intermittent flushing. A CT scan shows a C. Insulin tolerance test
calcified mass in the mesentery with a D. Oral glucose tolerance test
surrounding desmoplastic response and E. Saline suppression test
multiple hepatic metastases. Which of the
following would be the next initial investigation? 18.53. A 42 year old woman is referred with
g~lactorrhoea. Her periods have become
A. 5-hydroxyindoleacetic acid (5-HIAA)
irregular. She has two children who are 16 and
concentration in urine
21 years old and has been more stressed at
B. Capsule endoscopy
work than previously. She is not on any regular
C. Colonoscopy
medication. Blood tests reveal a prolactin level
D. Plasma insulin concentrations
of 12124 rniU/L (570 ng/rnl) with no evidence
E. Plasma somatostatin concentrations
of rnacroprolactin. What is the most appropriate
18.50. A 72 year old man with known pituitary treatment?
adenoma is admitted as an emergency with A. Monitor prolactin levels every year .I
sudden-onset headache, and visual B. Reassure and discharge j
disturbance. He is found to have bilateral C. Refer for pituitary surgery
ptosis. MRI imaging shows expansion of the D. Start on cabergoline 0.25 rng twice
tumour size secondary to haemorrhage, with weekly
invasion of the cavernous sinus. Which of the E. Start on carbirnazole 40 rng once daily
following sets of cranial nerves (CNs) are found
in the cavernous sinus? 18.54. A 53 year old man who is currently under
A. CN II and Ill investigation for bitemporal quandrantanopia
B. CN Ill and IV presents to the emergency department with
C. CN Ill, IV, V and VI sudden acute severe headache. What is the
D. CN IX, X, and XI most likely cause of his headache?
E. CN X, XI and XII A. Cerebrospinal fluid leak
B. Meningitis
18.51. A 17 year old who was diagnosed with C. Migraine
craniopharyngioma at the age of 2 years is D. Pituitary apoplexy
reviewed in clinic. His parents are concerned E. Subarachnoid, haemorrhage
about a recent dramatic increase in his weight
as well as polydypsia and polyuria. He is sent 18.55. A 33 year old man undergoes aCT scan
for an MRI pituitary to assess any interval for recurrent headaches, which reveals a
change in tumour size. Craniopharyngiomas 21 x 15 rnrn pituitary adenoma. On further
arise from remnants of Rathke's pouch. In questioning he has b.een feeling more tired over
embryological development of the pituitary recent months and feeling low in his mood. He
gland, which structure does Rathke's pouch is tested for pituitary dysfunction and visual field
develop into? disturbance. The secretion of which ho~mone is

downloaded from www.medicalbr.com


ENDOCRINOLOGY • 193

often the first to be affected due to mass effect She attends for a water deprivation test. Which
by a pituitary macroadenoma? of the following confirms a diagnosis of
diabetes insipidus?
A. ACTH
B. FSH A. 24-hour urine volume of 3 L
C. GH B. Plasma osmolality < 280 mOsm/kg at the
D. LH start of the test
E. TSH C. Plasma osmolality > 300 mOsm/kg and urine
osmolality < 600 mOsm/kg
18.56. A 55 year old woman is found to have a D. Plasma sodium concentration 145 mmoi!L
significant pituitary mass on MRI scanning. High E. Reduction in body weight of 1% over the
circulating levels of which hormone would direct test period
treatment for a pituitary macroadenoma down a
primarily medical, rather than surgical, route? 18.58. A 32 year old man is referred having
A.ACTH been found to have hypercalcaemia and a high
PTH level. He has recently been investigated
B. GH
C. LH for episodes of sweating, lightheadedness and
D. Prolactin confusion, which were helped by eating sugary
E. TSH foods. Which of the following is associated with
MEN 1?
18.57. 52 year old woman is referred to the A. Acromegaly
clinic with a 3-month history of polyuria. She B. Cerebellar haemangioblastoma
additionally complains of increased thirst, C. Marfinoid habitus
drinking up to 5 L per day. Fasting plasma D. Medullary thyroid carcinoma
glucose is normal at 4.2 mmoi/L (76 mg/dl). E. Phaeochromocytoma

Answers
~~
~I
18.1. Answer: D. 18.2. Answer: B.
Primary hypoadrenalism results in increased
synthesis and secretion of adrenocorticotrophic
High concentrations of insulin
(hyperinsulinaemia) exert proliferative effects
I
hormone (ACTH) from the pituitary gland. Due through the insulin-like growth factor-1 (IGF-1) I·
to the co-secretion of melanocyte-stimulating receptors, stimulating epidermal keratinocyte
hormone as part of the larger prohormone and dermal fibroblast proliferation in
(pro-opiomelanocortin; POMC), the axillae.
hyperpigmentation occurs, classically of the
palmar creases. Primary adrenal failure 18.3. Answer: A.
(Addison's disease) is usually autoimmune in Hyperprolactinaemia (often due to a
aetiology and this can be confirmed with the microprolactinoma) is a common cause of
detection of anti-adrenal autoantibodies. If secondary amenorrhoea in this age group.
diagnosed, potential coexisting autoimmune Prolactin is synthesised by lactotrophs in the
conditions should be looked for. Alternative anterior pituitary gland. Synthesis and release
causes of Addison's disease include of prolactin is under the tonic inhibition of
tuberculous adrenalitis, which should be dopamine, which is released from the
strongly considered in endemic areas and hypothalamus and passes down capillaries
when autoantibodies are negative. Abdominal surrounding the pituitary stalk to the anterior
imaging in such cases may reveal calcification pituitary.
of the adrenal glands. Secondary
hypoadrenalism due to pituitary pathology, with 18.4. Answer: A.
resultant low ACTH levels, is associated with Following overnight water deprivation, you
skin pallor. The dexamethasone suppression would expect urine to be.more concentrated,
test forms part of the workup for suspected with an osmolality of >600 mOsm/kg,
Cushing's syndrome. Imaging should not be particularly given that the plasma sodium level
done until the biochemical diagnosis is made. is at the upper end of the normal range and

downloaded from www.medicalbr.com


1 94 • ENDOCRINOLOGY

plasma osmolality is >300 mOsm/kg. This deafness). Thyroid peroxidase catalyses the
therefore confirms diabetes insipidus. The conversion of iodide ions into organic iodine
concentrating ability of the kidney is improved and couples it with tyrosine to form MIT and
following the administration of desmopressin DIT. This later step is inhibited by thionamides
(DDAVP), providing evidence of a central cranial such as carbimazole. DIT and MIT combine
cause. In nephrogenic diabetes insipidus, urine forming T4 and T3 . The organification of iodide
osmolality would show little improvement and coupling of iodinated tyrosine molecules
following administration of DDAVP. occurs on the surface of thyroglobulin. This is
subsequently cleaved, releasing thyroid
18.5. Answer: A. hormone. Uncoupled iodinated tyrosine can be
Arginine vasopressin acts via V2 receptors in dehalogenated, allowing recycling of the iodine.
the renal collecting ducts to cause the insertion The majority of T4 circulates bound to
of aquaporin-2 that allows increased water thyroxine-binding globulin (TBG).
permeability across the collecting ducts with
subsequent reabsorption of water, leading to 18.7. Answer: D.
the production of more concentrated urine. Through a classical negative feedback loop,
TSH secretion will increase with a reduction in
18.6. Answer: D. circulating free T4 or Ts. TSH synthesis follows
Iodide is actively transported into follicular cells a circadian rhythm with a peak at 0100 hrs
by a sodium/iodide transporter (Fig. 18.6). and nadir at 1100 hrs. Free T4 circulates at
Pendrin is found at the apical membrane, concentrations around three times that of free
where it transports iodide into colloid. A defect T3 , although Ts is a more potent activator of
in this transporter underlies Pend red's thyroid hormone receptors. Both play a role in
syndrome (congenital hypothyroidism and feeding back to the hypothalamus. Small

Colloid DIT DIT


MIT~--@-+-MIT-$- T3
T4

Target tissues

Blood

Negative
feedback

Fig. 18.6 Structure and function of the thyroid gland.


(1) Thyroglobulin (fg) is synthesised and secreted into the colloid of the follicle. (2) Inorganic iodide (11 is actively transported into the
follicular cell ('trapping'). (3) Iodide is transported on to the colloidal surface by a transporter (pendrin, defective in Pendred's syndrome)
and 'organified' by the thyroid peroxidase enzyme, which incorporates It into the amino acid tyrosine on the surface ofTg to form
monoiodotyrosine (Mil) and diiodotyrosine (011). (4) Iodinated tyrosines couple to form T3 and T4• (5) Tg· is endocytosed. (6) Tg is cleaved
by proteolysis to free the iodinated tyrosine and thyroid hormones. (7) Iodinated tyrosine is dehalogenated to recycle the.lodide. (8) T4 is
converted to T3 by 5'-monodeiodinase.

downloaded from www.medicalbr.com


ENDOCRINOLOGY • 195

changes in an individual's free T, will result in 18.11. Answer: A.


altered TSH secretion. Hypothyroidism can result in impaired left
ventricular contractility reducing cardiac output.
18.8. Answer: D. Reduced circulating thyroid hormone levels can
This woman has subclinical hypothyroidism. additionally cause reduced ventricular diastolic
There is no benefit in checking thyroglobulin relaxation, increasing diastolic pressure with
antibodies as it does not change her diagnosis subsequent increase in vascular smooth
or management. With positive thyroid muscle contractility and peripheral vascular.
peroxidase antibodies she has a higher risk of resistance. Through alteration in cholesterol
progression to overt thyroid failure. A trial of synthesis and metabolism, hypothyroidism
thyroxine replacement would be advocated if results in hypercholesterolaemia.
her TSH was >10 miU/L or she developed
symptoms suggestive of progression to clinical 18.12. Answer: B.
disease. Reviewing her biochemistry and Exogenous thyroxine and transient thyroiditis
symptomatology is therefore necessary. will result in low uptake of 99 mtechnetium on
scanning. Iodine deficiency can cause goitre
18.9. Answer: A. and raised TSH, but thyrotoxicosis would
This picture is in keeping with post-partum only potentially occur following iodine
thyroiditis. This typically presents 4-8 months supplementation. Although present in 80-95%
post-partum. Thyroid scintigraphy will of Graves' disease, TRAbs are not universal in
show negligible uptake and can be helpful in all cases, necessitating additional investigation
differentiating against Graves' disease. to confirm the diagnosis. A thyroid scintigraphy
Antithyroid drugs are ineffective. scan reveals uniform increased uptake in the
Symptomatic management with a setting of Graves' disease.
non-selective ~-adrenoceptor antagonist
(~-blocker) is the most appropriate treatment at 18.13. Answer: D.

--
this stage. She is likely to develop This is the classic image of a toxic adenoma
hypothyroidism, and close monitoring of her with increased uptake in the adenoma and
thyroid function to allow identification and early suppressed uptake in the remaining gland. The
treatment is advocated. Oral selenium is used presence of multiple nodules with high uptake /
for the treatment of mild to moderate of 99 mtechnetium would be consistent with toxic, 1
dysthyroid eye disease and is not appropriate multinodular goitre. 'Cold nodules' on · ~'
II
here. scintigraphy have a much greater likelihood of

18.1 o. Answer: E.
malignancy. Thyroglossal duct cysts are
typically located in the midline and are cold,
I
Exopthalmus but not lid retraction is specific whilst thyroiditis displays widespread reduced
to Graves' disease. Lid retraction is a uptake of tracer.
consequence of adrenergic stimulation
of the levator palpebrae muscles and may be 18.14. Answer: E.
seen in any form of thyrotoxicosis. Radioactive iodine is administered orally as a
Thyrotoxicosis can result in an increase capsule or liquid. The doses used for the
in sex hormone-binding globulin (SHBG), treatment of Graves' disease are much smaller
altering the ratio of free testosterone to than for the follow-up management of thyroid
oestradiol and, as such, result in cancer at 400-600 M Bq (approximately
gynaecomastia. Females have a greater 10-15 mCi). There is a significant risk of
susceptibility to autoimmune disease in general, hypothyroidism when using 131 1for the
including Graves' disease. Smoking is a treatment of Graves' disease; this is
significant risk factor for the development of approximately 40% after 1 year and 80% after
eye disease. The presence of a goitre in the 15 years. Failure to adjust energy consumption
setting of clinical and biochemical to the reduced metabolic rate associated with
hyperthydroidism is common with Graves' correction of hyperthyroidism may precipitate
disease but not universal or diagnostic. The weight gain. With a potential adverse effect on
presence of TRAbs or a classical picture of developing sperm, male individuals are advised
diffuse increased uptake on a to refrain from conceiving for a minimum of 6
scintigraphy scan are diagnostic. months post-treatment. There are reports that

downloaded from www.medicalbr.com


196 • ENDOCRINOLOGY

radioactive iodine can exacerbate active TSH both at the lower end of the reference
Graves' ophthalmopathy and so it is best range. Thyroiditis typically causes an initial
avoided in this situation if alternative treatment thyrotoxic phase followed on by a period of
options carry less risk. High-dose hypothyroidism, which may resolve or persist.
glucocorticoids can be used to reduce the risk Non-thyroidal illness results in reduced
of potentiating eye disease. peripheral conversion of T4 to T3 with reduced
secretion of TSH. The differential diagnosis for
18.15. Answer: D. this pattern would include a TSH-secreting
Simple diffuse goitre is a benign condition with pituitary tumour or thyroid hormone resistance,
hypertrophy of thyroid tissue. Thyroid function but in this scenario it is most likely that the
is normal and it will therefore not shrink patient has not been reliably taking
significantly with radioactive iodine treatment. levothyroxine for several weeks (causing the
Surgery is a better option if there is concern TSH to rise) and then in the few days prior to
about cosmetic appearance. It may enlarge in the blood test has taken an increased dose of
response to alterations in circulating oestrogens levothyroxine (resulting in the borderline
such as during pregnancy. The goitre usually elevated free T4).
regresses over time but may develop into a
multinodular goitre with autonomous function. 18.19. Answer: G.
In areas of endemic iodine deficiency, iodine In subacute thyroiditis, inflammation results in
supplementation may cause some regression of the release of colloid and stored thyroid
the goitre. hormone, resulting in thyrotoxicosis. Damage to
follicular cells impairs retention of iodine,
18.16. Answer: E. resulting in subsequent hypothyroidism and
Intestinal absorption of levothyroxine is impaired produces the classic picture of a 'cold' image
by co-ingestion of iron, colestyramine and on scintigraphy. Acute thyroiditis usually results
calcium supplements, but enhanced by vitamin from bacterial infection such as Staphylococcus
C. Increased clearance occurs with a variety of aureus and Streptococcus haemolyticus, whilst
medications, including antiepileptic medications subacute thyroiditis often follows a viral illness.
and rifampicin. Clearance is reduced with Management of hypothyroidism resulting from
increasing age, potentially necessitating smaller thyroiditis may involve at least temporary i
doses for replacement. replacement with levothyroxine and clos1
monitoring of thyroid function tests. After 4
18.17. Answer: E. months of TSH being within the reference
After commencement of levothyroxine for range, reduction in levothyroxine dose to 50 ~g
hypothyroidism, symptoms of dry skin and hair may be tried with further TSH monitoring after
can take 3-6 months to improve. Reduction in 6 weeks. If TSH remains within the normal
periorbital oedema occurs more rapidly. The range, a trial of levothyroxine can be attempted,
dose of levothyroxine should be adjusted to with repeat thyroid function testing after a
keep TSH within the reference range, with further 6 weeks.
serum T4 in the upper reference range.
Treatment with T3 is controversial, but may be 18.20. Answer: E.
considered in selected cases. The half-life of These results are in keeping with non-thyroidal
levothyroxine is around 7 days and it therefore illness or 'sick euthyroidism'. This occurs due
takes around 6 weeks following to decreased peripheral conversion of T4 to T3
commencement of levothyroxine to see as well as altered circulating levels and binding
resolution of thyroid function tests. There is no of thyroid hormone to thyroxine-binding
indication to check the patient's TRAbs. globulin, with resultant altered feedback on the
Malabsorption can result in under-treatment of hypothalamic-pituitary-thyroid axis. During
hypothyroidism, although there is nothing in this recovery from the systemic illness, the TSH
clinical scenario to suggest coeliac disease. may increase to levels associated with
hypothyroidism. Over time these will, however,
18.18. Answer: E. normalise; hence, vnless there is clinical
Central hypothyroidism results in both low TSH evidence of concomitant thyroidal disease,
and T4 . Amiodarone treatment usually causes repeated measurements and monitoring is
a mild elevation in free T4 , with free T3 and advised.

downloaded from www.medicalbr.com


ENDOCRINOLOGY • 197

18.21. Answer: B. shown equivalent efficacy for treatment with


Given the history and pattern of thyroid function 11 00 MBq and 3700 MBq (approximately 30
tests, this is likely to be amiodarone-induced and 100 rnCi, respectively) radioactive iodine
thyrotoxicosis. Arniodarone has a structure post-thyroidectomy for relatively low-risk thyroid
analogous to that of thyroxine. Standard daily carcinomas, but with extensive disease the
dosing of 200 rng daily contains around 600 higher dose should be used. Radioactive iodine
times the recommended daily requirement of should not be given unless TSH is elevated to
iodine. Amiodarone additionally has a direct >20rniU/L. This can either be achieved through
cytotoxic effect, inhibiting the conversion of discontinuation of levothyroxine or through
thyroxine (T4) to thyronine (T3). For these administration of recombinant TSH.
reasons, thyroxine production from the thyroid Thyroglobulin can be used as a tumour marker
gland is suppressed, resulting in globally following completion thyroidectomy and
reduced uptake on a scintigraphy scan. provides a sensitive marker of recurrence.
Ultrasound is a more sensitive imaging modality
18.22. Answer: C. for local recurrence than CT scanning.
It can be difficult to distinguish between type I Medullary thyroid cancer can be associated
and type II arniodarone-induced thyrotoxicosis. with multiple endocrine neoplasia (MEN) 2 (also
In type I, excess iodine induces increased known as MEN 2a) but there is no known
thyroid hormone synthesis and is usually genetic predisposition to date for the
responsive to antithyroid medication. In type II, development of well-differentiated thyroid
the picture is more of a thyroiditis due to cancer.
cytotoxic effect of arniodarone and is usually
responsive to glucocorticoid treatment. The 18.25. 'Answer: D.
excess iodine provided by amiodarone renders Regular menses imply, but do not confirm,
it unresponsive to treatment with radioactive ovulation. During a 28-day cycle, ovulation can
iodine. Arniodarone has a long half-life of 50-60 be predicted by a surge in LH on day 14 and
days and interim treatment is therefore needed, by measuring the responding increase in
even if amiodarone is withdrawn, as effects progesterone on day 21 (Fig. 18.25). Timings
may persist for 6-9 months. have to be adjusted depending on the length of
the woman's menstrual cycle. Alternatively,
18.23. Answer: A. ultrasound scanning can be used to track the
Anaplastic thyroid carcinoma is rapidly growth and development of follicles, thus
progressive with a median survival of only 7 predicting ovulation.
months and usually presents in those over 60
years of age. It may cause hoarseness due to 18.26. Answer: E.
recurrent laryngeal nerve compression and Functional hypothalamic amenorrhoea is
stridor due to tracheal compression. There is characterised by abnormal hypothalamic GnRH
no effective treatment, although surgery and secretion with resultant reduction in pulsation of
radiotherapy may be considered in some gonadotrophins. FSH levels are often within the
individuals. Follicular and papillary carcinoma
are much more indolent, with a 10-year survival
of around 95% and the majority are cured with Follicul..,-plln;a Ov•ttk>n LuteaJphase

~
surgery, with or without radio-iodine therapy.
These usually present as a single focus with or
without nodal involvement. Distant metastases,
greater age at presentation, nnale sex and ~::~ J\:.'. C\
certain histological subtypes have a worse
prognosis. The differential diagnosis in this case ~
0 Menses 7 14 21
Daysa!lerslarlollastmenslrualperiod
21!
'
is lymphoma.
Oestradiol OestradioiP~!~ed~e P~~j.:;.l'/
18.24. Answer: E.
With more extensive disease, levothyroxine
~ J., €) ~ &($
replacement is provided with an aim of
Fig. 18.25 Female reproductive physiology· and the normal
suppressing TSH, preventin@ stimulation of menstrual cycle. (FSH = follicle-stimulating hormone; LH =
any potential residual tissue. Studies have luteinising hormone.)

downloaded from www.medicalbr.com


198 • ENDOCRINOLOGY

normal range and can result in a high testicular volumes are not reduced to the
FSH-to-LH ratio. Functional hypothalamic extent seen here. Testicular trauma would be
amenorrhoea can be caused by eating associated with hypergonadotrophic
disorders, mental or physical stress or hypogonadism. Klinefelter's syndrome due to
over-exercising. Circulating levels of the 'satiety' XX'( karyotype results in small under-developed
hormone leptin have been shown to be testes with resultant elevation in
reduced in hypothalamic amenorrhoea, which gonadotrophins.
may impact on the production of LH.
Gonadotrophin-releasing hormone insensitivity 18.30. Answer: E.
is a rare autosomal recessive condition A variety of theories exist as to the aetiology of
that would present in a similar manner hyperandrogenism associated with PCOS.
but would be detected on genetic testing. Disordered gonadotrophin secretion has been ·
Hyperprolactinaemia accounts for around 1 .9% observed with a higher ratio of LH to FSH.
of hypogonadotrophic hypogonadism, but Androgens are synthesised by theca cells in the
again would be detected through plasma ovary under the influence of LH, whilst FSH
measurements. stimulates aromatisation of androgens by
granulosa cells. Challenging the pituitary gland
18.27. Answer: B. with GnRH has shown a preponderance for the
High gonadotrophins in the context of low production of LH but no evidence exists for
oestradiol and secondary amenorrhoea disruption in hypothalamic function of GnRH.
implicate premature ovarian failure. The most Hyperinsulinaemia leads to a reduction in
likely causes are acquired injury to the ovaries SHBG and resultant increase in metabolically
(e.g. previous chemotherapy), autoimmune or active free androgens. Mutations in
genetic disorders such as Turner's syndrome. 3~-hydroxysteroid dehydrogenase cause a
Karyotype and genetic screening is therefore virilising form of congenital adrenal hyperplasia.
necessary. MRI pituitary would be the
investigation of choice for hypogonadotrophic 18.31. Answer: D.
hypogonadism. The karyotype in this scenario is consistent with
Turner's syndrome and usually presents with
18.28. Answer: C. short stature and amenorrhoea. There is ~o
Constitutional delay is observed more frequently increased risk of ovarian cancer. Excess 1
in boys. Reduced growth velocity is observed androgen is not usually a feature. '
as early as 3-6 months. Due to delay in age of Cardiovascular malformations (especially aortic
pubertal growth spurt, height can drift further root dilatation) may go undiagnosed until later
from the growth chart at this time but catches in life and present a high risk of morbidity and
up once puberty is achieved. Bone age is mortality, especially during pregnancy with
consistent with age appropriate for height increased circulatory volume, Only up to 5% of
rather than chronological age. In childhood individuals with Turner's syndrome become
when long bones are still developing, the ratio pregnant spontaneously, with the majority
of upper-to-lower body is > 1, which is then requiring intervention such as egg donation.
reversed by adulthood. Constitutional delay Turner's syndrome is not associated with
has no effect on final height and can therefore mental retardation but is associated with
be predicted based on mid-parental heights. degrees of learning disability later in life. As it is
not heritable, Turner's syndrome cannot be
18.29. Answer: A. passed on to future generations with
Hypogonadotrophic hypogonadism in the unassisted pregnar:~cies.
context of small testes suggests absent,
incomplete or partial pubertal development 18.32. Answer: C.
due to Kallmann's syndrome or idiopathic Delayed puberty in.the context of
hypogonadotrophic hypogonadism. Individuals hypergonadotrophb hypogonadism in males is
with Kallmann's syndrome may have either usually due to Klinefelter's syndrome with
anosmia or hyposmia. This may, however, not karyotype 47XXY, and testing for this should
be obvious to the individual until more formally therefore be part of the next -line investigation.
tested. A similar biochemical pattern is seen in The differential diagnosis is acquired gonadal
men in the context of central obesity, but damage due to chemotherapy/radiotherapy,

downloaded from www.medicalbr.com


ENDOCRINOLOGY • 199

trauma or surgery, which should be apparent 18.36. Answer: C.


on taking a full history. Autoimmune gonadal With mild hypercalcaemia and
failure or post-infection orchitis due to mumps hypermagnesaemia along with a family history
or tuberculosis are differential diagnoses. of the same, the most likely underlying cause is
Alternative developmental or congenital gonadal familial hypocalciuric hypercalcaemia (FHH).
disorders include enzyme deficiency in the This may go undiagnosed until detected
biosynthesis of sex steroids or anorchidisrn/ incidentally when blood tests are taken for an
cryptorchidism. alternative reason. Diagnosis would be
confirmed by a urine calcium-to-creatinine ratio
18.33. Answer: C. of <0.01 and urine calcium <5 mmol/24 hrs
Klinefelter's syndrome results from genotype (<200 mg/24 hrs). The condition is caused
'IX{. This leads to hyalinisation and fibrosis by an inactivating mutation in the
of the seminiferous tubules. LH resistance calcium-sensing receptor gene. MEN
can occur rarely due to abnormalities in syndromes are associated with
LH receptor. Disordered migration of hyperparathyroidism, which is also inherited in
GnRH-producing neurons occurs in Kallrnann's an autosomal dominant manner. However,
syndrome, whilst defect in the steroidogenesis hypermagnesaemia would not be expected in
pathway underlies congenital adrenal this situation or in hyperparathyroidism
hyperplasia. TGF-~ has both a stimulatory and secondary to a parathyroid adenoma.
inhibitory effect on Leydig cell steroidogenesis. Dehydration can cause elevated serum calcium,
but this man's urea is not elevated, so he does
18.34. Answer: C. not appear dehydrated from a biochemical
Finasteride is used for the treatment of benign perspective.
prostatic hypertrophy and inhibits the conversion
of testosterone to its metabolically active form, 18.37. Answer: D.
dihydrotestosterone. This results in a reduced This woman has all the biochemical features of
testosterone-to-oestrogen ratio, resulting in primary hyperparathyroidism. The indications
gynaecornastia. Tarnoxifen and clornifene can be for consideration of surgery include:
used in the treatment of gynaecornastia by 0.25 mmoi/L (1 mg/dL) or above the normal
reducing oestradiol synthesis and function, reference range for serum calcium, urine
respectively, restoring a higher testosterone-to- calcium excretion > 10 mmol/24 hrs I
oestrogen ratio. Potassium-sparing, but not
thiazide, diuretics can cause gynaecornastia.
(>400 mg/24 hrs), a 30% reduction in
creatinine clearance, bone mineral density
~'
I
Through aromatisation, androgen replacement T-score below -2.5 at any site and age I
can lead to gynaecomastia but this should not younger than 50 years. Prior to surgery she
occur with the metabolically active form, should have imaging (by ultrasound and/or
dihydrotestosterone. sestamibi scanning) to try to localise a
parathyroid adenoma. Given her young age,
18.35. Answer: C. consideration should be given to screening for
This case describes secondary genetic causes of hyperparathyroidism. There is
hyperparathryoidism secondary to chronic renal nothing to suggest vitamin D deficiency (i.e.
failure. A similar biochemical picture is seen hypocalcaemia) and thiazide diuretics may
with vitamin D deficiency. Parathyroid hormone increase serum calcium concentrations. Loop
aims to maintain calcium homeostasis by diuretics may occasionally be used in the
reabsorbing calcium from bone through management of severe hypercalcaemia, but
altered activity of osteoblasts and osteoclasts. only under specialist supervision.
In the kidney, PTH activates the enzyme
1a-hydroxylase in the proximal tubules, which 18.38. Answer: E.
in turn activates 25-hydroxyvitamin D to These results are consistent with secondary
1,25-dihydroxyvitamin D. It is the latter that hyperparathyroidism as a result of vitamin D
enhances absorption of calcium and phosphate deficiency. With normal renal function and given
from the gut. A complex relationship exists her symptoms, the unde~lying diagnosis is
between feedback of magnesium on PTH osteomalacia. Parathyroid adenoma is
secretion, but PTH results in reabsorption of associated with primary hyperparathyroidism
magnesium from the distal <Gonvoluted tubule. and thus hypercalcaemia. All of the other

downloaded from www.medicalbr.com


-,.
I

200 • ENDOCRINOLOGY

answers cause hypoparathyroidism and result activation. Cortisol is a potent mineralocorticoid,


in hypocalcaemia. but is usually prevented from activating
mineralocorticoid receptors because it is
18.39. Answer: D. metabolised to inactive cortisone by the
The outermost layer of the adrenal cortex, enzyme ~ ~ ~-hydroxysteroid dehydrogenase
zona glomerulsoa, is the site of aldosterone type 2 (~ ~ ~-HSD2). In Cushing's syndrome, the
synthesis, whilst glucocorticoids are produced presence of high levels of cortisol overwhelms
in the middle zona fasciculata and androgens in the ~ ~ ~-HSD2 system.
the innermost zona reticularis. Chromaffin cells
in the adrenal medulla are the source of 18.44. Answer: C.
catecholamines. This clinical picture could Greatest risk of malignancy is seen in unilateral
result from disruption of the renin-angiotension lesions with an irregular surface, of size >4 cni,
system from medications such as with a lipid-poor consistency (Hounsfield
angiotensin-converting enzyme inhibitors or units > ~ 0 HU) and retention of contrast on
angiotensin receptor blockers or, more rarely, scanning.
due to defective synthesis of angiotensin due to
genetic mutations in the CYP11 82 gene. 18.45. Answer: E.
This presentation is in keeping with a
18.40. Answer: E. phaeochromocytoma. At such a young age
Females with milder forms of CAH develop genetic screening should be performed to
precocious puberty, cliteromega\y, accelerated detect any underlying inherited disorders.
growth or can present with o\igomenorrhoea, .Treatment is initially with an a-adrenoceptor
hirsutism and infertility later in life. Up to 95% of antagonist (a-blocker) such as
cases are due to 2 ~-hydroxylase deficiency. phenoxybenzamine. Initial prescription of
This results in reduced cortisol synthesis with ~-blocker therapy may cause a hypertensive
resultant increase in intermediate compounds crisis. Ketoconazole may used in the treatment
including ~ 7-hydroxyprogesterone and an of Cushing's syndrome. Dexamethasone and
increase in androgen synthesis. Severe forms fludrocortisone are synthetic glucocorticoids
can be detected from birth due to ambiguous and mineralocorticoids, respectively, and have
genitalia and can result in failure to thrive and no role here. 1
vomiting due to salt wasting.
18.46. Answer: B.
18.41. Answer: A. Initially considered a rarity, primary
The results of the overnight dexamethasone hyperaldosteronism is thought to underlie
suppression test suggest a possible diagnosis hypertension in 5- ~ 5% of cases. Diagnosis
of Cushing's syndrome. However, it is is suggested by finding an elevated
necessary to perform a second test to confirm aldosterone: renin ratio. Thiazide and loop
a diagnosis of Cushing's- either a 24-hour diuretics increase renin levels, whilst ~-blockers
urine collection for urine free cortisol or a inhibit renin secretion. Discontinuation of these
late-night salivary cortisol. Once Cushing's agents for at least 2 weeks is therefore
syndrome is confirmed, further investigations necessary before measuring plasma levels.
are required to determine the underlying cause. Substitution can be made with calcium channel
antagonists and a-blockers to control blood
18.42. Answer: B. pressure in the interim.
This case describes classic symptoms of
Cushing's syndrome, which can be associated 18.47. Answer: B.
with excess ACTH from a variety of cancers, Glucagon-like peptide-~ (GLP-1) and gastric
including small cell and carcinoid tumours of the inhibitory polypeptide are the two major incretin
lung. Other examples include islet cell tumours hormones secretE)d from the gastrointestinal
of the pancreas, medullary carcinoma of the tract which act on ~ cells within the pancreas,
thyroid and tumours of the thymus gland. increasing insulin secretion. Dipeptidyl
peptidase-4 is iniJ()\Ved in the degradation of
18.43. Answer: A. incretins, thereby reducing insulin secretion.
Hypercortisolaemia results in hypokalaemia as Leptin and somatostatin additionally have an
a result of increased mineralocorticoid receptor inhibitory role on insulin secretion.

downloaded from www.medicalbr.com


ENDOCRINOLOGY • 201

18.48. Answer: C. dysregulation and loss of thirst sensation


With high insulin concentrations and can be the result of hypothalamic
suppressed C-peptide, the likely diagnosis is involvement.
exogenous insulin use. An insulinoma would
result in elevated insulin and C-peptide 18.52. Answer: D.
concentrations, as would sulphonylurea In endocrinology, hormones are usually
ingestion. Alcohol and critical illness both result secreted in a pulsatile fashion. For the majority
in a picture of suppressed insulin and of hormones, samples taken at random times
C-peptide concentrations. can be hard to interpret. As a result, if there is
a suspicion of over-production of a hormone,
18.49. Answer: A. then tests should be performed to suppress
The history and clinical presentation are in secretion, and if there is thought to be
keeping with a neuroendocrine tumour and deficiency of a hormone, then tests should aim
subsequent metastases to the liver resulting in to enhance secretion. Growth hormone is
the classical features of carcinoid syndrome. secreted in a pulsatile manner with alterations
Measurement of 5-HIAA, the main breakdown in response to sleep, stress, exercise and
product of 5-hydroxytryptamine (5-HT, circulating glucose concentrations. Elevated
serotonin), in urine allows confirmation that this circulating glucose concentrations, as induced
is a carcinoid tumour and allows quantification through the use of an oral glucose tolerance
of its secretory activity. Care must be taken to test, should normally result in suppression of
ensure the avoidance of certain foods and growth hormone secretion. In the situation of a
medications that can result in false-positive pituitary tumour with autonomous growth
results when performing 24-hour urine hormone secretion, an oral glucose tolerance
collections for 5-HIAA. Although these tumours test will result in minimal or no suppression.
may be detected using imaging modalities such Dexamethasone suppression test and 24-hour
as CT, MRI or direct visualisation using urinary steroid profile are used in the diagnosis
colonoscopy or small bowel capsule of Cushing's syndrome. A saline suppression
endoscopy, this provides no assessment of test is used to diagnose hyperaldosteronism
type or hormonal activity of the tumour. There and an insulin tolerance test is used for the
is nothing in the history to suggest excess diagnosis of hypoadrenalism.
secretion of insulin or somatostatin to
necessitate their quantification. 18.53. Answer: D.
Prolactin levels over 5000 miU/L are usually
18.50. Answer: C. indicative of a macroadenoma, which secretes
From superior to inferior, the cavernous sinus prolactin (a macroprolactinoma). This should be
contains cranial nerves Ill (oculomotor), IV confirmed with MRI imaging. Macroprolactin is
(trochlear), VI (abducens) as well as V1 a physiologically inactive form of prolactin that
(ophthalmic branch of trigeminal) and V2 is complexed with immunoglobulin G (lgG)
(maxillary branch of trigeminal). antibody and, unless accounted for on
laboratory measurement, gives a false
18.51. Answer: A. impression of clinically relevant
Rathke's pouch separates and develops into hyperprolactinaemia. Given her symptoms and
the adenohypophysis (anterior pituitary). In the likelihood of this being a macroadenoma,
contrast, the neurohypophysis (posterior she should be commenced on dopamine
pituitary) develops from the infundibulum. The agonist therapy, as described in option D.
pituitary gland sits within the saddle-shaped Given the risk of growth and compression on
sella turcica and the superior aspect is covered surrounding structures, monitoring without
by the diaphragma sellae. treatment is not advised. Carbimazole is an
Craniopharyngiomas are benign tumours antithyroid medication usj'd for the. treatment of
and may be located in the sella turcica or Graves' disease. Prolactin levels are significantly
suprasellar space. They are often cystic with higher than would be expected simply as a
solid components. They commonly present as result of stress. Surgical pecompression is
the result of pressure on adjacent structures usually only necessary if a macroprolactinoma
causing hypopituitarism and/or cranial diabetes fails to respond sufficiently to dopamine agonist
insipidus. Hyperphagia, temperature therapy.

downloaded from www.medicalbr.com


202 • ENDOCRINOLOGY

18.54. Answer: D. her symptoms. A plasma sodium concentration


Bitemporal quadrantanopia implies the above the normal range and 24-hour urine
presence of a mass lesion compressing part of volume greater than 3 L can be an indicator of
the optic chiasm, most commonly a pituitary diabetes insipidus but are not diagnostic. A
macroadenoma. Pituitary apoplexy is a rare water deprivation test should be discontinued if
endocrine emergency characterised by a body weight falls by 3% or more and if plasma
sudden-onset headache due to acute osmolality rises to >300 rnOsm/kg with
haemorrhage or infarct of a pituitary gland, continued dilute urine (osmolality <300 rnOsm/
resulting in headache, visual disturbance and kg). DDAVP (2 j.!g intramuscularly) should
hormonal dysfunction. Treatment involves subsequently be administered to assess
urgent replacement of fluid, electrolytes response and determine if the underlying
and pituitary hormones, particularly diagnosis is cranial or nephrogenic diabetes
glucocorticoids. insipidus.

18.55. Answer: C. 18.58. Answer: A.


Growth hormone (GH) secretion is usually the MEN 1 is characterised by tumours involving
first affected, classically followed by LH, FSH, the parathyroid glands (95% of cases),
TSH, ACTH and prolactin, in that order. pancreas (neuroendocrine tumours, which may
However, in practice any pattern of hormone be non-functional or functioning; gastrinoma;
deficiency can occur. insulinoma; glucogonoma; VIPorna;
somatostatinoma) (30-80% cases), lung
18.56. Answer: D. (neuroendocrine carcinoid) (5%), gastrointestinal
As prolactinomas are responsive to dopamine tract (neuroendocrine carcinoid) (2%) and
agonist therapy, prolactin levels should always anterior pituitary tumours (commonly
be measured prior to undertaking surgical prolactinomas) (30%). The underlying genetic
resection of a pituitary macroadenorna. This defect is the MEN1 gene, which is on
may avoid the need for unnecessary pituitary chromosome 11. MEN 2 is characterised by
surgery, as prolactinomas will often shrink in tumours of the parathyroid and adrenal glands
size following commencement of a dopamine along with medullary thyroid carcinoma. MEN 3
agonist. can additionally be associated with 1
mucocutaneous neuromas and Marfanoid;
18.57. Answer: C. habitus. Hemangioblastomas are a feature of
Low plasma osmolality at the start of the test von Hippei-Lindau syndrome.
may suggest primary polydipsia as the cause of

downloaded from www.medicalbr.com


E Przybyszewski, A Shand

Nutritional factors in disease

Multiple Choice Questions


19.1. A 27 year old woman presents to clinic for A. Consumption of fats should represent
the first time. She has no known medical 15-30% of total daily energy intake
history and takes no medications, but B. Energy provided per gram of fat is less than
complains of lethargy and fatigue. She weighs the energy provided per gram of protein
97 kg and has a body mass index (BMI) of C. Fats are the main source of short -chain fatty
32 kg/m 2 . She states that her weight has been acid production by colonic bacteria
climbing over the past few years, more rapidly D. Increasing consumption of saturated fats
over the past few months. should lower circulating low-density
On examination, her temperature is 36.8°C, lipoprotein (LDL) concentrations
pulse is 70beats/min, and blood pressure is E. Trans fatty acids consumption promotes
128/78 mmHg. She has no conjunctival pallor, anti-inflammatory effects via prostaglandin
sclera are anicteric, examination of her neck production
is normal, there is no lymphadenopathy, and i
no cardiac murmurs are heard. Her abdomen
is obese without ascites and there is no
19.3. A 19 year old woman presents to her
family physician complaining of pain in her foot
~'
peripheral oedema. Which of the following is
the best next step in management?
when she runs. She is a competitive runner
and currently trains 60 miles per week.
I
··C'j

A. Check serum thyroid-stimulating hormone Her menstrual periods are-typically irregular,


(TSH) and she has not had her menstrual period
B. Measure triceps skinfold thickness in 6 months.
C. Prescribe orlistat On examination, her pulse is 50 beats/min,
D. Recommend a very-low-calorie diet blood pressure is 89/68 mmHg and her
E. Refer for a bone densitometry (dual X-ray BMI is 15.7 kg/m 2 • Beta-human chorionic
absorptiometry; DXA) scan gonadotrophin (hCG) testing is negative. The
following features may all be due to her
19.2. A 43 year old man with a history of nutritional state, except one. Which one is NOT
hypertension, hyperlipidaemia and gout likely to be a consequence of her nutritional
presents to his family physician for routine status?
follow-up. The man does not smoke or drink A. Amenorrhoea
alcohol and lives with his wife. He denies chest B. Decreased basal metabolic rate
pain and dyspnoea on exertion. He has a C. Foot pain /
BMI of 34 kg/m 2 and android distribution of D. High insulin state for maximal nutrient
body fat. Serum cholesterol is 6.3 mmoi/L delivery to cells
(244 mg/dl). The family physician recommends E. Increased lethargy for .energy conservation
decreasing his consumption of high-fat foods.
Which of the following is true regarding fat 19.4. A 63 year old man with a history of
metabolism? chronic alcohol use is admitted after being

downloaded from www.medicalbr.com


204 • NUTRITIONAL FACTORS IN DISEASE

found unresponsive in a park. He required evaluation with flexible sigmoidoscopy


several stitches in for repair of a forehead demonstrates a large tumour in the descending
laceration and later developed alcohol colon.
withdrawal seizures requiring diazepam for Which of the following is true regarding the
several days on the ward. After a prolonged most likely micronutrient deficiency associated
period of delirium and inattention, the decision with his anaemia?
is made to place a nasogastric tube for enteral A. This deficiency causes impaired wound
feeding. Three days after initiating feeding, he healing through defective collagen fibrils
becomes more alert, but complains of nausea, B. Duodenal hepcidin production is suppressed
weakness and palpitations. In addition to in states of this deficiency
obtaining an electrocardiogram (ECG), which of C. Absorption of this micronutrient is enhanced.
the following is the most appropriate next step by dietary calcium
in management? D. Absorption of this micronutrient is impaired
A. Check serum electrolytes by vitamin C
B. Obtain a computed tomography (CT) scan of E. Long-term deficiency can lead to cirrhosis
his head
C. Obtain an echocardiogram 19.7. A 34 year old overweight woman with a
D. Reassure him that he is getting adequate history of mild chronic asthma presents to a
nutrition via the nasogastric tube gastroenterologist complaining of bloating and
E. Treat with calcium gluconate, insulin and diarrhoea. Over the past year she has been
dextrose attempting to lose weight through lifestyle
modification with some success. She recently
19.5. A 34 year old woman with obstructive adjusted her diet to replace added sugar with
sleep apnoea and diabetes is referred to sugar-free foods and artificial sweeteners.
general surgery for a Roux-en-Y gastric bypass Which of the following is the most likely
procedure. Her BMI is 41 kg/m 2 and she has reason for her gastrointestinal complaints?
been unsuccessful in achieving weight loss with A. Consumption of non-starch polysaccharides
multiple attempts at lifestyle modification and has increased
pharmacological therapy. Which of the following B. She is no longer producing lactase
1
statements would be most appropriate in C. Short -chain fatty acid production has
counselling this patient? increased I
A. It will likely take years to see improvement in D. Sugar alcohols are not absorbed by the gut
diabetes control E. Sugar alcohols have a high glycaemic index
B. She is at low risk for post-surgical
complications such as wound infection 19.8. A 23 year old woman from northern
C. She should wait at least 2 years before Scotland presents to her family physician
considering pregnancy complaining of chronic tiredness. She has a
D. She will likely experience amenorrhoea history of right fibula fracture in childhood, but
post~.operatively is otherwise well and takes no medications or
E. Although effective for weight loss, bariatric supplements. She is up to date with routine
surgery is not associated with reduced vaccinations and is currently working as a
mortality librarian. Examination is normal. A P-hCG test is
negative.
19.6. A 72 year old man with a history of Which of the following nutritional
hypertension and hyperlipidaemia presents to recommendations is most appropriate at this
his family physician complaining of fatigue and time?
constipation. On further questioning, he admits A. Advise her to begin vitamin E
to recently chewing ice on occasions. Physical supplementation for cardioprotection
examination reveals conjunctival pallor and B. Begin daily vitamin 8,2 supplementation to
koilonychia. reduce the incidence of neural tube defects
Initial blood tests show white cell count C. Initiate supplementation with cholecalciferol
0fi/CC) 6.2 x 109/L, haemoglobin 96 g/L, mean (vitamin D), at least during winter months
cell volume (MCV) 72 fl, platelets 263 x 10 9/L, D. Initiate supplementation with retinol now in
international normalised ratio (INR) 1.0. Further the event that she becomes pregnant

downloaded from www.medicalbr.com


NUTRITIONAL FACTORS IN DISEASE • 205

E. Recommend treatment with ascorbic acid for chest is clear to auscultation, her abdomen is
the prevention of kidney stones soft but mildly tender to palpation in the right
upper quadrant and she has no oedema.
19.9. A 58 year old woman from Angola is seen Blood tests show wee 23 X 109/L, platelets
in clinic after recently emigrating to the UK. Her 340x109/L, INR 1.5, alanine aminotransferase
husband states that she has been confused (ALT) 32 U/L, albumin 34 g/L. Extrahepatic
and at times disorientated. On further biliary dilatation is seen on abdominal
questioning, the patient states that she has ultrasound.
nausea and diarrhoea, with seven bowel Which of the following is the most likely
movements daily. On examination, she is factor leading to her mild coagulopathy?
afebrile, anicteric, without conjunctival pallor A. Hepatic synthetic dysfunction
or lymphadenopathy. Her tongue appears B. Intracerebral haemorrhage
enlarged; no cardiac murmurs are heard; and C. Medication effect
her abdomen is soft, mildly tender and D. Obstruction of the biliary tree
non-distended. A dry cracking rash is seen on E. Previously undiagnosed coeliac disease
the skin of her neck and upper extremities
bilaterally. 19.12. A 68 year old man is being discharged
Which of the following treatments is most from the hospital after experiencing palpitations.
likely to improve her symptoms? An EeG has revealed new-onset atrial
A. Ascorbic acid 250 mg orally 3 times daily fibrillation and he is started on warfarin for
B. Folate 500 J.Lg orally once daily anticoagulation and stroke prophylaxis. His
C. Nicotinamide 100 mg orally 3 times daily cardiologist explains that warfarin works by
D. Pyridoxine hydrochloride 50 mg orally 3 antagonising vitamin K and that the patient
times daily should not vary his vitamin K intake while on
E. Riboflavin 10 mg orally once daily warfarin. Which of the following foods is highest
in vitamin K and should be consumed with the
least amount of variation?
19.10. A 45 year old man with a history of

-
alcoholic cirrhosis presents to the emergency A. Egg yolk
department after being found unresponsive B. Kale
at home. On examination he is minimally C. Liver
responsive, disorientated, anicteric, cachectic D. Pork
and has restricted horizontal eye movement E. Sunflower oil II·
bilaterally. His lab results are notable for mildly
elevated transaminases and low albumin. 19.13. A 32 year old woman gives birth to a I
Treatment is initiated with parenteral baby boy weighing 4.2 kg. She defaulted from
multivitamin therapy. antenatal care and takes no medications or
Which of the following is true regarding his supplements. On examination of the child after
most clinically significant vitamin deficiency? 10 minutes, his pulse is 136 beats/min and
A. Adults have limited stores of thiamine in the he is crying vigorously with arms and legs
liver and may manifest deficiency after a held in flexion. The skin appears pink with a
short period protuberant mass on his back in the midline at
B. Thiamine deficiency commonly leads to the level of L4. Treatment with a water-soluble
coagulopathy vitamin may have prevented this birth defect
C. Hepatocytes are most vulnerable to damage through which of the following mechanisms?
as a result of thiamine deficiency A. Accepting and donating hydrogen in
D. Thiamine acts as a co-factor in folate nicotinamide adenine dinucleotide (NAD)
co-enzyme recycling B. Decarboxylation of pyru':ate to acetyl-co-
E. Thiamine is fat soluble enzyme A to initiate the Krebs cycle
C. Donating a methyl gr6up in DNA and protein
19.11. A 41 year old previously healthy woman synthesis
is brought to the hospital by her husband who D. Facilitating absorption.of calcium in the small
states she has been febrile to 39oe for the past intestine
2 days and jaundiced for the past 7 days. On E. Hydroxylation of proline and lysine in the
examination she is delirious and jaundiced. Her formation of mature collagen

downloaded from www.medicalbr.com


206 • NUTRITIONAL FACTORS IN DISEASE

19.14. A 78 year old man presents to hospital


with an acute abdomen. At laparotomy he is
found to have extensive infarction of the
superior mesenteric artery territory. It is noted
C. Oral administration of a glucose/electrolyte
solution in place of oral fluids
D. Oral administration of loperamide 2 mg 4
times daily
l
at the time of surgery that he has 90 em of E. Restriction of oral fluid intake
healthy jejunum remaining, anastomosed to
an intact colon. 19.17. An 89 year old woman who lives
He may need additional treatments in the independently in her own second-storey flat
future as a result of this. Which of the following presents to her family physician with a
treatments is he LEAST likely to need? complaint of easy bruising on her forearms and
A. Daily oral administration of a glucose/ hands. On examination it is noted she has
electrolyte solution as a substitute for free extensive ecchymoses on the upper and lower
fluids limbs as well as a petechial rash. A full blood
B. Oral loperamide count and tests of clotting parameters are all
C. Parenteral (intravenous or subcutaneous) normal and the rash and bruising disappears
fluid rapidly with oral administration of vitamin C
D. Parenteral nutrition (ascorbic acid) 250 mg 3 times daily for 7 days.
E. Restriction of oral free fluid intake Which of the following clinical features is
associated with vitamin C deficiency (scurvy)?
19.15. An 85 year old woman suffers a total A. Gingival swelling and bleeding
anterior circulation stroke that leaves her B. Impaired cognitive function
unable to swallow safely. After a few days on .c. Lanugo hairs
intravenous fluids, her family request that she is D. Nail clubbing
fed intravenously. The attending multidisciplinary E. Spider naevi
team explain their preference is for nasogastric
feeding in the first instance. Which of the 19.18. Which of the following is an essential
following statements is true of the enteral route amino acid?
of nutrition versus the parenteral route? A. Alanine
A. It improves quality of life B. Asparagine
B. It is associated with fewer costs financially C. Glutamine
C. It leads to greater preservation of hepatic D. Glycine
and pulmonary immune function E. Tryptophan
D. It leads to reduced levels of systemic
inflammatory response 19.19. An 83 year old man with a long history of
E. It reduces the pathogenicity of intestinal Alzheimer's disease and progressive cognitive
microorganisms and functional impairment is noted to have lost
interest in food and is losing weight. Thorough
19. 16. A 49 year old woman with a 10-year investigation by CT scanning and gastroscopy
history of terminal ileal Grahn's disease treated has failed to demonstrate another underlying
with mesalazine is admitted with a complex physical explanation for this. Nasogastric tube
fistulating mass involving small bowel, bladder feeding is suggested. In this situation, which
and sigmoid colon. It is not possible to fashion statement below best describes the outcomes
a primary anastomosis and a proximal small with routine use of tube feeding, as based on
bowel stoma Gejunostomy) is brought out to strong evidence?
skin. Post-operatively, the fluid losses from this A. Improved cognitive function
stoma are >2000 mUday, leading rapidly to B. Improved muscle ~trength
dehydration. C. Improved quality of life
Which of the following measures is likely to D. Prolonged surviv<;~l
lead to a worsening of fluid losses from the E. There is little eviden,ce for improved survival or
high-output stoma? cognitive outcomes with routine tube feeding
A. Administration of oral omeprazole 20 mg
once daily 19.20. The long-term carers of a 40 year old
B. Advice to take oral fluids ad libitum to man with cerebral palsy report that he is finding
replace stomal losses it increasingly difficult to chew and swallow

downloaded from www.medicalbr.com


NUTRITIONAL FACTORS IN DISEASE • 207

food. The act of feeding is very tiring for him Which of the following is NOT a recognised
and is taking up much of the day. Increasingly, risk of gastrostomy insertion or gastrostomy
there are days when he may not eat or drink at feeding?
all and there are concerns that he is losing A. Aspiration pneumonia
weight and becoming dehydrated. After B. Colonic perforation
multidisciplinary assessment it is felt he should C. Insertion site infection
be fed by gastrostomy to allow adequate food D. Laceration of the liver
and fluid to be given on a daily basis. E. Pulmonary embolus

Answers
19.1. Answer: A. 19.2 WHD recommended population
This is an obese patient being evaluated for macronutrient goals
the first time with a history of weight gain
Target limits for
that has accelerated recently, along with average population
fatigue. This history suggests an underlying Nutrient (% of total energy intakes
disorder such as hypothyroidism or Cushing's unless indicated) Lower Upper
syndrome may be related to the weight gain Total fat 15 30
(Box 19.1). All obese patients should have Saturated fatty acids 0 10
thyroid function tests performed. Very-low- Polyunsaturated fatty acids 6 10
Trans 'tatty acids 0 2
calorie diets require the supervision of an
Dietary cholesterol (mg/day) 0 300
experienced physician and dietician and can be
Total carbohydrate 55 75
considered if short-term rapid weight loss is
Free sugars 0 10
required.
Complex carbohydrate 50 70
Dietary fibre (g/day):
As non-starch polysaccharides 16 24
As total dietary fibre 27 40
19.1 Potentially reversible causes of weight gain
Protein 10 15
Endocrine factors
Hypothyroidism
Cushing's syndrome 19.3. Answer: D.
lnsulinoma This female runner's presentation is concerning
Hypothalamic tumours or injury
for under-nutrition and the female athlete triad
Drug treatments
of disordered eating, amenorrhoea and
Atypical antipsychotics (e.g. olanzapine)
Sulphonylureas, thiazolidinediones, insulin decreased bone mineral density. Her foot pain
Pizotifen may be related to osteopenia and a stress
Glucocorticoids fracture. In a state of under-nutrition, her basal
Sodium valproate
metabolic rate will be decreased to minimise
~-blockers
further weight loss. In addition, she will have a
low-insulin state in order to promote liberation
of energy stores from tissues such as the liver,
19.2. Answer: A. muscle, and adipose tissue for systemic
Fats have the highest energy density of the utilisation (Fig. 19.3).
macronutrients and should represent 15-30%
of daily energy intake compared to 55-75% for 19.4. Answer: A.
carbohydrates and I 0-15% for proteins (Box This patient with a history of chronic alcoholism
19.2). High saturated fat consumption is is at risk for malnutrition ~rid refeeding
associated with higher levels of LDL syndrome due to restoration of carbohydrate
cholesterol. Whereas omega-3 fatty acids metabolism, insulin secretion and electrolyte
promote prostaglandin production and shifts into cells. Refeeding syndrome (Box 19.4)
anti-inflammatory cascade, trans fatty may present with nausea, vomiting, weakness,
acids are associated with cardiovascular seizures, respiratory depression, and cardiac
disease. arrhythmias or arrest. Initiation of nutrition

downloaded from www.medicalbr.com


1
208 • NUTRITIONAL FACTORS IN DISEASE

intraluminally and cause diarrhoea. Since sugar


alcohols are not absorbed, they do not
increase the blood glucose concentration and
have a very low glycaemic index.
I
19.8. Answer: C.
H~~~c ( Supplementation with vitamin D, m
~~~Satiety
cholecalciferol, is recommended for this woman
~
Eating behaviour
from northern Scotland. The combination of low
dietary vitamin D intake and limited sunlight
exposure in the UK has led to the UK
Fig. 19.3 Regulation of energy balance and its link with recommendation of i 0 ~g of vitamin D daily.
reproduction. a:> indicates factors that are stimulated by eating Vitamin D supplementation should be
and induce satiety. e indicates factors that are suppressed by
considered for everyone during winter months
eating and inhibit satiety.
(October to April) and all year for certain
groups: e.g. those who may not be outside
19.4 Complications of nasogastric tube feeding much, those who tend to cover up their skin,
Tube misplacement, e.g. tracheal or bronchial placement those with darker skins.
(rarely, intracranial placement) In countries where vitamin A deficiency is
Reflux of gastric contents and pulmonary aspiration endemic, pregnant women may be advised to
Interrupted feeding or inadequate feed volumes consume foods high in vitamin A. In contrast,
Refeeding syndrome
i':l countries where vitamin A deficiency is not
endemic, vitamin A supplementation is avoided,
should be done slowly with careful monitoring as moderate doses of retinol are teratogenic.
of electrolytes, especially potassium, phosphate Increased vitamin E intake has been shown to be
and magnesium, with repletion as appropriate. associated with lower rates of coronary artery
disease; however, randomised controlled trials
19.5. Answer: C. have not demonstrated significant benefits.
Bariatric surgery is the only anti-obesity Vitamin C exces::; is associated with the
intervention that has been shown to reduce formation of renal oxalate stones. Folate i
mortality. While diabetes control usually supplementation is used to reduce the incipence
improves rapidly, especially with gastric bypass of neural tube defects in pregnancy. In this case,
procedures, micronutrient deficiencies (e.g. the woman is of child-bearing age but clearly not
iron, folate, vitamin Bd may complicate the pregnant at the time (~-hCG test is negative).
post-procedure period. As a result,
pre-menopausal women should wait at least 2 19.9. Answer: C.
years for nutritional status to stabilise before This patient from Angola presents with
becoming pregnant. dementia, diarrhoea and dermatitis consistent
with pellagra, or niacin deficiency. The enlarged
19.6. Answer: B. tongue is due to non-infective inflammation of
This patient is presenting with anaemia in the the gastrointestinal tract leading to glossitis.
setting of chronic blood loss, koilonychias, and Pellagra can develop in certain parts of Africa
pica, consistent with iron deficiency anaemia. In and South America where corn-based diets
a state of iron deficiency, hepcidin production predominate. Treatrnent is with oral or
decreases in order to promote the transport of parenteral nicotinamide and usually results in
iron from the enterocyte basolateral surface into rapid improvement of syrnptorns.
circulation. Absorption of iron is facilitated by
vitamin C and impaired by dietary calcium. 19.10. Answer: A.
This malnourished i:Jitoholic patient presents
19.7. Answer: D. with clinical features suggestive of Wernicke's
Artificial sweeteners use sugar alcohols (e.g. encephalopathy (delirium and ophthalmoplegia),
sorbitol, xylitol) to provide sweet taste without which is a manifest2tion of thiamin (vitamin 8 1)
caloric contribution. Sugar alcohols are not deficiency. Thiamin is a water-soluble vitamin
absorbed by the gut and in high quantities can that acts as a co-factor in aerobic metabolism
act as an osmotic agent to move water of glucose. Neuronal cells are most Vulnerable

downloaded from www.medicalbr.com


NUTRITIONAL FACTORS IN DISEASE • 209

to damage from thiamin deficiency as they the synthesis of coagulation factors II, VII, IX
exclusively utilise glucose for energy and X. Deficiency can lead to coagulopathy.
requirements. The liver has very limited stores While cirrhosis, malabsorption and medications
of thiamin, so deficiency can manifest after only (e.g. warfarin) can also lead to coagulopathy,
1 month of a thiamin-free diet. this patient is previously healthy prior to this
acute episode.
19.11. Answer: D.
The patient is presenting with four features of 19.12. Answer: B.
Reynolds' pentad of ascending cholangitis Vitamin K is a fat-soluble vitamin produced by
(fever, right upper quadrant pain, jaundice, intestinal bacteria. Green leafy vegetables are
altered mental status), probably caused by rich sources of vitamin K, along with soya oil.
obstruction of the biliary tree. In obstructive Egg yolks represent a source of biotin and
jaundice, bile is unable to enter the gut lumen vitamin D. Liver is rich in vitamin A and
for fat digestion. As a result, the fat -soluble sunflower oil is rich in vitamin E (Box i 9. i 2).
vitamins, including vitamin K, are poorly
absorbed (Box i 9. i i ). Vitamin K is required in 19.13. Answer: C.
The child was born with a neural tube defect,
19.11 Gastrointestinal disorders that may be but is otherwise healthy. Folate deficiency in
associated with malabsorption of fat-soluble pregnancy is associated with spina bifida,
vitamins anencephaly and encephalocele due to
Biliary obstruction increased requirements during embryonic
Pancreatic exocrine insufficiency development. Pregnant women are advised to
Coeliac disease take folate supplements during the first
Ileal inflammation or resection
trimester. Mechanistically, folate acts as a

19.12 Summary of clinically important vitamins

Sources* Reference nutrient intake


Vitamin Rich Important (RNI)
Fat-soluble j
A (retinol) Liver Milk and milk products, 700 !lg men
eggs, fish oils 600 !lg women I
D (cholecalciferol) Fish oils Ultraviolet exposure to skin, 10 (.lg if >65 years or no
egg yolks, margarine, sunlight exposure
fortified cereals
E (tocopherol) Sunflower oil Vegetables, nuts, seed oils No RNI. Safe intake:
4 mg men
3 mg women
K (phylloquinone, Soya oil, menaquinones produced Green vegetables No RNI. Safe intake: 1 !lg/kg
menaquinone) by intestinal bacteria
Water-soluble
B1 (thiamin) Pork Cereals, grains, beans 0.8 mg per 9.68 MJ
(2000 kcal) energy intake
B, (riboflavin) Milk Milk and milk products, 1.3 mg men
breakfast cereals, bread 1.1 mg women
B, (niacin, nicotinic Meat, cereals 17 mg men
acid, nicotinamide) 13 mg women
B, (pyridoxine) Meat, fish, potatoes, bananas Vegetables, intestinal 1.4 mg men
microflora synthesis 1.2 mg women
Folate Liver Green leafy vegetables, 200 !lg
fortified breakfast cereals
B12 (cobalamin) Animal products Bacterial colonisation 1.5 !lg
Biotin Egg yolk Intestinal flora No ANI. Safe intake:
10-200 !19
C (ascorbic acid) Citrus fruit Fresh fruit, fresh and 4E mg
frozen vegetables
'Rich sources contain the nutrient in high concentration but are not generally eaten in large amounts; important sources contain less
but contribute most because larger arrounts are eaten.

downloaded from www.medicalbr.com


21 0 • NUTRITIONAL FACTORS IN DISEASE

methyl donor in the synthesis of DNA, RNA and fluid intake should be restricted in such patients
protein, with increased requirements during to 500 ml per day. A further 1 000 mL of a
cellular division. glucose/electrolyte solution with a sodium
concentration of at least 90 mmoi/L should be
19.14. Answer: D. given (e.g. St Mark's solution or Glucodrate,
The residual length of jejunum following Nestle) but no oral fluids over and above this
massive small bowel resection or bypass is a limit. Any subsequent deficit between intake
powerful predictor of the need for parenteral and output should be made up by parenteral
fluid or nutritional support. Those left with an fluid administration.
intact colon that can be anastomosed at the
time of initial surgery or at some time 19.17. Answer: A.
subsequently tend to fare better than those Deficiency of the water-soluble vitamin C
where the colon is lost (due to the further (ascorbic acid) has been shown to be prevalent
capacity of the colon to absorb water and in those aged >65 years living independently in
electrolytes). However, most patients with the UK. Its clinical presentation may be
<200 em of jejunum remaining will require oral precipitated by events such as trauma, surgery,
fluid restriction and the use of a glucose/ burns or infections and it tends to be more
electrolyte solution (with sodium concentration prevalent in those who smoke or use drugs
of 90-120 mmoi/L) to minimise diarrhoea and such as glucocorticoids or non-steroidal
maximise absorption of fluid and electrolytes. anti-inflammatory drugs. Ascorbic acid is very
In addition, those with < 100 em of jejunum heat labile and many traditional cooking
remaining will require a variable volume of methods lead to its degradation.
parenterally administered sodium chloride to Patients may notice poor healing of wounds.
maintain an adequate balance between what It may present with petechial or perifollicular
they can absorb orally and their overall fluid bleeding or larger ecchymoses. Gingival
requirements. Those with <75 em of jejunum swelling or haemorrhage may occur and, less
will also be unable to maintain their overall commonly, haemarthrosis or gastrointestinal
energy requirements by oral means and will bleeding. Anaemia is recognised.
require a variable amount of calories
administered parenterally (parenteral nutrition) in 19.18. Answer: E. J
addition to the other treatments above. There are nine essential amino acids (Bof
19 .18) that cannot be synthesised by the body
19.15. Answer: B. (e.g. through transamination) and must
Where the intestine is largely intact, functionally therefore be obtained in the diet. They are
normally and accessible to an enteral tube, the required in order to synthesise other proteins,
proven benefits of enteral over parenteral which have a variety of important functions.
nutrition are that the overall health-care costs Five other amino acids can only be synthesised
are less, that it is associated with fewer if there is an adequate dietary supply of their
episodes of infection, more rapid restoration of precursors. These are known as 'conditionally
normal intestinal function and a reduced length essential' amino acids.
of hospital stay.

19.16. Answer: B.
Although it may appear initially counterintuitive, 19.18 Amino acids
it is absolutely vital in this situation that further
Essential amino acids
intestinal losses are not incurred by Tryptophan Valine
inappropriate advice to simply drink more fluid. Histidine Phenylalanine
The jejunum is inherently 'leaky', allowing rapid Methionine Lysine
fluxes of sodium (and water) across the Threonine Leucine
Isoleucine
epithelial barrier. Taking fluids with a sodium
Conditionally essential amino acids and their precursors
concentration of < 90 mmoi/L results in a net
Cysteine: methionine, serine
efflux of sodium and water into the intestinal Tyrosine: phenylalanin~
lumen (and therefore greater stomal fluid Arginine: glutamine/glutamate, .aspartate
losses). Drinking a larger volume of such 'dilute' Proline: glutamate
Glycine: serine, choline
fluids leads to even more stomal losses. Oral

downloaded from www.medicalbr.com


NUTRITIONAL FACTORS IN DISEASE • 211

19.19. Answer: E. 19.20. Answer: E.


The evidence that tube feeding in advanced Gastrostomy is known to be associated with
dementia improves any of these parameters is risks of insertion (pain, bleeding, aspiration
very weak and inconsistent. Patients and their pneumonia, infection of the insertion site,
families are often caught up in a vicious cycle inadvertent damage to an intra-abdominal
of dementia and malnourishment (Fig. 19.19). organ or viscus, tube displacement) and
It is important to screen patients with dementia longer-term risks (infection at insertion site,
for malnutrition, to monitor body weight, tube displacement, aspiration of feed). It does
encourage an adequate intake of food and not offer any advantage over nasogastric
provide a pleasant home-like environment for feeding in terms of aspiration risk. The decision
meals. Oral nutritional supplements have been to go ahead should be taken when patients or
shown to be of benefit. Cases should be their carers are fully aware of the risks and
assessed individually. However, unless there is benefits involved.
some acute, reversible event (e.g. further
stroke, treatable infection), which may be
bridged by a short period of tube feeding, there
is little benefit to artificial nutritional support in
advanced cases of dementia.

Dementia Age-related
Frailty changes and
Cognitive
Sarcopenia impairment diseases

Intake!

Fig. 19.19 Malnutrition in dementia- a vicious circle. From


Volkert D, Chourdakis M, Faxen-/rving G, et a/. ESPEN guidelines
on nutrition in dementia. Clin Nutr 2015; 34:1052-1073.

downloaded from www.medicalbr.com


CM Farrell, C Thurtell

Diabetes mellitus
Multiple Choice Questions
20.1. A 110-kg 57 year old man presents with 25 and is dominantly inherited. One form of
1-month history of lethargy, urinary frequency MODY is due to mutation in glucokinase. How
and increased thirst. He has been dieting for should these patients be managed?
years but only recently has managed to lose A. Basal insulin
10 kg with little effort. What would be the B. Biguanide (metformin) alone
simplest test that could make the diagnosis? C. Diet alone
A. Autoantibodies to glutamic acid D. No treatment required
decarboxylase (GAD), protein tyrosine E. Sulphonylurea with meals
phosphatase islet antigen-2 (IA-2) and zinc
transporter 8 (ZnT8) 20.4. The diabetes team were asked to review
B. Capillary blood glucose a hyperglycaemic 37 year old man on the
C. Fasting venous blood glucose surgical ward who presented with abdomir:ml
D. Oral glucose tolerance test (OGTT) pain, general lethargy, weight loss and pal~
E. Random venous blood glucose loose bowel motions. He has been in hodpital a
number of times with upper abdominal pain
20.2. A 49 year old black woman with a body with a normal abdominal ultrasound scan. He
mass index (BMI) of 42 kg/m 2 presents with a has a family history of type 2 diabetes mellitus.
6-month history of fatigue and lethargy. Over He recently lost his job as a taxi driver as there
the past few days she has become increasingly was concern after he came to work smelling
thirsty, is getting up at night to pass urine and strongly of alcohol and was found to be above
has experienced some dysuria. On admission, the legal limit to drive. Mean corpuscular
blood glucose was measured at 40 mmoi/L volume (MCV) was raised on blood tests from
(720 mg/dl) with ketones of 4 mmoi/L and 3 years ago. On admission he had a blood
bicarbonate 12 mmoi/L. She is treated for glucose of 20 rnmoi/L (360 mg/dl) with
diabetic ketoacidosis (DKA), but what is the ketones of 3 mrnoi/L .and bicarbonate
most likely underlying diagnosis? 20 mmoi/L. What is the most likely
A. Impaired glucose tolerance diagnosis?
B. Latent autoimmune diabetes of adulthood A. Impaired glucose tolerance
(LADA) B.LADA
C. Metabolic syndrome C. Monogenic diabete$
D. Type 1 diabetes mellitus D. Pancreatic insuffi9iency
E. Type 2 diabetes mellitus E. Type 2 diabetes mellitus

20.3. The most common monogenic forms of 20.5. DKA is a medical emergency in people
diabetes are caused by defects in insulin with type 1 diabetes. What is the most
secretion. Maturity-onset diabetes of the young common mechanism of death in DKA in
(MODY) commonly develops under the age of children and adolescents?

downloaded from www.medicalbr.com


DIABETES MELLITUS • 213

A. Acute respiratory distress syndrome D. Maternal hyperglycaemia


B. Cerebral oedema E. Pregnancy-induced hyperphagia
c. Hypokalaemia
D. Pneumonia 20.1 0. A 38 year old man with poorly controlled
E. Septic shock type 1 diabetes suddenly develops double
vision. On examination he is unable to abduct
20.6. A 57 year old woman with a 5-year history his left eye. He undergoes an urgent CT scan
of diet-controlled diabetes is struggling to make of his brain, which is normal. What is the most
any further changes to her lifestyle. Her HbA1c likely diagnosis?
is above target. What would be the best A. Brain tumour
first-line pharmacological therapy? B. Diabetic mononeuropathy
A. Biguanide (e.g. metformin) C. Giant cell arteritis
B. Dipeptidyl peptidase-4 (DPP-4) inhibitor (e.g. D. Graves' eye disease
sitagliptin) E. Stroke
C. Insulin
D. Sodium and glucose co-transporter 2 20.11. A 21 year old student attends her family
(SGLT2) inhibitor (e.g. empagliflozin) physician feeling generally unwell with a 3-day
E. Sulphonylurea (e.g. gliclazide) history of vomiting. She has been trying to lose
weight and has been on a low-fat diet and
20.7. A 47 year old man with type 2 diabetes started to exercise daily. She felt unwell with
has been treated with a biguanide (metformin) diarrhoea and vomiting for 3 days following a
and a sulphonylurea (gliclazide) for a number of seafood meal. The family physician checked a
years. Unfortunately his glycaemic control has blood glucose level, which was 5.2 mmoi/L
deteriorated and his doctor is considering (94 mg/dl) and urine dipstick showed glucose
adding in a thiazolidinedione (pioglitazone). trace and ketones 3+.
What is its mechanism of action? What is the most likely cause of ketonuria?
A. Activation of peroxisome A. Diabetic ketoacidosis
proliferator-activated receptor y (PPARy) in B. Fasting
adipocytes C. High-carbohydrate diet
B. Delays carbohydrate absorption D. Repeated vomiting
C. Prevents breakdown of incretin hormones E. Strenuous exercise
D. Promotion of ~-cell insulin secretion ~·
E. Sensitises tissues to insulin 20.12. A 52 year old English lorry driver with
type 1 diabetes returns to clinic for his annual I
20.8. A 58 year old obese woman with an review. HbA1c is stable at 62 mmol!mol (7.8%)
8-year history of type 2 diabetes and an HbA1c and he reports no hypoglycaemic episodes. He
80 mmol/mol (9.5%) attends the diabetes clinic. is well aware of the need to check his glucose
She is recommended to start a glucagon-like prior to driving and does so; however, he does
peptide-1 (GLP-1) agonist. What are the not keep a record of them. Which of the
possible side effects? following is legally required in the UK for people
A. Bladder cancer prescribed insulin to enable them to drive a
B. Genital fungal infections heavy goods vehicle?
C. Hypoglycaemia A. Annual driving test
D. Pancreatitis B. Annual review with 3 months' glucose meter
E. Weight gain readings
C. Biannual diabetes review
20.9. A 24 year woman with type 1 diabetes is D. Continuous glucose monitoring
attending the antenatal clinic. She is 16 weeks E. Retinal screening
pregnant and is concerned. that her baby is
big for gestation. What is the most likely 20.13. A 56 year old woman has type 2 diabetes.
mechanism for this? She also has non-alcohotic fatty liver disease
A. Fetal hypoinsulinaemia (NAFLD), arthritis, coeliac disease, mild visual
B. Genetic factors impairment and chronic obstructive pulmonary
C. Hypertension disease (COPD). She smokes up to 20 cigarettes

downloaded from www.medicalbr.com


214 • DIABETES MELLITUS

a day. Which of the following conditions has a 20.11. A 32 year old woman attends the
pathophysiological link and is more common in antenatal clinic for her booking scan. She is 12
individuals with type 2 diabetes? weeks pregnant with twins and has been
A. Coeliac disease struggling with 'morning sickness'. She has a
B. COPD BMI of 36 kg/m 2 and undergoes an OGTI, the
C. NAFLD results of which are: fasting plasma glucose
D. Optic atrophy 4.8 mmoi/L (86 mg/dl); 2-hour plasma glucose
E. Rheumatoid arthritis 7.0 mmoi/L (126 mg/dl).
As part of her routine checks the midwife
20.14. A 67 year old female has had type 1 dips her urine and she has 2+ ketones. What is
diabetes for 50 years. She has an HbA1c of the most likely diagnosis?
42 mmol/mol (6%) and is very strict about her A. Diabetic ketoacidosis
diet. She was admitted for an elective total hip B. Gestational diabetes
replacement. On the day of surgery, she was C. Hyperemesis gravidarum
found by the junior doctor to be very drowsy D. Normal physiological response in pregnancy
with a capillary blood glucose of 2.2 mmoi/L E. Undiagnosed type 2 diabetes
(40 mg/dl). What should ideally happen next?
A. Cancel theatre 20.18. A 51 year old man with type 1 diabetes
B. Intravenous (IV) access and 100 ml of 20% returns to the foot clinic. He attends for regular
dextrose and repeat blood glucose in 15 review as he has an ulcer on his left heel. He
minutes has been on a walking holiday to the Amalfi
C. IV access and 100 ml of 50% dextrose c;oast for 2 weeks. The podiatrist asks for a
D. IV access and 200 ml of 20% dextrose medical' review as he is concerned that the left
E. Withhold insulin for rest of day foot is now warm and swollen. The ulceration
looks much improved and the patient feels well.
20.15. A woman at 20 weeks' gestation X-ray does not reveal any obvious bony
undergoes a 75-g oral glucose tolerance test abnormality. What is the most likely diagnosis?
with the following results: 0 minutes = A. Acute Charcot arthropathy
5.6 mmoi/L (1 01 mg/dl); 120 minutes = B. Deep vein thrombosis (DVT)
9.2 mmoi/L (166 mg/dl). According to the C. Dry gangrene
National Institute for Clinical Excellence (NICE) D. Gout
guidelines, what should be the immediate E. Osteomyelitis
management?
A. Dietary modification 20.19. A 47 year old woman with type 1
B. GLP-1 receptor agonist diabetes attends for annual review. She denies
C. Insulin any significant hypoglycaemia. Her results are
D. Metformin as follows: HbA1c 46 mmol/mol (6.4%); blood
E. Sulphonylurea, e.g. glibenclamide pressure (BP) 152/98 mmHg (average of 3);
weight 61 kg (BMI 24 kg/m 2); urinalysis: +
20.16. A frail 93 year old man with type 1 glucose, trace nitrites, albumin: creatinine ratio
diabetes for 46 years attends for review. His (ACR) 5 mg/rnrnol (previously early morning
HbA1c is 69 mmol/mol (8.5%). Blood pressure sample 6.2 mg/mmol); total cholesterol
is 152/82 mmHg for which he is taking an 3.8 mmoi/L (147 mg/dl).
angiotensin-converting enzyme (ACE) inhibitor Current medication: basal analogue insulin
(ramipril) and a calcium channel blocker (glargine), bolus/rapid-acting analogue insulin
(amlodipine). He has mild background diabetic (NovoRapid), ACE inhibitor (lisinopril), statin
retinopathy. Which of these treatment targets is (simvastatin).
most appropriate in this scenario? Which result is it most important to act
A. Avoidance of hypoglycaemia upon? '
B. HbA1c of 48 mmol/mol (6.5%) or less A. Blood pressure
C. HbA1c of 58 mmol/mol (7.5%) or less B. Cholesterol
D. Microvascular disease prevention C. HbA,c
E. No need to monitor blood glucose in view of D. Urinalysis
his age E. Weight

downloaded from www.medicalbr.com


DIABETES MELLITUS • 215

20.20. James is a 19 year old man from Ireland; D. Stimulation of hepatic gluconeogenesis
he has a family history of diabetes. His mother E. Stimulation of hepatic glucose uptake
developed diabetes later in life; he is unsure if
she required insulin but she often attended the 20.23. A 59 year old man with a BMI of 29 kg/
hospital. She died suddenly when he was m 2 is admitted to hospital with pleuritic chest
young. James is an active man but has recently pain and a productive cough and is found to
been hindered by general malaise, lethargy and have pneumonia. He has no history of diabetes
pain in his knees. He has had a steroid injection and takes no regular medication. As part of his
into his left knee with little improvement. The admission investigations, a plasma glucose is
following tests have been carried out: found to be 10.0 mmoi/L (180 mg/dL). Which
Haemoglobin 145 g/L Anti-GAD antibody: of the following is the most appropriate
(14.5 g/dL) negative management?
White blood cell count Anti-IA-2 antibody: A. Blood glucose monitoring with fasting
9
6.2 x 10 /L negative plasma glucose after recovery from infection
Urea 5.2 mmoi/L Antineutrophil B. Commence treatment with liraglutide
(31 mg/dL) cytoplasmic C. Commence treatment with metformin
Creatinine 62 JJmoi/L antibody D. No further assessment of glycaemic control
(0.70 mg/dL) (ANCA): negative E. Variable-rate intravenous insulin infusion
Glucose 11.4 mmoi/L Ferritin 1137 j.Jg/L
(205 mg/dL) 20.24. A 70 year old woman attends her family
HbA1c 51 mmol/mol physician complaining of excessive thirst and
(6.8%) fatigue. A random venous glucose is
What is the most likely diagnosis? 13.2 mmoi/L (238 mg/dL), confirming a
A. Hereditary haemochromatosis diagnosis of diabetes. She takes a number of
B. MODY medications for hypertension, ischaemic heart
C. Steroid-induced diabetes disease and polymyalgia rheumatica. Which of
D. Type 1 diabetes the following medications can precipitate
E. Type 2 diabetes hyperglycaemia?

._.
A. ACE inhibitor (e.g. ramipril)
20.21. Insulin is the main regulator of glucose B. Aspirin
metabolism and storage. It is secreted from C. Calcium channel blocker (e.g. amlodipine)
pancreatic ~ cells. These cells regulate blood D. Nitrate (e.g. isosorbide mononitrate)
glucose concentrations by coupling glucose E. Steroid (e.g. prednisolone)
with insulin secretion. Glucose enters the
20.25. An 18 year old female with type 1
I
pancreatic ~ cells by facilitated diffusion down
its concentration gradient through cell diabetes is admitted with suspected
pyelonephritis. She has not taken any insulin for
I
membrane glucose transporters (GLUTs).
24 hours during her acute illness. Her initial I
Through which GLUT does glucose enter
pancreatic ~ cells? blood tests include: plasma glucose 24 mmoi/L
(432 mg/dL), bicarbonate 12 mmoi/L and
A. GLUT1
ketones 5.5 mmoi/L. Which electrolyte will most
B. GLUT2
likely require regular monitoring and aggressive
C. GLUT3
intravenous supplementation?
D. GLUT4
E. GLUT5 A. Bicarbonate
B. Calcium
C. Magnesium
20.22. Blood glucose is tightly regulated in order
D. Phosphate
to provide a constant supply of glucose to the
E. Potassium
central nervous system. Following ingestion of
a meal containing carbohydrate, which of the
20.26. A 75 year old male with no prior diagnosis
following is most likely to occur in the normal
of diabetes is admitted to hospital because he
physiological state?
has become progressively more drowsy and
A. Inhibition of GLP-1 release unwell since being started on oral amoxicillinby
B. Inhibition of insulin release his family physician for a suspected chest
C. Stimulation of glucagon release infection 2 weeks ago. He appears clinically

downloaded from www.medicalbr.com


216 • DIABETES MELLITUS

dehydrated. His initial blood tests include: A. Advise him to avoid exercise
l
plasma glucose 55 mmoi/L (991 mg/dl), B. Always omit the short-acting insulin dose
ketones 0.1 mmoi!L, sodium 149 mmoi/L and after exercise
I serum osmolality 368 mmol!kg. Which of the C. Reduce his total daily insulin dose to relax
I'
following statements is correct with regard to the his glycaemic control
management of this patient? D. Refer for structured diabetes education
A. A solution of 10% dextrose is the initial programme
intravenous fluid of choice E. Refer to a tertiary centre for consideration of
B. Close monitoring of fluid balance is pancreatic islet transplantation
unnecessary
20.30. A 58 year old man with type 2 diabetes
C. Intravenous insulin is not required initially in
of 10 years' duration and a BMI of 33 kg/m 2
the absence of significant ketonaemia
attends clinic for review of his diabetes
D. Serum osmolality should normalise within 4
management. He has a suboptimal HbA1c
hours of treatment
of 69 mmol/mol (8.5%) on metformin
E. Thromboprophylaxis is contraindicated
monotherapy 1 g twice daily and would like to
discuss the addition of a second-line agent.
20.27. A 28 year old female has recently been
Which of the following options are the most
found to have hepatocyte nuclear factor 1 ex
appropriate if he wishes a strategy that
(HNF1cx) MODY. It is decided to treat
promotes weight loss?
her diabetes with gliclazide. Gliclazide, a
sulphonyulrea drug, exerts its hypoglycaemic A. DPP-4 inhibitor (e.g. sitagliptin)
effect by enhancing endogenous insulin B. GLP-1 agonist (e.g. liraglutide)
secretion. By which mechanism is this achieved? C. Insulin
D. PPARy agonistlthiazolidinedione (e.g.
A. Activation of PPARy
pioglitazone)
B. Activation of the GLP-1 receptor
E. Sulphonylurea (e.g. glipizide)
C. Closure of the transmembrane J3-cell KATP
channel 20.31. A 50 year old woman with type 2
D. Inhibition of DPP-4 diabetes presents to her family physician
E. Inhibition of SGLT2 complaining of genital thrush, which has ~ot
settled with topical antifungal treatment. She
20.28. A 21 year old female with type 1 had been started on a new oral hypoglydaemic
diabetes since childhood attends the diabetes drug 4 months earlier. Which of the following
clinic for review. She has been symptomatic of drugs is most likely to be responsible for her
hypoglycaemia several times since her last presentation?
appointment 6 months ago. Which of the
A. DPP-4 inhibitor (e.g. sitagliptin)
following is classed as a neuroglycopenic
B. Glucosidase inhibitor (e.g. acarbose)
symptom of hypoglycaemia?
C. PPARy agonistlthiazolidinedione (e.g.
A. Anxiety pioglitazone)
B. Confusion D. SGLT2 inhibitor (e.g. empagliflozin)
C. Headache E. Sulphonylurea (e.g. glimepiride)
D. Hunger
E. Sweating 20.32. A 35 year old .woman with type 1
diabetes of 20 years: duration presents with
20.29. A 24 year old male with type 1 diabetes chronic nausea, early satiety and intermittent
of 12 years' duration presents with frequent vomiting after meals.l She has a history of poor
episodes of hypoglycaemia. He goes running glycaemic control, retinopathy and peripheral
for up to 60 minutes 4 times per week and the neuropathy. Which of the following
hypoglycaemic episodes occur after exercise. investigations will b~,most helpful in
He has good awareness of hypoglycaemia and establishing a diagnosis?
is able to take corrective action on each A. Abdominal ultrasonography
occasion. He is on a basal-bolus insulin B. Anti-tissue transgl!Jtaminase (anti-tTG) antibody
regimen and his latest HbA1c is 62 mmol/mol C. Barium swallow
(7.8%). Which of the following interventions is D. Gastric emptying study
the most appropriate management? E. Plain chest radiograph

downloaded from www.medicalbr.com


DIABETES MELLITUS • 217

20.33. A 21 year old women with type 1 D. Centrally acting antihypertensive (e.g.
diabetes of 8 years' duration with good moxonidine)
glycaemic control - HbA1c 48 mmol/mol (6.5%) E. Thiazide diuretic (e.g. bendroflumethiazide)
- on basal-bolus insulin presents to her young
adult specialist clinic for routine review. She has 20.36. A 65 year old man with type 2 diabetes
been experiencing intermittent abdominal of 20 years' duration is referred to the specialist
bloating, diarrhoea and weight loss over the last diabetes foot clinic by his family physician with
3 months. Recent urea and electrolytes, liver an ulcer of the plantar surface of the right foot.
function tests and thyroid function tests were all The ulcer has been present for approximately
within normal limits. Which of the following is 6 weeks and there is a history of peripheral
the best next investigation to perform? diabetic neuropathy. On examination, there is a
A. Abdominal ultrasonography 2-cm diameter ulcer in proximity to the first
B. Anti-tTG antibody metatarsal head. It has an offensive odour and
c. Flexible sigmoidoscopy discharge. The area around the ulcer is hot and
D. Gastric emptying study erythematous. Which of the following features,
E. Upper Gl endoscopy if present, would most strongly indicate the
presence of osteomyelitis (bone infection)?
20.34. A 19 year old male with type 1 diabetes A. A normal plain foot radiograph
is admitted to hospital complaining of B. Elevated blood white cell count
generalised abdominal pain and vomiting. He is C. Increased skin temperature compared to the
apyrexial, tachycardic and clinically dehydrated. contralateral foot
There is no peritonism in the abdomen. He has D. Peripheral oedema
the following blood results: blood glucose E. The ulcer probing to the depth of bone
22 mmoi/L (396 mg/dl), ketones 4.3 mmoi/L,
bicarbonate 11 mmoi/L, alkaline phosphatase 20.37. A 72 year old man is admitted to hospital
250 U/L, white cell count 19 x 109/L and by his family physician for urgent investigation
haemoglobin 182 g/L. Which of the following of weight loss. He has a progressive 3-month
statements regarding interpretation of these history of back pain, jaundice, dark urine and
results is correct? anorexia. He has lost approximately 15 kg in
A. He can safely be discharged home weight. In the last 4 weeks he has developed
B. Measurement of venous pH will be normal increased thirst and is drinking excessively.
C. The elevated alkaline phosphatase A random venous glucose is 16.0 mmoi/L
enzyme invariably indicates vitamin D (288 mg/dl). Which investigation is most likely
deficiency to reveal the cause of his diabetes?
D. The elevated haemoglobin concentration will A. Anti-GAD and anti-IA-2 antibodies
likely normalise after intravenous fluid B. CT scan of the pancreas
administration C. Dexamethasone suppression test
E. The elevated white cell count invariably D. Faecal elastase
indicates underlying infection E. Serum C-peptide

20.35. A 48 year old man with type 1 diabetes 20.38. A 29 year old woman with type 1
of 30 years' duration attends clinic for routine diabetes for 18 years attends clinic for routine
review. He is on a basal-bolus insulin regimen review. She has poor glycaemic control with an
and has an HbA1c of 70 mmol/mol (8.6%). He HbA1c of 90 mmol/mol (1 0.4%). She is keen to
is on no other medication. Blood pressure is embark on stricter glycaemic management in
155/92 mmHg (repeated 3• times with similar advance of planning pregnancy. Which of the
results) and he has microalbuminuria with an following complications of diabetes would be
ACR of 7.3 mg/mmol. Estimated glomerular the most likely to deteriora!e· significantly should
filtration (eGFR) rate is 54 mUmin/1.73 m 2 . her glycaemic control impfove suddenly?
Which of the following drugs would be most A. Foot ulceration
beneficial? 1
B. Gastroparesis
A. ACE inhibitor (e.g. lisinopril) C. Microalbuminuria
B. ~-blocker (e.g. atenolol) ' D. Peripheral vascular disease
C. Calcium channel blocker (e.g. amlodipine) E. Retinopathy

downloaded from www.medicalbr.com


21 8 • DIABETES MELLITUS I i

20.39. An 18 year old woman with type 1 20.40. A 45 year old man with diabetes
diabetes attends her diabetes clinic to discuss presents with a 4-week history of weight loss,
the possibility of continuous subcutaneous polyuria and polydipsia. His blood results
insulin therapy (insulin pump therapy). She has include: random plasma glucose 20 mmoi/L
a suboptimal HbA1c of 68 mmol/mol (8.4%) and (360 mg/dl), ketones 2 mmoi/L and HbA1c
takes multiple daily injections of insulin. Which 110 mmol/mol (12.2%). He was diagnosed with
of the following statements is correct with diabetes 6 months ago at which point his BMI
regard to insulin pump therapy? was 23 kg/m 2 and HbA1c 65 mmol/mol (8.1 %).
A. A continuous glucose monitoring system There is no family history of diabetes. Since
(CGMS) is mandatory for all patients diagnosis he has been treated with metformin
B. DKA does not occur as insulin administration and a sulphonylurea. p-cell antibodies are
is constant checked and he is found to have a very high
G. Patients have to inject long-acting insulin in titre of anti-GAD antibodies. Which of the
addition to the pump-delivered insulin following diagnoses best fits with this scenario?
D. The rate of insulin delivery can be adjusted A.LADA
depending on the time of day B. Mitochondrial diabetes
E. There is an increased risk of microvascular G. MODY
disease compared to multiple daily D. Pancreatic disease
injections E. Type 2 diabetes

Answers
20.1. Answer: E. glucose but a normal post-prandial response.
This patient has osmotic symptoms in keeping Therefore, patients with glucokinase MODY
with hyperglycaemia. Given that he is generally have stable, mild hyperglycaemia, do
symptomatic, a random venous blood glucose not require treatment or monitoring and are at
of 2 11 .1 mmoi/L (2 200 mg/dl) is sufficient to very low risk of developing any diabetes ,
give the diagnosis of diabetes. This test will be complications.
the least burdensome to the patient and most
cost-effective. 20.4. Answer: D.
This patient has symptoms and signs in
20.2. Answer: E. keeping with pancreatic insufficiency, both
This patient is correctly treated for DKA as she endocrine and exocrine. The history of alcohol
has elevated blood glucose in keeping with excess is helpful to aid diagnosis. Alcohol
diabetes, elevated ketones and a metabolic excess can cause recurrent bouts of acute
acidosis. Given her ethnicity, BMI and pancreatitis, which can lead to progressive
prodromal illness, a diagnosis of 'ketosis-prone' destruction of the pancreas. Diabetes due to
diabetes (which is more common in patients of pancreatic insufficiency can sometimes be
African origin) is likely with an underlying managed with oral therapy but often requires
diagnosis of type 2 diabetes. This is important treatment with insulin.
as initially patients require insulin treatment but
as glucose levels are controlled and p cells 20.5. Answer: B.
recover, patients may be able to transfer off The average fluid loss in an adult with
insulin to oral hypoglycaemic agents. moderately severe DKA is 6 L. Patients are
therefore aggressively fluid replaced in the
20.3. Answer: D. first few hours. Cau9c>n is required in fluid
MODY is defined as non-insulin-dependent replacement in children and young adults
diabetes that develops under the age of 25 in due to the risk of cerebral oedema (a
one family member. Glucokinase is a pancreatic paediatric-specific DJ<A protocol should be
glucose sensor and patients with glucokinase used). The osmolar gradient caused by the high
mutations have an altered set-point for glucose. blood glucose results in water shift from the
This results in a slightly high fasting blood intracellular fluid to extracellular fluid and

downloaded from www.medicalbr.com


DIABETES MELLITUS • 219

contraction of cell volume. Correction of the drive fetal growth, resulting in an increased
blood glucose with insulin and fluids can result birth weight.
in a rapid reduction in the osmolarity, which in
turn reverses the fluid shift and development of 20.10. Answer: B.
cerebral oedema. It is thought that the cerebral Diabetic mononeuropathy is loss of a sensory
oedema is related to cerebral vasoconstriction, or motor function within a single peripheral or
brain ischaemia and hypoxia. As children's cranial nerve, in this case the 6th cranial nerve
brains have higher oxygen requirements than -resulting in sudden-onset diplopia. Given.that
adults, this may explain their unique the CT brain is normal and there are no other
susceptibility. Hypokalaemia-related cardiac symptoms or signs, it is unlikely to be in
events used to be a major cause of death but keeping with brain tumour or stroke. He would
potassium monitoring and replacement is now be unlikely to present with Graves' eye disease
much improved. with no features in keeping with thyrotoxicosis.
Giant cell arteritis commonly presents with
20.6. Answer: A. temporal tenderness and amaurosis fugax not
Metformin is a potent blood glucose-lowering diplopia.
treatment that is weight-neutral or can lead to
weight loss. It is low cost and does not cause 20.11. Answer: D.
hypoglycaemia. It is used as first line for type Ketone bodies are organic acids that are
2 diabetes in all patients who can tolerate it. formed during fat metabolism. When the body
The long-term benefits were shown in the UK has insufficient insulin or depletes its own
Prospective Diabetes Study. It is usually carbohydrate stores it will metabolise fat for
maintained when other medications are energy. Ketonuria may be found in normal
added. people who have been fasting or exercising
strenuously for long periods, who have been
20.7. Answer: A. vomiting repeatedly or who have been eating a
Thiazolidinediones predominantly work in diet high in fat and low in carbohydrate (all of
adipose tissue. They bind and activate PPARy. these circumstances can cause glycogenic

--;
This nuclear receptor regulates the expressions depletion). The history and glucose level in this
of many genes involved in metabolism. case are not in keeping with diabetic ketosis. 1
Thiazolidinediones enhance the action of The history here suggests vomiting to be the 1
endogenous insulin in the adipose cells but also most likely cause of ketonuria.
alter the release of adipokines, which adjust
insulin sensitivity in the liver. 20.12. Answer: B.
In the UK, according to the Driver and Vehicle
I
20.8. Answer: D. Licensing Agency (DVLA) it is a legal
All incretin-acting drugs have been reported to requirement for people on insulin who drive
be associated with an increased risk of larger vehicles such as buses or heavy goods
pancreatitis. Unlike sulphonylureas they do not vehicles to have an annual examination by a
cause hypoglycaemia as they only promote diabetes specialist along with a review of 3
insulin secretion when there is a glucose months of glucose meter readings. This patient
trigger. Sulphonylureas can lead to weight gain, should keep a record of his blood glucose
as can pioglitazone and insulin, but GLP-1 readings and the team should consider
agonists usually cause weight loss. Pioglitazone advocating a blood glucose meter, which can
is associated with an increased risk of bladder be electronically downloaded to provide this
cancer. SGLT2 inhibitors cause increased data at his annual review.
glycosuria, resulting in genital fungal infections
and increased risk of urine tract infections. 20.13. Answer: C.
Hypertension, NAFLD and
20.9. Answer: D. hypercholesterolaemia are associated with type
Maternal hyperglycaernia .causes fetal 2 diabetes due to insulin resistance. This
hyperglycaemia due to transmission of glucose cluster of conditions has. been termed the
across the placenta. Fetal insulin levels will 'insulin resistance syndrome' ~r 'metabolic
consequently rise. Insulin is a major fetal growth syndrome' and is much more common in
factor, and high levels of fetal insulin therefore obese individuals. Coeliac disease - gluten

downloaded from www.medicalbr.com


· 220 • DIABETES MELLITUS

intolerance - is an autoimmune condition disease, or those treated with insulin, given


affecting primarily the small bowel, caused by a risks of hypoglycaemia. The benefits of
reaction to gliadin, a gluten protein. Optic a lower target HbA1c need to be weighed up
atrophy and rheumatoid arthritis may also against the risk of hypoglycaemia in
have underlying autoimmune aetiologies. insulin-treated patients. Longer diabetes
Autoimmune diseases are more commonly duration and frequency of hypoglycaemic
associated with type 1 diabetes than type 2 episodes are risk factors for impairment of
diabetes. symptomatic response to hypoglycaemia.
Advanced age itself is a risk factor for
20.14. Answer: B. hypoglycaemia. Multiple comorbidities, reduced
Treatment of hypoglycaemia depends on the appetite and renal function can predispose to
severity and whether the patient is able to hypoglycaemia in older age. Given this patient's
swallow. Conscious patients should be treated age and duration of disease, the main aim of
with oral carbohydrate. If assistance is required treatment should be towards safety and
to treat hypoglycaemia this is called 'severe symptom control: i.e. to avoid
hypoglycaemia'. Patients should receive hypoglycaemia.
100-200 ml 20% dextrose or intramuscular
glucagon. If the patient is conscious and able 20.11. Answer: C.
to swallow, GlucoGel or refined sugar drink can The most likely diagnosis is hyperemesis
be used. Blood glucose should be re-checked gravidarum, a complication of pregnancy
after 15 minutes and re-treated if needed. characterised by severe nausea, vomiting and
Once conscious, an oral complex-carbohydrate dehydration. The diagnosis is usually clinical but
snack should be given. Insulin should not be has been defined as three of more episodes
omitted. If a dosing error was made, then of vomiting in a day such that weight loss of
adjustment may be required. Fifty per cent 5% has occurred and there is evidence of
glucose is generally discouraged due to risk of ketones in the urine. Risk factors include first
infusion vein irritation. pregnancy, multiple pregnancy, obesity and a
family history. When vomiting is severe it can
20.15. Answer: A. lead to dehydration and ketosis.
The aim of managing gestational diabetes is to A diagnosis of gestational diabetes is based
normalise the maternal blood glucose and upon maternal blood glucose measures thq.t
therefore reduce excessive fetal growth. The are associated with increased fetal growth.
first step is dietary modification, namely Women at high risk of developing gestational
reducing the amount of fast -acting refined diabetes include: BMI > 30 kg/m 2 ; previous
carbohydrate. Regular blood glucose macrosomic baby; previous gestational
monitoring is required. If diet alone does not diabetes; a first-degree relative with gestational
achieve targets, then metformin should be diabetes; and high-risk ethnicity. These women
started. If not tolerated or additional treatment should all undergo screening. This woman has
is required, insulin should be commenced. a BMI of 36 kg/m 2 , so warrants screening;
Glibenclamide can be used if metformin and however, her results did not reach the NICE
insulin are not tolerated, because it does not criteria for gestational or any other type of
cross the placenta. In some countries, diabetes.
glibenclamide and metformin are not licensed
for use in pregnancy. There is no evidence for 20.18. Answer: A.
safety of any other hypoglycaemic agents in Acute Charcot arthropathy almost always
pregnancy. presents with signs of inflammation - a hot,
red, swollen foot. Initial X-ray may show bony
20.16. Answer: A. destruction but can be normal. There is often a
The aims of treatment and target HbA1c history of peripheral n~uropathy. or previous
depends on the individual patient. Early in foot ulceration. As Charcot can be difficult to
diabetes a target HbA1c of 48 mmol/mol (6.5%) differentiate from osteomyelitis, MRI of the foot
or less may be appropriate to try to prevent can be helpful. The p<;~thophysiology is not well
microvascular disease. An HbA1c of 58 mmol/ understood but may involve. unperceived
mol (7.5%) or less may be more appropriate in trauma with underlying neuropathy leading to
some older patients with cardiovascular progressive destruction. In this case, minor

downloaded from www.medicalbr.com


DIABETES MELLITUS • 221

trauma may have occurred on his walking descent. The disease is inherited in an
holiday and repeated 'trauma' may have led to autosomal recessive pattern.
bonY destruction. In view of the recent ulcer,
osteomyelitis should be excluded (MRI should 20.21. Answer: B.
be considered), but there is some reassurance GLUT2 is present in renal tubular cells, liver
that the area of ulceration has improved and cells and pancreatic 13 cells. It is a bidirectional
the patient is systemically well. Although he has transporter, allowing glucose to flow in two
been on a flight, he has otherwise been active directions. This is required in pancreatic 13 cells
and has a swollen foot - not calf - making DVT so that the intracellular environment can
less likely. The history is not in keeping with accurately measure the serum glucose levels.
gout or gangrene. GLUT1 is expressed in erythrocytes and in the
endothelial cells of the blood-brain barrier. It is
20.19. Answer: A. responsible for the low level of basal glucose
Microalbuminuria is the presence of small uptake needed to maintain respiration in all
amounts of albumin in the urine at a cells. GLUT3 is mostly expressed in neurons
concentration not detected on standard and in the placenta. GLUT4 is found in adipose
urinalysis. Early morning urine is measured for tissue and striated muscle; it is regulated by
albumin: creatinine ratio. Microalburninuria is insulin and is responsible for insulin-regulated
present if ACR is 2.5-30 rng/mmol creatinine in glucose storage. GLUT5 is a fructose
men and 3.5-30 mg/mrnol creatinine in transporter expressed in enterocytes in the
women. False positives should be excluded small intestine.
and 2 out of 3 samples should be positive to
confirm the diagnosis (ideally an early morning 20.22. Answer: E.
sample on repeat). In the post -prandial period, there are rises in
Microalbuminuria is a good predictor of portal vein glucose and insulin and a fall in
progression to nephropathy in type 1 diabetes. glucagon. The production of glucose in the liver
The presence of established microalbuminuria is suppressed and the uptake of glucose in the
should prompt action to reduce the risk of liver and peripheral tissues is increased. The
progression of nephropathy and of incretin hormones - GLP-1 and gastric
cardiovascular disease. This should be done by inhibitory polypeptide (GIP) -augment insulin
aggressive reduction of blood pressure, secretion following oral glucose delivery.
optimising glycaemic control and cardiovascular
risk reduction. 20.23. Answer: A.
This patient has good glycaernic control The patient most likely has 'stress
and has a healthy BMI with normal level of hyperglycaernia' provoked by acute illness, in
cholesterol. Blood pressure rnay need further this case infection. Underlying impaired
validation in the first instance, but presuming it glycaemic control or diabetes, however, could
is persistently elevated, then it is diagnostic for also be present. A diagnosis of diabetes in the
hypertension and needs lowering for vascular asymptomatic individual requires follow-up
and renal protection. testing of plasma glucose. In this case, the
patient's capillary blood glucose levels should
20.20. Answer: A. be monitored and he ought to have a repeat
The raised ferritin in this case points to a assessment of his plasma glucose when
diagnosis of hereditary haemochromatosis, a recovered from the acute illness.
disease characterised by excessive intestinal
absorption of dietary iron, resulting in a 20.24. Answer: E.
pathological increase in total body iron stores. The only medication listed that may result in
Excess iron accumulates in tissues and organs, hyperglycaernia and the development of
disrupting their normal function. The most drug-induced diabetes is prednisolone, a
susceptible organs include liver, adrenal glands, glucocorticoid. There is a:n increased risk
heart, skin, gonads, joints and pancreas. of developing diabetes while taking
Patients can present with airrhosis, 13-adrenoceptor antagonis.ts (13-blockers) and
polyarthropathy, adrenal failure, heart failure or thiazide diuretics for blood pres~ure control, but
diabetes. The hereditary form is most common not with other antihypertensive agents. Other
in those of Northern European and Celtic groups of patients at risk of developing

downloaded from www.medicalbr.com


222 • DIABETES MELLITUS

diabetes are those taking atypical antipsychotic 20.28. Answer: B.


medications and transplant recipients taking Non-specific symptoms include nausea,
calcineurin inhibitors. headache and tiredness. Autonomic symptoms
result from activation of the autonomic nervous
20.25. Answer: E. system and include sweating, hunger,
There is marked whole-body depletion of trembling, anxiety and palpitation.
potassium in DKA and serum levels do not Neuroglycopenic symptoms result from the
correlate with the total body deficit. Both brain being deprived of glucose and include
hypokalaemia and hyperkalaemia can occur in delirium, drowsiness, speech disturbance,
DKA and be fatal. Hypokalaemia can be inability to concentrate, incoordination,
exacerbated by intravenous insulin, which irritability, visual disturbance and focal
moves potassium into the intracellular neurological deficit. Autonomic symptoms
compartment. Therefore, in DKA management, usually occur first to allow patients to recognise
serum potassium levels must be carefully the hypoglycaemia and treat it, but may be
monitored. Phosphate levels typically fall in DKA absent in individuals with impaired
but there is no need to routinely replace in the hypoglycaemia awareness.
absence of muscle weakness. Bicarbonate
levels normalise with fluid resuscitation; 20.29. Answer: D.
bicarbonate infusion is not recommended and Patient education is fundamental to the
may in fact be harmful. prevention and management of hypoglycaemia.
Structured education can provide assistance
20.26. Answer: C. with carbohydrate intake and insulin
Hyperosmolar hyperglycaemic state (HHS) is a management around the time of exercise. He
potentially fatal complication of type 2 diabetes, already has a suboptimal HbA1 c and relaxing
which sometimes occurs in patients with no this will put him at increased risk of
prior history of diabetes. Patients are extremely microvascular complications as well as offering
dehydrated and require fluid resuscitation, no genuine protection against hypoglycaemia.
initially with a solution of 0.9% sodium chloride, Exercise should not be discouraged as a
with careful monitoring of fluid balance. There is means of avoiding hypoglycaemia. Reduction of
a high risk of venous thromboembolism; hence insulin dose pre-exercise may be a useful,
low-molecular-weight heparin is indicated. strategy to avoid hypoglycaemia during ,
IV insulin infusion is necessary only in the exercise but structured education will equip him
presence of significant(> 1.0 mmoi/L) with a broader range of strategies for the
ketonaemia or metabolic acidosis (i.e. DKAI future. Pancreatic islet transplantation is only
HHS overlap). Infection is a common precipitant considered in individuals who experience
of HHS but antibiotics are not used recurrent severe hypoglycaemia despite
prophylactically in the absence of infection. optimisation of glycaemic control after
It typically takes 72 hours or longer for the structured education.
biochemical disturbances to correct with
treatment of HHS. 20.30. Answer: B.
Liraglutide, a GLR-1 agonist, may lead to
20.27. Answer: C. weight loss as well as improved glycaemic
Hypoglycaemic drugs have varied mechanisms control. SGLT2 inhibitors are also associated
of action. Sulphonylureas exert their effect with weight loss. The other options are
on pancreatic ~ cells by closing the KAw either weight-neutral (sitagliptin, a DPP-4
channel, which results in downstream effects inhibitor) or promote weight gain (glipizide, a
culminating in insulin exocytosis. Gliptins inhibit sulphonylurea; pioglitazone, a thiazolidinedione;
DPP-4, thereby potentiating the action of insulin).
endogenous incretins such as GLP-1 and
GIP. Thiazolidinediones activate PPARy in 20.31. Answer: D.
adipose tissue. GLP-1 analogues activate the Empagliflozin, an inhibitor of the sodium and
GLP-1 receptor. SGLT2 inhibitors have their glucose co-transporter 2 (SGLT2) in the kidney,
action in the kidney where they inhibit SGLT2, exerts its glycaemic effect by increasing the
thereby reducing re-uptake of glucose from amount of glucose in the urine. This can result
the urine. in fungal infection in approximately 5% of

downloaded from www.medicalbr.com


DIABETES MELLITUS • 223

patients taking the drug. Only if the problem appropriate antihypertensive agents in this
becomes recurrent or unacceptable to the case. These drugs confer additional benefit
patient is the drug withdrawn. The other drugs beyond simply lower blood pressure - they are
do not cause genitaltract infection but have associated with significantly reduced
their own class-specific side-effects. progression of nephropathy. Patients with
microalbuminuria benefit from aggressive
20.32. Answer: D. lowering of BP (often with multiple agents),
The most likely diagnosis is gastroparesis as a control of cardiovascular risk factors and
manifestation of autonomic dysfunction in optimisation of glycaemic control.
diabetes. An upper gastrointestinal (GI)
tract endoscopy is commonly performed 20.36. Answer: E.
as part of the diagnostic workup, but Features that potentially indicate osteomyelitis
definitive diagnosis is achieved by in the diabetic foot include: dactylitis (marked
demonstrating delayed gastric emptying by swelling of the entire digit), an ulcer that probes
oomtechnetium scintigraphy following a to the depth of bone and evidence of bony
solid-phase meal with imaging over 4 hours. destruction on a plain radiograph (X-ray). X-ray,
The other investigations will not provide a however, may be normal in 50% of cases. MRI
definitive diagnosis. is far more sensitive for osteomyelitis than a
plain X-ray. The presence of oedema, increased
20.33. Answer: B. heat and elevated inflammatory markers may
The most likely diagnosis is coeliac disease, occur in soft tissue infection alone and are not
which is strongly associated with type 1 specific for osteomyelitis. If there is clinical or
diabetes. Up to 1 in 20 people with type 1 radiological evidence of osteomyelitis, the
diabetes go on to develop coeliac disease. patient is typically treated with antibiotics for at
The best screening test for this condition is least 6 weeks.
anti-tTG antibody, which is typically present in
high titre in this condition, with the exception of 20.37. Answer: B.
concurrent immunoglobulin A deficiency. An The clinical scenario presented is that of
upper Gl endoscopy with 02 (second part of pancreatic carcinoma, which may present with
duodenum) biopsy is required to confirm the diabetes as a feature. Pancreatic insufficiency
diagnosis but only after initial anti-tTG testing. may also present with weight loss typically in a
Coeliac disease should be considered in those patient with a history of alcohol excess, but this
who present with weight loss, gastrointestinal condition is not associated with obstructive
symptoms, iron/folate deficiency anaemia, jaundice. This patient is likely to require insulin
infertility, osteoporosis/low bone mineral density therapy to control the osmotic symptoms of his
and malabsorption. diabetes.

20.34. Answer: D. 20.38. Answer: E.


The patient's clinical status and biochemical It is well recognised that when glycaemic
findings are consistent with DKA. He requires control improves rapidly, there can be a
urgent treatment in hospital. His pH on venous transient deterioration in the degree of
or arterial blood gas would be< 7.30 (W > retinopathy. This is thought to be due to loss of
50.1 nmoi/L) consistent with metabolic hyperglycaemia-induced hyperperfusion in the
acidosis. The white cell count and alkaline retinal circulation. The effect typically wears off
phosphatase levels are often elevated in DKA within 18 months. Therefore, in patients with
and subside with treatment. Due to significant established retinopathy, it is recommended that
volume depletion in this condition, haemoglobin improvement in glycaemic management should
concentrations are often elevated and will be effected gradually. Transjent worsening of
reduce with expansion of the intravascular peripheral neuropathy syr,pptoms can similarly
compartment following administration of be seen in this situation.
intravenous fluid.
20.39. Answer: D.
20.35. Answer: A. Insulin pump therapy is becoming more widely
ACE inhibitors such as lisinopril or angiotensin used for glycaemic control in type 1 diabetes.
receptor blockers would be the most The pump delivers a constant infusion of

downloaded from www.medicalbr.com


224 • DIABETES MELLITUS

short-acting insulin; there is no need for 20.40. Answer: A.


additional long-acting insulin. CGMS is not It is estimated that 50% of cases of type 1
mandatory, although it is a useful adjunct for diabetes present in adulthood. The patient is
monitoring glucose levels while on pump best described as having LADA, an insidious
therapy. As the technology becomes more form of type 1 diabetes that develops in
sophisticated, there will be more 'closed loop' adulthood. This condition initially presents like
systems, whereby the glucose monitor and type 2 diabetes but is characterised by a
pump communicate with one another to relatively rapid progression to insulin treatment
optimise insulin delivery. DKA is still a and the presence of ~-cell antibodies. MODY
possibility, for example in the case of pump and mitochondrial diabetes typically occur
failure. One of the attractions of pump therapy where there is a strong family history of
is the ability to alter the basal rate of insulin diabetes, sometimes with other clinical
depending on the time of day or level of manifestations such as sensorineural deafness
physical activity (i.e. lower basal rate at night in the case of mitochondrial disease. Pancreatic
or during exercise to reduce the risk of exocrine disease typically occurs in those
hypoglycaemia). The risk of microvascular with a history of conditions that affect
complications of diabetes depends on the pancreatic function such as pancreatitis,
HbA1c and glucose variability rather than the haemochromatosis, pancreatectomy and
mode of insulin delivery. pancreatic cancer.

downloaded from www.medicalbr.com


','

E EI-Omar, F Clegg,
MH Mclean

Gastroenterology
Multiple Choice Questions
21.1. A 38 year old woman, para 2+0 consults C. In Europe and North America, most cases
her family physician on account of new-onset are caused by infestation with Trypanosoma
painful mouth ulcers. Which statement about cruzi
mouth ulcers is correct? D. Manometry demonstrates failure of relaxation
A. They are a common feature of inflammatory of the lower oesophageal sphincter on
bowel disease swallowing and absent or weak simultaneous
B. They are malignant in I 0% of cases contractions in the oesophageal body after
C. They are managed with antibiotics in swallowing
recurrent cases E. Peroral endoscopic myotomy (POEM) is the
D. They are more common in pregnancy treatment of choice
E. They are particularly common in patients
with diverticulitis 21.4. A 32 year old man with a body mass
index of 32 kg/m 2 consults his family physician
21.2. A 19 year old female consults her family with a long history of heartburn and frequent
physician with recurrent oral thrush. He takes a use of over-the-counter antacids. The family
detailed clinical history, checks a number of
routine blood tests, offers her advice and starts
physician prescribes a 1-month course of
omeprazole, which cures his symptoms but I
her on an oral medication. Which statement is
correct?
they soon return after stopping the omeprazole.
The family physician refers him for an upper
I
A. Asking about dysphagia is irrelevant because
this is not caused by fungal infections
gastrointestinal (GI) endoscopy, which shows
evidence of a small hiatus hernia and Barrett's
I
B. Oral fluconazole would be an appropriate oesophagus. Which statement is true?
treatment in this case A. Acid is the only refluxate that causes injury to
C. The correct treatment is broad-spectrum the lower oesophageal mucosa
antibiotics B. Gastro-oesophageal reflux disease (GOAD)
D. The doctor should not start any treatment can be reliably diagnosed by symptoms
before confirming the presence of Candida C. Most patients who develop oesophagitis,
albicans on brushings or biopsies Barrett's oesophagus or peptic strictures
E. The oral contraceptive pill is the commonest have a hiatus hernia
cause of oral thrush in young females D. Patients are invariably obese
E. The incidence of GOAD is decreasing in
21.3. Which of the following statements about most populations /
achalasia is correct?
A. Barrett's oesophagus is a common finding 21.5. The patient in OuestiQn 21 .4 returns to his
on endoscopy family physician after the endoscopy with
B. Chest pain and heartburn are the usual considerable anxiety. He was alarmed by the
presenting symptoms mention of 'Barrett's oesophagus' in his

downloaded from www.medicalbr.com


226 • GASTROENTEROLOGY

endoscopy report, which his internet search settled on the highest dose of esorneprazole.
classified as a 'pre-malignant' condition. His Although he lives a very healthy lifestyle
maternal uncle died of 'gullet cancer' and he is (non-smoker, no alcohol), he is unwilling to
naturally very concerned about his own risks. abandon his body building and heavy exercise
Which statement about Barrett's oesophagus is regime. Which statement is correct?
correct? A. He should be referred for a POEM because
A. Annual surveillance endoscopy in all patients of his young age
with Barrett's oesophagus is mandatory B. He should be referred for an open 'Heller's
B. Barrett's oesophagus is a condition in which myotomy
the normal columnar mucosa of the lower C. He should be referred for oesophageal
oesophagus is replaced by squamous manometry and 24-hour pH studies with a
mucosa view to laparoscopic fundoplication
C. It is a pre-malignant condition with a D. His medical therapy should be optimised
1000-fold increased relative risk of with the addition of calcium channel
oesophageal cancer but with a lower blockers
absolute risk (5-1 0% per year) E. Long-term use of PPis is not a concern in
D. It is an entirely benign condition and his this young and healthy patient
family physician should reassure him that
it is not associated with oesophageal 21.8. A 56 year old man with no prior history of
cancer GORD presents with progressive dysphagia
E. Treatment of Barrett's oesophagus is only and weight loss of 10 kg over a 3-month
indicated for symptoms of reflux or period. He is a heavy smoker (40 pack years)
complications, such as stricture and consumes on average 40 units of alcohol
per week. He also complains of fits of coughing
21.6. A sprightly 82 year old woman with a past after swallowing. Which statement is correct?
history of a small hiatus hernia is recently A. He is likely suffering from a chronic food
diagnosed with osteoporosis and started on bolus obstruction
appropriate treatment. Three months later she B. He is more likely to have a squamous cell
complains to her family physician during a carcinoma than an oesophageal
routine visit of progressive dysphagia to solids, adenocarcinoma
especially to meat. The family physician notes C. He likely has 'Boerhaave's syndrome' ·
that she has lost 3 kg in weight although she D. He should be referred for an urgent barium
retains a good appetite. She has no other swallow
symptoms and clinical examination is otherwise E. The lack of GORD symptoms excludes
unremarkable. What is the most likely oesophageal adenocarcinoma arising on a
explanation? background of Barrett's oesophagus
A. She has an early oesophageal cancer
B. She has developed eosinophilic oesophagitis, 21.9. Considering oesophageal carcinoma,
a common condition at that age which statement is correct?
C. She has developed the Plummer-Vinson A. Approximately 70% of patients have
syndrome extensive disease at presentation
D. She is developing early dementia and B. Globally, adenocarcinoma is more common
forgetting to eat her meals than squamous cell carcinoma
E. She was started on bisphosphonates for the C. Metastases from oesophageal carcinoma are
osteoporosis usually localised to regional nodes adjacent
to the tumour
21.7. A 28 year old male body builder consults D. Oesophageal adenocarcinoma is particularly
his family physician on account of intractable common in the ?"iddle third of the
heartburn, severe regurgitation and retrosternal/ oesophagus
epigastric pain. An upper Gl endoscopy 2 years E. Risk factors for squamous cell carcinoma
previously confirmed the presence of moderate include achalasia, radiation oesophagitis,
oesophagitis. He had already received multiple caustic oesophageal stricture, Barrett's
courses of different proton pump inhibitors mucosa and Plummer-Vinson (Paterson-
(PPis) in escalating doses and symptoms have Brown-Kelly} syndrome

downloaded from www.medicalbr.com


--------------------------------.
GASTROENTEROLOGY • 227

21.10. A 74 year old man with dysphagia and A. She should arrange for him to have an
weight loss is diagnosed with oesophageal urgent barium meal
adenocarcinoma on upper Gl endoscopy. He B. She should check his H. pylori serology and
has a past medical history of hypertension, start him on eradication therapy if positive.
diet-controlled type 2 diabetes and mild C. She should start him immediately on H.
asthma. Which statement concerning his pylori eradication therapy
investigations and management is correct? D. She should start him on a course of PPis
A. Endoscopic ultrasound (EUS) is particularly and review him in 2 months for repeat blood
useful in assessing distant metastasis tests
B. Invasion of the aorta, major airways or E. She should start him on PPis and refer him
coeliac axis usually precludes surgery for an urgent upper Gl endoscopy
c. Oesophageal adenocarcinoma is very
sensitive to radiotherapy 21.14. The above patient undergoes upper Gl
D. Staging is futile, as his past medical history endoscopy, which shows a 2-cm chronic ulcer
precludes any operative intervention on the lesser curve of the stomach with no
E. The overall 5-year survival of oesophageal stigmata of recent haemorrhage. The rest of
adenocarcinoma is 50% the upper Gl tract is normal. Which statement
regarding his management is the most
21.11. Which statement is true regarding appropriate?
Helicobacter pylori (H. pylori) infection? A. He must have biopsies of the gastric ulcer to
A. Asymptomatic subjects are rarely infected by rule out malignancy and must have a repeat
H. pylori endoscopy 6-8 weeks later to confirm full
B. It is always present in patients with healing after treatment
dyspepsia B. He should avoid eating citrus fruits as these
C. It is always present in patients with peptic may delay healing of peptic ulcers
ulcers C. He should be referred to a surgeon for
D. It is usually acquired during early adulthood consideration of highly selective vagotomy
E. When present, it is always associated with D. He should only have antral biopsies to check
gastritis for presence of H. pylori infection, as gastric
ulcers are usually benign
E. There is no strong indication to stop

-
21.12. A 55 year old man presents with
progressive anorexia, weight loss, diarrhoea, smoking, as this has no impact on healing
nausea and vomiting, and profound peripheral rates
oedema. Blood tests show evidence of
anaemia and hypoalbuminaemia. Upper Gl 21.15. The patient in Question 21 .14 is found to . 'I
endoscopy shows enlarged, nodular and have H. pylori infection. Which statement about
coarse gastric folds. What is the most likely eradication therapy is correct?
diagnosis? A. Erythromycin is the most useful component
A. Classic NSAID gastropathy of eradication regimens
B. Crohn's disease of the stomach B. If first -line eradication therapy fails, the
C. Cronkhite-Canada syndrome same course should be repeated for another
D. Gl manifestations of thyrotoxicosis week
E. Menetrier's disease C. Metronidazole is no longer of benefit in
eradication regimens due to the very high
21.13. A 57 year old man who is a heavy resistance rates
smoker presents to his family physician with D. The inclusion of high-dose, twice-daily PPI
epigastric pain, occasional vomiting, tiredness therapy in eradication regimens increases
and easy fatigability. Clinical examination efficacy of treatment 1 ·
reveals signs of anaemia and epigastric E. The rate of success of eradication therapy is
tenderness but no masses or organomegaly. strongly dependent on the rate of amoxicillin
Routine blood tests confirm mild iron deficiency resistance in the popul?tion
anaemia but no other abnormalities.
Which action by the family physician is the 21.16. A 78 year old woman with osteoarthritis
most appropriate? and long-term indometacin therapy presents as

downloaded from www.medicalbr.com


228 • GASTROENTEROLOGY

an emergency with a short history of dizziness


and passage of black stools over 2 days.
Clinically, she has signs of anaemia,
gastro-oesophageal junction. Biopsies are
obtained from the tumour and from the
tumour-free distal stomach.
l
tachycardia and hypotension. Her haemoglobin Which statement is correct?
is 55 g/L and her urea is 21 mmoi/L (126 mg/ A. Biopsies from the tumour-free distal gastric
dl) with a creatinine of 85 f!moi/L (0.96 mg/ mucosa will invariably show evidence of
dl). What is the most likely diagnosis? H. pylori infection
A. She has acute on chronic renal failure due to B. Despite the large size of this tumour, it could
long-term NSAID therapy still be regarded as an early cancer because
B. She has bled from a gastric Dieulafoy lesion the chance of metastasis to distant organs is
C. She has had an acute lower Gl bleed from small
an NSAID-induced ulcer C. The bulge in the umbilicus is known as a
D. She has had an acute upper Gl bleed from Krukenberg tumour
an NSAID-induced ulcer D. The bulge in the umbilicus is likely a
E. She has suffered a silent myocardial metastatic nodule (Sister Joseph's
infarction, which is often asymptomatic in nodule)
older patients E. The most likely histology of this tumour is
squamous cell carcinoma because of its
21.17. Considering the acute management of proximity to the oesophagus
the patient in Question 21.16, what is the most
important action? 21.20. In relation to the patient in Question
A. She should be immediately referred to the 21.19, consider the management options for
on-call surgical team gastric cancer. Which of the statements below
B. She should be resuscitated with colloids/ is the most accurate?
crystalloids followed by blood products A. Endoscopic management is still possible if
C. She should be started on octreotide to stop local lymph node spread is limited
the bleeding B. For locally advanced tumours, partial
D. She should be started on tranexamic acid to gastrectomy with lymphadenectomy is the
stop the bleeding operation of choice
E. She should undergo immediate endoscopy C. In patients with inoperable tumours,
to treat any bleeding lesions chemotherapy is of no value in prolonging
survival
21.18. Gastric cancer is a leading cause of D. Proximal tumours involving the
cancer -related mortality worldwide. Which oesophago-gastric junction also require a
statement about this malignancy is correct? distal oesophagectomy
A. Africa has the highest incidence rates due to E. The biological agent trastuzumab may
the poor diet benefit some patients whose tumours
B. Diet is the most important risk factor over-express TP53
C. H. pylori infection is the most recognised
acquired risk factor 21.21. Which statement regarding gastric
D. The incidence has been steadily rising in lymphoma is correct?
most parts of the world in the past 3 A. EUS is a poor tool in staging gastric
decades lymphomas
E. The overall 5-year survival rates are now B. For lymphomas containing t(11 : 18)
around 50% chromosomal translocations, H. pylori
eradication alone is sufficient in controlling
21.19. A 65 year old man presents with a the disease
3-month history of nausea, vomiting, anorexia, C. H. pylori infectioryis the cause of all gastric
7 -kg weight loss and iron deficiency anaemia. lymphomas ·
Abdominal examination shows an epigastric D. High-grade B-cell lymphomas should be
mass and a bulging nodule in the umbilicus. treated by a combination of infliximab and
Upper Gl endoscopy confirms the presence radiotherapy
of a tumour occupying most of the E. The stomach is the most common site for
proximal stomach but not involving the extranodal non-Hodgkin lymphoma

downloaded from www.medicalbr.com


GASTROENTEROLOGY • 229

21.22. Which statement regarding B. Check human leucocyte antigen (HLA) status
gastrointestinal stromal cell tumours (GISTs) C. Commence a gluten-free diet and monitor
is correct? for symptoms
A. They are differentiated from other D. Endoscopy for gastric and jejunal biopsies
mesenchymal tumours by expression of the E. Recheck anti-tTG and lgA on a
c-kit proto-oncogene gluten-containing diet for 7 days
B. They are invariably benign and do not require
any specific management 21.26. Of those listed below, which
c. They are particularly aggressive and require pathophysiological process most likely leads to
resection and treatment with imatinib (a coeliac disease?
tyrosine kinase inhibitor) A. H. pylori colonisation of the small bowel
D. They arise from the interstitial cells of mucosa
Lieberkuhn B. Ingestion of gluten-containing foods in
E. They only bleed if patients also take genetically susceptible individuals leading to
NSAIOs aT-cell-mediated mucosal response in the
small bowel, associated with microbial
21.23. A 23 year old woman presents with dysbiosis
8-month history of bloating, loose stool and C. Ingestion of gluten-containing foods in
bowel-opening frequency of 3 times per day. individuals with HLA-002/008 status
There is no weight loss. Blood tests reveal a D. Ingestion of gluten-containing foods leading
haemoglobin of 108 g/L, a ferritin of 7 11g/L and to microbial dysbiosis and microbial
a folate of 1 11g/L and are otherwise normal. seqretion of short-chain fatty acids
What is the next best investigation? E. Interrupted T-cell tolerance in the colon
A. Abdominal X-ray leading to activation ofTh17 immune cells that
B. Coeliac serology with serum immunoglobulin react with gluten in the proximal small bowel
A(lgA)
C. Colonoscopy 21.27. A 50 year old man presents with
D. Stool calprotectin diarrhoea, low-grade fever and joint pains.
E. Stool culture A colonoscopy is normal. Biopsies from the
terminal ileum reveal the presence of foamy
21.24. A 42 year old man from the Indian macrophages. What is the appropriate
subcontinent presents with right iliac fossa pain management?


that has progressively increased in severity over A. Two weeks of intravenous ceftriaxone, then
the last few months. This is associated with
weight loss and low-grade fever. Blood analysis
oral antibiotics for 1 year
B. Seven days of oral metronidazole . I
reveals alkaline phosphatase (ALP) of 235 U/L C. Intravenous immunoglobulin
and y-glutamyl transferase (GG1) of 120 U/L. D. No treatment is re1 qui red - symptoms
Chest X-ray is normal. usually settle spontaneously
What is the most likely diagnosis? E. Oral omeprazole and restriction of dietary
A. Chronic appendicitis gluten
B. Crohn's disease
C. Human immunodeficiency virus (HIV) 21.28. An 82 year old man presents with
D. Ileocolonic tuberculosis (TB) persistent fresh rectal bleeding on passing
E. Whipple's disease stool. He has a past history of prostate cancer
diagnosed 2 years ago. Flexible sigmoidoscopy
21.25. A 25 year old woman, presents to the reveals a 10-cm segment of mucosal erythema
family physician requesting a test for coeliac associated with an abnormal vessel pattern and
disease as her sister has received a recent no ulceration. What is the most appropriate
diagnosis of this condition. Her blood management?
results show a mildly positive, anti-tissue A. 5-Aminosalicylate suppository
transglutaminase (tTG) with a low serum lgA. B. Endoscopic argon pla:;,ma coagulation
What is the next best investigation? C. Loperamide
A. Check anti-endomysia! antibody (anti-EMA) D. Predfoam enema
and immunoglobulin G (lgG) E. Sucralfate enema

downloaded from www.medicalbr.com


230 • GASTROENTEROLOGY

21.29. A 36 year old woman presents with an


intermittent history of diarrhoea and increased
bowel-opening frequency over a 2-year period.
A. Intramuscular glucocorticoids for I 0 days,
low-molecular-weight heparin (LMWH),
intravenous fluid and then reassess response
l
For the last 8 months, her symptoms have B. Intravenous glucocorticoids for 3 days,
worsened and become persistent. She is LMWH, intravenous fluid and then reassess
currently passing stool 8 times per day with response
urgency. The consistency of stool is watery and C. Intravenous vedolizumab infusion and
she reports that her stool is green in colour. smoking cessation
Her past medical history is mild asthma and D. Loperarnide and metronidazole
cholecystectomy 12 months ago. She has no E. Surgery with subtotal colectomy and
family history of note. ileostomy
What investigation is most likely to lead to
the diagnosis? 21.33. A 54 year old man with ulcerative colitis
A. Colonoscopy is receiving hospital treatment for acute severe
B. Endoscopy and duodenal biopsy ulcerative colitis. He becomes acutely unwell
C. Hydrogen breath test with sweating and shortness of breath.
D. 75
Se-homocholic acid taurine (SeHCAT) Pulse is IIObeats/min, blood pressure is
scan 120/68 mmHg, temperature is 37.7"C and
E. Thyroid function test respiratory rate is 28breaths/min. Oxygen
saturation is 94% on air. What is the most
21.30. A 19 year old woman presents with appropriate investigation to lead to the
a 6-month history of diarrhoea, right iliac diagnosis?
fossa pain and weight loss. Blood tests A. Chest X-ray
show her platelet count is 721 xI 09/L, B. CT pulmonary angiography
haemoglobin I 00 g/L and C-reactive protein C. Echocardiogram
(CRP) 62 mg/L. Investigation reveals a D. Electrocardiogram
diagnosis of terminal ileal Crohn's disease and E. Troponin I
her symptoms improve with a course of oral
prednisolone. What is the next most 21.34. A patient with inflammatory bowel
appropriate treatment? disease is commenced on azathioprine for
A. A 4-week course of ciprofloxacin maintenance immunosuppression. What Is the
B. Anti-a4J37 integrin biologic therapy appropriate advice to give the patient?
C. Azathioprine A. The patient requires an annual chest X-ray
D. Immunoglobulin and hepatitis serology
E. Methotrexate B. The patient requires blood analysis within I
week of starting this medication to test
21.31. Inflammatory bowel disease is associated amylase
with extra-intestinal manifestations of disease. C. The patient requires regular blood analysis to
Which of the following is most commonly test liver and bone marrow function
associated with ulcerative colitis? D. The patient should not conceive whilst on
A. Blepharitis this medication
B. Primary sclerosing cholangitis E. This medication reduces the risk of future
C. Psoriasis malignancy
D. Pyoderma gangrenosum
E. Renal calculi 21.35. Which of the following statements most
accurately describes a key pathophysiological
21.32. A 27 year old man with known ulcerative process underlying the development of Crohn's
colitis is admitted to hospital for management disease?
of an acute severe flare in colitis. His stool A. Activation of colonic nicotinic receptors by
frequency is I 0 times per day including smoking leads to altered host handling of
nocturnal bowel opening, and he has a mild adherent invasive. Escherichia coli
fever. Blood pressure is 122/78 mmHg, pulse B. Cytokines secreted from innate lymphoid
is 88beats/min; CRP is 82 mg/L. What is the cells cause mucosal inflammation
most appropriate initial management? throughout the colon

downloaded from www.medicalbr.com


GASTROENTEROLOGY • 231
I
C. It is caused by an autosomal recessive 21.39. A 68 year old man is referred with an
inherited interleukin-1 0 (IL -1 0) genetic 8-week change in bowel-opening habit with
mutation increased frequency to 4 times/day (baseline
D. The oral microbiome changes, with once/day) and change in consistency to looser
predominance of Bacteroides spp. stool. Blood analysis reveals haemoglobin
E. Transmural Th1 predominant adaptive 100 g/L and ferritin of 12 ~J-g/L. What is the
mucosal immune responses occur most appropriate investigation to lead to the
in association with colonic microbial diagnosis?
dysbiosis A. Barium enema
B. Colonoscopy
21.36. A 30 year old woman presents with C. CT scan of abdomen and pelvis
diarrhoea, associated with passing rectal D. Stool culture
mucus and blood. A diagnosis of mild E. Upper Gl endoscopy and duodenal biopsies
rectosigmoiditis is made on flexible
sigmoidoscopy. Biopsy histology shows chronic 21.40. Which statement is true when
inflammatory cell infiltrate, crypt abscesses and considering genetic aspects of colorectal
mild architectural distortion of the crypts. What cancer?
is the most appropriate initial treatment?
A. Colonic polyps are precursors to the
A. lnfliximab, anti-TNF biological therapy development of colorectal cancer and only
B. Oral 5-aminosalicylic acid (5-ASA) such as arise in individuals with APC gene mutation
mesalazine B. Familial adenomatous polyposis (FAP) is
C. Oral glucocorticoids most· commonly an autosomal dominant
D. Predfoam enema every 2. days condition with mutation within the APC gene
E. Ustekinumab, anti-p40 biological therapy C. Hereditary non-polyposis colon cancer
(HNPCC) occurs in up to 50% of colorectal
21.37. A 23 year old man has a known cancer cases and is associated with
diagnosis of terminal ileal Crohn's disease, mutation in mismatch repair genes
diagnosed 5 years previously and treated with D. Sporadic colonic cancer is directed solely by
azathioprine therapy. He presents with epigenetic changes rather than mutations of
increasing abdominal pain, bloating and nausea somatic genes
after eating. He has lost 3 kg in weight in the E. Sporadic colorectal cancer is associated with
last 4 months. What is the best investigation to mutation in the TP53 gene as an early
assess disease activity? occurrence
A. Barium enema
B. Calprotectin 21.41. A 46 year old woman with scleroderma
C. Capsule enteroscopy presents with frequent watery diarrhoea,
D. Small bowel barium meal and follow bloating and weight loss. Colonoscopy is
through normal with no abnormality found on biopsy
E. Small bowel magnetic resonance imaging histology. Coeliac serology is negative, thyroid
(MRI) enterography function normal and stool culture negative.
Bacterial overgrowth is suspected. What would
21.38. A 54 year old woman presents to clinic you expect to see on blood analysis to support
with urgency to pass stool and daily episodes this diagnosis?
of faecal incontinence. Flexible sigmoidoscopy A. Increase in immunoglobulins
does not reveal any mucosal pathology. Her B. Increased ferritin and normal haemoglobin
history includes vaginal birth of three children, C. Low vitamin B12 and increased mean cell
two of which required assisted delivery with volume
forceps. What is the most appropriate first D. Positive cytomegalovirus /serology
management? E. Reduced gastrin
A. Botox injection into internal anal sphincter
B. Defunctioning colostomy 21.42. A 23 year old woman. presents to the
C. Fluoxetine clinic with an 8-month history of variable
D. Pelvic floor exercises and biofeedback bowel-opening habit ranging from once to
E. Topical treatment with 2% diltiazem cream 4 times per day, passing soft stool in the

downloaded from www.medicalbr.com


232 • GASTROENTEROLOGY

morning and associated with bloating in the


evening. She describes left-sided crampy
21.46. A 40 year old male with a diagnosis of
chronic pancreatitis complains of foul-smelling
l
abdominal pain, relieved by defaecation. She is pale stools, which are difficult to flush. A
a single mother to a 3 year old boy. The deficiency in which of the following hormones/
company she works for has declared that jobs enzymes is responsible?
may not be stable during a management A. Chymotrypsin
change. Thyroid function is normal and coeliac B. Glucagon
serology is normal. C. Lipase
What is the next best investigation? D. Maltase
A. C-reactive protein E. Somatostatin
B. Platelet count
C. Small bowel MRI 21.47. A 42 year old man has recurrent
D. Stool calprotectin duodenal ulceration with bleeding despite
E. Upper Gl endoscopy and duodenal omeprazole therapy use. H. pylori serology has
biopsies been negative on three occasions. An
abnormality is noticed on imaging of his
21.43. A 30 year old woman has a diagnosis of pancreas. What other health problems might
diarrhoea-predominant IBS. Her symptoms are this man have?
negatively impacting her ability to work in the A. Marfan's syndrome
local supermarket and she is avoiding social B. Medullary thyroid cancer
functions for fear of unpredictable onset of C. Phaeochromocytoma
symptoms. She has tried a variety of dietary D. Pituitary adenoma
manipulations including a wheat-exclusion diet, E. Type 1 diabetes mellitus
a dairy-free diet and avoidance of caffeinated
drinks, with no improvement of her symptoms. 21.48. A 56 year old man with a history of
What is the next most appropriate treatment to heavy alcohol use presents with abdominal
consider? bloating following a recent episode of severe
A. 5-HT4 agonist, prucalopride central abdominal pain radiating to the back.
B. Nocturnal small-dose diazepam Clinical examination reveals resolving
C. Peppermint capsule periumbilical bruising and abdominal diste'nsion
D. Probiotics with shifting dullness. Blood tests demonstrate
E. Referral to dietician for consideration of a bilirubin 18 J.Lmol/L (1.05 mg/dL), alanine
low-FODMAP (fermentable oligo-, di- and aminotransferase (ALT) 50 U/L, ALP 200 U/L,
monosaccharides, and polyols) diet GGT 823 U/L, international normalised ratio
(INR) 1 .1. What initial test is most likely to give
21.44. Which of the statements is true the correct diagnosis?
when considering the normal function of the A. Alpha-fetoprotein
colon? B. Ascitic fluid albumin
A. Absorption of folic acid occurs in the C. Ascitic fluid amylase
colon D. Ascitic fluid cell count
B. The colon absorbs 50% of ingested protein E. Serum albumin
and fats
C. The colon absorbs electrolytes and water 21.49. Pancreatic fluid is normally excreted via
D. The colonic microbiota is acquired at birth the main pancreatic duct into the duodenum.
and is static throughout life In pancreas divisum, during embryonic
E. The pH of the colonic lumen is 3 development the dorsal and ventral ducts fail to
fuse, leading to main excretion via the dorsal
21.45. The arterial blood supply to the pancreas duct. Which structure does pancreatic fluid
is shared with which other organ/tissue? bypass in pancreaS' divisum?
A. Left kidney A. Ampulla of Vater ,
B. Ovaries B. Duodenum ·I
C. Psoas muscle C. Gallbladder
D. Sigmoid colon D. Hepatic portal vein
E. Terminal ileum E. Liver

downloaded from www.medicalbr.com


GASTROENTEROLOGY • 233

21.50. Twelve hours after endoscopic retrograde A. Amylase 2302 U/L


cholangiopancreatography (ERCP) for stent B. Calcium 2.42 mmoi/L
insertion due to biliary stricture, a patient C. CRP 120 mg/L
develops severe central abdominal pain. Blood D. Glucose 4 mmoi/L (72 mg/dl)
tests show bilirubin 20 J.lmoi/L, ALT 120 U/L, E. Pa0 2 7.8 kPa (59 mmHg)
ALP 80 U/L, GGT 98 U/L, amylase 1400 U/L,
haemoglobin 120 g/L, WCC 12.4x10 /L.
9 21.54. A 55 year old man presents with
What complication has she developed? obstructive jaundice and an abnormal pancreas
on CT. An elevation in which blood test would
A. Ascending cholangitis
suggest this condition may respond to
B. Common bile duct perforation
glucocorticoids?
c. Duodenal perforation
D. Haemorrhage A. Alpha-fetoprotein
E. Pancreatitis B. Bilirubin
C. Hepatitis E lgM
21.51. A 40 year old male with long-standing D.lgG4
alcohol excess has an episode of severe E. lgM
central abdominal pain. Six weeks later he
develops vomiting following eating. On 21.55. A 32 year old man with a previous
examination he has a gastric splash. What pituitary adenoma has repeated admissions
investigation will give the diagnosis? with duodenal ulceration despite proton
pump inhibitor. What is the likely genetic
A. Barium meal
mutati~n?
B. CT pancreas
C. Oesophageal manometry A. BRCA1
D. Upper gastrointestinal endoscopy B. BRCA2
E. Water-soluble contrast meal C.JAK2
D.MEN1
21.52. The CT scan below is from an 18 year E. RET
old with recurrent chest infections since
childhood and chronic diarrhoea. What 21.56. A 22 year old woman presents with
abnormality can you see? abdominal swelling and vomiting. Which
is the diagnosis that should be
excluded first?
A. Ascites
B. Bowel obstruction
C. Constipation
D. Obesity
E. Pregnancy

21.57. A 35 year old male underwent a gastric


bypass 6 months ago, and now presents with
fatigue and breathlessness. Blood tests
demonstrate haemoglobin 58 g/L, mean cell
volume 128 fl. A deficiency in which cells will
lead to these test result abnormalities?
A. Calcification of pancreas
B. Colonic diverticulae A. Chief cells
C. Common bile duct gallstones B. D cells
D. Distended gallbladder C. G cells
E. Small bowel stricture D. Oxyntic glands
E. Parietal cells
21.53. A 46 year old woman with a history of
gallstones presents with upper abdominal pain 21.58. Dysbiosis within the_intestinal microbiota
and is diagnosed with acute pancreatitis. Which has been associated with all of t.he following
of these features at presentation would suggest except one. Which of these conditions is NOT
a worse prognosis? associated with dysbiosis?

downloaded from www.medicalbr.com


234 • GASTROENTEROLOGY

A.
B.
Basal cell carcinoma
Crohn's disease
21.63. A 24 year old man presents with fresh
haematemesis the day after a night of heavy
l
C. Depression alcohol consumption, during which he had a
D. Hepatocellular carcinoma prolonged period of vomiting. Which of the
E. Type 2 diabetes mellitus following statements is correct?
A. A Mallory-Weiss tear never causes significant
21.59. Which of the following is a bleeding
defence mechanism that is unique to the B. In the absence of melaena, upper
stomach? gastrointestinal endoscopy is not indicated
A. Hydrochloric acid secretion C. Results showing urea 6 mmoi/L (36 mg/dl),
B. Immunoglobulins haemoglobin 131 g/L and blood pressure
C. Macrophages 112/65 mmHg mean the patient can be
D. Peyer's patches safely discharged
E. T lymphocytes D. The likely diagnosis is oesophageal varices
E. The majority of cases like this heal
21.60. A female patient with rheumatoid arthritis completely within 2 weeks
presents with dysphagia and is referred for
oesophago-gastroduodenoscopy (OGD). 21.64. A 60 year old with a background of
However, the endoscopist would prefer a non-alcoholic steatohepatitis with cirrhosis
barium swallow as this patient's first presents with fresh haematemesis, melaena
investigation. Of which complication of OGD and collapse. Endoscopy reveals oesophageal
might the patient be particularly at risk given ~;arices as the origin of bleeding. Which of the
her history? following is an appropriate treatment?
A. Acute severe colitis A. Band ligation
B. Atlantoaxial subluxation B. Clip placement
C. Cardiac arrhythmias C. Heater probe coagulation
D. Respiratory distress D. Injection of adrenaline (epinephrine)
E. Small bowel perforation E. Laparotomy

21.61. A 60 year old man presents with a 21.65. A 40 year old man presents with
history of dysphagia. Which of the following diarrhoea, weight loss, night sweats and multiple
features would suggest that the problem enlarged lymph nodes. He has recently had a
originates in the oesophagus? vesicular rash on his right torso. Which one
A. A sticking sensation retrosternally in investigation would give a unifying diagnosis?
response to oral intake A. Coeliac serology
B. Altered voice B. Colonoscopy
C. Drooling C. Faecal calprotectin
D. Nasal regurgitation D. HIV test
E. Symptoms worse with liquids E. Thyroid function tests

21.62. A 29 year patient with type 1 diabetes on 21.66. An individual with coeliac disease is
insulin with poor glycaemic control has a admitted to the high dependency unit with a
6-month history of vomiting around 1 hour pneumococcal bacteraemia. What disease
following food. What is the most likely associated with coeliac disease is likely
diagnosis? responsible?
A. Gastric outlet obstruction A. Primary biliary cirrhosis
B. Gastroparesis B. Sarcoidosis
C. H. pylori infection C. Small bowel lymphoma
D. Medication-induced vomiting D. Splenic atrophy
E. Raised intracranial pressure E. Type 1 diabetes mellitus

downloaded from www.medicalbr.com


GASTROENTEROLOGY • 235

Answers
21.1. Answer: A. POEM is a new endoscopic technique that is
The incidence of mouth ulcers is not increased performed in very specialised units and has not
during pregnancy but it is higher in women replaced the more established treatment
prior to menstruation. Although mouth ulcers modalities of pneumatic dilatation, endoscopic
are very common in the general population (up botulinum toxin injection of the lower
to 30%), malignancy is relatively rare. Mouth oesophageal sphincter and surgery (Heller's
ulcers are not related to diverticulitis but they myotomy).
are common in inflammatory bowel diseases
such as Crohn's disease and ulcerative colitis. 21.4. Answer: C.
There is no evidence that antibiotic therapy is Hiatus hernia is very common in GORD and
particularly useful in the management of mouth its complications, and is implicated in the
ulcers but topical (and occasionally oral) pathogenesis of reflux. Symptoms are
glucocorticoids are useful. notoriously misleading in the diagnosis. Acid,
bile and pepsin all cause injury, although acid is
21.2. Answer: B. the main factor. The incidence is rising in most
Dysphagia and odynophagia should always be populations. GORD can occur in very lean
asked about in case there is oesophageal individuals, although it is more common in
candidiasis. There is no convincing evidence overweight and obese individuals.
that the oral contraceptive pill, particularly in
young females, is associated with oral thrush. 21.5. Answer: E.
First -line treatment is with nystatin or Barrett's oesophagus is a pre-malignant
amphotericin suspensions or lozenges. condition in which the normal squamous lining
Resistant cases (as likely in this case with of the lower oesophagus is replaced by
recurrent infection) or immunosuppressed columnar mucosa that may contain areas of
patients may require oral fluconazole. Antibiotic intestinal metaplasia. The relative risk of
use is associated with increased risk of oral oesophageal cancer is 40- to 120-fold
thrush. The diagnosis is largely clinical and increased but the absolute risk is low
treatment could be started on the basis of the (0.1-0.5% per year). Surveillance is expensive

-
history and examination alone. Mycological and cost -effectiveness studies have been
confirmation could be sought in brushings or conflicting, but it is currently recommended that
biopsies but is not usually necessary. patients with Barrett's oesophagus with

21.3. Answer: D.
intestinal metaplasia, but without dysplasia,
should undergo endoscopy at 3- to 5-yearly
.. I
In South America, infestation by the parasite intervals if the length of the Barrettic segment is
T. cruzi causes a condition (Chagas' disease) less than 3 ern and at 2- to 3-yearly intervals if
I
that is clinically indistinguishable from achalasia. the length is greater than 3 em. Those with
In Europe and North America, the cause of low-grade dysplasia should be endoscoped at
achalasia is largely unknown. The commonest 6-rnonthly intervals. Neither potent acid
presenting symptoms are dysphagia, suppression nor anti-reflux surgery stops
regurgitation (indigested food) and weight loss. progression or induces regression of Barrett's
Chest pain is uncommon and heartburn is not oesophagus, and thus treatment is only
a feature, as acid reflux does not occur against indicated for symptoms of reflux or
a closed sphincter. As such, patients do not complications, such as stricture.
develop Barrett's oesophagus. However,
achalasia is regarded as a pre-malignant 21.6. Answer: E.
condition with a small risk of oesophageal Elderly patients generally/have a higher
cancer (squamous cell and adenocarcinoma), prevalence of oesophageal motility disorders
usually after 20 years. The classic manometric that affect swallowing. In this case, she is very
findings of achalasia are failure of relaxation of likely to have been starte.d on bisphosphonates
the lower oesophageal sphincter on swallowing for her osteoporosis. Bisphosphonates cause
and absent or weak simultaneous contractions oesophageal ulceration and should be used
in the oesophageal body after swallowing. with caution in patients with known

downloaded from www.medicalbr.com


. 236 • GASTROENTEROLOGY

21.9. Answer: A.
I I
oesophageal disorders. Patients should be
clearly instructed to always take their dose with The majority of oesophageal cancers worldwide
a full glass of water on an empty stomach, and are squamous cell carcinomas, but the
to stand or sit upright for at least 30 minutes incidence of adenocarcinoma in Western
after taking the dose. Eosinophilic oesophagitis countries now exceeds that of squamous
is much more common in children. This patient carcinoma. Unfortunately, oesophageal cancer
is less likely to have a cancer as she retains presents late and 70% of patients present with
her appetite and general well-being. The extensive and inoperable disease. All of the
Plummer-Vinson syndrome is a rare disease listed risk factors are pre-malignant lesions but
characterised by dysphagia, iron deficiency Barrett's metaplasia is associated with the
anaemia, angular stomatitis, atrophic glossitis, development of adenocarcinoma, not
cheilosis and oesophageal webs. She had none squamous carcinoma. Squamous carcinoma
of these features on clinical examination. can occur in any part of the oesophagus, and
almost all tumours in the upper oesophagus
21.7. Answer: C. are squamous cancers. Adenocarcinomas
Long-term PPI therapy is associated with typically arise in the lower third of the
reduced absorption of iron, vitamin B1 2 and oesophagus from Barrett's oesophagus or from
magnesium. The drugs also predispose to the cardia of the stomach. Oesophageal cancer
enteric infections with Salmonella, is very aggressive, invading locally and
Campylobacter and possibly Clostridium difficile metastasising to local and distant sites quite
and have recently been shown to have an early.
undesirable impact on the composition of the
gut microbiota. Heller's myotomy and POEM 21.10. Answer: B.
are procedures for treatment of achalasia not His past medical history is not unusual and
GORD. In this young and healthy patient, does not preclude surgical intervention. The
medical therapy has clearly failed and he patient should undergo extensive staging with
requires laparoscopic anti-reflux surgery. Some thoracic and abdominal CT, often combined
calcium channel blockers relax the lower with positron emission tomography (CT-PET).
oesophageal sphincter and can cause reflux This will identify metastatic spread and local
and heartburn. invasion. Patients with resectable disease o.n
imaging should undergo endoscopic ultrasound
21.8. Answer: B. (EUS) to determine the depth of penetration of
The diagnosis here is strongly in favour of a the tumour into the oesophageal wall and to
malignant oesophageal stricture. With such a detect locoregional lymph node involvement.
presentation, he should be referred for an EUS is clearly not suitable for assessing distant
urgent upper Gl endoscopy with biopsies in the metastasis. The overall 5-year survival of
first instance. Barium swallow demonstrates the oesophageal cancer is very poor (< 15%).
site and length of the stricture but adds little Squamous carcinoma is sensitive to
useful information. Food bolus obstruction radiotherapy, unlike adenocarcinoma, although
presents acutely and is an endoscopic radiotherapy could be used to palliate
emergency. Lack of GORD symptoms is obstructing tumours of both varieties.
frequently noted in patients who present with
oesophageal adenocarcinoma and Barrett's 21.11. Answer: E.
oesophagus, although GORD is a strong risk H. pylori is usually acquired during childhood;
factor for both. In this case, the combination of acquisition in adults is rare. Dyspeptic patients
lack of GORD symptoms, heavy smoking and have a higher prevalence of H. pylori compared
alcohol consumption favour a diagnosis of to asymptomatic subjects but many patients
oesophageal squamous cell cancer. with dyspepsia do not have the infection.
Boerhaave's syndrome is spontaneous Around 90% of duodenal ulcer patients and
oesophageal perforation that results from 70% of gastric ulcer patients are infected with
forceful vomiting and retching. In this case, the H. pylori. The remaining 30% of gastric ulcers
fits of coughing on swallowing are likely caused are caused by non-steroidal anti-inflammatory
by a fistula between the oesophagus and the drugs (NSAIDs)/aspirin. There are also more
trachea or bronchial tree. Fistulation can also rare causes of ulcers such as Zollinger-EIIison
lead to pneumonia and pleural effusion. syndrome and gastroduodenal Crohn's

downloaded from www.medicalbr.com


GASTROENTEROLOGY • 237

disease. Half of the world's population is perforation. Once a peptic ulcer forms, it is
infected with H pylori and the majority are more likely to cause complications and less
asymptomatic. The hallmark of H pylori likely to heal if the patient continues to smoke.
infection is the induction of gastritis, which is There is no specific dietary advice and citrus
the pathognomonic histological consequence of fruits have no relevance in this situation.
the infection. This histological gastritis may or
may not be endoscopically visible and may or 21.15. Answer: D.
may not cause symptoms. The rate of success of treatment is dependent
on several factors, including medication
21.12. Answer: E. adherence and antibiotic resistance. The most
These features are very consistent with important is resistance to clarithromycin.
Menetrier's disease, which is a rare condition of Amoxicillin resistance is very rare. Treatment is
unknown aetiology characterised by excessive based on a PPI taken simultaneously with two
production of transforming growth factor-alpha antibiotics (from amoxicillin, clarithromycin and
(TGF-a). As a result, the mucosal folds of the metronidazole) for at least 7 days. High-dose,
body and fundus are greatly enlarged. Whilst twice-daily PPI therapy increases efficacy of
some patients have upper gastrointestinal treatment, as does extending treatment to
symptoms, the majority present in middle or old 10-14 days. Metronidazole is still useful in
age with protein-losing enteropathy due to eradication regimens because in vitro
exudation from the gastric mucosa. Endoscopy resistance to the antibiotic could be overcome
shows enlarged, nodular and coarse folds. in· vivo by combination with other effective
Crohn's disease of the stomach usually antibiotics, especially clarithromycin. If the
presents with deep ulcers. Hyperthyroidism is first -line therapy fails, the same course should
associated with gastrointestinal symptoms but not be repeated but quadruple therapy,
does not usually cause any endoscopic consisting of a PPI, bismuth subcitrate,
feature,s. Cronkhite-Canada syndrome may metronidazole and tetracycline (OBMT) for
present with generalised gastrointestinal polyps, 10-14 days, is recommended. Erythromycin is
cutaneous pigmentation, alopecia and never used in eradication regimens but
onychodystrophy. NSAID gastropathy presents clarithromycin, another macrolide, is one of the
with gastritis, erosions or single/multiple most effective in use.
superficial ulcers.
21.16. Answer: D.
21.13. Answer: E. She has a high urea but her creatinine level is
This patient clearly requires urgent upper Gl within the upper range of normal for women.
endoscopy to rule out significant pathology, NSAIDs cause renal damage but the most
particularly gastric neoplasia. The absence of serious abnormality in this case is the
weight loss and persistent vomiting is suspected Gl bleed. A silent myocardial
reassuring but the presence of anaemia is infarction is a possible complication of an acute
alarming and should trigger urgent referral for and significant Gl bleed but it is not the most
endoscopy. Barium meal is rarely used. The likely diagnosis here. NSAIDs cause ulceration
other options are inappropriate because urgent throughout the Gl tract but lesions that bleed
endoscopy is mandatory in this situation and are most likely in the upper part, including
over-rides the other suggestions. gastric and duodenal ulcers. Melaena (black
tarry stool) is characteristic of an upper Gl
21.14. Answer: A. bleed. Lower Gl bleeds present with fresh
Gastric ulcers may occasionally be malignant blood. Dieulafoy lesions are rare causes of
and therefore must always be biopsied and upper Gl bleeding and are caused by a single
followed up to ensure healing. He should, of tortuous small artery in the submucosa that
course, have antral biopsies to check for H may erode through the mucosa and cause
pylori and this should be eradicated if positive, significant bleeding. These lesions are
but he should also have the ulcer edge diagnosed endoscopically.
biopsied. Surgery is no longer an option in the
management of peptic ulcer 'disease unless 21.17. Answer: B.
there are severe complications such as gastric This patient is clearly haemodynamically
outlet obstruction, uncontrollable bleeding or compromised and in shock. The most

downloaded from www.medicalbr.com


238 • GASTROENTEROLOGY

immediate management should focus


on resuscitation. Upper Gl endoscopy
21.21. Answer: E.
The stomach is the most common site for
1
should not be carried out before extranodal non-Hodgkin lymphoma and 60% of
resuscitation. all primary gastrointestinal lymphomas occur at
this site. H. pylori infection is closely associated
21.18. Answer: C. with the development of a low-grade lymphoma
The incidence of gastric cancer has, in fact, (classified as extranodal marginal-zone
been falling steadily across the world, although lymphomas of mucosa-associated lymphoid
it is still a global killer. H. pylori infection is the tissue (MALT) type). EUS plays an important
most important risk factor and this far role in staging these lesions by accurately
outweighs all the other traditional risk factors, defining the depth of invasion into the gastric
such as diet. H. pylori plays a key pathogenic wall. High-grade B-cell lymphomas should be
role and the infection has been classified by the treated by a combination of rituximab,
International Agency for Research on Cancer chemotherapy, surgery and radiotherapy. While
(IARC) as a definite human carcinogen. Despite initial treatment of low-grade lesions confined to
advances in treatment, the overall 5-year the superficial layers of the gastric wall consists
survival is still low {<30%). Africa has a low of H. pylori eradication and close observation,
incidence of gastric cancer (the so-called 25% contain t(11 : 18) chromosomal
African enigma) despite having high rates of H. translocations. In these cases, additional
pylori infection. radiotherapy or chemotherapy is usually
necessary.
21.19. Answer: D.
Virtually all gastric tumours are adenocarcinomas 21.22. Answer: A.
arising from mucus-secreting cells in the base of GISTs arise from the interstitial cells of Cajal.
the gastric crypts. Gastric cancers are very They are differentiated from other mesenchymal
aggressive and usually present quite late tumours by expression of the c-kit
with large sizes and with distant spread. While proto-oncogene. They are usually benign
most gastric cancers are initiated by H. pylori tumours, particularly the smaller lesions <2 em,
infection, the organism often disappears in the and asymptomatic, but the larger ones may
latter stages with onset of gastric atrophy have malignant potential. Very large lesions
and achlorhydria. Krukenberg tumours are should be treated pre-operatively with imatinib
metastatic deposits on the ovaries. Sister to reduce their size and make surgery easier.
Joseph's nodule is a metastatic deposit in the lmatinib can also be used for prolonged control
umbilicus usually from a cancer in the pelvis or of metastatic GISTs. They can bleed
abdomen. independently of NSAIDs.

21.20. Answer: D. 21.23. Answer: B.


Endoscopic management of gastric cancer is Given the duration of the symptoms, this is not
confined to very early mucosal or submucosal likely to be infection. X-ray is not likely to aid
disease with no spread elsewhere. For the diagnosis and exposes the patient to ionising
majority of patients with locally advanced radiation. Given the constellation of symptoms,
disease, total gastrectomy with mild anaemia and low folate in a patient of this
lymphadenectomy is the operation of choice. In age, coeliac disease should be considered
the surgical management of proximal gastric initially and therefore coeliac serology is the
cancers involving the oesophago-gastric next best investigation.
junction, distal oesophagectomy is also
required. The biological agent trastuzumab may 21.24. Answer: D.
benefit some patients whose tumours Abdominal TB is a common cause of these
over-express HER2. In patients with inoperable symptoms in the Indian subcontinent and other
tumours, survival can be improved and areas of the world where TB is endemic. In the
palliation of symptoms achieved with UK/US, Grahn's disease would be the most
chemotherapy using 5-fluorouracil and likely diagnosis. Grar:.1ulomatous hepatitis
cisplatin, ECF (epirubicin, cisplatin resulting in cholestatic liver function tests can
and fluorouracil) or other platinum- and be present in TB. Despite the diagnosis, chest
taxane-based regimens. X-ray can be normal.

downloaded from www.medicalbr.com


GASTROENTEROLOGY • 239

21.25. Answer: A. disease, as well as anti-tumour necrosis factor


Anti-tTG response is lgA mediated and (TNF) biological therapy. Methotrexate would
therefore can be false negative in lgA not be used here given toxicities that include
deficiency. Therefore, lgG-mediated antibodies teratogenicity in women of child-bearing age.
should be checked next. If positive, the patient Thiopurine immunosuppression with
should be offered an endoscopy for distal azathioprine is the next most appropriate
duodenal/duodenal bulb biopsies. HLA status is medication to prevent a further flare in
not exclusive and is not a reliable diagnostic disease.
test, although it can aid diagnosis in some
cases where serology/histology are equivocal. 21.31. Answer: B.
Inflammatory bowel disease {IBD) is a
21.26. Answer: B. multisystemic disorder with extra-intestinal
The pathophysiology of coeliac disease is manifestations (Fig. 21.31). Psoriasis is not
multifactorial. HLA status is associated but not usually associated with IBD but can be
exclusive. There is no evidence that H. pylori is associated with anti-TNF biological therapy.
linked to coeliac disease. Ocular manifestations are episcleritis and
anterior uveitis. Blepharitis is not associated
21.27. Answer: A. with IBD. Pyoderma gangrenosum and renal
These symptoms and the presence of small calculi are associated with Grahn's disease
bowel foamy macrophages histologically lead to rather than ulcerative colitis. Ulcerative colitis
a diagnosis of Whipple's disease. The with primary sclerosing cholangitis is associated
appropriate treatment is extended antibiotic with high risk of colon cancer and these
treatment with 2 weeks of intravenous patients require annual surveillance
ceftriaxone, then oral antibiotics for 1 year. The colonoscopy.
other treatments discussed would not be
appropriate.

·~~'"""""""'""'''""."'~
~
Unrelaledtolnftamm"'oty
lnflammat~ .. bowe]dlsenoeactlvily
21.28. Answer: E. ~~~

Conjun~m:
-;;'k
<
The diagnosis is radiation proctitis. This Eplscl..rtll:3~
Mouthulce!S {- - _,, __ _
Jwloimmunehepatitls

commonly occurs after radiotherapy for PrirnruyscJemsU.gcho\ang~is


andcholangloCllfCinoma
Fa~ liver (ulceralovecol~is)
prostate cancer. The aetiology is chronic Uv~abscesslporlalpyaemlo
Amyloldosisandcxalalecai<>JII
ischaemia rather than inflammation and
therefore steroid/anti-inflammatory treatments Mesenlericorporlal~rt~lnlhlombosis

~I
are ineffective. Endoscopic argon plasma
coagulation (APC) can be used for acute
bleeding but can increase risk of fistula
large·joinlarlhrilis
formation with repeated use. Evidence
suggests sucralfate enema is the best Erylhemanodosum

Pyodermogangrenosum
treatment and, if this fails, hyperbaric oxygen
therapy should be considered.
Fig. 21.31 Systemic complications of inflammatory bowel
disease.
21.29. Answer: D.
All these investigations can be considered in
this patient. Given the colour and consistency
of stool and previous cholecystectomy, the 21.32. Answer: B.
most likely diagnosis is bile acid malabsorption; Intravenous glucocorticoid therapy is indicated
therefore a radionuclide SeHCAT scan is the in this case of acute severe ulcerative colitis
correct answer in this scenario. (>6 bloody stools/24 hrs, raised CRP). Current
guidelines suggest revie';Y' at day 3 and then
21.30. Answer: C. day 5 for clinical and biochemical response. If
There is no evidence for use of antibiotics. non-response, rescue medical therapies with
Immunoglobulin is used for chronic Giardia anti-TNF biological therapy or ciclosporin can
infection associated with immunoglobulin be considered. Alternatively, nQn-response to
deficiency. Options B, C and E (all intravenous {IV) steroid therapy ± rescue
immunosuppressants) can be used in Grahn's medical therapies require a surgical review for

downloaded from www.medicalbr.com


240 • GASTROENTEROLOGY

consideration of subtotal colectomy and from these cells. Grahn's disease can affect the
ileostomy. LMWH is required as these patients whole gastrointestinal tract and is associated
are at increased risk of thrombosis. Loperamide with a predominant Th1 adaptive response and
may precipitate toxic megacolon. There is no ulcerative colitis is associated with a Th2
evidence for antibiotic therapy. Vedolizumab response. There is ongoing research into the
(anti-a4~7 integrin biological therapy) can be pathogenesis of IBD, including alterations in the
considered for maintenance treatment of IBD; oral microbiome and the role of specific
it has a long onset of action and therefore has bacteria, including E. coli.
no role in the management of acute severe
colitis. 21.36. Answer: B.
The distribution and histology features of the
21.33. Answer: B. inflammation are in keeping with ulcerative
A complication of acute severe IBD is colitis. In mild disease, a high-dose oral 5-ASA
thrombosis. This presentation and clinical is appropriate first-line therapy, reducing to a
status suggest a pulmonary embolus and maintenance dose when symptoms are under
therefore CTPA is most likely to lead to control. If symptoms do not settle, then
the diagnosis. Whilst some of the other addition of topical (enema) treatment with either
investigations may be appropriate in the acute 5-ASA or steroid, or a course of oral
scenario, they will not necessarily lead to the glucocorticoids, can be considered. Anti-TNF
diagnosis. therapy is a treatment for moderate to severe
ulcerative colitis, refractory to other
21.34. Answer: C. i;nmunosuppression including thiopurines (such
Regular blood monitoring is required to assess as azathioprine). Clinical trial data suggest that
for bone marrow suppression and liver ustekinumab may be a new treatment choice in
dysfunction. The interval between blood severe Grahn's disease but it is still in
analysis increases as time passes from development and is not yet used as a standard
induction. Very rarely, azathioprine can cause of care.
acute pancreatitis; amylase should be tested
if a patient presents with acute abdominal 21.37. Answer: E.
pain and vomiting soon after starting this This patient's symptoms are consistent with
medication. If a patient is stable on active small bowel Grahn's disease with
azathioprine, advice is to continue throughout subacute obstruction precipitated by eating and
pregnancy, as risk of teratogenicity is very low he needs investigation to assess disease
and a greater risk is uncontrolled IBD. activity and presence of stricturing. MRI
The side-effects of thiopurines include a enterography is a sensitive modality to assess
small increased risk of malignancy with mucosal inflammation and calibre of small
extended use, particularly lymphoma and bowel lumen. A small bowel barium meal and
non-melanoma skin cancer. A chest X-ray and follow through will also show this information
hepatitis serology should be checked prior to but is less sensitive than MRI and exposes this
anti-TNF biological therapy to avoid reactivation young patient to ionising radiation. A capsule
of latent TB or worsening of hepatitis B/C enteroscopy is contraindicated here, given
infection. suspicion of stricturing with risk of impaction.
Small bowel disease is suspected in this case
21.35. Answer: E. and a barium enema would assess the colon. If
The pathogenesis of IBD is complex and colonic investigation is required, a colonoscopy
involves breakdown of the epithelial barrier and would be first choice in a patient of this age.
disordered mucosal immune responses, Stool calprotectin would likely be high, in
associated with microbial dysbiosis in keeping with intestina,l inflammation, but would
genetically susceptible individuals. not give additional jriformation on structural
Genome-wide association studies have aspects of disease activity.
identified multiple polymorphisms in genes,
including cytokine genes, but there. is no one 21.38. Answer: D. •
somatic mutation driver gene identified. The This woman is likely to have damage to the
immune response involves both innate and pelvic floor/anal sphincter from previous
adaptive aspects and the cytokines secreted childbirth. Exercises to strengthen the pelvic

downloaded from www.medicalbr.com


GASTROENTEROLOGY • 241

floor along with biofeedback techniques to financial income to support her child and this
regulate bowel-opening pattern is the first may be impacting on these symptoms. In a
treatment to try. If unsuccessful, sacral nerve patient of this age, the main diagnostic
stimulators can be considered and some considerations are IBS and lBO. CRP and
patients require defunctioning colostomy. Botox platelet count may be elevated in the latter
injection and topical diltiazern cream are used but not always. A stool calprotectin is a
for relaxation of the anal sphincter to treat anal more sensitive marker of gastrointestinal
fissure. inflammation in this scenario to guide need for
further investigation and appropriate treatment.
21.39. Answer: B. Coeliac serology is negative, so there is
In a patient of this age with 'red flag' symptoms no indication for proceeding tci duodenal
of change in bowel-opening habit to loose stool biopsy.
and iron deficiency anaemia, the suspicion is
colorectal cancer and therefore colonoscopy is 21.43. Answer: E.
the best next investigation. CT scan will detect There is no evidence for the use of
large mass lesions and metastatic disease but benzodiazepines for IBS. An appropriate
is not sensitive for smaller localised mucosal next-line treatment would be a low-dose
lesions. Colonoscopy allows biopsies to be nocturnal tricyclic antidepressant such as
taken for histological analysis. amitriptyline. There is no evidence for the use
of probiotics. Peppermint capsules can often
21.40. Answer: B. help with bloating and flatus in IBS patients.
Most colorectal cancer is sporadic with no Prucalopride is used as a treatment in
identifiable genetic predisposition. Most constipation-predominant IBS that has
sporadic colorectal cancer arises from a benign failed to respond to laxative therapy. There
pre-malignant adenoma (polyp). Aetiology is is emerging evidence that a low-FODMAP
multifactorial, involving genetic and diet is effective in the treatment of IBS. This
environmental risk factors. The genetic risk should be supervised by a dietician and is
factors include acquired mutations of somatic initially very restrictive, with gradual
genes such as APC, TP53 and SMAD4, and reintroduction of food groups dependent on
these tend to occur sequentially over time, with symptom response.
APC mutations an early feature and TP53 a late

-
feature. Epigenetic changes can also lead to 21.44. Answer: C.
altered gene expression. These genetic Protein, fat and folic acid absorption occur in
changes are not mutually exclusive - not all the small bowel. The colonic luminal pH is
individuals with these mutations develop
colorectal cancer and not all colorectal
mildly acidic to neutral (5.5-7). The colonic
rnicrobiota is acquired at birth, matures to that
. !
tumours will contain all of these genetic
mutations. Approximately 5% of colorectal
of an adult by the age of 3 years and changes
over time as a natural part of ageing in the
I
cancers are attributable to genetic syndromes, elderly years. It can also change in relation to
the most common being FAP and environmental alterations such as use of
HNPCC. antibiotics. The colon acts to absorb water and
electrolytes from stool.
21.41. Answer: C.
Often patients with bacterial overgrowth are 21.45. Answer: E.
anaemic with a macrocytosis •and low vitamin The pancreatic body and tail receive supply
8,2 due to bacterial utilisation of the vitamin B 12 from the splenic artery derived from the coeliac
in the gastrointestinal lumen.· The other answers artery. The head of pancreas received supply
are not findings you would expect with this from the inferior pancreaticoduodenal artery
diagnosis. derived from the superio.~ mesenteric artery
(SMA). The SMA also supplies the ileum,
21.42. Answer: D. alongside jejunum, ascending and transverse
The history is consistent with irritable bowel colon. The left kidney is :;jupplied by the renal
syndrome (IBS). It is important to explore social artery, ovaries by the gonadal artery, the
history. She is experiencing stress in her life sigmoid colon by the inferior mesenteric artery
with regard to employment and worry of loss of and the psoas muscle by the lumbar arteries.

downloaded from www.medicalbr.com


,..
I

242 • GASTROENTEROLOGY

21.46. Answer C. 21.50. Answer: E.


This man gives a history of steatorrhoea, due to All the options listed are recognised
high lipid contents in his stools. Pancreatic complications of ERCP, but an extremely
lipase, in the presence of its co-factor, high amylase would point to pancreatitis.
colipase, cleaves long-chain triglycerides, The complication rate following ERCP is
yielding fatty acids and monoglycerides, which 5-10% with a 30-day mortality of 0.5-1 %,
are small enough to diffuse across enterocyte depending on the procedure complexity,
cell membranes. In chronic pancreatitis, underlying condition and comorbidities·.
inadequate pancreatic lipase production leads Acute pancreatitis is the most common
to high fat content in stools due to poor complication.
digestion. Chymotrypsin may be deficient as
well, but leads to protein deficiencies. Glucagon 21.51. Answer: B.
and somatostatin are endocrine hormones This man had alcohol-related acute pancreatitis
secreted by the pancreas and for unclear and has developed a pancreatic pseudocyst.
reasons tend not to be associated with Large pancreatic pseudocysts can cause
deficiency in chronic pancreatitis. Maltase compression of surrounding structures
is an enzyme found on small intestine brush and in this case gastric outlet obstruction.
border. Endoscopy, barium and water-soluble contrast
meals will all demonstrate dilatation of the
21.47. Answer: D. stomach, but not the cause. Oesophageal
In recurrent duodenal ulceration in a young manometry would be expected to be normal in
patient, Zollinger-EIIison syndrome should be this case.
considered. Gastrinoma as part of multiple
endocrine neoplasia (MEN) type 1 is a 21.52. Answer: A.
recognised cause of Zollinger-EIIison syndrome, The diagnosis here is that of cystic
of which pituitary adenoma is a common fibrosis, of which chronic pancreatitis is a
additional tumour. Phaechromocytoma and common complication. Calcification of the
medullary thyroid cancer are associated with pancreas is highly suggestive of chronic
MEN type 2 (not type 1) and Marfan's pancreatitis.
syndrome, so are not linked to gastrinoma.
21.53. Answer: E.
21.48. Answer C. Significant hypoxia, low serum calcium,
This man gives a history of acute pancreatitis, elevated serum glucose and very high CRP
probably secondary to alcohol, and now over 1 50 mg/L are associated with a poor
presents with ascites. His liver function tests prognosis in acute pancreatitis. Amylase level
show a normal liver synthetic function (bilirubin alone has not been proven to be prognostic.
and INR), which makes portal hypertension due Other adverse factors are given in Boxes
to chronic liver disease an unlikely cause of the 21.53A and 21.538.
ascites. Ruptured pancreatic pseudocyst
is a recognised complication of acute
pancreatitis and leads to an elevated ascitic
fluid amylase.

21.49. Answer: A. 21.53A Glasgow criteria for prognosis in acute


In a normal pancreas the pancreatic duct pancreatitis*
meets the common bile duct at the ampulla of Age > 55 years
Vater, where it enters the duodenum. The PO, < 8 kPa (60 mmHg)
pancreatic duct does not communicate White blood cell count> 15 x 109/L
Albumin < 32 g/L (3.2 g/pl)
directly with the liver, hepatic portal vein or
Serum calcium < 2 mmoi/L (8 mg/dl) (corrected)
gallbladder in a normal pancreas. In pancreas Glucose > 1o mmoi/L (180 mg/dL)
divisum, two pancreatic ducts remain present, Urea> 16 mmoi/L (45 mg/dl) (after rehydration)
the dominant dorsal duct draining directly Alanine aminotransfera~e > 200 U/L
Lactate dehydrogenase > 600 U/L
into the duodenum rather than joining the
common bile duct at the sphincter of Oddi, *Severity and prognosis worsen as the number of these
bypassing this. factors increases. More than three implies severe disease.

downloaded from www.medicalbr.com


GASTROENTEROLOGY • 243

c: 21.56. Answer: E.
21.53B Features that predict severe pancreatitis
In any young female, it is essential to consider
Initial assessment
pregnancy as a cause of abdominal symptoms
Clinical impression of severity
or vomiting. This will dictate what further
Body mass index > 30 kg/m 2
Pleural effusion on chest X-ray investigations and medications can be given
APACHE II score > 8 safely.
24 hours after admission
Clinical impression of severity 21.57. Answer: A.
APACHE II score > 8 The removal of a large portion of the stomach
Glasgow score > 3
Persisting organ failure, especially if multiple leads to a significant reduction in chief cells
CRP > 150 mg/L responsible for production of intrinsic factor,
48 hours after admission required to absorb vitamin B12 in the terminal
Clinical impression of severity ileum. They also have a role in pepsinogen
Glasgow score > 3 production, although protein absorption tends
CRP > 150 mg/L
to be affected less.
Persisting organ failure for 48 hours
Multiple or progressive organ failure
21.58. Answer: A.
(CRP = C-reactive protein)
Basal cell carcinoma has not been
demonstrated to be associated with the
intestinal microbiome. All the others have been
shown to be associated with dysbiosis.

21.54. Answer: D. 21.59: Answer: A.


lgG4 is elevated in autoimmune pancreatitis, Hydrochloric acid secretion is unique to the
which responds well to glucocorticoids. stomach. The others' main contributions occur
Autoimmune pancreatitis often mimics in the small intestine.
pancreatic cancers with an elevated bilirubin,
but bilirubin is not specific to the disease as it 21.60. Answer: B.
would be elevated in the main differential Rheumatoid arthritis may be associated with
diagnosis of carcinoma of head of pancreas. atlantoaxial subluxation due to flexion of the
Alpha-fetoprotein is elevated in hepatocellular neck during endoscopy. This can lead to high
carcinoma, germ cell tumours and pregnancy, spinal cord injury. Rheumatoid arthritis is the
none of which would be expected to respond most common cause of this complication in
to glucocorticoids. Hepatitis E lgM is elevated adults.
in acute infection and is not steroid responsive.
Serum lgM levels are not affected by 21.61. Answer: A.
autoimmune hepatitis. A sticking sensation retrostemally suggests an
oesophageal problem. All other features listed
21.55. Answer: D. suggest oropharygeal origin.
If there is recurrent duodenal ulceration in a
young patient, Zollinger-EIIison syndrome 21.62. Answer: B.
should be considered. Gastrinoma as part of Gastroparesis is an autonomic complication of
multiple endocrine neoplasia (MEN) type 1 is a poorly controlled diabetes, which presents with
recognised cause of Zollinger-EIIison syndrome, persistent vomiting. Symptoms may improve
of which pituitary adenoma is a common after a period of sustained improved glycaemic
additional tumour. MEN1 is a tumour control. There are no features in the history that
suppressor gene, mutations of which are seen suggest H. pylori infection or a cause for
in MEN type 1. Mutations in BRCA 1 and gastric outlet obstruction, e.g. ulcers, nor raised
BRCA2 are associated with breast cancer. intracranial pressure (positional headache worse
RET is a proto-oncogene, where mutations on lying flat). Insulin is not associated with
to increase activity lead to MEN types 2 vomiting as a side-effect.
and 3 (also known as MEN types 2a and 2b).
JAK2 mutations are associated with 21.63. Answer: C.
polycythaemia vera and myeloproliferative The Blatchford scoring system (Box 21.63) risk
disorders. stratifies upper gastrointestinal bleeding

downloaded from www.medicalbr.com


244 • GASTROENTEROLOGY

according to clinical features, initial blood tests 21.64. Answer: A.


and comorbidities, giving a score of 0 in this Band ligation is the endoscopic management of
case. This history is typical of a Mallory-Weiss choice for bleeding oesophageal varices.
tear. In the presence of a low Blatchford score, Options B-E are used in the management of
Mallory-Weiss tear can often be managed bleeding peptic ulcers or mucosal defects, but
without endoscopy. Whilst the majority of cases are likely to worsen bleeding secondary to
stop bleeding without intervention, those with varices.
elevated Blatchford scores and evidence of
ongoing bleeding require endoscopy and 21.65. Answer: D.
around 10% require endoscopic therapy due to Diarrhoea is one of the most common
life-threatening bleeding. symptoms associated with HIV and should be
considered in all individuals with unexplained
symptoms. A recent opportunistic herpes
zoster infection, alongside his other symptoms,
would support HIV as the unifying diagnosis.
21.63 Modified Blatchford score: risk
stratification in acute upper gastrointestinal 21.66. Answer: D.
bleeding
All these conditions are associated with coeliac
Score component disease (Box 21.66). However, splenic atrophy
Admission risk marker value
leads to a functional hyposplenism and leaves
Blood urea
the individual at risk of encapsulated organisms
~25 mmol/l (70 mg/dl) 6
10-25 mmol/l (28-70 mg/dl)
such as Streptococcus pneumoniae.
4
8-1 0 mmol/l (22.4-28 mg/dl) 3 Consequently, vaccination is now routinely
6.5-8 mmol/l (18.2-22.4 mg/dl) 2 recommended.
<6.5 mmol/l (18.2 mg/dl) 0
Haemoglobin for men 21.66 Disease associations of coeliac disease
< 100 gil (1 0 g/dl) 6
100-119 gil (1 0-11.9 g/dl) Type 1 diabetes mellitus (2-8%)
3
Thyroid disease (5%)
120-129 gil (12-12.9 g/dl) 1
Primary biliary cirrho_sis (3%)
~ 130 gil (13 g/dl) 0 Sjogren's syndrome (3%)
Haemoglobin for women Immunoglobulin A deficiency (2%)
< 100 gil (1 0 g/dl) 6 Pernicious anaemia
100-119 gil (10-11.9 g/dl) 1 Sarcoidosis
~ 120 gil (12 g/dl) 0 Neurological complications:
Encephalopathy
Systolic blood pressure
Cerebellar atrophy
<90 mmHg 3
Peripheral neuropathy
90-99 mmHg 2 Epilepsy
100-109 mmHg 1 Myasthenia gravis
>109 mmHg 0 Dermatitis herpetiformis
Other markers Down's syndrome
Presentation with syncope 2 Enteropathy-associated T-cell lymphoma
Small bowel carcinoma
Hepatic disease 2
Squamous carcinoma of oesophagus
Cardiac failure 2
Ulcerative jejunitis
Pulse ~ 100 beats/min 1 Pancreatic insufficiency
Presentation with melaena 1 Microscopic colitis
None of the above 0 Splenic atrophy

downloaded from www.medicalbr.com


DEJ Jones, QM Anstee,
J Hansi

Hepatology

Multiple Choice Questions


22.1. Which of the following is correct about the regarding the clinical presentation and progress
anatomy of the liver? of paracetamol overdose is true?
A. A fibrous capsule surrounds the liver lobule A. Deterioration from grade 1 to grade 4
in humans encephalopathy typically takes place over
B. The hepatic vein radicles accompany the several days
arteries in the portal tract B. It has a worse outcome than acute liver
C. The liver has significant regenerative capacity failure of other aetiologies
D. The liver is on the left side of the upper C. If the patient recovers from acute liver failure,
abdomen crossing the midline cirrhosis development is almost inevitable
E. The liver weighs around 0.75-1 kg in typical D. Jaundice typically occurs before prothrombin
adults time (PT) prolongation
E. Pancreatitis can be an accompanying
22.2. Which of the following is a marker of complication
the function of the liver that is in widespread
clinical use? 22.5. In the course of a 'well-woman' check
A. Alanine transaminase offered by her employer, a 50 year old woman
B. Fibroscan was found to have an antinuclear antibody with
C. Monoethylglycinexylidide (MEGX) test a multiple nuclear dot pattern and mildly
D. Platelet count abnormal liver function tests (LFTs). What is the
E. Prothrombin time most likely diagnosis?
A. Autoimmune hepatitis (AIH)
22.3. A 63 year old man presents with yellowing B. Lupus
of the skin and sclerae, dark urine and pale C. Non-alcoholic steatohepatitis (NASH)
stools. He has no significant past medical D. Primary biliary cirrhosis (PBC)
history, other than appendicectomy as a E. Primary sclerosing cholangitis (PSC)
teenager. What is the most likely diagnosis?
A. Autoimmune hepatitis 22.6. A 70 year old woman is admitted to
B. Gilbert's syndrome hospital and develops abnormal LFTs. She
C. Haemolysis has been on several different medications over
D. Hypothyroidism the last few days. Which of the following
E. Pancreatic carcinoma statements about drug-ihduced liver injury (DILl)
is true?
22.4. An 18 year old woman presents to the A. Amoxicillin is a commpn cause of acute liver
emergency department having taken an injury
overdose of an unknown quantity of B. DILl can be difficult to distinguish clinically
paracetamol. Which of the following statements from autoimmune hepatitis

downloaded from www.medicalbr.com


r
246 • HEPATOLOGY

found to be over 10000 U/L. What is the most


C. In unexplained jaundice, recently started
medications can be safely continued likely diagnosis?
provided there have been no reports of DILl A. Alcoholic hepatitis
associated with them B. Hepatitis 8 virus
D. It is seen with opiate use C. Hepatitis C virus
E. The risk of co-amoxiclav-induced liver injury D. Hepatitis E virus
goes down with each repeat exposure E. Wilson's disease

22.7. A 35 year old man is being considered 22.11. A 25 year old wornan with
for liver transplantation. Which of the following well-controlled, non-cirrhotic AIH attends your
is true? clinic to say she is pregnant. She is currently
A. Acute cellular rejection most commonly maintained on azathioprine monotherapy. What
occurs between days 2 and 5 advice would you give her?
B. At least partial human leucocyte antigen A. Disease flare-ups can occur following
(HLA) matching is essential for a good delivery
outcome B. Her child runs a significant risk of developing
C. Hepatorenal failure will typically improve AIH
following liver transplantation C. Her disease will deteriorate during pregnancy
D. Immunosuppression can be safely stopped D. She should immediately swap to MMF
in most patients at 5 years maintenance therapy
E. Mycophenolate mofetil (MMF) monotherapy E. She should undergo endoscopy
is a useful immunosuppression regime
22.12. A 53 year old man with known
22.8. A 42 year old man is in hospital with acute oesophageal varies presents with a large
liver failure. The team are considering his gastrointestinal (GI) bleed. You are the first
prognosis. In this situation, which of the attending clinician. What is the first step you
following liver functions, when deranged, has should take?
an important impact on outcome? A. Alert interventional radiology in case
A. Glucose regulation transjugular intrahepatic portosystemic stent
B. Innate immune response shunt (TIPSS) is required
C. Oxalate metabolism B. Arrange urgent cross-match
D. Red cell breakdown C. Arrange urgent endoscopy and banding
E. Steroid hormone clearance D. Insert large-bore cannula and give fluid
E. Organise bedside ultrasound to assess for
22.9. A 28 year old woman presents to her portal vein thrombosis
family physician with fatigue and itch. She has
obstructive LFTs. Which of the following is true 22.13. A patient with suspected variceal
about the autoimmune cholestatic liver disease bleeding cannot have an endoscopy because
primary biliary cholangitis? of lack of an available trained endoscopist. She
A. It is a different condition to primary biliary is becoming increasingly unstable. Which of
the following is a medical therapy appropriate
cirrhosis
B. It is commoner in men than in women for use in the first instance to establish
C. Patients usually show elevation of PT haemodynamic control?
D. The disease is more aggressive in younger A. Glypressin
patients B. Noradrenaline (norephinephrine)
E. Ursodeoxycholic acid (UDCA) is first-line C. Propranolol
therapy and should be used once the patient D. Subcutaneous octreotide
is symptomatic E. Tranexarnic acid'

22.10. A 20 year old student with no significant 22.14. A computed tomography (CT) scan
past medical history presents with a 1-week performed in a patient with chronic liver disease
history of acute lethargy and jaundice 2 weeks has identified a mass lesion suspicious of a
after returning from a week-long holiday in hepatocellular carcinoma. What is the next
Turkey. His alanine aminotransferase (AL1) is investigation you should consider?

downloaded from www.medicalbr.com


HEPATOLOGY • 247

A. A magnetic resonance imaging (MRI) scan suspected that he has primary sclerosing
B. Blood alpha-fetoprotein (AFP) measurement cholangitis. Which of the following statements
G. Laparoscopy about diagnosis and treatment is correct?
D. Liver biopsy A. First-line imaging technique for the bile ducts
E. Positron emission tomography (PEl} scan is endoscopic retrograde
cholangiopancreatography (ERCP)
22.15. A 45 year old woman presents with B. HCC is the characteristic complication
painful hepatomegaly and ascites. What G. The diagnosis is unlikely as the patient is
imaging findings would you predict when male
investigating? D. There is no proven therapy able to improve
A. An irregular liver on CT prognosis
B. Hepatic artery thrombosis E. UDCA is proven to reduce mortality
G. Hepatic venous thrombosis on triple-phase
CT 22.20. A 60 year old woman is being treated for
D. Isolated gastric varies cellulitis and has developed abnormal LFTs.
E. Reversal of portal blood flow on ultrasound Her family physician calls you for advice as she
suspects that the patient has flucloxacillin-
22.16. Twenty-four hours following liver induced liver injury. Which of the following
transplantation for autoimmune hepatitis, a statements is correct?
patient's ALT is climbing rapidly. What A. Characteristic blood test abnormalities are
diagnosis is the most likely? elevation in ALT and eosinophil count
A. Acute cellular rejection B. Loss of the small intrahepatic bile ducts can
B. Cytomegalovirus (CMV) infection occur
G. Delayed graft function G. Glucocorticoid therapy increases speed of
D. Hepatic artery thrombosis recovery
E. Recurrent AIH D. The patient is safe to take the drug again in
the future as the risk falls with repeat
22.17. A patient with primary biliary cholangitis exposure
is concerned about the prognosis of her E. The patient should avoid all penicillin-based
disease. Which of the following is predictive of drugs in the future
a poor outcome?
A. Alkaline phosphatase (ALP) level 22.21 An 18 year old medical student has his
B. AMA titre hepatitis B and C status checked as part of his
G. Large liver size on ultrasound occupational health screening for entrance to
D. Older age at disease onset medical school. His results are as follows:
LFTs:
~-
E. Presence of intercurrent autoimmune disease
Bilirubin 12 J.lmoi/L (0. 70 mg/dl) 1
22.18. A 38 year old woman presents in the ALT 19 U/L
third trimester of pregnancy with itch. She is HCV antibody not detected
found to have a rise in liver enzymes and serum Hepatitis B surface antigen (HBsAg) positive
bile acids. Which of the following statements is Antibody to HBsAg (anti-HBs) negative
correct? Antibody to hepatitis B core antigen
(anti-HBc) lgM negative
A. Acute viral hepatitis is a likely cause
Anti-HBc lgG positive
B. Fatty liver of pregnancy is the most likely
What is the next step in his management?
diagnosis
G. It is likely that the mother has underlying A. Check hepatitis B e antigen (HBeAg) and
chronic liver disease HBV DNA
D. There is a risk of intrauterine fetal death, B. Liver biopsy .
meaning that consideration should be given G. Repeat blood tests in '6 months
to early delivery D. Tenofovir
E. There is no effective drug treatment E. Vaccinate for hepatitis .B

22.19. A 76 year old man is referred to the 21.22. A 42 year old female with a new
outpatient clinic. His family physician has diagnosis of chronic hepatitis B has recently

_____________ ______________________
downloaded__._
from www.medicalbr.com
248 • HEPATOLOGY

had fluctuations in her LFTs with an abnormal


ALT despite a low viral load. Her delta serology
is checked and is positive with a high titre.
22.25. A 37 year old male prisoner is newly
diagnosed with HCV. He has no significant past
medical history or drug history. His results are
1
'
i
I
I
I

Which of the following statements is true about as follows:


I
the hepatitis D virus (HDV)? Full blood count:
A. All patients with HBV have HDV Haemoglobin 130 g/L
co-infection White cell count ryJCC) 4.2x10 9/L
B. HBV-HDV co-infection has the same annual Platelets 98 x 109 /L
rate of cirrhosis and HCC as HBV LFTs:
mono-infection Bilirubin 42 Jlmoi/L (2.46 mg/dL)
G. HDV can infect individuals simultaneously ALT 60 U/L
with any acute hepatitis ALP 64 U/L
D. HDV contains a single antigen to which Ultrasound liver: irregular, shrunken liver with
infected individuals make an antibody no focal lesion and no free fluid. Splenomegaly.
(anti-HDV) HCV genotype 1a. HCV RNA 750000 U/mL.
E. HDV is a DNA virus like HBV Elastography 25 kPa (2-7 kPa).
What is the next appropriate management
22.23. A 25 year old female who has chronic step?
hepatitis B and is under long-term follow-up in A. Genotype 1a is more difficult to treat
the hepatology clinic is 27 weeks pregnant, and therefore he should have interferon
about to enter her third trimester. She is therapy
worried she will give her baby hepatitis B and B. He does not have advanced fibrosis and
would like you to reassure her. You review her therefore can be discharged from the clinic
blood tests from this clinic appointment at 27 G. He needs a liver biopsy to accurately stage
weeks' gestation. his liver disease
LFTs: D. He should be offered oral antiviral therapy
Bilirubin 18 Jlmoi/L (1 .05 mg/dL) E. No treatment; patient should be observed
ALT 14 U/L with continued outpatient follow-up
ALP 84 U/L
HBsAg positive 22.26. A 64 year old man has been admitted to
HBeAg negative hospital feeling tired and unwell, suffering from
HBV DNA 48 U/mL (low titre) nausea and itching. A careful history elicits that
What is the most appropriate next step in he recently celebrated his wedding anniversary
her management plan? on a cruise ship from which they returned a
A. Advise elective caesarean delivery few weeks ago. He has recently noted dark
B. Interferon-alta in third trimester urine and pale stool. His wife is not unwell
G. Observe but she is vegetarian and he mentions he ate
D. Tenofovir in third trimester large amounts from the seafood buffet. He has
E. Vaccinate infant at birth no other risk factors for jaundice. His LFTs
confirm he is jaundiced with an acute hepatitis.
22.24. A 52 year old male ex-intravenous Which of the following is most likely to test
drug user with liver cirrhosis from hepatitis positive?
C virus has complications of diuretic-resistant A. Delta antibody
ascites and previous episodes of B. Hepatitis A virus (HAV) RNA in sweat
encephalopathy. He has been waiting for G. Hepatitis B surface antigen
treatment. Whilst he waits for treatment to be D. Hepatitis C antibody
initiated, what is the most appropriate E. Hepatitis E virus RNA in stool
management plan?
/
A. AFP and ultrasound in 12 months for HCC 22.27. There has been an outbreak of hepatitis
surveillance A virus (HAV) at a local nursery. The head
B. Assessment for liver transplantation teacher of a primary. school in the same area
G. Liver biopsy would like some advice on prevention of
D. Tenofovir secondary cases of hepatitis A infection. Of
E. Trial of interferon for 4 weeks note, no one at the primary school (child or

downloaded from www.medicalbr.com


HEPATOLOGY • 249

adult member of staff) has been identified as 22.30. A 62 year old male with cirrhosis from
unwell or an index case. What advice do you non-alcoholic fatty liver disease (NAFLD)
give to her? undergoes 6-monthly ultrasound for Hee
A. Everyone in her primary school should be surveillance and AFP. His AFP is normal but
vaccinated ultrasound shows a 3.5-cm lesion in the liver.
B. Good hygiene practice is the cornerstone of What is the next appropriate management
prevention step?
c. HAVis not highly infectious A. earcinoembryonic antigen
D. Infected individuals will always have B. PET scan
symptoms C. Repeat ultrasound liver by consultant
E. There is a risk of becoming a chronic carrier radiologist
D. Repeat ultrasound liver in 3 months
22.28. A 49 year old male with chronic hepatitis E. Triple-phase eT scan
B who has cirrhosis and developed
hepatocellular carcinoma has a liver transplant. 22.31. A 52 year old male is referred to
What treatment should be prescribed in the hepatology clinic due to an abnormal
post-operative period? ultrasound scan. He was originally referred to
A. Hepatitis B immunoglobulin haematology to be investigated for
B. Hepatitis B immunoglobulin and oral thrombocytopenia. He has a past medical
nucleoside history of type 2 diabetes, hypertension and
C. Hepatitis B vaccjnation body mass index (BMI) 33 kg/m 2 . He has mildly
D. Interferon-alta deranged LFTs (which prompted the ultrasound
E. Ribavirin scan request). Below are his results from clinic:
Full blood count:
Haemoglobin 106 g/L
22.29. A 79 year old male is admitted to the
wee 5.3x109/L
emergency department with rigors and
Platelets 89 x 109 /L
abdominal pain. He has a past medical history
LFTs:
of hypertension with no history of travel abroad.
Bilirubin 33 11moi/L (1.93 mg/dL)
On examination he is febrile with a temperature
ALT 54 U/L
of 40oe and tachycardic. He is tender in the
ALP 72 U/L
right upper quadrant but without guarding or
AFP 892 kU/L
rebound.
Ultrasound: 5-cm lesion in liver. Liver is
His inflammatory markers are raised with
shrunken and nodular. Further imaging is
wee 24x 109/L and e-reactive protein (eRP) of
requested.
300 mg/L. He has deranged LFTs with bilirubin
What is the likely description of the lesion on
64 11moi/L (3.74 mg/dL), ALT 74 U/L, ALP
a eT scan?
320 U/L. A liver screen was sent. An
ultmsound abdomen showed three cystic A. Enhances in arterial phase with portal venous
lesions in the right lobe of the liver with a washout
dilated common bile duct and associated filling B. Fluid-filled cystic lesion
defect suggestive of gallstones and inflamed C. Focal central scar
gallbladder. D. Low-density lesion with delayed arterial filling
Intravenous antibiotics and fluids are started. E. Thick-walled cyst with calcification
Stool microscopy for ova/cysts and parasites
was negative. Blood cultures were positive for 22.32. A 37 year old female presents to the
Escherichia coli and subsequent aspirate emergency department with colicky right upper
of cyst was positive for the same bacteria. quadrant pain and nausea. This is her fourth
Which of the following is the most likely presentation in 6 months. Ultrasound confirms
diagnosis? gallstones and she is referred to the surgeons
A. Amoebic liver abscess for elective cholecystectomy. Which of the
B. Hepatocellular carcinoma following dietary recomme.ndations do you
C. Hydatid cyst advise her to adopt?
D. Polycystic liver disease A. Fermentable, oligo-, di-, monosaccharides
E. Pyogenic liver abscess and polyols (FODMAP) diet

downloaded from www.medicalbr.com


250 • HEPATOLOGY
'f
I
B. Gluten-free diet 22.35. A 47 year old male with a BMI of 35 kg/
C. High-fat diet m 2 presents with jaundice, loss of appetite with
D. Low-fat diet abdominal pain and increasing abdominal girth
E. Low-oxalate diet over 3-4 weeks. He is teetotal with no other
past medical history. He does not take any
22.33. A 64 year old male with a background of regular medication. On examination he has
ulcerative colitis and primary sclerosing hepatosplenomegaly and ascites, with no
cholangitis has upper abdominal pain and peripheral oedema.
complains of dark urine and pale stool for a Full blood count:
month. On further questioning he reveals recent Haemoglobin I 90 g/L
weight loss. On examination he is afebrile, wee 13xi09/L
cachexic with a soft abdomen with no palpable Platelets 690 xI 09/L
abdominal mass. Further investigations are as LFTs:
follows. Bilirubin 72 f!moi/L (4.21 mg/dL)
LFTs: ALT 500 U/L
Bilirubin 240 f!moi/L (14.03 mg/dL) ALP 460 U/L
ALT 140 U/L Albumin 31 g/L
ALP 880 U/L Ascitic fluid albumin 22 g/L
Ultrasound: dilated intrahepatic bile ducts What is the most likely cause of his
with poorly demarcated echogenic tissue in presentation?
hepatic hilum. A. Alcoholic liver disease
Which is the next best investigation? a. Congestive cardiac failure
A. Carcinoembryonic antigen C. Haemochromatosis
B. Colonoscopy D. Hepatic vein thrombosis
C. CT abdomen E. NAFLD
D. ERCP
E. Liver biopsy 22.36. A 45 year old patient with alcoholic liver
disease is accompanied by his daughter to
clinic. He has been abstinent of alcohol for 4
22.34. A 42 year old female has attended the
years but developed diuretic-resistant ascites
emergency department numerous times with
and mild encephalopathy. Which of the
intermittent episodes of epigastric pain. She
following scoring systems should be used to
has been investigated as an outpatient with
identify and prioritise whether he should be
normal upper Gl endoscopy. On this
assessed for liver transplantation?
attendance, her results are as below:
Amylase 400 U/L A. King's College criteria
Bilirubin 45 f!moi/L (2.63 mg/dL) B. Maddrey score
ALT 56 U/L C. MELD (Model for End-Stage Liver Disease)
ALP 142 U/L D. Rockall score
CRP 2 mg/L E. Serum-ascites albumin gradient
Abdominal ultrasound: normal liver, slightly
'
dilated intrahepatic ducts and common bile 22.37. A 21 year old female is diagnosed with
duct. No stones or masses. MRCP: pancreas fibrolamellar hepat~cellular carcinoma of 3 em
normal but, as on ultrasound, dilated common with no underlying cirrhosis. Her AFP is normal.
bile duct with no stones or masses. Which is the best treatment option for her?
You diagnose her with sphincter of Oddi A. Chemotherapy
dysfunction (SOD). What is the next best B. Do nothing and observe
appropriate management step. C. Radiofrequency ablation (RFA)
A. Advise regular non-steroidal D. Surgical resection
anti-inflammatory drugs and no other E. Transarterial chemoembolisation (TACE)
intervention
B. Cholecystectomy 22.38. A 64 year old.female with recent change
C. Nifedipine in bowel habit and rectal bleeding who is
D. Observe awaiting lower Gl investigation presents to the
E. Sphincter of Oddi manometry emergency department with dark urine and

downloaded from www.medicalbr.com


HEPATOLOGY • 251

pale stool. On examination she is jaundiced 22.41. A patient has suspected NAFLD. Which
with hepatomegaly and moderate-volume of the following statements about diagnosis
ascites. and treatment is correct?
LFTs: A. Statins should be discontinued due to the
Bilirubin 240 ~-tmoi/L (14.03 mg/dl) risk of drug-induced liver injury
ALT 84 U/L B. Testing for the PNPLA3 rs738409 genetic
ALP 1400 U/L variant is part of the routine clinical testing of
Albumin 31 g/L patients
Carcinoembryonic antigen (CEA) 230 ~-tg/L C. The FIB-4 Score distinguishes between
Ascitic fluid: blood-stained alcoholic liver disease and NAFLD
Ultrasound: multiple liver lesions compressing D. The presence of individual features of the
bile ducts metabolic syndrome should be sought and
Which of the following is the most likely treated to reduce cardiovascular risk
diagnosis? E. Venesection should be commenced
A. Cholangiocarcinoma immediately if ferritin levels are raised, as this
B. Colorectal cancer indicates haemochromatosis
C. Focal nodular hyperplasia
D. Hepatic adenoma 22.42. A 57 year old man is admitted with
E. HCC presumed alcoholic hepatitis. Which of the
following statements about diagnosis and
22.39. A hospital porter is seen in the treatment is correct?
occupational health clinic following a A. A 1\(laddrey 'discriminate function' of less
needlestick injury. Investigations taken in the than 32 is indicative of a poor prognosis
clinic show that the patient is HBsAg negative, B. Alcohol consumption may safely continue
anti-HBc negative and anti-HBs positive. What C. Patients should be fasted to avoid stressing
is the correct interpretation of these results? the liver
A. Acute infection with hepatitis B D. The Steroids or Pentoxifylline for Alcoholic
B. Chronic dual infection with hepatitis B and Hepatitis (STOPAH) trial showed that
delta virus pentoxyfilline should be the first -line
C. Chronic mono-infection with hepatitis B treatment for alcoholic hepatitis
D. Previous immunisation against hepatitis B E. The STOPAH trial showed that prednisolone
without prior infection 40 mg daily for 28 days leads to a modest
E. Previous infection to hepatitis B but the reduction in short-term mortality
patient has cleared the virus
22.43. A 54 year old male patient with
22.40. When assessing the severity of NAFLD in alcohol-related cirrhosis is admitted with a
an overweight 65 year old type 2 diabetic 3-week history of increasing abdominal swelling
patient with hypertension, which of the and discomfort. He is mildly jaundiced and has
following statements is correct? a low-grade pyrexia but is haemodynamically
stable. Routine blood tests and a chest X-ray
A. A normal ALT level indicates that the disease
have been requested. What test would you
is mild and that there is unlikely to be
perform next?
significant scarring of the 'liver
B. A raised y-glutamyl transferase (GGT) A. Diagnostic paracentesis
indicates that the patient is probably B. Electroencephalogram
dependent on alcohol C. ERCP
C. The AST: ALT ratio is a useful indicator D. Triple-phase CT liver
of progressive liver fibrosis towards E. Upper Gl endoscopy
cirrhosis
D. The presence of obesity; hypertension and 22.44. An otherwise healthy 35 year old nurse
type 2 diabetes is not associated with a and part -time tattoo artist is referred with a
greater likelihood of steatohepatitis and liver persistent, fluctuating tral'!saminitis (ALT
fibrosis 40-120 U/L) that has been present for several
E. The use of routine ultrasound can distinguish years. What viral infection would you consider
between simple steatosis and steatohepatitis most likely?

downloaded from www.medicalbr.com


252 • HEPATOLOGY

A. Epstein-Barr virus (EBV) 22.48. A 48 year old male has recently been
B. Hepatitis A diagnosed with hemochromatosis with
C. Hepatitis B homozygous mutation of the HFE C282Y gene.
D. Hepatitis C His blood tests show a ferritin of 1950 11g/L
E. Hepatitis E and a transferrin saturation of 88%. What
treatment would be the most appropriate to
22.45. A 38-year old man is referred by his commence?
family physician to the outpatient clinic. His A. Ferrous sulphate 200 mg 3 times daily
father had haemochromatosis and he is about B. Fortnightly venesection
to get married, so he is wondering whether he C. Propranolol 20 mg 3 times daily
is likely to be affected. What would be the best D. Spironolactone I 00 mg once daily
first -line screening test in this case? E. Vitamin C supplement twice daily
A. CT liver
B. Ferritin 22.49. A patient with NAFLD cirrhosis
C. HFE genetic analysis undergoes screening upper endoscopy and is
D. Liver biopsy noted to have moderate (grade 2} oesophageal
E. Transferrin saturation varices with no signs of recent bleeding. What
would be the best next step?
22.46. A 45 year old woman presents with a A. Admit for TIPSS placement
5-day history of pale stools and dark urine B. Admit to intensive care unit and place
associated with cramping epigastric and right Sengstaken-Biakemore tube immediately
upper quadrant pains. Blood tests show C. Repeat upper Gl endoscopy in 6 months
bilirubin 120 11moi/L (7.02 mg/dl}, ALT 65 U/L, D. Start non-selective ~-blocker (propranolol
ALP 580 U/L, GGT 640 U/L. What would be 20 mg 3 times daily}
your first-line imaging investigation? E. Variceal banding
A. CT pancreas
B. Endoscopic ultrasound 22.50. A 53 year old bank employee in the UK
C. ERCP is found at a routine check-up to have
D. PET-CT abnormal LFTs. What is the most common
E. Ultrasound abdomen aetiology for abnormal liver biochemistry in
developed countries?
22.47. A 54 year old man with alcoholic A. Alcoholic liver disease
cirrhosis presents with haematemesis. The B. Autoimmune hepatitis
patient is commenced on terlipressin and C. Hepatitis C
emergency upper Gl endoscopy is performed, D. NAFLD
which demonstrates large oesophageal varices E. Primary biliary cholangitis
with active bleeding. The varices are banded
with good haemostatic effect. What medicine 22.51. A 60 year old man is found to have
would you start as secondary prophylaxis to hepatitis C and undergoes a liver biopsy that
reduce the chance of further variceal confirms stage 4 fibrosis (cirrhosis). He is
haemorrhage in the future? asymptomatic and subsequently receives
A. Atenolol antiviral therapy and successfully clears the
B. lsosorbide mononitrate virus (a sustained viral response). What further
C. Losartan test will he need?
D. Propranolol A. Cardiac angiogram
E. Ramipril B. Chest X-ray
C. Electrocardiogram (ECG)
D. HCV RNA annually to check for recurrence
E. Ultrasound every 6 months as part of routine
HCC surveillance

downloaded from www.medicalbr.com


HEPATOLOGY • 253

Answers
22.1. Answer: C. again not have an obstructive pattern.
The liver has significant regenerative capacity, Autoimmune hepatitis causes hepatocellular
with stem cells within the canals of Hering jaundice in aggressive forms, but not
playing a key role. This regenerative capacity obstructive jaundice. Hypothyroidism can cause
plays an important role in recovery from liver skin pigmentation that can be mistaken for .
failure and from liver resection. It also jaundice (although characteristically the sclerae
contributes to the phenotype of cirrhosis. The remain normal colour, in contrast to jaundice
liver is on the right side of the upper abdomen where yellowing is characteristic).
crossing the midline and weighs 1-2 kg,
depending on body size. Although drawings 22.4. Answer: E.
can give the impression that the liver lobule is Pancreatitis can be a rare complication of
encapsulated, this is not the case in humans paracetamol overdose and typically has a very
(although it is in pigs). The portal tracts contain poor outcome (also frequently preventing liver
portal vein radicles and arterioles, together with transplantation). Paracetarnol overdose causes
small bile ducts. Sinusoids cross the liver lobule acute liver injury, but if the acute event is
to the hepatic vein radicle. survived, the liver typically returns to normal.
Once encephalopathy develops, deterioration is
22.2. Answer: E. typically very rapid (hours or even minutes).
Synthesis of clotting factors by the liver makes Encephalopathy typically occurs after PT
prothrombin time (PT) a useful and easily prolongation and prior to the onset of jaundice.
available marker of hepatocyte function The outcome of acute liver failure in
(although watch for vitamin K deficiency and paracetamol is typically better than other
patients on warfarin). Alanine transaminase is a causes of acute liver failure.
marker of hepatocyte injury not function.
Platelet count lowering and elevation of 22.5. Answer: D.
Fibroscan values are markers of fibrosis/ The commonest autoantibody in PBC is
cirrhosis (through hypersplenism and liver antimitochondrial antibody (AMA; present in
fibrosis, respectively). MEGX, a dynamic test of over 95% of patients). A minority of patients
lidocaine metabolism, is a direct, active test of (around 20%) have characteristic antinuclear

.-.I
hepatocyte function but toxicity can be an issue antibodies that are reactive with either nuclear
and it is not in widespread clinical use. dot or nuclear rim antibodies. Where present,
these carry the same degree of diagnostic
22.3. Answer: E. value as AMA (and may suggest a worse
Yellowing of the sclera and skin are features prognosis). Both AIH and lupus are associated
suggestive of jaundice. The presence of dark with antinuclear antibodies but with a diffuse
urine and pale stools suggest a post -hepatic or nuclear staining pattern. Low-titre diffuse
obstructive jaundice (conjugated bilirubin is nuclear antibodies are seen frequently in NASH.
leaking back into the circulation and being The 'characteristic' autoantibody in PSG is
excreted through the kidney, causing urine perinuclear antineutrophil cytoplasmic antibody
darkening, whilst bilirubin metabolites are not (pANCA), although this is seen in only around
reaching the bowel, causing stools to be pale). 30% of patients.
Pancreatic carcinoma is a common cause of
this form of jaundice. Haemolysis is a cause of 22.6. Answer: B. ·
pre-hepatic jaundice through increased red DILl can be difficult to distinguish on liver
blood cell breakdown and bilirubin generation in biopsy from autoimmune hepatitis due to a
the spleen and thus would not give an number of shared features, including
obstructive pattern. Gilbert's syndrome is an parenchymal inflammation and eosinophilia.
inherited abnormality of bilirubin transport that Clinical context needs to be considered (e.g.
gives rise to clinically non-significant elevation autoimmune disease histQry and drug
of bilirubin (other liver biochemistry is typically exposure) and other immunological features of
normal), particularly in times of physiological AIH (elevated lgG and autoantibodies) sought.
stress such as intercurrent illness, which would Opiates are not reported to cause DILl; in ,

downloaded from www.medicalbr.com


254 • HEPATOLOGY

terms of drugs of abuse, the major risk is


ecstasy. When approaching a patient with
possible DILl, the precautionary principle should
condition. The name was changed following a
patient campaign in 2015. Typically thought of
as a disease of middle-aged and older women
1
apply and any potential risk therapy should be (it is 10 times commoner in women than men),
stopped until the causality becomes clearer. recent large cohort studies have identified an
important group of younger patients (aged
22.7. Answer: C. 20-50) who are more symptomatic and less
Hepatorenal failure complicating chronic liver likely to respond to UDCA. Younger patients
disease is best regarded as a vascular should be monitored closely and treated
consequence of cirrhosis and it typically aggressively. UDCA is first line-therapy and
improves when liver function returns to normal. should be used at a dose of 13-15 mg/kg in all
HLA matching is unnecessary in liver patients, regardless of symptom status or
transplantation (ABO and weight matching, in disease severity. The most characteristic blood
contrast, are important). Immunosuppression test change is elevation of the alkaline
regimes differ between centres but most phosphatase value, which reflects the
centres continue life-long immunosuppression. cholestatic nature of the disease. Transaminase
Research into safe discontinuation of elevation can be a feature of aggressive
immunosuppression has not translated into disease and overlap with AIH. PT, as well as
clinical practice. MMF monotherapy can be bilirubin and albumin levels, tend to be normal
associated with very resistant cellular rejection. until late in the disease, as hepatocellular
The most common time window for acute function is typically well preserved in this
cellular rejection is days 7-10. cholestatic disease.

22.8. Answer: B. 22.10. Answer: D.


The Kupffer cells play a key role in innate The clinical scenario is strongly suggestive of
immunity, in particular acting as a barrier for an acute viral hepatitis infection and the timing
gut bacteria/bacterial products entering the of onset would be most compatible with
portal vein. Sepsis is one of the major causes hepatitis E virus (HEV). The prodrome and
of death in acute liver failure. Steroid hormone onset in hepatitis B virus (HBV) are much more
clearance is a liver function and loss of prolonged than those described here (although
clearance function contributes to fluid retention/ clearly infection prior to the holiday is possible).
ascites (aldosterone) and feminisation in men Hepatitis C virus (HCV) almost exclusively
(oestrogen) with chronic liver failure. There is no causes chronic liver injury with no recognised
impact in the acute setting. The liver plays a acute infection event. Wilson's disease can
critical role in glucose homeostasis, buffering present with an acute hepatitis episode but is a
portal venous blood through glycogen rare condition and the acute presentation even
synthesis/breakdown. Hepatocytes are also key rarer. There is an almost automatic assumption
in gluconeogenesis. Acute liver failure is in some quarters that a hepatitis in a young
frequently characterised by hypoglycaemia but man returning from abroad will be
this is relatively easy to manage with alcohol-related. In this case this is very unlikely,
intravenous (IV) glucose in practice, and it is as an ALT of 10 000 U/L would be very atypical
only rarely so profound as to impact on (normal or even- low ALT is characteristic of
outcomes. Breakdown of red cells at the end alcoholic hepatitis).
of their life span is a function of the spleen not
the liver. Hyperoxaluria is an inborn error of 22.11. Answer: A.
oxalic acid metabolism, expressed in the liver AIH typically improves in terms of disease
but manifest as renal failure. Transplantation of activity during pregnancy. It can, however, flare
the kidney without the liver simply results in up during the early post-partum period. If
rapid renal failure in the graft. Combined patients have cirrhosis, then portal hypertension
transplant, in contrast, is highly effective. Oxalic can worsen during the third trimester, so
acid plays no role in acute liver failure. consideration should be given to endoscopy
as they enter the third trimester. This only
22.9. Answer: D. applies to cirrhosis. patients. MMF is teratogenic
Primary biliary cholangitis is the new name for and is contraindicated in pregnancy, so
primary biliary cirrhosis, so it is the same azathioprine should be continued. The

downloaded from www.medicalbr.com


HEPATOLOGY • 255

offspring of mothers with AIH run a slightly situations, in particular to explore for the
increased risk of AIH later in life (because of the presence of metastasis.
genetic contribution to pathogenesis). This
small risk should not impact on plans for 22.15. Answer: C.
pregnancy. This clinical presentation is typical of Budd-
Chiari syndrome (hepatic venous thrombosis).
22.12. Answer: D. Cirrhosis is typically associated with a small,
Variceal bleeding can be high pressure and can painless shrunken liver and reversal of portal
lead to the patient exsanguinating rapidly. It is venous flow; pain would be very unusual.
therefore essential to secure venous access Hepatic artery thrombus is a specific
early and commence fluid resuscitation. Delay complication of liver transplantation.
can lead to later failure to gain access.
Cross-matching is clearly urgent but should be 22.16. Answer: D.
done once access is secured. However, the ALT elevation following liver transplantation is,
acute intervention of choice is endoscopy and as in other settings, a marker of liver injury. The
banding; this should only be undertaken once commonest aetiology is dependent on the time
the patient is haemodynamically stable. TIPSS point post-transplant. Immediately post-surgery,
is a radiological intervention that is appropriate the commonest causes are thrombosis of the
after failed endoscopy or early rebleed. hepatic artery (a specific complication of liver
Ultrasound scan is a part of the workup to transplant) and primary graft dysfunction
explore triggers for bleeding - portal venous (typically a consequence of preservation injury).
thrombosis or occult hepatocellular carcinoma Acute cellular rejection would typically not be
(HCC) being potential factors - but should be seen until days 5-10. CMV infection is another
undertaken once the acute bleeding state is characteristic post-transplant challenge,
under control. typically when there is a mismatch between the
CMV status of the donor and recipient.
22.13. Answer: A. Prophylactic regimes in at-risk individuals are
Glypressin is recognised to reduce the severity effective. AIH recurrence post -liver transplant is
of acute variceal bleeding and can act as a described but is typically a late phenomenon.
bridge to endoscopy and an adjunct to
endoscopy (helping a clearer endoscopic field). 22.17. Answer: A.
Noradrenaline (norepinephrine) may be required Alkaline phosphatase level at presentation and
in the critical care setting to maintain in particular after therapy with ursodeoxycholic
cardiovascular status but is not primarily an acid is predictive of outcome, with clinically

~.
agent to reduce bleeding risk. Octreotide has relevant cut-offs identified and now in
benefits in variceal bleeding but its use has widespread clinical use. AMA is an important
been superseded by Glypressin: where used, it
has to be intravenous. Propranolol should never
diagnostic feature but titre is not predictive of
outcome. Intercurrent autoimmune disease is I
be used in acute bleeding but is an important common and requires management but has no
agent in the treatment of chronic portal impact of liver disease risk. Liver size is typically
hypertension. There is no clear evidence to increased in early PBC. Small liver size is a
support the use of tranexamic acid in Gl feature of cirrhosis with a worse outcome.
bleeding (including variceal bleeding) although Younger age is associated with a lower
trials are ongoing. likelihood of response to UDCA and thus
increased risk. PBC is typically benign in older
22.14. Answer: A. patients.
A second imaging modality is the key next
investigation in a case of suspected HCC. AFP 22.18. Answer: D.
has some use as a screening test in at-risk The combination of abnormal biochemistry,
patients but it can be normal in patients with pregnancy stage and, in/particular, elevation of
HCC, meaning it has no use in diagnosis. serum bile acids all point to cholestasis of
Laparoscopy and liver biopsy can be of use in pregnancy. The bile acid.elevation makes fatty
staging and planning therapy in specific cases, liver of pregnancy unlikely. UDCA therapy is
once the diagnosis is supported by dual effective and rifampicin has been used in
imaging. PET scan has utility in specific severe cases. There is no association with·

downloaded from www.medicalbr.com


256 • HEPATOLOGY

pre-existing maternal liver disease. Cholestasis


pruritus can be very severe. The rnajor clinical
concern is sudden intrauterine death due, it is
infection with normal transaminases and
negative anti-HBc lgM. Further management of
the patient will depend on HBeAg status and
r
thought, to a toxic effect of bile acids on the HBV DNA to distinguish disease phase.
fetal cardiac conducting apparatus. The risk
rises exponentially frorn 36 weeks' gestation 21.22. Answer: D.
onwards and there is a strong case for early HDV is an RNA-defective virus that requires the
delivery to avoid this risk. simultaneous presence of HBV for replication
and has the same sources and modes of
22.19. Answer: D. spread. HBV replication is usually suppressed
PSC is a fibrotic cholestatic liver disease of by HDV. Liver damage is believed to be due to
presumed autoimmune aetiology, which has a HDV and persistent HDV replication in
rnale predominance. Although UDCA is widely co-infected patients leads to higher annual
used, and typically improves the cholestatic rates of cirrhosis and HCC. Interferon-alta is the
liver function tests seen in PSC, there is no only effective drug against HDV but the optimal
evidence that it reduces mortality and there is duration of therapy is not well defined.
no evidence to support any drug therapy. HCC
can complicate cirrhosis PSC (as with cirrhosis 22.23. Answer: E.
of any cause) but the specific associated Pregnant women with chronic hepatitis B are
cancers are cholangiocarcinoma and colonic most infectious when markers of continuing
carcinoma, both of which should be screened viral replication, such as HBeAg, and high
for. Imaging of the bile ducts is a key part of hwels of HBV-DNA are present in the blood.
the diagnostic workup but should be through Tenofovir can be given if there are high levels of
magnetic resonance cholangiopancreatography HBV-DNA in the last trimester of pregnancy;
(MRCP) in the first instance because of the however, in this clinical scenario, the patient
risks associated with ERCP. The latter should has very low titres. Interferon-alia is
be reserved for therapeutic, rather than contraindicated in pregnancy. Neonates born to
diagnostic, procedures. hepatitis B-infected mothers should be
immunised at birth. In addition, to prevent
22.20. Answer: B. vertical transmission, hepatitis B
Flucloxacillin-induced liver injury is one of the immunoglobulin (HBig) is given to newborns of
commoner forms of drug-induced liver injury. It highly viraemic HBeAg-positive mothers.
is thought to have an immune pathogenesis Guidelines do not recommend elective
(there is a strong HLA association) and there is caesarean delivery for mothers with chronic
no crossover with other penicillins, which can hepatitis B infection.
be used safely. The immune element probably
underpins the characteristic pattern of 22.24. Answer: B.
worsening injury with repeat exposure - a Liver transplantation should be considered
phenomenon that means that repeat use is when complications of cirrhosis occur. He
absolutely contraindicated in patients should be undergoing 6-monthly interval AFP
suspected of the reaction. The process is and ultrasound for HCC surveillance. This
principally cholestatic, with elevation of alkaline surveillance will be ongoing even after his HCV
phosphatase being the characteristic is cured, as he will still have cirrhosis. Interferon
biochemical change (eosinophilia can be seen is contraindicated in decompensated cirrhosis,
as in all drug reactions). In severe cases, and tenofovir is a treatment used in hepatitis B.
intrahepatic bile duct loss can be seen and
UDCA therapy is advocated by some (although 22.25. Answer: D.
the evidence basis is limited). Glucocorticoid This patient is cirrhotic as evident by
therapy has no role. thrombocytopenia, Ultrasound imaging and high
elastography score. He does not need a liver
22.21. Answer: A. biopsy. He should be treated for his hepatitis C
This medical student has chronic hepatitis B. with new all-oral di~:ect -acting antiviral therapy.
Hepatitis B surface antigen is the hallmark of Combinations of these drugs have been
chronic HBV infection if present for more than targeted to be pan-genotypic with sustained
6 months. This patient is unlikely to have acute viral response (SVR) rates >90%. Once treated,

downloaded from www.medicalbr.com


HEPATOLOGY • 257

he will require life-long follow-up in clinic for 22.30. Answer: E.


HCC surveillance, and as he has only been AFP is produced by 60% of HCCs; therefore a
recently diagnosed with cirrhosis he should also negative AFP does not exclude HCC.
have an endoscopy to screen for varices. Carcinoembryonic antigen is a tumour marker
that can be raised in colorectal cancer.
22.26. Answer: E. Combination of imaging modalities more
The clinical presentation of hepatitis E is similar accurately diagnoses and stages the extent of
to that of hepatitis A with the likely source as disease, and using at least two modalities ·
shellfish in this clinical scenario. However, this (typically, CT or MRI following initial screening
question focuses on detection methods. ultrasound identification of a mass lesion) ·is
Hepatitis A virus is excreted in faeces or recommended.
diagnosis is made by detection of HAV lgM
antibodies in the blood, which persists for up to 22.31. Answer: A.
14 weeks after initial infection. Alternative In this clinical scenario, the patient has cirrhosis
samples such as serum and saliva can be used as evidenced by thrombocytopenia and
but assays are expensive and sensitivities vary. ultrasound imaging of liver. In this case, the
Therefore, answer E is the correct option. likely aetiology is NAFLD with risk factors of
diabetes, hypertension and obesity. The high
22.27. Answer: B. AFP is diagnostic for HCC. HCC are
HAV belongs to the picornavirus group of hypervascular in the arterial phase, followed by
enteroviruses. HAVis highly infectious and is washout in the portal venous phase. Option E
spread by the faecal-oral route, but a chronic is typical of a hydatid cyst and daughter cysts
carrier state does not occur. Infected may be present. Option C is the appearance of
individuals may be asymptomatic. Health a hepatic adenoma and focal nodular
departments will investigate outbreaks and hyperplasia can be differentiated from adenoma
advise regarding vaccination. In this scenario because of a focal central scar.
vaccinating everyone is not necessary, as no
one has been identified as the index case or 22.32. Answer: D.
close contact at the primary school. This patient has biliary colic. Gallstones are
conventionally classified into cholesterol or
22.28. Answer: B. pigment stones, although the majority are of
The use of post -liver transplant prophylaxis with mixed composition. Patients are generally
direct-acting antiviral agents and hepatitis B advised a low-fat diet to help reduce symptoms
immunoglobulins has reduced the reinfection as fat releases cholecystokinin, which
rate to 10% and increased 5-year survival to precipitates gallbladder contraction and might
80%, making transplantation an acceptable result in biliary pain. Low-oxalate diet is
treatment option. Hepatitis B vaccination is associated with renal stones. A FODMAP diet
ineffective in those already infected by HBV. is a treatment option for patients with irritable
Ribavirin is a treatment used in hepatitis C bowel syndrome and gluten-free diets are for
infection. patients with coeliac disease.

22.29. Answer: E. 22.33. Answer: C.


Hepatic abscesses are rare and clinical features Primary sclerosing cholangitis carries a lifetime
of pyogenic and amoebic liver abscesses can risk of cholangiocarcinoma of approximately
be similar. However, pyogenic abscesses are 20%. It can arise from anywhere in the biliary
most common in older patients and usually tree. Often the diagnosis is made by a
result from ascending infection due to biliary combination of CT and MRI. A liver biopsy may
obstruction (cholangitis). They are often cause tumour seeding. The patient will need an
described as more aggressive. E. coli and ERCP due to biliary obstrLJction but .is not
various streptococci, particularly Strep. milleri, currently septic and therefore the best next
are the most common organisms. Any investigation would be further imaging. Patients
associated biliary obstruction and cholangitis with ulcerative colitis and psc should undergo
require biliary drainage. Hepatic hydatid disease yearly colonoscopy as they are <;~t high risk of
is a parasitic zoonosis caused by the colorectal cancer, but this is not the focus of
Echinococcus tapeworm. this scenario.

downloaded from www.medicalbr.com


258 • HEPATOLOGY

22.34. Answer: E.
The patient has SOD type I. The gold standard
to the virus in the past but has been immunised
to the virus.
I I
for diagnosis is sphincter of Oddi manometry.
All biliary SOD patients with type I disease are 22.40. Answer: C.
treated with endoscopic sphincterotomy. NAFLD is an increasingly common liver disease
Medical therapies can be tried in SOD type II that is associated with features of the metabolic
patients. syndrome; the more features that an individual
possesses, the more likely they are to have
22.35. Answer: D. progressive disease. NAFLD is often
The patient has polycythaemia vera causing asymptomatic and may be associated with
Budd-Chiari syndrome. The ascitic fluid is an normal liver biochemistry, even when disease is
exudate with a serum-ascites albumin gradient advanced. AST rises and ALT falls as disease
(SAAG) of < 11 g/L. The other causes listed progresses towards cirrhosis. The AST: ALT
would have a high SAAG. ratio is included in calculated scores such as
the NAFLD fibrosis score and PIB-4 score that
22.36. Answer: C. are used to risk -stratify patients for presence of
The MELD score is used to identify and advanced fibrosis. GGT levels may be raised in
prioritise patients for liver transplantation. The NAFLD and so cannot be used to discriminate
Maddrey score enables the clinician to assess between alcoholic and non-alcoholic liver
prognosis in alcoholic hepatitis. The King's disease. Routine imaging modalities cannot
College criteria identify indices associated with distinguish between steatosis and
poor prognosis in patients with acute liver steatohepatitis; at present this can only be
failure. The Rockall score identifies patients reliably performed histologically.
needing intervention in upper Gl bleeds.
22.41. Answer: D.
22.37. Answer: D. NAFLD is a common, progressive liver disease
Fibrolamellar HCC occurs in young adults in the that is also associated with an increased risk of
absence of cirrhosis. The treatment of choice is cardiovascular disease. In patients with NAFLD,
surgical resection. The tumour biology is liver-related mortality is the third most common
different to standard HCC and, in the absence cause of death, after cardiovascular disease
of cirrhosis, surgical resection is less likely to and extrahepatic malignancy. If a patient is
cause liver failure. TACE and RFA are treatment found to have NAFLD, other features of the
options for HCC. metabolic syndrome (including type 2 diabetes,
hypertension, dyslipidaemia) should be sought
22.38. Answer: B. and treated. NAFLD is not associated with an
This patient has secondary liver disease. Given increased risk of statin-related liver injury and
the recent change in bowel habit and raised so statins are generally considered safe. Raised
CEA, the primary is likely to be colorectal ferritin levels may be seen in patients with
cancer. She has biliary obstruction from tumour NAFLD and do not necessarily indicate the
burden in liver. Options D and E are benign presence of haemochromatosis, which can be
lesions that would not present this way. Serum excluded by checking transferrin saturation.
levels of the tumour marker CA 19-9 are Although carriage of the PNPLA3 gene
elevated in cholangiocarcinoma. rs738409 variant is associated with more
severe NAFLD, it is not currently used as part
22.39. Answer: D. of routine clinical testing. The FIB-4 score is
HBV surface antigen (HBsAg) is a marker of used to risk-stratify patients for presence of
current infection. HBV surface antibody advanced fibrosis in NAFLD; it should not be
(anti-HBs) may be positive following previous used in patients with alcoholic liver disease.
infection with HBV or if the patient has been
immunised against HBV but immunisation 22.42. Answer: E.
against HBV does not induce HBV anti-core A Maddrey discriminate function greater than
(anti-HBc) antibody production. As this patient 32 is indicative of severe disease. Cessation of
is HBsAg negative, he is not currently infected. all alcohol consumption is essential. Good
The absence of anti-HBc despite the presence nutrition is very important, and enteral feeding
of anti-HBs indicates he has not been exposed via a fine-bore nasogastric tube may be needed

downloaded from www.medicalbr.com


HEPATOLOGY • 259

in severely ill patients. The STOPAH study was or passive protection against HCV. HCV is
a large multicentre double-blind randomised usually identified in asymptomatic individuals
trial to evaluate the relative merits of steroids screened because they have risk factors for
and/or a weak anti-tumour necrosis factor infection, such as previous injecting drug use,
(anti-TNF) agent (pentoxifylline), alone or in tattoos, needlestick injury, etc.
combination. In a cohort of II 03 patients, no
significant benefit from pentoxifylline treatment 22.45. Answer: E.
was identified; however, treatment with In hereditary haemochromatosis, iron is
prednisolone 40 mg daily for 28 days led to a deposited throughout the body and causes
modest reduction in short-term mortality, from damage to several organs, including the liver.
17% in placebo-treated patients to 14% in the Serum iron studies show a greatly increased
prednisolone group. These findings were ferritin, a raised plasma iron and saturated
consistent with earlier studies, where an plasma iron-binding capacity. The differential
improvement in 28-day survival from 52% to diagnoses for elevated ferritin includes
78% was seen when steroids were given to inflammatory disease, NAFLD or excess ethanol
those with a Glasgow score of more than 9. consumption for modest elevations
However, neither steroids nor pentoxifylline (< 1000 [.Lg/L). Transferrin saturation of more
improved survival at 90 days or I year. Sepsis than 45% is highly suggestive of iron overload
is the main side-effect of steroids, and existing and not affected by inflammatory state and so
sepsis and variceal haemorrhage are the main is more specific than ferritin. Genetic testing
contraindications to their use. can be considered later, but in the first instance
tests for iron overload would be first line and
22.43. Answer: A. best value.
The most likely diagnosis would be
spontaneous bacterial peritonitis. Diagnostic 22.46. Answer: E.
paracentesis (ascetic tap) may show cloudy The patient presents with a painful obstructive
fluid, and an ascites neutrophil count above jaundice. The blood tests represent a typical
250x 106/L almost invariably indicates infection. cholestatic picture with elevated ALP and GGT,
The source of infection cannot usually be making viral hepatitis unlikely. In this setting,
determined, but most organisms isolated are of ultrasound would be the first-line investigation
enteric origin and E. coli is most frequently to seek evidence of biliary obstruction with
found. Ascitic culture in blood culture bottles dilated common bile duct due to, for example,
gives the highest yield of organisms. gallstone disease. Depending on the findings, it
Spontaneous bacterial peritonitis (SBP) needs may then be necessary to proceed to MRCP or
to be differentiated from other intra-abdominal endoscopic ultrasound prior to ERCP if an
emergencies, and the finding of multiple obstruction is identified. Pancreatic cancer is
organisms on culture should arouse suspicion more classically associated with a painless
of a perforated viscus. obstructive jaundice.

22.44. Answer: D. 22.47. Answer: D.


Hepatitis A, hepatitis E and EBV infections Non-selective ~-adrenoceptor antagonists
usually cause only a short-lived, acute (~-blockers; e.g. propranolol) are used as a
hepatitis, with the exception that HEV may secondary measure to prevent recurrent
become chronic in patients that are variceal bleeding. Following successful
immunocompromised. Health-care workers are treatment by endoscopic therapy, patients
routinely immunised against HBV, making should also be entered into an oesophageal
chronic HBV infection less likely in this situation. banding programme with repeated sessions of
HCV is caused by an RNA flavivirus. Acute therapy at intervals until the varices are
symptomatic infection with hepatitis C is rare. obliterated. In selected inpividuals, TIPSS may
Most individuals are unaware of when they also be considered in this setting.
became infected and are only identified when
they develop chronic liver disease. Eighty per 22.48. Answer: B.
cent of individuals exposed to the virus become The patient carries two copies pf the most
chronically infected and late spontaneous viral common HFE variant, C282Y, and has
clearance is rare. Unlike HBV, there is no active evidence of iron overload. Venesection would

downloaded from www.medicalbr.com


260 • HEPATOLOGY

be the most appropriate treatment with


frequency adjusted according to fall in ferritin
and transferrin saturation. Vitamin C
22.50. Answer: D.
Increasingly sedentary lifestyles and changing
dietary patterns mean that the prevalence of
1
supplementation increases absorption of iron obesity and insulin resistance has increased
from the diet and so should be avoided. worldwide; thus, fat accumulation in the liver is
a common finding during abdominal imaging
22.49. Answer: D. studies and on liver biopsy. In the absence of
TIPSS is used in the management of refractory high alcohol consumption (typically a threshold
variceal bleeding that has not responded to of <20 g/day for women and <30 g/day for
other therapies. Placement of a Sengstaken- men is adopted), this is called non-alcoholic
Biakemore tube is an emergency holding fatty liver disease. Non-alcoholic fatty liver
measure used when there is an uncontrollable disease (NAFLD) is the most common cause
variceal haemorrhage - for example, while for abnormal LFTs worldwide, estimated to
plans are made for a TIPSS - and so is not affect 20-30% of the general population in
necessary in this situation. Western countries and 5-18% in Asia, with
If non-bleeding varices are identified at about 1 in 10 NAFLD cases exhibiting
endoscopy, ~-blocker therapy with propranolol non-alcoholic steatohepatitis (NASH). NAFLD is
(80-160 mg/day) or nadolol is effective in the leading cause of liver dysfunction in the
reducing portal venous pressure. Administration non-alcoholic, viral hepatitis-negative population
of these drugs at doses that reduce the heart in Europe and North America and is predicted
rate by 25% has been shown to be effective to become the main aetiology in patients
in the primary prevention of variceal bleeding. u.ndergoing liver transplantation during the next
In patients with cirrhosis, treatment with 5 years.
propranolol reduces variceal bleeding by 47%
(number needed to treat for benefit (NNT8 ) 10), 22.51. Answer: E.
death from bleeding by 45% (NNT 8 25) and Although the virus has been successfully
overall mortality by 22% (NNT8 16). The efficacy cleared, the patient was found to have cirrhosis
of ~-blockers in primary prevention is similar to and so remains at risk of developing
that of prophylactic banding, which may also hepatocellular carcinoma (HCC). He will require
be considered, particularly in patients unable to follow-up and management to check for risks
tolerate ~-blocker therapy. Carvedilol, a associated with cirrhosis, including HCC and
non-cardioselective vasodilating ~-blocker, is portal hypertension/varices.
also effective.

downloaded from www.medicalbr.com


HG Watson, DJ Culligan,
LM Manson

Haematology and
transfusion medicine
Multiple Choice Questions
23.1. In a patient with a vague history of weight 23.4. A 65 year old man presents with an
loss but little else on examination you find immul!e-mediated thrombocytopenia. He has
lymphadenopathy confined to the right axilla. been treated with antibiotics during a recent
Which of the following conditions is most likely? hospital admission. Which of the following is
A. Chronic lymphocytic leukaemia most likely implicated in the new development?
B. Follicular lymphoma A. Amoxicillin
C. Glandular fever B. Ciprofloxacin
D. Human immunodeficiency virus (HIV) C. Gentamicin
seroconversion illness D. Metronidazole
E. Metastatic breast cancer E. Vancomycin

23.2. In the investigation of a patient with 23.5. A patient with systemic lupus
suspected essential thrombocythaemia, in erythematosus (SLE) and immune
which of the following genes may you find thrombocytopenia (ITP) presents with a platelet
abnormalities? count of 5 x 109/L. Which of these is the most
A. BCL-2 likely presenting symptom?
B.BCR A. Haemarthrosis
C. c-MYC B. Intracranial haemorrhage
D. CAL-R C. Muscular haematoma
E. MYH-9 D. Oral mucosal bleeding
E. Retroperitoneal haematoma
23.3. Having made a new diagnosis of
polycythaemia rubra vera (PRV) you are 23.6. Which of the following anticoagulants
consulting with the patient regarding prognosis has a mechanism of action involving
and complications of the condition. He has antithrombin-dependent inhibition of thrombin
read the information booklet and wishes to and factor Xa?
know about common vascular complications of A. Apixaban
the condition. Which of the following is the B. Bivalirudin
most common vascular complication? C. Dabigatran
A. Budd-Chiari syndrome D. Dalteparin
B. lschaemic stroke E. Edoxaban
C. Livedo reticularis
D. Mesenteric vein thrombosis 23.7. A 72 year old woman who is on warfarin
E. Pulmonary embolism consults to ask if she can change to an

downloaded from www.medicalbr.com


262 • HAEMATOLOGY AND TRANSFUSION MEDICINE
T
I
alternative oral anticoagulant. You review her A. Haemophilia A carrier with low VIII levels
clinical case. In which of the following clinical B. Type 1 von Willebrand disease
circumstances would the answer be that she C. Type 2A von Willebrand disease
should stay on warfarin? D. Type 2N von Willebrand disease
A. Atrial fibrillation in a patient with a prosthetic E. Type 3 von Willebrand disease
mitral valve
B. Distal deep vein thrombosis (DVD following 23.11. In the context of routine clerking of a
plaster cast immobilisation pre-surgical patient, which of the following is
C. Lone atrial fibrillation with a CHA2 DS 2-VASc the most informative question in detecting an
score of 2 underlying bleeding disorder in a patient who
D. Proximal DVT following total knee reports a 'problem with bleeding'?
replacement A. Bleeding after shaving
E. Unprovoked pulmonary embolism B. Easy bruising
C. Epistaxis as a child
23.8. A patient who is admitted to the intensive D. Post-partum haemorrhage
care unit (ICU) with multi-organ failure and E. Previous post-surgical bleeding
sepsis is suspected of having heparin-induced
thrombocytopenia (HIT). Which of the following 23.12. A Dutch woman is referred following the
pieces of clinical information is most likely to development of a pulmonary embolism without
suggest an alternative diagnosis? obvious provoking factors. If she were
investigated further, which form of inherited
A. The heparin treatment commenced 16 days
thrombophilia would you be most likely to find
ago
in this case?
B. The patient has been receiving treatment
with unfractionated heparin (UFH) A. Antithrombin deficiency
C. The patient has had complex cardiac surgery B. Factor V Leiden
with cardiopulmonary bypass C. Protein C deficiency
D. The patient has sustained a post-operative D. Protein S deficiency
pulmonary embolus E. Prothrombin gene mutation
E. The platelet count has dropped from normal
to a level of 53 x 10 9/L 23.13. A 65 year old man is admitted as a
medical emergency. A diagnosis of pulmonary
23.9. What is the most likely diagnosis in a 2 embolism is made. Which of the following
year old boy who presents with an apparently features in his clinical history is most likely to
unprovoked knee haemarthrosis? have contributed most to his thrombosis risk?
,l Initial investigations show that he has a A. He had a total knee replacement (TKR) 9
normal platelet count and a normal prothrombin months previously
time (PT). His activated partial thromboplastin B. He had just flown back to the UK from a
time (APTT} is prolonged at 76 seconds. weekend break in Paris
A. Lupus anticoagulant C. He has recently started quinine for night
B. Severe factor XI deficiency cramps
C. Severe factor XII deficiency D. He was discharged from hospital 4 weeks
D. Severe haemophilia A (factor VIII deficiency) ago following treatment for congestive
E. Severe haemophilia 8 (factor IX deficiency) cardiac failure (CCF)
E. His 64 year old cousin has recently suffered
23.10. A 13 year old girl with significant from a pulmonary embolism following bowel
menorrhagia is investigated to see if there is an surgery
underlying bleeding disorder. Her investigations
show a normal platelet count, PT and 23.14. You are as~ed to see a 59 year old man
fibrinogen. Her APTI is prolonged and her following 3 months of anticoagulation for an
factor VIII level is 0.2 U/mL, her von Willebrand episode of proximal DVT that occurred without
factor ristocetin co-factor (vWF: RiCO) level is any identifiable risk factor. You are trying to
0.05 U/rnL and her von Willebrand factor decide whether you think he would benefit from
antigen (vWF:Ag) is 0.14 U/mL. What is the long-term anticoagulation or not. Of the
most likely diagnosis? following, which is the strongest risk factor for

downloaded from www.medicalbr.com


HAEMATOLOGY AND TRANSFUSION MEDICINE • 263

predicting recurrence following discontinuation D. Liver failure


of the treatment? E. Vitamin K deficiency
A. Age over 50 years
B. D-dimer level after stopping treatment 23.19. The blood film of a 67 year old man is
reported to show target cells, acanthocytes,
c. Hormone replacement therapy use
Howell-Jolly bodies, a leucocytosis and a mild
D. Male sex
thrombocytosis. What is the most likely cause
E. Unprovoked episode
of this picture?
23.15. A 32 year old woman who has been A. Alcoholic liver disease
previously well presents with apparent loss of B. Essential thrombocythaemia
consciousness. You suspect a diagnosis of C. Falciparum malaria
thrombotic thrombocytopenic purpura (TTP). D. Hyposplenism
Which of the following observations would be E. Myelofibrosis
LEAST common in fitting with that diagnosis?
A. Confusion 23.20. Consider the haemoglobin-oxygen
B. Fever dissociation curve, below. Which condition
C. Microangiopathic haemolytic anaemia results in increased oxygen release to hypoxic
D. Purpura tissue, i.e. moves the dissociation curve to
E. Thrombocytopenia the right?
Normal
23.16. Thrombin is generated as part of the venous P02

process of activation of coagulation. Thrombin


has many actions. Which of these actions
directly results in the cross-linking of fibrin clot?
A. Activation of factor VIII
B. Binding of endothelial cell-bound
thrombomodulin
C. Binding of the platelet thrombin receptor
D. Cleavage of factor XIII
2 4 6 8 10 12
E. Cleavage of the fibrinopeptides on fibrinogen P0 2 (kPa)

23.17. A 36 year old man presents with A. Haemoglobin S (HbS)


night-time cough and wheezing over the B. Hypothermia
previous 2 months. He has a previous history C. Low levels of 2,3-bisphosphoglycerate
of eczema. What finding in his full blood count (2,3-BPG)
would be in fitting with the scenario and help D. Low levels of C02
towards the diagnosis? E. Reduced pH
A. Basophilia
23.21. Which of the following changes is
B. Eosinophilia
commonly seen in normal pregnancy?
C. Lymphocytosis
D. Monocytosis A. Low protein C levels
E. Neutrophilia B. Low vitamin B12 levels
C. Lupus anticoagulant
23.18. A junior doctor phones you about a D. Polycythaemia
patient who is unwell after a,n episode of E. Reducing fibrinogen level with advancing
post-cholecystectomy pancreatitis. The patient gestation
has a normochromic normocytic anaemia, a
leucocytosis and a mild thrombocytosis. The 23.22. When examining the abdomen for the
prothrombin time is 55 seconds, the APTI is presence of splenomegaly, which of the
46 seconds and the fibrinog\')n is normal following statements is true?
(4 g/L). What is the most likely diagnosis? A. A palpable spleen moves.downwards on
A. Anticoagulation with LMWH expiration
B. Disseminated intravascular coagulation B. A palpable spleen moves upwards on
C. Factor XIII deficiency inspiration

downloaded from www.medicalbr.com


264 • HAEMATOLOGY AND TRANSFUSION MEDICINE
...
I
G. A splenic rub is frequently heard following E. Transformation to acute myeloid leukaemia
acute infarction of massive splenomegaly (AML) is diagnosed when the blast count
D. If you can get above a mass in the left upper rises to > 30%
quadrant, then it is likely to be a spleen
E. The spleen is usually palpable in small 23.26. Which of the following situations
people would be in keeping with a diagnosis of
monoclonal gammopathy of uncertain
23.23. Which of the following statements significance (MGUS)?
relating to normal haematopoietic stem cells A. A light chain only paraprotein with
is true? immunoparesis for immunoglobulin G (lgG)
A. Daughter cells of stem cells are always and immunoglobulin A (lgA)
destined to become mature blood cells B. A low-level lgG paraprotein associated with a
B. Stem cells are easily identified when single lytic lesion in the femur
examining the bone marrow down the G. A low-level lgG paraprotein with normal
microscope serum free light chain ratio
G. Stem cells can only give rise to daughter D. An lgA paraprotein in someone presenting
cells of a particular lineage with acute renal failure
D. Stem cells circulate in the blood and can be E. An immunoglobulin M (lgM) paraprotein in
mobilised into the blood in increased someone presenting with mild anaemia and
numbers by growth factors such as splenomegaly
granulocyte colony-stimulating factor (G-CSF)
E. They are the commonest cell found in the 23.27. Which of the following clinical situations
bone marrow represents a likely diagnosis of multiple
myeloma?
23.24. A patient is referred after being found A. A 10 year old child with acute renal failure
unexpectedly to have anaemia. Which of the and a positive urine test for Bence Jones
following statements is in keeping with a protein
diagnosis of acute myeloid leukaemia? B. Acute renal failure in a 65 year old woman
A. Bleeding into a knee joint in a 6 month old with hypercalcaemia, rib pain,
boy immunoparesis for lgG and lgA but no intact
B. Splenomegaly is frequently massive paraprotein
G. The leukaemic blasts proliferate excessively G. An lgG paraprotein of 10 g/L in a young
but differentiate normally woman with widespread joint pains and
D. The platelet count is most likely to be neutropenia
increased D. Hypercalcaemia in an elderly man with a high
E. The presence of a high white cell count alkaline phosphatase and sclerotic bone
but very low neutrophil count in an lesions
elderly man E. Pneumonia in a middle-aged man with
lymphopenia, increased polyclonal plasma
23.25. A 65 year old man presents to his doctor cells in the bone marrow and a polyclonal
with increasing shortness of breath and is rise in immunoglobulins
found to have an abnormal full blood count.
With regard to the myelodysplastic syndromes 23.28. A 56 year old man presents to hospital
(MDS), which of the following statements with abdominal discomfort and is found to have
is true? splenomegaly and an abnormal full blood
A. Anaemia is the most frequent presenting count. In CML, which of the following
abnormality and is commonly macrocytic statements is correct?
B. Erythropoietin (EPO) is not a useful therapy if A. Allogeneic stem cell transplantation
the renal function is normal remains the first -choice therapy in chronic
G. Ring sideroblasts are dysplastic white cells phase
and associated with mutations in the gene B. At presentation a majority of white cells are
SF381 blast cells
D. The median age of presentation is about G. The disease arises from a mutated stem cell
50 years containing the t(15;17) translocation

downloaded from www.medicalbr.com


HAEMATOLOGY AND TRANSFUSION MEDICINE • 265

D. The Philadelphia (Ph) chromosome is a small E. The presence of a TP53 mutation


chromosome 22 resulting from the predicts for a good response to
translocation t(9;22) chemotherapy
E. The platelet count is usually reduced
23.32. A 48 year old woman with general
23.29. Which of the following cases most likely malaise is found to have acute myeloid
represents a case of Hodgkin lymphoma (HL) leukaemia (AML). With regard to allogeneic
rather than non-Hodgkin lymphoma (NHL)? stem cell transplantation for patients with this
A. A 19 year old man with pancytopenia and a condition, which of the following statements
large mediastinal mass with pleural and is true?
pericardia! effusions A. Complications of the procedure lead to a
B. A 21 year old woman with painless, rubbery, treatment-related mortality (TRM) of about
cervical lymphadenopathy increasing over 3 1-5% by 2 years after transplantation
months and an asymptomatic mediastinal B. Older female donors are a better source of
mass on chest X-ray stem cells than younger male donors
c. A 25 year old woman with symptoms of C. Patients with chronic graft -versus-host
mediastinal obstruction and massive disease (GVHD) are more likely to relapse
mediastinal lymphadenopathy; the biopsy from AML after the transplant
shows sheets of centroblasts that are CD20 D. Reduced-intensity conditioning (RIC) is used
positive for very young, fit patients
D. A 50 year old woman with new-onset E. The anti-leukaemic effect originates
epilepsy and a mass in the right cerebral pri:Jdominantly from transplanted donor
hemisphere surrounded by oedema T lymphocytes (graft versus leukaemia;
E. A 60 year old man with widespread, GvL)
asymptomatic, painless lymphadenopathy
and splenomegaly 23.33. Which of the following best describes
the action of the anthracycline group of
23.30. A 68 year old man presents to his family chemotherapy drugs?
physician with fever, night sweats and A. They act as small-molecule inhibitors of
unexplained weight loss. Which one of the tyrosine kinases and include nilotinib
following is defined by low-grade, B. They act as antimetabolites and include
mature-looking lymphocytes that are CD5 and methotrexate
cyclin Dl positive but CD23 negative. C. They act as inhibitors of topoisomerase II
A. B-cell acute lymphoblastic leukaemia and include etoposide
(B-ALL) D. They act as inhibitors of tubulin
B. Chronic lymphocytic leukaemia (CLL) polymerisation and include vincristine
C. Diffuse large B-cell NHL (DLBL) E. They act to intercalate between
D. Follicular lymphoma double-stranded DNA base pairs and include
E. Mantle cell lymphoma daunorubicin

23.31. A 73 year old man is found to have 23.34. The first-line therapy for a 50 year old
some abnormalities in a full blood count taken man with severe aplastic anaemia is best
to investigate a symptom of fatigue. In patients described as follows?
with chronic lymphocytic leukaemia (CLL), A. A myeloablative allogeneic stem cell
which one of the following features is true? transplant from an unrelated donor
A. Most patients are symptomatic and require B. Immunosuppressive therapy with high-dose
treatment at presentation glucocorticoids
B. Patients with mutated immunoglobulin genes C. Immunosuppressive tperapy with horse
have a poorer prognosis than those with anti-thymocyte globulin (ATG) followed by
unmutated immunoglobulin genes ciclosporin
C. Signalling through the B-cell receptor complex D. Immunosuppressive tl:lerapy with rabbit
(BCR) is not a useful target for treatment ATG
D. The peripheral blood lymphocyte count is E. Supportive care only with red cells, platelets
persistently above 5x109/L and G-CSF

downloaded from www.medicalbr.com


266 • HAEMATOLOGY AND TRANSFUSION MEDICINE 1
I
23.35. A 68 year old woman has been found to Investigations show: haemoglobin 61 g/L;
have a low haemoglobin in the context of white cell count ryJCC) 5.2 x 109 /L; platelet
long-term rheumatoid arthritis and chronic count 84 x 109/L.
kidney disease. What does the mechanism of You request 4 units of packed red cells
the anaemia of chronic disease include? urgently. Group and screen 2 years previously
A. Chronic gastrointestinal (GI) tract bleeding showed group B Rhesus 0 (RhO)-positive with
B. High hepcidin levels inhibiting iron export a negative antibody screen.
from macrophages There is insufficient time to do compatibility
C. Iron toxicity to the bone marrow testing. What is the most appropriate blood to
D. Poor iron absorption because of transfuse?
achlorhydria A. AB RhO-negative
E. Red cell sequestration in a chronically B. B RhO-negative
enlarged spleen C. B RhO-positive
D. 0 RhO-negative
23.36. A 4 year old girl is brought to her doctor E. 0 RhO-positive
with fever and is found to have acute
leukaemia. Which of the following is considered 23.39. A 28 year old female, para 1 +0, presents
the most important prognostic feature for to obstetric triage at 27 weeks. All was well at
childhood acute lymphoblastic leukaemia? booking. She reports intermittent vaginal
A. T-cell rather than B-cell phenotype bleeding for the past 2 weeks. Other than
B. The age and sex of the child requiring a 3-unit red cell transfusion following
C. The chromosomal abnormalities acquired by a road traffic accident 3 years ago she has
the leukaemic cells been well.
D. The height of the white cell count at Investigations show: haemoglobin 121 g/L;
presentation WCC 8.4 x 109/l; platelet count 238 X 109/l.
E. The presence of minimal residual disease Group and screen: group A RhO-negative
(MRO) post induction therapy with a positive antibody screen.
Antibodies: anti-0 (titre 1/32)
23.37. A 6 year old patient has had severe What is the most likely reason for the
neutropenia (<0.2x109/L) lasting for more than development of the antibody?
7 days following initial treatment for AML. A. Fetal maternal haemorrhage during this
Which of the following is true? pregnancy
A. Azole prophylaxis against fungal infection B. Physiological increase in naturally occurring
with posaconazole is not recommended antibody
B. Gram-negative sepsis is more common than C. Previous pregnancy
Gram-positive sepsis D. Routine antenatal anti-0 prophylaxis
C. Indwelling Hickman lines are more commonly E. Transfusion
infected with Gram-negative organisms than
Gram-positive ones 23.40. A patient receives a blood transfusion
D. Neutropenic fever should be treated and has developed complications. Which of the
empirically prior to receiving the results of following combinations will result in least insult
blood cultures to the patient?
E. Ouinolone prophylaxis is not recommended A. Group AB red cells into a group 0 adult
to prevent sepsis B. Group B red cells into a group A adult
C. Group B red cells. into a group A neonate
23.38. A 42 year old woman with alcoholic liver D. Group 0 fresh frozen plasma (FFP) into a
disease presents as an emergency to hospital group A adult
with massive haematemesis. Her temperature is E. Group 0 FFP into a group B adult
3JDC, pulse rate 130 beats/min and blood
pressure is 70/40 mmHg. She is cool 23.41. A 28 year old female, para 1 + 1 , presents
peripherally. She has stigmata of chronic liver to obstetric triage at. 22 weeks. She reports
disease with a palpable spleen at 3 em below intermittent vaginal bleeding .for the past 24
the left costal margin. She is initially hours and thinks it is likely implantation
resuscitated with intravenous fluids. bleeding. There has been no preceding trauma.

downloaded from www.medicalbr.com


HAEMATOLOGY AND TRANSFUSION MEDICINE • 267

On examination she appears comfortable He is flushed but otherwise well.


at rest. Her temperature is 36.7"C, pulse Observations were normal before transfusion
rate is 88 beats/min and blood pressure is was started. How would you manage this
98/68 mmHg. Fundal height measures 22 em. clinical situation?
There are no concerns about the baby. A. Do nothing
Investigations: haemoglobin I 01 g/L; WCC B. Stop the transfusion, give chlorphenamine
8.4xi09/L; platelet count 238xi09/L. and, provided well, restart in 15 minutes
Group and screen: group 0 RhO-negative C. Stop the transfusion, give paracetamol and,
with a negative antibody screen. provided well, restart in 15 minutes
What is the most appropriate initial D. Stop the transfusion, initiate fluid
management for this patient? resuscitation and inform the hospital
A. Arrange a 2-unit red cell transfusion transfusion team
B. Arrange an urgent abdominal ultrasound to E. Stop the transfusion, take blood cultures,
exclude placental issues give antibiotics and return unit to blood
C. Commence her on oral iron bank as concern about bacterial
D. Reassure her that baby is well and bloods contamination
are normal for this stage of pregnancy
E. Take blood for a Kleihauer test and give 23.44. A 22 year old man is admitted following
anti-D a severe road traffic accident. He suffered
significant blood loss at the scene. He is
23.42. A 72 year old man with NHL attends the resuscitated with Gelofusine plasma expander
oncology assessment area post -chemotherapy and group 0 negative packed red cells.
with chest discomfort and shortness of breath Intra-operatively the anaesthetist phones you
at rest. His temperature is 37.4°C, pulse rate for advice.
is II 0 beats/min and blood pressure is Bloods show: haemoglobin 66 g/L; WCC
90/64 mmHg. 11.2xl 09/L; platelet count 83x 109/L; PT 28
Investigations: haemoglobin 81 g/L; WCC seconds; APTI 72 seconds; fibrinogen:
1.1 xi0 9/L; platelet count 40xi09/L. Troponin: 1.9 g/L.
awaited. ECG shows T-wave flattening. What is most appropriate to use to reduce
The patient agrees to a 2-unit red cell his bleeding risk?
transfusion. Which of the following actions is A. Cryoprecipitate
the most important one in ensuring a safe B. Fresh frozen plasma
transfusion without patient harm? C. Platelets
A. He confirms his name and date of birth D. Prothrombin complex concentrate
to the person taking his pre-transfusion E. Vitamin K
sample
B. He is provided with written patient 23.45. A 69 year old man with myelodysplasia is
information admitted with haernaturia. He is transfused with
C. He provides written consent 2 units of packed red cells. Eight hours
D. The reason for the transfusion is clearly following his transfusion you are called to
documented in the notes review him as he is now acutely unwell with
E. Transfusion is delayed until troponin is breathlessness and chest tightness.
available Observations reveal: temperature 37 .8°C,
pulse rate 130 beats/min, blood pressure
23.43. A 78 year old man with stable ischaemic 88/60 rnmHg, respiratory rate 34breaths/rnin
heart disease is admitted with a traumatic and oxygen saturation 82%. He is distressed
hip fracture. He undergoes a right with bilateral chest crepitations, R>L. Heart
hemi-arthroplasty. Twenty-four hours sounds are normal. Jugular venous pressure
post-operatively, investigations reveal a (JVP) is normal. ChestX-ray shows normal
haemoglobin of 65 g/L. Fifteen minutes into heart size, Kerley B lines and bilateral nodular
transfusion of the first unit of red cells, his infiltrates. Electrocardiography (ECG) shows
observations are: temperature 38.4°C; pulse sinus tachycardia.
rate 88 beats/min; blood pressure What is the most likely cause of the patient's
100/60 rnmHg. deterioration?

downloaded from www.medicalbr.com


268 • HAEMATOLOGY AND TRANSFUSION MEDICINE
t"
I
A. Myocardial infarction Investigations: haemoglobin 68 g/L; MCV
B. Pulmonary embolus 109 fl; WCC 5.3x109/L; platelet count
G. Sepsis 152 x 10 9/L; reticulocyte count 280 x 10 9/L;
D. Transfusion-associated circulatory overload alkaline phosphatase (ALP) 100 U/L; alanine
E. Transfusion-related acute lung injury aminotransferase (ALT) 54 U/L; bilirubin
32 J.!moi/L (1.87 mg/dl).
23.46. A 21 year old female medical student Blood film: polychromasia with spherocytes;
presents to her family physician with fatigue, no red cell fragments (see below).
shortness ofbreath and a sore mouth. On What is the most appropriate next
examination she is pale with a yellow tinge with investigation?
a palpable splenic tip on inspiration. Bloods are A. Bone marrow aspiration and trephine biopsy
checked. B. Direct antiglobulin (Coombs) test
Investigations show: haemoglobin 68 g/L; G. Hepatitis screen
mean cell volume (MCV) 118 fl; mean cell D. Serum vitamin B12 and folate
haemoglobin (MCH) 35 pg; WCC 1.9x109/L; E. Thyroid function tests
platelet count 92 x 109/L.
A blood film report reads: macrocytosis with 23.49. A 38 year old woman with known sickle-
macro-ovalocytes and hypersegmented cell disease presents to the emergency
neutrophils (see below). department on Christmas Eve with severe back
What is the most likely diagnosis? pain. Observations reveal: temperature 37.8"C,
A. Acute myeloid leukaemia pulse rate 90beats/min, blood pressure
B. Alcohol excess 118/80 mmHg, respiratory rate 18 breaths/min
G. Vitamin B12 deficiency and oxygen saturation 96%. On examination
D. Hyperthyroidism she is distressed. Radiological examination is
E. Iron deficiency unremarkable.
Investigations: haemoglobin 52 g/L; MCV
23.47. A 19 year old student presents to the 70 fl; WCC 12x109/L; platelet count
emergency department feeling sweaty and 384x10 9/L; C-reactive protein (CRP) 18 mg/L.
unwell over the last 24 hours. Her initial She is commenced on oxygen, broad-
observations show a temperature of 39.4"C, spectrum antibiotics, intravenous fluids and
pulse 120 beats/min, blood pressure analgesia, but fails to improve. What is the
94/64 mmHg, respiratory rate 22breaths/min most important next step in her management?
and oxygen saturations 99% on room air. A. Exchange transfusion
Investigations reveal: haemoglobin 118 g/L; B. Folic acid
MCV 84 fl; MCH 29 pg; WCC 33.0x10 9/L; G. LMWH
neutrophils 26x109/L; monocytes 4x10 9/L; D. Oral iron replacement
lymphocytes 3 x 1 09/L; platelet count E. Red cell transfusion
580x109 /L; blood glucose 6.9 mmoi/L
(124 mg/dl). 23.50. A couple attends for pregnancy
Blood film: neutrophilia with toxic granulation counselling. They both have beta-thalassaemia
(see below). trait.
What is the most likely cause of her Which of the following is correct?
leucocytosis? A. 1 in 2 chance the baby will be normal
A. Acute bacterial infection B. 1 in 4 chance the baby will develop hydrops
B. Acute myeloid leukaemia fetalis
G. Diabetic ketoacidosis G. 1 in 4 chance the baby will have
D. Infectious mononucleosis beta-thalassaemia trait
E. Systemic autoimmune disease D. 1 in 4 chance the baby will have
beta-thalassaemia major
23.48. A 56 year old Scottish woman, previously E. Pregnancy is unlikely to be successful
fit and well, presents to her family physician
with a 2-week history of increasing tiredness. 23.51. A 16 year old boy with known sickle-cell
On examination she is pale and jaundiced. disease is brought to the emergency
Bloods are checked. department profoundly unwell. He has ·a

downloaded from www.medicalbr.com


HAEMATOLOGY AND TRANSFUSION MEDICINE • 269

reduced conscious level, temperature 39°e, C. Infection


pulse rate 160 beats/min and blood pressure D. Intrauterine growth retardation
70/40 mmHg. E. Neonatal alloimmune thrombocytopenia
Investigations: haemoglobin 78 g/L; MeV
68 fl; wee 1 7 X 109/L; platelet count 23.54. A 25 year old man wishes to be a blood
480 x 109/L; reticulocyte count 120 x 109/L; eRP donor. Which of the following will his blood not
41 mg/L. be screened for evidence of exposure to?
Blood film: neutrophilia with toxic granulation; A. Hepatitis A
red cell changes consistent with sickle cell B. Hepatitis B
disease and hyposplenism (see below). C. Hepatitis e
What is the most likely clinical diagnosis? D. Human T-lymphotrophic virus (HTLV)
A. Aplastic crisis E. Syphilis
B. Infectious mononucleosis
c. Meningococcal sepsis 23.55. A 68 year old woman with ischaemic
D. Painful vasa-occlusive crisis heart disease and rheumatoid arthritis presents
E. Sickle chest syndrome with worsening angina.
Bloods show: haemoglobin 98 g/L; MeV
23.52. A pale and lethargic 2 year old boy is 80 fl; MeH 29 pg; wee 7x109/L; platelets
brought to the emergency department by his 300x 10 9/L; iron 6 ~moi/L (33.5 ~g/dl);
parents. He recently started nursery. His transferrin saturation 12%; total iron-binding
temperature is 36.9°e, pulse rate is 110 beats/ capacity 33 ~moi/L; vitamin B12 400 ng/L;
min and blood pressure is 90/40 mmHg. His folate 8 ~g/L.
urine is dark brown in colour. What is the most likely cause of her
Investigations: haemoglobin 60 g/L; MeV anaemia?
68 fl; MeH 31 pg; wee 9.3x 109/L; platelet A. Anaemia of chronic disease
count 330 x 109/L; reticulocyte count B. Autoimmune haemolysis
240x109/L; ALT 61 U/L; ALP 108 U/L; bilirubin C. Folate deficiency
60 ~moi/L (3.5 mg/dl). D. Iron deficiency
Blood film shows polychromasia with bite E. Myelodysplasia
and blister cells.
What is the most appropriate next 23.56. A 57 year old woman has just started
investigation? a blood transfusion following coronary
A. Direct antiglobulin test bypass grafting yesterday. What may
B. Donath-Landsteiner assay happen 30 minutes later as a result of the
C. Glucose-6-phosphate dehydrogenase transfusion?
(G6PD) assay A. She becomes hypothermic
D. Mid-stream urine (MSU) B. She becomes iron overloaded
E. Parvovirus serology C. She becomes thrombocythaemic
D. She develops an urticarial rash
23.53. A previously well 26 year old woman E. She develops hypokalaemia
presents at term, after an uneventful
pregnancy, in the late stages of labour. This is 23.57. A 36 year old man receives a platelet
her first pregnancy. Delivery is uneventful; the transfusion prior to Hickman line removal
baby is well but is noted to have a widespread following a stem cell transplantation for acute
petechial rash. myeloid leukaemia. Ten minutes after the
Investigations: haemoglobir;J 168 g/L; Mev transfusion starts, he becomes acutely
90 fl; wee 12x109/L; platelet count 7x109/L; breathless, hypotensive and febrile. Despite
PT 12 seconds; APTT 30 seconds; fibrinogen intensive resuscitation he..dies.
4.0 g/L. What is the most likely cause for his death?
Blood film: true thrombocytopenia. A. Bacterial contamination
What is the most likely cause of the low B. Metabolic-induced ca~diac arrhythmia
platelets? C. Pulmonary embolism
A. Disseminated intravascular coagulation D. Transfusion-associated circulatory overload
B. Immune thrombocytopenic purpura E. Transfusion of an ABO-incompatible unit

downloaded from www.medicalbr.com


270 • HAEMATOLOGY AND TRANSFUSION MEDICINE

Answers
23.1. Answer: E. 23. 7. Answer: A.
Follicular lymphoma and chronic lymphocytic All of the other circumstances are licensed
leukaemia (CLL) are systemic malignancies with indications for the use of rivaroxaban except for
involvement of lymphoid tissue in many sites. the management of patients with prosthetic
HIV and Epstein-Barr virus (EBV) infection heart valves. CHA,DS2 -VASc is a well"known
produce generalised lymphadenopathy and scoring system to evaluate risk of thrombosis in
systemic illness. Cancers tend to metastasise a patient with atrial fibrillation.
to local regional nodes draining a specific
tissue and so breast cancer tends to present 23.8. Answer: A.
with localised unilateral axillary HIT is most commonly seen in surgical patients,
lymphadenopathy. especially following major orthopaedic and
cardiac surgery. HIT is more commonly seen
23.2. Answer: D. when UFH is used compared with low-
All of these mutated genes are associated with molecular-weight heparin (LMWH); it is
haematological conditions: BCR with chronic commonly associated with moderate as
myeloid leukaemia (CML), c-MYC and BCL-2 opposed to severe thrombocytopenia and it is
with lymphoma, and MYH-9 with congenital associated, somewhat paradoxically, with
thrombocytopathy. Only CAL-R, calreticulin, is thrombotic events. The key period for
associated with the myeloproliferative neoplasm .developing HIT is after 5-10 days of exposure,
essential thrombocythaemia. with longer exposures less likely to be
associated.
23.3. Answer: B.
The most common complications of PRV 23.9. Answer: D.
involve vascular occlusion. All of the conditions All of these conditions can present with a
listed are associated with PRV, but the most normal PT and a prolonged APTI. Lupus
common is ischaemic stroke. anticoagulant is very rarely associated with a
bleeding diathesis. Factor XII deficiency causes
23.4. Answer: E. a very marked prolongation of the APTI but is
Vancomycin is associated with never associated with bleeding. Severe XI
immune-mediated thrombocytopenia - the deficiency is associated with variable severity of
others are not. bleeding and is rare. Factor VIII and IX
deficiencies present with identical phenotypes
23.5. Answer: D. but severe haemophilia A is 5 times more
Haemarthrosis, muscular haematoma and common than severe haemophilia B. The
retroperitoneal haemorrhage more commonly scenario is classical of the first presentation of
complicate bleeding disorders associated with severe haemophilia A or B.
reduced levels of coagulation factors.
Intracranial haemorrhage can complicate 23.10. Answer: C.
severe thrombocytopenia or severe coagulation She has a low factor VIII level, which is
factor deficiency but oral mucosal bleeding compatible with all the diagnoses given.
is by far the most common feature of However, she has a level of functional vWF
severe thrombocytopenia along with skin (RiCO) that is out of keeping with her vWF
purpura. antigen level (ratio is 0.35). This suggests a
dysfunctional molecule and therefore type 2
23.6. Answer: D. vWD. Type 2N vWD is associated with isolated
Dalteparin is a low-molecular-weight heparin low factor VIII and so' the most likely diagnosis
(LMWH), the effect of which is mediated by here is type 2A vWD. This is a common
enhanced avidity of antithrombin for its natural presentation in affected young women.
substrates, thrombin and factor Xa. Apixaban
and edoxaban are direct-acting inhibitors of 23.11. Answer: E.
factor Xa, while dabigatran and bivalirudin are Epistaxis, easy bruising and bleeding after
direct thrombin inhibitors. shaving are all symptoms commonly reported

downloaded from www.medicalbr.com


HAEMATOLOGY AND TRANSFUSION MEDICINE • 271

in patients without a bleeding disorder and so 23.17. Answer: B.


are very non-specific indicators of a bleeding Eosinophilia is often seen in association with
disorder. Post-partum haemorrhage is usually atopic and allergic conditions. It can also be
associated with other pregnancy-associated seen in pulmonary vasculitis. Other causes of
illness, uterine atony or vaginal trauma. reactive eosinophilia include drug reactions and
parasitic infections.
23.12. Answer: B.
Factor V Leiden is a gain-of-function mutation 23.18. Answer: E.
found in around 5% of northern Europeans. Anticoagulation with LMWH produces minor
The others are all less commonly found prolongation of APTT only. Disseminated
abnormalities associated with venous intravascular coagulation can complicate
thromboembolic (VTE) risk. pancreatitis but would be expected to be
associated with reduced platelet count and
23.13. Answer: D. fibrinogen. Liver failure, again, can result in
The TKR is too remote in time to be relevant; prolonged APTT and PT, but in advanced
however, it is a strong risk factor for VTE in the disease you would expect the fibrinogen level
35 days post-surgery. Quinine can result in the to fall. Vitamin K deficiency results in
development of thrombocytopenia and a prolongation of PT and APTT with a normal
systemic reaction but is not associated with fibrinogen level and is a common complication
VTE. Travel is a common but weak risk factor of hepatobiliary surgery.
for thrombosis and only journeys of >3 hours of
continuous travel are relevant. A cousin is not a 23.19. Answer: D.
first -degree relative and the event suffered by The described appearances are all part of a
this man's cousin was provoked. CCF, or hyposplenic picture. Irrespective of the cause of
indeed any significant hospitalisation, is a the hyposplenism or splenectomy. Liver disease
strong risk factor for subsequent VTE - The is associated with target cells but not with a
Million Women Study suggests the period of hyposplenic picture. Myelofibrosis is associated
attributable risk is about 12 weeks. with target cells, teardrop poikilocytes,
polychromasia and a leucoerythroblastic
23.14. Answer: E. appearance but not with the features of
The background to the event is the strongest hyposplenism.
predicting factor for recurrence. Unprovoked
events have a far higher rate of recurrence than 23.20. Answer: E.
provoked events where the provoking factor The oxygen dissociation curve shifts to the right
has been removed. In predicting recurrence - i.e. improves the release of oxygen to tissues
after an unprovoked event, male sex and - in conditions where the pH is decreased
elevated D-dimer are predictive of recurrence. (Fig. 23.20). The other changes would tend to
move the dissociation curve to the left, making
23.15. Answer: D.
Purpura is rare in TTP despite the name of the Normal
disorder. In conditions where prothrombotic ~ venous P0 2
~ 100
states are associated with thrombocytopenia, it :0
.Q Shift to left-
is often the case that there is a lack of purpura: 0>
0 75 12,3-BPG
E
e.g. in haemolytic uraemic syndrome (HUS) and Q) IW -Shift to right
heparin-induced thrombocytopenia. "'
.c
50 1co2 12,3-BPG
b I Temperature
c tw
0
I C02
23.16. Answer: D. ~::J 25 !Temperature
Factor XIII is cleaved and.the A subunit then 1ij
(f)
cross-links fibrin to form stable clot. 0
0 2 4 6/ s io 12
Thrombomodulin binding results in activation of
P02 (kPa)
the protein C pathway, which has an
anticoagulant effect. Activation of factor VIII Fig. 23.20 The haemoglobin-oxygen dissociation curve.
Factors are listed that shift the curve to the right (more oxygen
amplifies coagulation activation and the
released from blood) and to the left (less oxygen released) at
cleavage of fibrinopeptides results in production given P02. To convert kPa to mmHg, multiply by 7.5. (2,3-BPG =
of fibrin monomers. 2,3-bisphosphoglycerate)

downloaded from www.medicalbr.com


272 • HAEMATOLOGY AND TRANSFUSION MEDICINE

oxygen release to tissues less readily. HbS Splenomegaly is hardly ever massive, which is
does not have any affect on oxygen a feature of chronic myeloid leukaemia (CML).
dissociation. Bleeding is into skin and mucous membranes
because of thrombocytopenia, not joints as in
23.21. Answer: B. haemophilia.
Low measured vitamin B12 level is common in
pregnancy although vitamin B12 deficiency is 23.25. Answer: A.
rare - indeed decreased fertility is associated MDS are diseases of the elderly, with a median
with vitamin B12 deficiency. Protein C levels rise age of over 70 years. Anaemia is the
in pregnancy, protein S levels fall. Procoagulant commonest abnormality, occurring in 80% of
factors like factor VIII and fibrinogen increase as patients at some point, and is usually
pregnancy progresses. Pregnant patients tend macrocytic. Patients with anaemia and low
to have increased plasma volume and, if transfusion requirement and a baseline EPO
anything, tend to be anaemic rather than level of <200 U/L have a 70% chance of
polycythaemic. Lupus anticoagulant is never a responding to EPO therapy, independent of
normal finding. renal function. Dysplasia is present in more
than 10% of an affected bone marrow lineage
23.22. Answer: C. and ring sideroblasts are one form of dysplastic
The normal spleen is rarely if ever felt and a red cell. Mutations in SF381 (a splicing factor)
palpable spleen has the clinical features of are very strongly associated with the presence
moving down on inspiration as the diaphragm of ring sideroblasts. Progression to AML is
contracts and upwards (away from the hand) diagnosed when the blasts are over 20%.
on expiration. The examining hand cannot get
above the spleen and under the left costal 23.26. Answer: C.
margin. Inflammation of the splenic capsule In MGUS, the paraprotein is at a low level and
following infarction leads to an audible rub over with no evidence of end-organ damage, i.e. no
an acutely painful spleen. evidence of anaemia, bone disease or renal
disease. lgM paraproteins are associated with
23.23. Answer: D. low-grade lymphomas, most notably
Stem cells are rare, accounting for about 0.1% lymphoplasmacytic lymphoma, but can present
of marrow cells. They cannot be identified on as MGUS if there is no clinical evidence of
routine morphology and require immunological lymphoma. A normal serum free light chain
staining for identification. They have the ratio in MGUS carries a very good prognosis.
important characteristics of self-renewal
(daughter cells can remain as stem cells rather 23.27. Answer: B.
than differentiating into mature blood cells) and Myeloma is a disease of middle to old age. It is
pluripotency (they give rise to cells of different never seen in children and extremely rare under
lineages depending on requirements). Stem the age of 30 years. The diagnosis requires a
cells circulate in the blood in small numbers paraprotein (including light chain only as in
normally and these can be increased up to option B) with signs of end-organ damage and
I 000-fold by mobilisation procedures following usually with an increase in bone marrow
chemotherapy, G-CSF or plerixafor. In this way, monoclonal plasma cells. Option ·o is most
stem cells can be harvested from the blood for likely metastatic prostate cancer; option C is
transplantation. autoimmune disease, e.g. rheumatoid arthritis;
and option E would fit with HIV infection.
23.24. Answer: E. Normal light chains can appear in the urine in
The hallmark of acute leukaemia is bone acute renal failure because of failed
marrow failure: the presence of one or more of reabsorption by the renal tubules, as in
anaemia, thrombocytopenia and neutropenia. option A
This is because the leukaemic blast cells
proliferate but fail to differentiate normally. The 23.28. Answer: D.
disease is most common in older adults. The CML arises from a mutated stem cell
total white cell count can be low, normal or containing the Ph chromosome, which results
high, depending on how many of the blasts from the translocation t(9;22) and the resulting
escape the marrow into the blood. fusion gene BCR-ABL. The leukaemic clone

downloaded from www.medicalbr.com


HAEMATOLOGY AND TRANSFUSION MEDICINE • 273

proliferates and differentiates so in chronic 23.32. Answer: E.


phase the blast count is low and platelets The donor-derived T cells provide a new
tend to be high (sometimes very high: cell-mediated immune system that attacks
> 2000 xI 09/L). First -choice therapy for all residual leukaemic cells - GvL. Maintaining
patients in chronic phase is tyrosine kinase full (I 00%) donor chimerism in the T-cell
inhibition (TKI), e.g. imatinib, nilotinib or population promotes long-term remission of
dasatinib, with allogeneic stem cell AML and other haematological malignancies.
transplantation reserved for TKI failure. The The T cells also cause acute and chronic
translocation t(l5; 17) occurs in acute GVHD, which for similar reasons is associated
promyelocytic leukaemia (APL). with lower relapse rates. Older females are
more likely to have been pregnant and to be
23.29. Answer: B. sensitised to human leucocyte antigens (HLAs)
HL most commonly starts in the cervical region and therefore younger male donors are
and spreads contiguously along the lymphatic preferable, all other criteria such as HLA
channels. Young women with the nodular matching and cytomegalovirus (CMV) status
sclerosing subtype of HL typically present with being equal. RIC is less tissue damaging and
cervical and mediastinal lymphadenopathy. The safer for older patients; however, younger and
nodes are typically painless and rubbery or fitter patients benefit from full conditioning. The
sometimes firm or hard if there is lots of TRM for allogeneic transplants after 2 years is
sclerosis. All the other answers are more in about 20%, mainly from infection and GVHD.
keeping with forms of NHL: option C, primary The figure of 1-5% would apply to autologous
mediastinal large B-cell lymphoma; option E, stem cell transplantation.
follicular lymphoma; option D, primary central
nervous system lymphoma; option A, 23.33. Answer: E.
T-lymphoblastic lymphoma. Anthracyclines lead to the inhibition of DNA and
RNA synthesis by intercalating between base
23.30. Answer: E. pairs of the DNNRNA strand, thus preventing
Mantle cell lymphoma tends to present with the replication of cancer cells. Some
morphologically low-grade looking mature anthracyclines also inhibit topoisomerase II,
lymphocytes that are positive for CD5 but but etoposide is not an anthracycline class
negative for CD23. Mantle cell lymphoma of topoisomerase II inhibitor. All other
harbours the translocation t(ll ;14), which leads answers describe a different class of
to overexpression of cyclin Dl. CLL tends to chemotherapy drug.
be CD5 and CD23 positive, whilst all the other
conditions tend to be CD5 negative. ALL will 23.34. Answer: C.
have blast cells, not mature lymphocytes, and The treatment of choice is immunosuppression
DLBL is also a high-grade lymphoma with with horse ATG, which is safer than currently
centroblasts. available rabbit ATG. Horse ATG should be
followed by ciclosporin as longer-term
23.31. Answer: D. immunosuppression. Allogeneic stem cell
This is a requirement for CLL diagnosis. transplantation is reserved as first -line treatment
Patients with lower CLL lymphocyte counts are for young patients (<30 years), ideally with a
termed monoclonal B-cell lymphocytosis of fully matched sibling donor. Supportive care is
uncertain significance. Some patients present important but not as sole therapy and there is
with lymphadenopathy and lymphocyte counts no proven useful role for G-CSF. Long-term
< 5 x 109 /L and are called small cell lymphocytic glucocorticoids dramatically increase the risk of
lymphoma. Most patients are asymptomatic at fatal fungal infection in neutropenic patients.
presentation and undergo watch and wait.
Mutated immunoglobulin genes carry a good 23.35. Answer: B.
prognosis and TP53 mutations carry a poor The anaemia of chronic disease largely results
prognosis and a tendency to chemotherapy from induced hepcidin production from the
resistance. Targeting the BCR complex with liver. This occurs secondaf),' to increased
drugs such as ibrutinib (Bruton's tyrosine interleukin-6 (IL-6) levels in inflammation. Iron is
kinase inhibitor) is a successful new form of trapped in iron-exporting cells, including
therapy. duodenal enterocytes and macrophages. Iron

downloaded from www.medicalbr.com


T'
· 274 • HAEMATOLOGY AND TRANSFUSION MEDICINE

does not cause direct marrow toxicity. Chronic RhO-negative can safely be given to any patient
Gl tract bleeding leads to iron deficiency
anaemia, as does achlorhydria, e.g.
as it lacks isohaemoglutinins that can react
against the patient's red cell surface antigens.
I
post-gastrectomy. Option E describes
I
hypersplenism. 23.39. Answer: A. !
A female of child-bearing age would have
23.36. Answer: E. received RhO- and Kell-negative units. Routine
All have been recognised prognostic factors at antenatal anti-0 prophylaxis is administered at
one time or another but the presence of MRO 28 weeks. Anti-0 only occurs in RhO-negative
is now recognised as the most significant individuals exposed to exogenous 0 antigen,
prognostic factor and used to modify therapy and has clinical implications. She has not been
accordingly. pregnant before. It is most likely that her
partner is 0 antigen positive and the fetus has
23.37. Answer: D. inherited this antigen causing maternal
Nowadays the use of indwelling lines and allo-anti-0 to form.
antibiotic prophylaxis with quinolone antibiotics
(e.g. levofloxacin, ciprofloxacin) leads to more 23.40. Answer: C.
identified Gram-positive infections than Group 0 FFP contains both anti-A and anti-B,
Gram-negative, although Gram-negative which will react with the recipient's red cells
infections are still more life-threatening. expressing A or B antigens. Group 0 patients
Quinolone prophylaxis and posaconazole have anti-A and anti-B that will react with A
prophylaxis have both been shown to be and or B antigens expressed on transfused
beneficial and to reduce mortality during cells. Group A patients have anti-B, which will
induction therapy for AML. Indwelling plastic react with B antigens expressed on transfused
lines are more commonly infected with cells. The neonate immune system does not
Gram-positive organisms, e.g. Staphylococcus produce antibodies when exposed to
epidermidis. Neutropenic sepsis is a medical exogenous antigens and so neonates tolerate
emergency and must be treated empirically, ABO incompatibility better than adults.
e.g. with piperacillin/tazobactam with or without
aminoglycosides, whilst waiting for culture 23.41. Answer: E.
results. Positive cultures only occur in about A Kleihauer test or acid elution test is a blood
30% of episodes of neutropenic sepsis. test used to measure the amount of fetal
haemoglobin transferred from a fetus to a
23.38. Answer: D. mother's blood stream. It is vital not to miss
In an emergency, group 0 red cells can be this sensitising event. Implantation bleeds occur
given safely to any patient as the group 0 red in the first trimester. Although exclusion of
cells lack surface antigens against which the placental abnormality is required, the priority is
patient's isohaemoglutinins can react (Box to minimise the likelihood of RhO sensitisation
23.38}. There is a historic ABO and Rhesus 0 occurring. Normal pregnancy causes a fall in
type available; however, group-specific units the haemoglobin through haemodilution for
can only be issued when the group has been which transfusion is not indicated. Red cell
confirmed from a current sample. As this parameters will guide the need for iron
woman has child-bearing potential, she should replacement.
receive RhO-negative. Plasma of group AB
23.42. Answer: A.
It is good practice to explain and document the
reason for transfusion to the patient and to
23.38 ABO blood group antigens and antibodies obtain the patient's consent but this may
ABO blood Red cell. A or Antibodies UK frequency not always be possible. Key steps in the
group B antigens in plasma (%) transfusion process are to positively identify the
0 None Anti-A and 46 patient at the bedside prior to taking a blood
anti-B sample and before administering a blood
A A Anti-B 42 component. As he has evidence of
B B Anti-A 9 cardiorespiratory compromise, transfusion
AB A and B None 3
should not be withheld.

downloaded from www.medicalbr.com


HAEMATOLOGY AND TRANSFUSION MEDICINE • 275

23.43. Answer: C. 23.48. Answer: B.


All symptoms and signs of a possible Investigations are in keeping with haemolysis. A
transfusion reaction require rapid clinical positive direct antiglobulin (Coombs) test would
assessment of the patient after the transfusion support an autoimmune aetiology, the most
has been stopped (Fig. 23.43). His symptoms likely cause of this woman's haemolysis.
are consistent with a febrile non-haemolytic A bone marrow aspirate is not indicated.
transfusion reaction. Spherocytes are not associated with
hepatitis, vitamin B 12 or folate deficiency, or
23.44. Answer: B. hypothyroidism.
His PT and APTI are significantly prolonged;
therefore, FFP is indicated at a dose of 15 mU 23.49. Answer: E.
kg. He will likely require platelets; however, This patient is experiencing a vasa-occlusive
at present, they are greater than the crisis with compromised blood flow to the small
recommended threshold of 50 x 109/L. vessels. A red cell transfusion will facilitate
Cryoprecipitate is primarily used to replace oxygenation and reverse the sickling process.
fibrinogen when below 1 .5 g/L. Prothrombin Exchange is reserved for life-threatening crises.
complex concentrate is used to reverse Folic acid and LMWH are indicated but not the
warfarin. Vitamin K has a 4- to 6-hour onset of most important next step. Sickle-cell disease is
action. associated with small cells and normal iron
metabolism.
23.45. Answer: E.
Transfusion-related acute lung injury is 23.50. Answer: D.
characterised by the sudden onset of Beta-thalassaemia trait is not associated with
breathlessness within 6-24 hours of transfusion reduced fertility. With appropriate antenatal
with bilateral nodular infiltrates on X-ray. This care, pregnancy outcome is successful.
condition results in pulmonary oedema but is Hydrops fetalis is associated with alpha-chain
non-cardiac in origin. Temperature, JVP and abnormalities. The baby has a 1 in 4 chance it
EGG are normal. There is no history of pleuritic will not inherit the beta-thalassaemia gene, a 1
chest pain. Although the other conditions in 2 chance it will inherit one beta-thalassaemia
should be considered, the pattern is most gene and a 1 in 4 chance it will inherit two
consistent with transfusion-related acute lung beta-thalassaemia genes and develop
injury. beta-thalassaemia major.

23.46. Answer: C. 23.51. Answer: C.


Vitamin B12 deficiency results in ineffective Patients with sickle-cell disease have
haematopoiesis with nucleocytoplasmic compromised splenic function resulting from

~
asynchrony. Iron deficiency results in anaemia microvascular occlusion. This puts them at risk
with small, pale red cells. Acute myeloid of life-threatening infections from capsulated
leukaemia can cause a pancytopenia but is organisms. Aplastic crises produce a very low
associated with circulating immature blast cells. haemoglobin with a reticulocytopenia. There is
Alcohol excess can result in a macrocytosis no history of joint pain or shortness of breath to I
and poor diet, usually leading to folate suggest a vasa-occlusive painful crisis or sickle
deficiency. Hypothyroidism can cause a chest syndrome.
macrocytosis and fatigue with normal neutrophil
appearances on film. 23.52. Answer: C.
This child has non-spherocytic haemolysis.
23.47. Answer: A. G6PD deficiency is the most common cause in
The blood film shows an increase in neutrophils this age group. The Donath-Landsteiner
with heavily staining granules consistent with antibody is found in paroxysmal cold
bacterial infection. There are no immature blast haemoglobinuria. The direct antiglobulin test
cells. Infectious mononucleosis is associated detects antibody-coated red cells found in
with a lymphocytosis. Whilst diabetic autoimmune haemolysis .•Parvovirus is
ketoacidosis can cause a neutrophilia, the associated with anaemia and reticulocytopenia.
blood glucose is normal. Autoimmune disease The urine discolouration is caused by
is associated with a neutropenia. haemosiderinuria.

downloaded from www.medicalbr.com


'+'
I
276 • HAEMATOLOGY AND TRANSFUSION MEDICINE i
'
I
Symptoms/signs of possible acute transfusion reaction
o Fever, chills, tachycardia, hyper- or hypotension, collapse, rigors, flushing,
urticaria, bone, muscle, chest and/or abdominal pain, shortness of breath,
nausea, generally feeling unwell, respiratory distress

Stop the transfusion


o Undertake rapid clinical assessment, including temperature, pulse, BP,
respiratory rate and 0 2 saturation
o Check the identity of recipient details on the unit and compatibility form

Reaction involves mild fever


Fever or urticarial rash only? Urticaria'

Febrile non-haemolytic transfusion reaction Mild pruritus/rash


If Isolated temperature~ 3B"C, or rise of 1-2'C, o Give chlorphenamine 1 0 mg slowly llr
: obseryatloos are stable and patient is otherwise well No i o Restart the transfusion at a slower rate and observe
o Give paracetamot• more frequently
o Restart Infusion at a slower rate and observe more
frequently

Suspected ABO incompatibility?


:~Yes\ oWrong blood pack infused
oHaemoglobinuria

ABO incompatibility No'


o Take down unit and giving set; return
intact to blood bank
o Commence IV saline Infusion
• Monitor urine output/catheterise
Maintain urine output at> 100 mUhr Yes,
Give furosemide if urine output falls•
• Treat DIG wHh appropriate blood
compenents
• lnfonn hospital transfusien department No'
' immediately

Bacterial contamination?
o Blood pack discoloured or damaged
; Yes i o Rapid onset of hyper- or hypotension, rigors or collapse
o Temperature;;, 39'C or rise of;;, 2'C -

' Bacterial infection of unit Severe allergic reaction


· o Take down unit and giving set/return intact to blood o Discontinue transfusion
bank with all other used/unused units o, Give chlorphenamine 10 mg slowly tv•
• Take blood cultures, repeat blood group/cross-match/ o Commence 0 2 and fluid support
, FBC, coagulation screen, biochemistry, urinalysis i, No o Give salbutamol nebuliser
· o Monitor urine output o If severe hypotension or bronchospasm, give
o Commence broad-spectrum antibiotics If suspected adrenaline (epinephrine) 0.5 mg tM•
, bl!()terial infection (Ch. 6) o Send clotted blood sample to transfusion laboratory
• Commence oxygen and fluid support • Take down unit and giving set, and return intact to
! •:seek advice · blood bank with all other used/unused units

If acute dyspnoea/hypotension
o Monitor blood gases
o Perform chest X-ray , .Normal\
o Measure central venous/pulmonary ' CVP
capillary pressure

Fluid overload Transfusion-related acute lung injury (TRALI)


: oGlva oxygen and furosemide 40-BO mg IV" • Typically within 6-24 hrs of transfusion
• Breathlessness, non-producttve cough
o Chest X-ray bilateral nodular infiltration
o Discontinue transfusion
• o Give 1DO% oxygen
, o Treat as ARDS -ventilate if severely hypoxaemic

Fig. 23.43 Investigation and management of acute transfusion reactions. ·use size-appropriate dose in children. (ARDS ~ acute
respiratory distress syndrome; BP =blood pressure; CVP =central venous pressure; DIG~ disseminated intravascular coagulation; FBC
~ lull blood count; IV ~ intravenous)

downloaded from www.medicalbr.com


HAEMATOLOGY AND TRANSFUSION MEDICINE • 277

23.53. Answer: E. chronic disease. Anaemia of chronic disease


Neonatal alloimmune thrombocytopenia occurs results from impaired iron handling. In iron
when maternal alloantibodies form against deficiency anaemia, the patient is iron depleted,
fetal platelet antigens inherited from the father. leading to a raised iron-binding capacity in the
This can result in severe life-threatening context of low iron and transferrin saturation.
thrombocytopenia in utero or at delivery, and in Folate deficiency, myelodysplasia and
first pregnancy. Immune thrombocytopenic autoimmune haemolysis are associated with an
purpura is unlikely given the absence of a elevated MCV.
history of maternal immune thrombocytopenia.
The coagulation is normal. Aside from the 23.56. Answer: D.
petechial rash, there are no concerns regarding Non-specific allergic reactions can occur during
the baby. The platelet count will recover a transfusion. Iron overload is associated with
spontaneously. repeated red cell transfusion. Massive
transfusion is associated with hyperkalaemia,
23.54. Answer: A. hypothermia and thrombocytopenia.
Hepatitis A is transmitted through the
faeco-oral route. All the other infective agents 23.57. Answer: A.
are transmitted through direct contact with Bacterial contamination of platelets can result in
infected blood or bodily fluids. The rate of rapid onset of acute circulatory compromise.
transfusion-transmitted infection is monitored It is thought to occur secondary to exposure
through national haemovigilance schemes. The to significant bacterial toxin within the
UK Serious Hazards of Transfusion (SHOT) contaminated platelet unit rather than the
scheme has reported on proven or suspected bacterial load itself. Transfusion-associated
cases of infection transmission arising from circulatory overload is associated with
blood and component use in the UK since transfusion of large volumes relative to the
1996. patient. Whilst transfusion can cause
hyperkalaemia, this is in the context of massive
23.55. Answer: A. transfusion. Pulmonary emboli are not generally
She has a normochromic, normocytic anaemia associated with fever. ABO incompatibility
with low iron, iron-binding capacity and resulting in catastrophic circulatory collapse is
transferrin saturation in keeping with anaemia of most often associated with red cell transfusion.

downloaded from www.medicalbr.com


SH Ralston, GPR Clunie

Rheumatology and
bone disease

Multiple Choice Questions


24.1. Which cell in the bone microenvironment C.· Gabapentin
is primarily responsible for coordinating the D. Paracetamol
regulation of bone remodelling? E. Tramadol
A. Bone lining cells
B. Bone marrow stromal cells 24.4. A 65 year old woman with osteoarthritis
C. Osteoblasts attends her family physician because of knee
D. Osteoclasts pain worse on going up- and downstairs. She is
E. Osteocytes a non-smoker, does not drink alcohol, but is
taking simvastatin 20 mg daily for high
cholesterol and lisinopril 10 mg daily for
24.2. Within the bone microenvironment, which hypertension. On examination her height is
of the following receptors are responsible for 154 em and weight is 95 kg. What would be the
activation of bone resorption and bone most appropriate treatment for her knee pain?
formation, respectively?
A. Arthroplasty
A. Osteoprotegerin and Wnt B. Cognitive behavioural therapy (CBT)
B. Osteoprotegerin and sclerostin C. Regular weight-bearing exercise
C. Receptor activator of nuclear factor kappa B D. Surgical synovectomy
ligand (RANKL) and osteoprotegerin E. Weight loss
D. Receptor activator of nuclear factor kappa B
(RANK) and lipoprotein receptor-related 24.5. Which of the following disease-modifying
protein 5 (LRP5) antirheumatic drugs (DMARDs) does NOT
E. RANKL and sclerostin require any monitoring of full blood count or
liver function tests?
24.3. A 75 year old man presents with pain of A. Azathioprine
moderate severity affecting the base of both B. Hydroxychloroquine
thumbs. Hand X-rays shows evidence of C. Leflunomide
osteoarthritis at both first carpometacarpal D. Methotrexate
(CMC) joints. Local application of gel containing E. Myocrisin
diclofenac has not helped his symptoms. What
would be the next most appropriate drug 24.6. A previously healthy 45 year old man is
treatment for his pain? woken up at 0400 hrs with acute pain, swelling
A. Co-codamol and redness of the right ankle with no obvious
B. Diclofenac trigger factor. Height is 168 em, weight 104 kg.

downloaded from www.medicalbr.com


RHEUMATOLOGY AND BONE DISEASE • 279

He has a history hypertension treated with (MCP) joints of the hands, gradually worsening
bendroflumethazide 2.5 mg daily. He drinks over a period of 6-8 weeks. On examination,
2-3 pints of beer each night and consumes 26 there is symmetrical swelling and tenderness of
units of alcohol per week. What is the most both wrists and the MCP and PIP joints of the
likely diagnosis? hands. Investigations show that anti-citrullinated
peptide antibodies (ACPAs) and rheumatoid
A. Gout
B. Osteoarthritis factor are negative, but that she has an elevated
c. Psoriatic arthritis ESR (25 rnrn/hr) and a raised CRP (65 rng/L). ·
D. Rheumatoid arthritis X-rays of the hands are normal. Which of the
E. Septic arthritis following statements is correct?
A. Magnetic resonance imaging (MRI) of the
24.7. A 77 year old woman with a history of hands should be done to clarify the diagnosis
generalised osteoarthritis (OA) is admitted to B. Rheumatoid arthritis is excluded by the
hospital with a delirious episode associated negative ACPA test and normal radiographs
with dehydration and a urinary tract infection. C. The joint pain and swelling is most likely due
During the admission, she develops pain, to generalised osteoarthritis
swelling and redness of the left wrist gradually D. The presentation is consistent with
worsening over a period of 4-6 hours. Blood polymyalgia rheurnatica (PMR)
tests reveal a neutrophilia (white cell count E. The presentation is typical of seronegative
12.5x109/L), a raised erythrocyte sedimentation rheumatoid arthritis
rate (ESR; 65 mm/hr) and a raised C-reactive
protein (CRP; 154 mg/L). 24.10. Which of the following is a common
What would be the most likely diagnosis? complication of seronegative (ACPA and
rheumatoid factor negative) rheumatoid
A. Calcium pyrophosphate deposition disease
arthritis?
B. Gout
C. Reactive arthritis A. Felty's syndrome
D. Septic arthritis B. Osteoporosis
E. Vasculitis C. Rheumatoid nodules
D. Uveitis
24.8. A 63 year old woman with a 10-year E. Vasculitis
history of rheumatoid arthritis presents with
gradually worsening pain and swelling of the left 24.11. A 36 year old woman presents with
knee joint over a period of 2-3 days. Her 3-month history of joint pain and swelling
arthritis has generally been under good control affecting the wrists, MCPJs and proximal
with methotrexate 20 mg weekly and the interphalangeal joints (PIPJs) of the hands, both
tumour necrosis factor alpha (TNF-a) inhibitor shoulders, both knees and the MCPJs of both
etanercept 50 mg weekly. On examination the feet. Laboratory investigations reveal an ACPA
knee is warm and swollen, with signs of an level of 145, an ESR of 68 rnm/hr and a CRP
effusion. What would be the most appropriate of 84 rng/L. On examination she has 22 tender
course of action? and 16 swollen joints and rates the activity of
A. Aspirate the knee and inject with 80 mg her arthritis as 65/100, giving a Disease Activity
methylprednisolone? Score 28 (DAS28) of 7.54. What would be the
B. Aspirate the knee and send the synovial fluid most appropriate initial treatment?
for culture and microscopy A. Adalimumab 40 rng every 2 weeks and
C. Commence treatment with a broad-spectrum prednisolone 5 mg daily
antibiotic B. Hydroxychloroquine 200 mg twice daily and
D. Commence treatment with diclofenac 75 mg ibuprofen 400 mg 3 times daily
twice daily C. Methotrexate 15 rng weekly, folic acid 5 rng
E. Increase the dose of methotrexate to 25 mg weekly and prednisolone 30 rng daily
weekly D. Prednisolone 30 rng daily, ibuprofen 400 mg
3 times daily and omepr~zole 30 mg daily
24.9. A 66 year old woman presents with pain E. Rituxirnab 1000 mg on two occasions a
and stiffness affecting the wrists, proximal fortnight apart combined with prednisolone
interphalangeal (PIP) and metacarpophalangeal 5 mg daily

downloaded from www.medicalbr.com


280 • RHEUMATOLOGY AND BONE DISEASE
r !

24.12. Which one of the following statements 24.14. A 60 year old woman suffers a low
is true with respect to post-menopausal trauma fracture of the right wrist after a fall.
osteoporosis? She is a non-smoker and drinks 8 units of
A. Bone pain is the most common presenting alcohol per week. Her menopause occurred at
feature aged 52. She is on no current medication and
B. Calcium and vitamin D supplements can has no significant medical history but reports
prevent its development that her mother, aged 79, has recently suffered
C. It is a rare complication of polymyalgia a hip fracture.
rheumatica What would be the most appropriate course
D. Obesity is an important risk factor of action?
E. Patients are usually asymptomatic until a A. Advise her to stop drinking alcohol
fracture occurs completely
B. Commence treatment with alendronic acid
24.13. A 73 year old woman presents to her C. Commence treatment with calcium and
family physician with sudden onset of pain in vitamin D supplements
the lower back region that developed after D. Request a dual X-ray absorptiometry (DXA)
removing weeds in her garden. She has a scan
history of breast cancer treated 10 years E. Request a spine radiograph
previously with surgery and radiotherapy
followed by tamoxifen for 5 years. She has a 24.15. Which one of the following is a common
history of hypertension controlled with adverse effect of oral bisphosphonate therapy
bendroflumethazide 2.5 mg daily. Her height is ,in patients with osteoporosis?
154 em, weight 53 kg and physical examination A. Atypical subtrochanteric fractures
is unremarkable. A spine radiograph is shown B. Iritis
below. C. Leucopenia
D. Osteonecrosis of the jaw
E. Upper gastrointestinal upset

24.16. A 48 year old woman with seropositive


rheumatoid arthritis (RA) has persistent pain
and swelling of the wrists, elbows, shoulders
and MCP joints of both hands despite 12
months' therapy with methotrexate 25 mg
weekly, folic acid 5 mg weekly, sulfasalazine
3 g daily and hydroxychloroquine 400 mg daily.
Examination reveals tenderness and swelling of
10 affected joints, and a raised ESR (45 mm/
hr). She rates the activity of her arthritis as
80/100 on a visual analogue scale yielding a
DAS28 of 6.45.
What would be the most appropriate next
course of action?
A. Commence treatment with aprernilast 30 mg
twice daily
B. Commence treatment with the interleukin
(IL)-17 blocker secukinumab 150 mg monthly
C. Commence treatment with the TNFa inhibitor
etanercept 50 mg weekly
What is the most likely cause of the back
D. Increase the frequency of methotrexate to 3
pain?
times weekly
A. Diffuse interstitial skeletal hyperostosis E. Stop sulfasalazine and substitute leflunomide
B. Metastatic bone disease 10 mg daily
C. Osteoarthritis of the spine
D. Osteoporotic vertebral fractures 24.17. A 56 year old man presents with acute
E. Spondylolisthesis pain, redness and swelling of the first

downloaded from www.medicalbr.com


RHEUMATOLOGY AND BONE DISEASE • 281

metatarsophalangeal (MTP) joint of the right (DIPJs) of both hands. Investigations show a
toot. Investigations show an elevated ESR haemoglobin of 118 g/L, white cell count
(35 mm/hr), CRP of 56 mg/L, a mild 6.3x 109/L, platelets 355 x 109/L and ESR
neutrophilia (12.1 x 109/L), serum creatinine 20 mm/hr. An X-ray of the hands and wrists is
75 J.Lmoi/L (0.85 mg/dL), estimated glomerular performed.
filtration rate (eGFR) >60 mUmin/1.73 m and
2 Which radiological features are typical of
a serum uric acid level of 450 J.Lmoi/L (7.6 mg/ osteoarthritis?
dL). Radiographs reveal evidence of erosions in A. Irregularity and fusion of the sacroiliac joints
the affected joint. B. Joint space narrowing and subchondral
What would be the most appropriate sclerosis of the PIPJ and DIPJ of the hands
treatment? G. Marginal erosions affecting the MCPJ of the
A. Allopurinol 100 mg daily initially gradually hands
increasing in dose until uric acid falls below D. Periarticular osteoporosis affecting the PIPJs
360 J.Lmoi/L (6.1 mg/dL) and DIPJs in the hands
B. Colchicine 500 mg 3 times daily until E. Punched-out erosions of the first MTP joint
symptoms settle followed by colchicine of the feet
500 mg daily on a long-term basis 24.20. A 65 year old man presents to his family
G. Colchicine 500 mg 3 times daily until physician complaining of pain in the left knee,
symptoms settle followed by long-term worse on ascending and descending stairs. He
diclofenac 75 mg twice daily smokes 15 cigarettes a day, and drinks about
D. Diclofenac 75 mg twice daily followed by 4 units of alcohol daily (28 units per week). He
long-term low-dose aspirin 75 mg/day is a former amateur soccer player who suffered
E. Etoricoxib 60 mg daily followed by allopurinol a cruciate ligament tear in his 30s. He has lived
starting at 100 mg daily, gradually increasing alone since his wife died 5 years previously. He
in dose until uric acid falls below 360 J.Lmoi/L has been avoiding dairy products since he
(6.1 mg/dL) thinks they cause gastrointestinal upset.
Which of the following risk factors predispose
24.18. A 35 year old woman with well-controlled
to the development of osteoarthritis?
RA states that she wishes to become pregnant.
Her medication consists of methotrexate 20 mg A. Alcohol intake >21 units per week
weekly, folic acid 5 mg weekly and ibuprofen B. Cigarette smoking
400 mg 3 times daily. What advice would you G. lmmobilisation
give with regard to her plans to conceive and D. Low dietary calcium intake
her medication? E. Previous anterior cruciate ligament tear
A. She can go ahead and try to conceive so 24.21. A 66 year old woman is referred to the
long as she reduces the dose of rheumatology clinic with pain in the right hip of
methotrexate to 10 mg weekly and ibuprofen gradual onset over the past 2 years, worse on
to 200 mg 3 times daily weight-bearing. Examination reveals limitation
B. She should stop the ibuprofen for at least 3 and pain on internal rotation of the right .hip.
months before trying to conceive but can Her height is 154 em and weight is 82 kg
continue the methotrexat~ (body mass index (BMI) 34.6 kg/m 2 ). A pelvic
G. She should stop the methotrexate and the X-ray shows joint space narrowing and
ibuprofen and then go ahead and try to osteophytes of the right hip joint, consistent
conceive with osteoarthritis.
D. She should stop the methotrexate for at Which one of the following statements is true
least 12 months before trying to conceive with regard to the treatment of osteoarthritis of
but can continue the ibuprofen the hip?
E. She should stop the methotrexate for at
A. A cyclo-oxygenase 2(COX-2) selective
least 3 months before trying to conceive but
NSAID is more likely to be effective than a
can continue the ibuprofen
non-selective NSAID in the treatment of pain
24.19. A 75 year old woman is referred to the B. Joint replacement surgery is indicated if the
rheumatology clinic complaining of pain and response to paracetamol is inadequate
swelling affecting the proximal interphalangeal G. Long-term prophylactic NSAID therapy has a
joints (PIPJs) and distal interphalangeal joints disease-modifying effect

downloaded from www.medicalbr.com


282 • RHEUMATOLOGY AND BONE DISEASE

D. Oral glucosamine is an effective treatment of gradually worsening pain, weakness and


E. Oral NSAIDs are more effective than stiffness affecting the muscles of the shoulder
paracetamol in controlling symptoms and pelvic girdle. She has a past medical
history of type 2 diabetes controlled with diet
24.22. A 67 year old woman attends her family and metformin 500 rng twice daily. She is also
physician complaining of generalised aches and taking atorvastatin 40 rng daily and has been
pains and feeling tired all the time. She has recently prescribed trimethoprim for a urinary
been under a lot of stress at work and has also tract infection.
been concerned that her son is unemployed Investigations show a normal full blood
and unable to get a job. Physical examination is count, ESR 13 mm/hr, normal urea and
unremarkable. electrolytes, serum creatine kinase 100 U/L,
Routine biochemistry and haematology tests serum adjusted calcium 2.25 mmoi/L (9.0 mg/
are completely normal with the exception of the dL), serum 25(0H)D <20 nrnoi!L (8 ng/mL),
serum 25-hydroxyvitamin D (25(0H)D) value, phosphate 0. 70 mmoi/L (2.17 mg/dL); bilirubin
which is 30 nmoi/L (12 ng/mL). This is reported 10 IJ.rnoi/L (0.58 mg/dL), aspartate
by the biochemistry laboratory as showing aminotransferase (AST) 20 U/L, alkaline
evidence of vitamin D insufficiency. phosphatase (ALP) 275 U/L, parathyroid
Which of the following statements is true? hormone (PTH) 35 prnoi/L (330 pg/mL).
A. Circulating serum 25(0H)D levels are highest What is the most likely diagnosis?
in the spring and lowest in the autumn A. Osteomalacia
B. Circulating serum 25(0H)D levels remain B. Osteoporosis
constant in most people throughout the year C. Polymyalgia rheumatica
C. Circulating vitamin D is mainly derived from D. Statin-induced myopathy
dietary intake of fish and meat E. Vitamin D insufficiency
D. Circulating vitamin D is mainly derived from
synthesis from 7 -dehydrocholesterol in the 24.25. A 56 year old woman presents with
skin by the action of ultraviolet (UV) light generalised musculoskeletal pain associated
E. It is likely that her symptoms will respond with multiple focal tender spots on local
well to supplementation with vitamin D, so pressure of the muscle on the trunk and upper
long as the circulating levels are restored to arms and is diagnosed as having probable
above 50 nmoi/L (20 ng/mL) fibromyalgia. Investigations reveal normal urea
and electrolytes, normal full blood count, ESR
24.23. Which of the following statements is 10 mm/hr, and normal bilirubin, AST and ALP.
correct with regard to metabolism of vitamin D? Serum 25(0H)D level is 25 nmoi/L (1 0 ng/mL),
A. The biologically active form 25(0H) vitamin D serum calcium is 2.41 mmoi/L (9.7 mg/dL),
is hydroxylated in the kidney by the enzyme phosphate 1.2 rnmoi/L (3. 72 mg/dL) and PTH
CYP27B1 to generate the inactive metabolite is 5.2 pmoi/L (49 pg/mL). Which of the
1,25(0HhD following statements is true?
B. The biologically active form 25(0H) vitamin D A. Fibromyalgia is associated with vitamin D
is inactivated by the enzyme CYP24A 1 in the deficiency
kidney B. High-dose vitamin D supplements (25 000
C. The biologically inactive form 25(0H) vitamin units per week) 'are likely to be an effective
D is hydroxylated in the kidney by the treatment for the musculoskeletal pain
enzyme CYP24A1 to give the active C. Low-dose vitamin D supplements (800 units
metabolite 1 ,25(0HhD daily) are likely to be an effective treatment
D. The biologically inactive form 25(0H) vitamin for the musculoskeletal pain
D is hydroxylated in the kidney by the D. The low vitamin D levels are likely to be
enzyme CYP27B1 to give the active secondary to the fibromyalgia
metabolite 1 ,25(0HhD E. The muscle pain and tenderness are likely to
E. The vitamin D metabolite 24,25(0HhD is be due to osteomalacia
biologically inactive and stimulates intestinal
calcium absorption and bone resorption 24.26. A 25 year old.man is referred to the clinic
because of focal bone pain affecting the
24.24. A 66 year old Muslim woman of Middle mid-shaft of the left femur. On examination he
Eastern origin presents with a 2-month history has short stature and, bilateral bowing

downloaded from www.medicalbr.com


RHEUMATOLOGY AND BONE DISEASE • 283

deformities of the lower limbs. He was What is the most likely cause of the pain?
diagnosed as having childhood rickets and A. Osteoarthritis
treated with vitamin D metabolites but stopped B. Osteomalacia
treatment aged 16 years and was lost to C. Osteoporosis
follow-up. There is a family history of rickets D. Paget's disease
affecting his mother and brother. E. Renal osteodystrophy
Investigations reveal a serum calcium of
2.25 mmoi/L (9.0 mg/dL), phosphate 24.29. Which of the following clinical or
0.60 mmoi/L (1.86 mg/dL), PTH 12.5 pmoi/L radiographic features is consistent with a
(118 pg/mL), ALP 160 U/L and serum 25(0H)D diagnosis of Scheuermann's disease?
54 nmoi/L (22 ng/mL).
A. At least two wedge deformities in the
What is the most likely diagnosis?
thoracic spine with aT-score of <-2.5 at
A. Tumour-induced osteomalacia either spine or hip on DXA examination
B. Vitamin D-deficient rickets B. Crush deformity affecting at least three
c. Vitamin D-resistant rickets type I vertebrae in the lumbar spine
D. Vitamin D-resistant rickets type II c. Disc space narrowing in the lumbar spine
E. X-linked hypophosphataemic rickets with marked osteophyte formation
D. Two vertebral crush deformities in the
24.27. Which of the following statements is true
thoracic spine and one in the lumbar spine
with regard to Paget's disease of bone?
with evidence of osteopenia on radiographs
A. Dietary calcium deficiency and smoking are E. Wedge deformity affecting several adjacent
recognised risk factors vertebrae in the thoracic spine with disc
B. It can be inherited in families in association space narrowing
with mutations in the SQSTM1 gene
c. It is a focal skeletal disorder characterised by 24.30. A 32 year old man is referred to the
inhibition of bone formation and an increased rheumatology clinic having sustained a fracture
risk of fracture of the right femur after falling when he tripped
D. It is a systemic skeletal disorder characterised over an uneven pavement. Examination is
by a generalised increase in bone turnover unremarkable apart from the fact that he has
E. There is a strong genetic component blue sclerae. He is known to have osteogenesis
mediated by variants at the human leucocyte imperfecta (01) and has a history of low trauma
antigen (HLA) locus on chromosome 6 fractures dating back to childhood.
Which of the following statements is true with
24.28. A 68 year old man presents with gradually
regard to this condition?
worsening pain in the right hip region, which is
present at rest and worsens slightly on A. Bisphosphonates are highly effective at
weight-bearing. lnvestigations,reveal a creatinine preventing fractures in adults with 01
of 140 J.lmol/L (1 .58 mg/dL) and an eGFR of B. Fractures of the vertebrae are an uncommon
35 mUmin/1.73 m2 but otherwise normal urea complication
and electrolytes, normal serum calcium and C. The diagnosis can be excluded if the sclerae
phosphate, but an ALP of 350 U/L. The full are of normal colour
blood count is normal. A pelvic radiograph is D. The incidence of fractures increases
performed and is shown below. progressively with age
E. The incidence of fractures is highest in childhood

24.31. A 23 year old woman presents with pain,


swelling and deformity affecting the distal tibia
of the right leg. On examination she has areas
of cafe-au-lait pigmentation on her right
shoulder and left forearm. The past medical
history is unremarkable and she is on no
medication apart from th~ oral contraceptive
pill. Radiographs reveal expansion of the left
tibia with alternating areas of osteolysis and
osteosclerosis.

downloaded from www.medicalbr.com


284 • RHEUMATOLOGY AND BONE DISEASE

What is the most likely diagnosis? of fine touch on examination of the affected
A. Camurati-Engelmann disease digits in the right hand. Investigations are as
B. Fibrous dysplasia follows: haemoglobin 120 g/L, white cell count
C. Osteomyelitis 6.5x 109/L, platelets 456x 109/L, ESR 20 mm/
D. Osteopetrosis hr, CRP 6 mg/L.
E. Paget's disease of the tibia What is the most likely cause of the
symptoms?

24.32. A 58 year old man with haemochromatosis A. Bone erosions secondary to the
is referred to the rheumatology clinic with a long-standing RA
3-year history of pain mainly affecting the small B. Median nerve compression
joints of the hands and wrists. Four weeks C. Mononeuritis associated with rheumatoid
previously he had developed acute pain, swelling vasculitis
and redness of the right wrist, which had D. Osteoarthritis of the first CMC joint
responded to treatment with naproxen 500 mg E. Ulnar nerve compression
3 times daily. He has a history of type 2 diabetes
treated with diet and metformin and has been 24.34. Which one of the following statements is
treated with regular venesection for the previous true with regard to joint hypermobility?
3 years. A. Affected patients may experience episodes
Clinical examination of his hands is of postural hypotension accompanied by
unremarkable with no evidence of synovitis. tachycardia
Laboratory Investigations are as follows: B. It is a rare complication of osteogenesis
haemoglobin 115 g/L, white cell count imperfecta
8.2x109/L, platelets 345x10 9/L, ESR 20 mm/ C. Mutations in the FBN1 gene are the most
hr, serum iron 100 [!moi/L (558 [!g/dL), AST common cause
35 U/L, bilirubin 15 [.Lmoi/L (0.88 mg/dL) and D. The diagnosis can be confirmed by a
ALP 150 U/L. Beighton score of more than 4 in patents
Radiographs of the hands and wrists show who have dislocated at least one joint
joint space narrowing and subchondral cysts E. Treatment with NSAIDs is highly effective in
affecting the MCP and the radiocarpal joints controlling ligament and joint pain in affected
with no osteophyte formation. patients
Which of the following statements is true?
I A. The arthritis is an incidental finding unrelated 24.35. A 67 year old woman with a 15-year
I to the diagnosis of haemochromatosis
B. The clinical picture is consistent with
history of type 2 diabetes treated with diet,
metformin and sitagliptin presents with gradual
I rheumatoid arthritis
C. The most likely cause for his joint symptoms
onset of pain and deformity affecting the right
ankle and foot.
is diabetic cheiroarthropathy On general examination, blood pressure is
D. The most likely explanation for the acute flare 145/85 mmHg, pulse 85 beats/min, height
in his joint symptoms is calcium 153 em and weight 89 kg. Neurological
pyrophosphate deposition disease examination reveals absent ankle jerks and
E. The risk of further flares in symptoms can be impairment of fine touch and proprioception in
reduced by continued venesection and both feet. Peripheral pulses are absent below
restoration of serum iron levels to normal the femoral arteries. Examination of the ankle
joint reveals swelling and deformity of the ankle
24.33. A 64 year old woman with a 10-year joint and a severe valgus deformity.
. history of rheumatoid arthritis affecting the Investigations show moderate renal
hands and wrists, which is controlled with dysfunction with a serum creatinine of
sulfasalazine 3 g daily and hydroxychloroquine 165 [.Lmoi/L (1 .87 'mg/dL) and eGFR of 25 mU
200 mg twice daily, presents with a disturbance min/1 .73 m2 . Serum AST is 20 U/L, ALT
of sensation and tingling affecting the thumb 85 U/L, bilirubin 12 [.Lmoi/L (0. 70 mg/dL) and
and anterior aspects of the index and second serum uric acid 400 [!moi/L (6.7 mg/dL). Full
fingers of the right hand. blood count shows mild anaemia with a
On examination there is no evidence of haemoglobin of 11 0 g/L and an ESR of
active synovitis but there is altered perception 25 mm/hr.
I
downloaded from www.medicalbr.com
j
RHEUMATOLOGY AND BONE DISEASE • 285

Radiographs show severe destruction of the neutrophils and there are negatively birefringent
ankle and the mid-foot joints with bony crystals in the fluid.' What is the correct
fragments within the joint. management?
What is the most likely cause of the ankle A. Antibiotics and steroids should be used
pain? together
A. Calcium pyrophosphate deposition disease B. Intra-articular steroids can be given straight
B. Charcot joint away because the diagnosis is reactive
C. Diabetic cheiroarthropathy arthritis
D. Gout C. The patient can be treated for gout because
E. Osteoarthritis infection has been excluded
D. The joint should be drained and analgesia
24.36. A 17 year old male presents with pain given but steroids should be withheld until
and swelling of the middle of the right tibia the culture result is available
that has been gradually increasing in severity E. The results favour a diagnosis of
over a period of 6-8 weeks. An X-ray shows pseudogout
expansion of the bone and a soft tissue mass
containing islands of calcification. 24.40. Psoriatic arthopathy contains a number
What is the most likely diagnosis? of the radiographic signs in the answers below.
A. Fibrous dysplasia Which one of the following radiographic signs is
B. Hypertrophic pulmonary osteoarthropathy NOT typically recognised in psoriatic arthritis
C. Metastatic bone disease (PsA)?
D. Osteosarcoma A. Bone sclerosis
E. Paget's disease B. Calcification of peri-odontoid ligaments
C. Juxta-articular new bone formation
24.37. Which physiological process is primarily D. Sacroiliac erosions
responsible for the development of E. Syndesmophytes
osteoporosis in patients on long-term
glucocorticoid therapy? 24.41. A young man has a history of chronic
A. Increased degradation of 25(0H)D low back pain and stiffness, which disturbs his
B. Increased osteoclastic bone resorption sleep and takes time to wear off in the morning
C. Inhibition of 25(0H)D production after waking. In making a diagnosis, which is
D. Inhibition of bone formation the most appropriate combination of tests to
E. Secondary hyperparathyroidism do after clinical assessment?
A. HLA-827 and ESR
24.38. Which of the following environmental B. MRI lumbar spine and sacroiliac joints and
exposures has been associated with bone scintigraphy
susceptibility to, and severity of, rheumatoid C. MRI SIJs and ESR
arthritis? D. Pelvis radiograph and HLA-827
A. Cigarette smoking E. Pelvis radiograph, whole-spine and SIJs MRI,
B. Excessive alcohol intake (>21 units per and HLA-827
week)
C. Human immunodeficiency virus (HIV) 24.42. Which one of the following is a
infection recognised use of ultrasound in rheumatology
D. Obesity (BMI >30) practice?
E. Vitamin D insufficiency A. Adding information to the diagnostic workup
of patients with polyrnyalgia rheurnatica
24.39. In investigating the cause of an acute B. Detection of the vascularity of synovitis in
monoarthritis in a 50 year old man in a MCP joints
non-tuberculosis (TB) endemic region, synovial C. Diagnosing sacroiliac inflammation
fluid from the swollen joint is sent for Gram D. Discrimination of hip adductor tendonitis
stain, culture and polarised microscopy. The from symphysitis
laboratory staff call with the results: they say E. Guiding needle placement to a lumbar spine
'Gram stain negative; culture results are not facet joint in treating facet joint arthritis with
available for another 48 hours. There are many injectable steroid

downloaded from www.medicalbr.com


286 • RHEUMATOLOGY AND BONE DISEASE

24.43. A woman, aged 51 years, has never had


a fracture but has been on intermittent steroids
mild weight loss and some altered cognitive
function.
l
(5 x 4-week courses) for her Crohn's disease Which one of the following diagnoses is least
over the last 2 years. Her DXA scan results are: likely?
lumbar spine bone mineral density (BMD) A. Autoimmune connective tissue disease
T-score -2.0; femoral neck BMD T-score -1.5; B. Fibromyalgia
and total hip BMD T-score -2.2. C. Inflammatory bowel disease
Which is the correct statement? D. Malignancy
A. Calcium and vitamin D should be the only E. Sarcoidosis
therapy considered
B. FRAX assessment will be useful in this case 24.47. A 65 year old man presents with low
C. Lateral spinal X-rays should NOT be thoracic back pain for the first time. It has been
obtained present for about 3 months, starting initially
D. Nothing should be done as T -scores are over the course of a week and now at a
>-2.5, except DXA to be arranged in 5 years constant level (no worsening, but no
time improvement). His sleep is disturbed. There are
E. Steroids should NOT be used to treat her no systemic symptoms and no leg pains.
Crohn's disease Of the following, which is the most likely
diagnosis?
24.44. Blood is taken from a pre-menopausal A. Axial spondyloarthritis
woman with joint pains, xerostomia and fatigue. B. Intervertebral disc prolapse
Antinuclear antibody (ANA) is positive; anti-DNA C. Osteoporotic fracture
antibody titre 1 U/L, anti-Ro(SSA) positive, D. Septic discitis
anti-La(SSB) positive, complement C3 and C4 E. Spondylolisthesis
normal, rheumatoid factor positive, ACPA
negative. Based on the following immunology 24.48. A 13 year old girl presents with limp and
results, which is the most likely diagnosis? examination evidence of left knee swelling.
A. Primary Sjogren's syndrome (PSS) There are no systemic symptoms, sore throat
B.RA or rash. Blood tests show normal full blood
C. RA and systemic lupus erythematosus (SLE) count, CRP of 10 mg/L, ESR of 10 mm/hr,
together negative rheumatoid factor, ACPA and ANA
D.SLE autoantibodies.
E. Systemic sclerosis (SScl) Which is most likely to be correct?
A. As there is no psoriasis, the condition is
24.45. In discriminating causes of inflammatory unlikely to be PsA
polyarthritis, which statement is most likely to B. Methotrexate should be started immediately
be true? C. Screening for uveitis is not necessary
A. An absence of joint synovitis on examination D. The condition is best classified as
rules out RA oligoarticular juvenile idiopathic arthritis (JIA)
B. Enthesitis occurring in PsA always causes E. The girl has juvenile RA
pain or local tenderness
C. PolyartiCular joint involvement may be a 24.49. Which of the following statements about
typical presentation of gout in an older JIA is most likely to be correct?
woman A. Oligoarthritis in the presence of high acute
D. Pseudogout/calciurn pyrophosphate phase response measures is not a
deposition disease affects only peripheral presenting feature of leukaemia or
joints inflammatory bowel disease (lBO)
E. Synovitis in PIP and some MCP joints rules B. Only ANA-poqitive JIA patients need an
out OA ophthalmological examination
C. Systemic JIA is associated with
24.46. A woman aged 34 years presents with haernophagOGytic syndrome
a 24-month fluctuating history of fatigue, D. Systemic JIA is us8ally associated with a
widespread pains, poor appetite, non-specific positive ANA
bowel symptoms, non-specific skin rashes, E. The prevalence of JIA is about 1 in 10 000

downloaded from www.medicalbr.com


RHEUMATOLOGY AND BONE DISEASE • 287

24.50. Which of the following treatments has G. Enthesitis does not improve with
NOT shown efficacy in either ankylosing secukinumab (anti-IL-17 DMARD)
spondylitis or psoriatic arthritis? D. Enthesitis only occurs in SpA patients who
A. Anti-IL-l? A monoclonal (secukinumab) are HLA-827
B. Anti-IL-23/12 monoclonal (ustekinumab) E. Enthesitis only occurs in lower limbs
G. Anti-TNF-a
D. Apremilast (phosphodiesterase-4 inhibitor) 24.55. Apremilast - a treatment developed for
E. Rituximab (anti-CD20/anti-B-cell therapy) psoriatic arthritis - is a small molecule that ·
directly inhibits which of the following?

24.51. A 35 year old man develops low back, A. Mitogen-activated protein (MAP) kinases
posterior heel pain and a swollen knee and has B. Phosphodiesterase 4 (PDE4)
a pustular skin rash on the soles of his feet. G. RANK ligand
There are no preceding illnesses, no previous D. Signal transducer and activator of
psoriasis or family history of it. What is the transcription 3 (STAT3)
most likely diagnosis? E. T-cell CD80/86 binding

A. Ankylosing spondylitis
24.56. Which of the following interventions lack
B. Gout
evidence of efficacy in the treatment of any
G. Post -streptococcal arthritis
components of fibromyalgia (pain, fatigue,
D. Psoriatic arthritis
physical functioning)?
E. Sexually acquired reactive arthritis
A. Cannabis
24.52. Most of the genes below are implicated B. Cognitive behavioural therapy (CBT)
in influencing either susceptibility for, or severity
G. Gabapentin
of, ankylosing spondylitis. However, which of
D. Supervised aerobic exercise training
these genes below has NOT been implicated?
E. The serotonin and noradrenaline reuptake
inhibitor (SNRI) duloxetine
A. ERAP-1
B. HLA-827 24.57. Which of the following is thought to
G. ANK-H be a consequence of, or associated with,
D. /L -23 receptor constitutive substantial connective tissue laxity
E. STAT-3 (hypermobility syndrome (HMS)/
hypermobile-type Ehlers-Danlos)?
24.53. Which combination of features below
A. Enthesitis
is most likely to be relevant to a diagnosis
B. Fibromyalgia
of axSpA?
G. Hypertension
A. Achilles tendon enthesitis, anterior uveitis D. Plantar fasciitis
and pubic symphysitis E. Uterine fibroids
B. An aunt who has psoriasis, rheumatoid
factor and fatigue 24.58. The autoantibody profile: ANA positive,
G. Back pain, joint swelling and stiffness, DNA!Sm/Ro(SSA)/La(SSB) negative,
scleritis, fatigue and positive ACPA ribonucleoprotein (RNP) positive, is most likely
D. High ESR, anterior uveitis, ankle swelling, to be associated with which autoimmune
raised serum angiotensin-converting enzyme connective tissue disease?
(ACE)
A. Mixed connective tissue disease (MCTD)
E. Low back pain, rosacea, prostatism and
B. Polymyositis
diarrhoea
G. Primary Sjogren's syndrome (PSS)
D. Systemic lupus erythematosus (SLE)
24.54. Enthesitis is the hallmark musculoskeletal E. Systemic sclerosis (SScl)
lesion of all spondyloarthritides (SpAs). Which
one of the following is characteristic of 24.59. A 34 year old woman (currently
enthesitis in the context of an SpA condition? mid-menstrual cycle) pres.ents with small joint
A. Enthesitis can occur in PsA without causing pain and stiffness, a UV-sensitive erythematous
any symptoms skin rash on exposed skin surfaces, fatigue,
B. Enthesitis cannot be detected by US mouth ulcers, some ankle swelling and a

downloaded from www.medicalbr.com


----------------------------------------------------------------------------------1
288 • RHEUMATOLOGY AND BONE DISEASE
I
normal examination otherwise, apart from 3+ D. Transthoracic echocardiogram and RFTs in
protein on urine dipstick and blood pressure of SScl I
145/90 mmHg. E. Yearly whole-body CT in polymyositis I

Which is the most appropriate immediate


course of action? 24.62. Which of the following is most likely to be I

true in relation to primary Sjogren's syndrome?


A. Check routine lab tests including
autoimmune serology and complement A. Overall anti-TNF-a therapy has been shown
and arrange routine review in a few to be useful
weeks B. PSS is not associated with malignancy
B. Check routine lab tests including C. Severe oral dryness results from
autoimmune serology and complement and immunological destruction of the salivary
start steroids immediately glands
C. Check routine lab tests including D. The environment does NOT play a role in
autoimmune serology and complement, influencing a patient's symptoms
start hydroxychloroquine and review E. Topical steroids should never be used for
progress in 3 months xerophthalmia
D. Check routine lab tests including
autoimmune serology and complement, 24.63. A 52 year old non-smoking man presents
arrange Gram stain and culture of urine, with a 6-week history of arthralgia and a
quantify urinary protein and arrange renal palpable purpuric rash appearing on his limbs.
ultrasound He has a year -long history of recurrent fevers,
E. Discharge her from your care with sinusitis, cough and sore throats treated with
reassurance and no follow-up repeated courses of antibiotics and steroids.
What is the most likely diagnosis?
24.60. In addition to obtaining routine A. Adult-onset Still's disease
haematology, renal and liver laboratory tests B. Malignancy
with autoimmune serology (including C. Post -streptococcal reactive arthritis
myositis-specific antibodies) and muscle D. Microscopic polyangiitis (MPA)
creatine kinase, which is the most appropriate E. Rheumatic fever
combination of tests when investigating for
polymyositis? 24.64. Which one of the following diseases is
A. Muscle and cardiac MRI, open muscle NOT typically associated with autoantibody

I biopsy, barium swallow


B. Muscle electromyogram (EMG), needle
production?
A. Antiphospholipid syndrome
I muscle biopsy, whole-body CT
C. Muscle MRI and EMG, chest X-ray, open
B. Giant cell arteritis (GCA)
C. Granulomatosis with polyangiitis (GPA;
~ muscle biopsy formerly known as Wegener's

l D. Muscle MRI and EMG, needle muscle


biopsy, whole-body CT scan, cardiac
granulomatosis)
D. Microscopic polyangiitis (MPA)
transthoracic echocardiogram, barium E. Polymyositis
swallow
E. Muscle MRI and EMG, open muscle biopsy, 24.65. A 40 year old Armenian woman presents
whole-body CT scan, cardiac transthoracic with recurrent bouts of fatigue, arthralgia
echocardiogram, barium swallow (oligoarticular: elbow, wrists and a knee),
headaches and crops of painful mouth ulcers,
24.61. Of the following, what proactive occasional bluish-red raised tender blotchy skin
investigational monitoring of internal organ lesions and a single sore erythematous eye.
function, despite the absence of symptoms, is She does not hai(Ei diarrhoea.
essential in managing autoimmune connective ESR is 25 mm/hr but CRP is normal; full
tissue disease? blood count shows slight lymphopenia,
A. Respiratory function tests (RFTs) and chest normal neutrophil-and platelet counts. Serum
X-ray in dermatomyositis adjusted calcium and PTH are in the reference
B. RFTs in SLE range.
C. Slit lamp ocular examination in SLE Which is the most likely diagnosis?

downloaded from www.medicalbr.com


,.-----_
r RHEUMATOLOGY AND BONE DISEASE • 289

A. Beh<;:et's disease A. Calcium pyrophosphate disorder disease/


B. Grahn's disease arthritis
c. Sarcoidosis B. Gout
D. Tuberculosis C. Osteoarthritis
E. Viral infection D. Psoriatic arthritis
E. Rheumatoid arthritis
24.66. Which of the following has NOT been
implicated in contributing to the pathogenesis 24.70. A 75 year old woman with established
of vasculitis? osteoporosis, with previous forearm fracture,
A. Cannabis use known stable chronic kidney disease (CKD)
B. Hepatitis C stage 3 (eGFR approx. 48 mUmin/1. 73 m 2)
C. HIV infection and no history of TB (she was immunised
D. HLA-851 against TB as a child and has had no recent
E. lgA TB contacts), presents with an acutely
swollen very painful knee. She feels generally
24.67. Which association between a unwell, had dysuria a month previously treated
microorganism and an autoinflammatory or with antibiotics but has had no rash or
autoimmune condition is speculative and diarrhoea.
LEAST well proven? Investigations show haemoglobin 122 g/L,
white cell count/neutrophils slightly raised,
A. Chlamydia trachomatis and sexually acquired platelet count 450x109/L and normal
reactive arthritis
lymphocytes with ESR 45 mm/hr. Liver and
B. Group A streptococci and rheumatic fever
thyroid function tests are normal.
C. Hepatitis C and cryoglobulinaemic vasculitis
Which of the following statements is
D. Propionibacterium acnes and SAPHO correct?
(synovitis-acne-pustulosis-hyperostosis-
osteitis) syndrome A. Background renal impairment has no
E. Staphylococcus and septic arthritis relevance to the current presentation
B. Blood cultures are likely to give the
diagnosis
24.68. Which of the following conditions is most
C. Septic arthritis is unlikely
likely to lead to development of an associated
D. The dysuria has no relevance to the current
malignancy?
presentation
A. Diffuse idiopathic skeletal hyperostosis E. Uric acid should be tested
{DISH)
B. Giant cell arteritis
C. Primary Sjogren's syndrome 24.71. Conditions and their therapies have been
matched, below; all are correct except one.
D. Relapsing polychondritis
Which therapy is NOT appropriate for its listed
E. Rheumatoid arthritis
condition?

24.69. The abnormality on the wrist radiograph A. Barbotage for cutaneous calcinosis in
below (arrowed) suggests which condition? dermatomyositis
B. Local glucocorticoid injection for chronic
plantar fasciitis
C. Oral pilocarpine for primary Sjogren's
syndrome
D. Rituximab (anti-CD20 monoclonal) for
ANCA-associated vasculitis
E. Thalidomide for Beh<;:et's disease

24.72. A 22 year old man presents with a


6-week history of persistent back pain and
stiffness occurring with and ~fter immobility,
and heel pains. In interpreting investigations,
which of the following is most likely to
be true?

downloaded from www.medicalbr.com


290 • RHEUMATOLOGY AND BONE DISEASE

A. A negative HLA-B27 rules out axSpA D. He may have sexually acquired reactive
B. A normal CRP rules out ankylosing arthritis
spondylitis E. SIJ radiographs will reveal the diagnosis
C. A previous diagnosis of sterile urethritis is
irrelevant information to making a diagnosis

Answers
24.1. Answer: E. single joint and risk factors of obesity,
Osteoclasts are responsible for resorbing bone thiazide therapy and excessive alcohol intake.
and osteoblasts for bone formation but The pattern of involvement and acute onset is
osteocytes are responsible for coordinating not consistent with rheumatoid arthritis,
osteoblast and osteoclast activity. psoriatic arthritis or OA. Septic arthritis is
possible but unlikely in·the absence of a
24.2. Answer: D. previous history of joint disease or site of
RANK and LRP5 are key receptors involved infection.
in the activation of osteoclastic bone
resorption and bone formation, respectively. 24.7. Answer: A.
Osteoprotegerin inhibits bone resorption. Wnt Calcium pyrophosphate deposition disease is
stimulates bone formation but is a ligand, not a strongly associated with OA and typically
receptor. RANKL is a ligand that stimulates affects those aged >65 years, affecting women
bone resorption. more commonly than men. The onset is
sudden with join pain and swelling developing
24.3. Answer: D. over a period of 4-6 hours. Dehydration is a
Paracetamol is the first-line systemic analgesic common precipitating factor, and investigations
for mild to moderate pain. Non-steroidal typically reveal a neutrophilia and a raised ESR
anti-inflammatory drugs (NSAIDs) such as and CRP. Gout and reactive arthritis can
diclofenac should be used with great caution in present in a similar manner but would be less
the elderly and there would be a higher risk of likely in a woman of this age. Septic arthritis is
adverse effects with co-codamol, gabapentin possible but usually has a subacute onset and
and tramadol. tends to develop more slowly, over a period of
24-48 hours.
24.4. Answer: E.
Obesity aggravates joint pain in osteoarthritis 24.8. Answer: B.
and weight loss is one of the most effective The history is suggesting of septic arthritis
therapies for osteoarthritis (OA) of the lower given the history of rheumatoid arthritis (RA)
limbs. Weight-bearing exercise is unlikely to and immunosuppressive therapy with
help and may worsen symptoms. Surgical methotrexate and a TN F-a inhibitor. Options A,
synovectomy is not indicated in OA and D and E would not be appropriate until
arthroplasty would only be indicated for infection had been excluded, nor would
advanced OA resistant to medical therapy. option C.
Cognitive behavioural therapy would be unlikely
to help. 24.9. Answer: E.
The negative ACPA and normal radiograph
24.5. Answer. B. does not exclude RA since ACPA and
Blood monitoring is not required for rheumatoid factor are negative in about
hydroxychloroquine but is required for all the one-third of patient~ and radiographs are
other drugs to screen for blood dyscrasias and normal in early RA. The presentation would not
abnormal liver function tests. be consistent with PMR and the distribution of
involvement (wrists,.metacarpophalangeal joint;
24.6. Answer: A. MCPJ) excludes OA. MRI.would not be
The clinical presentation is typical of gout, necessary since the patient has typical signs of
given the acute onset, involvement of a synovitis.
I

downloaded from www.medicalbr.com


RHEUMATOLOGY AND BONE DISEASE • 291

24.10. Answer: B. 24.16. Answer: C.


Osteoporosis is a common complication The patient has failed to respond adequately to
of RA whether seropositive or seronegative. triple therapy and progression to biologic
Felty's syndrome, vasculitis and nodules can treatment would be indicated. Neither
occur in RA but they are rare complications secukinumab nor apremilast is effective in RA
and occur in seropositive RA. Uveitis is a and would not be appropriate choices.
feature of axial spondyloarthritis (axSpA), Substitution of sulfasalazine with leflunomide is
not RA. unlikely to be effective. Increasing the frequeflcy
of methotrexate would not be indicated since
24.11. Answer: C. the patient is already on the maximum
Methotrexate is the core disease-modifying recommended dose.
antirheumatic drug (DMARD) in the
management of RA and it is often combined 24.17. Answer: E.
with prednisolone therapy at first presentation Acute gout can be managed with either NSAID
to gain disease control. Option B would be therapy or colchicine, but urate-lowering
inappropriate in view of the very active disease therapy with allopurinol is indicated to control
given that hydroxychloroquine has relatively hyperuricaemia in the long term, to reduce the
weak immunosuppressive effects. Options A risk of recurrence and prevent long-term joint
and E would not be indicated as first-line damage. Allopurinol alone would not be
treatments. Option D would not give adequate appropriate since it may cause a further flare in
disease control. acute gout due to the change in uric acid
levels.
24. 12. Answer: E.
Obesity is protective against osteoporosis 24.18. Answer: E.
and osteoporosis does not cause bone pain Methotrexate is teratogenic and must be
unless a fracture has occurred. Calcium and stopped completely at least 3 months before
vitamin D supplements are used in the attempting to become pregnant. It is not
treatment of osteoporosis, mainly as an necessary to stop ibuprofen before becoming
adjunct to other treatments, but alone pregnant but NSAIDs are contraindicated after
they are ineffective in the prevention of 20 weeks.
osteoporosis. Osteoporosis is a common
complication of PMR, not a rare 24.19. Answer: B.
complication. Osteoarthritis is characterised by joint space
narrowing due to cartilage erosion and
24.13. Answer: D. subchondral sclerosis. Erosions at all MCPJs
The radiograph shows typical features of and periarticular osteoporosis are more typical
osteoporotic vertebral fractures with biconcave of RA (though inflammatory OA typically causes
deformities of L4, L3 and L2, and a wedge inflammation at index and middle finger MCPJs)
deformity of L 1 . whereas punched-out erosions of the first MTP
joint of the feet suggest gout. Irregularity and
24.14. Answer: D. fusion of the sacroiliac joint (SIJ) suggests axial
The patient is at increased risk of osteoporosis spondyloarthritis (axSpA) or psoriatic arthritis
because of the low trauma fracture and family (PsA).
history of hip fracture. If DXA shows
osteoporosis, treatment would be indicated. 24.20. Answer: E.
There is no indication at present to start Previous joint injury is a strong risk factor for
treatment with either alendronate or calcium osteoarthritis due to destabilisation of the joint.
and vitamin D supplements, r:1or is there a None of the other factors significantly influences
reason to advise her to stop alcohol since she the development of osteoarthritis.
has a moderate intake.
24.21. Answer: E.
24.15. Answer: E. Systemic NSAIDs are more effective that
Options A, B and D are rare adverse effects. paracetamol in controlling pain in OA. There
Leucopenia is not a recognised adverse effect is no evidence that COX-2 selective and
of oral bisphosphonates. non-selective NSAIDs differ in efficacy. There is

downloaded from www.medicalbr.com


292 • RHEUMATOLOGY AND BONE DISEASE

some evidence that glucosamine has a weak


beneficial effect in knee OA but it has not been
recessive disorders, which would be
inconsistent with the family history of an
1
I

studied in hip OA. Joint replacement surgery is affected mother and brother.
a recognised treatment for OA, but would only
be indicated when optimal medical therapy was 24.27. Answer: B.
ineffective. In about I 0-15% of cases the disease is
inherited in families due to mutations in the
24.22. Answer: D. SQSTM1 gene. There is no proven association
Although diet accounts for a proportion of with the HLA locus. Paget's is a focal skeletal
circulating vitamin D, sunlight exposure is the disorder characterised by increased bone
most important source in most people. resorption and formation (not reduced bone
Synthesis of 25(0H)D in the skin under the formation). Although environmental factors play
influence of UV light accounts for the fact that a role in Paget's disease of bone, the triggers
circulating levels are highest in the summer and are unclear and there is no evidence that
lowest in the winter. It is very unlikely that this alcohol intake or smoking predispose to the
patient's symptoms are related to the level of disease.
vitamin D.
24.28. Answer: D.
24.23. Answer: D. The radiograph shows changes typical of
The inactive metabolite 25(0H) vitamin D Paget's disease with alternating areas of
(25(0H)D) is hydroxylated in the kidney at the osteosclerosis and osteolysis and expansion of
I a. position by the enzyme CYP27BI to give the femur. There is also a pseudofracture on
the active metabolite I ,25(0HhD. Although the lateral femoral cortex. The site of Paget's
25(0H)D is hydroxylated at the I and 24 corresponds with the location of the pain, and
positions, the 24,25(0HhD metabolite is not the elevated ALP level indicates increased
biologically active. metabolic activity, suggesting that the pain may
be caused by Paget's disease of bone. There
24.24. Answer: A. is no evidence of OA, which makes this unlikely
Osteomalacia is suggested by the symptoms, as the cause of the pain. The biochemistry
the patient's ethnic background, the low does not support a diagnosis of osteomalacia
25(0H)D, high PTH, low phosphate and high and renal osteodystrophy would not be
ALP. Vitamin D insufficiency is a biochemical expected in a patient with mild renal
diagnosis in patients with serum 25(0H)D levels impairment.
of 25-50 nmoi/L (I 0-20 ng/mL). Polymyalgia is
unlikely in view of the normal ESR, and 24.29. Answer: E.
statin-induced myopathy is unlikely in view of Scheuermann's is characteristically
the normal creatine kinase. The symptoms and accompanied by wedge deformities of several
biochemical abnormalities are not consistent adjacent thoracic vertebrae with disc space
with osteoporosis. narrowing. It typically results in contiguous
vertebral paramarginal syndesmophytes.
24.25. Answer: D. Options B, D and E would be consistent with
It is likely that the low 25(0H)D levels are osteoporosis. Option C would be consistent
secondary to fibromyalgia since vitamin D with osteoarthritis.
deficiency is common as a secondary feature
of many diseases due to lack of sunlight 24.30. Answer: E.
exposure and a poor diet. There is no evidence Fractures occur most commonly in childhood,
that vitamin D supplements help in fibromyalgia decrease during adolescence and adulthood
or that fibromyalgia is a complication of vitamin but increase again with ageing. Option A
D deficiency. is incorrect: it is un~nown whether
bisphosphonates reduce fracture risk in adults
24.26. Answer: E. with 01. Option B is imcorrect: vertebral
Option A is unlikely in view of the positive family fractures occur collJmonly. Option C is
history. Vitamin D-deficiency rickets is unlikely incorrect: blue sclerae are typical of type I 01
in view of the positive family history and the but normal sclerae do not exclude other
normal 25(0H)D level. Options C and D are subtypes.

downloaded from www.medicalbr.com


RHEUMATOLOGY AND BONE DISEASE • 293

24.31. Answer: B. Ehlers-Danlos syndrome type Ill, which is a


The presentation is typical of fibrous dysplasia, polygenic disorder. Option B is incorrect since
which is caused by a somatic mutation in hypermobility is common in osteogenesis
GNAS1, which activates signalling through the imperfecta. Option D is incorrect. Hypermobility
PTH receptor causing a focal increase in bone is diagnosed with a Beighton score of 4 or
turnover and osteolytic lesions. It is also above in the presence of arthralgia. Joint
associated with cafe-au-lait pigmentation due dislocations are not a prerequisite to make the
to activation of signalling through the diagnosis.
melanocyte-stimulating hormone receptor and
endocrine abnormalities. Paget's disease can 24.35. Answer: B.
also present with focal bone deformity but The presentation is typical of a Charcot
would be extremely unusual in a patient of this joint secondary to peripheral neuropathy
age. Osteomyelitis is unlikely in the absence of associated with the diabetes. Although the
a previous history of infection. Camurati- patient has hyperuricaemia, the history does
Engelmann disease can present similarly but is not suggest gout. The history is also
bilateral rather than unilateral. The diagnosis is inconsistent with calcium pyrophosphate
not consistent with osteopetrosis, which is a deposition disease. Diabetic cheiroarthropathy
systemic rather than local disorder affects the hands, not the ankle. The clinical
characterised by high bone mass. picture ·is not consistent with OA, which in
the lower limbs typically affects the hips
24.32. Answer: D. and knees.
Calcium pyrophosphate deposition disease
(CPPD) is a common feature of the arthropathy 24.36. Answer: D.
associated with haemochromatosis and the Osteosarcoma is a rare tumour that
presentation with acute pain and swelling that predominantly affects people under 30 years of
settles with NSAID treatment is consistent with age, presenting with pain and a soft tissue
CPPD-associated arthropathy (pseudogout). mass, often with islands of calcification within
Diabetes can cause an arthropathy but has the lesion. Fibrous dysplasia can cause an
different features and RA is unlikely as expansile bone lesion but is not associated with
symptoms have settled and there is no a soft tissue mass, The patient is too young for
synovitis. Although venesection should Paget's disease. Metastatic bone disease
be continued as treatment for the presents differently. Although hypertrophic
haemochromatosis, there is no evidence that pulmonary osteoarthropathy can present with
this will improve the arthritis. Option A is pain and a periosteal reaction in the limbs, it
incorrect since an arthropathy is associated does not cause a soft tissue mass.
with haemochromatosis in up to 50% of cases.
24.37. Answer: D.
24.33. Answer: B. The main mechanism is inhibition of bone
The symptoms are typical of median nerve formation due to osteoblast and osteocyte
compression syndrome, which is a recognised apoptosis. Osteoclastic bone resorption
complication of RA. Mononeuritis can occur can be increased due to secondary
secondary to vasculitis in RA but this is unlikely hyperparathyroidism but this is not the main
since the disease is under good control. mechanism of bone loss. Options A, C and
Osteoarthritis of the CMC joint presents with E are all incorrect; glucocorticoids do not
local pain rather than neurological symptoms. affect vitamin D metabolism or cause
Bone erosions are a cause of pain in RA but hypoparathyroidism.
not neurological symptoms.
24.38. Answer: A.
24.34. Answer: A. Smoking has been associated with severity and
Postural orthostatic hypotension syndrome susceptibility to RA as well as response to
(POTS) is a recognised complication of treatment. There is no evidence that alcohol,
hypermobility. Option C is incorrect. Although body weight or HIV infection predispose to RA.
FBN1 mutations cause hypermobility as part of Although vitamin D insufficiency is common in
Marfan's syndrome, this is a rare cause of RA, there is no evidence that it plays a causal
hypermobility - the most common cause is role or influences disease activity.

downloaded from www.medicalbr.com


294 • RHEUMATOLOGY AND BONE DISEASE "'*'
'

24.39. Answer: D. spinal fractures can be relatively clinically silent


The available results are consistent with acute - causing few symptoms - so often need
gout (urate crystals are negatively birefringent) pro-action 'ruling out' with imaging. If present,
but do not exclude the possibility that infection a fragility spine fracture will elevate her further
may also be present. In sepsis, often bacterial fracture risk considerably.
identification is not possible until culture is FRAX (www.shef.ac.uk!FRAX) allows an
available. A negative Gram stain suggests there overall quantification of fracture risk using the
might not be infection but does not rule it out. main BMD-independent risk factors for· fracture.
The cause of pseudogout is calcium-containing It is well established that the level of BMD at
crystals (usually pyrophosphate) and these which fragility fractures are likely is higher than
crystals are positively birefringent. Reactive would otherwise be expected if steroids were
arthritis is certainly possible but intervention not being taken.
should be delayed until it is clear it is not Patients with Crohn's disease, and patients
septic arthritis. on steroids, can be calcium and/or vitamin D
deficient and supplements should be
24.40. Answer: B. considered but antiresorptive therapies should
Calcification of peri-odontoid ligaments is a also be considered if overall risk warrants.
feature of crowned dens syndrome, which is a Whilst steroids should be minimised, there may
lesion seen in CPPD. Syndesmophytes are the be no other option for treatment of her acute
hallmark radiographic sign in advanced Crohn's flare-ups.
ankylosing spondylitis. Bone sclerosis is a
recognised feature of PsA; juxta-articular new 24.44. Answer: A.
bone formation is very common in PsA and Autoimmune serology results interpreted alone
is included in the diagnostic (CASPAR) out of clinical context are rarely, if ever,
classification. Sacroiliac disease is a common diagnostic. The serology is typical of PSS.
feature in all spondyloarthritides (SpA). Positive anti-DNA antibodies are typically
associated with SLE. Anti-Ro can be present in
24.41. Answer: E. both SLE and PSS but if both Ro and La are
The history suggests inflammatory back pain positive, PSS is [Tlore likely. Although patients
and hence either axSpA or ankylosing with SScl can be ANA positive, antibodies more
spondylitis (AS) is possible. AS and axSpA can specific for SScl are anti-centromere or
be associated with normal ESR. Diagnosis of anti-topoisomerase (Scl-70). Rheumatoid factor
AS requires an abnormality of SIJs on x-ray to is not specific for RA and in the context of
be present but a diagnosis of axSpA can be these other serology results and negative
made before SIJ X-ray changes are present ACPA, the rheumatoid factor is far more likely
using axial skeletal MRI. Although HLA-827 to represent PSS.
does not diagnose either disease, its presence
increases the likelihood of either axSpA or AS 24.45. Answer: C.
in the appropriate clinical context. The presentation of gout in men and women
is typically different; particularly in post-
24.42. Answer: B. menopausal women, the first presentation may
Ultrasound has poor ability to detect soft tissue be polyarticular. Synovitis in RA can be subtle
abnormalities if there is extensive bone (which and is not always clinically obvious - MRI and
appears as an interface linear high signal with a ultrasound (US) are useful in confirming early
reflectance void beyond it, i.e. black!). There synovial disease. Axial skeletal forms of CPPD
are no characteristic features of PMR-related include crowned dens syndrome, intervertebral
lesions on ultrasound. With Doppler, ultrasound disc inflammation and ligament flavum
is useful for gauging the vascularity of joint and inflammation/thickening. Studies using US
tendon synovitis. have shown subclinical inflammation at
entheses in PsA.
24.43. Answer: B. Generalised OA can present as an
Her osteoporosis risk may be quite high given inflammatory 'storm' of symptoms in small
her age (likely peri-menopausal), steroids and joints with synovial inflammation. Involvement of
systemic inflammatory disease, and despite just PIPJs and DIPJs is more common than
an osteopenia-level BMD. Mild but definite MCPJs, but often the index and sometimes

downloaded from www.medicalbr.com


--
RHEUMATOLOGY AND BONE DISEASE • 295

third finger MCPJs are involved (RA often picks 24.49. Answer: C.
out the fifth MCPJ early on in the course of the All JIA patients should be referred for
disease). ophthalmological examination to rule out uveitis.
Both lBO and leukaemia can present with
24.46. Answer: D. oligoarthritis. Systemic JIA is regarded as an
The features are not unusual for a antibody-negative condition and is analogous to
rheumatology referral! The differential adult-onset Still's disease. The prevalence of
diagnosis can be wide and includes JIA (1 : 1000) is similar to the prevalence of
inflammatory and autoimmune disease. Also, diabetes in children and adolescents (1 : 700).
significant somatic and functional effects from
psychosocial triggers in a vulnerable person 24.50. Answer: E.
can provide such a symptom complex. Rituximab is 'B-cell depletion' therapy. B-cell
Rheumatology assessment requires a broad proliferation and B-cell antigen presentation are
approach and judicious use of investigations not a major part of the pathophysiology of
based on a stratified differential diagnosis either AS or PsA. Other therapies mentioned
based on clinical assessment. A 2-year history have alternatively been shown to have some
of an illness due to malignancy would be clinical effectiveness in one or both conditions.
expected to cause progressive clinical
deterioration. 24.51. Answer: E.
A pustular plantar foot rash occurring
24.47. Answer: C. simultaneously with inflammatory back pain,
Men do get osteoporosis and the commonest enthesitis and synovitis does suggest reactive
vertebral fracture sites are low thoracic spine or arthritis. A sexual history may not be
L1 or L2. Fracture pain can be mild to severe volunteered but should be sought - with direct
but often starts acutely or subacutely, and questions about new recent sexual encounters,
persists. It would be rare for axSpA to present penile discharge, dysuria and other genital
at this age for the first time. Disc prolapse symptoms, if necessary.
lesions occur mainly in younger people and are
rare in the thoracic spine - the commonest 24.52. Answer: C.
levels being L5/S1, L4/L5, L3/L4. The absence Ankylosing spondylitis (AS) and all
of systemic features is chiefly against this being spondyloarthropathies are generally
sepsis - patients with this diagnosis are often autoinflammatory conditions characterised by
generally quite unwell. Like prolapsed discs, the abnormalities in antigen processing (HLA-B27,
main sites of spondylolistheses are lumbar ERAP-1 ), antigen presentation (ERAP-1,
spine and sometimes in the neck, but very HLA-B27) and the stimulation and activity of
rarely in the thoracic spine. Malignancy is not type 17 T cells (IL-23r, STAT-3). ANK-H is
on the list but should be considered in anyone associated with calcium pyrophosphate
this age presenting with non-trivial/self-limiting deposition disease (CPPD). ANK-H codes
back pain for the first time. for a transmembrane protein important in
transporting inorganic pyrophosphate.
24.48. Answer: D.
Oligoarthritis is the most common form of JIA, 24.53. Answer: A.
accounting for 60% of cases. Monoarthritis is Scleritis and ACPAs are features of RA.
an unusual presentation of RA, especially if Uveitis, ankle swelling and raised ACE are
both rheumatoid factor and ACPA antibodies typical of sarcoid. Fatigue is a feature of all
are negative. Ophthalmological screening is autoinflammatory and autoimmune conditions.
recommended in all cases of JIA, regardless of A small minority of PsA patients may have a
whether ANA is positive or negative. As in positive rheumatoid factor. Achilles insertional
adults, PsA may be the cause of monoarthritis/ tendonitis (enthesitis) and symphysitis are
oligoarthritis, whether or not psoriasis is recognised axSpA lesions.
present, i.e. PsA has to be .considered
possible. Initial management should be 24.54. Answer: A.
with an NSAID and consider intra-articular Direct evidence from randomised. controlled trial
steroid injection (under light general data suggest TNF-a inhibitors ustekinumab
anaesthetic). (anti-IL-12/23 monoclonal) and secukinumab

downloaded from www.medicalbr.com


296 • RHEUMATOLOGY AND BONE DISEASE

(anti-IL-17) treat enthesitis successfully, although


evidence that non-biologic immunotherapies do
is scant. Ultrasound studies have shown
-of MCTD. Patients with MCTD have some
features of SScl, SLE and myositis but the
sclerodactyly differs in appearance to the
l I

inflammation at painful and at symptomless sclerodactyly of SScl. Antibodies to DNA and


entheses in PsA. MRI and US can detect Sm ('Smith') are characteristic of SLE and
enthesitis. Enthesitis can occur at any site of positivity to Ro(SSA) and La(SSB) together
entheseal tissue - which includes any soft tissue suggests PSS.
structure attachment to bone and at the nail
bed-distal interphalangeal joint tissue complex. 24.59. Answer: D.
Enthesitis occurs in both HLA-827 positive and The patient may well have SLE and an
negative individuals with spondyloarthritis. associated glomerulonephritis needs to be
promptly identified/ruled out. Steroids may be
24.55. Answer: B. indicated but more initial information is needed
Apremilast inhibits PDE4, which then - and that may include kidney biopsy to grade
secondarily reduces pro-inflammatory cytokine glomerulonephritis and guide therapy choices.
production including IL-17 and TNF-a but also Any delay in obtaining information in a potential
increases production of the anti-inflammatory case of lupus nephritis can be dangerous.
cytokine IL-1 0. Inhibition of RANK ligand is
achieved by denosumab monoclonal antibody, 24.60. Answer: E.
used in treating osteoporosis. CDS0/86 binding Investigation of patients with polymyositis
(T-cell co-stimulatory signalling) by abatacept necessarily needs to be comprehensive.
(CTLA-Ig), blocks T-cell function, and is licensed Skeletal muscle (MRI and then open biopsy at
for use in treating RA. STAT3 is a potential MRI-identified abnormal muscle site), cardiac
therapy target in a number of autoinflammatory muscle (echocardiogram but not cardiac MRI
and autoimmune diseases given its role in initially) and gastrointestinal tract muscle
promoting differentiation of Th17 cells. {barium swallow) need investigating for disease
involvement. Insufficient skeletal muscle may be
24.56. Answer: A. obtained for all required analyses from needle
There is no evidence that cannabinoids improve biopsy. In a minority of cases, polymyositis is
any aspect of fibromyalgia. The quality of associated with malignancy, so whole-body CT
evidence (see http://www.cochranelibrary.com/ may screen for deep-organ malignancy and
topic/Rheumatology/Fibromyalgia/) for all lymph gland enlargement. DXA scan is required
treatments in fibromyalgia is generally poor, to guide steroid-induced osteoporosis risk
perhaps with the exception of supervised management - most cases of adult
aerobic exercise. However, some evidence polymyositis require high-dose and persistent
exists showing responses in a minority of steroid therapy.
studied patient populations for a wide range of
interventions. 24.61. Answer: D.
Echocardiography and RFTs in SScl screen for
24.57. Answer: B. features of pulmonary hypertension (PHT),
Tenderness at entheses can occur, as there will which in SScl may not present with dyspnoea
be tenderness elsewhere, if there is pain until the lesion is quite advanced. Proactive
sensitisation (as seen in fibrornyalgia, which is screening identifies early right heart effects and
associated); however, inflammation at entheses pre-symptomatic reduced lung gas transfer.
is not a recognised feature of HMS. Lax internal PHT is life-threatening but can be treated with
supportive connective tissue can result in a prostacyclin analogue (e.g. iloprost), sildenafil
uterine, vaginal or rectal prolapse. Hypertension (inhibits cGMP-specific phosphodiesterase type
does not occur but postural hypotension 5) or bosentan (endothelin-1 mediated
may be part of a syndrome of autonomic vasoconstriction blocking). Respiratory lesions
dysfunction (postural orthostatic tachycardia should be investigated in SLE and
syndrome; POTS) associated with HMS. dermatomyositis if there are any relevant
symptoms and on ti:Je basis of examination
24.58. Answer: A. signs (i.e. of interstitial lung disease). Uveitis
Antibodies to RNP antigens (e.g. U1 RNP) are is not a common feature of lupus in adults. The
suggestive - in the appropriate clinical context presentation of polymyositis is associated with

downloaded from www.medicalbr.com


RHEUMATOLOGY AND BONE DISEASE • 297

malignancy so initially CT screening is helpful; headache and other cranial symptoms is not
however, the merit of regular yearly CT uncommon in the disease. Sarcoid is possible
monitoring in the absence of detecting - as the rash may be erythema nodosum,
malignancy thereafter has not been which is common to sarcoid and BD - but
substantiated. there is no hypercalcaemia or ankle joint
involvement here, which would be more typical
24.62. Answer: C. in sarcoid. Sinus thrombosis in BD can be
Even in severe disease, structurally normal detected by either head CT or MRI but sarcoid
parts of salivary glands can be seen. Their in the brain often affects meninges at the
sub-function may be a consequence of base of the brain and ideally requires
cytokine inhibition of neurotransmitter function. gadolinium-enhanced sequences on MRI to
Humidity, blink rate (and therefore tasks being disclose lesions adequately.
undertaken) and air conditioning all affect the
degree to which surface moisture from eyes 24.66. Answer: A.
and mucous membranes evaporates, and Cannabis use has been linked to causing an
therefore affects symptoms. Patients who have occlusive vascu/opathy similar to
previously failed to benefit from eye lubricants thromboarteritis obliterans. Hepatitis C is
managed to do so after a trial of topical eye associated most commonly with
drop steroids. PSS is significantly associated cryoglobulinaemic vasculitis. HIV and indeed
with the development of lymphoma. many different viruses are considered potential
triggers of vasculitis. HLA-851 is associated
24.63. Answer: D. with Behc;;et's disease -the main manifestation
In theory, all the features can conceivably occur of which is a vasculitis. lgA production and
in all the conditions but the likelihood of a deposition in vasculitis lesions is a characteristic
subacute, relapsing/remitting condition involving of Henoch-Schonlein purpura vasculitis.
different tissues arising at different times
supports the diagnosis of granulomatosis with 24.67. Answer: D.
polyangiitis (GPA; formerly Wegener's) here. Case examples and rationale exist to support
Fever and rash are generally temporally related an association of Propionibacterium acnes and
in post -streptococcal reactive arthritis, SAPHO syndrome but the level of proof that
adult-onset Still's disease and rheumatic fever. the organism is responsible for, or associated
Lung malignancy would not be common in a with, a substantial number of cases of SAPHO
non-smoker and features of it unlikely to remit is not high. SAPHO is thought to represent a
over time. spectrum of pathophysiological features
possibly contiguous with features seen in
24.64. Answer: B. childhood chronic relapsing multifocal
The diagnostic terminology for GPA and MPA osteomyelitis (CRMO). Associations between
have been subsumed under the new diagnostic the other microorganisms and their
classification 'ANCA-associated vasculitis autoinflammatory or autoimmune condition are
(AAV)', partly owing to their association with more robust, based on good epidemiological,
autoantibodies to neutrophil antigens clinical and immunological data or pathogenetic
(antineutrophil cytoplasmic antibody (ANCA) vs. principles.
intracellular antigens proteinase-3 (PR3) and
myeloperoxidase (MPO) for GPA and MPA, 24.68. Answer: C.
respectively). A substantial number of patients PSS is associated with about a 15% risk of
with polymyositis have antinuclear antibodies lymphoma (PSS with mucosa-associated
(ANAs) and myositis-specific antibodies. GCA is lymphoid tissue lymphoma; 'MALToma'). RA is
not associated with autoantibodies. associated with malignancy with a standardised
incidence ratio (SIR) of 1.1 . The risk is greatest
24.65. Answer: A. for Hodgkin lymphoma (SIR 3.21) and lung
The features are suggestive of Behc;;et's cancer (SIR 1.64). Relapsing polychondritis can
disease (BD). A history of genital lesions may be associated with coincident hematological
not be volunteered. Variollis inflammatory eye malignancy (particularly myelodysplasia) in a
lesions can occur in BD and cerebral venous small minority of cases. DISH is associated with
sinus thrombosis as a cause of non-specific diabetes and possibly with CPPD disease, but

downloaded from www.medicalbr.com


, I
298 • RHEUMATOLOGY AND BONE DISEASE

not with malignancy. Giant cell arteritis occurs examining for crystals (urate-causing gout or
in the elderly, a population in which malignancy calcium-containing causing pseudogout). CKD
is not unusual but there is no known stage 3b-5 is associated with urate and
association. calcium-containing crystal-induced
musculoskeletal disease. Secondary joint
24.69. Answer: D. infection following incompletely treated urine
There is new bone formation at ligament infection is possible but also previous infection
attachments at the distal ulna (this is a can trigger subsequent bouts of crystal arthritis.
non-articular part of the carpus) typical of PsA.
The feature of juxta-articular ('fluffy') new bone 24. 71. Answer: A.
adjacent to joints is highlighted in the CASPAR Barbotage is a procedure usually done under
classification criteria for PsA (Box 24.69). The ultrasound guidance whereby needle disruption
radiocarpal joint space is reduced here from of calcific deposits in tendons is undertaken
PsA also. There is an absence of subchondral (e.g. calcific supraspinatus tendonitis). The
cysts or sclerosis and osteophytes (thus technique usually involves repeated
unlikely to be primary OA or CPPD), and no RA high-pressure fluid injection and aspiration. It
or gout erosions present, nor periarticular has not been shown beneficial for calcinosis
osteopenia, as seen in active RA. cutis. Thalidomide is an extremely effective
treatment for the severe mucosal ulcers
in Behr;:et's disease. Local glucocorticoid
24.69 The CASPAR criteria for psoriatic arthritis
injection is useful for treating a number of
Inflammatory articular disease ijoint, spine or enthesis) .non-inflammatory enthesopathic lesions (such
with ~ 3 points from the following (1 point each unless
stated):
as plantar fasciitis and elbow epicondylitis).
Current psoriasis (scores 2 points) Rituximab has now been shown useful in some
History of psoriasis in first- or second-degree relative patients with AAV, inducing as well as
Psoriatic nail dystrophy maintaining remission. Oral pilocarpine can
Negative lgM rheumatoid factor*
improve salivary and other glandular secretion
Current dactylitis
History of dactylitis in all but late PSS. A trial of therapy 5-1 0 mg 3
Juxta-articular new bone1 times daily can be attempted over a month.
*Established by any method except latex. 11/1-defined
ossification near joint margins (excluding osteophytes) on 24.72. Answer: D.
X-rays of hands or feet. Reactive arthritis can present like axSpA or AS
(CASPAR= ClASsification for Psoriatic ARthritis)
with low back axial symptoms. Radiographs are
frequently normal in early SpA conditions.

I 24.70. Answer: E.
The clinical features are fairly non-specific and
HLA-827 is positive in 95% of people with an
old (modified New York criteria) definition of AS
I gout, septic arthritis and pseudogout are but is less prevalent in cohorts of patients

-~
possible diagnoses. Indeed, severe gout or diagnosed with axSpA. In axSpA (and AS) and
pseudogout can cause systemic symptoms all SpA conditions, the acute phase response
identical to those caused by infection. The may be normal and associated clinical
diagnosis is made on knee fluid aspiration and problems (current or previous) include anterior
then Gram stain and culture of the fluid, but uveitis, psoriasis, inflammatory bowel disease,
also polarised light microscopy of joint fluid enthesitis and sterile urethritis.

downloaded from www.medicalbr.com


JP Leach, RJ Davenport

Neurology
Multiple Choice Questions
25.1. A 44 year old woman is admitted with B. Dissociated sensory loss is always
abrupt onset of neurological deficit. The associated with reflex changes
referring physician suspects a brainstem stroke. C. Dissociated sensory loss is usually a sign of
Which of the following combinations of signs brainstem pathology
and symptoms would be most likely to originate D. Dissociated sensory loss means
from a brainstem lesion? loss of sensation over one-half of
A. Bilateral optic neuropathy the body
B. Cranial nerve signs with sensory and upper E. Dissociated sensory loss requires testing of
motor neuron signs in all four limbs pin-prick, light touch, proprioception and
C. Horner's syndrome with ipsilateral arm pain vibration in all four limbs
D. Lower motor signs in both arms only
E. Upper motor neuron signs in both legs only 25.4. A 25 year old woman presents to the
emergency department with a rapidly evolving
25.2. A 34 year old man is admitted with severe headache. She has a family history of
worsening weakness in both legs. Over the cerebral neoplasm and is worried that this is
course of 3 days he has found it increasingly the cause of her headaches.
difficult to walk upstairs. In the last 12 hours he Which of these accompanying features
complains of feeling mildly breathless. would suggest a diagnosis other than
The referring physician finds weakness in all migraine?
four limbs and cannot elicit reflexes. He can A. Asymmetrical reflexes
find no sensory problems. B. Exacerbation by exercise
Which diagnosis is most likely? C. Photophobia
A. Guillain-Barre syndrome (GBS) D. Unilateral site
B. Inflammatory myopathy E. Vomiting
C. Myasthenia gravis
D. Peripheral neuropathy 25.5. An 18 year old male presents with
E. Spinal stroke numbness in both legs evolving over a few
weeks. He reports some variable sensory
25.3. A 38 year old man presents with sensory alteration and is worried that this might
changes in both legs, which he finds difficult to represent multiple sclerosis. Gait is slower than
characterise. The referring physician asks you usual but there is no reported weakness.
to review him 'to assess him for a dissociated Which of the following would suggest a
sensory loss'. lesion outside the spinal cord?
Which of the following statements is true A. Band of hyperaesthesia across the trunk
about dissociated sensory loss? B. Loss of reflexes
A. Dissociated sensory loss is a sign of C. Preserved vibration but loss of pin-prick
peripheral nerve disease sensation over both legs

downloaded from www.medicalbr.com


.,
300 • NEUROLOGY I

D. Sphincter disturbance C. Long duration of symptoms (more than


E. Wasting of both quadriceps 2 years)
D. Loss of biceps reflex on the symptomatic left
25.6. A 23 year old man complains of some side
excessive daytime sleepiness. Which clinical E. Restriction of lateral neck flexion
features may suggest an underlying diagnosis
of narcolepsy. 25.10. A 78 year old male presents with a
A. Hypnic jerks year-long history of lower back pain
B. Myoclonus on awakening occasionally radiating down one or both legs.
C. Periodic limbs movements in sleep What feature would suggest that MRI of the
D. Restless legs lumbosacral spine is indicated?
E. Sleep paralysis A. A band of altered sensation across the
costal margin
25.7. A 58 year old man presents having had B. Impaired vibration sense in both legs
three unprovoked generalised tonic-clonic C. Localised tenderness over the lower back
seizures in the last 3 weeks. He has a history D. Loss of both ankle jerks
of severe head injury with frontal contusions E. Recent onset of urinary incontinence
some years before. His wife reports some blank
episodes lasting a few minutes with some 25.11. A 64 year old woman presents to
manual automatisms and lip smacking for 1-2 hospital with a 3-day history of worsening
minutes. He has no history of any other 'confusion' and disorientation. She has no past
medical problems. medical history other than hypertension. She
Which antiepileptic drug would be has had no recent foreign travel.
recommended as first line? Examination shows her to be drowsy, but
A. Clobazam even when awakened she is unable to answer
B. Lamotrigine questions, although she can follow direction
C. Levetiracetam and complies with the examination. Her
D. Pregabalin temperature is 38.5°C. She has some neck
E. Sodium valproate stiffness but no rash.
Cranial nerve examination is normal. She has
25.8. A 19 year old woman presents having had bilaterally upgoing plantar reflexes but no
three unprovoked generalised tonic-clonic apparent other neurological signs.
seizures on awakening in the last 3 weeks. What is the likely causative organism?
She has a history of some brief twitching A. Borrelia burgdorferi
movements occurring in the mornings after B. Herpes simplex
some sleep deprivation but no other episodes C. Herpes zoster
of altered awareness. She has no history of any D. Neisseria meningitides
other medical problems. E. Streptococcus pneumoniae
Which antiepileptic drug would be
recommended as first line? 25.12. An 18 year old male student has been
A. Carbamazepine horne from university for 3 days, during which
B. Levetiracetarn time he has become increasingly drowsy. He is
C. Phenytoin able to be roused but is disorientated.
D. Sodium valproate He has a temperature of 39°C, has marked
E. Topirarnate neck stiffness and a positive Kernig's sign. He
has developed a spotting, non-blanching rash
25.9. A 44 year old woman presents with a over his anterior chest wall. He has no focal
history of neck pain and some tingling in the neurological deficit.
upper limbs. Which of the following features What is the most appropriate next course of
would make you keen to carry out magnetic action?
resonance imaging (MRI}? A. Administer intravenous (IV} benzylpenicillin
A. Awakening with tingling in one or both hands B. Arrange for a computed tomography (CT)
B. History of her hearing 'clunking' on neck brain scan
movements C. Carry out a lumbar puncture

downloaded from www.medicalbr.com


NEUROLOGY • 301

D. Puncture one of the purpuric lesions for Examination shows motor signs in the legs
microscopic analysis only with increased reflexes and upgoing
E. Take blood for viral polymerase chain plantars. All modalities of sensation are reduced
reaction (PCR) test below the costal margin.
What is the likely underlying process?
25.13. With regard to the patient in Question A. Cerebral metastasis
25.12, IV benzylpenicillin has now been B. Metastatic spinal cord compression
administered and his cerebral imaging has C. Paraneoplastic encephalopathy
been shown to be normal. A lumbar puncture D. Paraneoplastic Guillain-Barre syndrome
has been carried out. E. Paraneoplastic neuropathy
What is the most likely pattern of abnormality
to emerge in cerebrospinal fluid (CSF)? 25.16. A 33 year old female has had a severe
A. Normal white cells, normal protein, low pain over her left shoulder, which increased
glucose gradually over the initial 24 hours, corning on 2
B. Normal white cells, raised protein, normal weeks after an influenza vaccination. It is a dull
glucose unremitting ache for which she was given
C. Raised white cells (90% lymphocytes), raised opiate analgesia for several weeks.
protein, low glucose Since the pain subsided she has had some
D. Raised white cells (90% neutrophils), normal weakness of hand movements - most
protein, normal glucose particularly in holding and turning a door key.
E. Raised white cells (90% neutrophils), raised She has reduced reflexes in the left arm, with
protein, low glucose somE) subjective decrease in pin-prick sensation
over all dermatomes in the left arm.
25.14. A 38 year old man presents to his family What is the most likely diagnosis?
physician with a 3-rnonth history of a change in A. Brachial neuralgia
sensation in both arms. His wife has been B. Cervical radiculopathy
trying to get him to seek help for worsening C. Guillain-Barre syndrome
hand weakness and progressive gait D. Herpes zoster-related neuralgia
difficulties. E. Transverse myelitis
Examination shows him to have no cranial
nerve signs. He has marked wasting of intrinsic 25.17. An 18 year old female is referred by her
muscles of both hands and brisk leg reflexes optician after an abnormal visual field test. She
with upgoing plantar responses. Sensory had her vision checked after complaining of
examination shows him to have lost pin-prick headaches and formal perimetry has shown
sensation over both arms and the upper half of enlargement of both blind spots.
his trunk. Vibration and proprioception are Further clarification of her symptoms has
normal. revealed a 6-month history of worsening daily
What is the likely pathology? headaches, increased on bending and
coughing, sometimes accompanied by transient ~
A. Metastatic lesion in the upper spinal cord
flashing lights lasting seconds at a time. ~
B. Motor neuron disease
C. Peripheral neuropathy Neurological examination confirms the
D. Spinal cord stroke enlargement of blind spots with some
E. Syringomyelia accompanying papilloedema. No other focal
deficit was found.
What is the likely diagnosis?
25.15. A 64 year old man is referred to the
emergency department by his family physician. A. Cerebral venous sinus thrombosis
He has been undergoing radiotherapy for B. Idiopathic intracranial hypertension (IIH)
a small cell carcinoma of lung for the last C. Intracranial neoplasm
2 months. D. Migraine with aura
He sought help this morning for some back E. Optic neuritis
pain and gait difficulty evolving over the last
day. He has no symptoms in the arms. He 25.18. A 21 year old man was involved in a
reports some recent difficulty in initiating clash of heads while playing football. He was
urination. unconscious for about a minute but recovered

downloaded from www.medicalbr.com


302 • NEUROLOGY 1 I
i

and was able to play on for the remaining half precipitated by rising from a lying position,
hour. He did not report any concussive building up over 4-5 minutes each time and
symptoms and was able to go out for a meal necessitating that she lie back down.
with a few friends where he consumed two She is distressed and cannot sit up for any
pints of beer. length of time. Examination shows no change
The next morning his friends cannot rouse in cranial nerves. Her reflexes are generally
him from sleep. An ambulance is called and brisk but plantar responses are downgoing,
takes him immediately to hospital. On and there is no other deficit in the limbs·.
admission he is apyrexial and has a Glasgow She has normal blood tests and a normal CT
Coma Scale (GCS) score of E2 V3 M2. His of brain but no other investigation.
pupils are symmetrical and reacting to light. What is the likely cause of her headache?
Plantar response is upgoing on the right. A. Cerebral venous sinus thrombosis
What is the likely diagnosis? B. Cluster migraine
A. Alcoholic coma C. Intracranial tumour
B. Extradural haernatorna D. Spontaneous intracranial hypotension
C. Post -traumatic tonic-clonic seizure E. Subarachnoid haemorrhage
D. Subdural haernatorna
E. Viral encephalitis 25.21. A 44 year old man has been in
hospital for 3 weeks for management of
25.19. A 75 year old woman had a diagnosis of decompensated alcoholic liver disease.
Alzheimer's disease made 3 years ago. He awakens with an inability to dorsiflex the
Recently her mobility has begun to deteriorate right ankle.
and she has had a number of falls, twice Examination shows normal movements
having her skull X-rayed in the emergency otherwise bilaterally. There is no wasting and
department as a result of her injuries. reflexes are intact. Sensory examination shows
She has a history of hypertension and reduced pin-prick sensation of the right lateral
transient ischaernic attacks and is on aspirin shin. He has slight tenderness over the lower
and rarnipril. Her daughter says that her back bilaterally but no other findings.
memory and concentration are much worse What is the likely cause of his weakness?
over the last 2 weeks and she can go for long A. Alcoholic neuropathy
spells where she is difficult to rouse. B. Cerebral infarct
On examination she is apyrexial and drowsy, C. Common peroneal nerve lesion
and she is disorientated in time and place. Her D. Sciatic nerve lesion
GCS score is E5 V4 M5. There are no cranial E. Tibial nerve lesion
nerve abnormalities, but she is weaker on the
left, with generally brisk reflexes and upgoing 25.22. A 23 year old woman presents havi11g
plantar reflexes. She has some frontal release had three generalised tonic-clonic seizures in
signs (pout and grasp reflexes) bilaterally. the previous 3 weeks. Which of the following
What is the most likely explanation for her would suggest a focal origin to her epilepsy?
decline? A. History of 'blank spells' in childhood
A. Alzheimer's disease B. History of morning myoclonus
B. Extradural haernatorna C. Prolonged post -ictal dysphasia
C. lschaernic stroke D. Prolonged seizure (lasting 2-3 hours)
D. Metabolic encephalopathy E. Seizures on awakening
E. Subdural haernatorna
25.23. A 56 year old right -handed man is
25.20. A 34 year old woman has a long history admitted with an abrupt onset of loss of
of migraine with aura happening three or four speech. Comprehension appears to be
times per year. After a recent episode where preserved and he can follow direction with no
she had visual aura, typical severe headache, difficulties. He cannot repeat words or phrases.
recurrent vomiting with photophobia and an Where is the abno~mality most likely to be
intolerance of noise, she is left with a different situated on imaging?
character of headache over the subsequent 10 A. Left and right frontal lobes
days. This is a severe pounding headache B. Left frontal lobe

downloaded from www.medicalbr.com


NEUROLOGY • 303

c. Left temporal lobe What are his chances of being rendered


D. Right frontal lobe seizure-free by a single antiepileptic drug?
E. Right temporal lobe A. 1%
B. 10%
25.24. A 78 year old man presents with speech c. 40%
difficulty. His response to speech is intact, but D. 60%
the words are poorly formed and at times E. 80%
difficult to understand. The clinical diagnosis of
a dysarthria is made.
25.28. A 30 year old woman presents having
Which of the following would suggest a
had a single generalised tonic-clonic seizure on
cerebellar cause of his dysarthria?
awakening a week ago. She exhibits no
A. Associated pseudobulbar features neurological deficit and initial investigation with
B. Fatiguability (worsening as the day CT brain and electrocardiogram (ECG) have
goes on) been normal.
c. Nasal regurgitation when swallowing fluids She volunteers some other symptoms.
D. Scanning speech (lack of variation in Which of these would be considered
cadence) physiological rather than epileptiform?
E. Stammering rapid speech
A. Bitten tongue on awakening
B. Episodes of lost time with automatic
25.25. A 38 year old man has come to see you
behaviour
because of some progressive gait difficulties.
C. Episodes of lost time with no automatisms
He has had several falls at home. On calling
D. Jerking of whole body on falling asleep
him through to your consulting room you notice
E. Jerks of both arms on awakening
that his walk is abnormal. (Note: it is always
good practice to watch your patients as they
make their way in.) 25.29. A 22 year old man presents with a
Which of these features would be most likely history of fatigue and muscle cramps. He now
to suggest a peripheral nerve problem as the feels unable to take part in I 0 km runs with his
cause of his walking difficulties? friends.
Which of these symptoms or signs is most
A. Broad-based shuffling gait
likely to suggest an underlying muscular
B. Circumduction of one foot
problem?
C. Inability to turn quickly
D. Rapid small steps with no festination A. Dark urine after prolonged exertion
E. Slapping one foot B. History of 'dropping things'
C. Intermittent focal fasciculations over 5 years
25.26. A 38 year old woman presents to her D. Nocturnal leg cramps
optician with a history of blurred vision over E. Strong family history of diabetes mellitus
several months. Computerised visual field
testing shows that she has bilaterally enlarged 25.30. A 28 year old woman complains of
blind spots. excessive daytime sleepiness. She often finds
Where will the pathological process occur herself falling asleep in lectures during the day.
that will cause this selective change? She is worried that she might have a diagnosis
A. Macula of narcolepsy/cataplexy.
B. Optic chiasm Which of the following features is a normal
C. Optic discs phenomenon and would help you to reassure
D. Optic nerves her?
E. Peripheral retina A. Family history of excessive daytime
sleepiness
25.27. A 42 year old man presents with a B. Hallucinations - experiencing visions of
history of two nocturnal and one daytime intimidating people on awakening
generalised tonic-clonic seizures over the C. Sleep paralysis - awak~ning unable to move
previous 3 months. He is reluctant to start for 45 seconds
treatment because of the. risk of side-effects D. Sudden collapses related to fright
and his uncertainty of efficacy. E. Sudden whole-body jerking on falling asleep

downloaded from www.medicalbr.com


304 • NEUROLOGY
I I
I
25.31. A 35 year old female presents with a Which feature would suggest a lesion within
6-day history of delirium and disorientation. the spinal cord as the cause of his problems?
She is pyrexial but aside from being unable to A. Bilateral lower limb hypertonicity,
answer questions or follow direction, exhibits hyper-reflexia, and upgoing plantar reflexes
no neurological deficit. B. Circumduction of the left foot
After normal imaging has been carried out, a C. Difficulty with heel-toe waking
lumbar puncture is done, which shows the D. High-stepping gait
following results: white cell count 35x109/L; E. Slapping of the feet against the ground
blood film - 90% lymphocytes; CSF protein
0.65 g/L; CSF glucose 4.2 mmoi/L (76 mg/dL); 25.35. A 35 year old patient presents with
serum glucose 6.0 mmoi!L (1 OS mg/dL) (normal weeks of progressively worsening left-sided
CSF glucose is > 60% of contemporary serum facial pain. Which of the following would
glucose). suggest that the cause is outside the superior
Which process would be a likely cause of orbital fissure?
this picture?
A. Diplopia on gaze to the left
A. Brainstem encephalitis B. Diplopia on gaze to the right
B. Meningococcal meningitis C. Left proptosis
C. Subarachnoid haemorrhage D. Reduced visual acuity in the left eye
D. Tuberculous meningitis E. Sensory alteration over the left eye
E. Viral encephalitis
25.36. A 43 year old woman presents with a
25.32. A 76 year old man presents with history of vertigo and vomiting.
a left-sided facial weakness of rapid Which of the following features would be
onset. He has no past medical history most in keeping with a diagnosis of acute
of note. labyrinthitis?
Which feature would suggest that the
A. Evolving symptoms over weeks
deficit is caused by an upper motor neuron
B. Improvement in symptoms following an Epley
lesion?
manoeuvre
A. Deviation of the tongue to the left on C. Ipsilateral sensorineural hearing loss
protrusion D. Nystagmus worsened by change in position
B. Hyperacusis on the left E. Precipitation by minor head injury
C. Loss of taste in the anterior two-thirds of the
tongue on the left 25.37. A 28 year old right-handed man
D. Preservation of eyebrow elevation on the complains of visual disturbance after sustaining
affected side a significant head injury in a road traffic
E. Weakness of eyebrow elevation on the accident. CT scan shows a fracture in the left
opposite side parietal region with an underlying cerebral
contusion and extradural haematoma.
25.33. A 64 year old woman presents with a left What visual symptoms would you expect to
foot drop of gradual onset. result from this injury?
Which of the following would suggest that
A. Diplopia on looking to the right
the responsible lesion is in the common
B. Left -sided neglect
peroneal nerve rather than a more proximal
C. Left superior quadrantanopia
lesion?
D. Reduced visual acuity on the left
A. Reduced left ankle jerk E. Right inferior quadrantanopia
B. Reduced pin-prick in lateral shin
C. Reduced pin-prick sensation in the medial 25.38. A 54 year old female presents with a
shin 6-month history of recurrent headaches. They
D. Tinel's sign over the fibular neck affect the right periorbital region, with a
E. Weakness of ankle inversion throbbing quality, and make her feel sick.
Sometimes the rigbt eyelid appears droopy with
25.34. A 57 year old man is referred to the clinic the pain, and she prefers to go to bed and
because of some difficulty with unsteadiness on sleep it off. It usually lasts a few hours. When
walking. younger, she recalled headaches with her

downloaded from www.medicalbr.com


NEUROLOGY • 305

periods for which she would take analgesia, but C. Functional sensory symptoms
these were different from the current D. Multiple sclerosis
symptoms. In between attacks, she is well and E. Ulnar entrapment neuropathy
on no medication.
Which of the following is the most likely 25.42. An 18 year old male presents to a
diagnosis? remote hospital 3 hours after being felled by a
A. Carotid artery dissection single punch. He was briefly knocked out,
B. Cluster headache seemed to recover, before becoming
C. Migraine increasingly drowsy, then losing
D. Temporal arteritis consciousness.
E. Tension-type headache On arrival in the emergency department,
his neurological examination shows: no eye
25.39. A 66 year old man presented with 6 opening, incomprehensible sounds, flexing to
weeks of intermittent diplopia, improved by pain on the right, extending on the left. His
closing one eye. His family physician has pulse is 50 beats/min, regular; blood pressure
checked a variety of blood tests - all were is 210/115 mmHg. His right pupil is fixed and
normal except antibodies to the acetylcholine dilated. His airway is compromised and he is
receptor (AChR), which returned strongly intubated and ventilated. The nearest hospital
positive with a high titre of antibodies. with a neurosurgeon and scanner is 6 hours
What is the next most relevant test? away by ambulance.
What is the best course of action?
A. Antibodies to muscle-specific kinase (MuSK)
B. CT chest A. Burr hole on the left side of the head
C. Electromyography (EMG) B. Burr hole on the right side of the head
D. MRI head C. Palliative care
E. Tensilon test D. Transfer him to the nearest hospital as soon
as possible
25.40. A 70 year old female presents with E. Treat him with mannitol and intensive care
variable weakness of her legs; she has lost a
significant amount of weight recently, complains 25.43. A 74 year old woman presents with a
of a dry mouth and, more recently, a cough, 12-month history of tremor affecting her right
occasionally with blood. There is little to find on arm only.
examination, and there is uncertainty about Which feature is the most supportive of a
whether her leg reflexes are present. There diagnosis of Parkinson's disease?
are no other signs, although she looks unwell A. Family history of learning disabilities
and thin. B. Her father had a tremor
Antibodies to which of the following are most C. Her husband reports that for the last few
likely to be present? years she has occasionally lashed out or
A. Acetylcholine receptor (AChR) grabbed him while asleep
D. Tremor improves with small amounts of

~.
B. Muscle-specific kinase (MuSK)
C. N-methyl-o-aspartate (NMDA) receptor alcohol
D. Thyroid peroxidase E. Tremor is most apparent when using the
E. Voltage-gated calcium channel (VGCC) arm

25.41. A 28 year old woman presents in the 25.44. A 65 year old man has been diagnosed
I
sixth month of her first pregnancy with with Parkinson's disease. He is reluctant to I
unpleasant tingling affecting the ring and little start treatment, as he has heard that such
fingers, mainly on the left hand and to a lesser treatment only lasts a short time before he will
extent the right, which keeps her awake at become immune to it.
night. She has developed gestational diabetes, Which statement is most correct?
but is otherwise well, with no previous A. He should avoid treatment as long as he
problems. She is on no medication. can, as there is a short therapeutic window
What is the most likely diagnosis? once he has started it
A. Carpal tunnel syndrome (CTS) B. He should delay treatment until his
B. Cervical spondylosis symptoms are interfering with everyday life

downloaded from www.medicalbr.com


306 • NEUROLOGY

C. He should pursue deep brain stimulation otherwise well, and his only medication is
(DBS) surgery rather than medication, as this thyroxine.
is far more likely to be successful What is the likely diagnosis?
D. He should start a non-dopaminergic therapy A. Dystonic tremor
such as trihexyphenidyl B. Enhanced physiological tremor
E. He should start treatment now, as C. Essential tremor
dopaminergic therapies are disease D. Hyperthyroid-associated tremor
modifying E. Parkinson's disease

25.45. A 72 year old male presents with a 25.48. A 66 year old female is brought to the
12-month history of a right arm tremor at rest, emergency department by her worried husband .
micrographia and generalised slowness. He is in the late afternoon. She had been well when
finding it increasingly difficult to turn over in they got up that morning; he had left to do
bed. He is on no medication, and the some shopping at 1000 hrs, returning an hour
examination reveals mainly right-sided later, expecting her to be ready for a planned
parkinsonism. His grandmother was said visit to see old friends. However, she was still in
to have had Parkinson's disease, and died her dressing gown and seemed to have
aged 82. forgotten they were due to go out. Although he
What is the most appropriate next explained the proposed visit to her several
investigation? times, she kept asking him why he wanted her
A. CT head to get dressed. Shortly thereafter, their
B. Genetic testing for the known mutations neighbour knocked on the door to borrow a
associated with parkinsonism ladder- his wife did not recognise him, and
C. None when told it was their neighbour, she was
D. Serum caeruloplasmin adamant it was not, as she remembered their
E. Single-photon emission computed neighbour as someone quite different. Her
tomography (SPECT) imaging (DaTscan) husband realised she was referring to the
previous neighbour who had moved out 2
25.46. A 69 year old woman has developed odd years before. She got dressed unaided, they
involuntary chewing and 'gurning' movements visited their friends, whom she recognised, but
of her mouth and jaw over the last few months, she seemed to have forgotten a number of
which cause embarrassment. Three years recent events, and kept asking the same
previously, she suffered a minor stroke, but questions repeatedly. By the time she reached
made a good recovery. She has had hospital, she seemed to have recovered back
intermittent vertigo for many years. She takes to normal, although could not recall the
simvastatin, clopidogrel, lisinopril, previous few hours. The examination was
bendroflumethiazide and metoclopramide. normal.
What is the most likely cause? What is the likely diagnosis?
A. Drug-induced dyskinesia A. Early Alzheimer's disease
B. Functional (psychogenic) movement B. Functional (psychogenic) amnesia
disorder C. Post-ictal state following an unwitnessed
C. Huntington's disease seizure
D. Parkinson's disease D. Transient global amnesia (TGA)
E. Post -stroke chorea E. Transient ischaernic attack (TIA)

25.47. A 52 year old male describes a 10-year 25.49. A 45 year old man presents to his family
history of tremor affecting both arms, and more physician very worried about his memory. He
recently his head. His father has a similar but describes difficulty remembering words, making
more mild tremor, as does his older brother. silly errors whilst typing and occasionally
Although his brother claims that alcohol helps forgetting names, albeit transiently. His
his tremor, this patient has never noted such grandfather died in a nursing home having
an effect. It embarrasses him as he is a waiter, gone 'senile' at the age of.87. He is otherwise
and people notice him shaking as he tries to well. He is able to work, although worried
serve; he has, on occasion, split things. He is about his job as there have been recent

downloaded from www.medicalbr.com


NEUROLOGY • 307

redundancies, and he has two young children lost weight. Her family have been alarmed by
and his wife does not work. her sudden bouts of laughing or crying, often
The doctor speaks to his wife (with his with little provocation, and this is apparent in
permission), who is surprised that her husband clinic. She was previously well. Examination is
was at the surgery, as she was unaware of any normal except for a small shrivelled tongue,
problems. She feels that he is stressed, no which moves slowly, barely intelligible speech
more than usual, and confirms that he is and a brisk jaw jerk.
sleeping well. He is on no medicatio11, does not What is the likely diagnosis?
smoke and drinks only occasionally. He A. Brainstern stroke
dropped 2 points on the Montreal Cognitive B. Motor neuron disease
Assessment, both on immediate recall. C. Myasthenia gravis
What is the likely explanation? D. Olfactory groove meningioma
A. Depression E. Polymyositis
B. Early Alzheimer's disease
C. Functional memory disturbance 25.52. A 59 year male has noted complete loss
D. Minimal cognitive impairment of smell and taste in the last few months. He is
E. Sleep apnoea sure this has only been present since he
slipped on ice outside a fishmongers and
25.50. A 79 year old male arrives in the banged the back of his head; he thinks he may
emergency department having developed an have briefly lost consciousness. He had a
acute movement disorder affecting his left arm headache for a few weeks after this and,
and leg that day. He is fully conscious but although he recovered well, he is pursuing legal
distressed. He has recurrent and apparently action against the fishmonger who he maintains
uncontrollable movements mainly of his left was negligent. He is a heavy smoker, but
arm, which suddenly shoots out at odd angles, otherwise well.
and flails, before being still for a few seconds, What is the most likely cause of his anosmia
then repeats the wild movements. There is a and ageusia?
similar but less dramatic pattern in his leg. He A. Idiopathic
does not appear weak, and has no other B. Malingering
symptoms or signs. He underwent triple C. Parkinson's disease
coronary artery bypass grafting 10 years D. Post-head injury
previously, and is on treatment for E. Smoking
hypertension. He is normally independent and
generally very well. He is seen by the stroke 25.53. A 32 year old woman with multiple
team who do not think this is the result of a sclerosis (MS) has developed urinary problems.
stroke, and a neurology consult is requested. She frequently feels she needs to pass urine,
Where is the likely lesion? although often passes only small amounts.
A. Brainstem She is intermittently incontinent, and has had
B. Left motor strip several proven urinary tract infections in the last ~

C. Left parietal lobe (angular gyrus) 12 months. A post-micturition ultrasound scan ~

D. Right motor strip reveals a residual volume of 182 mL of urine.


E. Right subthalamic nucleus What is the most appropriate treatment?
A. Bladder-stabilising (anticholinergic) drug
25.51. An 82 year old female was seen in the B. In-dwelling catheter
neurology clinic with a progressive history of C. Intermittent self-catheterisation
speech and swallowing problems over the D. Long-term antibiotic prophylaxis
previous 9 months. Initially she had some E. Pelvic floor exercises
slurring of speech, and was seen in an ear,
nose and throat (ENT) clinic; no vocal cord 25.54. A 36 year old male presents with a
pathology was found. 3-week history of headaches, typically
Her speech deteriorated, and is now difficult awakening him in the ea.ly hours. They are
to understand, with a rather squeaky severe, affecting the right periorbital region, last
characteristic. Her swallowing has also about an hour and are very distressing; he is
deteriorated with frequent choking, and she has unable to find a comfortable position. He has

downloaded from www.medicalbr.com


308 • NEUROLOGY

noticed that his right eye waters and goes red


with the pain. Occasionally he gets an attack
during the day. He thinks he had something
A.
B.
C.
Aquaporin-4 antibody
Lumbar puncture for oligoclonal bands
Paraneoplastic antibodies
l
similar about 2 years ago but it disappeared D. Repeat imaging with contrast
after a week or so. He is otherwise well, but E. Visual evoked potentials
terrified that he has a brain tumour such is the
severity of the pain. 25.57. A 21 year old female was diagnosed with
What is the likely diagnosis? MS 2 years ago after presenting with ·ataxia,
from which she recovered fully; she declined
A. Cluster headache
further treatment at the time as she was
B. Hypnic headache
C. Migraine considering starting a family. She has now
D. Paroxysmal hemicrania developed numbness of her left arm, which has
E. Temporal arteritis alarmed her but is not compromising her
function. She has no other symptoms.
What is the most appropriate immediate
25.55. A 50 year old male describes a 3-week
management?
history of dizziness, often occurring in bed.
Shortly before the dizziness started, he had A. Broad-spectrum antibiotic
walked into a glass door, 'seen stars' but not B. Conservative
lost consciousness. On closer questioning, he C. High-dose oral glucocorticoids
has noted that getting into or out of bed and D. Physiotherapy
rolling over to the left can trigger his symptoms, E. Start a disease-modifying drug of high
which is brief vertigo lasting a few seconds. He efficacy such as natalizumab
has fallen on two occasions as he got out of
bed as a result. He occasionally gets it during 25.58. A 30 year old female presents to
the day, usually when getting into his sports her family physician. Her identical twin
car, but otherwise he is generally well. He is sister was diagnosed with MS last year, and
on no medication, but drinks about 60 units of she has read that her own risk of MS is
alcohol per week. therefore increased, and she is enquiring
What is the appropriate management? about this.
Approximately what is her risk of developing
A. Alcohol abstention
MS?
B. Betahistine
C. Low-salt diet A.5%
D. Short course of glucocorticoids B. 20%
E. Vestibular repositioning (e.g. Epley c. 35%
I manoeuvre) D. 50%
E. 85%
I 25.56. A 36 year old female presents with
f progressive difficulty walking over the previous
week, and now has difficulty passing urine.
25.59. Which of the following scenarios would
represent a reasonable case for requesting an
One year previously she had an episode of electroencephalogram (EEG)?
monocular visual loss which lasted about 2 A. A 15 year old female with a single
weeks. She was abroad at the time, and by the unwitnessed blackout thought to be
time she returned, her vision was recovering so syncope but with associated urinary
she did not seek attention. On examination she incontinence
has an upper motor neuron pattern weakness B. A 24 year old male in a psychiatric unit on
in both legs, with brisk reflexes and upgoing multiple antipsychotics and consistently
plantar responses, and a palpable bladder; drowsy in the mornings
her right optic disc is pale. Her non-contrast C. A 64 year old female having had a single
MRI head is normal, but there is a long lesion generalised tonic-clonic seizure
seen in the thoracic spine, stretching over D. A 68 year old female with two witnessed
several spinal segments, thought to be generalised tonic-clonic seizures
inflammatory. E. A 68 year old male with a previous left
Which investigation is most likely to make hemisphere stroke and recent episodes of
confirm a diagnosis? unwitnessed collapse

downloaded from www.medicalbr.com


NEUROLOGY • 309

25.60. A 28 year old man describes evolving A. Bilateral optic neuropathy


weakness of all four limbs over 8 weeks, and B. Diabetic retinopathy
most recently some dyspnoea. C. Idiopathic intracranial hypertension
Blood tests are normal and lumbar D. Intracranial space occupying lesion
puncture shows a raised CSF protein but no E. Neuromyelitis optica
cells. A diagnosis is made of chronic
inflammatory demyelinating polyneuropathy 25.63. A 32 year old man has a 3-month history
(CIDP). of slowly worsening headaches after sustaining
Which of the following patterns of a minor head injury while playing football. These
abnormality on nerve conduction studies and headaches were worse if he exercises or
EMG is likely to be present at the time of coughs and are sometimes associated with
diagnosis? vomiting and nausea. He has no other systemic
A. Delayed conduction in motor and sensory symptoms.
nerves with denervation on EMG Neurological examination shows him to have
B. Delayed conduction in sensory and motor bilateral papilloedema but no focal neurological
nerves with normal EMG deficit. MRI is reported as showing dilated
C. Normal nerve conduction and EMG studies lateral and third ventricles but no increase in
D. Normal nerve conduction studies but size of the fourth ventricle.
denervation changes on EMG What is the likely diagnosis?
E. Small sensory nerve and compound motor A. Cerebral venous sinus thrombosis
action potentials but normal EMG B. Idiopathic intracranial hypertension
C. Intracranial space-occupying lesion
25.61. A 68 year old woman presents to her D. Normal pressure hydrocephalus
family physician with a 3-day history of E. Stenosis of the aqueduct of Sylvius
'dizziness', by which she means extreme
vertigo, with a sensation of the room spinning 25.64. A 44 year old woman is referred to see
to the right, made worse with any movement. you because of worsening headaches. These
She has vomited on occasions and has been have been present for around 4 years and have
bed-bound but reports no focal neurological increased in frequency and severity such that
deficit. Things have improved slightly in the last she is now self-medicating with doses
24 hours and she can now make her way of paracetamol and codeine tablets 3-4
round the house with some assistance. times daily.
What is the likely explanation for her Her headaches fall into two types: a constant
symptoms? underlying 'throbbing, aching' pain present
A. Acute labyrinthitis every moment of every day, and episodes of
B. Benign paroxysmal positional vertigo severe pulsating headache with photophobia,
C. Brainstem stroke phonophobia and nausea. She has been off

-
D. Meniere's disease work for the last 3 months and has now
E. Migraine become withdrawn, depressed and weepy.
Neurological examination is normal.
What is the likely diagnosis?
25.62. A 26 year old woman saw her optician
because of recent headaches. These are A. Chronic Daily Headache
present on stooping or coughing and often B. Chronic migraine
associated with visual symptoms (small flashing C. Functional headache disorder
dots or brief loss of vision). She has no history D. Intracranial space occupying lesion
of vomiting or other neurological symptoms. E. Subarachnoid haemorrhage
She has no cognitive or depressive symptoms.
Examination shows her body mass index to 25.65. A 28 year old man presents
be 40 kg/m 2 . She has bilateral papilloedema with episodes of unsteadiness lasting
and no venous pulsation on fundoscopy. Blind days to hours at a time on around 3-4
spots are enlarged in both eyes. Neurological occasions per year. The~e are not be
examination is otherwise normal. worsened by changes in position, and he is
What is the likely cause of her fundal not aware of any precipitants. There is no
abnormalities? accompanying pain or visual symptoms and in

downloaded from www.medicalbr.com


31 0 • NEUROLOGY

between these bouts he reports no A. Brain imaging


symptoms. B. Further investigation of neutrophilia and
His father had similar symptoms in later life, hyponatraemia
and his younger brother has started to display G. No further investigations necessary
similarly intermittent symptoms. Neurological D. Sleep-deprived EEG
examination is normal. E. Standard EEG
What is the likely diagnosis?
25.68. A 29 year old male presents 8 weeks
A. Benign paroxysmal positional vertigo
after being hit on the head by a golf ball. He fell
B. Diabetic neuropathy
to the ground but it was unclear whether he
G. Episodic ataxia
lost consciousness or not (if he did, it must
D. Migraine
have been very brief). He went to hospital that·
E. Spinocerebellar ataxia
day, and had a minor laceration to his scalp
glued. He has since been troubled by gradually
25.66. A 62 year old man has a 3-month history worsening headache, poor concentration,
of worsening unsteadiness, causing him to fall irritability and poor sleep, and now he is very
on several occasions. He has not sustained any
miserable and low. He is off work, and
serious injury but now requires more help to spending much of his time asleep on the couch
cook and look after himself in his home. during the day.
He has multiple medical conditions and He has returned to the emergency
requires a range of regular medications, which department on several occasions due to his
he supervises himself. symptoms, and had aCT head 2 weeks ago
Neurological examination reveals nystagmus which was normal. Yesterday he saw an
on lateral gaze with jerky pursuit movements acupuncturist who recommended he seek an
and diplopia on lateral gaze (both directions). MRI head scan.
His speech is slurred but he has no other What is the appropriate management?
neurological deficit. Power and reflexes are
normal in all four limbs. He has an intention
A. Explanation of the 'post -concussion
syndrome'
tremor, past-pointing in both arms and cannot
heel-toe walk because he is so unsteady.
B. MRI head
G. Neurosurgical referral
Which of his medications is most likely to be
contributing to his unsteadiness?
D. Psychiatric referral
E. Tramadol for his headache
A. Diazepam
B. Digoxin 25.69. A 24 year old woman had first seen her
G. Phenytoin family physician a year ago with a history of
D. Prochlorperazine anxiety and panic attacks. In the subsequent 6
E. Salbutamol months she and her family had begun to notice
some cognitive difficulties and short-term
25.67. A 68 year old female presents with a mernory problems (losing the ability to carry out
witnessed sleep-onset convulsion, with typical complex tasks at work, not recognising family
post-event confusion, lateral tongue biting and members).
subsequent myalgia. She reports a single When you see her in the clinic, she is
collapse at the age of 16, and remembers orientated in time and person but is hesitant in
having a 'brain wave' test, which was normal, her answers, being also disorientated in place.
apparently. She is desperate not to lose her She exhibits difficulties in all domains of
driving licence, and believes this was related cognitive testing, especially in tasks requiring
to dehydration caused by the hot weather working memory and concentration.
that day. On examination she has frequent twitching
In the emergency department on the day of movements involvil)g 'all four limbs (her family
presentation, her ECG and routine blood tests have noticed these over the last 2 weeks).
are all normal except for a neutrophilia (total Concentration difficulties limit the usefulness of
white cell count 19.8x 109/L) and sodium of visual examination,.but otherwise she has
133 mmol/L. generally brisk reflexes and upgoing plantars.
What is the next most appropriate What is the most likely cause of her cognitive
investigation? problems?

downloaded from www.medicalbr.com


NEUROLOGY • 311

A. Alzheimer's disease unwell, with some agitation and anxiety,


B. Anxiety disorder with a marked startle response to sound or
c. Hyperthyroidism touch.
D. Sporadic Creutzfeldt-Jakob disease (CJD) He is apyrexial and shows no signs of
E. Variant CJD localised infection. Blood pressure is
165/11 0 mmHg but cardiovascular examination
25.70. A 40 year old right-handed man presents is otherwise normal. Neurological examination
after a single generalised tonic-clonic seizure. is difficult but shows generalised stiffness when
He has no risk factors for development of he is moved or but no other focal neurological
epilepsy but on closer questioning has had deficit.
spontaneous bursts of altered sensation What is the likely cause of his symptoms?
('like a burning') in the left arm lasting seconds A. Botulism
at a time. B. Dental abscess with extension into the basal
Examination is normal except for an skull
asymptomatic homonymous left lower temporal C. Functional illness
quadrantanopia. He has had a CT head which D. Juvenile myoclonic epilepsy
shows a non-enhancing abnormality in the E. Tetanus
substance of the brain.
What is the likely nature of the lesion? 25.73. A 44 year old woman returned from a
A. Basal skull meningioma holiday with her German cousins 1 week ago.
B. Glioblastoma rnultiforme in right parietal lobe In the last 5 days she has begun to notice
C. Low-grade glioma in right parietal lobe worsening diplopia in all directions of gaze. She
D. Medulloblastoma in the right parietal lobe has had some difficulty in swallowing (fluid
E. Optic nerve glioma sometimes coming up through the nose) over
the last 3 days and has had worsening
25.71. A 26 year old woman presents to the weakness in legs and arms over the last 24
emergency department with a rapid onset of hours. She has noticed that she is becoming
headache. There had been a slowly evolving breathless, even at rest. The cousin with whom
scotoma in the right side of her vision, which she was staying has begun to notice the same
had spread over around 35 minutes with some symptoms in the last 3 days.
associated flickering in the vision. The Examination shows her to be apyrexial with
headache afterwards had been severe, and she normal pulse and blood pressure. Forced vital
retired to bed where she vomited on several capacity and forced expiratory volume in 1
occasions. Her family had noted that her second (FEVJ) are reduced. She exhibits
speech had been slurred during the headache bilateral ptosis with reduced eye movements
phase and she felt that she had some in all directions. Pupils are equal and reactive,
associated altered sensation in the and palatal movement is reduced. There is

-
right hand. weakness with preserved reflexes in all limbs.
You see her 24 hours later, at which point What is the likely cause of her symptoms?
she feels drained but otherwise back to normal. A. Botulism
Neurological examination is normal. B. Brainstem stroke
What is the most likely cause of her C. Miller Fisher syndrome
symptoms? D. Multiple sclerosis I
A. Cerebral venous sinus thrombosis E. Myasthenia gravis
B. Focal seizure arising in the occipital region I
C. Migraine with aura 25.74. A 34 year old woman was involved in a
D. Subarachnoid haemorrhage minor road traffic accident 24 hours previously.
E. Transient ischaemic attack Her car had been hit from behind but she had
not lost consciousness and had not suffered a
25.72. A 21 year old male has been working on direct blow to the head. On awakening the next
a farm for the summer. He has been on day she was aware of a 'ight-sided headache
treatment with flucloxacillin for jaw stiffness that with associated neck pain. She had no diplopia
had been diagnosed as an early dental or visual symptoms and no other focal
abscess. Over 5 days, he has become more neurological symptoms.

downloaded from www.medicalbr.com


312 • NEUROLOGY

Examination shows her to have a small right


pupil with mild degree of right ptosis. Eye
movements are full and fundal examination is
normal. Cranial nerve examination is otherwise
with loss of vibration sensation to the knees
and reduced proprioception in toes and ankles.
Initial investigation shows: random glucose
6.8 mmoi/L (123 mg/dl); full blood count
l I
normal. normal; erythrocyte sedimentation rate (ESR)
What is the most likely cause of her 78 mm/hr.
symptoms? What is the likely cause of his symptoms?
A. Extradural haematoma A. Alcoholic neuropathy
B. Subarachnoid haemorrhage B. Diabetic neuropathy
G. Subdural haematoma G. Motor neuron disease
D. Traumatic brachial plexopathy D. Myelopathy secondary to lymphomatous
E. Traumatic dissection of the extracranial deposits
carotid artery E. Neuropathy secondary to myeloma

25.75. A 44 year old woman has a 5-year 25.77. A 54 year old man presents after a
history of progressive left-sided deafness. second bout of left-sided facial palsy in 4 years.
She has been travelling round Asia with her He is a gamekeeper in the Highlands of
work as an aid worker and has not sought Scotland but has been off work with increasing
medical help. fatigue for the last 4 months. He has had some
She has finally presented to her family increasing hyperacusis for the 3 days of his
physician after noticing some mild left-sided recent facial weakness.
clumsiness in her hand and severe headaches, His only other past history is of some
worse on exercising and stooping. She had no assessment at a rheumatology clinic for
family history of neurological disease. generalised aches and pains with worsening
Examination confirms the presence of blurred fatigue.
disc margins bilaterally, a left-sided Examination shows him to be apyrexial with
sensorineural deafness and cerebellar signs in a weakness involving the whole of the left face,
the left arm and leg. with normal fundi and normal eye movements.
What is the likely cause of her progressive Cranial nerve examination is otherwise normal
symptoms? and he has symmetrically normal reflexes and
A. Acoustic neuroma sensory examination.
B. Brainstem stroke What is the likely cause of his symptoms?
G. Meniere's disease A. Bell's palsy
D. Migraine without aura B. Lyme disease
E. Multiple sclerosis G. Multiple sclerosis
D. Stroke
25.76. A 68 year old man presents with E. Syphilis
progressive numbness over 6 months initially
affecting his feet, and spreading up his legs. 25.78. A 48 year old man complains of
Over the last 3 months, his hands have worsening gait difficulty over the last 3 months.
become affected with both numbness and Which of the following symptoms or signs
weakness. He admits to drinking around 12 would suggest that the cause is sited in the
units of alcohol per week. He has no cranial spinal cord?
nerve symptoms and sphincter function is A. Evolution of symptoms over seconds
normal. B. Progressively worsening urinary incontinence
Examination shows him to have reduced G. Sensory loss distally in upper and lower
power symmetrically in both legs distally and in limbs
finger abduction and adduction. Reflexes are D. Widespread upper and lower motor neuron
reduced in all four limbs with downgoing signs
plantars. Sensory testing shows symmetrically E. Worsening diplopia
reduced pin-prick sensation below both knees,

downloaded from www.medicalbr.com


NEUROLOGY • 313

Answers
25.1. Answer: B. 25.5. Answer: E.
The brainstem is a packed centre from where Spinal cord lesions can cause upper motor
most cranial nerve nuclei originate (III-XII) and neuron findings, usually with some degree of
all long tracts pass through. Combination of all sensory change and sphincter dysfunction.
these signs will either signify widespread While reflexes can be lost in the immediate.
neurological disease or a lesion restricted to the aftermath of a spinal cord lesion (so-called
brainstem. 'spinal shock'), lower motor neuron changes
Horner's syndrome and arm pain would (wasting, areflexia) would not occur in isolation
suggest a brachial plexus lesion, while cervical with spinal cord pathology.
spine changes will cause lower motor neuron
signs in the arms (perhaps with upper motor 25.6. Answer: E.
neuron deficit in the legs). The optic nerves only The narcolepsy tetrad is excessive daytime
interact with the brainstem to serve the sleepiness, cataplexy, sleep paralysis and
pupillary light reflex. hypnagogic hallucinations. Hypnic jerks are a
normal phenomenon, while awakening
25.2. Answer: A. myoclonus is a feature of the generalised
Gradual onset of weakness without sensory epilepsy syndromes. Restless legs
signs and loss of reflexes is most in keeping can accompany a range of medical
with GBS. Strokes usually present abruptly and conditions (parkinsonism, iron deficiency,
would not cause lower motor neuron signs. neuropathy) and, like periodic limb movements
Breathlessness would be unusual in a peripheral in sleep, are not associated with narcolepsy/
neuropathy, even if onset is rapid. A myopathy cataplexy.
should not reduce reflexes. Myasthenia gravis
can cause weakness but onset is usually slower 25.7. Answer: B.
with some degree of fatigability. The scenario is highly suggestive of a
focal-onset epilepsy (previous head injury,
25.3. Answer: E. focal-onset seizures with altered awareness
Dissociation of sensory loss arises because of and automatisms). One of the first-line
the different decussation of dorsal columnar treatments is likely to be effective, and
pathways (vibration and proprioception) and lamotrigine is recognised as having an excellent
spinothalamic pathways (temperature/pin-prick). safety profile. Levetiracetam and sodium
Lesions can affect one pathway more than the valproate can be used in focal epilepsies but
other. It can occur in brainstem lesions, but is would not usually be first line. Clobazam and
usually a sign of spinal disease. Effects on pregabalin are considered adjunctive treatments
sensory pathways can be isolated and have no (used alongside other antiepileptic drugs).
effects on reflexes.

~I
25.8. Answer: B.
25.4. Answer: A. The story is highly suggestive of a genetic
Migraine is a recurrent condition that leads to generalised epilepsy, and treatment choice is
episodic headaches with or without warning. dictated by two issues. Firstly, genetic
The phase of severe headaches is usually
associated with heightened sensitivity to stimuli
generalised epilepsies (GGE) can be made
worse (particularly eliciting myoclonus) by the
I
I
(photophobia, phonophobia, •osmophobia). sodium channel blocking drugs, and
Avoidance of any stimuli is commonly carbamazepine and phenytoin should be
described by migraineurs (lying still in a dark, avoided in such cases. Lamotrigine's efficacy in
quiet room). Loss of function {dysarthria, Genetic Generalised Epilepsy is less than with
transient weakness, visual scotomata) can Valproate and this may deter first-line usage
occur but may merit investigation at the first here. Broad-spectrum antiepileptic drugs
occurrence, or if they emerge as an isolated should be used in these Qircumstances.
symptom. Abnormality of the reflexes (either Secondly, the possibility of future pregnancy
tendon reflexes of plantars) would not be (any drug will usually be required long term)
expected in migraine. should prompt avoidance of sodium valproate

downloaded from www.medicalbr.com


31 4 • NEUROLOGY

and any drugs with an uncertain safety profile


in pregnancy. While topiramate can be useful in
GGE, the safety in pregnancy is uncertain and
necessary, a lumbar puncture can be carried
out in due course, but only once the serious
underlying sepsis has been addressed.
1
it is therefore best avoided in the first instance.
I
25.13. Answer: E.
25.9. Answer: D. Acute bacterial meningitis (unless already
The main reason to carry out cervical spine treated with antibiotics) will cause a rise in
imaging is to provide information on any lesion neutrophils in CSF. Bacterial infection·will
that might be operable. It is in such cases that usually raise protein and lower glucose. Viral
a good history and neurological examination encephalitis will cause a rise in CSF
are most important. Where clinical features of lymphocytes and often have little effect on
motor radiculopathy are found (as here) and are protein and glucose. A raised CSF protein and
persistent, then imaging may be considered. no effect on white cells is characteristic of
Signs or symptoms of spinal cord compression inflammatory processes such as Guillain-Barre
(upper motor neuron signs in the legs, or syndrome.
sphincter dysfunction) would make imaging
more important. 'Clunking' sensations in the 25.14. Answer: E.
neck are common and unrelated to specific Syringomyelia is a slowly progressive problem
pathologies. Nocturnal hand symptoms are with expansion of the central canal causing
more likely with carpal tunnel syndrome, while destruction of the cells and tracts in the
prolonged duration of symptoms is common anterior spinal cord. The occurrence of
with benign non-progressive spinal disease. selective spinothalamic loss and anterior horn
cell loss localises this process to the anterior
25.1 0. Answer: E. spinal cord. The lack of leg symptoms makes
Assessment of lower back pain is another this a process in the cervical and thoracic cord.
occasion when a good history and neurological The slow evolution would not be in keeping
examination are required. New onset of urinary with a vascular origin. The sensory findings (as
symptoms or focal radiculopathy may suggest well as the limitation to the arms) makes motor
a structural cause. Localised tenderness is neuron disease much less likely.
common in muscular back pain, while costal
hyperaesthesia would suggest a thoracic 25.15. Answer: B.
localisation. Neurological examination in old age The concurrence of leg symptoms and
is likely to elicit some 'deficits' such as distal sphincter dysfunction with or without pain in
reflex loss and sensory disturbance, even in such a patient is highly suggestive of a
asymptomatic patients. malignant spinal cord compression. Urgent
imaging is needed to allow intervention to

I 25.11. Answer: B.
The pyrexia and evolving neurological deficit
prevent progression to destruction of the spinal
cord. Note that this should include the thoracic
f makes it likely that there is an encephalitic and spinal cord as there may be a 'dropped'

·~ process rather than just a rnening itis. It should


be noted that the patient is not 'confused'; she
spinal level.

is dysphasic. Herpes simplex is the most 25.16. Answer: A.


common cause of a viral encephalitis. Bacterial The scenario of severe pain followed by some
meningitis can cause some isolated cranial minor neurological deficit is typical of brachial
nerve deficits, but the cortical nature of her neuralgia (neuralgic amyotrophy). Cervical
expressive dysphasia would not be expected in radiculopathies can cause pain, but any
a pure meningitis. neurological deficit will coincide with this rather
than follow its remission. There is no rash to
25.12. Answer: A. suggest a herpes zoster infection (which would
This is a classic scenario with rapidly evolving usually also cause a persisting rather than an
clinical disease where it is important to treat early-then-remitting' neuralgic pain). Transverse
first and think later! The rapid development of myelitis may caus& 1back and neck pain but
meningococcal sepsis is a severe threat to life would usually have more marked sensory
and limb and requires immediate treatment. deficit, perhaps with long tract signs in the
Investigation with imaging, bloods and, if lower limbs.

downloaded from www.medicalbr.com


NEUROLOGY • 315

25.17. Answer: B. Cerebral venous sinus thrombosis can cause


The gradual onset of headache with features of severe postural headache but, like other causes
high pressure (papilloedema, worsening with of headache in the list, will usually cause
moves that increase intracranial pressure), and headache precipitated by manoeuvres that
no focal neurological deficit make IIH the most raise intracranial pressure.
likely diagnosis. This is more likely when
patients are obese and where there has been 25.21. Answer: C.
exposure to steroids, oral contraceptives or The singular loss of ankle dorsiflexion makes
tetracyclines. sciatic nerve or radicular involvement unlikely. If
While serious conditions like intracranial ankle inversion is intact (usually tibialis posterior
tumour and cerebral venous sinus thrombosis - supplied by L5 and tibial nerve), then the
should be excluded, the lack of other features likely cause is common peroneal weakness
in someone of this age and gender make IIH selectively affecting tibialis anterior. Alcohol
most likely. There is no episodicity to her excess may make nerves more susceptible to
headaches and no clear features to suggest localised damage, but the focal nature of the
migraine with aura. Optic neuritis can cause deficit would make a generalised process an
optic disc changes and some ocular pain, unlikely cause. It would be unlikely that a
but the pressure features will mitigate cortical lesion would cause such a focal
against this. combined lower motor neuron and sensory
deficit.
25.18. Answer: B.
The story of waking from unconsciousness 25.22. Answer: C.
followed by deterioration, after a head injury, is Prolonged post-ictal dysphasia would suggest
classically that of an arterial bleed into the a focus of onset in the dominant hemisphere
extradural space following head injury. This (either Broca's or Wernicke's areas).
requires urgent imaging and probably acute Generalised epilepsies are thought to be
drainage/decompression. genetic in origin, and will often be accompanied
There is nothing to suggest any extra alcohol by other seizure types such as myoclonus or
intake and the presence of focal signs would absences. Very prolonged episodes lasting
be against this. There is no suggestion of an hours or more are rare with epilepsy and
infective process and there are no markers to (especially where awareness is retained)
suggest a seizure - this would be a diagnosis may result from non-epileptic attack
of exclusion in a case like this. disorder.

25.19. Answer: E. 25.23. Answer: B.


The effect of minor head injury, especially in the In right-handed patients, the dominant
presence of cerebral atrophy and aspirin hemisphere for both limb movement and
treatment, is a risk factor for chronic speech is usually the left. Speech
development of a subdural haematoma, which comprehension is usually sited in Heschl's
is impairing consciousness. Alzheimer's disease gyrus in superior temporal lobe, while
itself would not usually cause such a rapid Broca's area in the frontal lobe is devoted to
decline, and it is unusual for ischaemic strokes formation of speech. Damage to the former will
to impair consciousness. The presence of focal result in so-called receptive or fluent dysphasia
neurological signs would go against her (inability to understand auditory input from
drowsiness having an encephalopathic speech) and the latter with expressive
cause. dysphasia.

25.20. Answer: D. 25.24. Answer: D.


The history is key here - providing evidence Dysarthria can be caused by many
that the headache is related to low intracranial things (Box 25.24) but cerebellar deficits
pressure. A dural tear is likely to have will cause consistently poor articulation
happened while vomiting, caused by the with impaired cadence (sGanning) as this
original migraine, and the clear postural worsens. Poor coordination of limb
association of the subsequent headache makes movement would also usually be noted at
this the prime diagnostic consideration. these times.

downloaded from www.medicalbr.com


316 • NEUROLOGY

25.24 Causes of dysarthria


l i

Type Site Characteristics Associated features


Myopathic Muscles of speech Indistinct, poor articulation Weakness of face, tongue and neck
Myasthenic Motor end plate Indistinct with fatigue and dysphonia Ptosis, diplopia, facial and neck
Fluctuating severity weakness
Bulbar Brainstem Indistinct, slurred, often nasal Dysphagia, diplopia, ataxia
'Scanning' Cerebellum Slurred, impaired timing and Ataxia of limbs and gait, tremor of
cadence, 'sing-song' head/limbs
Nystagmus
Spastic Pyramidal tracts Indistinct, nasal tone, mumbling Poor rapid tongue movements,
('pseudobulbar') increased reflexes and jaw jerk
Parkinsonian Basal ganglia Indistinct, rapid, stammering, quiet Tremor, rigidity, slow shuffling gait
Dystonic Basal ganglia Strained, slow, high-pitched Dystonia, athetosis

Fatiguability of speech is characteristic of scotomata in the corresponding area of the


myasthenia gravis, while pseudobulbar features visual field.
would be most commonly accompanying
multi-infarct states. Stammering rapid speech is 25.27. Answer: D.
a feature of parkinsonian syndromes, while Most patients will be rendered completely
nasal regurgitation is a sign of palatal weakness seizure-free by the first or second drug they are
of whatever cause. prescribed. Some will have a brief relapse
(perhaps related to reduced adherence) while
25.25. Answer: E. others will require long-term treatment with two
Slapping one foot while walking (often after or more antiepileptic drugs in combination.
lifting the affected leg higher than the other) Where there is incomplete or inadequate
is a sign of a foot drop, which is most response, steps should be taken to ensure the
commonly related to common peroneal nerve diagnosis and classification are correct, that the
damage. patient is taking the treatment, and that there
Circumduction is a compensatory movement are no other confounding factors (e.g. evolving
to avoid tripping over a plantar-flexed foot lesion, use of alcohol or epileptogenic drugs).
(most usually related to upper motor neuron In cases where there is an established single
effects), while inability or difficulty in turning is focus, surgical workup may be indicated.
most likely with parkinsonian disorders. Rapid
small steps (marche a petits pas) is a sign 25.28. Answer: D.
of generalised cortical problems (most A crucial part of the assessment of patients
usually vascular), while a broad base is a after a single seizure is to clinically assess
compensation for the impaired coordination of whether there has been any other seizure
cerebellar disease. activity experienced by the patient.
Nocturnal seizures may only be signalled by
25.26. Answer: c. some tongue biting or enuresis (often
The physiological blind spot corresponds to the associated with sdme myalgia or headache).
area of the retina where the optic nerve passes Daytime seizures of absence epilepsy or focal
through and spreads fibres radially (the optic onset with altered consciousness will usually
disc). Any process that swells the optic disc have the patient sitting still or fidgeting,
will cause papilloedema and an increased respectively. Morning myoclonus is
blind spot. characteristic of genetic generalised epilepsy
Macular lesions will cause a reduction in (particularly juvenile myoclonic epilepsy), while
visual acuity (as noted on the Snellen chart) hypnic jerks are, of 8ourse, an entirely normal
while optic chiasm lesions (most usually a phenomenon and not indicative of any
pituitary problem) will cause a bitemporal pathology (other than fatigue).
hemianopia. Optic nerve lesions can cause
blindness when severe but may show only 25.29. Answer: A.
reduction in colour vision on the affected A metabolic muscular problem means that
side(s). Peripheral retinal problems will cause prolonged exercise is liable to cause muscle

downloaded from www.medicalbr.com


r damage resulting in myoglobinuria: presence of
myoglobin causing the black urine.
NEUROLOGY • 317

Bilateral facial weakness should raise the


suspicion of either myasthenia or Guillain-Barre
Diabetes mellitus can occur in mitochondrial syndrome, while ipsilateral protrusion of the
conditions, but it is common and unlikely to be tongue occurs often with a facial palsy and
directly relevant unless there are other clues without other signs would not imply
(accompanying systemic disease, maternal involvement of any other cranial nerves.
transmission and deafness). Nocturnal leg
cramps are common and history of dropping 25.33. Answer: D.
things is non-specific for muscular problems. The common peroneal nerve supplies tibialis
anterior, the toe extensors and sensation
25.30. Answer: E. to the lateral aspect of the shin. The nerve runs
The occurrence of daytime sleepiness in round the fibular neck and irritation at that
narcolepsy is usually accompanied by some of point can produce a Tinel's sign (localised
the other three components of the narcolepsy tenderness of dysaesthesia over a site of nerve
tetrad: namely, cataplexy (collapses with fright damage) in a nerve that is affected early. The
or laughter), hypnagogic hallucinations (visual common peroneal nerve is mostly derived from
hallucinations often with an emotional content) L5 roots, so clinical differentiation can be
and sleep paralysis (loss of voluntary movement difficult.
on awakening). The genetic basis for this An L5 lesion will cause a foot drop alongside
condition means that a family history of reduced pin-prick in the lateral shin, as well as
excessive sleepiness may help diagnosis. Of weakness of ankle inversion (as it supplies
the tetrad components, sleep paralysis is the tibialis posterior).
one that is most likely to be physiological, and The ankle jerk depends on contraction of
isolated sleep paralysis should not in itself soleus and gastrocnemius, which are supplied
trigger invasive or prolonged testing. by tibial nerve and S1 root.
Hypnic jerks on falling asleep are a normal
phenomenon. The important treatable cause of 25.34. Answer: A.
daytime sleepiness is lack of night-time sleep A lesion of the spinal cord is likely to affect
- exclusion of obstructive sleep apnoea may be both legs equally in provoking upper motor
required. neuron signs, leading to a spastic paraparesis
with increased tone, increased reflexes and
25.31. Answer: E. extensor plantar reflexes.
A raised neutrophil count in CSF is strongly Selective or particular difficulty with heel-toe
associated with a bacterial infection, although walking results from cerebellar dysfunction,
the very earliest stages of a viral encephalitis while a high-stepping gait and slapping gait will,
may cause neutrophils to rise. respectively, compensate for or result from
Infections by mycobacteria, viruses, or impaired ankle dorsiflexion (i.e. a foot drop),
partially treated bacterial meningitis may be which results most usually from a common
associated with lymphocytic CSF. Where there peroneal or radicular lesion.
has been a large subarachnoid haemorrhage, Unilateral circumduction is a sign of a
the irritant effect of blood on the meninges may unilateral upper motor neuron lesion, where the
cause a lymphocytic CSF, but would more leg has to drift sideways while walking to
likely have an abrupt onset and mild (or no) compensate for the partial plantar flexion
pyrexia. caused by increased tone in soleus and
gastrocnemius.
25.32. Answer: D.
Supply of the upper face is bilateral (from both 25.35. Answer: D.
cerebral cortices), so a unilateral facial The superior orbital fissure is formed by the
weakness caused by an upper motor lesion will cleft between the lesser and greater wings of
be modified by the residual supply from the the sphenoid bone. Lesions in this will affect
ipsilateral cortex. the structures that pass through, including
Hyperacusis and dysguesia (altered taste) cranial nerves Ill, IV and VI. which, if affected,
originate from lesion of the facial nerve and will cause diplopia on gaze to either side or on
would not be prominent with an upper motor downgaze. Compression of the orbital vein
neuron facial weakness. would lead to proptosis, and any effect on the

downloaded from www.medicalbr.com


,..
31 8 • NEUROLOGY

I
fifth nerve will cause sensory alteration over the (false-positive tests are very rare - the antibody
upper face. is very specific). Imaging his head will add
Pathology of the optic nerve will cause a nothing, as this is an autoimmune disease of
reduction in acuity in one eye, but this does not the neuromuscular junction. Antibodies to
pass through the superior orbital fissure. Any MuSK are rnuch less commonly found in
effect on acuity alongside some disturbance of myasthenia gravis, and never when the AChR
ocular motility would suggest pathology in the antibody is positive. Whilst he may have an
cavernous sinus. abnormal single-fibre EMG, the diagnosis is
already made with the antibody result, so the
25.36. Answer: D. EMG will add little or nothing; similarly, a
Labyrinthitis (also known as acute vestibular Tensilon test may well be positive, but
failure) presents with abrupt onset of vertigo adds nothing to what we already know.
that tends to be most severe for a few days, Myasthenia gravis is, however, associated with
severe enough to cause the patient to be thymic abnormalities and in older men
bed-bound. thymomas are not uncommon; hence, he
Meniere's disease is an idiopathic chronically requires imaging of his chest for this reason
recurring disorder involving episodic vertigo (either CT or MRI).
with tinnitus and a progressive deafness.
Benign paroxysmal positional vertigo can be 25.40. Answer: E.
precipitated by minor head injury and results in Whilst the differential on this limited history is
vertigo that is typically precipitated by specific wide, there are clues to suggest Lambert-
head positions (as in the Hallpike Test). This . Eaton myasthenic syndrome (LEMS). The
responds well in most cases to the Epley weakness is variable, in keeping with a
manoeuvre or more chronic rehabilitation. mysathenic syndrome, there are no sensory
features, and the dry rnouth suggests
25.37. Answer: E. autonomic involvement, which is common in
A lesion in the parietal region will cause a LEMS. LEMS may be paraneoplastic, and there
quadrantanopia - due to its effect on the are alarm bells for cancer, with weight loss,
superior fibres, the quadrantanopia will be in unwellness and haemoptysis (lung cancer is the
the contralateral inferior visual field. commonest malignancy seen with LEMS). The
Neglect will result frorn a parietal lesion but reflex uncertainty reflects the classic reflex
this is contralateral to the lesion. Reduced potentiation seen in LEMS, whereby reflexes
acuity results from a reduction in macular appear absent, but rnay return (potentiate) with
function and rnay be a manifestation of an optic exercise. The diagnosis is supported by the
neuropathy. presence of VGCC antibodies.
Diplopia results from a disturbance of ocular
motility - this is unlikely to be caused by a 25.41. Answer: E.
cortical abnormality. The symptoms conform to the distribution of
the ulnar nerve and, although carpal tunnel
25.38. Answer: G. syndrome (median nerve) is common in
1!'
'I This is a typical scenario for migraine without pregnancy, in this case the distribution
aura. The additional non-headache symptoms suggests ulnar not median. The most likely site
would not occur in tension-type headache, and of entrapment is at the elbow (as opposed to
neither temporal arteritis nor dissection behave the wrist in CTS).
in such a paroxysmal manner. (She is also too
young for temporal arteritis.) Although she gets 25.42. Answer: B.
occasional ptosis, the other autonomic features This is very suggestive of an extradural
of cluster are absent and the headache lasts haematoma, localising to the right side of his
too long; patients are usually very agitated with head. He is coning, and will not survive 6 hours
cluster and usually male. in ambulance. The immediate life-saving
procedure is a burr hole to evacuate the clot.
25.39. Answer: B.
The story of variable binocular diplopia is 25.43. Answer: G.
suggestive of myasthenia gravis, and the The sleep disturbance is very suggestive of an
positive AChR antibody confirms this REM sleep behavioural disturbance, now a

- downloaded from www.medicalbr.com


NEUROLOGY • 319

well-recognised pre-motor symptom of although usually there is a family history, which


Parkinson's disease (PD), often preceding the may be suppressed.
motor features by years. A tremor that
responds to alcohol and is mainly with action 25.47. Answer: C.
suggests essential tremor (ET; usual bilateral), This is a very typical Essential tremor (ET)
and whilst PD can be familial, the family history history- both arms involved, postural and
of a tremor would also fit ET better. Family kinetic components, autosomal dominant
history of learning disabilities suggests possible pattern of inheritance and an alcohol response
fragile X tremor ataxia syndromes, which can in some members (only about 50% note such a
manifest sometimes in women. response). Parkinson's tremor is more typically
asymmetrical, and at rest. It can be difficult to
25.44. Answer: B. distinguish an enhanced physiological tremor
There remains some controversy about when from a mild ET, as they look similar, although
best to start treatment in PD, although there is the other features help (family history, alcohol
consensus that presently we have no proven responsiveness). Whilst sensible to check his
disease-modifying therapies. Anticholinergics thyroid status, it is unlikely to explain a 10-year
are no longer favoured due to their adverse history.
effect profile and poor efficacy. Whilst the
response to dopaminergic therapies becomes 25.48. Answer: D.
both attenuated and complicated as PD This is a typical story for TGA, with a profound
progresses, people do not become 'immune' anterograde amnesia lasting several hours
to it. Whilst DBS is a very effective treatment leading to repetitive (and irritating) questions,
for tremor (where drugs often fail), few would and retrograde amnesia stretching back at least
advocate this approach prior to a trial of 2 years, but not so long that she had forgotten
medication. Most would recommend her friends or husband. Psychogenic amnesia
dopaminergic therapy sooner or later, and in often involves loss of self-identity (functional
the UK we would be inclined to wait until his fugue state); the post-ictal state is usually
symptoms trouble him, although there is confusion, rather than this very specific isolated
greater enthusiasm for earlier treatment amnestic syndrome. Isolated amnesia is almost
elsewhere. never due to a TIA, and Alzheimer's presents in
a much more insidious way.
25.45. Answer: C.
Parkinson's disease is a clinical diagnosis, and 25.49. Answer: C.
tests are rarely helpful. This is unlikely to be In general, people who worry about their
Wilson's disease (caeruloplasrnin) with this age memory, which no one else has noticed, rarely
at presentation, and the scenario is not have an underlying disease; clinicians should
suggestive of a genetic cause (in any case, one worry much more about the family who bring a
would need to undertake genetic counselling patient who seems blithely unaware of any
first before any genetic testing). Structural problem. Minimal cognitive impairment (MCI) is
imaging is rarely indicated in a typical story a controversial entity, although some will
such as this, and, similarly, functional imaging progress to dementia. Depression can present
with either SPECT or positron emission with a pseudo-dementia, but there are no
tomography (PET} is unnecessary when the specific features of depression here. Sleep
diagnosis is clear clinically. apnoea can disturb memory but is usually
associated with excessive daytime sleepiness,
25.46. Answer: A. and a sleep history of snoring and apnoeic
This is likely to be secondary to long-term spells.
metoclopramide use, even though many
doctors and patients think of it as an innocuous 25.50. Answer: E.
drug. Chorea can occur with stroke, but is He has developed acute hemiballism, which
usually unilateral and acute. Such dyskinesias usually localises to the contralateral subthalamic
may complicate Parkinson's disease when nucleus in the basal gangUa. In this case, it is
treated with levodopa, but are not a presenting almost certainly due to a stroke, but is often
feature in patients not on treatment. not recognised as such as the symptoms are
Huntington's disease can present this late, unusual and may be missed by inexperienced

downloaded from www.medicalbr.com


320 • NEUROLOGY

clinicians. Lesions in the motor strip would


cause weakness, and lesions affecting the
(fAGs). The pain is always severe, lasting
between 30 and 180 minutes, associated with
"
i
I
!

angular gyrus in the dominant parietal lobe are autonomic activation and agitation. Cluster
associated with Gerstmann's syndrome headaches typically awaken people from sleep,
(agraphia, acalculia, finger agnosia and inability clusters last weeks, with months to years of
to differentiate left from right). remission in between. They are more common
in male smokers. Migraine can awaken people
25.51. Answer: B. from sleep, but usually patients want- to lie
The symptoms and signs suggest a quietly in a dark room, the opposite of cluster
pseudobulbar palsy, but the progression over patients, and autonomic activation is rare.
several months excludes a stroke; a structural Hypnic headache also awakens people from
lesion could potentially cause this, but not in the sleep, but usually affects older women, and is·
frontal region. Polymyositis may affect not associated with agitation or autonomic
swallowing but not speech, and would not activation. Temporal arteritis does not occur
cause these signs or emotional incontinence. under the age of 50 years and does not
Whilst myasthenia gravis can present with produce such a paroxysmal history. Paroxysmal
bulbar symptoms, the upper motor neuron hemicrania is another form of TAG, but the
signs and emotionalism do not fit. Unfortunately, symptoms are much shorter and affect women
this sounds very likely to be a pseudobulbar more commonly.
presentation of motor neuron disease.
25.55. Answer: E.
25.52. Answer: D. This is a typical story for benign paroxysmal
Disturbance of sense of smell (and taste, which positional vertigo (BPPV), the clues being the
is crucially dependent upon smell) is common short-lasting vertigo induced by changes in
after minor head injury, most typically to the posture, typically in bed. About half of cases
occipital region, as the shearing forces cause are triggered by minor head trauma (there is no
disruption to the olfactory fibres as they pass indication for brain imaging). Treatment with an
through the cribriform plate in the anterior cranial Epley manoeuvre or similar is easy (there are
fossa. (Patients are often mystified as to why a plenty of examples on You Tube!) and highly
bang to the back of their head might affect their likely to be successful, unlike drug treatment.
nose.) It would be an unusual malingering Although he should be advised to reduce his
symptom, and malingering is a forensic rather alcohol intake, which might perhaps have
than medical diagnosis. Parkinson's disease is explained the initial accident, this is not directly
often preceded by hyposmia, although patients an alcohol-related problem.
rarely, if ever, present at this stage. Whilst
smokers often have less acute senses of smell 25.56. Answer: A.
and taste, they rarely notice this. For most The story of an episode of optic neuritis,
patients presenting with reduced sense of smell followed by a spinal cord syndrome, with an
and no apparent triggers, the causes are either extensive longitudinal inflammatory lesion in the
ENT related or idiopathic. spinal cord is very suggestive of neuromyelitis
optica (NMO), which is commonly associated
25.53. Answer: c. with the aquaporin-4 antibody. This does not
The scan confirms that she is retaining urine, sound like a paraneoplastic syndrome, and
with incomplete bladder emptying. Thus the whilst the other tests may add further
optimal treatment would be regular intermittent information, they are unlikely to be diagnostic.
self-catheterisation, providing that her arm/hand NMO is different from MS, and requires a
function is not compromised by her MS. different approach to treatment. Indeed some
Antibiotics would not affect her bladder MS treatments can make NMO worse, so
function, and anticholinergic drugs would distinction is important.
exacerbate the problem. A long-term catheter
would ideally be avoided. 25.57. Answer: B.
This is a non-disabling relapse, and thus there
25.54. Answer: A. is no immediate indication for treatment,
This is a typical scenario for cluster headache, although it should trigger reconsideration of
one of the trigeminal autonomic cephalalgias disease-modifying drugs. Although infection can

downloaded from www.medicalbr.com


NEUROLOGY • 321

trigger relapses, you should only treat a proven, 25.61. Answer: A.


symptomatic infection. Physiotherapy will not An acute onset of vertigo that begins to resolve
help sensory symptoms. over days is likely to be related to an acute
vestibular syndrome.
25.58. Answer: C. BPPV is a chronic condition precipitated
The risk of developing MS is increased by usually by a specific movement in each
10- to 25-fold in first -degree relatives of people individual, and would merit treatment with the
with MS, but varies depending upon the Epley manoeuvre. A brainstem stroke would
kinship. The highest risk is in female usually cause more widespread neurological
monozygotic (MZ) twins (in male MZ twins it is changes, while Meniere's disease is a chronic
about a 6% risk of occurring). condition causing vertigo with associated
deafness and tinnitus.
25.59. Answer: C. Vertigo associated with migraine is a
Of course in an adult with new onset seizures recurring condition of relatively short -lived
imaging is the important investigation, but EEG vertigo most usually associated with headache
can have a role in some cases. In someone and/or other migrainous symptoms.
with a single seizure, a timely EEG (within 4
weeks) can help inform the risk of recurrence, 25.62. Answer: C.
and so will be worthwhile after a single The coexistence of headache and papilloedema
generalised tonic-clonic seizure at any age. will always merit imaging to exclude an
In patients under the age of about 30 years intracranial lesion, but the patient's age and
with new-onset epilepsy (either multiple morphology, intermittent nature of the
seizures or single seizure with high risk of symptoms and lack of other findings would
recurrence), the EEG can help with make IIH most likely. Transient loss of vision on
classification of epilepsy and so will carry bending (or other manoeuvres that transiently
therapeutic implications. raise intracranial pressure; ICP) are
In elderly patients with multiple seizures, characteristic, and called visual obscurations.
epilepsy is almost certainly going to have a Optic neuropathy would cause disc pallor and
focal origin and the EEG is unlikely to be useful. reduced colour vision, while retinopathy would
In anyone of any age with unwitnessed or cause field defects or, if affecting the maculae,
indeterminate transient loss of consciousness, reduced visual acuity. Neuromyelitis optica is an
the inter-ictal EEG may not only fail to show inflammatory condition causing neurological
abnormalities but also any resultant deficit but not headache.
'abnormalities' may be red herrings; this applies
especially to patients with focal injury or to 25.63. Answer: E.
younger females who have a higher chance of The aqueduct of Sylvius is the small channel
displaying epileptiform features on EEG such as that allows CSF to travel from the third to the
photosensitivity. fourth ventricle. Stenosis can become apparent
in adult life and lead to symptoms of raised ICP.
25.60. Answer: B. Imaging will show that the ventricles 'upstream'
ClOP is a condition that ca1:1ses loss of myelin (lateral and third ventricles) will be dilated, but
in peripheral nerves. Nerve conduction studies the fourth will be small or of normal size.
can demonstrate demyelination of peripheral IIH is associated with normal or small
nerves (via slowed conduction) or axonal ventricular size, while space-occupying lesions
damage to sensory or motor nerves (with will be apparent on imaging, and if severe
reduced numbers of functioning axons leading enough to cause raised intracranial pressure
to reduced amplitude of response). EMG will often cause other neurological deficits.
shows spontaneous activity in muscle Venous sinus thrombosis will usually be
(fasciculation or positive sharp waves) when apparent on imaging (often resulting in
nerve supply to muscle is lost due to axonal haemorrhage). Normal pressure hydrocephalus
damage, but not as a consequence of usually occurs in older patients, and will have
demyelination. The peripheral demyelination no features of acutely rai:'\_ed ICP, but rather a
therefore would be expected to cause only triad of reduced cognition, urinary incontinence
abnormal nerve conduction velocities and not and gait abnormalities, resulting in dilation of all
EMG changes. ventricles apparent on imaging.

downloaded from www.medicalbr.com


322 • NEUROLOGY

25.64. Answer: A. cause for her seizure. The event at the age of
Chronic Daily Headache (sometimes known 16 is removed enough to have little relevance,
as medication overuse headache) is an while the borderline hyponatraemia is not
increasingly common condition, made worse by severe enough to cause seizures. A neutrophilia
ease of access to paracetamol and compound is common after a seizure. While an EEG may
analgesics. The unrelenting nature of the slowly provide some prognostic information, the most
progressive pain with no neurological or important role of investigation is to exclude a
systemic features and associated high intake of primary intracerebral lesion.
analgesia will give good clues to the diagnosis.
While migraine headache syndromes can 25.68. Answer: A.
transform with time, many of this patient's People may develop a constellation of
painful episodes have no other migrainous symptoms after head injury, including
features. It would be an unusual person who headache, fatigue, dizziness, poor
was not rendered weepy or low by such severe concentration/memory, emotionalism and
headaches, and the concurrence of a mood numerous other symptoms. These are not
disorder should not allow the physician to specific to head injury. They are often persistent
make a hasty attribution of symptoms to and may get worse, especially if the diagnosis
psychological causes, particularly in the is not explained. The management requires a
presence of other diagnostic features. careful explanation of the diagnosis, as well as
Subarachnoid haemorrhage would cause an reassurance that they have not suffered any
abrupt -onset acute headache rather than a irreversible brain damage (http://www
relentless one. .headinjurysymptoms.orgl). Unfortunately, in
such scenarios, many (well-meaning) health-
25.65. Answer: C. care workers and other professionals may
The episodic ataxias are inherited exacerbate the situation by recommending
channelopathies that result in prolonged more intervention, as in this case. Neither a
paroxysms of ataxia in affected individuals, psychiatric nor neurosurgical consult will be of
usually with normal intervening neurological any value, and tramadol is a poor choice in this
examination (although some patients can situation.
develop a slowly progressive ataxia). The family
history is key in this case, suggesting an 25.69. Answer: E.
autosomal dominant disorder. The history is of a progressive degenerative
Peripheral neuropathies are unlikely to be disorder and, in this age, with evolution of
paroxysmal and would not cause an ataxia, choreiform movements, variant CJD would be
while migraine can cause episodic vertigo, but most likely. Myoclonus tends to be more
usually alongside other migrainous phenomena. prominent in sporadic CJD, but this involves
BPPV would usually have direct positional an older population, as does Alzheimer's
precipitants. disease (which has a slower course and
only shows neurological signs at a late
25.66. Answer: C. stage).
Cerebellar function can be acutely affected by a The presence of significant cognitive decline
number of medications including the older and upgoing plantars would not be in keeping
antiepileptic drugs (phenytoin, carbamazepine with a psychological cause. While age
and valproate), lithium and amiodarone. and gender would be in keeping with
Digoxin and diazepam at higher doses can hyperthyroidism, the presence of neurological
be associated with sedation and drowsiness signs and severity of the cognitive dysfunction
but not cerebellar ataxia. Antiemetics and would make this less likely.
antipsychotics can be associated with
movement disorders such as chorea and 25. 70. Answer: C.
athetosis but not usually ataxia; ~ 2 -agonists will The only localising features here are the
cause tremor in acute stage. symptoms of left sensory change and the
quadrantanopia, wf.lich would both suggest a
25.67. Answer: A. right parietal lesion. Seizures are more likely
At this age, the key investigation is brain with low-grade gliomas, while highly malignant
imaging (ideally MRI) to exclude a structural lesions such as glioblastomas will often have a

downloaded from www.medicalbr.com


NEUROLOGY • 323

rapid onset of neurological symptoms with, for botulinum toxin from Clostridium botulinum.
reasons that are not entirely clear, a lower risk (Her cousin had obviously eaten the same
of seizures than low-grade lesions. poorly prepared food!)
Meningiomata, by definition, are situated Brainstem stroke would not arrive in such a
outside the brain, while medulloblastomas are progressive manner, and Miller Fisher syndrome
tumours more common in childhood, most would cause ataxia and areflexia along with any
likely to be situated in the posterior fossa ophthalmoplegia (nor is it contagious).
(cerebellum). Visual changes related to optic Myasthenia gravis would have an onset with
nerve problems will be monocular rather than some fatigability. Multiple sclerosis would be
homonymous. unlikely to cause isolated weakness in such a
progressive manner (although this can be
25.71. Answer: C. increased in family members, the simultaneous
Migraine is a common disorder, and preceding onset is a clue to a recent infection as the
visual symptoms that disappear with onset of cause).
headache are characteristic of migraine with
aura. Somatic sensations and dysarthria are 25.74. Answer: E.
common with migraine, but on a first Carotid dissection can be precipitated by a
occurrence would justify imaging to exclude a surprisingly minor trauma. Association of
primary structural cause. unilateral pain with Horner's syndrome is
Subarachnoid haemorrhage will not usually characteristic of this disorder.
have focal signs but may need excluding in Signs are too focal and would be expected
abrupt-onset headache as a first or worst to be more severe with subarachnoid
occurrence. TIAs are not usually associated haemorrhage. Subdural and extradural
with headache, while the prominent headache haematomas would be unlikely to cause an
and widespread other symptoms would not be isolated Horner's syndrome. A brachial
in keeping with focal seizure. Cerebral venous plexopathy would also be expected to
sinus thrombosis would not usually recover cause symptoms and signs in the
completely so quickly, and any focal signs or ipsilateral arm.
symptoms would tend to evolve with the
headache rather than precede it. 25.75. Answer: A.
Acoustic neuromas may be discovered
25.72. Answer: E. incidentally on imaging or on investigation of
The occurrence of jaw stiffness in advance of sensorineural deafness. If uncovered late, there
generalised stiffness and spasms would be may be compression of the brainstem,
characteristic of tetanus, in this case most sometimes with compression of the fourth
likely as the result of an infected wound. ventricle causing hydrocephalus and
Diagnosis and adequate treatment is raised ICP.
paramount as this disease can still be fatal Brainstem stroke would have a much more
even if treated. abrupt onset, and deafness is a very rare
Dental abscesses are usually very painful, feature of multiple sclerosis. Migraine would
and would be vanishingly unlikely to extend have a more episodic course, with no
intracranially. While the anxiety may make neurological deficit (and normal fundi). Most
doctors think of a functional illness, the acoustic neuromas are spontaneous and not
characteristic pattern of intermittent jaw related to neurofibromatosis.
symptoms followed by generalised symptoms
would be unlikely in functional disorders. 25.76. Answer: E.
Botulism is also caused by a bacterial toxin The symptoms of a rapidly progressive distal
(from Clostridium botulinum) but more usually sensorimotor loss would be most in keeping
causes ocular and bulbar weakness rather than with a neuropathy. The levels of alcohol intake
spasms. and random glucose are too modest to
account for an alcoholic or diabetic neuropathy,
25.73. Answer: A. respectively. A raised ES8 would highlight an
The rapidly progressive generalised weakness immune-related cause and, with no rash or
preceded by ocular and bulbar paralysis is arthropathy, such a raised level would be most
characteristic of weakness caused by in keeping with myeloma.

downloaded from www.medicalbr.com


324 • NEUROLOGY

There are no features to suggest a 25.78. Answer: B.


myelopathy (no upper motor neuron symptoms Spinal cord problems will often cause motor,
or sphincter deficit) and the presence of sensory and sphincter problems. Any sensory
sensory symptoms and signs would not be problems caused by spinal cord lesions may
suggestive of motor neuron disease. relate to dissociated sensory loss, which may
be asymmetrical if only half of the cord is
25.77. Answer: B. affected. Motor deficit caused by spinal cord
The symptoms and signs suggest a lower problems are upper motor neuron lesion in
motor neuron facial palsy on the affected side. character below the lesion and may be lower
Recurrent facial palsy in someone at high risk motor neuron character at the level of the
of Borrelia burgdorferi infection makes Lyme lesion, rather than a widespread mixture of
disease the likely cause here. The onset and upper motor neuron and lower motor neuron
relapse would be unusual for a vascular cause, signs.
and multiple sclerosis causes upper motor Distal sensory loss is usually caused by a
neuron problems, being unlikely to cause an peripheral neuropathy, and cranial nerve deficits
isolated facial palsy. Syphilis can cause some (e.g. diplopia) will require involvement superior
vasculitic central nervous system problems, but to the spinal cord.
the typical pattern would involve a myelopathy Timing of the evolution of the symptoms
and some brainstem signs. Even with the tends to give information on the nature rather
recent rise in incidence, it remains rarer than than the site of the lesion.
that other spirochete, Borrelia.

II
'
.,(.1

downloaded from www.medicalbr.com


P Langhorne

Stroke medicine

Multiple Choice Questions


26.1. A patient is referred by his doctor to the 26.4. Which of the following ischaemic stroke
hospital stroke service. Which of the following patients who have undergone carotid duplex
transient symptoms would allow you to make scanning would be most likely to benefit from
a clinical diagnosis of transient ischaemic left carotid endarterectomy?
attack (TlA)? A. Left cerebellar hemisphere infarct with good
A. Dizziness functional recovery, 80% left carotid artery
B. Expressive dysphasia stenosis
C. Loss of consciousness B. Left middle cerebral artery territory infarct
D. Slurred speech (dysarthria) with good functional recovery, 60% left
E. Transient confusion carotid artery stenosis
C. Left middle cerebral artery territory infarct
with good functional recovery, 80% left
26.2. A 70 year old man is admitted overnight carotid artery stenosis
with a 24-hour history of left hemiparesis. D. Left middle cerebral artery territory infarct
Which one of these drug therapies would with persistent dense right hemiparesis and
mandate urgent computed tomography (CT) profound dysphasia, 90% left carotid artery
head scanning? stenosis
A. Aspirin E. Right middle cerebral artery territory infarct
B. Clopidogrel with good functional recovery, 75% left
C. Ramipril carotid artery stenosis
D. Sirnvastatin
E. Warfarin 26.5. A 73 year old man presents with a right
hemiparesis and expressive dysphasia
secondary to an infarct in the territory of the left
26.3. A 43 year old woman attends the hospital middle cerebral artery. Blood pressure is
accident and emergency department with a 153/82 mmHg, serum cholesterol is
1-hour history of sudden-onset severe occipital 4.4 mmol/L (170 mg/dL) and the
headache. On examination she is photophobic electrocardiogram (EGG) shows sinus rhythm
with neck stiffness. What is the most with no abnormalities. Which of the following
appropriate first -line investigation? secondary prevention medications is usually
A. Carotid Doppler ultrasound (duplex) avoided in such a patient?
scan A. Amlodipine
B. CT cerebral angiography B. Aspirin
C. CT head scan C. Ramipril
D. Lumbar puncture D. Simvastatin
E. Skull X-ray E. Warfarin

downloaded from www.medicalbr.com


T'
. 326 • STROKE MEDICINE I
I
26.6. The ROSIER (Rule Out Stroke In the A. It can be offered to most stroke patients
Emergency Room) clinical stroke tool can be B. It is effective in intracerebral haemorrhage
used to triage patients with clinical suspicion of C. It is effective in large-vessel occlusion
stroke. Which of the following features are D. It requires less technological support than
given a negative score on the ROSIER scale intravenous thrombolysis
(i.e. are not thought to be consistent with a E. It is a treatment that is widely available
clinical diagnosis of stroke)?
A. Leg weakness 26.11. A 38 year old man is brought to·the
B. Loss of speech emergency department with a suspected
C. Seizure intracerebral haemorrhage. Which of the
D. Unilateral arm weakness following are recognised risk factors for
E. Visual field defect intracerebral haemorrhage?
A. Antiphospholipid abnormality
26.7. A patient is admitted with a clinical picture B. Cardiac embolism
of acute stroke. You request a plain CT scan C. Carotid artery stenosis
as initial emergency brain imaging. What D. Cocaine use
information can you get from plain E. Raised cholesterol
(non-contrast) CT brain scanning in acute
stroke patients? 26.12. A 70 year old man with recent minor
stroke is found to be in atrial fibrillation. When
A. Distinguishes acute stroke from TIA
advising him on anticoagulant therapy, which of
B. Reliably detects intracerebral blood
t~e following features would make you favour
C. Reliably detects subtle acute ischaemic
warfarin over a direct oral anticoagulant
changes
(DOAC)?
D. Shows brain function (functional imaging)
E. Shows blood flow in vessels A. Fewer drug interactions
B. Lower drug costs
C. Lower risk of intracerebral haemorrhage
26.8. A 65 year old woman with a previous
D. More effective at preventing embolism
history of diabetes and breast carcinoma is
E. Simpler dosing regimes
found collapsed at home with drowsiness and
a left hemiparesis. Which of the following
26.13. A 45 year old woman is admitted to
should be carried out first?
hospital with symptoms of raised intracranial
A. Check blood glucose level pressure, seizures and focal neurological
B. Check temperature symptoms. Which of the following is correct
C. Clarify breast carcinoma history about the suspected diagnosis of cerebral vein
D. Examine for peripheral neuropathy thrombosis?
E. Examine for symmetrical plantar responses
A. CT brain scanning is the definitive
imaging
26.9. An 83 year old woman is brought to the B. It can include an associated haemorrhage
emergency department after becoming unwell C. It is never caused by infection
at home. Which of the following is true of total
D. It is rarely treated with anticoagulation
anterior circulation stroke? E. It usually presents like arterial stroke
A. It is caused by occlusion of small perforating
arteries 26.14. A 62 year old man with a stroke is being
B. It includes higher cerebral dysfunction and considered for thrombolysis therapy. Which of
motor loss the following is true of intravenous thrombolysis
C. It involves isolated homonymous hemianopia with recombinant tissue plasminogen activator
D. It is not caused by cerebral embolism (rt-PA)?
E. It is a pure motor stroke A. It can be given up to 12 hours after
symptom onset
26.10. A patient with an acute stroke is B. It can be offered to most stroke
admitted to the specialist stroke unit. Which of patients
the following currently apply to mechanical clot C. It improves the chance of recovery of
retrieval (thrombectomy)? independence

downloaded from www.medicalbr.com


STROKE MEDICINE • 327

o. It reduces the risk of early death advise best medical therapy rather than carotid
E. It reduces the risk of early intracerebral endarterectomy?
haemorrhage A. Her age - she is too old to benefit
B. She has a history of diabetes
26.15. An 81 year old woman with diabetes, G. The carotid stenosis is on the asymptomatic
hypertension and a minor left hemisphere side
ischaemic stroke 1 week ago is found to have D. The stroke impact is only minor
a right carotid artery stenosis of 70%. Which of E. There has been too long a delay since h·er
the following features would cause you to stroke onset .

Answers .
26.1. Answer: B. haemorrhage will have a normal CT scan; in
The definition of a TIA is the rapid onset of a these cases, a lumbar puncture should be
focal neurological deficit, of presumed vascular performed 12 hours following the onset of
origin, that resolves within 24 hours. It also headache to look for xanthochromia
includes transient monocular blindness due to (breakdown products of red blood cells).
vascular occlusion in the retina (amaurosis
fugax). Dysphasia is caused by a deficit in the 26.4. Answer: G.
dominant cerebral hemisphere. Delirium, Carotid endarterectomy reduces the risk of
dizziness, lone dysarthria and loss of stroke in patients with severe stenosis of the
consciousness are not focal deficits when internal carotid artery but carries a significant
present on their own. Loss of consciousness risk of perioperative mortality and stroke.
can very rarely be caused by basilar ischaemia Decisions on whether to operate must,
but is rarely transient. As a general rule, the therefore, be based on a careful benefit/risk
diagnosis of TIA should not be made in patients analysis. The absolute reduction in risk of future
who present with episodes of syncope, stroke is greatest for symptomatic patients with
dizziness or delirium, as these do not reflect 70-99% stenosis and, in general, outweighs
focal cerebral dysfunction. the risk of surgical complications. Importantly,
symptomatic patients are defined as patients
26.2. Answer: E. with a TIA or non-disabling stroke in the
Early CT scanning is the ideal for all patients territory of the carotid artery on the same side
with suspected stroke to help plan acute
therapies and plan secondary prevention. This
as the stenosis in the preceding 6 months. The
evidence to support surgery in symptomatic
~
man is beyond the time window for acute patients with moderate (50-69%) stenosis and I
therapies. Patients who are anticoagulated with
warfarin or who have a non-iatrogenic
asymptomatic patients with severe stenosis is
less conclusive, as these patients have a I
coagulopathy require urgent imaging of the smaller benefit/risk ratio than patients with
brain to rule out the possibility of an severe symptomatic stenosis. Patients with
intracerebral haemorrhage. stenosis of less than 50% do not benefit from
carotid endarterectomy, irrespective of
26.3. Answer: G. symptoms. Finally, a patient with severe
The clinical signs and symptoms in this patient residual disability would Qain little benefit from
are suggestive of a subarachnoid haemorrhage. preventing a further stroke within the same
An emergency head CT scan is essential. territory and may have a greater risk of surgical
About 5-10% of patients with a subarachnoid complications.

downloaded from www.medicalbr.com


328 • STROKE MEDICINE

Stroke CT brain scan within


or atypical or multiple 24 hours of onset;
cerebral TIAs MRI if later than 7 days

Single
typical TIA
lschaemic Haemorrhagic I

ECG Sinus rhythm Carotid duplex

Refer if> 70%


stenosis on
symptomatic side

Antiplatelet drugs4 LowerBP 1


• Aspirin 300 mg at once if BP > 130/70 mmHg
then 75 mg daily 1-2 weeks after onset
• Clopidogrel 75 mg daily • Thiazide diuretic
is effective alternative J:o~~k1i(i~~Eirola • ACE inhibitor
if foh¥J:cli01esler6l (check U&Es)
>-3.5mmo!IL • Other agents
(135 mgfdL) with
If contraindications s.rmvastatln 40 mg
Consider to anticoagulation, nocte, After checking
• Cardioversion e.g. bleeding, falls, liverftmotkm tests
• Anti-arrhythmic binge drinking,
poor adherence Lifestyle
• Smoking cessation
• Lower salt intake
• Lower fat intake
• Lower excess
alcohol intake
• Increase exercise
• Lose excess weight

Fig. 26.5 Strategies for secondary prevention of stroke. (1) Lower blood pressure with caution in patients with postural hypotension,
renal impairment or bilateral carotid stenosis. (2) Other statins can be used as an alternative to sirnvastatin in patients on warfarin or
digoxin. (3) Warfarin and aspirin have been used in combination in patients with prosthetic heart valves. (4) The combination of aspirin
and clopidogrel is indicated only in patients with unstable angina or those with a temporary high risk of recurrence (e.g. carotid stenosis).
(ACE= angiotensin-converting enzyme; BP =blood pressure; CT =computed tomography; EGG= electrocardiogram; INR = international
normalised ratio; MRI =magnetic resonance imaging; TIA =transient ischaemic attack; U&Es =urea and electrolytes)

I
f 26.5. Answer: E. mimic of stroke. For that reason, it scores -1
In addition to lifestyle modifications, anti platelet, on the ROSIER clinical stroke tool (Box 26.6),
lipid-lowering and antihypertensive therapy form as does loss of consciousness, whereas all the
the cornerstone of secondary prevention for other options score +1 .
most patients with an ischaemic stroke (Fig.
26.5). Recent large-scale randomised trials have 26.7. Answer: B.
demonstrated the benefit of statins and The main advantages of plain CT scanning are
antihypertensive medication in these patients, its speed and tolerability and it can rapidly
even with blood pressure and cholesterol levels detect intracranial bleeding plus some stroke
within the 'normal' range. Patients in atrial mimics. Magnetic resonance imaging (MRI) is
fibrillation benefit from anticoagulation with often needed to show more subtle ischaemia,
warfarin following ischaemic stroke, but there is while MR angiography or CT angiography are
no such benefit in those who are in sinus rhythm. usually required to show vessel occlusion.

26.6. Answer: C. 26.8. Answer: A.


Although seizure does occur in <5% of acute Hypoglycaemia can mimic stroke, is a medical
stroke patients, seizure (with post-seizure emergency and is easily corrected. Although
paresis) is more commonly recognised as a we do not know what drugs this patient takes,

downloaded from www.medicalbr.com


STROKE MEDICINE • 329

~,

26.11. Answer: D.
26.6 Rapid assessment of suspected stroke
The main recognised risk factors for
ROSIER scale
intracerebral haemorrhage include high blood
can be used by emergency staff to indicate probability of
pressure, smoking, excess alcohol intake,
a stroke in acute presentations:
structural abnormalities, coagulopathies and
Unilateral facial weakness +1
drugs such as cocaine and amphetamines.
Unilateral grip weakness +1
Unilateral arm weakness
Raised cholesterol, antiphospholipid
+1
Unilateral leg weakness +1
abnormality, cardiac embolism and carotid
Speech loss +1 artery stenosis are recognised risk factors for
Visual field defect +1 ischaemic stroke.
Loss of consciousness -1
Seizure -1 26.12. Answer: B.
Total (-2 to +6); score of> 0 indicates stroke is Warfarin is a less expensive drug option but
possible cause does require regular monitoring. Also, at
Exclusion of hypoglycaemia present we cannot easily monitor and reverse
Bedside blood glucose testing with BMstix
anticoagulation levels with DOACs (although
Language deficit
new agents are being developed). DOACs have
History and examination may indicate a language deficit
simpler dosing regimes with fewer drug
Check comprehension ('lift your arms, close your eyes') to
identity a receptive dysphasia interactions and appear to have a better
Ask patient to name people/objects (e.g. nurse, watch, balance of effectiveness and safety than
pen) to identify a nominal dysphasia warfarin.
Check articulation (ask patient to repeat phrases after you)
for dysarthria
Motor deficit
26.13. Answer: B.
Subtle pyramidal signs: Cerebral venous sinus thrombosis usually
Check for pronator drift: ask patient to hold out arms and presents with symptoms of raised intracranial
maintain their position with eyes closed pressure, seizures and focal neurological
Check for clumsiness of fine finger movements symptoms. It often includes associated
Sensory and visual inattention
haemorrhage. MR venography demonstrates a
Establish that sensation/visual field is intact on testing one
filling defect in the affected vessel. About 10%
side at a time
Retest sensation/visual fields on simultaneous testing of of cerebral venous sinus thrombosis is
both sides; the affected side will no longer be feiVseen associated with infection requiring antibiotic
Perform clock drawing test treatment. Otherwise, the treatment of choice is
Truncal ataxia usually anticoagulation.
Check if patient can sit up or stand without support
26.14. Answer: C.
Intravenous thrombolysis with rt-PA increases
hypoglycaemia must be excluded. While the risk of early haemorrhagic transformation of
important, the other actions do not take priority the cerebral infarct with potentially fatal results.
over checking the blood glucose. However, if it is given within 4.5 hours of
symptom onset to carefully selected patients
26.9. Answer: B. (about 20% of ischaemic stroke patients), the
Classification of a stroke is helpful for both haemorrhagic risk is more than offset by an
clinical and research purposes. A total anterior improvement in overall outcome The earlier the
circulation stroke results in a mix of motor treatment is given, the greater the benefit.
deficit, higher cerebral dysfunction and
homonymous hemianopia caused by occlusion 26.15. Answer: C.
of a major cerebral artery. An embolic cause is To benefit from carotid artery surgery, the
often found. patient needs to have an expectation of several
years of reasonable quality of life to offset the
26.10. Answer: C. risks of surgery. The stroke impact being minor
Mechanical clot retrieval (thrombectomy) and her age and other risk factors would not
appears to be particularly effective in cerebral influence this decision. The key contraindication
ischaemia caused by large-vessel occlusion. is that the carotid artery stenosis is on the
However, it requires careful patien~selection opposite side from the patient's symptoms and
and considerable support from imaging so this is a lower-risk asymptomatic carotid
investigations and catheter laboratories. stenosis.

l downloaded from www.medicalbr.com


R Darbyshire, J Olson

Medical ophthalmology
Multiple Choice Questions
27.1. A 23 year old male presents to the 27.4. A 53 year old man attends his family
emergency department following an alleged physician for ongoing neck pain, which has
assault. He is intoxicated, his nose is bleeding occurred since he was involved in a road traffic
and he has a large left periorbital haematoma accident 6 months ago. During the consultation
that prevents spontaneous eyelid opening. his wife mentions his left eyelid is drooping. On
Alongside assessment for traumatic brain injury, . examination, the pupil on this side is 1-2 mm
which of the following ocular conditions is it smaller. Which is the most appropriate
most important to exclude? investigation?
A. Hyphaema A. Chest X-ray
B. Medial orbital wall fracture B. Computed tomography (CT) angiogram of
C. Orbital floor fracture the aortic arch, carotid arteries and
D. Retinal detachment intracranial vessels
E. Retrobulbar haemorrhage C. CT head
D. Doppler ultrasound of the carotid artery
27.2. A 36 year old male primary school teacher E. Magnetic resonance imaging (MRI) head
presents with a 3-day history of bilateral red,
watery, painful eyes. His vision is 6/7.5 in both 27.5. A 34 year old female is admitted with a
eyes. He is usually fit and well with no past life-threatening attack of asthma. After
ocular history. He mentions one of the children stabilisation she is transferred to the
in his class had a similar condition a week ago. intensive care unit where she remains
What is the most likely diagnosis? intubated and ventilated. The admitting doctor
A. Allergic conjunctivitis notices the left pupil, is dilated and minimally
B. Bacterial conjunctivitis responsive to light. There is no other
C. Episcleritis neurological abnormality. What is the most
D. Microbial keratitis likely cause?
E. Viral conjunctivitis A. An Adie's pupil
B. Argyll Robertson syndrome
27.3. An 18 year old female presents with a C. Horner's syndrome
24-hour history of a severely photophobic, D. Pharmacological mydriasis
watery and injected right eye. Her visual acuity E. Physiological anisocoria
is reduced to 6/18 in the affected eye. Which
feature of the clinical history will most affect 27.6. An 18 year old female has been referred
immediate management? following a routine visit to her optician, who
A. Contact lens wear noted anisocoria. Pupil measurements are as
B. Foreign travel follows:
C. Other unwell contacts The direct and consensual reflex in the left
D. Previous cold sores around the nose or mouth pupil is sluggish and the pupil constricts slowly
E. Previous ocular history in response to accommodation. There is no

downloaded from www.medicalbr.com


MEDICAL OPHTHALMOLOGY • 331

Right pupil Left pupil exacerbated by eye movement. Her visual


Conditions diameter (mm) diameter (mm) acuity is 6/24 in the right eye and 6/6 in
Light 3 7 the left. Examination of the right eye
Dark 7 8 reveals reduction in colour vision, a relative
afferent pupillary defect (RAPD) and the optic
nerve appears normal. Visual fields show
abnormality noted on examination of a central scotoma. What is this is a typical
extraocular movements and no evidence case of?
of ptosis. Slit-lamp examination reveals A. Demyelinating optic neuritis
vermiform movements at the pupillary B. Infectious optic neuritis
border in the left eye. What diagnosis is C. Leber's hereditary optic neuropathy
this consistent with? D. Neuroretinitis
A. Adie's pupil E. Non-arteritic anterior ischaemic optic
B. Episodic mydriasis neuropathy
C. Horner's syndrome
D. Pharmacological mydriasis 27.10. A 78 year old woman visits her
E. Physiological anisocoria family physician with a 3-week history of a
severe right-sided headache. The area
27.7. An anxious 53 year old with high over her right forehead and scalp is so
myopia has a 1-day history of new-onset tender she cannot bear to brush her hair
large central floater in her right eye and and is unable to sleep on that side. She
temporal photopsia, most noticeable in low reports numerous episodes where the vision
light conditions. Her visual acuity is 6/9 in in her right eye has become very blurred
the affected eye and visual fields are full to and then spontaneously recovered.
confrontation. Which investigation is considered the
What is the most likely diagnosis? definitive diagnostic test for this
condition?
A. Asteroid hyalosis
B. Posterior vitreous detachment A. C-reactive protein
C. Visual snow B. Erythrocyte sedimentation rate
D. Vitreous haemorrhage C. Plasma viscosity
E. Vitritis D. Temporal artery biopsy
E. Temporal artery Doppler ultrasound
27.8. A 73 year old male attends his family
physician having had cataract surgery 27.11. An 18 year old is referred from her
performed on his left eye 4 days ago. He was optician querying papilloedema. She has a
very pleased with the vision initially, but it 3-month history of headaches. All of the
seems to be worsening over the past 48 hours following are causes of papilloedema except
and he has noticed some n'ew floaters. The one. Which one is NOT a cause of
eye is becoming more painful. His visual papilloedema?
acuity is 6/60 and on his last clinic letter A. A space-occupying lesion
from ophthalmology it was 6/12. A B. Accelerated hypertension
same-day ophthalmological referral is C. Idiopathic intracranial hypertension
needed to exclude which of the following D. Obstructive hydrocephalus
conditions? E. Venous sinus thrombosis
A. Corneal oedema
B. Endophthalmitis 27.12. A 78 year old man attends for
C. Post -operative acute anterior uveitis consideration of cataract surgery,
D. Raised intraocular pressure describing gradual visual deterioration
E. Retinal detachment over the past few years. Past medical
history includes hypertension and diabetes.
27.9. A 34 year old woman presents with a He is also a smoker. Dilated fundal
1-week history of progressive blurring of examination is shown below. Which of the
central vision in the right eye, flashing lights following would have the greatest impact in
and an aching pain behind the eye managing his condition?

downloaded from www.medicalbr.com


't
332 • MEDICAL OPHTHALMOLOGY

'
She attends the diabetic retinal screening
programme for annual retinal photographs.
The latest image is show below.

A. Control of hypertension
B. lntravitreal anti-vascular endothelial growth
factor (anti-VEGF) therapy
G. Smoking cessation What is the earliest feature of diabetic
D. Tighter glycaemic control retinopathy visible on fundus fluorescein
E. Vitamin supplementation with high-dose angiography (FFA)?
antioxidants and zinc
A. Capillary occlusion
B. lntraretinal microvascular anomalies
27.13. A 69 year old male presents with a 2-day
G. Microaneurysms
history of sudden-onset, painless blurred vision
D. Venous beading
in his left eye. His visual acuity is 6/18 in the
E. Venous reduplication
affected eye. His past medical history includes
hypercholesterolaemia, chronic obstructive
27.15. A 31 year old patient with poorly
pulmonary disease (COPD), osteoarthritis and
controlled type 1 diabetes attends the eye
gastro-oesophageal reflux disease (GORD). He
casualty department complaining of blurred
has no past ocular history of note. The fundal
vision and floaters in the left eye. She manages
image is shown below.
her diabetes on a basal-bolus injection regime
with insulin Lantus and NovoRapid, but admits
her blood sugar levels have been high recently.
Her left fundus is shown in the image below.

What is the most common aetiology of this


condition?
A. Arteriosclerosis
B. Glaucoma Adequate treatment with which of the
G. Hyperviscosity following will induce a permanent regression of
D. Inflammation this condition?
E. Thrombophilia A. Diffuse macular laser
B. Focal macular Ieser
27.14. A 62 year old female was diagnosed with G. lntravitreal anti-VEGF therapy
type 2 diabetes mellitus 5 years ago. Her current D. Pan-retinal laser therapy (photocoagulation)
medications include metformin and sitagliptin. E. Photodynamic therapy (PDT) with verteporfin

downloaded from www.medicalbr.com


MEDICAL OPHTHALMOLOGY • 333

Answers
27.1. Answer: E. asthma attack. lpratropiurn is an antirnuscarinic
All of the above conditions may have occurred agent. This may therefore cause dilation of the
following the inciting injury. Retrobulbar pupil if vaporised drug leaks from the mask.
haemorrhage is a sight-threatening emergency. The effect may last up to 24 hours. The
Bleeding behind the globe, in the absence of diagnosis of a pharmacological mydriasis can
any decompressing fracture, raises intraorbital be confirmed if there is little or no pupillary
pressure, which irreversibly damages the optic constriction following instillation of 1%
nerve. Typical clinical features include: severe pilocarpine. The other answers would be less
pain, progressive proptosis, reducing visual likely given the timing and clinical
acuity, ophthalmoplegia, diplopia and an scenario.
unreactive pupil. Emergency decompression
surgery is required to preserve optic nerve 27.6. Answer: A.
function. An Adie's tonic pupil is caused by loss of the
parasympathetic innervation to the sphincter
27.2. Answer: E. pupillae muscle in the iris and ciliary body.
Viral conjunctivitis is most commonly caused by The direct and consensual pupillary light reflex
adenovirus, a non-enveloped double-stranded are sluggish. The pupil slowly constricts to
DNA virus. The clinical presentation varies from near focus. The syndrome typically occurs in
subclinical to severe inflammation. The young females after a viral illness. Application
condition is highly contagious. Classic features of dilute pilocarpine 0.125% (a muscarinic
include prominent conjunctival hyperaemia agonist agent) to both eyes has no effect in
and follicles, petechial haemorrhage and the normal eye but causes constriction of
pseudomembranes. Corneal involvement is the Adie's pupil due to denervation
characterised by epithelial microcysts, punctate hypersensitivity.
epithelial keratitis and focal subepithelial
infiltrates. 27.7. Answer: B.
With age, the vitreous progressively liquefies, a
27.3. Answer: A. process known as syneresis. When a break in
A presentation of red eye in a known contact the posterior hyaloid face occurs, escaping
lens wearer should prompt a same-day referral fluid separates the vitreous from the retina,
for ophthalmological assessment including causing a posterior vitreous detachment
slit-lamp biomicroscopy. The cornea must be (PVD). An impression where the posterior
examined in detail for any features of microbial hyaloid face was once attached to the optic
keratitis. nerve becomes visible to the patient as a
large central floater and to the clinician on
27.4. Answer: B. examination as a circular opacity or Weiss
This patient has Horner's syndrome. Horner's ring. PVD occurs earlier in myopia, collagen
syndrome is a triad of ptosis, miosis and and connective tissue disorders, and may be
anhydrosis of the affected side of the face and triggered by trauma or inflammation. PVD is a
neck. It can be caused by interruption of the risk factor for a retinal tear and subsequent

~I
sympathetic fibres at any point along their detachment. A thorough examination of the
protracted course from their origin in the retina is required.
posterior hypothalamus through to their
synapse in the superior cervical ganglion and 27.8. Answer: B.
then to the eye. This patient has a history of Although all of these complications are possible
significant trauma; therefore a dissection of the after cataract surgery, this is a presentation
internal carotid artery must be excluded by CT
or MR angiography in the first instance.
of endophthalrnitis until proven otherwise and
requires specialist review. The worrying features I
are the initial subjectively good vision, followed I
27.5. Answer: D. by rapid deterioration, new floaters, which may
Nebulised salbutarnol and ipratropiurn are suggest infection in the vitreous, and increasing
involved in the management of a life-threatening pain.

downloaded from www.medicalbr.com


-.
334 • MEDICAL OPHTHALMOLOGY

27.9. Answer: A. • stage 2: the temporal disc head becomes


Optic neuritis is an acute inflammatory process involved, creating a 360° circumferential
affecting the optic nerve. It usually presents swelling
with sudden monocular visual loss over hours • stage 3: vessel obscuration occurs at the
to days and eye pain in young adults, more disc margin
commonly in women. The Optic Neuritis • stage 4: vessel obscuration occurs at the
Treatment Trial (ONTT) elucidated the typical disc head
features of a demyelinating optic neuritis as
In established papilloedema, circumferential
follows:
retinal folds or Paton lines may form.
• Age 20-50 years Haemorrhage and cotton wool spots represent
• Unilateral ischaemic damage. In this setting, optic nerve
• Worsens over hours/days function will be reduced on examination and ,
• Recovery starts within 2 weeks may not recover. Severe hypertension causes
• Retrobulbar pain bilateral optic disc swelling in the absence of
raised intracranial pressure.
• Reduced colour vision
• Relative afferent pupillary defect
27.12. Answer: C.
In two-thirds of cases, the optic disc itself
The fundus image shows the early changes of
appears normal because the area of
dry age-related macular degeneration (AMD).
inflammation is deeper within the nerve -
Smoking is associated with a 2- to 3-fold
this is described as a retrobulbar optic
increased risk of developing age-related
neuritis.
macular degeneration. Use of the AREDS
Leber's hereditary optic neuropathy is a rare
(Age-Related Eye Disease Study) vitamin
progressive hereditary optic neuropathy: those
formulation was found to reduce the
affected are usually male and present with
progression to severe AMD by 25% in those
painless severe unilateral loss of central vision;
with advanced AMD in the fellow eye. There is
the second eye may become affected within
currently no evidence to suggest that
weeks to months of the first.
modification of hypertension or glycaemic
control has an effect on progression of AMD.
27.10. Answer: D. lntravitreal anti-VEGF therapy is currently
The history is convincing of giant cell arteritis. reserved for patients with neovascular AMD.
Whilst inflammatory markers are usually raised,
these are non-specific. A patient with suspicion
27.13. Answer: A.
of giant cell arteritis with raised inflammatory
The image shows retinal vein occlusion. Unlike
markers should be commenced on high-dose
vasculature elsewhere in the body, retinal
glucocorticoid without delay. This is due to the
I serious nature of the condition and risk of
ocular involvement. Temporal artery biopsy
arteries and veins share their adventitial sheath.
This means that arteriosclerotic changes in the
I remains the gold standard investigation.
artery can directly compromise the lumen of
the vein. This is the most common mechanism
Histopathological features include inflammation
where hypertension, hypercholesterolaemia,
of the arterial wall with fragmentation and
diabetes, smoking and obesity contribute the
disruption of the internal elastic lamina. The
majority of risk. In younger patients,
vessel may be affected in a 'patchy' manner;
inflammatory conditions that result in
therefore, obtaining a specimen of at least
inflammation of the vessel wall must be
1 em in length is recommended whenever
considered. Less common associations with
possible.
retinal vein occlusion include hyperviscosity,
inherited or acquired thrornbophilias. The most
27.11. Answer: B.
common predisposing ocular condition is
Papilloedema describes bilateral optic disc
glaucoma.
swelling secondary to raised intracranial
pressure. Severity is described by Frisen's 27.14. Answer: A.
scale where: The earliest featur~ of diabetic retinopathy on
• stage 1 : the optic disc is raised superiorly, retinal angiography is ischaemia characterised
nasally and inferiorly, creating a C-shape by capillary dropout. Microaneurysms can

downloaded from www.medicalbr.com


MEDICAL OPHTHALMOLOGY • 335

appear in background diabetic retinopathy, lntravitreal anti-VEGF injection can cause


whereas intraretinal microvascular anomalies, temporary regression of neovascularisation.
venous changes and lipid exudate characterise Focal and diffuse macular laser are treatments
more advanced disease. for diabetic macular oedema, although this is
being rapidly succeeded by intravitreal
27.15. Answer: D. anti-VEGF therapy. Photodynamic therapy with
Pan-retinal photocoagulation ablates the verteporfin is a treatment for other macular
peripheral ischaemic retina and permanently disorders, predominantly central serous
reduces production of pro-angiogenic factors. chorioretinopathy.

downloaded from www.medicalbr.com


RM Steel, SM Lawrie

Medical psychiatry
Multiple Choice Questions
28.1. A psychiatric history differs from a general 28.4. You are working in an emergency
medical history in which of the following key department. An elderly woman who has
respects? presented with a pretibial laceration is loudly
A. 'Drug history' refers to recreational drugs demanding that she be given priority treatment
rather that prescribed medication on the grounds that she is a close personal
B. 'Family history' refers to relationships within friend of the Prime Minister. A psychiatric
the family rather than illnesses affecting diagnosis of 'persistent delusional disorder' is
first- and second-degree relatives that might recorded in her case notes. Which of the
indicate genetic risk following statements best describes a delusion?
C. 'Past medical history' is less important A. A recurrent and intrusive thought that enters
D. Much of the examination is conducted during the patient's mind against their conscious
the course of history taking resistance and is recognised by the patient
E. The psychiatric history does not include as being a product of their own mind
'history of presenting complaint' B. An understandable belief that a patient
becomes preoccupied with to an
28.2. You are working in an emergency unreasonable extent
department. A 30 year old man presents C. An unshakeable false belief that is not
with excoriations on both forearms and tells accepted by other members of the patient's
you that he is experiencing a sensation of culture
something crawling under his skin. When D. A patient's perception and/or belief that
documenting this patient's mental state, under thoughts are being implanted into his/her
which heading would you record his tactile own head by someone or something else
hallucinations? E. When a patient's stream of thought shifts
A. Cognition suddenly from one thought to another very
B. Insight loosely or entirely unrelated thought
C. Mood
D. Perception 28.5. When reviewing a patient's neurology
E. Thought case notes, you read that her temporal lobe
epilepsy is characterised by a prodrome
28.3. Which of the following psychiatric comprising olfactory hallucinations. Which of
presentations is rare amongst general medical the following most accurately describes an
inpatients? hallucination?
A. Adjustment reactions A. A belief that has no rational basis
B. Alcohol-related disorders B. A false perce[)tion experienced by the patient
C. Delirium as arising in his/her own mind
D. Depression C. A fixed, false belief out of keeping with a
E. Schizophrenia patient's cultural background

downloaded from www.medicalbr.com


MEDICAL PSYCHIATRY • 337

D. A misperception of real external stimuli C. Alcoholic hallucinosis


E. A sensory perception occurring without an D. Delirium tremens
external stimulus E. Wernicke-Korsakoff syndrome

28.6. When on-call over the weekend in a large 28.1 0. A 28 year old businesswoman presents
general hospital, you are asked to attend the to the emergency department with chest pain
toxicology unit. Which of the following is true of and various other symptoms. She admits to the
self-harm? doctor that she has been taking cocaine and
A. Incidence increases with age some other recreational drugs. Which of the
B. It is more common in men than women following combination of features could be
C. It is the term psychiatrists use for 'attempted attributable to cocaine intoxication?
suicide' A. Auditory hallucinations and hypothermia
D. Methods that carry high risk of death are more B. Constricted pupils and sedation
likely to be associated with mental disorder C. Formication and auditory hallucinations
than are methods that carry low risk of death D. Hypothermia and constricted pupils
E. There is a lower incidence in lower E. Sedation and formication
socioeconomic groups
28.11. A 46 year old man is brought to the
28.7. An 80 year old retired lawyer who lives emergency department by emergency
independently is brought to the emergency ambulance. He says he is unable to breathe,
department by a neighbour who found him his hands and feet are tingling, he feels that he
wandering on the street in the early hours of is about to collapse and possibly die. On
the morning. He has no past psychiatric examination he has sinus tachycardia. Oxygen
history. As you attempt to interview him, the saturation is 100%. You notice that thi;:; is his
man says, 'This is a wonderful party. It is great fifth attendance at the emergency department
to see all those young people dancing.' Which in 3 months. Which is the most likely
is the most likely diagnosis? diagnosis?
A. Delirium A. Factitious disorder
B. Dementia B. Generalised anxiety disorder
C. Histrionic personality disorder C. Hypochondriacal disorder
D. Mania D. Obsessive-compulsive disorder
E. Schizophrenia E. Panic disorder

28.8. As a 35 year old man wakes from sleep 28.12. A 32 year old man with diabetes mellitus
he briefly sees a lion at the foot of his bed. survives an 8-day admission to critical care with
Which of the following most accurately overwhelming sepsis and ketoacidosis. On
describes his experience? discharge from hospital he appears happy and
A. Autoscopic hallucination glad to be alive. You review him at the diabetic
B. Functional hallucination clinic 2 months later and he tells you that he is
C. Hypnagogic hallucination waking in the middle of the night with vivid
D. Hypnopompic hallucination nightmares. He is now struggling to sleep, he
E. feels anxious and jumpy all of the time and

--
Kinaesthetic hallucination
finds himself bursting into tears very easily. His
28.9. A 48 year old barman is brought to the mother has been admitted to hospital but the
emergency department by his wife from whom thought of visiting her on a hospital ward
he has recently separated. She is concerned
that he is confused and 'talking nonsense'.
terrifies him. Which is the most likely diagnosis?
A. Acute stress reaction
1
He has an unsteady gait yet his breath B. Adjustment disorder
alcohol level is zero. On examination he has
ophthalmoplegia and is disorientated in time.
His liver function tests are deranged.
C.
D.
Delirium
Depression
I
E. Post -traumatic stress disorder
Which is the most likely diagnosis?
A. Alcohol withdrawal 28.13. You review a 55 year old man in the
B. Alcoholic dementia cardiology outpatient clinic. Two months ago

downloaded from www.medicalbr.com


338 • MEDICAL PSYCHIATRY
.,
he suffered an acute myocardial infarct 28.16. A 45 year old man presents with dry,
requiring thrombolysis and subsequent broken skin on both hands. He reports a
coronary artery stenting. He appears to be 10-month history of distressing repetitive
making a good physical recovery but he tells thoughts with a theme of hygiene. He
you that for the past few weeks he has been recognises that these are his own thoughts.
unable to experience pleasure from activities He describes a short -lived reduction in distress
that he would ordinarily enjoy (such as following hand washing and says that in recent
watching his favourite football team score a weeks he has been washing his hands more
goal). Which of the following terms most and more. What are the man's thoughts most
accurately describes this symptom? likely to be?
A. Anhedonia A. Auditory hallucinations
B. Depression B. Catastrophisations
C. Dysphoria C. Compulsions
D. Euthymia D. Obsessions
E. Hypomania E. Ruminations

28.14. Which one of the following statements 28.17. A 19 year old female student is brought
about psychiatric treatment is true? to the emergency department at midnight by
A. Most patients treated with psychiatric her friends. They had been out drinking
medication suffer significant sedation as a together but when she became so intoxicated
side-effect that she was unable to walk they became
B. Psychotropic medications can be prescribed worried about her and took her to hospital.
by psychiatrists and psychologists She is admitted overnight for observation
C. The majority of psychiatric patients are given (temperature, blood pressure, heart rate and
treatment against their will respiratory rate are all normal) and you review
D. There is considerable randomised controlled her on the ward round the following day.
trial evidence to support use of cognitive Her urea and electrolytes, liver function tests,
behavioural therapy (CBT) for depression and thyroid function tests and full blood count are
anxiety disorders all normal. On examination she is extremely
E. There is little randomised controlled trial thin, weight 38 kg (body mass index 16 kg/m 2 )
evidence to support pharmacological with lanugo hair on her arms and back. You tell
interventions in psychiatry her you are concerned about her low weight
and ask her about her eating. She tearfully tells
28.15. A 43 year old woman attends your you that she started dieting 8 months ago but
general medical clinic for investigation of it now dominates her life and she thinks she
severe and persistent fatigue. No abnormalities has developed anorexia nervosa. Unfortunately
are evident on examination or investigation. the hospital has no psychiatric liaison service.
On reviewing her medical record you see What is the most appropriate immediate
f] that over the past 25 years she has had management?
,,.i numerous visits to hospital, including to A. Dietetic review and referral for urgent
the ear, nose and throat (ENT) department, psychiatric outpatient assessment
where she was diagnosed with B. Mental health act detention and compulsory
temporomandibular joint dysfunction; refeeding
psychiatry, where she was diagnosed with C. Prescribe rnirtazapine as an antidepressant
depression; gynaecology, where a and appetite enhancer
hysterectomy was performed for menorrhagia; D. Transfer to the local psychiatric hospital for
and gastroenterology, where she was specialist inpatient treatment
diagnosed with irritable bowel syndrome. What E. Voluntary refeeding as a medical inpatient
is the most likely diagnosis?
A. Factitious disorder 28.18. A 30 year old man brings his 27 year old
B. Fibromyalgia wife and 10 day old son to the emergency
C. Hypochondriacal disorder department He says that over the past 2 days
D. Malingering his wife has not been her normal self. Initially
E. Somatisation disorder she appeared unusually anxious about the

downloaded from www.medicalbr.com


MEDICAL PSYCHIATRY • 339

baby, unable to put him down for more than a D. Genetic mutations that cause frontotemporal
few minutes. Last night she stayed up through dementia are also associated with
the night and this morning she refused to amyotrophic lateral sclerosis
accept any of the food or drink that he offered E. The genetic basis of Alzheimer's disease is
her and will not let him hold the baby. She will unknown
not tell him what is wrong but agreed to come
to the hospital as she said she would 'feel safer 28.22. A 50 year old woman with alcohol
there'. What is the likely diagnosis? dependence syndrome is admitted to hospital
with cellulitis in her foot. When the nurse gives
A. Post-partum blues
her lunch, she comments that this is the first
B. Post-partum depression
meal she has had for weeks as she has been
C. Post-traumatic stress disorder
spending all of her money on cider. On
D. Puerperal psychosis
examination she is fully orientated and does not
E. Schizophrenia
appear confused. She has a tremor, is
28.19. Which of the following statements is true sweating and tachycardic. What would
about the biological basis of psychiatric appropriate management comprise?
disorders? A. Acarnprosate and diazepam
A. A large number of conditions have an B. Diazepam and parenteral vitamins (Pabrinex)
identified single genetic cause C. Disulfiram and acamprosate
B. Depression is associated with focal D. Haloperidol and disulfiram
reductions in 5-hydroxytryptamine (5-HT, E. Parenteral vitamins (Pabrinex) and haloperidol
serotonin) receptor binding
C. Most disorders have a discrete underlying
28.23. A 22 year old male is brought into the
emergency department. He is agitated, difficult
abnormality on neuroimaging
to converse with, smells of alcohol and says
D. Schizophrenia is associated with increased
post-synaptic dopamine D2 receptor binding that he is being persecuted by secret services;
E. Task-based functional magnetic resonance however, he is fully oriented. Which of the
following is the most likely diagnosis?
imaging (MRI) is the technique of choice for
analysing the interactions between multiple A. Alcohol withdrawal-
brain regions B. Bipolar affective disorder
C. Drug intoxication
28.20. Which of the following complaints are D. Drug-induced psychosis
later (as opposed to earlier) manifestations in E. Schizophrenia
the natural history of dementia?
A. Difficulty getting dressed
28.24. A patient with schizophrenia getting
B. Getting lost in familiar surroundings treated with clozapine wants to speak to you
C. Personality change about her treatment. Which of the following
D. Subjective memory problems statements are true of clozapine?
E. Disinhibited behaviour A. Electrocardiogram (ECG) monitoring is
mandatory as clozapine commonly causes
28.21. A 70 year old widowed woman is cardiac arrythmias
brought to your clinic with a history of memory B. It can cause dry mouth
impairment and aggressive behaviour for C. It is a first-line treatment for schizophrenia
investigation and treatment. Which of the D. It is associated with constipation
following statements is true of the E. It is associated with myeloproliferation
pathophysiology and management of
dementia? 28.25. A 30 year old woman with a history of
A. Anticholinesterase mediCation is indicated to bipolar disorder treated with lithium wants to
treat memory impairment in Pick's disease have a child and wonders if she should stay on
B. Anticholinesterases may of some benefit in the treatment. Which of the following
the late stages of Alzheimer's disease statements are true of lit~ium salts?
C. Creutzfeldt-Jakob disease has characteristic A. Hypopararathyroidism is a potential risk
electroencephalogram (EEG) abnormalities of B. They are contraindicated in pregnancy
generalised slow waves because of a risk of neural tube defects

downloaded from www.medicalbr.com


340 • MEDICAL PSYCHIATRY
,
C. They have a wide therapeutic range statements relating to depression is true and
D. They should be reserved for might guide diagnosis and management?
treatment -resistant cases of bipolar A. Antidepressants do not work if patients have
disorder ongoing medical problems
E. Toxic effects include nausea, vomiting, B. CBT and other psychotherapies are less
tremor and convulsions effective for depression than antidepressants
C. Electroconvulsive therapy (ECT) is the
28.26. A 55 year old man with a history of treatment of choice for severe depression
ischaemic heart disease complains of low D. Depression has a similar prevalence in
mood and an inability to derive pleasure from people with chronic medical complaints as in
I
activities he used to enjoy, as well as fatigue, the general population
disturbed sleep, poor concentration and E. Tricyclic antidepressants and SSRis can
reduced appetite. Which of the following cause OTc interval prolongation

Answers
28.1. Answer: D.
28.1 How to structure a psychiatric interview
'Drug hi?tory' refers to both recreational and
prescribed medication (and over-the-counter Presenting problem
and herbal preparations!). This is true of both a Reason for referral
Why the patient has been referred and by whom
general medical and a psychiatric history. In the
Presenting complaints
psychiatric history, 'family history' refers to both
The patient should be asked to describe the main
familial conditions and relationships. Much of problems for which help is requested and what they
the mental state examination is conducted want the doctor to do
during the course of psychiatric history taking, History of present illness
rather than as a separate set of procedures at The patient should be asked to describe the course of the
the end (Box 28.1). 'History of presenting illness from when symptoms were first noticed
The interviewer asks direct questions to determine the
complaint' is as prominent in a psychiatric
nature, duration and severity of symptoms, and any
history as it is in clinical histories taken in other associated factors
specialties, as is 'past medical history'. Background
Family history
28.2. Answer: D. Description of parents and siblings, and a record of any
A tactile hallucination is the experience of mental illness in relatives
perceiving touch in the absence of a touch Personal history
stimulus. It is an abnormality of perception. Birth and early developmental history, major events in
The mental state examination (MSE) is a childhood, education, occupational history,
relationship(s), marriage, children, current social
systematic examination of the patient's thinking, circumstances
emotion and behaviour. As with the clinical Previous medical and psychiatric history
examination in other areas of medicine, the aim Previous health, accidents and operations
is to elicit objective clinical signs. Whilst many Use of alcohol, tobacco and other drugs
aspects of the patient's mental state may be Direct questions may be needed concerning previous
psychiatric history since this may not be volunteered:
observed as the history is being taken, specific
'Have you ever been treated for depression or nerves?'
enquiries about important features should or 'Have you ever suffered a nervous breakdown?'
always be made. Previous personality
The patterns of behaviour and thinking that characterise a
28.3. Answer: E. person, including their relationships with other people
Adjustment reactions, alcohol-related disorders, and reactions to stress (useful information may be
obtained from an informant who has known the patient
delirium and depression are all very common well for many years)
within the general medical inpatient population
(Box 28.3). Rates of schizophrenia in the general
medical inpatient population are similar to rates in
the general population. The lifetime prevalence of
schizophrenia is approximately 1%.

downloaded from www.medicalbr.com


MEDICAL PSYCHIATRY • 341

28.3 Prevalence of psychiatric disorders by medical setting


Medical/surgical
General practice Outpatients Inpatients General psychiatric services
Delirium +++
Alcohol/substance abuse ++ ++ +++ +++
Schizophrenia +++
Bipolar affective disorder +++
Depression ++ ++ +++ +++
Anxiety disorders ++ ++ ++ +++
Adjustment disorders ++ ++ +++ +
Somatoform disorders + +++ ++
Personality disorders + + + +++
- 'rare' (2%); + 'uncommon' (2-5%); ++ 'common' (5-10%); +++ 'vel}' common' (10%)

28.4. Answer: C. Most cases of SH that come to medical


A delusion is a false belief, out of keeping with attention involve overdose, but other methods
a patient's cultural background, which is held include asphyxiation, drowning, hanging,
with conviction despite evidence to the jumping from a height or in front of a moving
contrary. It is common to classify delusions on vehicle, and firearms. Methods that carry a high
the basis of their content. They may be: chance of being fatal are more likely to be
• persecutory - such as a conviction that associated with serious psychiatric disorder.
others are out to harm one Self-cutting is common and often repetitive, but
• hypochondriacal - such as an unfounded rarely leads to contact with medical services.
conviction that one has cancer SH is more common in women than men, in
young adults than the elderly, and in lower
• grandiose - such as a belief that one has
socioeconomic groups. In contrast, completed
special powers or status
suicide is more common in men and the elderly
• nihilistic- such as 'My head is missing',
(Box 28.6).
'I have no body' or 'I am dead'
Option A describes an obsessional thought, 28.6 Risk factors for suicide
B describes an over-valued idea, D describes
Psychiatric illness (depressive illness, schizophrenia)
thought insertion and E describes loosening of Older age
associations. Male sex
Living alone
Unemployment
28.5. Answer: E. Recent bereavement, divorce or separation
A sensory perception arising without an Chronic physical ill health
external stimulus is an hallucination. Meanwhile, Drug or alcohol misuse
an external stimulus that is rnisperceived is an Suicide note written
History of previous attempts (especially if a violent method
illusion. A false perception that does not have
was used)
the characteristics of a normal sensory
perception (such as a voice heard in one's 28.7. Answer: A.
head rather than in external space) is called a The main differentials in a man of this age with
pseudo-hallucination. A belief that has no no previous psychiatric history are delirium and
rational basis is simply a false belief. 'A fixed, dementia. The acute onset, disrupted sleep-
false belief out of keeping with a patient's wake cycle and visual hallucinations are all
cultural background' is the conventional suggestive of delirium.
definition of a delusion. Personality disorders, bipolar affective disorder
and schizophrenia typically emerge during
28.6. Answer: D. adolescence or early adult life; they are unlikely
Self-harm (SH) is a common reason for to present for the first time at the age of 80.
presentation to medical services. 'Self-harm'
and 'attempted suicide' are not synonymous. 28.8. Answer: D.
Whilst some patients who self-harm are An autoscopic hallucination is the experience of
motivated by a desire to end their life, many seeing an image of oneself in external space. A
have other motivations. functional hallucination is a false perception that

downloaded from www.medicalbr.com


1
342 • MEDICAL PSYCHIATRY

is triggered by a (normal) sensory stimulus: for Obsessive-compulsive disorder is characterised


example, a flash of light causing an olfactory by repetitive, intrusive, anxiety-provoking
hallucination. A hypnagogic hallucination is a thoughts, images or impulses that give rise to
brief hallucination experienced whilst going to repeated behaviours (compulsions) such as
sleep, whereas a hypnopompic hallucination is checking, washing or counting, which initially
experienced whilst waking from sleep. A reduce the anxiety but over time become
kinaesthetic hallucination is a false perception problematic in themselves.
of joint or muscle sense (proprioception): for
example, a hypnagogic hallucination of falling 28.12. Answer: E.
through space. Post-traumatic stress disorder (PTSD) is
relatively common in patients who recover after
28.9. Answer: E. being in critical care. It is characterised by a
Wernicke-Korsakoff syndrome is classically period of latency followed by intrusive
described as a combination of ataxia, flashbacks or nightmares, psychophysiological
delirium and ophthalmoplegia (Wernicke's hyper-arousal and avoidance.
encephalopathy) with short-term memory loss 'Acute stress reaction' is the diagnostic term
leading to disorientation and confabulation for what is colloquially termed 'shock' -a
(Korsakoff's syndrome). transient sense of bewilderment or 'daze'
This man is presenting with acute signs arising in the context of a profoundly stressful
suggestive of Wernicke's encephalopathy, situation. An adjustment disorder is a state of
which is a medical emergency. Urgent distress and emotional disturbance (that can
treatment with parenteral thiamin is needed to include symptoms of depression and/or anxiety)
prevent permanent brain damage. arising during a period of adaptation to a
significant life event. Delirium is a state of
28.10. Answer: C. altered brain function arising from physical
Cocaine intoxication causes heightened arousal illness or other physiological challenge.
(not sedation) and pupillary dilatation (not
constriction). At higher doses, the heightened 28.13. Answer: A.
arousal can lead to agitation and to psychotic 'Anhedonia' is classically described as the
symptoms such as auditory hallucinations and inability to feel pleasure in normally pleasurable
formication (the sensation of ants crawling activities. The term 'depression' is often used in
under one's skin). Cocaine can also cause a non-medical context to mean 'low mood' but
hyperthermia (rather than hypothermia). in a medical context it is a diagnosis rather
than a symptom. This man is not reporting low
28.11. Answer: E. mood per se but rather an inability to feel
The clinical presentation is highly suggestive of pleasure. 'Dysphoria' is a feeling of being ill at
a panic attack. Anxiety leads to hyperventilation ease. 'Euthymia' describes mood that is neither
and a respiratory alkalosis that triggers the high nor low. 'Hypomania' is a milder form of
I physical symptoms described. These physical
symptoms are catastrophically interpreted,
mania.

thereby generating more anxiety and a vicious 28.14. Answer: D.


cycle is established. Psychiatrists are medical practitioners and are
Acute treatment involves helping patients to therefore licensed to prescribe medication.
control their breathing, sometimes even getting Psychologists are not medically qualified;
them to re-breathe exhaled air (using a paper therefore, in the majority of jurisdictions,
bag) to correct the respiratory alkalosis. Panic psychologists are not licensed to prescribe.
attacks are differentiated from general anxiety There is a great deal of randomised controlled
disorder by the episodic rather than chronic trial evidence to support both psychotropic
nature of the anxiety, although the two can medications and psychotherapies such as CBT.
coexist. A relatively small minority of patients require
Factitious disorder is the deliberate feigning compulsory treatment (detention under mental
of physical symptoms or induction of physical health legislation). Sedation can occur with
signs for no obvious gain. The core feature of some medications (e.g. high-dose
hypochondriacal disorder is a fear of a specific antipsychotics) but most of the commonly
serious disease (such as cancer). prescribed medications (for example, selective

downloaded from www.medicalbr.com


MEDICAL PSYCHIATRY • 343

serotonin re-uptake inhibitors; SSRls) do not few weeks after delivery. Post-partum
cause sedation. depression is depression arising following
childbirth. Post-traumatic stress disorder is a
28.15. Answer: E. delayed reaction to an extremely stressful
Somatisation disorder is the diagnostic term for event. It is characterised by flashbacks,
patients who, over many years, experience (and avoidance and hyper -arousal.
present to medical services with) somatoform
(medically unexplained) symptoms affecting 28.19. Answer: B.
more than one system. Such patients are not Very few psychiatric disorders have a single
faking their symptoms (in contrast to patients cause of any sort; most are multifactorial and
with factitious disorder or malingering) and they polygenic. Very few conditions have a discrete
are not usually worried about a possible serious underlying brain lesion: abnormalities on
underlying condition (hypochondriasis). When neuroimaging, even in dementia or
assessing patients with somatoform symptoms schizophrenia, occur in < 10% and most
it is important to recognise and acknowledge disorders are characterised by 'dysconnectivity'
that their symptoms are real, distressing and (abnormal interactions between brain regions).
disabling and to explain that doctors often see Schizophrenia is associated with increased
patients whose symptoms cannot be explained pre-synaptic dopamine synthesis and turnover.
by disease. Resting-state functional MRI is the technique of
choice for analysing the interactions between
28.16. Answer: D. multiple brain regions.
This is a typical description of obsessive-
compulsive disorder (OCD), and the thoughts 28.20: Answer: A.
are 'obsessions'. The repetitive hand washing Subjective memory complaints are a common
sustained by the temporary relief from anxiety is early manifestation of all dementias. Getting lost
the 'compulsion'. 'Rumination' refers to the in familiar surroundings is a common presenting
focusing of attention on one's symptoms. feature of Alzheimer's. Difficulty getting dressed
Catastrophising is viewing a situation as much and other dyspraxias are usually more of a
worse than it actually is (relatively common in problem in the later stages of Alzheimer's
depression and anxiety). disease. Behaviour and personality change are
common early manifestations of the
28.17. Answer: A. frontotemporal dementias.
Despite her low weight, this woman is not
acutely medically unwell: hence neither 28.21. Answer: D.
psychiatric nor ongoing medical admission are Creutzfeldt-Jakob disease is characterised by a
indicated. She needs to be supported to gain generalised periodic sharp wave pattern on
weight but this is best done collaboratively as EEG. Mutations in several genes have
an outpatient. Antidepressant medication does been described in Alzheimer's disease but
not have a major role to play in the most are rare and/or of small effect (such
management of anorexia nervosa. as apolipoprotein E epsilon 4; APOe4).
Anticholinesterases have been shown to be
28.18. Answer: D. of some benefit at slowing progression of
Puerperal psychosis affects approximately 1 in cognitive impairment, but only in the early
500 women, with a peak onset in the first 2 stages of the disease, while post-synaptic
weeks after birth. It usually takes the form of an cholinergic receptors are still available. Although
affective psychosis (manic, depressive or Alzheimer's and Pick's (as one of the
mixed) but can sometimes resemble frontotemporal dementias; FTDs) share certain
schizophrenia. The onset is often rapid, with symptoms, they cannot be treated with the
transition from normal mental state to same pharmacological agents because the
psychosis in a matter of days. Women often cholinergic systems are not affected in FTD.
conceal their symptoms, which can place both
the woman and her baby at significant risk. 28.22. Answer: B.
Post-partum blues is the term used to This woman is at risk of Wernicke-Korsakoff's
describe a transient, self-limiting period of syndrome as a consequence of alcohol
increased emotional reactivity during the first dependence and poor nutrition. She should be

downloaded from www.medicalbr.com


344 • MEDICAL PSYCHIATRY

given prophylactic parenteral vitamins Clozapine is not typically associated with


(Pabrinex). She is also showing early signs of cardiac arrhythmias. EGG monitoring is good
alcohol withdrawal and should therefore be medical practice, and myocarditis and
prescribed an alcohol withdrawal regime circulatory collapse are rare but may arise in
(diazepam or similar). Haloperidol is not the first months of treatment.
effective in alcohol withdrawal and may
precipitate alcohol withdrawal seizures. 28.25. Answer: E.
Disulfiram and acamprosate are both used to Lithium is the treatment of choice for bipolar
support long-term abstinence in alcohol disorder, as it has proven efficacy in both
dependence syndrome. They have no role in phases of the disorder. Lithium salts are
acute presentations. associated with hyperparathyroidism and have
a narrow therapeutic range. They may be
28.23. Answer: E. teratogenic, although this has probably been
Alcohol addiction sufficient to cause a exaggerated over the years and the risk (if
withdrawal syndrome is rare in the young and anything) is with Ebstein's anomaly; thus,
the patient smells of alcohol, suggesting he still pregnant mothers with bipolar disorder may be
has some in his system. Cannabis, stimulant or better treated than untreated, although this is a
hallucinatory drug intoxication or induced clinical decision that should be made in
psychosis could cause such a presentation, but consultation with a specialist following
would usually also be associated with some exploration of the risks and benefits for the
degree of cognitive impairment and/or patient and her unborn child. Neural tube
disorientation. Bipolar disorder is possible but defects are associated with valproate and
the content of any delusions present is usually · carbamazepine, which are therefore
mood-congruent, for example, a patient with contraindicated in pregnancy.
mania might report being persecuted because
of some special talents or identity. 28.26. Answer: E.
Depression affects about 20% of patients with
28.24. Answer: D. chronic medical complaints, but these patients
Clozapine is the only treatment licensed for the are still likely to benefit from antidepressant
treatment of schizophrenia that has not treatment. CBT and other psychotherapies are
responded to antipsychotic drugs. It is not a about as effective as antidepressants for
first-line treatment because of a range of depression in general, although they are
dangerous adverse effects. Hypersalivation is a probably less effective in severe depression
common adverse effect. White blood cell where antidepressants are the first -line
monitoring is required to monitor for the treatment. ECT is the treatment of choice for
uncommon adverse effect of agranulocytosis. severe depression that has not responded to

I Constipation is a common adverse effect. antidepressants and for psychotic depression.

I
:~
'i'f
i!:.

downloaded from www.medicalbr.com


SH Ibbotson

Dermatology
Multiple Choice Questions
29.1. A 23 year old woman presents with a 29.2. A 45 year old farmer is prescribed
history of an itchy papular rash (see below), doxycycline 200 mg orally daily for rosacea.
which developed on her chest and arms in the A week later he experiences a severe sunburn
evening, after sitting in the sun for 2 hours, reaction after being out one morning on his
earlier that day. The eruption lasts for 3 days tractor on a cloudy day (see below). What is
before fully resolving. What is the most likely the most likely finding on investigation?
diagnosis?

A. Abnormal plasma porphyrin scan


B. Positive antinuclear antibodies (ANA) and
antibodies to extractable nuclear antigens
(ENA)
C. Positive patch testing to doxycycline
D. Positive ultraviolet B (UVB) sensitivity on
monochromator phototesting
E. Ultraviolet A (UVA) sensitivity on
monochromator phototesting

29.3. A 22 year old pregnant woman presents


I
to her family physician at 32 weeks' gestation
with a 1-week history of intense generalised
A. Chronic actinic dermatitis pruritus, which is preventing her from sleeping.
B. Erythema multiforme Her family physician exa~ines her and finds no
C. Lupus erythematosus abnormalities.
D. Polymorphic light eruption Which of the following would be the most
E. Solar urticaria important investigation to undertake?

downloaded from www.medicalbr.com


346 • DERMATOLOGY

A. ANA and complement B. Carbamazepine is the likely cause


B. Full blood count G. Full recovery is likely for patients who are
G. Liver function tests under 60 years of age
D. Thyroid function tests D. Skin pain suggests a possible viral trigger
E. Urinalysis E. There is good evidence to support the use
intravenous immunoglobulins (IVIg)
29.4. An 86 year old man is referred to the
dermatology clinic with a 2-week history of 29.6. A 75 year old woman presents to clinic
bilateral lower leg redness, scaling and swelling. with 12-month history of a crusted plaque on
He has a history of leg ulcers on both lower her lower leg, which has increased in size
legs for the last 2 years and full-layer despite the use of potent topical corticosteroid.
compression stockings were introduced 6 Examination reveals a well-defined crusty
weeks previously. He is otherwise well and on plaque 2 em in diameter that is considered
examination there is sharply defined bilateral most likely to be a patch of Bowen's disease
erythema and scaling, with associated oedema (see below). The patient also has a history of
of the lower legs and a sharp cut -off below the varicose veins and ankle swelling, which are
knee. There is also ulceration of both medial evident on examination.
lower legs, which is long-standing with
surrounding lipodermatosclerosis.
What is the most likely diagnosis?
A. Acute ischaemia
B. Bacterial cellulitis
G. Allergic contact dermatitis
D. Deep venous thrombosis
E. Necrotising fasciitis
29.5. A 35 year old woman presents to the
emergency department with a 2-day history of
painful erythema, which commenced on the trunk
and spread rapidly to the limbs, and a 1-day
history of soreness of the mouth and dysuria. She Which of the following would be the most
had been commenced on carbamazepine for appropriate management plan?
epilepsy 3 weeks earlier and trimethoprim 3 days A. Cryotherapy
ago. On assessment there is extensive erythema, B. Photodynamic therapy
with incipient blistering and denudation of the G. Radiotherapy
epidermis affecting about 60% body surface area D. Refer for excisional surgery
and a positive Nikolsky sign. She is tachycardic, E. Topical ingenol mebutate
normotensive and afebrile.
29.7. A 33 year old woman presents with a
3-month history of an enlarging pigmented
nodule on the forearm. On examination by the
naked eye and under a dermatoscope, a
diagnosis of melanoma is suspected. What
would be the most appropriate next step in
management?

A diagnosis of drug-induced toxic epidermal


necrolysis (TEN) is considered most likely.
Which of the following statements is true?
A. Antibiotics should be prescribed, given the
n
symptom of dysuria 3
downloaded from www.medicalbr.com
DERMATOLOGY • 347

A. Curettage and cautery


B. Excision with a 1 em margin
C. Excision with a 2 em margin
D. Excision with a 2 mm margin
E. Initial incisional biopsy

29.8. A 23 year old woman is attending for


narrowband UVB phototherapy for the treatment
of guttate psoriasis (see below). After the third
treatment she reports the development of an
itchy papulovesicular rash on the arms, legs and
trunk, and a clinical diagnosis of polymorphic
light eruption (PLE) is made. Her only medication
is the contraceptive pill.

29.10. A 16 year old girl presents with a history


of persistent papulopustular inflammatory acne,
which has not responded to topical antibiotics or
Which of the following statements is benzoyl peroxide preparations (see below). She
correct? is otherwise well but is very troubled by her acne
A. Phototherapy should be discontinued and it is stopping her from socialising. What is
B. PLE is a common cause of the most appropriate next management step?
photo-aggravation of psoriasis
C. PLE is more common in patients on the
contraceptive pill
D. PLE is more common in patients with
psoriasis
E. Systemic prednisolone should be
prescribed

29.9. A 68 year old man presents with a 2-year


history of a nodular lesion on the right cheek,
which has gradually increased in size but
otherwise does not trouble him. He is well, A. Combined oestrogen/anti-androgen orally for
although he is on warfarin as he has a history 3 months
of transient ischemic attacks. Examination B. Erythromycin 250 mg orally daily for 3
reveals a translucent nodule on the right months
nasolabial fold region, with telangiectatic C. lsotretinoin 0.5 mg/kg orally daily for 4
vessels (see below). What is the most months
appropriate management approach? D. Lymecycline 408 mg daily for 3 months
A. Mohs' micrographic surgery E. Minocycline 100 mg daily for 3 months
B. Photodynamic therapy
C. Standard excisional surgery 29.11. A 12 year old girl complains of itching of
D. Topical imiquimod the scalp and has recently returned from a
E. Tapical ingenol mebutate residential school camp. Her mother thought

downloaded from www.medicalbr.com


. .-----------------------------------------------------------------------------1
348 • DERMATOLOGY I I
I

her daughter had more dandruff than usual and Which of the following statements is correct?
tried her with an over-the-counter anti-dandruff A. Examination with Wood's light will show areas
shampoo, which did not help. She then asked of fluorescence if the endothrix is involved
the advice of her family physician, who thought B. If the diagnosis is confirmed he should be
the daughter may have 'nits'. treated with topical terbinafine
Which of the following statements is correct? C. Oral griseofulvin is the antifungal agent of
A. 'Nits' are the active head louse Pediculus choice for children in the UK
humanus capitis and are easily seen on the D. Systemic glucocorticoids will prevent any
scalp and often confused with dandruff further hair loss
B. All cases of head lice need intensive E. Tinea capitis is a dermatophyte fungal
treatment with insecticides infection of the scalp hair bulb
C. Head lice infestation is highly contagious and
all members of the family and all classmates 29.13. A 62 year old woman presents with a
should be treated at the same time 6-week history of a rapidly enlarging lesion on
D. Malathion would be the insecticide of choice the right cheek, which is otherwise
for active treatment asymptomatic (see below). She lived in South
E. Thorough combing of wet, conditioned hair Africa until the age of 20 years and has
may be effective previously had three BCCs excised. What would
be the most appropriate course of action?
29.12. A 10 year old boy presents with a patch
of inflammation and hair loss in the scalp,
which was noticed when he went for a haircut.
He is otherwise well, with no medical history
and lives at home with his mother, brother and
pets. They have just returned home from a
holiday on a farm. On examination, there is a
boggy area of inflammation in the scalp, with
overlying pustules and hair loss (see below).
This is thought most likely to be tinea capitis.

CM
A. Biopsy and photodynamic therapy
B. Biopsy and topical irniquimod
C. Excisional surgery
D. Mohs' micrographic surgery
E. Observation

29.14. A 65 year old man presents with an


enlarging, darkening. patch of brown
pigmentation adjacent to the eye, and biopsy
confirms lentigo maligna. The patient is
otherwise fit and well.

downloaded from www.medicalbr.com


I
I

DERMATOLOGY • 349

Which statement is correct? What is the most likely diagnosis?


A. Excisional surgery is usually curative A. Contact allergy
B. Histological disease often persists despite B. Fixed drug eruption
clinical clearance G. Herpes simplex virus
G. Lentigo maligna is best treated non-surgically D. Molluscum contagiosum
by topical imiquimod . E. Tinea corporis
D. Lentigo maligna should be treated by
photodynamic therapy 29.17. A 64 year old man presents with three
E. There is high risk of invasive SCC if left episodes of angioedema occurring over 6
untreated weeks. He has a complex medical history with
hypertension, diabetes mellitus and ischaemic
heart disease, but has not previously had any
29.15. A 65 year old female presents with a dermatological disease. His medications include
2-year history of recurrent crops of pustules on
ramipril, insulin, bendroflumethiazide,
the palms and soles (see below). There is no
clopidogrel and warfarin.
past history of dermatological disease. She has
Which of the following statements is correct?
diabetes and is hypertensive but is otherwise
well and has recently been trying to stop A. It is unusual that he does not also have
smoking. She is informed that the diagnosis is urticaria
palmoplantar pustulosis. B. Patch testing would be the investigation of
choice
G. Ramipril-induced angioedema is the most
likely diagnosis
D. There is an increased incidence of
angioedema in patients with diabetes
E. Bendroflumethiazide-induced angioedema is
the most likely diagnosis

29.18. Which cells in the skin are primarily


involved in antigen presentation?
A. B lymphocytes
B. Keratinocytes
G. Langerhans' cells
·D. Merkel cells
E. T lymphocytes
Which of the following is correct?
A. Bacterial swabs from pustules usually grow 29.19. The basement membrane serves as an
Staphylococcus aureus anchor for the epidermis and allows movement
B. Palmoplantar pustulosis is usually associated of cells and nutrients between the dermis and
with chronic plaque psoriasis the epidermis. Which of the following
G. PUVA (psoralen and ultraviolet A) treatment statements is correct?
is usually effective
A. The anchoring fibrils consist of type IV
D. Smoking cessation is likely to result in
collagen
disease improvement
B. The lamina densa contains mainly type VII
E. Tapical glucocorticoids should be avoided
collagen
due to potential flaring of psoriasis
G. The lamina Iucida lies above the basal cell
membrane
29.16. A 32 year old woman presents with a D. The main hemidesmosomal collagen is
painful, well-demarcated erythematous plaque type XIV
on the buttock, which occurs most months E. The tonofilaments mainly consist of keratin 5
pre-menstrually. She is otherwise well and has and 14
recently started the oral contraceptive for
menorrhagia. On close inspection, there is a 29.20. The parents of a child with xeroderma
well-defined plaque on the right buttock with pigmentosum, a rare genetic disease
clustered vesicles evident. associated with severe photosensitivity, have

downloaded from www.medicalbr.com


350 • DERMATOLOGY 1
I
I
I

found out through the internet that vitarnin D B. Non-sedating antihistamines are usually
deficiency can lead to irnpaired bone health, effective
rickets and osteomalacia. Which of the C. She is advised that she should be delivered
following statements is correct? early because of fetal risk
A. Dietary vitarnin D intake is effective for D. She is advised that this is unlikely to be a
vitarnin D deficiency problern in subsequent pregnancies
B. Skin fibroblasts are the rnain source of E. Topical glucocorticoids should be avoided
vitarnin D synthesis due to potential adverse fetal effects
C. UVA exposure is required for cutaneous
vitarnin D synthesis 29.23. A 63 year old wornan presents with a
D. UVB exposure is required for cutaneous pigmented lesion on the anterior chest noted
vitarnin D synthesis by her farnily physician during Well Wornan
E. Vitarnin D toxicity is a risk for patients with screening. She had been aware of the lesion
increased photosensitivity for I 0 years and did not think it had
significantly changed, although on closer
29.21. A 54 year old wornan with chronic questioning, she said it had enlarged over the
urticaria, without obvious history of trigger, last year. Excisional surgery confirrns a
is referred to dermatology for investigation diagnosis of invasive superficial spreading
and rnanagernent (see below). With the malignant rnelanorna (Breslow thickness
exception of hay fever and long-standing 0.8 rnrn).
vitiligo, she is otherwise well and is taking no
medications.

Which would be the rnost appropriate


investigation?
A. Patch testing
B. Prick tests
C. Specific irnrnunoglobulin E (lgE)
D. Thyroid function tests
E. TotallgE

29.22. A 28 year old wornan, 32 weeks into


her first pregnancy, presents with an itchy
I
I erythematous papular eruption that she first Which of the following statements is correct?
noticed in the stretch rnarks on her abdornen A. Breslow thickness is the rnost important
'I and thighs 2 weeks earlier. A diagnosis of prognostic factor
pruritic urticarial papules and plaques of B. Five-year disease-free survival is about
pregnancy (polymorphic eruption of pregnancy) 85%
is rnade and she is offered advice and C. The patient will need indefinite follow-up
treatment. D. Total body cornp,uted tomography (CT) scan
Which of the following statements is correct? is recornrnended
A. Abnormalities in liver function tests E. Wide local excision with a 0.5-crn rnargin is
cornrnonly occur advised

downloaded from www.medicalbr.com


DERMATOLOGY • 351

29.24. A 62 year old man presents with a D. Severity of nail involvement


2-month history of blistering and fragility on E. Type of psoriasis
the back of his hands (see below). He is
otherwise well and enjoys socialising with his 29.27. Which of the following conditions would
friends. be likely to flare if treated with PUVA
photochemotherapy?
A. Atopic dermatitis
B. Bullous pemphigoid
C. Chronic urticaria
D. Lichen planus
E. Pustular psoriasis

29.2B. A 56 year old man presents with an


ill-defined nodular lesion on the nasal bridge
abutting the medial canthus (see below). What
would be the most appropriate management
approach?
Which of the following investigations would
be most useful?
A. Lupus serology
B. Patch testing
C. Porphyrin plasma scan
D. Skin biopsy and direct immunofluorescence
E. Urinalysis

29.25. A 39 year old woman with psoriasis,


which is controlled by acitretin, wishes to
become pregnant. What advice would
you give her with regard to her plans to
conceive?
A. Acitretin is not as teratogenic as isotretinoin
so she can conceive on drug
B. She can try to conceive once she has been
off acitretin for 1 year
C. She can try to conceive once she has been
off acitretin for 6 months
D. She can try to conceive once she has been
off acitretin for 3 years
E. She can try to conceive once she has been
off acitretin for 2 months
A. Curettage and cautery
29.26. A 64 year old man with recalcitrant B. Mohs' surgery

--
psoriasis, despite UVB, PUVA and C. Photodynamic therapy
methotrexate, is being considered for biological D. Radiotherapy
therapy. He has extensiv~ chronic plaque E. Topical imiquimod
psoriasis, nail and scalp involvement and
troublesome psoriatic art.hritis. On assessing 29.29. Which of the following is a normal variant
the Psoriasis Area and Severity Index (PASI), of nail growth?
which of the following is included in the A. Longitudinal ridging
1
assessment? B. Nail furrowing
A. Area of involvement C. Onycholysis
B. Degree of lichenification D. Pitting
C. Severity of joint involvement E. Transverse ridging

downloaded from www.medicalbr.com


352 • DERMATOLOGY

29.30. A 35 year old woman presents with a C. lmiquimod


6-month history of sudden onset of hair loss. D. lngenol mebutate
On examination there is complete loss of scalp E. No treatment
hair, eyebrows, eyelashes and hair at all other
body sites. There is no evidence of scalp 29.33. A 21 year old woman presents following
inflammation, scarring or regrowth. She is an episode of lip, tongue and facial swelling,
otherwise well, although she has a history of occurring within minutes of blowing up a
vitiligo and treated hypothyroidism. balloon. The episode resolved within ·an hour of
Which of the following management taking a non-sedating antihistamine. The patient
approaches would be most appropriate? asks whether this can be related to the rubber
A. Discuss consideration of a wig balloon and is keen to know what the most
B. Finasteride useful method of investigation would be?
C. lntralesional corticosteroids A. Oral challenge
D. Topical minoxidil B. Patch testing
E. UVB phototherapy C. Prick testing
D. Specific lgE testing
29.31. A 42 year old man with chronic plaque E. Specific lgG testing
psoriasis has been managed with methotrexate,
with partial control of his skin disease, for the 29.34. The skin changes as people age and
last 5 years. He had previously failed to there are many important differences between
respond to UVB and PUVA He is otherwise young and old skin. Which of the following
well, although he has a history of hypertension statements relating to the skin of the elderly is
and is a smoker. He has increasingly had correct?
problems with pain and swelling of joints in the A. Absorption and clearance of topical .
hands and feet and in the sacroiliac joints. On medications is reduced
examination he has extensive plaque psoriasis B. Photo-ageing is accelerated intrinsic ageing
and evidence of synovitis and dactylitis in due to sunlight exposure
fingers and toes. The PASI is 15 and C. Skin cancer is more common because of
Dermatology Life Quality Index (DLOI) 25, increased immune reactions in the skin
despite methotrexate 25 mg/week D. The skin thickens as it ages
subcutaneously. E. There is less risk of developing irritant
What would be the most appropriate next dermatitis
treatment approach?
A. Switch to adalimumab 29.35. A 62 year old man presents with a
B. Switch to apremilast 6-week history of itchy blistering rash occurring
C. Switch to ciclosporin on the trunk and limbs (see below). He is
D. Switch to fumaric acid esters otherwise in good health apart from controlled

I E. Switch to PUVA plus acitretin hypertension. His medications are ramipril and
aspirin. On examination he has several tense,

~' 29.32. A 93 year old woman is noted to have


two lesions on the lower legs. The lesions have
clear and haemorrhagic-fluid filled blisters
arising on erythematous plaques on the trunk
been unchanged and asymptomatic for 2 years and limbs. A diagnosis of bullous pemphigoid is
and don't trouble her. Her daughter brought suspected.
her to clinic for an opinion. On examination she
has two well-defined erythematous scaly
plaques on the lower legs, with dermatoscopic
features consistent with Bowen's disease. She
additionally has pitting ankle oedema, varicose
veins and absent foot pulses. Her daughter
asks for advice as to what would be the most
appropriate treatment option.
Which of the following is most appropriate?
A. Cryotherapy
B. Excision

downloaded from www.medicalbr.com


DERMATOLOGY • 353

Which of the following would you expect B. Oxygen is required for the photodynamic
to find? reaction
A. Mucosal involvement C. PDT should not be used for nodular BCC
B. Neutrophilia D. PDT should not be used in elderly frail
C. Positive circulating anti-epidermal antibodies patients
D. Positive Nikolsky sign E. Red laser light is required for irradiation
E. Subcomeal blister on histology during PDT

29.38. There has been a rising epidemic of skin


29.36. An 85 year old woman presents with a cancer in white-skinned populations. Which of
6-month history of generalised itch, but no the following statements is correct?
rash. Investigations do not identify a cause and
A. Malignant melanoma usually arises on sites
symptomatic management is offered. Which is
of chronic sun exposure
likely to be the most effective management
B. Most patients with sporadic BCC carry a
option?
germline mutation in the PTCH1 gene
A. High-dose tricyclic antidepressants C. There is a greatly increased risk of BCC in
B. High-dose antihistamines immunosuppressed patients
C. Topical capsaicin D. There is convincing evidence that sunscreen
D. UVB phototherapy use lowers risk of BCC development
E. Very potent topical glucocorticoids E. There is marked genetic heterogeneity in
sec
29.37. A 45 year old man attends clinic for
topical photodynamic therapy (PDT) for a 29.39. Actinic keratoses are extremely common
superficial BCC on his leg (see below). He is in white-skinned populations and occur as
keen to ask about how the treatment works. scaly erythematous lesions on chronically
PDT is increasingly used to treat superficial sun-exposed sites (see below). Histological
non-melanoma skin cancer. Which of the evidence of dysplasia is the hallmark and
following statements is correct? progression to Bowen's disease and/or invasive
sec may occur. Which of the following
statements is correct?

A. A cream containing a photosensitiser is


applied

downloaded from www.medicalbr.com


354 • DERMATOLOGY
T
I
I
A. Approximately 3% of actinic keratosis What is the most likely diagnosis?
transform into invasive sec A. Epidermolysis bullosa
B. Field-directed treatment is typically required B. Impetigo
C. Hyperkeratotic actinic keratosis usually C. Staphylococcal scalded skin syndrome
responds to cryotherapy (SSSS)
D. Invasive SCC invariably arises from actinic D. Stevens-Johnson syndrome
keratosis E. Toxic epidermal necrolysis
E. Spontaneous resolution of actinic keratosis
may occur

29.40. Which of the following statements is 29.42. A 32 year old man with chronic plaque
correct with respect to malignant melanoma? psoriasis has been attending for UVB
A. Acral melanoma is less common in phototherapy. He was starting to respond to
dark-skinned populations treatment but in the second week commented
B. Lentigo maligna melanoma usually occurs in that he had developed a new asymptomatic
younger patients rash on the back and chest. On examination,
C. Most patients with melanoma have a positive in addition to chronic plaque psoriasis affecting
family history of melanoma extensor surfaces, sacral area and buttocks,
D. Nodular melanoma is most common in a rash is evident on the upper trunk, consisting
men of oval scaly macules and hypopigmentation
E. The majority of melanomas arise from a (see image). What is the most likely
pre-existing naevus diagnosis?

29.41. A 3 year old boy presents with a 2-day


history of tense blisters and superficial erosions
arising on a background of erythema affecting
the trunk and face (see below). He has been
off his food and is febrile. On examination,
extensive superficial erosions are evident in the
groins and affected large areas on the trunk.
There is no mucosal involvement.

I(:
·I
~,
I

A. Pityriasis rosea
B. Pityriasis versic;olor I

C. Polymorphic light eruption


D. Psoriasis
E. Secondary syphilis I
downloaded from www.medicalbr.com
j
DERMATOLOGY • 355

29.43. A 35 year old woman returns to clinic for history of relevance and she is not known to be
management of chronic atopic eczema (see atopic. What is the most likely diagnosis?
image). She has a life-long history of eczema A. Allergic contact dermatitis
with whole-body and facial activity over the B. Dermatophyte fungal infection
last 5 years, despite topical emollients, C. Irritant contact dermatitis
glucocorticoids, phototherapy and PUVA. She D. Late-onset atopic dermatitis
is otherwise in good health, works in an office E. Progesterone dermatitis
and does not smoke or drink. She needs a
considerable amount of time off work because
of her skin, which is adversely impacting on her 29.46. A 14 year old boy attends the paediatric
life, with a DLOI score of 24. What would be dermatology clinic with his mother who is
the next most appropriate management step to seeking a second opinion for treatment of
consider? long-standing chronic atopic eczema. On
examination he has extensive eczema affecting
the trunk and flexor and extensor surfaces of
the limbs, with chronic inflammation and
lichenification (see below). He is using
Eumovate ointment daily, approximately 100 g
per fortnight to the trunk and limbs, emulsifying
ointment as emollient and fexofenadine at night
to help with itch. Which of the following
changes to his management is likely to be most
effective?

A. Acitretin
B. Apremilast
C. Ciclosporin
D. Dupilumab
E. Methotrexate

29.44. Early clinical trial data on biological


agents for severe recalcitrant atopic dermatitis
look promising. Which of the following
pathways is implicated for therapeutic response
to biological agents in atopic dermatitis?
A. IL-12 inhibition

-I
B. IL-13 inhibition
C. IL-17 inhibition
D. IL-23 inhibition
A. Changing from emulsifying ointment to
E. TNF-a inhibition
aqueous cream
B. Changing vehicle formulation to Eumovate
29.45. A 28 year old mother of 6 month old cream
twins presents with a rash on the back of C. Increasing the amount of topical
hands and between the fingers. Examination glucocorticoid use I
reveals erythema and scaling at these sites, D. lnc'reasing the dose of fexofer:Jadine
with bilateral involvement and a relatively sharp E. Increasing the potency of topical I
cut -off at the wrists. There is no previous glucocorticoid, e.g. to Betnovate ointment

downloaded from www.medicalbr.com


356 • DERMATOLOGY

29.47. A 72 year old man with previously A. Aspirin


stable chronic plaque psoriasis presents with B. Bendroflumethiazide
extensive erythroderma and sheets of pustules C. Doxycycline
affecting the trunk, limbs and flexures (see D. Furosemide
below). He has a history of chronic obstructive E. Ranitidine
pulmonary disease and has recently completed
a course of prednisolone for a respiratory
exacerbation. Topical treatment with a vitamin
29.49. A 29 year old man attends a consultation
D agonist to the skin has been ineffective.
for advice with respect to further management
Which of the following statements is correct?
of chronic urticaria that has troubled him for 6
months. There are no known triggers
and investigations are normal. He has
not responded well to H1 antihistamines.
Which of the following treatment options
has been shown to be effective in this
setting?
A. Montelukast
B. Narrowband UVB phototherapy
C. Oral prednisolone
D. PUVA photochemotherapy
E. Ranitidine

29.50. An 85 year old woman presents with a


6-week history of severe oral ulceration. On
A. Coal tar and UVB phototherapy should be examination it is also noted that she has an
commenced extensive erythematous rash on the neck and
B. He should be admitted and treated with upper trunk, with erosions evident and Nikolsky
topical dithranol sign positive. Skin biopsy is undertaken and
C. Methotrexate would be the systemic shows intra-epidermal blistering, acantholysis
immunosuppressant of choice because of and lgG and C3 deposition within the epidermis
rapid onset of action on immunofluorescence. Which of the following
D. The flare of pustular psoriasis may be related statements is correct?
to prednisolone use A. Long-term treatment is often
E. Vitamin D analogues are the topical required
treatment of choice for pustular psoriasis B. Low-dose systemic glucocorticoids are
usually effective
29.48. A 49 year old man presents with an itchy C. Positive Nikolsky sign usually indicates a
violaceous papular rash affecting the flexural drug-induced cause
surfaces of forearms and shins. Skin biopsy D. The disease can be treated with
shows a lichenoid reaction pattern and a drug penicillamine
cause is considered. Which of the following E. Underlying malignancy is found in most
drugs is the most likely to be the culprit? cases

29.51. A 27 year old woman presents with


progressive symmetrical depigmentation
affecting the face, hands and feet (see below).
There is no history of preceding inflammation
and no altered sensation within the areas of
hypopigmentation. A diagnosis of vitiligo is
established. Which of the following is
associated with an improved prognosis for
repigmentation in vitiligo?

downloaded from www.medicalbr.com


DERMATOLOGY • 357

she has multiple erythematous annular targetoid


lesions. She is otherwise well, with no
preceding medical history. A diagnosis of
erythema multiforme is suspected. Which of the
following is most likely to have provoked this?

A. Associated hypothyroidism
B. Facial involvement
C. Involvement of the distal limbs A. Adenovirus
D. The presence of a trichrome pattern B. Herpes simplex virus
E. The presence of leucotrichia C. Ibuprofen
D. Paracetamol
29.52. A 53 year old woman with a known E. Oral contraceptive
diagnosis of hepatitis C attends a general
medical clinic for routine review. On enquiry 29.54. A 66 year old woman was commenced
she comments that she has been aware of on carbamazepine and co-codamol 10 days
blistering occurring on the backs of her hands, previously for trigeminal neuralgia. She was
in particular when she had minor trauma to the otherwise well, having just returned from a
skin. On examination there is nothing much to holiday in the Caribbean. She presents to
see other than mild scarring and milia. What out-of-hours primary care with an acute onset
would be the most appropriate course of action? of extensive rash, systemic malaise and high
A. Check renal function, as this may indicate fever. Examination reveals a widespread
renal failure erythematous maculopapular and purpuric
B. Patch testing, as this may be allergic contact rash, with prominent facial involvement and
dermatitis facial oedema. She has generalised
C. Porphyrin investigations, as this may be a lymphadenopathy and a temperature of 40°C.
cutaneous porphyria Initial investigations show an eosinophilia and
D. Reassurance that this is likely due to the marked elevation of liver function tests. She is
hepatitis C infection admitted to an infectious disease department.
E. Skin biopsy and direct immunofluorescence What would be the most likely diagnosis?
to exclude immunobullous disease A. Carbamazepine-induced drug reaction
B. Co-codamol-induced drug reaction

-
29.53. A 25 year old woman presents to her C. Hepatitis
family physician with a 2-day history of a rash D. Leptospirosis
on her hands and forearms. On examination E. Meningococcal sepsis

Answers
I
I

29.1. Answer: D. sun exposure and lasting. a few days before


This immunological photoderrnatosis occurs in resolving. In contrast, solar urticaria will almost
18% of the population in Northern Europe. It invariably occur within 15 minutes of sun
presents as a delayed-onset papulovesicular exposure and will last only a few hours before
eruption, typically occurring a few hours after resolving. Chronic actinic dermatitis can occur

downloaded from www.medicalbr.com


358 • DERMATOLOGY
f
I

at any age but typically presents in elderly but liver function tests would be most important
males and the morphology of rash is a diagnostically. In the absence of other relevant
dermatitis. It usually takes a few hours to days history, there would be no specific indication to
of sun exposure to develop and it persists check ANA and complement.
until treated. Lupus erythematosus is a
photo-aggravated autoimmune disorder and 29.4. Answer: C.
the skin features more typically develop a day, The scenario of the relatively recent introduction
or so, after sun exposure and persist for of compression bandaging a few weeks· earlier
weeks. Erythema multiforme is a raises the po.ssibility of allergic contact
photo-aggravated disease often triggered by dermatitis. Specifically, rubber additives or
herpes simplex virus infection and can be most preservatives in any of the topical preparations
prominent on sun-exposed sites although could be culprits. All of the other diagnoses
usually affects sun-protected sites as well. The should, of course, be considered and excluded
rash is usually more targetoid and less papular. but a bilateral symmetrical presentation of each
of these diagnoses would be extremely unlikely.
29.2. Answer: E.
Most drugs, including doxycycline, 29.5. Answer: B.
photosensitise maximally in the UVA region The Nikolsky sign is when gentle lateral
and this would usually be detected on pressure on stroking the skin results in
monochromator phototesting. Patch testing is epidermal detachment. Carbamazepine is a
not the investigation of choice for suspected drug that is associated with many cutaneous
systemic drug-induced photosensitivity. Both adverse effects and is one of the most
false-positive and false-negative results mean common culprits in TEN. Evidence relating to
that patch testing using topical delivery of a the use of IVIg in TEN is controversial at best
drug that has been used systemically is an and overall not advised (see British Association
unreliable investigation. It is usually not of Dermatology guidelines). The symptom of
indicated nor of clinical relevance. Positive ANA dysuria is much more likely to be due to
and ENA autoantibodies can be seen with inflammation and desquamation of the
some systemic drug photosensitisers: for uroepithelial tract due to involvement in the
example, thiazides or proton pump inhibitors. TEN process. Catheterisation should be
However, this is not typically the case with avoided unless necessary, such as for
doxycycline and, indeed, the most common monitoring fluid balance - and in that instance
presentation of drug-induced photosensitivity is should be performed with caution. A
through a phototoxic non-immunological mid-stream urine should be undertaken, but
mechanism. UVB provocation testing may be antibiotics should not be prescribed empirically.
positive in drug-induced photosensitivity but is Skin pain is a characteristic feature of TEN and
much less likely to be abnormal than UVA the prognosis is better for patients who are less
phototesting, which is the main part of the than 40 years of age, although other prognostic
ultraviolet spectrum implicated in drug-induced indicators need to be taken into account when
photosensitivity. Whilst some drugs can cause assessing the disease severity score
minor derangements of porphyrins, this is not (SCORTEN), which is predictive of risk of
the case with doxycycline and one would not mortality (Box 29.5).
expect abnormal porphyrins in doxycycline-
induced photosensitivity. 29.6. Answer: B.
Photodynamic therapy is approved for use in
29.3. Answer: C. Bowen's disease. Given its relative specificity of
Acute cholestasis of pregnancy is uncommon treatment and improved healing compared with
but usually presents in the third trimester of other treatments such· as cryotherapy and
pregnancy. It is essential to diagnose this 5-f\uorouracil, this would be the treatment of
promptly as there is increased fetal and choice on a lower leg site where there is
maternal risk; urgent diagnosis and treatment, coexistent oedema and vascular insufficiency,
which may include early delivery, are required. either venous and/oF arterial. Definitive surgical
The other investigations of full blood count, excision would be an option but in this instance
urinalysis and thyroid function tests would all be a non-surgical approach would usually be
important to undertake in this clinical setting advised in order to reduce morbidity, given the

downloaded from www.medicalbr.com


DERMATOLOGY • 359

' may occur. Indeed, most cases of


29.5 Disease severity score for toxic epidermal
necrolysis: SGORTEN photo-aggravation of psoriasis are through the
mechanism of induced PLE. If PLE develops
Factor
Age > 40 years
during a course of phototherapy, then it is
Heart rate > 120 beats/min advisable to stop treatment until the rash has
Cancer or haematological malignancy settled and then to restart at a lower dose, with
Involved body surface area > 10% reduced dose increments and, if necessary, a
Blood urea > 10 mmoi/L (28 mg/dl)
topical corticosteroid application after
Serum bicarbonate < 20 mmoi/L (20 mEq/L)
Blood glucose~ 14 mmoi/L (252 mg/dl) treatment. Systemic corticosteroids are
Mortality rates uncommonly required for PLE occurring during
0-1 factor present = 3% a course of phototherapy and might cause a
2 factors = 12% rebound flare of psoriasis on cessation.
3 factors = 35%
4 factors = 58%
~5 factors = 90%
29.9. Answer: G.
The image shows a well-defined, nodular basal
From Bastufi-Garin S, Fouchard N, Bertocchi M, eta!.
SCORTEN: a severity-of-illness score for toxic epidermal cell carcinoma (BCC). If he is agreeable to
necrolysis. J Invest Oermato/2000; 115:149-153. surgery, then the most appropriate approach
would be excision of the lesion. Mohs'
very low risk of malignant transformation. micrographic surgery would only be required if
Cryosurgery to a lesion of this size, on an the lesion is poorly defined, in order to ensure
oedematous lower leg site with venous complete excision and preservation of normal
insufficiency, would be associated with a higher tissue.
risk of poor healing and ulceration. Radiotherapy Warfarin does not preclude a surgical
would not usually be used on a lower leg site approach as long as the international
with these coexistent morbidities as the risk of normalised ratio (INR) is checked 48 hours
radionecrosis, scarring and ulceration are before the procedure in order to ensure the
relatively high. lngenol mebutate is not licensed patient is not over-anticoagulated and as long
or approved for use in Bowen's disease. as additional precautionary measures are taken
at the time of surgery in order to ensure
29.7. Answer: D. haemostasis. Medical treatments, including
The first step in this situation would be an initial photodynamic therapy, imiquimod and ingenol
excision with a narrow margin in order to mebutate, would be unlikely to be effective with
establish the diagnosis. Thereafter, the next this prominently nodular BCC. Medical
step would be to re-excise with wider margins treatments are best reserved for either
(1-2 em) once the diagnosis of melanoma was superficial BCC or thin, nodular BCC, and
confirmed, and dependent on the Breslow would only be considered for a lesion such as
thickness. An initial wide excision would not this if surgery was contraindicated or the
usually be advocated in case the clinical patient refused to consider a surgical approach.
diagnosis was incorrect. lncisional biopsy is not
advised when melanoma is suspected if it can 29.10. Answer: D.
be readily excised. Curettage would also not be In the first instance, as a trial, lymecycline
recommended if melanoma was suspected. would be an appropriate next step.
Erythromycin would be an alternative but the
29.8. Answer: B. dose of 250 mg daily is sub-therapeutic and

-I
There is no evidence that PLE is more common would be unlikely to be effective. Minocycline is
either in patients taking the contraceptive pill not the first antibiotic of choice given the risk of
or in psoriasis. The prevalence of PLE is skin pigmentation and of drug-induced lupus.
approximately 18% in Northern Europe and, as Antibiotics will need to be continued for several
it most commonly occurs in young females of months, and a trial of at least 3 months is
child-bearing age, the contraceptive pill and required. Combined oestrogen/anti-androgen
PLE are commonly associated but there is no contraceptives, such as those including
evidence to indicate a causal relationship. PLE cyproterone acetate, may be appropriate but I
does commonly occur during a course of would usually only be consider(ld or added in if
phototherapy and, if this is the case, there there was an inadequate response to a trii:ll of
is concern that Ki:ibnerisation of psoriasis systemic antibiotics. lsotretinoin would not be

downloaded from www.medicalbr.com


+'
I
360 • DERMATOLOGY

considered at this early stage in management regress. However, given that spontaneous
of a patient with papulopustular acne. The resolution of keratoacanthoma often leaves
hope would be that this case would respond cosmetically unacceptable scars and that it is
well to systemic antibiotics; systemic retinoids impossible to distinguish from invasive sec,
would only be required if there was a failure to active intervention and removal is important. An
respond to 3-6 months of antibiotic treatment. incisional biopsy may not clearly distinguish
between keratoacanthoma and invasive SCC:
29.11. Answer: E. the distinction often remains difficult; thus,
Scalp infestation with the head louse Pediculus usually these lesions are definitively excised. A
humanus capitis is very common. A diagnosis nodular lesion such as this would be unlikely to
is confirmed by identifying a living louse or respond to treatment with either topical
nymph. However, the 'nits' are actually empty imiquimod or photodynamic therapy.
egg cases, not the head lice themselves, and
are signs of there having been an infestation. 29.14. Answer: B.
'Nits' are yellowish in colour and can be Lentigo maligna may be very difficult to treat and
confused with dandruff. Not all cases require to achieve clinical and histological clearance. It is
treatment with insecticide, as regular wet not unusual for clinical response to occur but
combing of conditioned hair may be effective in abnormal cells to remain histologically.
physical removal of lice. Malathion would not Treatment of choice would be definitive surgical
be the insecticide of choice as resistance is excision although this can be difficult as
fairly common; alternatives such as dimeticone dysplastic cells often persist at the margins,
may be used. Whilst the infestation is highly ~rising as part of field change carcinogenesis.
contagious, treatment is only recommended for lmiquimod may be used if surgery is not
the affected individual and close contacts, such appropriate but the risk of recurrence is higher.
as family members or close school class Left untreated, there is a significant risk of
members where there has been direct invasive melanoma developing into lentigo
head-to-head contact. Treatment of all maligna. Pigmented lesions with metastatic
classmates is not required. potential would not be appropriately treated by
photodynamic therapy, and melanin absorbs red
29.12. Answer: G. light; thus efficacy would not be expected.
Fluorescence with Wood's light is only seen with
some species of dermatophyte infection and not 29.15. Answer: G.
with those involving the endothrix (within the hair Whilst some consider palmoplantar pustulosis
shaft). Dermatophyte infection of the scalp hair to be a variant of psoriasis, most patients with
affects the shaft as opposed to the bulb. Topical this condition do not have other features of
treatment will not be sufficient for clearance of chronic plaque psoriasis and there is increasing
inflammatory active fungal infection within the evidence to suggest that the two conditions are
hair-bearing scalp. Griseofulvin is the only distinct. Palmoplantar pustulosis is almost
systemic antifungal agent licensed for use in invariably associated with smoking but there is
children in the UK. There is no convincing no convincing evidence that stopping smoking
evidence that systemic glucocorticoids reduce results in disease improvement. Topical
hair loss associated with tinea capitis. glucocorticoids are usually a mainstay
treatment in palmoplantar pustulosis. Bacterial
29.13. Answer: G. swabs from the pustules are usually sterile.
From the history, this lesion is most likely a PUVA may be effective for disease suppression
benign keratoacanthoma, but it is impossible to in this condition.
distinguish this from a rapidly growing
squamous cell carcinoma (SCC). Given her 29.16. Answer: G.
history of significant sun exposure and previous Clustered painful vesicles recurring at the same
BCC, it would be important to excise in order site in association with the pre-menstrual period
to exclude invasive SCC. Mohs' micrographic are most likely to be extra-labial herpes simplex
surgery would not usually be required for this virus infection. Pain and clustered vesicles would
well-defined tumour and is most commonly not be expected in fixed drug eruption, although
used, in particular, for poorly defined BCC. this should certainly be something to be
Observation is an option, because if this is a considered, particularly if the patient is taking
benign keratoacanthoma, then it should paracetamol or non-steroidal anti-inflammatory

downloaded from www.medicalbr.com


DERMATOLOGY • 361

medication in the pre-menstrual phase. In tinea with chronic urticaria do not have an obvious
corporis or dermatophyte fungal infection, the trigger and total lgE and specific lgE testing are
most likely presentation would be a raised edge unlikely to be helpful in the absence of a history
with pustules and scaling and central clearing, suggestive of trigger factors. Prick testing would,
and it would be unlikely to clear and recur at the again, be unlikely to be contributory unless there
same site each month or be painful. In was a specific trigger identified in the history.
molluscum contagiosum, whilst these are due to
a pox virus infection, the lesions are not vesicular 29.22. Answer: D.
but are usually solid umbilicated papules and The condition is usually treated with topical
they are not usually painful or intermittent. Whilst glucocorticoids, which can be safely prescribed
an acute vesicular eczema can occur in in pregnancy. The condition is not known to be
association with contact allergy, the intermittent associated with any adverse effects to the fetus
and isolated nature of this would make this and early delivery is not generally required.
diagnosis unlikely. Abnormal liver function tests are not associated
with polymorphic eruption of pregnancy.
29.17. Answer: C. Sedating antihistamines, such as
A relatively common side-effect of chlorphenamine, may be required but as this is
angiotensin-converting enzyme (ACE) inhibitors not primarily a histamine-mediated disease,
is angioedema and this usually occurs without non-sedating antihistamines are not advised as
associated urticaria. Thiazide diuretics do not their safety in pregnancy is unproven.
typically cause angioedema. Patch testing is Polymorphic eruption of pregnancy usually
used to investigate type IV delayed persists until delivery and may even continue
hypersensitivity and not type I immunological for some time into the post-partum period
reactions. There is no evidence that angioedema before spontaneous resolution. It does not
is increased in patients with diabetes. usually occur in subsequent pregnancies.

29.18. Answer: C. 29.23. Answer: A.


Langerhans' cells have the primary function of Breslow thickness is the most important
antigen presentation to lymphocytes. prognostic factor. For a tumour less than 1 mm
in histological thickness, routine CT scanning
29.19. Answer: E. and sentinel node biopsy would not be
The tonofilaments mainly consist of keratins 5 indicated. Prognosis should be around 95%
and 14. The lamina Iucida lies immediately below disease-free survival at 5 years. Wide local
the basal cell membrane. ·The lamina densa is excision with a 1-cm margin is advised. For a
made up mainly of type IV collagen. The main good prognosis in melanoma such as this,
hemi-desmosomal collagen is type XVII. The long-term follow-up is not required.
anchoring fibrils consist of type VII collagen.
29.24. Answer: C.
29.20. Answer: D. This history is very suggestive of a diagnosis of
The main cell type involved in vitamin D porphyria cutanea tarda, and this would be an
photosynthesis in skin is the keratinocyte. UVB easy screening test. If this proved positive, then
is required for this. UVA exposure does not more detailed investigations would be required
result in adequate cutaneous vitamin D including urine porphyrins and investigating for
production. Dietary absorption of vitamin D is an underlying cause of iron overload and liver

-I
poor and vitamin D and calcium supplements disease. In this case it may be due to
are required in vitamin D deficiency. Vitamin D alcohol-induced liver disease. Skin biopsy and
deficiency is a potential concern for patients immunofluorescence may show characteristic
with photosensitivity diseases. changes of subepidermal blistering and periodic
acid-Schiff (PAS) staining but would not be the
29.21. Answer: D. investigation of choice. Patch testing, urinalysis
This patient is atopic and has autoimmune and lupus serology would not be specifically
disease. Urticaria may be a manifestation of indicated. I
autoimmune hypo- or hyperthyroidism. Patch
testing is the investigation of choice for delayed 29.25. Answer: D. I
type IV cell-mediated hypersensitivity but not Acitretin has a long half-life and high lipid
type I antibody-mediated allergy. Most patients bioavailability; pregnancy should be avoided for

downloaded from www.medicalbr.com


i'
362 • DERMATOLOGY

3 years after the drug has been stopped, which 29.31. Answer: A.
is why it is not often used in women of As tumour necrosis factor alpha (TNF-a)
child-bearing age. Acitretin is at least as antagonists have efficacy both in psoriasis and
teratogenic as isotretinoin and the effect is of psoriatic arthritis, adalimumab would be the
longer duration. most appropriate next treatment approach to
consider. Ciclosporin could only be used
29.26. Answer: A. short-term and other treatment options would
The body is divided into four areas and each is not be effective for psoriatic arthritis.
scored individually based on area involved and
the redness, thickness and scaling of psoriatic 29.32. Answer: E.
plaques. Joint and nail involvement are not Given that this patient is 93 years old and has
assessed and neither is the type of psoriasis. other comorbidities, active treatment may result
Lichenification is taken into account in eczema in ulceration and poor healing. Given that these
severity scores. lesions have not significantly changed and are
asymptomatic and not bothering her, the most
29.27. Answer: B. appropriate treatment option would likely be to
The autoimmune bullous diseases are usually
leave these untreated, as the risk of significant
exacerbated by light-based therapies and change and development of invasive sec is
should be avoided. All of the other diseases very low (approximately 3%). This case
may respond therapeutically to PUVA. demonstrates how every patient must be
individually assessed in order to ascertain what
29.28. Answer: B.
i~ most appropriate in any clinical scenario.
The image is of a BCC. Mohs' surgery, if
available, would be preferred to excisional
29.33. Answer: D.
surgery at this site given the need for
Specific lgE testing is also known as the
preservation of normal tissue and structures.
radioallergosorbent test (RAST) and could be
Curettage and cautery may lead to
helpful in this situation, which is most likely to
unacceptable scarring at this site and is less
have been caused by type I latex rubber
likely to result in tumour clearance. Topical
allergy. Patch testing is used to investigate
imiquimod may cause significant inflammation,
delayed type IV cell-mediated hypersensitivity.
blepharitis and conjunctivitis at this site.
Prick testing may be positive and helpful in type
Photodynamic therapy, given the nodular
I allergy but is risky to undertake in a patient
nature of the tumour, would be unlikely to
who has already experienced angioedema as
result in complete clearance. Radiotherapy
this may trigger anaphylaxis and should not be
would be likely to result in poorer cosmetic
undertaken without anaesthetic support
outcome and risk damage to the medial
available. Thus, the specific lgE test would be a
canthus and lacrimal duct.
safe initial investigation. It is important to be
29.29. Answer: A. aware that both specific lgE and prick testing
Longitudinal ridging can be a normal part of the can be falsely positive or falsely negative. If
ageing process. Pitting, onycholysis, transverse specific lgE testing is negative but clinical
ridging and nail furrowing are associated with suspicion is high, further investigations with
pathological processes. prick testing in a controlled situation could be
indicated in order to try and clarify the
29.30. Answer: A. diagnosis. lgG antibodies are not elevated in
lntralesional corticosteroids are unlikely to be type I hypersensitivity reactions as the
effective in sudden-onset alopecia universalis mechanism is mediated via lgE immediate
;j and is mainly indicated for patchy alopecia antibody reactions. Oral challenge would not be
areata. There is no evidence for efficacy of appropriate and would, again, carry an
topical minoxidil, finasteride or UVB unnecessary risk of anaphylaxis.
phototherapy in the treatment of alopecia
areata. The most appropriate option is 29.34. Answer: A.
psychological support and discussion of As people get older ~here is reduced absorption
realistic expectations for hair regrowth, which and clearance of topical medications. Skin
may include whether she wishes to consider immune reactions are reduced with ageing.
use of a wig, given the complete alopecia. Photo-ageing is a different process to intrinsic

downloaded from www.medicalbr.com


DERMATOLOGY • 363

ageing but is superimposed on intrinsic ageing. evidence to suggest that regular sunscreen use
There is increased susceptibility to irritants and reduces the risk of actinic keratosis (AK) and
irritant dermatitis. The skin becomes thin and sec, although whilst assumed that it will
atrophic with ageing. reduce BCC risk, there is no good evidence to
support this. In most BCCs, the PTCH1 gene
29.35. Answer: C. mutations are somatic and not germline. SCC
Circulating anti-epidermal antibodies may be is a highly genetically heterogeneous tumour.
present in bullous pemphigoid. In this condition,
the split is below the basement membrane and 29.39. Answer: E.
therefore the subepidermal blisters are tense and Invasive SCC may arise de novo or from the
intact. Nikolsky sign is thus negative. Mucous background of AK. The risk of transformation of
membrane involvement is uncommon in bullous AK into invasive SCC is < 1%. A field-directed
pemphigoid. Eosinophilia is usually evident. approach, such as with 5-fluorouracil, PDT or
imiquimod, is required for multiple AK and
29.36. Answer: D. field-change carcinogenesis. Isolated lesions
Great caution needs to be taken with high may be treated with a lesion-directed approach
doses of antihistamines in elderly patients such as cryotherapy. Hyperkeratotic AK usually
because of the risk of over-sedation, delirium does not respond well to cryotherapy, and
and falls. Likewise, low-dose tricyclic curettage and cautery and preparations
antidepressants may be considered but a containing salicylic acid in combination with
high-dose approach would not be advisable. 5-fluorourcil are usually required. Spontaneous
In the absence of rash, very potent topical resolution of AK may occur.
glucocorticoids would be unlikely to be of
therapeutic benefit and, in the elderly, adverse 29.40. Answer: D.
effects of striae and purpura may occur. Most patients with melanoma do not have a
Topical capsaicin would be unlikely to be of positive family history for melanoma and
benefit for generalised pruritus as it can only be approximately 50% of melanomas arise from
applied to localised areas. pre-existing naevus. Lentigo maligna melanoma
tends to occur in the elderly, and acral
29.37. Answer: B. melanoma is more common in dark-skinned
Oxygen is required for the photodynamic populations. Nodular melanoma is more
therapy effect. The cream contains a common in men.
photosensitiser prodrug and not the
photosensitiser itself as the prodrug needs to 29.41. Answer: C.
be taken up and converted to the The description of the rash, the age of onset,
photosensitiser in the skin cells. Laser light is associated systemic features of irritability, fever,
not required for irradiation and most extensive areas of rash with erosions and
dermatological PDT is undertaken using blistering, and systemic upset are most in
broadband and light-emitting diode (LED) light keeping with staphylococcal scalded skin
sources. Nodular BCC can be treated with syndrome. Toxic epidermal necrolysis is much
PDT, particularly if surgery is contraindicated. less likely in this age group and usually occurs in
However, recurrence rates at 5 years are higher association with drug ingestion. Stevens-
following PDT for nodular BCC than for surgical Johnson syndrome typically has mucosal
excision. PDT is often the most appropriate involvement, lesions are more targetoid and

-I
treatment choice for elderly frail patients. It can often there is a precipitant of herpes simplex
be used to treat large areas, on an outpatient virus infection. Epidermolysis bullosa is a
basis, without the need for surgery and with genetically inherited blistering disease, which
improved healing. does occur in children but is not associated with
systemic upset. Impetigo is a localised form of
29.38. Answer: E. superficial bacterial infection, usually due to
Malignant melanoma usually occurs on Staphylococcus aureus.
intermittently sun-exposed sites. The I
immunosuppressed patient population is most
at risk of sec, with only a slight increased risk
29.42. Answer: B.
The description of the rash and its distribution I
of BCC. There is good epidemiological are consistent with pityriasis versicolor and it is

downloaded from www.medicalbr.com


+
I

364 • DERMATOLOGY

likely that, as the patient has started to tan with dermatitis would be unusual, although not
phototherapy, the areas of hypopigmentation impossible at this age. The distribution, however,
have become more obvious, making him aware is more suggestive of external causes. The
of this second diagnosis. Pityriasis rosea would distribution would be unusual for dermatophyte
usually be more widespread, not just restricted fungal infection as this is usually unilateral.
to the central trunk, and would usually be Progesterone dermatitis is thought to be due to
associated with a herald patch. In addition, autoimmune sensitisation to progesterone and
lesions would be erythematous and not occurs cyclically with the menstrual cycle but
hypopigmented. Polymorphic light eruption this distribution would be unusual.
commonly occurs during phototherapy but is a
papulovesicular eruption. This is unlikely to be 29.46. Answer: E.
psoriasis because it is hypopigmented and also Moderate-strength topical glucocorticoid such
other sites of psoriasis are improving with as Eurnovate would usually not be sufficient to
phototherapy. Secondary syphilis should always gain adequate control of significant eczema
be considered with new development of activity on the trunk and limbs, and increasing
erythematous scaly rash, but lesions do not the potency of glucocorticoid to a potent agent
tend to be hypopigmented and are usually such as betamethasone 17 -valerate (Betnovate)
more prominently found on distal sites, would be likely to be more effective. Increasing
including palms and soles. the amount but not the potency would be
unlikely to suffice. For chronic lichenified
29.43. Answer: E. eczema, ointments are the preferred vehicle.
Methotrexate can be very effective for chronic Emulsifying ointment is a very good emollient
management of eczema, including atopic arid barrier for chronic lichenified eczema.
eczema. It would not be sensible to move to the Changing to aqueous cream, which can be
newer drugs - apremilast or the biological agent irritant, would not be advisable. Increasing the
dupilumab - without having a therapeutic trial of dose of fexofenadine would be unlikely to be
more conventional immunosuppressants such beneficial, as fexofenadine is a non-sedating
as methotrexate. Ciclosporin can be very antihistamine and, as atopic eczema is not
effective for clearing eczema; however, for primarily a histamine-mediated disease, the
chronic disease activity without acute flare, the beneficial effect of antihistamines would usually
use of a drug that can only be continued in the be via sedating antihistamines, in order to
short term would not be ideal. Acitretin would break the itch/scratch cycle at night.
not usually be used in a woman of child-bearing
age unless she had definitely completed her 29.47. Answer: D.
family, as there is a requirement for her to Pustular psoriasis can often be triggered as a
abstain from pregnancy for 3 years after rebound secondary to commencement and
cessation of drug because of teratogenicity. sudden cessation of systemic glucocorticoids.
It can also be triggered by topical use of
29.44. Answer: B. glucocorticoids and other irritants, which
Biological agents blocking and inhibiting include dithranol, coal tar and vitamin D
interleukin (IL)-4R and IL-13 are being trialled analogues. These should all be avoided in
for use in atopic dermatitis. TNF-a inhibition pustular psoriasis. Whilst PUVA light therapy
and inhibition of IL-12, IL-23 and IL-17 can often be effectively used in pustular
pathways by biological agents have been psoriasis, UVB, although highly effective for
shown to be effective in psoriasis. chronic plaque psoriasis, often causes further
flaring of unstable pustular disease. The effects
29.45. Answer: C. of methotrexate may take several weeks to
In the absence of a history of atopic dermatitis, become established as it does not have a rapid
bilateral dermatitis developing on the backs of onset of action.
hands and between the fingers in a young
woman on maternity leave and likely to have a 29.48. Answer: B.
lot of exposure to water and detergents is most There are many common culprits for
likely to be irritant contact dermatitis. Allergic drug-induced lichenoid reactions, which include
contact dermatitis is a possibility and if irritant gold, penicillamine, thiazides, [3-adrenoceptor
avoidance does not suffice then patch testing antagonists ([3-blockers), ACE inhibitors, proton
should be considered. Late-onset atopic pump inhibitors, non-steroidal anti-

downloaded from www.medicalbr.com


DERMATOLOGY • 365

inflammatories, antituberculous drugs, immunobullous disease such as bullous


sulphonamides, lithium and antimalarials. The pemphigoid would not be the investigation of
other drugs listed are not common culprits. choice in this clinical setting as porphyria
cutanea tarda should be suspected. There are
29.49. Answer: B. characteristic changes of porphyria cutanea
With narrowband UVB phototherapy there is tarda on histology but, if the porphyrin
randomised controlled trial evidence to show investigations are abnormal, then skin biopsy
efficacy in recalcitrant chronic urticaria. There is would not necessarily be indicated, although it
no conclusive evidence for the use of H2 would be confir.matory. Blistering can occur in
blockers, such as ranitidine, nor of montelukast, allergic contact dermatitis but blisters are
although both are widely used. There is no usually multilocular and milia and scarring are
randomised controlled trial evidence showing uncommon. Thus, patch testing would not be
efficacy with PUVA. the investigation of choice. In chronic renal
failure, patients can present with a porphyria
29.50. Answer: A. cutanea tarda-like presentation due to uraemia
Long-term treatment is often required as this and this may be associated with some
condition, pemphigus, is an autoimmune elevation of porphyrins due to impaired
blistering disorder that tends to last for many elimination. However, in a patient with
months or years; it requires long-term known underlying chronic liver disease, the
treatment to prevent disease relapse. The most likely diagnosis would be porphyria
condition is more difficult to treat than bullous cutanea tarda.
pemphigoid and higher doses of systemic
glucocorticoids are required. There may be 29.53. Answer: B.
underlying malignancy but not in most cases. Erythema multiforme may have multiple
Investigations should be with this in mind. triggers, which include infections and drugs.
Penicillamine may actually trigger pemphigus However, in many cases, a trigger is not
and a positive Nikolsky sign indicates that there identified. Where there is an evident provoking
is superficial intra-epidermal blistering but does factor, this is most usually herpes simplex virus
not necessarily indicate a drug-induced cause. infection.

29.51. Answer: D. 29.54. Answer: A.


The presence of a trichrome pattern, where The description of her presentation is very
normal skin colour, hypopigrnentation and much consistent with drug reaction and
depigmentation are present, is a good eosinophilia with systemic symptoms (DRESS).
prognostic factor. All of the others are poorer This diagnosis is often confused with an
prognostic factors. infectious cause and patients are not
uncommonly admitted to infectious diseases
29.52. Answer: C. units for investigation. High fever can occur due
The description of tense blistering, scarring and to drug hypersensitivity and can be misleading.
milia on photo-exposed sites of back of hands Facial oedema and lymphadenopathy, with
in a patient with underlying liver disease should eosinophilia and systemic involvement such as
strongly raise the suspicion of a diagnosis of hepatitis, are classical features of DRESS.
porphyria cutanea tarda, which is almost Carbamazepine is one of the most common
invariably associated with chronic liver disease culprits for DRESS. Co-codamol is not a typical
and iron overload. Examination findings are culprit. Patients are usually investigated for
often not striking if the disease is relatively underlying infection, including meningococcal
quiescent but the presence of scarring and
milia should raise suspicions. The screening
infection given the purpura, and other causes
of hepatitis. Overseas travel should always raise
~
investigation of choice would be porphyrin
plasma scan as a first step. The abnormal
porphyrins in porphyria cutanea tarda are water
soluble and thus biochemical analysis of urine
the possibility of an infectious cause. However,
these patients are often extensively investigated
for underlying infection because of high fever.
The diagnosis of DRESS ~s often not reached
I I
will also show raised porphyrins in urine.
Hepatitis C, per se, does not cause blistering
until a dermatological consultation is requested
or infection screening is negative, so this is a
I
and most patients with hepatitis C will not have diagnosis to be aware of.
porphyria cutanea tarda. Skin biopsy to exclude

downloaded from www.medicalbr.com


T

L Mackillop, F Neuberger

Maternal medicine
Multiple Choice Questions
30.1. A 25 year old woman with a 5-year history C. She has stopped her medication for fear of
of rheumatoid arthritis is planning her first teratogenicity
pregnancy. Which of the following drugs should D. She is anxious about the pregnancy and is
be avoided in pregnancy? having pseudoseizures
A. Azathioprine E, Sleep deprivation has caused a worsening of
B. Hydroxychloroquine her epilepsy
C. Methotrexate
D. Prednisolone 30.4. A 35 year old primiparous woman attends
E. Sulfasalazine the antenatal day unit at 18 weeks' gestation
with a history of vomiting, dysuria, left loin pain
30.2. A 40 year old woman is 6 weeks pregnant. and rigors for 24 hours. A urine dipstick is
She has a diagnosis of epilepsy. Which of the positive for leucocytes and nitrites. Her
following pieces of advice is correct? C-reactive protein is 140 mg/L. Which of the
following findings on examination and
A. Drug doses should routinely be doubled in
investigation would require urgent attention?
the second trimester
B. Pregnancy reduces the frequency of seizures A. A decrease in urea and creatinine values
C. She should start high-dose folic acid from her pre-pregnancy levels
D. She should stop anticonvulsant therapy B. A mild respiratory alkalosis on the arterial
E. Sodium valproate is the antiepileptic drug of blood gas
choice in pregnancy C. A raised alkaline phosphatase result
D. A respiratory rate of 24 breaths/min
30.3. An 18 year old woman is admitted to the E. The presence of a systolic murmur
gynaecology ward at 8 weeks' gestation in her
first pregnancy. She has a known diagnosis of 30.5. A 22 year old woman is 9 weeks into her
epilepsy and takes levetiracetam. Her epilepsy first pregnancy, and presents with vomiting.
is usually well controlled but she has had three Which of the following is a feature of
seizures in the last 2 days. Four of the answers hyperemesis gravidarum?
below need to be considered as possible A. Abdominal pain
contributing factors; however, one is unlikely. B. Hyper-reflexia
I Which of the following factors is LEAST likely to C. Lactate >2 mmoi/L (18.0 mg/dL)
have contributed to the increasing frequency of D. Vomiting intermittently
seizures? E. Weight loss >5%
A. Serum drug levels have reduced due to
increased renal clearance and increased 30.6. A 34 year old primiparous woman is
plasma volume admitted to hospital at 32 weeks' gestation
B. She has nausea and vomiting of pregnancy with central crushing chest pain, ST segment
and cannot keep her tablets down elevation on her electrocardiogram (EGG) and a

downloaded from www.medicalbr.com


MATERNAL MEDICINE • 367

troponin T of 840 ng/L. She has a background C. Intravenous magnesium sulphate is


of smoking a pack of cigarettes daily for contraindicated
18 years, essential hypertension and a family D. Nebulised salbutarnol can be given safely
history of ischaemic heart disease. Which E. Peak flow measurement is not helpful in
of the following is the appropriate pregnancy
management?
A. Aspirin 300 mg 30.10. A 36 year old woman with type 2
B. Aspirin, clopidogrel and fondaparinux diabetes is planning her first pregnancy. She
G. Primary percutaneous coronary intervention takes metforrnin 500 mg 3 times daily and her
(PPCI) HbA1c is 45 mrnol/mol (6.3%). Which of the
D. Therapeutic low-molecular-weight heparin following pieces of advice should she be given
E. Thrombolysis with alteplase regarding medication?
A. Double her metforrnin dose before she
30.7. A 19 year old woman is contemplating her conceives
first pregnancy. She has lupus nephritis, which B. Start aspirin 75 mg daily at 12 weeks'
is stable and takes azathioprine and tacrolimus. gestation
Which of the following pieces of advice should G. Start gliclazide 40 mg daily after she has
she be given? conceived
A. She is at increased risk of fetal growth D. Start subcutaneous insulin at 12 weeks'
restriction gestation
B. She is likely to be infertile E. Stop rnetformin after she has conceived
G. She should be warned that dialysis is
contraindicated in pregnancy 30.11. A 40 year old woman is examined at 32
D. She should start antihypertensive therapy weeks' gestation in the antenatal clinic. She
prior to conception has upper abdominal pain and pruritus. Which
E. She should stop her tacrolirnus of the following physical signs can be part of
normal pregnancy?
30.8. A 30 year old woman, who is normally A. Ascites
well, is 22 weeks into her second pregnancy B. Palmar pigmentation
and is seen on the acute medical unit with C. Spider naevi
cough wi_th green sputum for the last 3 days. D. Upper abdominal tenderness
Her respiratory rate is 12 breaths/min, oxygen E. Yellow sclerae
saturations 98% on air, blood pressure
110/76 mmHg, heart rate 90beats/min and 30.12. An 18 year old woman presents at 36
temperature 38.4°C. Her chest X-ray shows weeks' gestation with 24 hours of vomiting.
right middle lobe consolidation. The differential diagnosis includes viral
Which of the following is the most gastroenteritis, pre-eclampsia/haemolysis,
appropriate treatment? elevated liver enzymes, and low platelets
A. Amoxicillin 500 mg 3 times daily and (HELLP) and acute fatty liver of pregnancy.
clarithromycin 500 mg twice daily Which of the following would be in keeping with
B. Doxycycline 200 rng immediately, then a diagnosis of acute fatty liver of pregnancy?
100 rng daily thereafter A. Blood glucose 3.2 rnmoi/L (58 rng/dl)
C. Oseltamivir 75 mg twice daily for 5 days B. Creatinine 100 ~moi!L (1 .13 rng/dl)
D. Piperacillin/tazobactam (Tazocin) 4.5 g C. Oliguria
intravenously (IV) 3 times daily D. Platelets 85 x 109 /L
E. Trimethoprim 200 rng twice daily E. Tern perature 39°C

30.9. A 17 year old woman who is 30 weeks 30.13. A 25 year old woman with ulcerative colitis
into her first pregnancy is admitted to hospital is planning her first pregnancy, and attends clinic
with acute severe asthma. Which of the for pre-pJregnancy counselling. Which of the
following statements is TRUE? following pieces of advicejs TRUE?
A. Chest X-ray is generally avoided in this A. She should be advised against pregnancy
situation B. She should stop taking sulfasalazine during
B. Inhalers should be stopped in pregnancy the first trimester

downloaded from www.medicalbr.com


y:s
368 • MATERNAL MEDICINE

G. She should deliver by caesarean section irritability. On examination she has a fine
D. She should stop infliximab once she has tremor. What is the most likely diagnosis?
conceived A. Anxiety
E. She should stop taking methotrexate 3 B. Graves' disease
months prior to conception G. Hashimotos's thyroiditis
D. Post-partum depression
30.14. A 42 year old woman presents at 12 E. Post-partum thyroiditis
weeks post-partum with palpitations and

Answers
30.1. Answer: G. levetiracetam. Non-adherence is common
Methotrexate should be stopped 3 months due to concerns over teratogencity and
before pregnancy and throughout pregnancy non-reassurance or reticence to prescribe by
and breastfeeding. All other medications can be health-care professionals. Nausea and vomiting
taken during pregnancy and breastfeeding. is very common in early pregnancy and
Women taking sulfasalazine should also receive antiemetics may be used safely to allow regular
high-dose (5 mg daily) folic acid from medication to be given. All these factors need
pre-conception until at least 12 weeks' to be thought about and addressed to ensure
gestation. women feel confident and comfortable in
their decisions and their chronic condition
30.2. Answer: G. can be optimally managed in pregnancy.
Women with epilepsy should take high-dose Pseudoseizures in this scenario would be very
folic acid prior to conception and throughout uncommon.
the pregnancy. This is because women with
epilepsy who take antiepileptic drugs (AEDs) 30.4. Answer: D.
that induce cytochrome P450 (for example Pregnancy does not cause a significant
phenytoin, carbamazepine) are at risk of low increase in respiratory rate. A respiratory rate
levels of folic acid. Anticonvulsant therapy > 20 breaths/min is abnormal in pregnancy. All
should be reviewed prior to conception, and of the others are part of normal physiological
should not be stopped. Sodium valproate is changes of pregnancy.
associated with a higher risk of major
congenital malformations compared to other 30.5. Answer: E.
AEDs and there should be a discussion Hyperemesis gravidarurn (HG) can be
between the woman and her epilepsy specialist diagnosed in the first trimester of pregnancy,
about switching to another AED prior to when other causes of persistent nausea and
pregnancy. Pregnancy does not reduce the vomiting have been excluded. It is associated
frequency of seizures. Women with with > 5% pre-pregnancy weight loss,
well-controlled epilepsy are not more likely to electrolyte imbalance and dehydration. Nausea
have increased seizures in pregnancy, but and vomiting in pregnancy (NVP) is common,
those with poorly controlled epilepsy may find but not all of these women have HG.
their condition deteriorates. There is no
rationale for routinely doubling drug doses in 30.6. Answer: G.
I the second trimester, although some AEDs, for Myocardial infarction is more common in
example lamotrigine, may need a dose increase pregnancy compared to age-matched controls.
during pregnancy. PPCI is not contraindicated in pregnancy, and
should be carried out where benefits outweigh
30.3. Answer: D. risks. Both chest X-rays and EGGs are useful
Profound physiological changes in pregnancy investigations for chest pain in pregnancy.
can cause a significant reduction in serum Management of chest pain. where an acute
concentrations of some drugs - this is coronary syndrome is suspected should be the
particularly true for lamotrigine and same as in a non-pregnant woman.

downloaded from www.medicalbr.com


-'
MATERNAL MEDICINE • 369

30.7. Answer: A. 30.11. Answer: C.


Many women with chronic kidney disease Spider naevi and palmar erythema (not
(CKD) have successful pregnancies. However, pigmentation) can be part of normal pregnancy,
there are increased risks of pre-eclampsia, fetal and are also signs of chronic liver disease. The
growth restriction, miscarriage, pre-term other physical signs are not seen in normal
delivery and fetal death for these women. pregnancy. Ascites is observed in chronic liver
Women with CKD are more likely to require disease, and yellow sclerae indicate raised
antihypertensives, but they do not need to be bilirubin, which is not part of normal
started routinely. Women who are already pregnancy.
taking angiotensin-converting enzyme inhibitors
should stop this drug at conception and switch 30.12. Answer: A.
to an alternative therapy, such as labetalol, Oliguria could be associated with viral
methyldopa or nifedipine. gastroenteritis if there was an associated acute
kidney injury (AKI). Creatinine of 100 11moi/L
30.8. Answer: A. (1 .13 mg/dl) could be seen in pre-eclampsia!
The diagnosis is community-acquired HELLP and gastroenteritis with an AKI. A blood
pneumonia. Doxycycline is a tetracycline and is glucose of <4 mmoi/L (72 mg/dl) is a feature
associated with discolouration of infant teeth of acute fatty liver of pregnancy (AFLP), and is
when used in the second and third trimesters. one of the 'Swansea Criteria' for diagnosis of
Intravenous Tazocin is unnecessary, and oral AFLP. This condition is associated with
therapy is more appropriate for this woman. polydipsia/polyuria. Creatinine > 150 11moi/L
Oseltamivir is not indicated for community- (> 1.70 mg/dl) is a feature of AFLP. It is
acquired pneumonia. Trimethoprim is not an associated with hypoglycaemia, but not
appropriate antibiotic for community-acquired typically thrombocytopenia or a fever.
pneumonia: it is avoided in the first trimester Thrombocytopenia is more commonly
due to its anti-folate effects. Amoxicillin and associated with pre-eclampsia!HELLP.
clarithromycin are the most appropriate choice
of antibiotic for community-acquired 30.13. Answer: E.
pneumonia, and both are safe in pregnancy. Some women with complex ulcerative colitis
will require a caesarean section, but this mode
30.9. Answer: D. of delivery is usually reserved for obstetric
Chest X-ray should be carried out for the same indications. Methotrexate is teratogenic and
reasons as outside of pregnancy, and is safe. should be stopped 3 months prior to
Nebulised salbutamol and ipratropium, steroids, conception. Most women with well-controlled
magnesium sulphate and aminophylline can all ulcerative colitis will have uneventful
be given safely in pregnancy. Peak flow pregnancies. Sulfasalazine can be taken safely
measurement is valid in pregnancy, and should throughout pregnancy. lnfliximab can be taken
be carried out. Women should be advised to safely in the first and second trimesters.
continue their inhalers in pregnancy, and aim
for freedom from symptoms. 30.14. Answer: E.
Post-partum thyroiditis commonly presents at
30.10. Answer: B. 3-4 months post-delivery, although it can
Women with type 2 diabetes should take present up to 6 months post-delivery. It can
aspirin 75 mg once daily from 12 weeks' present with a transient hyperthyroidism,
gestation to delivery to reduce the risk of hypothyroidism, or a biphasic pattern, with a
developing pre-eclampsia. Insulin and period of hyperthyroidism followed by
metformin are both used to manage women hypothyroidism. Graves' disease is possible,
with type 2 diabetes in pregnancy. Tight but post-partum thyroiditis is more likely at 12
glycaemic control is advocated prior to weeks post-partum. Hashimoto's thyroiditis
pregnancy to reduce the risk of major leads to symptoms of fatigue, weight gain, cold
congenital malformations and miscarriage but intolerance, constipation and depression (in
the decision to alter medication is based on association with bioche~T-~ical hypothyroidism).
blood glucose levels (BGs); therefore the first Post-partum depression and anxiety can only
step would be to start monitoring BGs before be diagnosed once organic causes of
deciding on treatment. symptoms and signs have been excluded.

downloaded from www.medicalbr.com


T

R Mann

Adolescent and
transition medicine

Multiple Choice Questions


31.1. Adolescence is a complex developmental D. Investigation should be considered if she
stage characterised by physical, biochemical has not started menstruating by the age of
and emotional changes that see a child or 18 years
young person transition to adulthood. Which of E. The ovary produces both oestrogen and
the following statements is true about male testosterone and there is a rise in adrenal
pubertal physiology? androgen production
A. Follicle-stimulating hormone (FSH) production
ultimately stimulates the growth of pubic, 31.3. Adolescence is a period of time
facial and axillary hair when adherence with medications and
B. Gonadatrophin-releasing hormone (GnRH) treatment regimes can fall significantly.
produced in the pituitary stimulates Which factors might predict that a young
luteinising hormone (LH) and FSH release person is at particular risk of low adherence?
from the hypothalamus A. Acceptance of the seriousness of their health
C. Growth hormone increases skeletal growth problems
and promotes development of the male B. Female gender
genital organs C. High self-esteem and high levels of
D. Leydig cells in the testis produce self-confidence and motivation
testosterone D. Patients treated with complex medicines
E. Puberty is initiated by pulsatile release of E. Use of treatments with good short-term
testosterone in the testis symptom relief

31.2. In relation to the normal adolescent 31.4. On a global basis, what is the commonest
female, which of the following statements is cause of death in adolescents?
most correct? A. Complications of pregnancy
A. A fall in growth hormone levels is associated B. Infective gastroenteritis
with a climb in insulin-like growth factors 1 C. Late effects of childhood cancer treatment
and 2 (IGF-1 and IGF-2) D. Malaria
B. Breast bud development and the E. Road injury
development of pubic hair are seen around
the time of menarche 31.5. Which of the following characteristics are
C. Insulin levels fall by around 30%, coinciding particularly associated with risk-taking
with an increased risk of type 2 diabetes behaviours in teenagers?

downloaded from www.medicalbr.com


ADOLESCENT AND TRANSITION MEDICINE • 371

A. Female gender B. In the UK he will not be able to drive unless


B. Having good health and high levels of he is seizure-free for 2 years
physical well-being C. Sodium valproate can affect spermatogenesis
C. Higher educational status and is teratogenic when taken by men
D. Maturation of the prefontal cortex, resulting D. There is no evidence that alcohol
in development of frontal lobe control independently affects seizure control, so he
E. Maturity of the frontostriatal reward circuits, does not need to be concerned about his
encouraging novel and adult-like activities alcohol intake
E. There are no restrictions on application for
31.6. Which of the following is a key component jobs in the armed forces
of a successful transition programme?
31.9. A 17 year old male with a diagnosis of
A. A dedicated transition nurse to support
ulcerative colitis has been referred to your
patient education throughout adolescence
clinic. He is currently well and completely
B. A range of detailed patient information
asymptomatic on mesalazine. The only relevant
leaflets about the patient's medical condition
history is that his healthy brother has recently
and treatment and prognosis
been investigated for a raised bilirubin. Routine
C. A well-developed website/online support
liver function tests are as detailed:
service with up-to-date information about the
Alkaline phosphatase 420 U/L, alanine
service and transition arrangements
aminotransferase 36 U/L, bilirubin 34 J-Lmoi/L
D. A written transition policy, developed in
(1.99 mg/dl), albumin 48 g/L, y-glutamyl
conjunction with young patients and
transferase (GGT) 25 U/L.
implemented with a staff training package
Which of the following is true?
E. Access to a patient support group
A. Gilbert's syndrome is a common cause of
31.7. At your first appointment with a 17 year hyperbilirubinaemia and is the most likely
explanation of these liver function tests
old female kidney transplant recipient, she
discloses that she is pregnant. The pregnancy
B. It is important to check hepatitis serology
is unplanned. Her current treatment comprises C. It is important to undertake alkaline
alternate-day prednisolone, mycophenolate phosphatase isoen:z:ymes to exclude bone or
liver disease
mofetil, enalapril, arnlodipine and azathioprine.
Which of the following statements is most
D. The albumin suggests active inflammatory
bowel disease
correct in relation to the pregnancy?
E. The alkaline phosphatase result suggests
A. Mycophenolate is the immunosuppressant of vitamin D deficiency
choice in pregnancy
B. Prednisolone easily crosses the placenta and 31.10. A 20 year old woman with learning
causes a risk of impaired fetal growth difficulties and epilepsy is attending your clinic
C. Stable blood pressure control is essential for the first time. She is accompanied by her
and enalapril should be continued new boyfriend who she has just started living
D. The patient needs to be aware that pregnancy with. She is currently taking high-dose sodium
adversely affects long-term renal allograft valproate and has generalised seizures every
survival so close monitoring is necessary few weeks. What is your highest priority to
E. There is an increased risk of pre-eclampsia clarify or assess during your consultation?
in women who have received a renal
A. Assess her sodium valproate levels as you
transplant are concerned about adherence
B. Find out whether she is taking any
31.8. You have just completed a consultation recreational drugs
with a 17 year old male with epilepsy. He is C. Procure a detailed family history, particularly
about to leave college and asks for advice regarding epilepsy risk
about employment and driving. He is seeking D. Request an up-to-date electroencephalo-
further information about other lifestyle choices. gram report
Which of the following statements is true? E. Review the patient's and her partner's
A. Illicit drugs can affect seizure threshold, as understanding of teratogenicity of sodium
well as affect adherence to antiepileptic valproate, their plans for having children and/
drugs (AEDs) or whether they are using contraception

downloaded from www.medicalbr.com


372 • ADOLESCENT AND TRANSITION MEDICINE
.,
31.11. You are taking over the care of a 18 year transmembrane conductance regulator
old male with severe spastic quadriplegic (CFTR) protein
cerebral palsy. Which of the following D. She is likely to be infertile, and the chances
statements is true? of pregnancy are low
A. A common cause of mortality is renal failure E. The OCP seems to be safe and effective for
related to recurrent urinary tract infections most patients with CF
B. Formal assessment of respiratory function
such as spirometery and peak flows will help 31.14. An 18 year old female has beeri referred
assess respiratory risks to your late-effects clinic, after treatment for
C. Gastro-oesophageal reflux is an important acute lymphoblastic leukaemia (ALL) in
comorbidity that needs to be assessed and childhood, ultimately requiring treatment with
treated if necessary total body irradiation and bone marrow
D. Now .that he is 18 years old he automatically transplantation at 15 years of age. She is
assumes the capacity to consent to currently treated with thyroxine but her general
treatment and decisions about his care health seems good. Which of the following
E. Nutritional support is unlikely to be necessary statements is true?
now growth has been completed A. As her periods are regular she can be
reassured of normal fertility in the future
31.12. An 18 year old male with Duchenne B. Her risk of future malignancy is no higher
muscular dystrophy is admitted in end-stage than the normal population
respiratory failure. After treatment with C. She has previously been treated with
antibiotics and stabilisation on non-invasive high-dose doxorubicin and therefore is at
respiratory support, he is ready for discharge risk of cardiomyopathy
with ongoing respiratory support and careful D. She needs 7 years more follow-up before
long-term follow-up. At his next clinic discharge, as 10-year disease-free survival
appointment, he asks for a discussion about equates to cure in ALL
the genetics of his condition and how it might E. When plotted on a centile chart, her growth
involve his wider family - his younger brother is is 50th centile for height and weight,
also a Duchenne sufferer. He has a 10 year old suggesting she has normal growth hormone
sister. levels
Which of the following statements are true?
A. He will be infertile 31.15. A 16 year old girl has undergone liver
B. His father must be a carrier of the Duchenne transplant following a Kasai procedure for biliary
gene atresia in the newborn period. She has been
C. His sister could be affected well for 8 years post-transplant. She had a viral
D. His sister should be referred for genetic illness characterised by lymphadenopathy,
testing for carrier status malaise and hepatosplenomegaly 6 months
E. There are no genetic implications for ago and now has persistent palpable cervical
second-generation family members lymphadenopathy. Which of these statements
is rnost correct?
31.13. A 17 year old female with cystic fibrosis
A. If her lymphadenopathy persists you need to
(CF) is referred to your adolescent respiratory
consider blood tests, including a full blood
clinic. She has good nutritional status and is
count, liver function tests, Epstein-Barr virus
generally well, attending full-time school. Which
(EBV) and cytomegalovirus (CMV) PCR and
of the following are true regarding her current
serology
status?
B. She is at increased risk of chronic fatigue
A. As she has good nutritional status and syndrome
growth, her bone mineral density is likely to C. T-cell activation and proliferation is the likely
be normal and treatment with vitamin D is underlying pathological process in
not necessary post -transplant lyrnphoproliferative disorder
B. Clarithromycin will affect oral contraceptive (PTLD), often triggered by EBV infection
pill (OCP) effectiveness D. The most likely cause of her infection was
C. Most patients gain benefit from newer hepatitis B infection
treatments that rectify defects in the CF E. There is a 10% chance of this being a PTLD

downloaded from www.medicalbr.com


ADOLESCENT AND TRANSITION MEDICINE • 373

31.16. A 16 year old boy presents with an B. Eighty per cent of adolescents follow their
8-week history of diarrhoea, weight loss and diet reasonably well
raised inflammatory markers. A biopsy is C. In females, concern about body image,
consistent with Grahn's disease. Which of the including the desire for weight loss, can be a
following is the most correct statement in significant factor in non-adherence to insulin
relation to this case? therapy
A. Anti-tumour necrosis factor (TNF) therapy, for D. Microvascular complications can begin from
example infliximab, is more commonly 20 years after diagnosis, and so can already
needed for adolescents, and he has about a be emerging in patients in their 20s and 30s
50% chance of needing treatment with a E. The majority of patients do not take their
biological agent. insulin injections reliably and this results in an
B. First-line treatment is with steroid therapy, increased admission rate with diabetic
most commonly oral prednisolone or pulsed ketoacidosis (DKA)
intravenous methylprednisolone
C. Methotrexate is a helpful first -line 31.18. When planning adult services for a young
maintenance therapy person with juvenile idiopathic arthritis (JIA),
D. There is a 50% chance of him requiring which one of the following statements is most
surgery in the next 5 years accurate?
E. When offering lifestyle advice, particular A. All children with oligoarticular juvenile arthritis
emphasis should be given to reducing will require long-term follow-up into adulthood
smoking, as smoking increases disease B. Antinuclear antibody (ANA)-positive patients
activity and reduces effectiveness of need ophthalmic screening for eye
biological agents involvement
C. Methotrexate is a first -line treatment if
31.17. Adherence with treatment is a particular multiple joints are affected, and it should be
challenge for teenagers and young adults with used early, particularly in polyarticular JIA
long-term medical conditions such as diabetes. D. Systemic JIA can often be treated with a
Which of the following statements is most combination of long-term NSAIDs and
correct in relation to studies in adolescents with systemic glucocorticoids
diabetes? E. When offering lifestyle advice, particular
A. About 15% of adolescents do not check emphasis should be given to reducing
blood glucose levels regularly and fabricate alcohol intake as it increases disease activity
results for the medical team looking after and reduces effectiveness of biological
them agents

Answers
31.1. Answer: D. changes are breast bud development and early
Puberty is initiated by pulsatile GnRH pubic hair growth, which can be seen from
production, which stimulates FSH and LH around 10 years of age. Menarche arises
production in the pituitary gland. LH stimulates relatively late in puberty, but an adolescent who
Leydig cells in the testis to produce has not started her periods by 16 years of age
testosterone, which causes androgenisation should be investigated for delayed puberty.
and skeletal growth. FSH acts on Sertoli cells Growth hormone levels, IGF-1 and IGF-2 levels
to stimulate spermatogenesis and not to climb steadily during puberty, as do insulin
increase androgenisation. levels by about 30%.

31.2. Answer: E. 31.3. Answer: D.


Adolescent females have a increase in Complex medicine and treatment regimes
testosterone and androgen production make it harder for young people to adhere
(manifest as the development of pubic hair, strictly to their treatment. The other factors are
increased sweating, acne) from the ovaries and all associated with better patient adherence
the adrenal glands. The earliest pubertal with medication and treatment regimes, and in

downloaded from www.medicalbr.com


V*
37 4 • ADOLESCENT AND TRANSITION MEDICINE

many conditions have been associated with a mycophenolate nor azathioprine are safe during
better long-term outlook. Younger patients find pregnancy. When pregnancy is diagnosed,
it harder to adhere to medicines where the angiotensin-converting enzyme (ACE) inhibitors
long-term health benefits are considerable if should be stopped immediately, and alternative
there is no short-term improvement in therapy commenced, due to human fetotoxicity.
symptoms. With careful monitoring, transplant survival rates
are good.
31.4. Answer: E.
Road injury/fatal road traffic accidents account 31.8. Answer: A.
for a significant proportion of adolescent deaths Illicit drugs have an adverse affect on seizure
on a worldwide basis. Death from road injury is control through both lowering seizure threshold
independently associated with alcohol and drug and adversely affecting adherence. Alcohol
ingestion. Whilst malaria and infective does not seem to independently increase
gastroenteritis are important causes of death in seizure activity, but binge drinking can be
younger children, teenagers have better associated with significant sleep disturbance
immune responses and these conditions are a and reduced AED compliance. In the UK,
common cause of morbidity but have a lower drivers are not permitted to drive unless they
mortality rate than in younger children. Lower have had no daytime seizures for 1 year, but if
respiratory tract infections and suicide are also seizures only occur during sleep then driving
important global causes of adolescent mortality. can be considered. There is no evidence of
teratogenicity in men taking sodium valproate
31.5. Answer: E. but it reduces sperm count in some. In most
The highest-risk adolescents, in terms of parts of the world there are restrictions on entry
risk-taking and self-harming behaviour such as to the armed forces, driving heavy goods
heavy alcohol intake, illicit drug ingestion and vehicles and driving emergency vehicles.
non-adherence to treatment regimes, are
males, older adolescents and those with 31.9. Answer: A.
serious long-term health conditions. It is The liver function tests (including raised alkaline
thought that maturation of the frontostriatal phosphatase) are normal for a male of this age,
reward circuits in early/mid-adolescence drives with the exception of the isolated raised
individuals towards impulsive and pleasure- bilirubin. Hypoalbuminaemia can be a marker of
seeking behaviours that place the adolescent at inflammatory bowel disease, but this albumin is
risk. With time, frontal lobe control of impulsivity normal. There is no need to check alkaline
improves and more stable and safe behaviour phosphatase isoenzymes as the GGT is
patterns develop. normal. Gilbert's syndrome affects 5-1 0% of
the Western European population, and is one
31.6. Answer: D. of the commonest causes of isolated elevation
The starting point in the development of an in bilirubin. It is autosomally recessively
effective transition policy is the local inherited and so his brother's jaundice is likely
development of a programme that meets the to also be due to Gilbert's syndrome.
medical, social and cultural needs of your local
population. The other measures may support 31.10. Answer: E.
the implementation of your transition All these issues may be important. However, in
programme, but often services overly focus a young person who has recently started a
upon a series of information-giving interventions sexual relationship, establishing his/her
rather than developing an ethos of patient understanding of the reproductive implications
autonomy and control. of his/her condition and treatment is of vital
importance - particularly with an agent as
31.7. Answer: E. teratogenic as sodium valproate.
There is an increased risk of hypertension
during pregnancy in all women with renal 31.11. Answer: C.
disease, so they require close monitoring Gastro-oesophageal wflux is common in
throughout - the pre-eclampsia rate is around patients with severe neurodisability. It places
30%. Prednisolone crosses the placenta poorly patients at significant risk of aspiration as they
and is not a particular risk to the fetus. Neither may not have adequate airway-protective

downloaded from www.medicalbr.com


ADOLESCENT AND TRANSITION MEDICINE • 375

reflexes. Most patients with four-limb cerebral 31.15. Answer: E.


palsy are also intellectually impaired, and many PTLD is a well-recognised complication
do not have the capacity to give informed occurring in more than 10% of solid organ
consent. They are unlikely to be able to comply recipients - particularly those who receive a
with formal lung function tests, although solid organ in childhood when they are
respiratory disease, often complicated by commonly EBV seronegative. It is often
recurrent chest infections or aspiration and triggered by EBV infection leading to
scoliois, is a common cause of death. Many do uncontrolled B-cell proliferation and tumour·
not manage to ingest their full nutritional proliferation, including development of
requirements and need additional/supportive lymphoma. Blood tests need to be undertaken
feeding, often by gastrostomy. immediately in this high-risk patient and further
investigation for possible lymphoma is
31.12. Answer: C. necessary.
Female carriers of Duchenne muscular
dystrophy are at risk of cardiomyopathy and 31.16. Answer: E.
around 10% also experience muscle weakness Smoking is a particular risk factor for
and fatigue. They should be referred for genetic exacerbation and reduces effectiveness of
testing when they are able to understand the many immunosuppressant therapies, as well as
implications of the diagnosis, possibly in the increasing the risk of steroid and other therapy.
mid-teens. This patient's mother must be a Adolescents/teenagers do have more
carrier of the Duchenne muscular dystrophy aggressive disease than older patients, and
gene, not his father, and his sister has a 50% about 20% will require surgery or treatment
risk of also being a carrier. Fertility is normal in with biological agents. First-line therapy is a
males. There are potentially risks for the wider 6- to 8-week trial of elemental diet;
family, with his mother's sisters having a 50% methotrexate is used as maintenance therapy
chance of being carriers of the Duchenne but is not a first -line agent.
genetic mutation.
31.17. Answer: C.
31.13. Answer: E. A few adolescents present with repeated
The OCP is contraindicated in patients with episodes of DKA because of non-adherence
pulmonary hypertension but is safe and with insulin therapy. In females it is thought that
effective in most patients with CF. There is motivating factors might include weight loss/
increasing evidence of effectiveness of CFTR concerns about body image. The extent of
modification in patients with specified (G551D) non-adherence amongst adolescents is high,
CFTR gene mutations, but these affect a very with 25% not taking their insulin as prescribed,
small proportion of CF sufferers. Females with 80% not following their diet and around 30%
CF usually have normal fertility if their general not checking blood glucose and/or submitting
nutrition and health are good; males have fabricated results. Microvascular complications
obstructive azoospermia. All patients are at can develop within 10 years of diagnosis, so
significant risk of osteoporosis. may already be present in later teenage years.

31.14. Answer: C. 31.18. Answer: C.


There is a well-recognised risk of Methotrexate and earlier use of anti-TNF agents
cardiomyopathy in patients exposed to such as etanercept have significantly improved
high-dose anthracyclines such as doxorubicin, outcome in the 30% of patients with
so long-term follow-up is recommended. There polyarticular JIA. All patients, even those who
is also an increased risk of second malignancy. are not ANA positive, need screening for
Many female survivors of childhood cancer uveitis, which can be silent and persist into
have reduced long-term fertility due to reduced adulthood (Box 31.18). As many as 50% of
ovarian reserves, and total-body irradiation children with oligoarticular JIA go into
makes this patient at risk of pituitary remission, so long-term follow-up into
dysfunction, including growth hormone adulthood may not be required.
deficiency. The hypothyroidism might be central
or peripheral in aetiology.

downloaded from www.medicalbr.com


'T
376 • ADOLESCENT AND TRANSITION MEDICINE

31.18 Juvenile idiopathic arthritis in adolescence


Uveitis: may be clinically silent and persist into adulthood. data on fraclure risk and the evidence base for
All (not just those who are ANA positive) need treatment is poor.
ophthalmic screening for eye involvement. Therapy: methotrexate is standard treatment, used after
Persistence into adulthood: occurs in 50% of cases, NSAIDs alone are insufficient. Anti-TNF therapy is
especially in systemic disease. Specific supportive effective in all forms of juvenile idiopathic arthritis but
management through transition from adolescence to long-term safety remains unclear.
adulthood should be planned.
Reduced peak bone mass: common in polyarthritis and
systemic juvenile idiopathic arthritis but there are few
(ANA ~ antinuclear antibody; NSA/Ds ~ non-steroidal anti-inflammatory drugs; TNF ~ tumour necrosis factor)

downloaded from www.medicalbr.com


MD Witham

Ageing and disease

Multiple Choice Questions


32.1. A 79 year old man who lives in his own D. Transient ischaemic attack
home presents with several recent falls, all of E. Vestibular neuronitis
which have taken place whilst walking in town.
Witnesses report no loss of consciousness. He 32.4. An 84 year old man presents with falls
takes thyroxine and paracetamol but no other and ankle swelling. He was started on
medications. Which of the following interventions amlodipine 3 months ago for hypertension, and
is most likely to reduce his risk of falls? started furosemide 2 weeks ago for the ankle
A. Calcium and vitamin D supplementation swelling. He complains of feeling lightheaded
B. Cardiac pacemaker when standing, and thinks he might have lost
G. Hip protectors consciousness before the last fall. His blood
D. Home environment modification pressure is 165/97 mmHg lying, 142/88 mmHg
E. Strength and balance training standing.
What changes to his medication are most
32.2. An 86 year old woman complains of appropriate?
incontinence for the last 2 years. She describes A. Add an angiotensin-converting enzyme (ACE)
wanting to pass urine 10-12 times a day, has inhibitor, continue the amlodipine and
to rush to the toilet, and if she does not get furosemide
there in time, urine is passed in a flood. Which B. Add an ACE inhibitor and stop the
of the following would be the most useful amlodipine
treatment for her symptoms? G. Stop the furosemide and amlodipine
A. Antimuscarinic medication D. Stop the furosemide and amlodipine and
B. Long-term prophylactic antibiotics add an a-adrenoceptor antagonist
G. Long-term urinary catheter (a-blocker)
D. Pelvic floor muscle training E. Stop the furosemide and continue the
E. Tension-free vaginal tape surgery amlodipine

32.3. A 68 year old woman presents 32.5. Which of the following is an essential
complaining of dizziness. She says that this component of a successful rehabilitation
started 2 days ago, and that she cannot walk programme?
in a straight line. She feels sick and the room is A. A dedicated rehabilitation ward
spinning. What is the most likely cause? B. Clearly defined diagnoses
A. Fast atrial fibrillation G. Goal setting
B. Lumbar nerve root entrapment D. Medical leadership
G. Orthostatic hypotension E. The Barthel Index

downloaded from www.medicalbr.com


y
378 • AGEING AND DISEASE

32.6. Which of the following outcomes does D. Hip extension range


Comprehensive Geriatric Assessment (CGA) E. Supine blood pressure
improve?
A. Chance of living independently at 6 months 32.11. An 82 year old man presents with
B. Mortality at 3 years delirium, which has been gradually worsening
C. Speed of recovery from surgery for 6 weeks. His niece thinks that it started
D. Time to onset of dementia after his last hospital stay, when he was treated
E. Time to onset of frailty for pneumonia. His blood tests show a sodium
of 122 rnrnoi/L. What is the most likely cause
32.7. A 76 year old man is admitted to hospital for his hyponatraemia and delirium?
with a urinary tract infection. As the admitting A. Addison's disease
doctor, you are concerned that he is frail. B. Bendroflumethiazide
Which of the following measurements would C. Carcinoma of the lung
most assist you in assessing whether he D. Ibuprofen
is frail? E. Inadequate salt intake
A. Blood pressure
B. Body mass index 32.12. A 92 year old woman presents with
C. Hand grip strength urinary incontinence, which the nursing home
D. Number of medications staff are finding difficult to manage. She has
E. Six-minute walk distance severe dementia, and the incontinence has
been gradually worsening for several years. She
32.8. You see an 87 year old woman in your often declines to wear incontinence pads,
clinic, newly diagnosed with heart failure with appears to be unaware that she needs to pass
preserved systolic function. She is sceptical urine, and consequently is found by the staff to
that her condition is real, and would prefer to have been incontinent sitting in her chair. Which
attribute it to old age. Which one of the management strategy is most likely to improve
following cardiovascular changes is attributable her continence?
to normal ageing? A. A course of antibiotics
A. Development of left ventricular hypertrophy B. Intermittent selfccatheterisation
B. Fatty infiltration of the myocardium C. Long-term urinary catheter
C. Increased left ventricular end diastolic volume D. Pelvic floor exercises
D. Reduced left ventricular ejection fraction E. Regular prompted toileting
E. Reduced maximum heart rate
32.13. You are assessing an 83 year old woman,
32.9. You are assessing a 78 year old woman who wishes to undergo a hip replacement. She
as part of Comprehensive Geriatric has not lost any weight in the last 12 months,
Assessment. Which one of the following her grip strength is 15 kg, and her 5 m walk
measurements would best allow you to predict time is 6 seconds. She admits to feeling tired a
her falls risk? lot of the time, but still manages to do her own
A. Abbreviated Mental Test score shopping, a little gardening, and goes out to
B. Barthel Index play cards 3 evenings a week. Which description
C. Hand grip strength best fits her current functional status?
D. Six-minute walk test A. Disabled
E. Timed 'get up and go' test B. Frail
C. Functionally impaired
32.10. You are assessing a 91 year old man D. Not frail
who recently fractured his hip in a fall. Which E. Pre-frail
one of the following components of
examination is most likely to influence your
32.14. A 77 year old woman complains of
immediate management of his falls risk?
unsteadiness on her feet, which started a few
A. Cardiac auscultation months ago and has. gradually worsened. She
B. Geriatric Depression Scale finds it difficult to walk in a straight line, and
C. Hallpike manoeuvre often overbalances when she turns. On

downloaded from www.medicalbr.com


AGEING AND DISEASE • 379

examination, she has no nystagmus or past 32.18. An 82 year old woman with advanced
pointing, tone and power are normal, but dementia is noticed by the nursing home staff
Romberg's test is positive. What is the most to look rather pale. She does not complain of
likely cause for her unsteadiness? breathlessness or tiredness; she had a severe
A. Benign positional vertigo stroke 3 years ago and has been unable to
B. Cerebellar infarction walk since; she sits in a wheelchair during the
C. Parkinson's disease day and is helped into bed by two helpers at
D. Peripheral neuropathy night. Her bowels are open normally and she
E. Vestibular neuronitis does not complain of indigestion. She has been
in hospital twice. in the last 3 months and
32.15. A 93 year old man presents having fallen during her last admission stated a wish to be
three times in the last week. He has significant allowed to die. What is the most appropriate
bruising over the side of his face from the last investigation for this woman?
fall. His wife saw the last fall; she is sure that A. Abdominal ultrasonography
her husband lost consciousness for a few B. Full blood count
seconds, but came round after 2-3 minutes on C. No investigation
the ground. Lying and standing blood pressure D. Upper and lower gastrointestinal endoscopy
are 155/92 mmHg and 148/90 mmHg, E. Upper gastrointestinal endoscopy
respectively, and cardiac auscultation is normal.
Which course of action would be most 32.19. An 86 year old woman presents having
appropriate for this man? taken to her bed for the last 2 days. She is
A. 24-Hour electrocardiogram (ECG) monitoring normally mobile around the house using a walking
B. Echocardiography frame, but does not usually leave the house.
C. Referral to physiotherapist for strength and Carers come to help her wash and dress twice a
balance training day. On examination, her pulse is 110 beats/min,
D. Start calcium and vitamin D supplementation blood pressure 90/50 mmHg, respiratory rate
E. Tilt table testing 24 breaths/min, oxygen saturations 96% on air.
Her temperature is 37.0°C. Her chest is clear, she
32.16. An 85 year old woman presents with has a gallop rhythm on cardiac auscultation, and
diarrhoea and vomiting. Her blood tests show her jugular venous pressure is not elevated. She
acute kidney injury. Which one of the following is disoriented, drowsy, but able to move all her
changes in kidney structure is attributable to limbs. She opens her eyes when you raise your
ageing, rather than to an underlying disease voice. Her ECG shows deep T-wave inversion
process? across the anterior leads. What is the most likely
diagnosis?
A. Glomerulosclerosis
B. Porosity of the glomerular filtration barrier A. Depression
C. Reduction in nephron numbers B. Myocardial infarction
D. Renal arteriolar hyaline deposition C. Parkinson's disease
E. Stenosis of the renal arteries D. Pneumonia
E. Pulmonary embolism
32.17. A 94 year old man presents with three
falls over a 2-day period. On assessment, he is 32.20. A 77 year old man complains of difficulty
disoriented and dehydrated. His chest is clear walking. On inspection of his gait, he struggles to
to auscultation, temperature is 35.2°C, pulse start walking, but then accelerates into a series of
90 beats/min, blood pressure 11 0/50 mmHg. small steps, and fails to lift his feet very far from the
His respiratory rate is 18 breaths/min and his floor. He does not swing his arms when walking,
oxygen saturations are 89% on air. What is the and has difficulty turning at the end of the walk.
most likely cause for his falls? What is the most likely explanation for his gait?
A. Cerebral infarction A. Bilateral parietal lobe stroke disease
B. Pneumonia B. Cerebellar stroke
C. Poor fluid intake C. Hip osteoarthritis
D. Spinal cord compression D. Parkinson's disease
E. Subdural haematoma E. Peripheral neuropathy

downloaded from www.medicalbr.com


y
380 • AGEING AND DISEASE

Answers
32.1. Answer: E. Leadership is necessary, but does not have to
His falls do not occur at home: thus, home be medical or doctor leadership. Assessment of
modification is unlikely to help in this case. needs is necessary, but this does not have to
Calcium and vitamin D is effective only in be via the Barthel score. Rehabilitation can take
patients in institutional care, who are those with place in many settings, including the patient's
the lowest vitamin D levels. Hip protectors do home; a ward is not necessary. It is essential to
not reduce falls, and current evidence suggests define the patient's disabilities and functional
that they do not reduce fractures either. A capabilities; this is more important than the
pacemaker would help only if cardioinhibitory precise underlying diagnoses.
carotid sinus hypersensitivity was
demonstrated. 32.6. Answer: A.
CGA reduces short-term mortality or adverse
32.2. Answer: A. outcomes, but not these outcomes at 12
She is describing urge incontinence. months. CGA can improve cognition in the
Antimuscarinic medication can be helpful but medium term, but there is no evidence as to
carries a high burden of side-effects. Her whether it delays the onset of dementia or not.
symptoms have been continuous for 2 years; CGA might improve the speed of recovery from
they are not therefore due to infection and surgery, but there is no trial evidence to prove
antibiotic therapy is inappropriate. For most this. It is a key component, however, in
people, long-term catheters bring as much maximising function in older people recovering
harm as benefit. Tension-free vaginal tape from surgery. CGA is usually offered to those
and pelvic floor training are useful interventions patients who are already frail or pre-frail; thus it
for stress incontinence but not for urge is unlikely to affect time to onset of frailty.
incontinence.
32.7. Answer: C.
32.3. Answer: E. Hand grip strength forms part of the Fried frailty
She is describing vertigo, which may be due to phenotype, and is a powerful independent
either labyrinth or brainstem disease. As her predictor of frailty-related outcomes in older
symptoms have persisted for 2 days, a people. Blood pressure is not part of frailty
transient ischaemic brainstem attack is less syndromes. Although weight loss is part of
likely than vestibular neuronitis - although note frailty measurements, current body mass index
that a completed stroke involving the brainstem is not. Similarly, walk speed over a short
might produce similar symptoms. distance (4 or 5 m) is part of frailty assessment,
but 6-minute walk distance is not commonly
32.4. Answer: C. used; this is a measure of endurance exercise
This is a classic case of treating drug side- capacity and is more useful in assessing
effects with further drugs. The amlodipine has disease severity of cardiorespiratory illnesses
caused ankle oedema; the furosemide has then such as heart failure and chronic obstructive
caused intravascular volume depletion and pulmonary disease. Number of medications is
orthostatic hypotension. The safest course of related to multimorbidity, not to frailty.
action is to stop both agents, then reassess
the blood pressure (perhaps using a 24-hour 32.8. Answer: E.
blood pressure monitor). If the blood pressure All of the other changes are due to
is still high, an alternative agent (such as an cardiovascular pathology; all are more common
ACE inhibitor) could be considered. a-Blockers with age, but can be attributed to disease
are particularly likely to worsen orthostatic processes such as atherosclerosis,
hypotension. hypertension, obesity and myocardial
dysfunction.
32.5. Answer: C.
Goal setting (and regular review of goals) 32.9. Answer: E.
is an essential component of successful Timed 'get up and go' test is a good predictor
rehabilitation. The other components are not. of future falls risk, and also allows observation

downloaded from www.medicalbr.com


AGEING AND DISEASE • 381

of the gait for unsteadiness. Six-minute walk very unlikely that urinary infection is playing any
test measures endurance rather than part in her symptoms.
'fast-twitch' lower limb function (which is more
closely correlated with balance and falls risk). 32.13. Answer: E.
The Barthel Index measures dependency in She has 2 of the 5 Fried Frailty criteria - low
activities of daily living, and although hand grip grip strength and self-reported exhaustion.
is a good measure of overall physical status Three criteria are required to diagnose frailty,
(and forms part of the criteria for frailty), it is but the presence of 1 or 2 criteria is sometimes
less directly relevant to falls risk. categorised as 'pre-frail'. You are not given any
information to suggest that she has functional
32.10. Answer: C. impairment - she continues to undertake
Benign positional vertigo is common and activities of daily living. Similarly, you are not
amenable to treatment with simple positional told anything that suggests the presence of a
manoeuvres. Supine blood pressure alone will specific disability.
tell you little; postural blood pressure is more
important. Finding a reduced hip extension 32.13 How to assess a Fried Frailty score
range would be unsurprising after recent hip
Hand grip strength in bottom 20% of healthy elderly
surgery. Whilst depression is important, finding
distribution*
it will not directly influence your plans for Walking speed in bottom 20% of healthy elderly
reducing his falls risk. Cardiac auscultation may distribution*
uncover a murmur of aortic stenosis - a cause Self -reported exhaustion
Physical inactivity
of syncopal episodes potentially amenable to
At least 4.5 kg weight loss within 1 year
intervention - but this is less likely than option Patient is defined as frail if 3 or more factors are present;
C, and even if you find severe aortic stenosis, 1-2 factors indicate a 'pre-frail' state.
comorbid disease and frailty might prevent you *Varies between populations. Grip cut-off is 30 kg for men
from intervening successfully. and 18 kg for women in US adults; 5 m walk time cut-off is
7 seconds in US adults for both sexes.
32.11. Answer: B.
Drugs are the most common cause of 32.14. Answer: D.
hyponatraemia in older people - and thiazide The lack of nystagmus or past pointing argues
diuretics are one of the commonest drug against this being due to middle ear, brainstem
causes. Ibuprofen is a less likely cause, unless or cerebellar disease. The normal tone makes
acute kidney injury has been precipitated by its Parkinsonian syndromes less likely, although
use. Both carcinoma of the lung and Addison's you are not given specific information about
disease can cause hyponatraemia, but are both bradykinesia. A peripheral neuropathy or dorsal
much less common causes than drugs. column spinal cord disease would explain the
Inadequate salt intake is very unlikely to lead to unsteadiness and positive Romberg's test.
low serum sodium levels.
32.15. Answer: A.
32.12. Answer: E. The witness account suggests that this was a
Her dementia is likely to be severe enough that syncopal episode; this requires investigation.
she is unaware of needing to pass urine; the 24-Hour ECG monitoring is a reasonable first
normal inhibitory signals preventing bladder investigation; if this does not uncover a reason,
emptying are lost and the signals indicating that then further investigation (e.g. tilt table testing)
the bladder is full are either not processed or may be required. Echocardiography is likely to
not acted on. Regular toilet visits (e.g. every be less useful, especially given that no murmur
2-3 hours) can be helpful in ensuring that is audible.
voiding occurs before the bladder is full. Pelvic
floor exercises are useful in stress incontinence, 32.16. Answer: C.
but require active participation and The other structural changes are due to
understanding by the patient. Catheterisation is disease, not ageing. Glomerulosclerosis may be
not the first choice for any continence problem, caused by a range of dis~ases, including
and this woman is unlikely to have sufficient diabetes mellitus and infections; diabetes may
cognitive function to self-catheterise. The similarly cause porosity of the filtration barrier,
long-standing nature of the problem makes it leading to proteinuria. Hypertension leads to

downloaded from www.medicalbr.com


f
382 • AGEING AND DISEASE

arteriolar hyaline deposition, and renal artery decisions on her behalf should she lack
stenosis may be caused by atherosclerosis or capacity to make decisions about her medical
fibromuscular dysplasia. care.

32.17. Answer: B. 32.19. Answer: B.


Onset of falls, particularly several falls in quick The ECG is suggestive of myocardial infarction
succession, should suggest intercurrent illness. - perhaps 2-3 days ago; this would also
Acute illness in older people may present explain her gallop rhythm. Myocardial infarction
atypically, as here - but there are still clues that may present without chest pain in older people
this is pneumonia. He has a low temperature - especially older women - and atypical
(equivalent to a fever of 38.8°C), a raised pulse symptoms such as tiredness and delirium are
rate and low oxygen saturations. The other common, as in this case, where she suffers
options do not explain all of these features; in from hypoactive delirium. The problem is acute,
particular, they do not explain his hypoxia. making depression or Parkinson's disease
unlikely; the normal oxygen saturations make
32.18. Answer: C. pneumonia and pulmonary embolism less likely
It is unlikely that performing any of the listed diagnoses.
investigations will improve the quality of this
woman's life. She is asymptomatic: therefore 32.20. Answer: D
even if anaemia was discovered on a full blood The gait described is festinant (slow start, then
count, it is debatable as to whether transfusion accelerating), and shuffling (not lifting the feet).
would improve her quality of life. Clearly if she The lack of arm swing and difficulty turning are
were to become symptomatic, this would also consistent with a Parkinsonian gait.
change. Endoscopy would, in addition, be Cerebellar lesions cause ataxia; bilateral parietal
burdensome given her frailty, and you have lobe stroke disease may cause apraxia (e.g.
some indication from her last illness that she difficulties starting to walk) or marche a petits
might not want further medical intervention. pas and peripheral neuropathy can cause a
Even if you did decide to investigate anaemia, stamping gait, which may be high stepping if
ultrasonography is unlikely to find the cause. In foot drop is present. Hip osteoarthritis would
real life, the decision-making process would, of typically cause an antalgic gait, where the
course, need to be informed by the wishes of weight-bearing phase is shortened for the
the patient, and of those deputed to make affected leg (a 'limp').

downloaded from www.medicalbr.com


GG Dark

Oncology

Multiple Choice Questions


33.1. A 52 year old woman presents to her D. M
family physician concerned about her risk of E. S
cancer from her smoking. She has smoked 20
cigarettes per day for the last 35 years and 33.4. A 23 year old woman presents to her
several of her family members have died of family physician. She is 8 weeks pregnant and
cancer. She is in good health. the family physician advises her to take folic
What site of cancer is most likely to be acid daily. She asks if it is safe to take other
caused by inhaled carcinogens from cigarette vitamin supplements.
smoke? Which vitamin in large doses can
A. Bladder be teratogenic and so should be
B. Breast avoided?
G. Central nervous system A. Vitamin A
D. Colon B. Vitamin B12
E. Ovary G. Vitamin C
D. Vitamin D
33.2. A 44 year old man presents to the urology E. Vitamin E
clinic for assessment. He is suspected of
having a bladder tumour in the renal tract and 33.5. A 71 year old man presents to the
the consultant requests a urine cytology dermatology clinic with an ulcer over his
specimen. left temple. It has been growing slowly
What is the best sample for cytological over the previous 3 years. He worked as a
assessment? farmer and spent most of the working day
A. 24-Hour urine collection outdoors.
B. First urine sample of the morning On clinical examination the lesion appears
G. Fresh, full voided sample as a non-healing, indolent, punched-out,
D. Mid-stream urine clean-looking 2-cm ulcer over the left temple.
E. Urine that is frozen immediately There are no enlarged lymph nodes in the head
and neck.
33.3. A research student is studying the effects What is the most appropriate next step in
of a new chemotherapy agent on cell cycle and the patient care?
cellular division. What phase of the cell cycle is A. Perform a full-thickness biopsy of the centre
most likely to correspond with nuclear division of this lesion
followed by cytokinesis? B. Perform a full-thickness biopsy of the edge
A. Go of the lesion
B. G1 G. Perform scrapings and culture from the ulcer
G. G2 base

downloaded from www.medicalbr.com


384 • ONCOLOGY t
I
D. Refer for radiotherapy treatment to this A. Adrenaline (epinephrine)
lesion B. Lactate dehydrogenase
E. Resection of the whole lesion with a 1-cm C. Plasma osmolality
clear margin D. Prolactin
E. Renin
33.6. A 52 year old man presents to the
emergency department complaining of 33.9. A 65 year old man presents to the
lightheadedness. He has a past medical history emergency department complaining of gradual
of lung cancer, which was diagnosed a month loss of sensation in his right hand, weakness of
previously and found to be metastatic, involving the left lower leg and visual disturbance. He
the bone and pericardium. has smoked two packs of cigarettes daily for
On clinical examination, his blood pressure 40 years. His past medical history includes
is 70/40 mmHg and his pulse is I OObeats/min. asthma and emphysema.
His heart sounds are distant and soft. His On clinical examination, he is afebrile, pulse
electrocardiogram shows low-voltage 86 beats/min, blood pressure 137/86 mmHg
complexes and electrical altemans is present. and respiration rate 24 breaths/min. Eye
A chest X-ray film shows that the cardiac
fundoscopy is normal.
silhouette is enlarged. A magnetic resonance imaging (MRI) scan of
What is the most appropriate next step in the brain shows five different intracerebral
this patient's care? lesions, ranging from I to 3 em in diameter and
A. Intravenous dexamethasone located at the gray-white matter junction in
B. Intravenous fluid challenge both cerebral hemispheres. The lesions are
C. Intravenous furosemide sharply demarcated and contrast enhancement
D. Oral ibuprofen after gadolinium administration is present in all
E. Pericardiocentesis of them.
What is the most likely diagnosis?
33.7. A 66 year old woman presents to the
A. Arteriovenous malformations
emergency department holding her right arm
B. Embolic infarcts
with a deformity that signifies an obvious
C. Multifocal glioblastoma multiforme
fracture. She described that she was shopping
D. Multiple abscesses
and picked up a bag out of the trolley to place
E. Multiple metastases
it in her car. She then felt a sharp, sudden pain
in the middle of her arm and her humerus
suddenly gave way. 33.10. A 65 year old woman presents to the
What is the most likely reason for the emergency department with constant, severe
fracture? abdominal pain that has worsened over the
previous week. She has no other associated
A. Bone metastasis in the humerus from breast
symptoms but has noticed that her daily urine
cancer
output had decreased significantly. She has
B. Osteitis fibrosa cystica from parathyroid
had a constant desire to urinate, but when she
disease
tries, only a small amount of bloody urine is
C. Osteomalacia from nutritional deficiency
discharged. She is a long-time smoker, having
D. Osteoporosis
smoked three packs per day for more than
E. Primary malignant bone tumour of the
45 years.
humerus
On clinical examination she has a suprapubic
33.8. A 51 year old woman presents to the mass arising from the bladder and appears to
endocrinology clinic for investigation of weight be in urinary retention.
gain. She is found to have an elevated 24-hour What is most likely to be detected
·urinary free cortisol, elevated serum upon imaging the patient's genitourinary
adrenocorticotrophic hormone (ACTH) and system?
elevated serum cortisol. The serum cortisol A. Bilateral hydronephrosis
does not fall when she is given high-dose B. Bladder dilation
dexamethasone. C. Bladder dyskinesis
What serum investigation is most likely to be D. Unilateral hydronephrosis
elevated in this patient? E. Urethral dilation

downloaded from www.medicalbr.com


ONCOLOGY • 385

33.11. A 54 year old woman presents to the What factor is the most likely cause of this
emergency department complaining of severe problem in the UK?
lower abdominal pain and distension over a
24-hour period. Her bowels had not moved
over the same time period and her abdomen
has become visibly swollen with associated
nausea and vomiting. Over the previous 4
months, she has lost 9 kg in weight and has
noted progressive symptoms of constipation.
She reports that on several occasions she has
passed blood mixed in with her bowel
movements, which have become thinner in
calibre. She denies any recent travel, use of
antibiotics, or fevers.
On clinical examination, she appears acutely
uncomfortable and has a temperature of
38.3°C. Her abdomen is diffusely distended
and tender to palpation in the left lower
quadrant. There are hyperactive rushing bowel
sounds. On rectal examination, her stool is
brown and tests positive for blood. A plain A. Arsenic
abdominal X-ray film shows multiple small B. Benzene
bowel air fluid levels and a dilated colon C. Human papilloma virus (HPV)
proximal to the sigmoid colon. D. Ultraviolet (UV) radiation
What is the most likely diagnosis? E. Vinyl chloride
A. Amoebic abscess
33.14. A 42 year old man previously worked at
B. Colonic polyp
the Fukushima Daiichi Nuclear Power Plant
C~ Diverticulitis
and received radiation exposure as a result of
D. Diverticulosis
the damage to the reaCtor caused by an
E. Sigmoid carcinoma
earthquake and the subsequent leakage of
nuclear material. He has concerns about his
33.12. A 39 year old woman completed her last future cancer risk as a direct result of his
course of adjuvant chemotherapy for breast exposure.
cancer 2 years earlier. She presents to the What statement in relation to radiation
oncology clinic complaining of constant back exposure is the most accurate?
pain for 3 weeks. On clinical examination she is
tender to palpation over two well-circumscribed
A. Large exposure is required to develop the
most serious malignancies
areas in the thoracic and lumbar spine. There is
no neurological deficit.
B. Leukaemia has the shortest latency period of
all malignancies
What is the most appropriate next step
C. Malignancies always occur within 10 years of
in investigation, assuming rapid availability
exposure
of all?
D. Malignancy risk increases with advancing
A. Computed tomography (CT) scan of whole age at the time of exposure
spine
E. Therapeutic radiation therapy given without
B. Isotope bone scan
chemotherapy does not increase the risk of
C. Needle biopsy of the affected areas a second malignancy
D. Plain film X-rays of the affected areas
E. Ultrasour;Jd of the affected areas 33.15. A 22 year old man presents to his family
physician complaining of breathlessness
33.13. A 62 year old woman has noticed worsening over the previous 7 days. He has no
a lesion on her face that has persisted for cough and denies smoking. A chest X-ray is
more than a month. It appears as an performed, shown below.
ulcerated lesion with a raised, rolled edge
(see figure).
What is the most likely histological type of
malignancy? . .I
downloaded from www.medicalbr.com
-.1
+
386 • ONCOLOGY

demonstrated, extending from the buttocks


down the back of the legs and onto the soles
of the feet.
What is the most likely site of the lesion?
A. A dorsal column of the spinal cord
B. Cauda equina
C. Distal spinal cord
D. Proximal spinal cord
E. Sciatic nerves

33.18. A 49 year old man presents with a


1-week history of increasing nausea, vomiting
and lethargy. He has an extensive smoking
history of 30 pack years and was recently
A. Adenocarcinoma
diagnosed with lung cancer. He is not taking
B. Carcinosarcoma
any medications and has not yet initiated
C. Immature teratoma
chemotherapy.
D. Papillary serous carcinoma
On clinical examination, he is afebrile and
E. Squamous cell cancer
somnolent. His lungs are clear to auscultation
and his heart is regular in rate and rhythm. His
33.16. A 28 year old woman presents to her
skin demonstrates loss of elasticity. Laboratory
family physician after finding a breast lump on
re;o;ults indicate a serum calcium level of
self-examination. She found the lump 2 months
2.95 mmoi/L (11.8 mg/dl).
previously, and although she was not initially
What is the most appropriate initial step in
concerned, has laterly become worried about
management?
the possibility of cancer. On further questioning,
she reports that the lump had neither increased A. Calcitonin
nor decreased in size since she first noticed it B. Hydrochlorothiazide
and that she had no family history of breast C. Intravenous normal saline
cancer. D. Prednisolone
On clinical examination there was a 2 x 2 em E. Zoledronic acid
firm, non-tender, mobile lump in the left breast.
What is the most appropriate next action for 33.19. A 37 year old accountant presents to his
the family physician to take? family physician to ask for advice regarding the
future management of his ulcerative colitis. He
A. Perform a fine needle aspiration of the lump
has had pancolitis for the past 19 years and
himself for cytological assessment
has been told that he is at an increased risk for
B. Reassure the patient that the lump is
developing colorectal cancer. He asks for
probably benign and arrange a 1-month
recommendation regarding appropriate
follow-up
surveillance.
C. Refer the patient to a breast cancer
What is the most appropriate investigation
multidisciplinary team
for regular surveillance?
D. Request a mammogram
E. Request an ultrasound of the breast A. Barium enema
B. Colonoscopy
33.17. A 64 year old man with known lung C. Colonoscopy and multiple biopsies
cancer presents with a history of progressive D. Faecal occult blood testing
leg weakness and numbness, which began in E. Flexible sigmoidoscopy with multiple
his buttocks and gradually spread down the biopsies
back of his legs into the soles of his feet. He
has recently become impotent and incontinent 33.20. A 72 year old woman presents with
of both faeces and urine. abdominal distension, feeling bloated and
On examination there is wasting of the getting full quickly when eating. Her past
buttocks and calf muscles with bilateral medical history includes hypertension and she
weakness of hip extension, ankle dorsiflexion is prescribed an angiotensin-converting enzyme
and plantar flexion. A sensory deficit is (ACE) inhibitor.

downloaded from www.medicalbr.com


ONCOLOGY • 387

Clinical examination reveals abdominal 33.23. A 44 year old woman presents to her
distension with shifting dullness. Pelvic family physician complaining of a severe
examination reveals a large, non-tender right headache that had been present for several
adnexal mass. weeks and had not responded to the usual
Abdominal CT scan shows masses arising over-the-counter headache remedies. She
on both ovaries, ascites and omental locates the headache to the centre of her head
thickening. Serum cancer antigen 125 (CA-125) and describes it as constant but worse in the
level is 2000 U/ml. Serum alpha-fetoprotein mornings. She has no other neurological signs
(AFP) and human chorionic gonadotrophin or symptoms. She has had 'tension headaches'
(hCG) are normal. previously but those were located in the back
What is the most likely diagnosis? of her head and felt different from the present
A. Choriocarcinoma pain. She has a past history of breast cancer 2
B. Dermoid cyst (cystic teratoma) years previously, which was treated with
G. Epithelial ovarian cancer surgery followed by adjuvant chemotherapy.
D. Ovarian sarcoma What is the most appropriate next step in
E. Sertoli stromal cell tumour diagnosis?
A. Carotid arteriogram
33.21. A 25 year old woman, gravida 2, para 2 B. CT scan of the head
presents to her family physician to discuss G. Lumbar puncture
contraception. She has no medical problems, D. Psychiatric evaluation
is on no medications and has no family history E. Skull X-rays
of cancer. All clinical examinations are normal.
After a discussion with the family physician, 33.24. A 43 year old woman presents to the
she chooses to take the oral contraceptive pill specialist breast clinic with a breast lump that
(OCP) and stays on the pill for the following she noticed on self-examination. She has a
5 years. 2-cm, firm, non-tender mass in the left breast,
What cancer' has the greatest reduction in which is movable from the chest wall, but not
risk as a result of this medication? movable within the breast. She has no prior
A. Bone sarcoma history of breast disease.
B. Breast cancer What is the most appropriate initial step?
G. Cervical cancer A. Arrange a mammogram to find any other
D. Endometrial cancer lesions that might also need to be addressed
E. Hepatocellular carcinoma B. Arrange an ultrasound scan and advise the
patient she is unlikely to need a biopsy
33.22. A 73 year old man presents to his family G. Discuss the surgical options in case cancer
physician complaining of a drooping right eye is found
lid. He has a 70-pack year history and his D. Obtain a fine needle aspirate and discharge
family physician has been seeing him for more the patient if no malignant cells are found
than 10 years for management of his E. Wait for two menstrual cycles to see whether
symptoms of chronic obstructive pulmonary there is spontaneous resolution
disease (COPD). On clinical examination, he
has ptosis of the right eye with a constricted 33.25. A 70 year old man presents to his family
right pupil. The remainder of the eye and physician with an episode of visible haematuria.
cranial nerve examination is normal. He denies prior episodes and had been
What is the most likely finding on a chest previously healthy. He is not on any medication.
X-ray of this patient? Urinalysis confirms gross haematuria without
A. A calcified granuloma in the left mid-lung proteinuria or casts. The patient denies any
field pain and all physical examination is normal.
B. A left-sided pleural effusion What is the most appropriate next step?
G. A right upper lobe pneumonia A. CT scan of the pelvis
D. An irregularly shaped mass at the apex of B. Cystoscopy
the left lung G. Renal angiogram -
E. An irregularly shaped mass at the apex of
the right lung
D. Transrectal prostatic biopsy
E. Trimethoprim-sulfamethoxazole I
downloaded from www.medicalbr.com
f
388 • ONCOLOGY

33.26. A 73 year old man presents to the chest 33.28. A 26 year old woman presents to her
clinic for annual review for asbestosis. He has a family physician complaining of facial hair on
long smoking history and was diagnosed with her upper lip. This has been present for many
asbestosis on biopsy 4 years previously. He years and has not bothered her before. She
has no change in his symptoms but continues has been trying to conceive for some time
to smoke cigarettes and denies any cough or without success and previously has taken the
shortness of breath. His chest X-ray shows left OCP for irregular periods.
lower lobe pleural thickening with calcifications On clinical examination, her body mass index
at the level of the diaphragm. (BMI) is 32 kg/rn 2 • Her blood pressure is
He has many questions about his disease 135/88 rnmHg, pulse is 72 beats/min and skin
and wants to discuss his risk for malignancy examination reveals acanthosis nigricans, mild
and long-term prognosis. What explanation is acne and scattered plucked chin with facial hair
most appropriate? on the upper lip. Abdominal examination is
A. Asbestosis itself (without smoking) is unlikely normal.
to progress to cancer This woman is at greatest risk for what
B. His risk of cancer is greater than 70 times condition?
that of the normal population A. Diabetes mellitus
C. Mesothelioma is the most common cancer B. Gastric cancer
associated with asbestosis and smoking C. Ovarian cancer
D. Small cell lung cancer is the most common D. Ovarian torsion
cancer associated with asbestosis and E. Uterine cancer
smoking
E. Steroids may slow progression of his disease 33.29. A 59 year old man presents to his family
physician with a 3-week history of dyspnoea,
33.27. A 42 year old woman presents to the particularly on exertion, and had an occasional
clinic to discuss her concerns regarding breast cough, which is dry and unproductive.
cancer. She has no symptoms at review, but He describes some chest tightness and
previously she had noted bilateral breast discomfort, which was mostly dull in
tenderness prior to her menses, which has nature.
since abated. She has had two caesarean On clinical examination there is nicotine
deliveries but no other operations. She is taking staining of the left index and second fingers.
a low-dose OCP and has no known drug There is no peripheral lymphadenopathy, no
allergies. She does not smoke and has no evidence of heart failure, the jugulovenous
family history of cancer. All clinical examinations pressure is not raised and heart sounds are
are normal. normal. On chest examination there is reduced
She wants to know whether BRCA 1 and expansion on the right, with decreased tactile
BRCA2 screening would be appropriate for her vocal fremitus, dullness to percussion and
in addition to routine screening starting at age diminished breath sounds. Examination of
50. What is the most appropriate response? the left hemithorax is unremarkable. Peak flow
A. BRCA 1 and BRCA2 screening is not rate is 450 Umin. Abdominal examination is
recommended normal.
B. BRCA 1 and BRCA2 screening should be What is the most likely diagnosis?
performed after age 50 A. Collapse of the right lung
C. BRCA 1 and BRCA2 screening should be B. Consolidation of the right lung
perfomied if breast pain recurs C. Interstitial fibrosis throughout right lung field
D. BRCA 1 screening is recommended D. Left tension pneumothorax
E. BRCA2 screening is recommended E. Right pleural effusion

Answers
33.1. Answer: A. and are then processed in the liver to become
Inhaled carcinogens are absorbed across the more water-soluble. The metabolised
bronchial mucosa and enter the blood stream carcinogens are then filtered by the kidney and

downloaded from www.medicalbr.com


ONCOLOGY • 389

sit in the bladder for hours. After more than 10 aspiration, usually under echocardiograrn
years, the risk of bladder cancer is significantly guidance.
elevated. The same '1s true for breast cancer,
as carcinogens are secreted into the breast 33.7. Answer: A.
ducts, but the incidence of breast cancer This patient has no prior history of illness and
caused by this aetiology is not as great as that the fracture has occurred spontaneously, i.e.
for bladder cancer. without any trauma. In view of her gender and
Ovarian cancer is not affected by smoking age, of the options listed, this is most likely to
but the risk of endometrial cancer is lower in be due to breast cancer (1 in 8 lifetime risk).
smokers than non-smokers. Options C and D
have no significant linkage to smoking. The 33.8. Answer: B.
best answer is option A. The clinical indicators suggest that this patient
has Cushing's syndrome. There are four
33.2. Answer: C. possible causes of Cushing's. These are:
Options A and E would allow the cells to die exogenous steroids, adrenal adenoma, ectopic
and therefore be unsuitable for cytological ACTH and a pituitary adenoma. Only the latter
assessment. Option 8 results in cells sitting in two give a high ACTH and only ectopic
the bladder overnight with some also dying off. production does not fall on a high-dose
This is, however, the best option for suspected suppression test. Therefore, the clinical
Mycobacterium infection. Option Dis the scenario is describing Cushing's syndrome with
best sample for culture as it minimises ectopic ACTH production. The most likely
contamination at the start and end of stream. cau'le of that is small cell lung cancer (SCLC).
Option C gives the best yield for cytological LDH is an intracellular enzyme that is released
assessment. during necrosis as a pathological process;
therefore, in rapidly growing tumours (like
33.3. Answer: D. SCLC), this can be elevated in a serum sample.
The second growth phase precedes nuclear Patients with SCLC can develop the
division, which is in mitosis (M}, and is followed syndrome of inappropriate antidiuretic hormone
by cytokinesis, which is still in mitosis (M). (vasopressin) secretion, but that would
decrease plasma osmolality. Renin may be
33.4. Answer: A. increased in some tumours but not lung.
It is important to understand which drugs are Adrenaline is increased in neuroendocrine
safe in pregnancy, and vitamin A taken in large tumours of the adrenal gland
doses can cause fetal abnormalities. Other (phaeochromocytoma) but not neuroendocrine
vitamins mentioned are not thought to have any tumours of the lung (SCLC). Prolactin can be
teratogenic effect. produced as a result of an ACTH-producing
pituitary tumour causing loss of prolactin
33.5. Answer: B. inhibitory factor (due to pituitary stalk
This patient is likely to have a basal cell compression), but ACTH would fall with
carcinoma from the description. high-dose dexamethasone in that scenario.
Options D and E relate to management but
identification is required first, particularly before 33.9. Answer: E.
delivering invasive treatment. Option C would The clinical features do not suggest infection
be used for a fungal lesion. Option A would (option D) and option A would be more likely to
biopsy the central necrotic portion and may not cause a subarachnoid haemorrhage. Option B
yield a diagnosis, whereas option 8 would is more likely to have sudden onset. Option B,
sample the proliferative edge and therefore is C and E are possible from the clinical history
best for histological diagnosis. but the radiological description is more in
keeping with option E.
33.6. Answer: E.
The clinical presentation of this patient 33.10. Answer: A.
describes a pericardia! effusion resulting from A long smoking history increases the exposure
his malignancy: hence the increased cardiac of the urological epithelium to inflammatory
silhouette. His blood pressure is low as he is mediators such as carcinogens in tobacco.
developing cardiac tamponade. This requires After more than 1 0 years, this increases the

downloaded from www.medicalbr.com


v
390 • ONCOLOGY

risk of developing a bladder cancer, which in myeloma. Vinyl chloride is hepatotoxic and has
turn is causing urinary retention. Given the time been associated with hepatic angiosarcoma.
course, it is most likely that bilateral
hydronephrosis will be present. 33.14. Answer: B.
The carcinogenic effect of radiation exposure is
33.11. Answer: E. related to the exposure rate. When we consider
This patient is in bowel obstruction and the this in relation to therapeutic radiation, larger
clinical history suggests many features to locate doses can be given in lots of small fractions
this to the sigmoid colon. Option A is unlikely in over more time to lessen the effect (55 Gy over
the absence of foreign travel and the symptoms 5 weeks on average for radiotherapy treatment,
would be right upper quadrant pain. Option B whereas 8 Gy to the whole body over 30
is unlikely to cause thin stools and obstruction. seconds could prove fatal). Second
Option C (-itis) has inflammation and could malignancies induced by radiotherapy usually
result in abscess formation, even perforation, take more than 10 years to manifest. Patients
but would not fully explain the stool history (thin that are young (< 18 years) have tissues that
calibre). Option D may explain the increasing are still developing and are therefore at highest
constipation over time but not the acute risk of transformation. Chemotherapy is
presentation. Option E is the best answer for all administered with radiotherapy to enhance the
symptoms and progression into an emergency biologically effective dose, i.e. it produces a
presentation to hospital in bowel obstruction. greater effect on tissue than the dose of
A high temperature can be seen in malignancy radiation on its own. This acts as a sensitiser
or when secondary infection is present. for the tissue and would therefore increase the
risk of malignant transformation. Leukaemia has
33.12. Answer: B. the shortest latency period.
This patient is likely to be pre-menopausal and
therefore osteoporosis is less likely. She has a 33.15. Answer: C.
diagnosis of breast cancer and could have This question is about understanding the
progressive recurrent disease and therefore the natural history of malignancy. Papillary serous
onset of back pain requires investigation. The carcinoma is mostcommonly associated with
first step in investigation is to assess the whole gynaecological cancers and is therefore unlikely
skeleton to see if this is isolated or widespread in a male patient, although it can arise in the
and that is best done with a radioisotope bone pancreas in older patients. Carcinosarcoma
scan. This will be followed with plain film contains malignant elements from epithelial
imaging of any hot spots and, if suspicious, tissue (carcinoma) and connective tissue
thereafter consider a biopsy of the (sarcoma) and is most commonly found in the
abnormalities. gynaecological system, although rarely it is
CT imaging can show bone detail but would found as a component of de-differentiated
be less sensitive than a bone scan. Ultrasound carcinoma of the lung, but not at this age.
would not be helpful. MRI would be best if Adenocarcinoma can develop from many
there was also a neurological deficit, to look for primary sites, including the lung, where it can
cord compression or to distinguish osteoporotic arise in the periphery or hilar region and is not
collapse from metastatic involvement. associated with tobacco products. This, too, is
less likely at this age. Testicular immature
33.13. Answer: D. teratoma is most likely to cause a large-volume
Each of these substances is associated with lung metastases in a young male.
malignancy but UV exposure is most
associated with a basal cell carcinoma. These 33.16. Answer: C.
tumours are therefore more common in According to the National Institute for Health
individuals that work outdoors. Although and Care Excellence (NICE) guidance, the lump
arsenic is associated with skin c~mcer, it is requires follow-up and investigation, not
most likely to be squamous cell carcinoma. reassessment in a month's time by the same
HPV is associated with head and neck cancer doctor. However, whilst one of the listed
and cervical cancer. Benzene is associated investigations is the most appropriate (option
with leukaemia, particularly acute myeloid A), it should be performed and interpreted by
leukaemia but also non-Hodgkin lymphoma and specialists and not in the primary care setting.

downloaded from www.medicalbr.com


ONCOLOGY • 391

33.17. Answer: B. may slightly increase the risk of hepatocellular


This patient has clinical signs that suggest carcinoma.
lower motor neuron weakness and as the
spinal cord ends at L1~L2, this abnormality is 33.22. Answer: E.
at a lower level than L2. Lung cancer will The clinical features are that of Horner's
commonly metastasise to bone and although it syndrome and the clinical signs will be
can cause spinal cord compression, the ipsilateral, therefore suggesting an invasive
neurology in this presentation infers that this is lesion in the apex of the right lung (option E).
at a level where it results in compression of the The remaining options are non-invasive or in
cauda equina. the wrong location and therefore should not
cause impairment of the sympathetic
33.18. Answer: C. innervation on the right side. The presence of
This patient is presenting with hypercalcaemia Horner's syndrome in a patient with a chest
and the immediate management should infection is suggestive of an underlying cancer.
be to rehydrate the patient (option C).
Bisphosphonate therapy (option E) is indicated 33.23. Answer: B.
once his hydration state improves. Loop The past history of breast cancer should raise
diuretics can be used if fully rehydrated but not an index of suspicion for metastasis and the
a thiazide diuretic (option B) as it could increase new headache is different to her previous
the serum calcium. In patients with lung cancer, episodes. This should be investigated as brain
hypercalcaemia is most commonly associated metastasis and the best initial investigation of
with squamous cell carcinoma. those listed is CT scan (option B). The other
options would not enable this diagnosis to be
33.19. Answer: C. made.
Patients with ulcerative colitis have an
increased risk of developing a colonic 33.24. Answer: A.
carcinoma due to the chronic inflammation. It The first step is to look for other lesions and to
often will have a lead time of 10 years and is examine for calcification and spiculation (option
more likely in those with more significant A). There is no need to wait for the menstrual
inflammation. In order to assess the whole cycle as the patient has not presented with
colon and biopsy for histology, option C is the cyclical changes in the breast. A fine needle
best investigation. Option E will not assess the aspiration will be required but a negative result
whole colon and the remaining options (without does not exclude malignancy. A biopsy will be
biopsy) do not allow histological assessment. required, whether a mammogram is performed
or not. In practice, patients attending a
33.20. Answer: C. one-stop specialist breast clinic will have a
Epithelial ovarian cancer is most likely to cause mammogram or ultrasound, clinical examination
a rise in CA-125 and although this tumour and a fine needle biopsy at the same visit.
marker is rarely diagnostic it can assist in
disease activity monitoring. The remaining 33.25. Answer: B.
tumours do not normally cause a rise in A single episode of frank haematuria requires
CA-125 unless it is a false positive due to investigation and is unlikely to be a simple
inflammatory changes arising from the tumour. urinary infection (option E). CT imaging may not
Choriocarcinoma would cause a rise in serum detect small mucosal lesions (option A) and
hCG. without stream impairment it is unlikely that this
is related to prostatic enlargement (option D).
33.21. Answer: D. The most likely diagnosis is a bladder lesion or
The risk of breast cancer and bone sarcoma cancer and that requires cystoscopy for
remains the same in patients taking the OCP. inspection and biopsy.
The risk of cervical cancer is slightly increased,
not by the medication but due to the increased 33.26. Answer: B.
sexual activity of this population of patients. Non-small cell cancer is the IJlost likely
The risk of endometrial cancer is reduced due malignancy that occurs in patients with
to the reduced stimulation of this tissue. The asbestosis. Although asbestosis is due to
use of a steroid-based drug such as oestrogen previous asbestos exposure, only a small

downloaded from www.medicalbr.com


f
392 • ONCOLOGY

percentage of patients subsequently develop middle-aged patients and males more than
mesothelioma and the continued smoking will females. However, in younger patients it is
increase the risk of lung cancer. Moreover, more associated with insulin resistance and
smoking cessation will not negate the risk of thus an increased risk of diabetes mellitus and
malignancy. Steroid therapy may improve the polycystic ovary syndrome, which may explain
symptoms but does not alter the natural history some of the other signs and symptomatology.
of asbestosis. Her BMI will increase her risk of uterine
cancer but only after she has become
33.27. Answer: A. post -menopausal.
Screening of patients for breast and ovarian
susceptibility genes is indicated in individuals 33.29. Answer: E.
that have a personal history of both cancers, or The clinical features at presentation are those
that have a personal diagnosis of either breast of a pleural effusion: reduced expansion,
or ovarian cancer and a first -degree relative diminished tactile vocal fremitus (vocal
with either breast or ovarian cancer. Routine resonance), dullness to percussion and
screening of BRCA genes in a patient with no diminished breath sounds. Tracheal deviation
history would not be indicated. may be away from the side of the lesion in
massive effusion but shift of the lower
33.28. Answer: A. mediastinum (apex beat) is also likely to be
Acanthosis nigricans is a paraneoplastic away from the side of the effusion.
phenomenon associated with gastric cancer in

downloaded from www.medicalbr.com


L Colvin

Pain and palliative care


Multiple Choice Questions
34.1. Which of the following is normally involved into his legs. On examination, light touch is
in peripheral pain processing? painful, and he has reduced sensation to
A. A~ fibres pin-prick testing in a stocking distribution,
B. Calcitonin gene-related peptide (CGRP)- typical of peripheral diabetic neuropathy. His
containing C fibres blood. glucose control is not good, his renal
C. lnterneurons function is impaired and he is overweight. He is
D. Meissner's corpuscles reluctant to walk because of the pain, and lives
E. Pacinian corpuscles alone, becoming increasingly socially isolated.
What factor may make pharmacological
34.2. The pain system can change in response management more difficult?
to tissue injury. Which of the following A. Impaired renal function
neurotransmitters plays a key role in the B. Obesity
process of central sensitisation? C. Pin-prick hypoalgesia
A. Galanin D. Reduced mobility
B. Glutamate E. Social isolation
C. Glycine
D. ~-Endorphin 34.5. A range of patient -reported outcome
E. y-Aminobutyric acid (GABA) measures have been validated for use in
patients with chronic pain. Which one of the
34.3. A previously fit 72 year old man presents following assessment tools is likely to be most
with severe pain affecting his right chest wall, helpful in making the diagnosis of neuropathic
such that he is struggling to remain living pain?
independently. From his history you discover A. Beck Depression Inventory
that his clothes touching his skin is B. Brief Pain Inventory
excruciatingly painful. His family physician has C. Pain Catastrophising Scale
started him on tramadol 50 rng twice daily, D. Pain Detect
without significant benefit. What symptom is he E. Tampa Scale of Kinesiophobia
describing?
A. Breakthrough pain 34.6. A 79 year old woman has osteoporosis
B. Formication with vertebral collapse at T1 0, demonstrated on
C. Hyperalgesia plain X-ray. She is now struggling with washing
D. Mechanical allodynia and dressing herself because of severe pain.
E. Spontaneous pain She lives alone, although she has a daughter
who visits her every day, .who has been giving
34.4. A 64 year old man with type 2 diabetes her two co-codamol 30/500 (30 mg codeine
mellitus is complaining of numbness, and 500 mg of paracetamol), morning and
paraesthesia and pain in both feet, spreading evening. Her daughter is concerned that her

downloaded from www.medicalbr.com


f
. 394 • PAIN AND PALLIATIVE CARE

mother's memory is not as good as it used to B. Develop a management plan with the patient
be. What is the likeliest cause of the memory to support her in using self-management
impairment? strategies
A. Borderline cognitive impairment exacerbated C. Ensure she has a thorough assessment by a
by opioid medication dietician and advice on diet
B. Depression D. Increase oxycodone IR to 15 mg with an
C. Lack of sleep due to pain increase in frequency, as required for the
D. New onset of Alzheimer's disease pain, up to 8 times a day.
E. Undiagnosed malignant disease, with brain E. Stop her strong opioids, as they may be
metastases causing the constipation

34.7. A 27 year old man had a severe injury to 34.9. A 49 year old man with an advanced
his left arm in a motorcycle accident 4 years oropharyngeal tumour has severe pain in his
ago. He had extensive surgery, complicated by mouth and jaw, and is also struggling to eat.
post-operative infection, and required a high He is taking soluble co-codamol, which helped
dose of opioids to manage it at that time. He initially but is not really working now. He has a
has had persistent pain since then, being past history of peptic ulcer disease. What type
unable to return to his job as a builder. When of analgesic would you choose next?
assessed in the pain clinic he has very limited A. A strong opioid should be considered, with
movement, mechanical allodynia and appropriate formulation or route of
intermittent swelling (below). The affected limb administration (e.g. suspension or liquid;
is noticeably colder than the other arm, with transdermal)
increased sweating in his hand. B. Diazepam should be given to help with any
anxiety
C. Diclofenac should be started at maximum
dose to reduce any inflammation
D. Low-dose amitriptyline, or other tricyclic
antidepressant, should be started in case
there is any neuropathic pain
E. Paracetamol should be added in

34.10. A 65 year old man with inoperable lung


cancer has increasing breathlessness. He is
struggling to climb the stairs to his bedroom
What is the likeliest diagnosis? and more recently has found holding a
A. Chronic osteomyelitis conversation difficult. What would the first step
B. Complex regional pain syndrome (CRPS) be in managing his symptoms?
type 1 A. A comprehensive assessment to determine if
C. CRPS type 2 there is a reversible cause
D. Secondary osteoarthritis B. Breathing retraining should be used to
E. Ulnar neuropathy improve mechanical effectiveness of the
respiratory system
34.8. A 24 year old woman with long-standing C. Use of a hand-held fan to cool the air flowing
Crohn's disease, currently quiescent, has over nasal receptors
chronic abdominal pain. She is on D. Referral for physical therapy to reduce
modified-release oxycodone 30 mg twice daily, disability and strengthen muscles
plus immediate-release (IR) oxycodone E. Reassurance and involvement of a
5-10 mg as required for pain, up to 6 times a psychologist to manage anxiety
day. She is regularly admitted with a flare-up in
her pain, despite no evidence of active disease, 34.11. A 72 year old man with metastatic
although she is often very constipated. What is prostate cancer is admitted with severe nausea
the best long-term management option for her? and some delirium. He is on morphine for pain
A. Add in trarnadol 100 mg 4 times daily for control, for bony metastases. What
additional short-acting analgesia investigations should you do initially?

downloaded from www.medicalbr.com


PAIN AND PALLIATIVE CARE • 395

A. Blood test, including full blood count, urea side-effects associated with the medication.
and electrolytes, calcium and albumin Which of the following pieces of advice is true?
B. Computed tomography (CT) scan of A. Drowsiness after a dose increase is common
abdomen and pelvis and may improve within a few days
C. CT scan of head B. Morphine is the opiate of choice regardless
D. Electrocardiogram (ECG) and of renal function
echocardiogram C. Once established on the right dose, furt~er
E. Endoscopy adjustments will not usually be necessary
D. The dry mouth associated with her morphine
34.12. A 68 year old patient with chronic prescription will improve within a week of
obstructive pulmonary disease (COPD) attends starting the drug
your outpatient clinic after a recent admission E. The nausea and vomiting are likely to persist
to the high dependency unit. He remains short and she should take long-term antiemetic
of breath on minimal exertion. His daughter medication in addition
asks whether he might be referred to the local
palliative care team. Which of the following 34.15. A 71 year old woman with lung cancer
statements applies? and end-stage COPD is becoming increasingly
A. He should be judged to be in the last 6 distressed by dyspnoea and is referred to the
months of his life in order to benefit from respiratory team for assessment. She is
specialist palliative care team input tachypnoeic and anxious. Her symptoms are
B. He should have a diagnosis of cancer to be no longer relieved by inhaled bronchodilators.
suitable for referral She has a cough productive of grey phlegm,
C. He should have up-to-date pulmonary which is unchanged from her normal situation.
function tests before referral Her husband supports her at home; both of
D. He would benefit from advice on anticipatory them continue to smoke.
planning for future exacerbations of his Her chest X-ray shows hyperinflation of both
disease lungs and the known tumour at the left apex.
E. Opiate medication is the likely treatment of Observations are unremarkable other than
choice for this patient oxygen saturations of 89%.
Which of the following might play a role in
34.13. A 79 year old man is in the ward. He has helping to manage her current condition?
presented with right flank pain and a sense of A. An oxygen concentrator, for use as required
abdominal fullness. His liver function tests are at home
abnormal and an ultrasound shows multiple B. Antibiotic therapy
lesions in the liver. He is tender over the right c. Initiation of citalopram medication
upper quadrant and tells the medical team that D. Oral diuretic therapy to treat any coexisting
his pain is not helped by paracetamol. cardiac failure
He has a past medical history of ischaemic E. Sublingual lorazepam, to be taken as
heart disease, gout, total hip replacement and required
a resection of a colonic cancer 2 years ago.
Which of the following would be a
34.16. A 76 year old woman is an inpatient in
reasonable strategy if his pain persists?
the general medical unit. She is known to have
A. A glucocorticoid such as prednisolone or multiple myeloma with bony metastases and
dexamethasone has presented with vomiting. Her bowels have
B. An NSAID not moved for 8 days. She is delirious and
C. Antispasmodic medication such as hyoscine looks as though she may be dying.
butyl bromide Her initial blood results are as follows:
D. Gabapentin haemoglobin 79 g/L, white cell count
E. Oral morphine solution 6.7x10 9/L, platelets 314x109/L; urea
18.3 mmoi/L (11 0 mg/dl), sodium 143 mmoi/L,
34.14. A 52 year old woman has metastatic potassium 4.2 mmoi/L, creatinine 213 11moi/L
cancer with bony metastases throughout her (2.4 mg/dl), calcium 2.94 mmoi/L (11.8 mg/
pelvis. She is requiring increasing doses of her dl), albumin 23 g/L, adjusted calcium
opiate medication, and is concerned about the 3.28 mmoi/L (13.1 mg/dl).

downloaded from www.medicalbr.com


v
396 • PAIN AND PALLIATIVE CARE

Which of the following initial treatments and is now unable to communicate his needs
would be most helpful to this patient? to his family or the nursing team caring for him.
A. A subcutaneous infusion of haloperidol for He is currently undistressed.
her delirium and nausea Which of the following statements applies to
B. Blood transfusion to bring haemoglobin his ongoing management?
above 100 g/L and addition of a proton A. As he is unconscious, there is now no need
pump inhibitor for religious or spiritual support in this
C. Intramuscular cyclizine situation
D. Intravenous fluids and bisphosphonate B. He should have his urea and electrolytes
therapy checked at least twice weekly to check for
E. Intravenous fluids and laxatives to address worsening of his renal function
the constipation C. If he is unable to swallow medication, then
he should receive his usual dose of diuretics
34.17. You are asked by hospital colleagues to by an intravenous route
undertake a palliative care review of a 77 year D. Parenteral medication should be available as
old man who is dying of end-stage cardiac required for any symptoms that might arise
failure in one of the general hospital wards. E. The family can be advised that he is likely to
He is no longer able to eat or drink and is die within the next 2-3 days
completely bed bound. He is now unconscious

Answers
34.1. Answer: B. such as CGRP. Spinal interneurons modulate
Normally, light touch and pressure cause input from peripheral nerves.
activation of specialised mechanoreceptors
such as Pacinian and Meissner's corpuscles, 34.2. Answer: B.
with transmission of sensation such as light A wide range of neurotransmitters are involved
touch being via large myelinated A~ fibres. in pain processing (Box 34.2), with changes
Painful stimuli activate high-threshold occurring in response to tissue injury.
nociceptors, found on small unmyelinated C Glutamate, acting via the N-methyl-o-aspartate
fibres. C fibres may contain a range of (NMDA) receptor plays a key role in central
neuropeptides involved in pain processing, sensitisation (Fig. 34.2), with increased neuronal

NMDA receptor

lll
Amplified
s1gnal

Regulation of
pain response

changes

Fig. 34.2 Mechanisms of central sensitisation. Post-synaptic activation of the N-methyl-0-aspartate (NMDA) receptor by the amino
acids glycine and glutamate, which bind to the NR1 and NR2 subunits, respectively, amplify pain signals at the level of the spinal cord.
In contrast, magnesium ions block receptor activation.

downloaded from www.medicalbr.com


PAIN AND PALLIATIVE CARE • 397

34.2 Neurotransmitters and receptors involved in pain processing in the spinal cord
Neurotransmitter Receptor(s) Receptor type Comments*
Amino acids
Glutamate AMPA ion channel Excitatory; permeable to cations: can be Ca'•, Na• or
K•, depending on subunit structure
NMDA lon channel Excitatory; blocked by Mg'+ at resting state; block can
be altered if membrane potential changes; permeable
to Ca'•, Na' and K'
Kainate lon channel Post synaptic - excitatory
Gp I GPCR Pre-synaptic - inhibitory through GABA release;
permeable to Na' and K'
Gp II GPCR Activates a range of signalling pathways; long-term
effects on synaptic excitability
(lp Ill GPCR Probably inhibitory; can decrease cAMP production;
pre-synaptic; decreases glutamate release
Glycine GlyR lon channel Mainly inhibitory; permeable to cl- blocked by
caffeine
y-aminobutyric acid GABA, lon channel Mainly inhibitory in spinal cord; permeable to Cl-;
indirectly modulated by benzodiazepines (increased
ion channel opening); not specifically involved in
nociception, generally depressant effect on spinal
cord activity
GABA, GPCR Predominantly inhibitory; activated by baclofen
Neuropeptides
Substance P Neurokinin receptors GPCR Mainly excitatory; increased in inflammation,
decreased in neuropathic pain
Cholecystokinin CCKRs1-8 GPCR Excitatory; clinical trials of antagonists in progress
Calcitonin gene-related CALCRL GPCR Excitatory; slows degradation of substance P;
peptide implicated in migraine
Opioids
Dynorphin OP1 (kappa) GPCR Excitatory??; may be pro-nociceptive
~-endorphin OP3 (mu) GPCR Inhibitory
Nociceptin ORL-1 GPCR Inhibitory; also expressed by immune cells
'Excitatory = increased pain; inhibitory= reduced pain.
(AMPA = a-amino-3-hydroxy-5-methy/-4-isoxazolepropionic acid; CALCRL = calcitonin receptor-/ike receptor,· cAMP= cyclic
adenosine monophosphate; GABA = y-aminobutyric acid; Gp = group; GPCR = G-protein-coupled receptor,· NMDA = N-methy/-o-
aspartate; OP = opioid; ORL -1 = opioid receptor-/ike 1)

activity at the spinal cord level. Inhibitory amino is not controlled by the background analgesia.
acid neurotransmitters include glycine and Hyperalgesia occurs in a painful area when the
GABA, with neuropeptides such as j3-endorphin pain experienced is much greater than would
and galanin having inhibitory actions, although be expected from the painful stimulus. Pain
these may be altered in some chronic pain can also occur spontaneously without any
states. precipitating stimulus, and may be related to
spontaneously occurring electrical discharges in
34.3. Answer: D. injured nerves.
When normally non-painful stimuli become
painful (either thermal or mechanical), the term 34.4. Answer: A.
'allodynia' is used. This can occur in Assessment has shown features typical of
neuropathic pain, and may be associated with diabetic neuropathy: stocking distribution,
other sensory changes, resulting in other reduced sensation to pin-prick and mechanical
symptoms such as formication - the sensation allodynia. This man needs multidisciplinary
of insects crawling over the skin. Breakthrough management to address tl;le range of issues
pain tends to occur when the usual pain is that are affecting him. Psychosocial factors
controlled by analgesia and something (such as need to be considered with support in active
movement) precipitates an increase in pain that rehabilitation, and use of non-pharmacological

downloaded from www.medicalbr.com


'f
398 • PAIN AND PALLIATIVE CARE

techniques to self-manage his pain. Support in assess physical and psychological effects of
managing his diabetes better will reduce the chronic pain in general (Box 34.5).
risk of worsening symptoms, and lifestyle/
dietary advice is needed to increase what he is 34.6. Answer: A.
able to do. The renal impairment means that While all of the above may impact on memory,
anti-neuropathic drugs such as pregabalin and the likeliest cause is borderline opioid toxicity.
duloxetine need to be used with care, as Elderly patients with limited reserve in terms of
toxicity may result on lower doses than cognitive function are much more sensitive to
expected. even low doses of opioids (Box 34.6).

34.5. Answer: D. 34.7. Answer: B.


The Brief Pain Inventory was originally designed The diagnosis of CRPS type 1 requires a
for use in cancer patients and measures pain combination of sensory, vasomotor, sudomotor
intensity and its interference with different and motor changes to be present. It can occur
aspects of life. It does not assess the character after an injury such as a fracture, without
of the pain, which is necessary to make a definite peripheral nerve lesion. CRPS type 2
diagnosis of neuropathic pain. Pain Detect occurs if there is a defined peripheral nerve
does do this, and has been validated in a range lesion. CRPS is commoner in females between
of neuropathic pain types. The other measures the ages of 35 and 50, occurring in about 20

34.5 Instruments used in the assessment of pain and its impact


Instrument Comments
Brief Pain Inventory Developed for use in cancer pain, validated and widely employed for chronic pain; based
on 0-10 ratings of pain intensity and the impact of pain on a range of domains, including
sleep, work and enjoyment of life
Pain Detect, s-LANSS, DN-4 A number of screening questionnaires to aid diagnosis of neuropathic pain
Pain Catastrophising Scale Developed to assess individual levels of catastrophising, encompassing three different
domains: helplessness, rumination and magnilication
Tampa Scale of Kinesiophobia Measures how much an individual is fearful of movement
Pain Self-efficacy Assesses individual beliefs about self-efficacy in the context of chronic pain, and how this
Questionnaire impacts on function
Visual analogue scale (VAS) Patient marks pain intensity on a horizontal Iine
Localisation of pain Body chart, allowing the patient to indicate where pain is situated
Beck Depression Inventory Assesses emotional function
SF-36/EQ-5D Assesses health-related quality of life
(DN-4 = Douleur Neuropathique questionnaire; EQ-50 = EuroOol 5-Domain questionnaire; SF-36 = Short Form 36; s-IANSS =
Leeds Assessment of Neuropathic Signs and Symptoms)

34.6 Challenges in pain assessment in particular patient populations


Patient population Challenges Co mments
Paediatric Assessment needs to be appropriate to Co nsider visual tools to aid pain assessment
develop mental stage
Elderly May have impaired cognitive function Co nsider formal assessment of cognitive
Cultural factors may reduce self-reporting of pain fun ction
Risk of adverse effects of medication increased Co nsider non-verbal assessment
Co nsider visual tools to assess pain
Em ploy a number of tools assessing pain
be haviours
Cognitive Reporting and expression of pain may change Pe rform formal assessment of cognitive
impairment Increased sensitivity to central nervous system fun ction
effects of analgesics Use non-verbal assessment: facial expressions,
vo calisations, body movements, changes in
so cial interactions
Substance Response to analgesics altered Se ek specialisfsupport early
misuse Increased tolerance En sure prescribing is sate
Increased risk of addiction
Substance misuse may affect reporting of pain

downloaded from www.medicalbr.com


I
PAIN AND PALLIATIVE CARE • 399

per I 00 000 individuals. Active rehabilitation 34.12. Answer: D.


with early physiotherapy is important in Palliative care is the active total care for
management. patients with incurable disease. A palliative care
approach is likely to be suitable where
34.8. Answer: B. symptom management becomes more
This is a challenging pain syndrome to manage, important than aggressive treatment of the
with very limited evidence for efficacy of underlying disease, although the two goals of
long-term opioids, and growing evidence of treatment can exist side by side. Although
possible harms. While opioids may be required initially targeted at patients with malignant
for short-term use, regular use of short-acting disease, patients with other life-limiting
strong opioids should be avoided if possible. conditions such as COPD are now often
Support in developing pain management suitable for palliative care input. There are
strategies, maintaining function, ensuring usually no stringent guidelines regarding
adequate nutrition and reducing opioids should physiological parameters. Prognosis is often
be the goal. Strong opioids should not be difficult to judge, but key events such as an
stopped abruptly unless the patient is at risk of admission to higher-level care are good
overdose. opportunities to consider the future and
make anticipatory care plans for the next
34.9. Answer: A. exacerbation. Opiate medication is a possibility,
The pain has responded to a weak opioid, but but in the first instance he may benefit from
either some tolerance has developed or there is non-pharmacological treatments and an
disease progression. He is, therefore, likely to assessment as to whether his inhaled
get better analgesia from a strong opioid. medication is optimised.
Diazepam is not an analgesic, although it can
be used for short-term anxiolysis. While there 34.13. Answer: A.
may be neuropathic features, a strong opioid It is likely that this patient has liver capsule pain
should be started first. Non-steroidal as a result of metastases from his previous
anti-inflammatory drugs (NSAIDs) should be colonic carcinoma. This type of pain responds
avoided if there is a history of peptic ulcer poorly to opiates and is best treated by
disease. Adding in paracetamol is unlikely to be glucocorticoids. An antispasmodic will not
effective, as he is already taking paracetamol in relieve the stretch of the liver capsule. An
the compound preparation (co-codamol) and NSAID is better suited to ischaemic or bone
may result in exceeding safe dosing limits. pain and gabapentin to neuropathic pain
(Box 34.13).
34.1 0. Answer: A.
While all of the options given can be used in 34.14. Answer: A.
the management of breathlessness, it is Nausea and vomiting can occur initially with
important to assess for treatable causes first, opiate therapy but usually settle after a few
such as pleural effusion, cardiac failure or days. Dry mouth and constipation are
bronchospasm, even though the patient is long-term effects, however, and will need
being managed palliatively. Options B-E ongoing treatment. Tolerance usually develops
can all be useful in the management of to drowsiness, so this problem is often
breathlessness, but it is important to diagnose transient after an increase in dose. Opiate
and start to treat any potentially reversible toxicity is an ongoing concern for any patient
underlying cause first. on this medication and so follow-up review
is advisable. This patient may need
34.11. Answer: A. adjustments to her medication over time
There are many potential causes for confusion depending on increasing pain, changes in
and nausea, including raised intracranial renal function or development of toxicity,
pressure and hypoxaemia due to a if it arises.
cardiovascular problem. While many of these Patients who develop renal failure are at
tests may be appropriate, rapid checking of particular risk of opiate toxicity and
blood results allows exclusion of correctable consideration should be given to ·swapping
causes such as electrolyte imbalance and, from morphine to an alternative opiate such as
particularly, hypercalcaemia. alfentanil.

downloaded from www.medicalbr.com


400 • PAIN AND PALLIATIVE CARE

34.13 Common types of pain in cancer


Type of pain Features Management options
Bone pain Tender area over bone NSAIDs
Possible pain on movement Bisphosphonates
Radiotherapy
Increased intracranial Headache, worse in the morning, associated with vomiting Glucocorticoids
pressure and occasionally confusion Radiotherapy
Codeine
Abdominal colic Intermittent, severe, spasmodic, associated with nausea or Antispasmodics
vomiting Hyoscine butylbromide
Liver capsule pain Right upper quadrant abdominal pain, often associated with Glucocorticoids
tender enlarged liver
Responds poorly to opioids
Neuropathic pain Spontaneous pain Anticonvulsants:
Light touch, pressure and temperature changes are painful; Gabapentin
increased pain on pin-prick Pregabalin
Numbness, tingling or loss of temperature sensation Antidepressants
Skin feels abnormal Amitriptyline
Duloxetine
Ketamine
lschaemic pain Diffuse, severe, aching pain associated with evidence of NSAIDs
poor perfusion Ketamine
Responds poorly to opioids
Incident pain Episodic pain usually related to movement or bowel spasm Intermittent
short-acting opioids
Nerve block
(NSA/Ds ~ non-steroidal anti-inflammatory drugs)

34.15. Answer: E. oxygen therapy reduces the sensation of


Breathlessness is one of the most common breathlessness in advanced cancer any better
symptoms in palliative care and is distressing than cool air flow, and oxygen is indicated only
for both patients and carers. Patients with if there is significant hypoxia. In this case,
breathlessness should be fully assessed to oxygen is also likely to be precluded by both
determine whether there is a reversible cause, the husband and wife continuing to smoke in
such as a pleural effusion, heart failure or the house.
bronchospasm; if so, this should be managed Opioids, through both their central and their
in the normal way. There is no suggestion in peripheral action, can palliate breathlessness
the scenario here of fluid overload or and might be an alternative in this scenario. If
new infection to justify antibiotics or anxiety is considered to be playing a significant
diuretics. role, a quick-acting benzodiazepine, such as
There are many potential causes of lorazepam (used sublingually for rapid
dyspnoea in cancer patients and in other absorption), may also be useful. Citalopram
chronic diseases; apart from direct involvement would be useful for longer-term treatment of
of the lungs, muscle loss secondary to established anxiety or depression, but would
cachexia, anxiety and fear can all contribute. not give instant relief in the short term in the
~ A cycle of panic and breathlessness, often
associated with fear of dying, can be dominant.
way that lorazepam would.

I Exploration of precipitating factors is important


and patient education about breathlessness
34.16. Answer: D.
Although this patient is very unwell, her
and effective breathing has been shown to be hypercalcaemia may be amenable to treatment.
I effective. Non-pharmacological approaches that Even if it is not life.saving, it will make her feel
include using a hand-held fan, pacing, and
I following a tailored exercise programme can
more comfortable and reducing her calcium
level will be the most effective way to help her

I help. There is no evidence to suggest that vomiting. The other treatments may have a role

downloaded from www.medicalbr.com


PAIN AND PALLIATIVE CARE • 401

34.17 How to manage a patient who is dying


Patient and family awareness Support for family
Assess patient's and family's awareness of the situation Make sure you have contact details for family, that you know
Ensure family understands plan of care when they want to be contacted and that they are aware of
Medical interventions facilities available to them
Stop non-essential medications that do not contribute to Religious and spiritual needs
symptom control Make sure any particular wishes are identified and followed
Stop inappropriate investigations and interventions, including Ongoing assessment
routine observations Family's awareness of condition
Resuscitation Management of symptoms
Complete Do Not Attempt Cardiopulmonary Resuscitation Need for parenteral hydration
(DNACPR) form Care after death
Deactivate implantable defibrillator Make sure family know what they have to do
Symptom control Notify other appropriate health professionals
Ensure availability of parenteral medication for symptom
relief

in due course, but would be of lower priority if they contribute to these ends. In the case of
than addressing her hypercalcaemia. this patient, he has no oral intake and is
undistressed; therefore diuretics are unlikely to
34.17. Answer: D. help and intravenous treatment will be
When patients with any advanced condition unnece~sarily burdensome. Religious and
become comatose and unable to take spiritual support are very important in this
medication or oral intake with no reversible situation, for the family as much as the patient.
cause, they are likely to be dying. Although Priority should be given to checking the
many will die within 2-3 days, this stage understanding of family members regarding
of life is often unpredictable and doctors should the situation and their wishes regarding
be cautious in any prognosis they give to care, visiting and how they wish to be
families. contacted.
Once the conclusion has been reached that Although the patient does not currently have
a patient is dying, there is a significant change any symptoms at present, it is possible that he
in management (Box 34.17). Symptom control, may develop them at some point in future. It is
relief of distress and care for the family become important to ensure availability of parenteral
the most important elements of care. medication for symptom relief so that it can be
Medication and investigation are only justifiable given without delay should the need arise.

downloaded from www.medicalbr.com


S. Jenks

Laboratory reference ranges


Notes on the International System of Units (SI Units)
Systeme International (81) d'Unites are a Exceptions to the use of Sl units
specific subset of the metre-kilogram-second By convention, blood pressure is excluded from
system of units and were agreed on as the the 81 unit system and is measured in mrnHg
everyday currency for commercial and scientific (rnillimetres of mercury) rather than pascals.
work in 1960, following a series of international Mass concentrations such as g/L and j.lg/L
conferences organised by the International are used in preference to molar concentrations
Bureau of Weights and Measures. 81 units have for all protein measurements and for
been adopted widely in clinical laboratories but substances that do not have a sufficiently
non-81 units are still used in many countries. well-defined composition.
For that reason, values in both units are given Some enzymes and hormones are measured
for common measurements throughout this by 'bioassay', in which the activity in the
textbook and commonly used non-81 units are sample is compared with the activity (rather
shown in this chapter. The 81 unit system is, than the mass) of a standard sample that is
however, recommended. provided from a central source. For these
Examples of basic Sl units assays, results are given in standardised 'units'
Length metre (m) (U/L), or 'international units' (IU/L), which
Mass kilogram (kg) depend on the activity in the standard sample
Amount of substance mole (mol)
and may not be readily converted to mass
Energy joule (J)
Pressure pascal (Pa) units.
Volume The basic Sl unit of volume is the
cubic metre (1000 litres). For
convenience, however, the litre (L) is
used as the unit of volume in
laboratory work.

Examples of decimal multiples and submultiples of


Sl units
Factor Name Prefix
106 mega- M
103 kilo- k
10-1 deci- d
10-' centi- c
10-3 milli- m
10-6 micro- ll
to-' nano- n
to-" pico- p
to-15 femto- f
I

I
I
downloaded from www.medicalbr.com
LABORATORY REFERENCE RANGES • 403

Laboratory reference ranges in adults


Reference ranges are largely those used
35.1 Urea and electrolytes in venous blood
in the Departments of Clinical Biochemistry
Reference range
and Haematology, Lothian Health
Analysis Sl units Non-SI units
University Hospitals Division, Edinburgh,
Sodium 135-145 mmoi/L 135-145 mEq/L
UK. Values are shown in both 81 units and,
Potassium• 3.6-5.0 mmoi/L 3.6-5.0 mEq/L
where appropriate, non-81 units. Many Chloride 95-1 07 mmoi/L 95-1 07 mEq/L
reference ranges vary between laboratories, Urea 2.5-6.6 mmoi/L 15-40 mg/dl
depending on the assay method used and on Creatinine
other factors; this is especially the case Male 64-111 j.!moi/L 0. 72-1.26 mg/dl
for enzyme assays. Reference ranges Female 50-98 j.!moi/L 0.57-1.11 mg/dl
and the definition of 'abnormal' results are 'Serum values are, on average, 0. 3 mmoi/L higher than
based on distributions in the population plasma values.
of interest and may differ between settings.
No details are given here of the collection 35.2 Analytes in arterial blood
requirements, which may be critical to Reference range
obtaining a meaningful result. Unless Analysis Sl units Non-SI units
otherwise stated, reference ranges shown Bicarbonate 21-29 mmoi/L 21-29 mEq/L
apply to adults; values in children may be Hydrogen ion 37-45 nmoi/L pH 7.35-7.43
different. PaGOz 4.5--{).0 kPa 34-45 mmHg
Many analytes can be measured in PaOz 12-15 kPa 90-113 mmHg
either serum (the supernatant of clotted Oxygen saturation >97%
blood) or plasma (the supernatant of
anticoagulated blood). A specific requirement
for one or the other may depend on a kit
manufacturer's recommendations. In other
instances, the distinction is critical. An example
is fibrinogen, where plasma is required, since
fibrinogen is largely absent from serum. In
contrast, serum is required for electrophoresis
to detect paraproteins because fibrinogen
migrates as a discrete band in the zone of
interest.

35.3 Hormones in venous blood


Reference range
Hormone Sl units Non-SI units
Adrenocorticotrophic hormone 1.5-13.9 pmoi/L (0700-1 000 hrs) 63 ng/L
(ACTH) (plasma)
Aldosterone
Supine (at least 30 mins) 30-440 pmoi/L 1.09-15.9 ng/dl
Erect (at least 1 hr) 11 0-860 pmoi/L 3.97-31.0 ng/dl
Cortisol Dynamic tests are required
Plasma cortisol > 500 nmoi/L (approximately 18 j.!g/dl)* either at baseline or
at 30 mins post 250 !!9 ACTH 1_24 (Synacthen) by IM injection
Follicle-stimulating hormone (FSH)
Male 1.0-10.0 lUlL . 0.2-2.2 ng/ml
Female 3.0-1 0.0 IU/L (early follicular) 0.7-2.2 ng/ml
>30 IU/L (post-menopausal) >6.7 ng/ml
Continued

downloaded from www.medicalbr.com


f
. 404 • LABORATORY REFERENCE RANGES

35.3 Hormones in venous blood - cont'd


Reference range
Hormone Sl units Non-SI units
Gastrin (plasma, fasting) <40 pmoi/L <83 pg/ml
Growth hormone (GH) Dynamic tests are usually required - see Ch. 18
< 0.5 flg/L excludes acromegaly (if insulin-like growth factor < 2 miU/L
1 (IGF-1) in reference range)
> 6 flg/L excludes GH deficiency > 18 miU/L
Insulin Highly variable and interpretable only in relation to plasma glucose and body
habitus
C-peptide Highly variable and interpretable only in relation to plasma glucose and body
habitus. However very low levels are found in conditions of insulin deficiency
e.g. type 1 diabetes
Luteinising hormone (LH}
Male 1.0-9.0 IU/L 0.1-1.0 f.Lg/L
Female 2.0-9.0 IU/L (early follicular) 0.2-1.0 f.Lg/L
> 20 IU/L (post-menopausal) > 2.2 f.Lg/L
17p-oestradiol
Male <160 pmoi/L <43 pg/ml
Female: early follicular 75-140 pmoi!L 20-38 pg/mL
mid-follicular 100-453 pmoi/L 27-123 pg/ml
post-menopausal <150 pmoi/L <41 pg/ml
Parathyroid hormone (PTH} 1.6-6.9 pmoi/L 16-69 pg/ml
Progesterone (in luteal phase in women)
Consistent with ovulation >30 nmoi/L >9.3 ng/ml
Probable ovulatory cycle 15-30 nmoi!L 4.7-9.3 ng/mL
Anovulatory cycle <10 nmoi!L <3 ng/mL
Prolactin (PRL} 60-500 miU/L -
Renin concentration
Supine (at least 30 mins) 5-40m1U/L -
Sitting (at least 15 mins) 5-45miU/L -
Erect (at least 1 hr) 16-63miU/L -
Testosterone
Male 10-38 nmoi/L 2.9-10.9 ng/ml
Female 0.3-1.9 nmoi!L 0.1-0.9 ng/ml
Thyroid-stimulating hormone (TSH} 0.2-4.5 miUIL -
Thyroxine (free}, (free T4) 9-21 pmoi!L 700-1632 pg/dl
Triiodothyronine (free}, (free T3} 2.6-6.2 pmoi/L 160-400 pg/dl

Notes gender (e.g. testosterone), age (e.g. FSH in


1 . A number of hormones are unstable and women), pregnancy (e.g. thyroid function tests,
collection details are critical to obtaining a prolactin), time of day (e.g. cortisol) or
meaningful result. Refer to local laboratory regulatory hormones (e.g. insulin and glucose,
handbook. PTH and [Ca2•]).
2. Values in the table are only a guideline; 3. Reference ranges are usually dependent on
hormone levels can often be meaningfully the method used for analysis.
understood only in relation to factors such as

downloaded from www.medicalbr.com


LABORATORY REFERENCE RANGES • 405

35.4 Other common analytes in venous blood

Analyte
a, -Antitrypsin
Alanine aminotransferase (ALT)
Albumin
Reference range
Sl units
1.1-2.1 g/L
10-50 U/L
35-50 gil
Non-SI units
11 0-21 0 mg/dl
-
3.5-5.0 g/dl
-
Alpha-fetoprotein <10 ng/ml 1000 ng/ dL
Alkaline phosphatase 40-125 U/L -
Amylase <100 U/L -
Aspartate aminotransferase (AST) 10-45 U/L -
Bilirubin (total) 3-16 j.!mOI/L 0.18-0.94 mg/dl
Calcium (total) 2.1-2.6 mmoi/L 4.2-5.2 mEq/L
or 8.5-10.5 mg/dl
Carboxyhaemoglobin 0.1-3.0% -
Levels of up to 8% may be found in
heavy smokers
Caeruloplasmin 0.16-0.47 g/L 16-47 mg/dl
Cholesterol Target cholesterol levels will vary according to individual cardiovascular
risk, with stricter goals for those most at risk. Normal ranges are
misleading as they cover both a healthy and unhealthy population. A
rough guide to treatment targets is given below, but further guidance,
e.g. National Cholesterol Education Programme Adult Treatment Panel Ill
(ATPIII) guidelines, should be used for individual case management. See
Box 35.15 for treatment guidelines in diabetes
Approx Treatment Targets:
Cholesterol (total) (see also Fig. 35.1) <5-5.2 mmol/1 <200 mg/dl
LDL -cholesterol <2-2.5 mmol/1 <100 mg/dL
HDL-cholesterol
Low <1.0 mmoi/L <40 mg/dl
High (desirable) ~1.5 mmoi/L ~60 mg/dl
Complement
C3 0.81-1.57 g/L -
C4 0.13-1.39 g/L -
Total haemolytic complement 0.086-0.410 g/L -
CA-125 <35 U/ml
CEA <3 ng/ml
Copper 10-22 j.!moi/L 64-140 j.!g/dl
C-reactive protein (CRP) <5 mg/L
Highly sensitive CRP assays also exist that measure lower values and
may be useful in estimating cardiovascular risk
Creatine kinase (CK; total)
Male 55-170 U/L -
Female 30-135 U/L -
Creatine kinase MB isoenzyme < 6% of total CK -
Ethanol Not normally detectable
Marked intoxication 65-87 mmoi/L 300-400 mg/dl
Stupor 87-109 mmoi/L 400-500 mg/dl
Coma >109 mmoi/L >500 mg/dl
y-glutamyl transferase (GGT) Male 10-55 U/L -
Female 5-35 U/L
Glucose (fasting) 3.6-5.8 mmoi/L 65-1 04 mg/dL
See Box 35.15 tor definitions of impaired glucose tolerance and diabetes
mellitus. Hypoglycaemia is defined as a blood glucose of less than
3.9 mmoi/L (70 mg/dl)
Glycated haemoglobin (HbA1,) 4.0-6.0% -
20-42 mmol/mol Hb
See Box 35.15 tor diagnosis of diabetes mellitus
Ketones <1 mg/dl
Note: levels can be mildly physiologically
elevated during periods of starvation
Continued

downloaded from www.medicalbr.com


__J
f
. 406 • LABORATORY REFERENCE RANGES

35.4 Other common analytes in venous blood - cont'd


Reference range
Analyte Sl units Non-SI units
Lactate 0.6-2.4 mmoi!L 5.40-21.6 mg/dL
Lactate dehydrogenase (LDH; total) 125-220 U/L -
Lead <0.5 (lmoi/L <10 (lg/dL
Magnesium 0.75-1.0 mmoi/L 1.5-2.0 mEq/L
or 1.82-2.43 mg/dL
Osmolality 280-296 mOsmollkg -
Osmolarity 280-296 mOsmoi/L -
Phosphate (fasting) 0.8-1.4 mmoi/L 2.48-4.34 mg/dl
Protein (total) 60-80 gil 6-8 g/dL
Sex hormone-binding globulin (SHBG) 18.4-75.6 nmoi/L 175-718 (lgidl
Triglycerides (fasting) 0.6-1.7 mmoi/L 53-150 mg/dL
Troponins Values consistent with myocardial infarction are crucially dependent on
which troponin is measured (I or T) and on the method employed.
Interpret in context of clinical presentation.
Troponin I and troponin T are structural cardiac muscle proteins that are
released during myocyte damage and necrosis, and represent the
cornerstone of the diagnosis of acute myocardial infarction. Modern
assays are extremely sensitive, however, and can detect minor degrees
of myocardial damage, so that elevated plasma troponin concentrations
may be observed in conditions other than acute Ml, such as pulmonary
embolus, septic shock and pulmonary oedema.
Tryptase 0-135 mg/L -
Urate
Male 0.12-0.42 mmoi!L 2.0-7 .o mg/dl
Female 0.12-0.36 mmoi!L 2.0--£.0 mg/dl
Vitamin D (25(0H)D)
Normal >50 nmoi!L >20 ng/mL
Insufficiency 25-50 nmoi/L 10-20 ng/ml
Deficiency <25 nmoi/L <10 ng/ml
Zinc 10-18 (lmoi!L 65-118 (lg/dl

Calculated blood values


Anion gap when not including K+ and ~ 0-20 mEq/L when
The anion gap is the difference between the including K+
primary measured cations (sodium Na+ and
potassium K+) and the primary measured anions eGFR
(chloride CJ- and bicarbonate HC03-) in serum The estimated glomera\ filtration rate (eGFR)
A normal anion gap using ion selective is calculated from age, sex and blood creatinine
electrodes is 3- ~ 1 mEq/L level. An adjustment is needed for certain
Previous measures of anion gap using ethnic groups. eGFR levels have been used to
colorimetry gave a normal range of 8-16 mEq/L categorise degrees of renal impairment

Stage of chronic
eGFR (mUmin/1.73 m') kidney disease (CKD) Description
>90 Stage 1 Normal
60-89 Stage 2 Mild reduction (not considered CKD)
45-59 CKD stage 3A Moderately reduced function
30-44 CKD stage 38 Moderately reduced function
15-29 CKD stage 4 Severely reduced function
<15 CKD stage 5 Very severely reduced function/end-stage
kidney failure

downloaded from www.medicalbr.com


LABORATORY REFERENCE RANGES • 407 ·.·'!•i
,I

180

160
Non-smoker
Non-diabetic men

Age under 50 years


180

160
Smoker
-
SBP140 SBP140

120 120

100 100
345678910 345678910
TC:HDL TC:HDL
Age 5o-59 years
180 180

160 160

SBP 140 SBP 140

120 120

100 100
345678910 345678910
TC:HDL TC:HDL
Age 60 years and over
180 180

160 160

SBP 140 SBP 140

120 120

100 100
345678910 345678910
TC:HDL TC:HDL

D CVD risk < 10% over next 10 years


D CVD risk 10-20% over next 10 years
• CVD risk > 20% over next 10 years
,j
SBP = systolic blood pressure (mmHg)
TC:HDL =serum total cholesterol to HDL cholesterol ratio
CVD risk over
II
next 10 years I
'30%

10% 20%

Fig. 35.1

Serum osmolality bicarbonate, glucose and urea. Osmoles per


The serum or plasma osmolality is a kilogram of water defines osmolality.
measurement of the different solutes in plasma. Normal range is 280-296 mOsm/kg.
It is primarily determined by sodium, chloride,

downloaded from www.medicalbr.com


f
. 408 • LABORATORY REFERENCE RANGES

35.5 Common analytes in urine


Reference range
Analyte Sl units Non-SI units
Albumin Definitions of microalbuminuria are given in Box 35.6
Proteinuria is defined below
Calcium (normal diet) Up to 7.5 mmol/24 hrs Up to 15 mEq/24 hrs
or 300 mg/24 hrs
Copper < 0.6 !lmol/24 hrs <38 !lg/24 hrs
Cortisol 20-180 nmol/24 hrs 7.2-65 !lg/24 hrs
Creatinine
Male 6.3-23 mmol/24 hrs 712-2600 mg/24 hrs
Female 4.1-15 mmol/24 hrs 463-1695 mg/24 hrs
5-hydroxyindole-3-ace!ic acid (5-HIAA) 10-42 !lmol/24 hrs 1.9-8.1 mg/24 hrs
Metadrenalines
Normetadrenaline (normetanephrine) 0.4-3.4 !lmol/24 hrs 73-620 !lg/24 hrs
Metadrenaline (metanephrine) 0.3-1.7!-lmol/24 hrs 59-335 !lg/24 hrs
Oxalate 0.04-0.49 mmol/24 hrs 3.6-44 mg/24 hrs
Phosphate 15-50 mmol/24 hrs 465-1548 mg/24 hrs
Potassium* 25-1 00 mmol/24 hrs 25-1 00 mEq/24 hrs
Protein <0.3 g/L <0.03 g/dl
Sodium• (random sample) 40-220 mmoi/L 40-220 mEq/L
Sodium* 100-200 mmol/24 hrs 100-200 mEq/24 hrs
Urate 1.2-3.0 mmol/24 hrs 202-504 mg/24 hrs
Urea 170-6li0 mmol/24 hrs 10.2-36.0 g/24 hrs
Zinc 3-21 !lmol/24 hrs 195-1365 !lg/24 hrs
*The urinary output of electrolytes such as sodium and potassium is normally a reflection of dietary intake. This can vary widely. The
values quoted are appropriate to a 'Western' diet.

Urinalysis
Urinalysis is a point of care test normally
Calculated urine values
assessing the pH (normal range pH 4.6 to pH
8.0), specific gravity (normal range Urine Osmolality
1.005-1.030) and the prescence of Urine osmolality is a measure of the
components such as blood, protein, glucose, concentration of osmotically active particles,
ketones, nitrites, leukocyte esterase, bilirubin principally sodium, chloride, potassium and
and urobilirubin. A normal result is negative for urea; glucose can contribute significantly to the
all parameters. If present, a scale of + to ++++ osmolality when present in substantial
is used to describe the degree of positivity. amounts in urine

35.6 Albumin: Creatinine Ratio (ACR)


Typical dipstick
I
ACR' Category PCR' results' Significance I
I
<2.5 mg/mmol (males) A1 Normal < 25 - Normal
<3.5 mg/mmol (females)
2.5-30 mg/mmol (males) A2 Mildly raised 25-50 - Moderately elevated albuminuria '
3.5-30 mg/mmol (females) Known as 'Microalbuminuria'
30-70 A3 Moderately raised 50-100 +IO++ Dipstick positive
Known as 'Proteinuria' I
70-300 100-350 ++tO+++ Glomerular disease more likely; !
equivalent to > 1 g/24 hrs
Known as 'Heavy Proteinuria'
> 300 > 350 +++to++++ Nephrotic range: almost always
glomerular disease, equivalent to
> 3.5 g/24 hrs
1
Urinary albumin (mg!L)Iurine creatinine (mmoi/L). 2Urine protein (mg!L)Iurine creatinine (mmoi/L). (If urine creatinine is measured in
mg!dL, reference values tor PCR and ACR can be derived by dividing by 11.31.) 'Dipstick results are affected by urine concentration
and are occasionally weakly positive on normal samples. I

downloaded from www.medicalbr.com


- ---
I 1
r Random urine osmolality should average
300-900 mOsm/kg of water
LABORATORY REFERENCE RANGES • 409

35.7 Analytes in cerebrospinal fluid (CSF)


Reference range
Analysis Sl units Non-SI units
Calcium clearance: creatinine clearance ratio
Cells <5x106 cells/L <5cells/mm 3
This test is generally done in the context of (all mononuclear)
hypercalcaemia in association with a normal or Glucose' 2.3-4.5 mmoi/L 41-81 mg/dl
raised plasma PTH level. lgG index' <0.65 -
The ratio is calculated using the following Total protein 0.14-0.45 g/L 0.014-0.045 g/dl
formula: 'Interpret in relation to plasma glucose. Values in CSF are
urine calcium (mmoi/L) x [serum creatinine typically approximately two-thirds of plasma levels.
2
(Jlmoi/L) /1 000] A crude index of increase in immunoglobulin G (lgG)
serum calcium (mmol) x urine creatinine attributable to intrathecal synthesis.
(mmoi/L)
Interpretation 35.8 Analytes in faeces
• Calcium creatinine clearance ratio > 0.01 is Reference range
suggestive of primary hyperparathyroidism Analyte Sl units Non-SI units
(PHPT). Calprotectin <50 f.Lg/g
• Calcium creatinine clearance ratio < 0.01 is Elastase
suggestive of familial hypocalciuric
hypocalcaemia (FHH) but may also be seen
35.9 Analytes in pleural fluid
in PHPT with vitamin D deficiency.
Reference range
• Urine calcium is also usually <200 mg/24 hrs
Analyte Sl units Non-SI units
(50 mmol/24 hrs) in FHH.
Lactate dehydrogenase 125-220 U/L

35.10 Haematological values


Reference range
Analysis Sl units Non-SI units
Activated partial thromboplastin lime (APTT) 24-34secs -
Bleeding time (Ivy) <8 mins -
Blood volume
Male 65-85 mUkg -
Female 60-80 mUkg -
CD4 count >500 cells/mm 3 -
Coagulation screen
Prothrombin lime (Pl) 10.5-13.5 sees -
Activated partial thromboplastin time 26-36secs -
(APTl)
D-dimers
Interpret in relation to clinical presentation <200 ng/ml -
Erythrocyte sedimentation rate (ESR) Higher values in older patients are not necessarily abnormal
Adult male 0-10 mm/hr -
Adult female 3-15 mm/hr -
Erythropoietin (EPO) (adults) 4.1-19.5 mU/mL
Factor VIII RR is 50-150% of normal
Ferritin
Male (and post-menopausal female) 20-300 f.LQ/L 20-300 ng/ml
Female (pre-menopausal) 15-200 f.LQ/L 15-200 ng/ml
Fibrinogen 1.5-4.0 g/L 0.15-0.4 g/dl
Folate
Serum 2.8-20 f.LQ/L 2.8-20 ng/ml
Red cell
Haemoglobin
120-500 f.Lg/L
. 120-500 ng/ml

Male 130-180 g/L 13-18 g/dl


Female 115-165 g/L 11.5-16.5 g/dl I
Haptoglobin 0.4-2.4 g/L 0.04-0.24 g/dl
Continued
I
downloaded from www.medicalbr.com
j
410 • LABORATORY REFERENCE RANGES

35.10 Haematological values- cont'd


Reference range
Analysis Sl units Non-S! units
Iron
Male 14-32 Jlmoi/L 78-1781J.g/dl
Female 10-28 Jlmoi/L 56-157 Jlgldl
Leucocytes (adults) 4.0-11.0x109/L 4.0-11.0x10'/mm'
Differential white cell count
Neutrophil granulocytes 2.0-7.5 X 109/L 2.0-7.5 x 103/mm'
Lymphocytes 1.5-4.0 X 1091L 1.5-4.0 x 103/mm'
Monocytes 0.2-0.8 X 109/L 0.2-0.8 x 103/mm'
Eosinophil granulocytes 0.04-0.4 X 109/L 0.04-0.4 x 103/mm'
Basophil granulocytes 0.01-0.1 x10 9/L 0.01-0.1 x 103/mm'
Mean cell haemoglobin (MCH) 27-32 pg -
Mean cell volume (MCV) 78-98 II -
Packed cell volume (PCV) or
haematocrit
Male 0.40-0.54 -
Female 0.37-0.47 -
Platelets 150-350 X 109/L 150-350 x 103/mm'
Prothrombin time (PT) 10-13.5 sees -
Red cell count
Male 4.5-6.5 X 1012/L 4.5--£.5 x 1O'/mm'
Female 3:8-5.8x1012/L 3.8-5.8 x 106/mm'
Red cell lifespan
Mean 120 days -
Half-life (51 Cr) 25-35 days -
Reticulocytes (adults) 25-85x 109/L 25-85 x 103/mm'
Total iron-binding capacity 43-81 Jlmoi/L 240-453 11g/dl
Transferrin 2.0-4.0 g/L 0.2-0.4 g/dl
Transferrin saturation
Male 25-50% -
Female 14-50% -
Vitamin B,
Normal >210 ng/L -
lntermediatev 180-200 ng/L -
Low <180 ng/L -
von Willebrand factor activity RR is 50-200%
(ristocetin co-factor)

35.11 Immunological reference ranges


Anti-tissue transglutaminase antibody (anti-tTG) Negative <20 U, positive >75 U
Anti-citrullinated peptide antibodies (ACPA) Normal <4.5 U
Anti-DNA antibody titre 0-51U/L
TSH receptor antibodies (TRAbs) 0-1.61UIL
Serum allergen testing (Specific lgE) Interpretation
<0.35 kUIL - no specific lgE antibody detected
0.35-0.70 kU/L - low level
0.70-3.50 kUIL- moderate level
3.50-17.5 kUIL - high level
17.5-100 kUIL- very high level
>100.0 kU/L - extremely high level
Immunoglobulins (lg)
lgA 0.8-4.5 gil
lgE 0-250 kU/L
lgG 6.0-15.0 g/L
lgM 0.35-2.90 g/L

downloaded from www.medicalbr.com


r Laboratory reference ranges in pregnancy
LABORATORY REFERENCE RANGES • 411

The levels of many analytes in blood vary appropriate and it is important for the clinician
during pregnancy, when many hormonal and reviewing the results to be aware of this to
metabolic changes occur. The standard adult enable appropriate interpretation and patient
reference ranges may therefore not be management.

35.12 Analytes that may be significantly affected by pregnancy*


Reference range
Analyte First trimester Second trimester Third trimester
Alkaline phosphatase 17-88 U!L 25-126 U!L 38-229 U/L
Packed cell volume (PCV) or haematocrit 0.31-0.41 0.30-0.39 0.28-0.40
Haemoglobin 116-139 g/L 97-148 g/L 95-150 g/L
Human chorionic gonadotrophin 4 weeks: 16-156 lUlL 4270-1 03 000 IU/L 2700-78 300 IU/L
4-9 weeks:
101-233 000 lUlL
9-13 weeks:
20 900-291 000 IU/L
17p-Oestradiol 690-9166 pmoi/L 4691-26 401 pmoi/L 12 701-22 528 pmoi/L
(188-2497 pg/mL) (1278-7192 pg/mL) (3460-6137 pg/mL)
Progesterone 25-153 nmoi/L Not available 314-1088 nmoi/L
(8-48 ng/mL) (99-342 ng/mL)
Prolactin 765-4532 miU/L 2340-7021 miU/L 2914-7914 miU/L
Thyroid-stimulating hormone (TSH) 0.60-3.40 miU/L 0.37-3.60 miU/L 0.38-4.04miU/L
Thyroxine (free), (free T4) 10-18 pmoi/L 9-16 pmoi/L 8-14 pmoi/L
(777-1399 pg/dL) (699-1243 pg/dL) (621-1 088 pg/dL)
Won-Sf equivalents are given in brackets where appropriate.

Laboratory reference ranges in childhood and adolescence


The levels of many analytes in blood vary and puberty on interpretation of the results.
due to the physiological changes that occur For example, a creatinine of 70 11moi/L
during growth and adolescence. Hospital (0.79 mg/dl) is perfectly normal for the
laboratories may provide reference ranges majority of adults but may indicate significant
that are age-adjusted or based on pubertal renal impairment in a child. Reference
stage but this is not always the case. It is ranges for hormone results are described
therefore important for the doctor requesting according to the Tanner stages of puberty
these tests to understand the impact of age (see Fig. 35.2).

35.13 Analytes that may be significantly affected by growth and puberty*


Analyte Age/pubertal stage Gender Reference range
Alkaline phosphatase (ALP) <1 year M, F 80-580 U/L
1-16 years M, F 100-400 U/L
16-20 years M 50-250 U!L
F 40-200 U/L
Creatinine <1 year M, F 12-39 [!moi/L
(0.14-0.44 mg/dL)
1-4 years M, F 13-42 [!moi/L
(0.15-Q.48 mg/dL)
4-12 years M, F 20-57 [!moi/L
(0.23-Q.64 mg/dL)
12-15 years M, F 31-67 [!moi/L
(0.35-0. 76 mg/dL)
Continued

downloaded from www.medicalbr.com


4 ~ 2 • LABORATORY REFERENCE RANGES

35.13 Analytes that may be significantly affected by growth and puberty•- cont'd

Analyte Age/pubertal stage Gender Reference range


15-18 years M 39-92 11moi/L
(0.44-1.04 mg/dL)
F 34-72 11moi/L
(0.38-0.81 mg/dL)
Follicle-stimulating hormone Prepubertal M <3.0 IU/L
(<0.6 ng/mL)
F <3.21U/L
(<0.64 ng/ml)
Pubertal stage 2 M <6.61U/L
(< 1.32 ng/ml)
F <4.11U/L
(<0.82 ng/ml)
Pubertal stage 3 M 0.7-5.0 IU/L
(0.14-1 ng/ml)
Pubertal stages 4-5 M 1.5-6.0 IU/L
(0.3-1.2 ng/ml)
Pubertal stages 3-5 F 2.5-13.5 IU/L
(0.5-2. 7 ng/ml)
Insulin-like growth factor 1 <7 years M 15-3491-lg/L
F 17-2721-lg/L
8-16 years M 67-510 11g/L
F 59-5021-lg/L
Luteinising hormone Prepubertal M <1.0 IU/L
(<0.11lg/L)
Pubertal stage 2 M <3.0 IU/L
(<0.31lg/L)
Prepubertal and pubertal stage 2 F <1.0 IU/L
(<0.1 llg/L)
Pubertal stage 3 M 1.0-4.0 IU/L
(0.1-0.4 llg/L)
Pubertal stages 4-5 M 1.0-5.0 IU/L
(0.1-0.6 11g/L)
Pubertal stages 3-5 F 1.0-8.0 IU/L
(0.1-0.9 119/L)
17p-oestradiol Prepubertal and pubertal stages 2-3 M <75 pmoi/L
(<20 pg/ml)
Prepubertal and pubertal stage 2 F <100 pmoi/L
(<27 pg/mL)
Pubertal stages 4-5 M <130 pmoi/L
(<35 pg/ml)
Pubertal stages 3-5 F <150 pmoi/L
(<41 pg/ml)
Testosterone Prepubertal M <0.5 nmoi/L
(< 0.1 ng/ml)
F <0.6 nmoi/L
(< 0.2 ng/ml)
Pubertal stage 2 M <10.6 nmoi/L
(< 3.1 ng/ml)
F <1.4 nmoi/L
(< 0.4 ng/ml)
Pubertal stage 3 M 0.4-30 nmoi/L
(0.1-8.7 ng/ml)
Pubertal stage 4 M 5.6-30 nmoi/L
(1.6-8. 7 ng/ml)
Pubertal stage 5 M 10-30 nmoi/L
(2.9-8. 7 ng/ml)
Pubertal stages 3-5 F 0.4-1.9 nmoi/L
(0.1-0.5 ng/ml)
'Non-S/ equivalents are given in brackets where appropriate.

downloaded from www.medicalbr.com


r
'
LABORATORY REFERENCE RANGES • 413

Tanner
stage
Ill IV v
Female

Mature stage.
Elevation of breast Further enlargement Projection of Projection of papilla
Pre-adolescent and papilla as a of breast and areola areola and papilla with recession of
Breast
small mound with no separation to form mound areola to contour
of contours above breast of breast

y r r T T
Darker, coarse and
curled hair but Dark, coarse and
Pubic None Sparse, long and Darker, coarse and covering smaller curled hair
hair straight curled hair area than in adult. extending to inner
No spread to medial thighs
surface of thighs
Male

r
\_ / \ /

r l ) )
Genitalia

Pubic
hair
Pre-adolescent

None
Growth oftestes
and scrotum. Skin
on scrotum
reddens and
becomes wrinkled

Sparse, long and


straight
' '
Growth of penis and
further growth of
testes and scrotum.
Skin of scrotum
becomes darker and
more wrinkled

Darker, coarse and


curled hair
Further growth in
length and width
of penis, testes
and scrotum

Darker, coarse and


curled hair but
covering smaller
Penis, testes
and scrotum of
adult size

Dark, coarse and


curled hair
extending towards
area than in adult umbilicus

Fig. 35.2

35.14 Diagnostic cut-offs in diabetes and other glucose-related disorders


Diabetes is confirmed by:
• either plasma glucose in random sample or 2 hrs after a 75 g glucose load ~11.1 mmoi/L (200 mg/dl) or
• fasting plasma glucose~ 7.0 mmoi/L (126 mg/dl) or
• HbA1, ~ 48 mmol/mol
In asymptomatic patients, two diagnostic tests are required to confirm diabetes; the second test should be the same as the
first test to avoid confusion
'Pre-diabetes' is classified as:
• impaired fasting glucose= fasting plasma glucose~ 6.1 mmoi/L (110 mg/dl) and< 7.0 mmoi/L (126 mg/dl)
• impaired glucose tolerance= fasting plasma glucose < 7.0 mmoi/L (126 mg/dl) and 2-hr glucose after 75 g oral glucose
drink 7.8-11.1 mmoi/L (140-200 mg/dl)
HbA1, criteria for pre-diabetes vary. The National Institute for Health and Care Excellence (NICE) guidelines (UK) recommend
considering an HbA1, range of 42-47 mmol/mol to be indicative of pre-diabetes; the American Diabetes Association (ADA)
guidelines suggest a range of 39-47 mmol/mol. The ADA also suggests a lower fasting plasma glucose limit of~ 5.6 mmoi/L
(100 mg/dl) for impaired fasting glucose.
Gestational diabetes is classified by:
Plasma glucose ~5.6 mmoi/L (100 mg/dl) (fasting) 27.8 mmoi/L (140 mg/dl) 120 mins post 75 g OGTl according to NICE
2015 guidelines, but other regions may differ

downloaded from www.medicalbr.com


l
f
414 • LABORATORY REFERENCE RANGES

There are many national and international and local policy may differ. Targets given are
guidelines for diabetes. The targets/threshold according to the NICE 2015 diabetes guidelines
discussed below relate to current UK guidelines unless otherwise specified

35.15 Diabetes target ranges


HbA,c Type 2 diabetes managed by lifestyle/ diet+/- a single drug not associated with
Targets should be personalized to hypoglycaemia: aim for an HbA1c level of 48 mmol/mol (6.5%)
the individual. However broad Type 2 diabetes managed with a drug associated with hypoglycaemia: aim for an
guidance is given here HbA1c level of 53 mmol/mol (7.0%)
Type 1 diabetes: aim for a target HbA1c level of 48 mmol/mol (6.5%) or lower
Blood pressure *Type 2 diabetes: <140/80 mmHg, or <135/75 mmHg if microalbuminuria or
proteinuria is present
Type 1 diabetes: <135/85 mmHg, or <130/80 mmHg with nephropathy
Cholesterol There is no absolute agreed guidelines target levels for cholesterol levels but all
patients over 40 years should be offered stalin therapy. See Table 35.4 for
cholesterol targets
BMI Normal range (Caucasian) = 20-25 kg/m'
(lower in high-risk ethnic groups: e.g. 20-23 kg/m' in Asian populations)

Further information Tanner JM, Whitehouse RH. Clinical longitudinal


Abbassi-Ghanavati M, Greer LG, Cunningham standards for height, weight, height velocity,
FG. Pregnancy and laboratory studies: a weight velocity, and stages of puberty. Arch Dis
reference table for clinicians. Obstet Gynecol Child 1976; 51:170-179.
2009; 114:1326-1331.

downloaded from www.medicalbr.com


Colour illustrations
These illustrations are shown here in colour. They are also reproduced in black and white
alongside their associated questions.

Fig. 16.21

111!JI[[[[01
:UlLLLUJllilU
Fig. 13.5

v~

Fig. 16.25

downloaded from www.medicalbr.com


4 16 • COLOUR ILLUSTRATIONS

Fig. 17.15a Fig. 17.15b

downloaded from www.medicalbr.com


COLOUR ILLUSTRATIONS • 417

Fig. 27.12 Fig. 27.15

Fig. 27.13

downloaded from www.medicalbr.com


418 • COLOUR ILLUSTRATIONS

(")
3
Fig. 29.7

Fig. 29.1

Fig. 29.5

Fig. 29.6 Fig. 29.10

downloaded from www.medicalbr.com


COLOUR ILLUSTRATIONS • 419

CM
Fig. 29.13 Fig. 29.28

Fig. 29.23 Fig. 29.37

downloaded from www.medicalbr.com


420 • COLOUR ILLUSTRATIONS

Fig. 29.48

Fig. 29.42

Fig. 29.53

Fig. 29.46

downloaded from www.medicalbr.com


COLOUR ILLUSTRATIONS • 421

Fig. 34.7

downloaded from www.medicalbr.com


downloaded from www.medicalbr.com
Index
Page numbers followed by "I" indicate figures, "t" indicate tables, and "b" indicate boxes.

A Acute stress reaction, 342


Abacavir, 101, 101b Acute systemic type I hypersensitivity, unusual clinic features of,
Abdominal aortic aneurysm 23,26
diabetes mellitus and, 139, 150 Acute tubular necrosis (ATN), 127
smoking cessation for, 139, 150 Acute viral pericarditis, 136, 147
Abdominal colic, 400b Adalimumab, for plaque psoriasis, 352, 362
ABO blood group, 266, 274, 274b Adaptive immunity, 22, 25
Abrus precatorius Gequirity bean), 45b Addison's disease, 193
Acamprosate, for Wernicke-Korsakoff syndrome, 343- 344 in hyponatraemia, 381
Acanthosis nigricans, 392 Adenocarcinoma, 236, 390
due to hyperinsulinaemia, 185, 193 Adenoma, toxic, 187, 1871, 195
Acclimatisation, 51-52 Adenosine, 145
ACE inhibitor, 126, 129, 150, 223, 361 Adie's pupil, 330-331, 331 t, 333
Acetylcysteine, 42b Adjustment disorders, 342
Achalasia, 225, 235 prevalence of, 341b
Aciclovir, 33, 35, 103, 105 Adolescent
for herpes B virus infection, 85, 95 cause of death in, 370, 374
Acitretin w ith diabetes, 373, 375
for eczema, 364 juvenile idiopathic arthritis in, 373, 375, 376b
pregnancy and, 351, 361-362 risk of low adherence, 370, 373-374
Aconitum terox (Indian aconite, bikh), 45b risk-taking behaviors in, 370-371, 374
Aconitum nape/Ius (aconite, wolf's bane, monkshood), 45b transition medicine and, 370-375
Acoustic neuroma, 312, 323 Adrenaline (epinephrine), 46, 48-49, 389
ACPA. see Anti-citrullinated peptide antibodies in anaphylaxis, 61-62, 71
Acquired immune deficiency syndrome (AIDS), 28, 30 ~-Adrenoceptor, causing lichenoid reactions, 364-365
ACR. see Albumin:creatinine ratio ~-Adrenoceptor antagonists, 134, 145, 195
Acromegaly, MEN 1 and, 193, 202 heart rate control with, 134, 145
ACTH. see Adrenocorticotrophic hormone Adrenocorticotrophic hormone (ACTH), in venous blood,
Actinic keratosis, spontaneous resolution of, 353--354, 3531, 403b-404b
363 Advanced dementia, investigation in, 379, 382
Actinomyces israelii, 92 Adverse drug reactions, hypersensitivity, 7, 10
Activated partial thromboplastin time (APTI), reference range Ageing, and disease, 377-382
of, 409b-41 Ob ~2 -agonists, 322
Acute bacterial infection, causing leucocytosis, 268, 275 AIDS. see Acquired immune deficiency syndrome
Acute cellular rejection, 119-120, 127-128 Alanine aminotransferase (ALT], in venous blood, 405b-406b
Acute Charcot arthropathy, 214, 220-221 Alanine transaminase, 245, 253
Acute gout, 280-281, 291 Albendazole, for neurocysticercosis, 75, 87
Acute hemiballism, 307, 319-320 Albumin
Acute interstitial nephritis, 119, 127 in urine, 408b
Acute kidney injury, Hantavirus infection and, 82, 93 in venous blood, 405b-406b
Acute labyrinthitis, 304, 309, 318, 321 Albumin:creatinine ratio (ACR), 408b
Acute medicine, 54-72 Alcohol
Acute metabolic acidosis, 11 0, 113- 114 abuse, prevalence of, 341b
Acute myeloid leukaemia, 264, 272 excessive consumption of
allogeneic stem cell transplantation and, 265, 273 atrial fibrillation and, 137, 148
childhood, 266, 274 dilated cardiomyopathy due to, 136, 147
Acute myocardial infarction, 148 use, 28, 30
Acute pancreatitis, 233, 242, 242b- 243b Alcohol dehydrogenase inhibitors, 42b
Acute papillary muscle rupture, 138- 139, 149 Alcoholic hepatitis, diagnosis and treatment of, 251,
Acute pericarditis, 149 258-259
Acute respiratory distress syndrome (ARDS) Alcohol-related acute pancreatitis, CJ for, 233, 242
Berlin definition of, 56, 65, 70b Aldosterone, 11 3
variables denoting, 61, 70, 70b in venous blood, 403b-404b
weaning and, 55, 65 Aldosterone synthase, 190, 200
Acute rheumatic fever, c-reactive protein and, 140, 151 Alendronic acid, 162
Acute right ventricular failure, 145 Alfentanil, for intracranial pressure, 63, 71

downloaded from www.medicalbr.com I


"!''_
424 • INDEX

Alkaline phosphatase (ALP) Angiotensin-converting enzyme (ACE) inhibitor, 214, 223,


in growth and puberty, 411 b-412b 386-387
in pregnancy, 411b causing lichenoid reactions, 364-365
in venous blood, 405b-406b Anhedonia, 337-338, 342
Allergic rhinitis, 173, 184 Anion gap, 406
Allodynia, 397 largest, 38, 41-43, 43b
Allopurinol, for acute gout, 280-281, 291 Anions, 406
Allopurinol hypersensitivity reaction, 79, 90-91 Ankylosing spondylitis (AS), 294
Alopecia totalis, wig for, 352, 362 Anorexia nervosa, management of, 338, 343
ALP. see Alkaline phosphatase Anosmia and ageusia, 307, 320
Alpha-fetoprotein, in venous blood, 405b-406b Antalgic gait, 382
Alport's syndrome (hereditary nephritis), 121, 129 Anterior cruciate ligament tear, previous, in development of
ALT. see Alanine aminotransferase osteoarthritis, 281 , 291
Alveolar oedema, 143 Anterior uveitis, as complication of Ebola virus disease, 74 ,
Amanita phailoides (death cap mushroom), 45b 85-86
Amantad ine, 89 Anthracycline, 265, 273
Ambrisentan, 184 chemotherapy, dilated cardiomyopathy and, 142, 152
Amenorrhoea Anthrax, cutaneous, 82, 93
anterior pituitary brain lesion in, 185, 193 Anti-arrhythmic drugs, 40b, 45b
functional hypothalamic, 189, 197-198 Antibiotics, 235, 261, 380
Amino acids, in pain processing, in spinal cord, 397b for papulopustular acne, 359-360
y-Aminobutyric acid, in pain processing, in spinal cord, Antibody fragments, 42b
397b Anticholinergic agents, 40b, 45b
5-aminosalicylic acid (5-ASA), oral, 231, 240 Anticholinesterases, dementia and, 343
Amiodarone, 40b, 45b, 144-147, 196 Anti -citrullinated peptide antibodies (ACPA), reference range of,
moxifloxacin and, 7, 11 410b
thyrotoxicosis induced by, 188, 197 Anticonvulsant therapy, 368
glucocorticoids for, 188, 197 Anticonvulsants, 40b, 44, 45b
Amitriptyline, 232, 241 for neuropathic pain, 400b
for neuropathic pain, 400b Antidepressants, 40b, 45b
Amlodipine, 116,119, 170,191,371,377,380. see also for depression, 340, 344
Calcium channel blockers for neuropathic pain, 400b
Amoxicillin, 105, 159, 165, 176, 215- 216 Anti-DNA antibody titre, reference range of, 41 Ob
for community-acquired pneumonia, during pregnancy, 367, Antiemetics, 322
369 Anti-endomysia! antibody (anti-EMA), coeliac disease and, 229,
for H. pylori infection, 227, 237 239
Amphetamines, 41b-42b, 44b Antiepileptic drug, 44, 368
Amphotericin, 235 for generalised tonic-clonic seizures, 303, 316
liposomal, for leishmaniasis, 81, 92 Antifungal agent, 180
Amphotericin B, 98-99 Anti-glomerular basement membrane (GBM), 116, 124-125
Ampicillin, 151-152 Antihistamines, 41b, 44b, 49
Ampulla of Vater, 232, 242 for eczema, 364
Amylase, in venous blood, 405b- 406b non-sedat1ng, 352
Amyloidosis, 116, 124 Antimalarials, 40b, 45b
ANA. see Antinuclear antibody causing lichenoid reactions, 364-365
Anaemia Antimicrobials, indications for, 84, 95
of chronic disease, 269, 277 Antimuscarinic medication, for incontinence, 377, 380
duodenal hepcidin production and, 204, 208 Anti-neuropathic drugs, 397-398
Anal cancer, 96, 100 Antineutrophil cytoplasmic antibodies (ANCA), 116, 124-125
Anal sphincter, damage to, management of, 231, 240-241 Antinuclear antibody (ANA), 116, 124-125
Analgesia Antiplatelet drugs, 138, 146
in intensive care, 55, 65 Antipsychotic drugs, 40b-41b, 44, 44b-45b, 322
for pain, in oropharyngeal tumour, 394, 399 Antispasmodics, for abdominal colic, 400b
Analytes Antithymocyte globulin (ATG), horse, for severe aplastic
affected by growth and puberty, 411b-412b anaemia, 265, 273
affected by pregnancy, 411 b Antithyroid drugs, 195
in arterial blood, 403b Anti-tissue transglutaminase antibody (anti-tTG), reference
in cerebrospinal fiuid, 409b range of, 41 Ob
in faeces, 409b cx1-antitrypsin, in venous blood, 405b-406b
measuring, 403 Anti-TIG antibody , for coeliac disease, in type 1 diabetes,
in pleural fluid, 409b 217, 223
in urine, 408b Antituberculous drugs, 39b, 170
in venous blood, 405b-406b causing lichenoid reactions, 364-365
Anaphase, definition of, 14, 18 Anti-vascular endothelial growth factor (anti-VEGF) therapy,
Anaphylaxis, 23, 26 intravitreal, 335
Anaplastic carcinoma, 188, 197 Antiviral therapy, oral, for thrombocytopenia, 248, 256-257
ANCA. see Antineutrophil cytoplasmic antibodies Anuria, 118, 127
Anchoring, definition of, 2-3, 5 Anxiety disorders
Androgen, 198 prevalence of, 341 b
production of, in adolescent females, 370, 373 Aortic dissection, 132-143
Angina pectoris, exertional, low risk of future events and, 142, pregnancy and, 139, 150
153 screening for, Turner's syndrome and, 189, 198
Angioedema, ramipril-induced, 349, 361 Aortic incompetence, 143 •
Angioplasty and Stenting for Renal Artery Lesions (ASTRAL), Aortic regurgitation, 143
125 Aortic root dilatation, 143
Angiostrongylus cantonensis, eosinophilic meningitis and, 79, Aortic stenosis, 381
91 parasternal thrill and, 140, 151

downloaded from www.medicalbr.com


INDEX • 425

Aortic valve, infective endocarditis affecting, 133, 144 Autosomal dominant inheritance, feature of, 16, 19
Apixaban, 143, 157, 270 Autosomal dominant polycystic kidney disease (APKO), 121,
cessation of, for right-sided pleural effusion, 157, 175 129
for preventing stroke in patiens with atrial fibrillation, 135, AV block, second degree, Mobitz type II, 134, 145
146 Axial spondyloarthritis (axSpA), 287, 294-295
APKD. see Autosomal dominant polycystic kidney disease Azathioprine, 246, 254-255, 367, 371
Aplasia, parvovirus B19 and, 75-76, 88 for Crohn 's disease, 230, 239
Apremilast, 287, 291, 296 for inflammatory bowel disease, 230, 240
for eczema, 364 monotherapy, for autoimmune hepatitis in pregnancy, 246,
Aquaporin-4 antibody, in optic neuritis, 308, 320 254-255
Aqueduct of Sylvius, stenosis of, 309, 321 Azithromycin, 105
ARDS. see Acute respiratory distress syndrome for Campylobacter infection, 84- 85, 95
Arginine vasopressin (AVP), effect on kidney of, 185, 194
Array comparative genome hybridisation (CGH), 17, 20 8
for chromosomal abnormalities, 15, 19 Babesia microti, as causative agent of babesiosis, 84, 95
parental, 17, 20 Babesiosis, 84, 95
Arsenic, 390 Bacillary angiomatosis, 81, 92
Arterial blood, analytes in, 403b Bacillus Calmette-Guerin (BCG) vaccine, 167
Arterial blood gas, dysfunctional breathing and, 177 Bacterial contamination, of platelets, 269, 277
AS. see Ankylosing spondylitis Bacterial overgrowth, 231, 241
Asbestosis, 388, 391-392 Banana spider, 50
Ascending aorta, emergency repair of, 139, 150 Band ligation, for bleeding oesophageal varices, 234, 244
Ascitic fluid amylase, 232, 242 Barbiturates, 42b
Ascorbic acid. see Vitamin C Barbotage, 289, 298
Aspartate aminotransferase (AST), in venous blood, 405b-406b Bariatric surgery, pregnancy and, 204, 208
Aspergillosis, 165, 1651, 180 Barium swallow, for polymyositis, 288, 296
voriconazole for, 167-168, 181 Barotrauma, 71
Aspiration, in septic arthritis, 279, 290 Barrett's oesophagus, 225-226, 235
Aspirin, 132- 133, 143, 147, 170-172, 183, 236-237 Barthel Index, 380-381
for acute pericarditis, 136, 14 7 Bartter's syndrome, 11 0, 11 3
for diabetes, during pregnancy, 367, 369 Basal analogue insulin (glargine), 214
on enzyme target, 6--9 Basal cell carcinoma, nodular
for rheumatic fever, 140, 151 excisional surgery for, 347, 3471, 351, 3511, 359, 362
sensitivity, 183 photodynamic therapy for, 353, 3531, 363
Asthma, 30, 177 Basement membrane, 349, 361
exacerbation of, 132, 143 Basic life support, 133, 144
montelukast for, 165, 179 Basophil granulocytes, reference range of, 409b-41 Ob I
I,
pregnancy and, prednisolone for, 164- 165, 179 Bat bites, 82, 93 I
triggered by pollen, 160-161, 177 Bayes' Theorem, 3-4
1
ASTRAL. see Angioplasty and Stenting for Renal Artery B-blockers, 40b, 45b
Lesions Beck Depression Inventory, 398b
Asymmetrical reflexes, in migraine, 299, 313 Becker muscular dystrophy, genetic pedigrees and, 15-16, 19
Asymptomatic bacteriuria, in pregnant woman, 122, 130 Beclometasone, 171
Atazanavir, 101b Beh9et's disease, 288-289, 297
Atelectotrauma, 71 Belladonna, 41 b, 44b
Atherosclerosis, causing retinal vein occlusion, 332, 3321, 334 Bendroflumethiazide, 116, 190, 278-279, 378, 381
Atlantoaxial subluxation, rheumatoid arthritis and, 234, 243 causing lichenoid reaction, 356, 364-365
ATN. see Acute tubular necrosis plasma renin concentrations increased by, 191 , 200
Atopic dermatitis Benign paroxysmal positional vertigo (BPPV), 308, 320
IL-13 inhibition in, 355, 364 Benign positional vertigo, 381
late-onset, 364 Benzatropine, 40b-41b, 44b- 45b
Atorvastatin, 108, 119, 121 Benzene, 390
Atovaquone plus proguanil, 33-35 Benznidazole, 95
Atracurium, 72 Benzodiazepines, 40b-42b, 44b-45b, 241
Atrial fibrillation, 132-133, 143-144, 147 for acute alcohol withdrawal, 63, 71
excessive alcohol consumption and, 137, 148 for breathlessness, 400
limb ischaemia and, 150 Benzylpenicillin, 151 - 152
preventing stroke in patients with, apixaban for, 135, 146 intravenous (IV), for meningococcal sepsis, 300-301 , 314
Atrial myxoma, 137, 148 Benzylpiperazine, 42b
Atrial septal defect, 146-147 Berylliosis, 171, 182
myocardial infarction and, 149 Beta-thalassaemia trait, 268, 275
Atrioventricular block, 138, 149 Betnovate ointment, for eczema, 355, 3551, 364
Atropine, 42b Bicarbonate, 111, 114
Auditory hallucinations, cocaine intoxication causing, 337, 342 in arterial blood, 403b
Augmented immunosuppression (ATG and high-dose steroids), Biceps reflex, loss of, for magnetic resonance imaging (MRI),
120, 128 300, 314
Autism, CGH and, 17, 20 Biguanide, for type 2 diabetes mellitus, 213, 219
Autocrine stimulation, in tumour formation, 17, 20 Bilateral hydronephrosis, in genitourinary system, 384,
Autoimmune connective tissue disease, long-term management 389-390
of, 288, 296 Bilateral lower limb hypertonicity, 304, 317
Autoimmune disease, 23, 26 Bile acid malabsorption, SeHCAT for, 230, 239
disease-modifying therapy in, 23- 24, 26 Biliary colic, dietary recommendatiCl[ls for, 249-250, 257
Autoimmune gonadal failure, 198-199 Biliary tree obstruction, 205, 209, 209b
Autoimmune hepatitis (AIH), in pregnancy, azathioprine Bilirubin, in venous blood, 405b- 406b
monotherapy for, 246, 254-255 Bioassay, 402
Autoimmune pancreatitis, lgG4 in, 233, 243 Biopsy, bland urine sediment with interstitial fibrosis on, 121,
Autoscopic hallucination, 341-342 128

downloaded from www.medicalbr.com


4 26 • INDEX

Biotin, 209b Bronchial carcinoma, 155, 1551, 174-175


Bipolar affective disorder, prevalence of, 341b adenomatous, 168, 181
Bipolar disorder, lithium salts for, 339-340, 344 CT scan for, 155, 1551, 174-175
Bipolar stain, for Gram-negative coccobacilli, 77, 89 right-sided , 175
Bismuth subcitrate, 237 Bronchiectasis, 156-157, 1571, 175
Bisoprolol, 119, 191 Bronchoalveolar lavage, 25, 27
Bisphosphonates, 159, 226, 235- 236, 391 Bronchodilators, nebulised, 162-163, 180
for bone pain, 400b Bronchoscopy, 174-17 5
for hypercalcaemia, 395- 396, 400-401 bronchiectasis and, 179
oral, for osteoporosis, 280, 291 flexible, 176
Bivalirudin, 270 haemoptysis and, 178
Black widow spiders (Latrodectus spp.), 50 interstitial pneumonia and, 176
Bladder cancer, 122, 130, 383-389 pleural disease and, 176
Blatchford scoring system, 234, 243-244, 244b spontaneous pneumothorax and, 184
Bleeding Bronchospasm, 178
oesophageal varices, band ligation for, 234, 244 Brown recluse spiders (Loxosceles spp.), 50
oral mucosal, in immune thrombocytopenia, 261, 270 Brucellosis, in pregnancy, 86b
time, reference range of, 409b-41 Ob Brugmansia spp. (angel's trumpet), 45b
a-Blockers, 380 BUB1B mutation, mosaic variegated aneuploidy due to, 14, 18
P-Biockers, 134, 145 Budd-Chiari syndrome (hepatic venous thrombosis), 247, 255
heart rate control with, 134, 145 Buerger's disease, 139, 150
risk of sudden death reduced by, 136-137, 147 Bullous pemphigoid
Blood, gas carriage in, 56, 66 Nikolsky sign in, 352-353, 3521, 363
Blood alpha-fetoprotein (AFP) measurement, for HCC, PUVA photochemotherapy for, 351 , 362
246-247, 249, 255, 257 Bumetanide, effects of, 134, 145
Blood film Buprenorphine, 42b
for lymphatic filariasis diagnosis, 78, 90 Bupropion, 41b , 44b
for Plasmodium vivax infection diagnosis, 73, 85 Burkholderia pseudomaiiei, 180
for Trypanosoma brucei gambiense infection diagnosis, Buspirone, 41b, 44b
78-79, 90
Blood test, for nausea, and delirium, 394-395, 399 c
Blood volume, reference range of, 409b-41 Ob C fibres, calcitonin gene-related peptide (CGRP)-containing, in
Bloody diarrhoea, azithromycin and, 84-85, 95 peripheral pain processing, 393-396
Body, jerking of, in generalised tonic-clonic seizures, 303, Cabergoline, for macroadenoma, 192, 201
316-317 Caeruloplasmin, in venous blood, 405b-406b
Bolus/rapid-acting analogue insulin (NovoRapid), 214, 217 Calcification, of peri-odontoid ligaments, 285, 294
Bone disease, 278-298 Calcitonin gene, expression of, 14, 18
Bone erosions, 293 Calcitonin gene-related peptide, in pain processing, in spinal
Bone formation, inhibition of, 285, 293 cord, 397b
Bone metastasis, in humerus, from breast cancer, 384, Calcitonin gene-related peptide (CGRP)-containing C fibres, in
389 peripheral pain processing, 393- 396
Bone pain, 400b Calcium, 34- 35
Bone sarcoma, from oral contraceptive pill, 391 in falls, 380
Bone sclerosis, 294 in osteoporosis, 286, 291, 294
Borderline cognitive impairment, exacerbated by opioid supplements for, 159, 162
medication, 393-394, 398, 398b in urine, 408b
Bosentan, 184 in venous blood, 405b-406b
for pulmonary hypertension, 173, 184 Calcium channel blockers, 40b, 45b
Botulism, 311, 323 for type 1 diabetes, 214
Bowman's capsule, 108, 112 Calcium clearance:creatinine clearance ratio, 409
BPPV. see Benign paroxysmal positional vertigo Calcium pyrophosphate deposition disease (CPPD), 279, 284,
Brachial neuralgia, 301, 314 290, 293
Bradycardia, 51, 53 Calculated low-density lipoprotein (LDL) cholesterol, 107, 11 2
Brain abscess Calprotectin, in faeces, 409b
Nocardia asteroides infection and, 81 , 92 CAL-R genes, thrombocythaemia and, 261 , 270
staphylococcal, 93 Campylobacter infection, ulcerative colitis and, 81, 92
Brain imaging, 310, 322 Camurati-Engelmann disease, 293
Brain-death, testing for, 57-58, 67 Cancer screening, serial whole-body computed tomography
prevention of, 63, 72 for, 51-53
Brainstem stroke, 299-313 Candida oesophagitis, 98, 102
Brazilian wandering spider (Phoneutria nigriventer), 47-48, 50 Candidaemia, 90
BRCA 1 screening, in breast cancer, 388, 392 Cannabis, 42b
BRCA2 screening, in breast cancer, 388, 392 in fibromyalgia, 287, 296
Breakthrough pain, 397 in vasculitis, 289, 297
Breast cancer, 386-389 Capillary blood glucose, on routinely used early warning
bone metastasis in the humerus from, 384, 389 systems, 60, 69
formation of, 17, 20 Capillary occlusion, in diabetic retinopathy, 332, 3321, 334-335
multidisciplinary team, in breast lump, 386, 390 Capillary refill time, 139, 150
next-generation sequencing and, 17, 20 Carbamates, 42b
from oral contraceptive pill, 391 Carbamazepine
Breathing, during sleep, 173, 184 drug reaction induced by, 357, 365
Breathlessness, 400 toxic epidermal necrolysis caused by, 346, 358
·in Guillain-Barre syndrome (GBS), 299, 313 Carbimazole, 188, 192, 201 •
in inoperable lung cancer, 394, 399 thyroid hormone synthesis pathway and, 186, 194, 1941
Breslow thickness, of malignant melanoma, 350, 3501, 361 Carbon monoxide, 69
Brief Pain Inventory, 398b Carboxyhaemoglobin, 59, 69
Bronchial artery angiography, for cystic fibrosis, 163, 178 in venous blood , 405b-406b

downloaded from www.medicalbr.com


INDEX • 427

Carcinogenesis, 52 Chromaffin cells, 200


Carcinoid tumour, 168, 181 , 191, 200 Chromatids, attachment to mitotic spindle, 14, 18
Carcinoma, in hyponatraemia, 381 Chronic cardiac failure, associated with renin-angiotensin-
Carcinosarcoma, 390 aldosterone system (RAAS), 134, 145
Cardiac arrest, treatment for, 55, 65 Chronic daily headache, 309, 322
Cardiac tamponade, 62, 71, 138 Chronic hepatitis B, management of, 247, 256
Cardiac transplantation, 136, 147 after liver transplant, 249, 257
Cardiac transthoracic echocardiogram, for polymyositis, 288, in pregnant women, 248, 256
296 Chronic inflammatory demyelinating polyneuropathy (ClOP),
Cardiomyopathy 309,321
cardiac transplantation and, 136, 147 Chronic lymphocytic leukaemia, 261-270, 273
dilated, 136, 147 Chronic obstructive pulmonary disorder (COPO), 162, 1621,
anthracycline chemotherapy and, 142, 152 177-178 .
causes of, 136, 147 LAMAILABA combination inhaler for, 165, 179
hypertrophic, 136, 147 palliative care for, 395, 399
inheritance of, 141, 152 Chronic pericardia! constriction, 137, 148
ventricular arrhythmia and, 136, 147 Chronic thromboembolic disease, 178
restrictive, 138, 149 Chymotrypsin, 242
risk of, from doxorubicin, 372, 375 Ciclosporin, for severe aplastic anaemia, 265, 273
Takotsubo (stress), 137, 147- 148 Cidofovir, 85
Cardiopulmonary resuscitation (CPR), for cardiac arrest, ClOP. see Chronic inflammatory demyelinating polyneuropathy
62, 71 Cigarette smoking, 147
Cardiovascular Outcomes in Renal Atherosclerotic Lesions in rheumatoid arthritis, 285, 293
(CORAL) trials, 125 Ciguatera poisoning, 37, 40, 94
Carotid artery stenosis, medical therapy for, 327, 329 Ciprofloxacin, 93, 104-1 05, 274
Carotid endarterectomy, 325, 327 Cisplatin, 228, 238
Caspofungin, 34 Citalopram, 400
Cast nephropathy, 128 Clarithromycin, 237
Cataract surgery, complications of, 331, 333 for community-acquired pneumonia, during pregnancy, 367,
Catecholamine release, increase, 108- 109, 112 369
Cathinones, 42b Clindamycin, 93
Cauda equina, lesion in, 386, 391 for toxic shock syndrome, 77, 89
Cavernous sinus, cranial nerves in, 192, 201 Clinical biochemistry and metabolic medicine, 107-11 4
CBT see Cognitive behavioural therapy Clinical decision-making, 1-5
C04 count, reference range of, 409b-41 Ob errors in, 2, 4
CEA, in venous blood, 405b- 406b Clinical genetics, 14-21
Ceftriaxone, for antimicrobial infection, 78, 89 Clinical immunology, 22- 27
Central chemoreceptors, activation of, 174 Clinical pharmacology, 6-1 3
Central cyanosis, 146 Clomifene, 199
Central venous access, noradrenaline and, 57, 66 Clostridium botulinum, 93
Central venous catheter (CVC), infection due to, 74- 75, 87 Clostridium p erfringens, 92
Cerebral dysfunction, in total anterior circulation stroke, 326, as causative agent of gas gangrene, 83, 93- 94
329 as causative agent of necrotising colitis, 84, 95
Cerebral oedema, OKA and, 212-213, 218-219 Clozapine, for schizophrenia, 339, 344
Cerebral toxoplasmosis, 98, 101, 101 f Clubbing, familial, 163, 178
Cerebral vein thrombosis, 326, 329 Cluster headache, 307-308, 320
Cerebrospinal fluid (CSF) Clustered vesicles, in herpes simplex virus infection, 349,
abnormality in, meningococcal sepsis and, 301, 314 360-361
analytes in, 409b Coagulation screen, reference range of, 409b-41 0b
cryptococcal antigen test on, 76, 88 Coagulation studies, haemoptysis and, 178
in Listeria monocytogenes infection diagnosis, 83, 94 Co-amoxiclav, 166
Cervical spine imaging, 300, 314 Cobalamin. see Vitamin 8 12
CGA. see Comprehensive Geriatric Assessment Cocaine, 42b
CHA20S2-VASc score, 135, 145-146, 261-262, 270 intoxication, 337, 342
Charcot joint, 284-285, 293 use, intracerebral haemorrhage and, 326, 329
Chelating agents, 42b Co-codamol, 393- 394
Chemokine receptor inhibitor, 101b for pain, in oropharyngeal tumour, 394, 399
Chemotherapy, 390 Cod liver oil, capsules of, 162
Chest pain Codeine, 42b, 393- 394
acute, 143 for increased intracranial pressure, 400b
due to pericarditis, 58, 67 in Western world populations, 7, 10
Chief cells, deficiency of, 233, 243 Codeine phosphate, severe liver disease and, 8, 12
Chlamydia trachomatis, 103- 105 Coeliac disease
infection, in pregnancy, 86b disease association of, 234, 244, 244b
Chlamydia! salpingitis, diagnosis of, 104, 106 pathophysiology of, 229, 239
Chloride, in venous blood, 403b Coeliac serology, for coeliac disease, 229, 238
Chloroquine, 35 Cognitive behavioural therapy (CBT), 290, 338, 342-343
Chlorphenamine, for polymorphic eruption of pregnancy, 361 Cognitive biases, 4
Cholecalciferol. see Vitamin 0 Cohort studies, 29-30
Cholecystokinin, in pain processing, in spinal cord, 397b Colchicum autumnale (autumn crocus), 45b
Cholestasis, acute, of pregnancy, 345- 346, 358 Cold nodules, 195
Cholesterol Colistin, 34, 36
embolisation, 11 6-117, 125 Collagen, type IV, 361
in venous blood, 405b-406b , 407f Colon, normal function of, 232, 241
Cholestyramine, 112 Colonoscopy
Cholinergic features, in poisoning, 39, 43, 44b for colorectal cancer, 231, 241
Cholinesterase reactivators, 42b in ulcerative colitis, 386, 391

downloaded from www.medicalbr.com


.428 • INDEX

Colorectal cancer, 25G-251, 258 Cough


genetic aspects of, 231, 241 chronic
Common peroneal nerve lesion, 302, 315 chest X-ray for, 161, 177
Common source outbreak, 33, 35 CT scan for, 165, 179
Communication, human factors and, 2, 5 from gastro-oesophageal reflux, 162, 177
Compartment syndrome, 62-63, 71 Coxiella burnetii infection, as occupational disease, 78, 90
Complement deficiency, 23, 26 C-peptide, in venous blood, 403b-404b
Complement system, 23, 26 CPPD. see Calcium pyrophosphate deposition disease
in venous blood, 405b-406b Cranial nerve
Complex regional pain syndrome (CRPS) type 1, 394, 3941, in cavernous sinus, 192, 201
398-399 palsies, CT angiogram and, 55, 65
Comprehensive Geriatric Assessment (CGA), 378, 380 signs, in brainstem stroke, 299-313
Computed tomography (CT], 390 Craniopharyngiomas, 201
of abdomen, 60, 69 C-reactive protein (CRP)
for acute stroke, 326, 328 acute rheumatic fever and, 140, 151
chest, for diplopia, 305, 318 in venous blood, 405b-406b
grey-white differentiation in, 55, 65 Creatine kinase (CK)
of head, in brain metastasis, 387, 391 in rhabdomyolysis, 60, 70
for Horner's syndrome, 330, 333 in venous blood, 405b-406b
in salmonellosis diagnosis, 80, 91 -92 Creatine kinase MB isoenzyme, in venous blood, 405b-406b
serial whole-body, for cancer screening, 51 -53 Creatinine
for subarachnoid haemorrhage, 325, 327 in growth and puberty, 411 b- 412b
warfarin for, 325, 327 in urine, 408b
Computed tomography (CT] angiography, of circle of Willis, 55, in venous blood, 403b
65 Creutzfeldt-Jakob disease (CJD), 343
Computed tomography (CT] pulmonary ang1ography variant, 31 G-311 , 322
(CT-PA) Critical illness, 54-72
haemoptysis and, 178 Critical illness myopathy (CIM), 57, 66-67
for thrombosis, in IBD, 230, 240 Critical illness polyneuropathy, weaning and, 63, 72
Confined placental mosaicism, as cause of false-positive result Grahn's disease, 294
in aneuploidy screening, 17, 20 pathophysiology of, 23G-231, 240
Confirmation bias, 2, 4 smoking and, 373, 375
Confusion Cryoglobulinaemia, type II, 25, 27
in hypoglycaemia, 216, 222 Cryoprecipitate, 275
in thrombotic thrombocytopenic purpura (TTP), Cryptococcal antigen test, on cerebrospinal fluid, 76, 88
263, 271 Cushing's syndrome, 193, 389
Congenital adrenal hyperplasia, 21 -hydroxylase deficiency and, 24-hour urine free cortisol test for, 191, 200
19G-191, 200 hypokalaemia associated with, 191, 200
Congenital anomalies, 28-30 weight gain and, 203-207
Congestive cardiac failure (CCF), risk factor for venous Cyanide, 39, 42b-43b, 44
thromboembolism, 262, 271 poisoning, 37- 38,41 ,44
Conium macu/atum (hemlock), 45b Cyclizine, 173, 184
Conservative management, of non-disabling relapse, 308, Cyclophosphamide, 128
32G-321 Cyproterone acetate, 359-360
Consolidation Cystic fibrosis
left-sided, 160, 1601, 176 bronchial artery angiography for, 163, 178
left-sided extensive, 159, 1591, 176 oral contraceptive pill and, 372, 375
causative organism of, 159, 176 Pseudomonas aeruginosa and, 166, 180
CURB-65 score for, 159, 176 Cystoscopy, 130
Constitutional delay, 189, 198 in haematuria, 387, 391
Contact allergic dermatitis, 36G-361, 364 Cysts, Taenia so/ium and, 80, 91
diagnosis of, 346, 358 Cytochrome P450, induced by rifampicin, 6, 9
Contact lens wear, 330, 333 Cytochrome P450 2D6 (CYP2D6), 7, 10
Contraceptive pill, 347 Cytokines, key function of, 22, 25
Contractility, stroke volume and, 60, 69 Cytomegalovirus (CMV)
COPD. see Chronic obstructive pulmonary disorder colitis, 96, 100
Copper infection, in pregnancy, 86b
in urine, 408b retinitis, 98, 101
in venous blood, 405b- 406b Cytotoxic chemotherapy, 167-168
CORAL trials. see Cardiovascular Outcomes in Renal
Atherosclerotic Lesions (CORAL) trials D
Corneal involvement, in viral conjunctivitis, 333 Dabigatran, 270
Coronal papillae, 103, 1031, 105 Dalteparin, antithrombin-dependent inhibition of, 261 , 270
Coronary artery bypass graft surgery, best outcome/ Daptomycin, 170
improvements with, 141- 142, 152 Dark urine, in muscular problem, 303, 316-317
Corpuscular volume, in hookworm infection, 75, 87 Darunavir, 101b
Corticosteroids, 156-157, 180 Datura stramonium (Jimson weed, thorn apple), 45b
intermittent oral, 156-157 D-dimer test. 3, 5
intralesional, for alopecia totalis, 362 reference range of, 409b-41 Ob
systemic, 359 de Musset's sign, 14G-141 , 151
Cortinarius spp. , 45b Deadly nightshade (Atropa belladonna), 39, 43-44, 44b-45b
Cortisol Decision-making, confirmation bias and, 2, 4
in-urine, 408b Decompression, 53 ...
in venous blood, 403b-404b Deep vein thrombosis, 59, 68
Coryza, acute, rhinovirus and, 166, 180 Defibrillation, 144
Co-trimoxazole, 97 Dehydration, 199, 290
for Nocardia infection, 84, 94-95 Delayed puberty, karyotype for, 189-190, 198-199

downloaded from www.medicalbr.com


INDEX • 429

Delirium, 337, 341-342 Dipeptidyl peptidase-4, 200


alternative diagnosis of, 58, 67, 67b Dipipanone, 42b
prevalence of, 341 b Dipstick test, for proteinuria, 46, 49
reversibility and, 58, 68 Dipyridamole, 138, 148-149
Delusion, 336, 341 Direct antiglobulin (Coombs) test, for haemolysis, 268, 275
Dementia Direct oral anticoagulants, for stroke, 326, 329
malnutrition in, 206, 211, 211f Disease, ageing and, 377-382
manifestations of, 339, 343 Disease-modifying therapy, in autoimmune disease, 23-24, 26
pathophysiology and management of, 339, 343 Disseminated gonococcal infection (DGI), 103, 105
Demyelinating optic neuritis, 331, 334 Dissociated sensory loss, 299, 313
Dengue Distal tubule, early, abnormal function of, 109, 113
criteria for, 75, 88 Disulfiram, for Wernicke-Korsakoff syndrome, 343-344
in pregnancy, 86b Diuretics, 145, 401
'Dense deposits', 118-119 Diving, risks involved in, 52-53
Deoxyribonucleic acid (DNA), repair of, 14-18 OMSA. see Dimercaptosuccinic acid
Depression, 342 Dobutarnine, 152
diagnosis and management of, 340, 344 Dolutegravir, 98, 101b
post-partum, 343 Dominant negative mutation, definition of, 15, 19
Dermatitis, 33, 35 Domperidone, 11 9
Dermatology, 345-365 Dopamine, 49, 64
Dermatophyte fungal infection, 364 Dose expression, violation in prescribing practice, 8 , 12
Desferrioxamine, 42b Double vision, wound botulism and, 82, 93
Deterioration, sign of clinical, 60, 69 Douleur Neuropathique questionnaire (DN-4), 398b
Developmental delay Doxazosin, 191
CGH and, 17, 20 Doxorubicin, for acute lymphoblastic leukaemia, 372, 375
parental array CGH and, 17, 20 Doxycycline, 34-35, 105, 166, 369
Dexamethasone, 191, 20Q-201, 384, 389 for Lyme borreliosis, 79, 91
for liver capsule pain, 395, 399, 400b unpasteurised milk and, 76, 88
suppression test, 201 Driver and Vehicle Licensing Agency (DVLA), for people
Dextropropoxyphene, 42b prescribed insulin, 213, 219
DGI. see Disseminated gonococcal infection Drowning, 52-53
Diabetes insipidus Drug fever, 9Q-91
cranial, water deprivation test for, 185, 1851, 193-194 Drug history, 340
diagnosis of, 193, 202 Drug reaction, carbamazepine-induced, 357, 365
Diabetes mellitus, 212-224, 388, 392 Drug users, anthrax amongst, 82, 93
abdominal aortic aneurysm and, 139, 150 Drug-induced chronic eosinophilic pneumonia, 170, 182
diagnostic cut-offs in, 41 3b Drug-induced dyskinesia, 306, 319
limb ischaemia and, 150 Drug-induced liver injury (DILl), 245-246, 253-254
during pregnancy, medication for, 367, 369 Dual process theory, type 2 (analytical) thinking in, 2, 4
target ranges of, 414b Dual X-ray absorptiometry (DXA) scan, for osteoporosis, 280,
type 2, diabetic ketoacidosis and, 212, 218 291
use of statins and, 108, 11 2 Duchenne muscular dystrophy, 372, 375
Diabetic cheiroarthropathy, 293 Duloxetine, 397-398
Diabetic ketoacidosis, type 2 diabetes mellitus and, 212, 218 for neuropathic pain, 400b
Diabetic mononeuropathy, 213, 219 Dupilumab, for eczema, 364
Diabetic nephropathy, 120, 128 Dusky toes, 125
Diabetic retinopathy, capillary occlusion in, 332, 332f, 334-335 DVLA. see Driver and Vehicle Licensing Agency
Diagnostic error, occurrence of, 1-3 Dynorphin, in pain processing, in spinal cord, 397b
Dialysis, 119, 127 Dysarthria, 303, 315- 316, 316b
Diaphragmatic failure, 154, 17 4 Dysbiosis, 233-234, 243
Diarrhoea, 110,1 13, 114f Dysconnectivity, 343
acute, 110,113 Dysfunctional breathing, studies for, 162, 177
HIV and, 234, 244 Dyskinesia, drug-induced, 306, 319
Diastolic hypertension, hypothyroidism due to, 186, 195 Dysphasia, in transient ischaemic attack, 325-327
Diastolic murmur, early, 132 Dysphoria, 342
Diazepam, 40b, 44, 45b, 203-204, 322, 399
for Wernicke-Korsakoff syndrome, 339, 343-344 E
DichloNos, 38, 43 Early bronchoscopy, 25, 27
Diclofenac, 147, 278, 290 Ebola virus disease, complications of, 74, 85-86
in elderly patients, 8, 11 Ebstein 's anomaly, 143
Dicobalt edetate, 42b EBUS-FNA. see Endobronchial ultrasound-guided fine-needle
Dietary modification, for gestational diabetes, 214, 220 aspiration
Differential white cell count, reference range of, 409b-41Ob Echocardiography, 17 4- 175
Diffuse inflammatory lymphocytosis syndrome (DILS), 97, 101 for dysfunctional breathing, 177
Digitalis purpurea (foxglove), 45b for source of enterococcal infection, 84, 94
Digoxin, 42b, 44, 119, 132- 133, 322 ECMO. see Extracorporeal membrane oxygenation
Fab fragments, 42b Eczema, topical glucocorticoid for, 355, 355f, 364
reduction on dosage of, 8 , 12 Efavirenz, 101 b
Dihydrocodeine, 42b eGFR. see Estimated glomerular filtration rate
Dihydrotestosterone, 190, 199 Eisenmenger's syndrome, 135, 146
Dilated cardiomyopathy, 136, 147 Elastase, in faeces, 409b
anthracycline chemotherapy and, 142, 152 Elastic recoil, loss of, in COPD, 154• 174
causeS' of, 136, 147 Elderly, skin changes in, 352, 362-363
DILl. see Drug-induced liver injury Electric shock, for snakebite, 49
Diltiazem, 147, 191 Electroencephalogram (EEG), 308, 321
Dimercaptosuccinic acid (DMSA), 38, 41, 42b Electrolytes, in venous blood, 403b
Dimeticone, for head louse, 360 Elvitegravir, 98, 101b

downloaded from www.medicalbr.com


!I_
430 • INDEX

Emollients, 39b, 355 Ethylene glycol, 42b-43b


Emotional stress, acute, 137, 147-148 Etoricoxib, for acute gout, 280-281, 291
Empagliflozin, and genital thrush, 216, 222- 223 Eumovate ointment, 355, 364
Empyema, 155, 1551, 175 European adder (1/ipera berus), 47, 50
pleural fluid pH and, 164, 1641 EuroOol 5-Domain questionnaire (EQ-50), 398b
Emtricitabine, 101, 101b Euthymia, 342
Emulsifying ointment, for eczema, 364 Evidence-based medicine, patient-centred, 2, 4
Enalapril, 371 Exacerbation, of asthma, 132, 143
Endobronchial ultrasound, lung cancer and, 183 Exercise-induced bronchospasm, 174
Endobronchial ultrasound-guided fine-needle aspiration Exertional heat illness, 53
(EBUS-FNA), 160, 176 Extracellular fluid (ECF), 108, 112
Endocarditis, 147 Extracorporeal membrane oxygenation (ECMO). venous-arterial,
infective, 167, 180 for viral myocarditis, 56, 66
Endocrine system, haemoconcentration mediated through, Extracranial carotid artery, traumatic dissection of, 311-312,
51-53 323
Endometrial cancer, 387, 391 Extradural haematoma, 301-302, 305, 315, 318
Endophthalmitis, 331, 333 Eye movements, wound botulism and, 82, 93
B-Endorphin, in pain processing, in spinal cord, 397b Eyebrow elevation, relative preservation of, on facial weakness,
Endothelin inhibitors, 184 304, 317
Endotracheal intubation, during mediastinoscopy, hoarseness
of voice and, 173, 184 F
Energy, regulation of, 203, 207, 2081 Facial dysmorphism, parental array CGH and, 17, 20
Enterococcal infection, 84, 94 Factitious disorder, 342
Enterohepatic circulation, of active drug, 7 , 10 Factor V Leiden, 262, 271
Enthesitis, in psoriatic arthritis (PsA), 287, 295-296 Factor VIII, reference range of, 409b-41 Ob
Envenomation, 46-50 Factor XIII, fibnn clot and, 263, 271
suspected, 46-48 Faecal transplantation, for C. dilficile infection, 76-77, 88
Environmental medicine, 51-53 Faeces, analytes in, 409b
Eosinophil granulocytes, reference range of, 409b-41 Ob Falls
Eosinophilia, 263, 271 CT abdomen and, 60, 69
Eosinophilic meningitis, Angiostrongylus cantonensis and, 79, examination for, 378, 381
91 predicting, 378, 380-381
Eosinophilic oesophagitis, 235-236 reducing , interventions for, 377-380
Ephedrine, 42b Famciclovir, 105
Epidermolysis bullosa, 363 Family history, 340
Epiglottitis, management of, 58, 67- 68 Fasciola hepatica, serology for, 76, 88
Epilepsy, 30 'FAST HUG' checklist, 62, 71
folic acid for, 366, 368 Fat metabolism, 203, 207, 207b
illicit drugs and, 371, 374 Fatigue, decision-making and, 2, 4
Epinephrine. see Adrenaline Fatty liver, acute, of pregnancy, 367, 369
Epirubicin, cisplatin and fluorouracil (ECF), 228, 238 Febrile non-haemolytic transfusion reaction, 267, 275, 2761
Episodic ataxia, 309-310, 322 Felty's syndrome, 291
Epistaxis, 262, 270-271 Ferritin, reference range of, 409b-41 Ob
Epithelial ovarian cancer, 386-387, 391 Fetal maternal haemorrhage, during pregnancy, 266, 274
EQ-50. see EuroQol 5-Domain questionnaire Fever
Erectile dysfunction (EO), 123, 131 likelihood ratio for, 1, 3
Erlotinib, 168, 181 in thrombotic thrombocytopenic purpura (TTP). 263, 271
Errors Fexofenadine, 355
in clinical decision-making, 2, 4 for eczema, 364
diagnostic, occurrence of, 1-3 Fibrillin, mutations in, Marfan's syndrome and, 141, 152
medication reconci liation and, 7, 11 Fibrinogen, 403
Erythema multiforme, herpes simplex virus infection and, 357, reference range of, 409b-41 Ob
365 Fibrolamellar hepatocellular carcinoma, treatment for,
Erythematous scaly plaques, treatment of, 352, 362 250, 258
Erythrocyte sedimentation rate (ESR), reference range of, Fibromyalgia
409b-410b and constitutive substantial connective tissue laxity, 287,
Erythromycin, 11, 40b, 45b, 105, 237 296
for papulopustular acne, 359-360 low vitamin 0 and, 282, 292
Erythropoiesis, 51-53 Fibrous dysplasia, 283-284, 293
Erythropoietin (EPO) Finasteride, 123, 131, 362
deficiency, 126 gynaecomastia and, 190, 199
reference range of, 409b-41 Ob Fixed drug eruption, 360-361
Eschars, without rash, as sign of Rickettsia africae infection, 78, Flecainide, 145
90 Flexible bronchoscopy , lung cancer and, 183
ESR. see Erythrocyte sedimentation rate Flexible cystoscopy, 123, 130
Essential amino acid, 206, 210, 21 Ob Flixotide, 165
Essential hypertension, 132, 143 Flucloxacillin, 151-152
Essential thrombocythaemia, 263, 271 as infusion, 33, 35
Essential tremor, 306, 319 for S. aureus infection, 80, 91
Estimated glomerular filtration rate (eGFR), 406, 4061 Flucloxacillin-induced liver injury, 247, 256
metformin and, 8, 12 Fluconazole, 98, 235
Estimated volume of distribution ry,), of drug, 6, 9-10 oral, 235
Etanercept, 173, 279 Flucytosine, 98-99
for seropositive rheumatoid arthritis, 280, 291 Fludrocortisone, 191, 200
Ethambutol, optic neuritis secondary to, 167, 180 Flumazenil, 42b
Ethanol, 42b-43b 5-fluorouracil, 228, 238, 363
in venous blood, 405b-406b Focal-onset epilepsy, 300, 313

downloaded from www.medicalbr.com


INDEX • 431

FODMAP (fermentable oligo-, di- and monosaccharides, and Giant cell arteritis (GCA), 288, 297
polyols) diet, for IBD, 232, 241 temporal artery biopsy for, 331, 334
Folate, 209b Gilbert's syndrome, 245, 253, 371, 374
deficiency, during pregnancy, 205, 209-210 GIST. see Gastrointestinal stromal cell tumour
reference range of, 409b-41 Ob Glargine. see Basal analogue insulin
Folic acid, for rheumatoid arthritis, 279, 291 Glasgow Coma Scale (GCS), 58, 67
Follicle-stimulating hormone (FSH) Glibenclamide, 220
in growth and puberty, 411b-412b Gliclazide, 216
in venous blood, 403b-404b Gliptins, 222
Follicular and papillary carcinoma, 197 Glomeruli, normal, 121-122, 129
Follicular lymphoma, 261-270 Glomerulosclerosis, 381-382
Fomepizole, 42b 'Glove and stocking' sensory disturbance, bacterial index and,
Formication, cocaine intoxication causing, 337, 342 77-78, 89
Foscarnet, 85 Glucagon, 242
Fracture, in post-menopausal osteoporosis, 280, 291 Glucagon-like peptide-1 , 191, 200
Francisefla tularensis analogues, 222
infection, 83-84, 94 Glucagon-like peptide-1 (GLP-1) agonist, pancreatitis and, 213,
muskrat contact and, 172, 183 219
FRAX, 294 Glucocorticoids, 88, 120-121, 128, 195-196,200, 239-240,
Free wall rupture, 149 298, 355
Fresh, full voided sample, for cytological assessment, 383, 389 for amiodarone-induced thyrotoxicosis, 188, 197
Fresh frozen plasma, for bleeding risk, 267, 275 for generalised itch, 363
Fried Frailty score, 381b for increased intracranial pressure, 400b
Frisen's scale, 334 intravenous therapy, for ulcerative colitis, 230-231 , 239-240
FSH. see Follicle-stimulating hormone for liver capsule pain, 395, 399, 400b
Full-thickness biopsy, of edge of the les1on, 383-384, 389 for palmoplantar pustulosis, 360
Functional hallucination, 341-342 for polymorphic eruption of pregnancy, 361
Functional hypothalamic amenorrhoea, 189, 197-198 systemic, for pemphigus, 365
Functional memory disturbance, 306-307, 319 for tinea capitis, 360
Furosemide, 64, 121, 148, 170, 377, 380 topical, for eczema, 355, 3551, 364
effects of, 134, 145 Glucodrate®, 210
Glucokinase, maturity-onset diabetes of the young, 212, 218
G Glucosamine, 291-292
Gabapentin, for neuropathic pain, 400b Glucose, in venous blood, 405b-406b
Gait, 382 Glucose-6-phosphate dehydrogenase (G6PD) assay, 269, 275
Gamma hydroxybutyrate (GHB), 38, 42b, 43 Glucose-6-phosphate dehydrogenase (G6PD) deficiency, 269,
I
Gas carriage, in blood, 56, 66 275
Gas gangrene, 93
Clostridium perfringens as causative agent of, 83, 93-94
test for, 80, 91
Glucose-related disorders, diagnostic cut-offs in, 413b
II
Gastric cancer, 228, 238 GLUT2, in pancreatic ~ cells, 215, 221
management options for, 228, 238 Glutamate, in central sensitisation, 393, 396-397, 3961, 397b
Gastric emptying study, for gastroparesis, 216, 223 Glutamine, 206, 210
Gastric lymphoma, 228, 238 y-glutamyl transferase (GGD, in venous blood, 405b-406b
Gastric ulcers, 227, 237 Glutathione repleters, 42b
Gastrin, in venous blood, 403b-404b Glycated haemoglobin (HbA,d, in venous blood, 405b-406b
Gastroenteritis, 10 Glyceryl trinitrate, 171-172
Gastroenterology, 225-244 Glycine, in pain processing, in spinal cord, 397b
Gastrointestinal stromal cell tumour (GISD. 229, 238 Glycogen storage diseases (glycogenoses), investigation of,
Gastro-oesophageal reflux 107, 111
cough from, 162, 177 Glypressin, for variceal bleeding, 246, 255
in severe neurodisability, 372, 374- 375 Gnathostomata spinigerum , 73-85
Gastroparesis, 234, 243 Goal setting, in rehabilitation programme, 377, 380
Gastrostomy, risk of, 206-207, 211 Goitre
GBS. see Guillain-Barre syndrome diffuse symmetrical, pregnancy and, 187, 196
GCA. see Giant cell arteritis Graves' disease and, 195
GCS. see Glasgow Coma Scale Gold, causing lichenoid reactions, 364-365
Gene expression, multi-functionality and, 14, 18 Gonadotrophin-releasing hormone (GnRH) agonist therapy,
Gene therapy, primary immune deficiency and, 25-26 123, 130
Generalised osteoarthritis, 294-295 Gonadotrophin-releasing hormone insensitivity, 197-198
Generalised tonic-clonic seizures Gout, 278-279, 290, 293-294
antiepileptic drug for, 303, 316 Grandiose delusion, 341
electroencephalogram (EEG) for, 308, 321 Granulomatous cerebral angiitis, 83, 94
in jerking of whole body, 303, 316-317 Granulomatous interstitial nephritis, 120, 128
Generic international non-proprietary name (INN), ciclosporin Granulomatous polyangiitis, 170, 182
and, 8, 12 Grass snake (Natrix natrix), 47, 50
Genes, ankylosing spondylitis and, 287, 295 Graves' disease, 186-187, 1861, 195
Genetic counselling, for myotonic dystrophy type 1, 15, 18- 19 hypothyroidism due to, 186, 195
Genetic defect, of alternative complement pathway, 118-119, radioactive iodine for, 187, 195-196
127 Griseofulvin, for tinea capitis, 348, 3481, 360
Genetic generalised epilepsy, 300, 313- 314 Group B streptococcal infection, in pregnancy, 86b
Genetic pedigrees, 15-16, 19 Group 0 fresh frozen plasma (FFP), 266, 274
Genioglossus, palatoglossus and, 173, 184 Group 0 red cells, 266, 274, 274b
Gentamicin, 151-152 Growth hormone (GH)
monitoring of, 9, 12 pituitary macroadenoma affecting, 192-193, 202
unpasteurised milk and, 76, 88 in venous blood, 403b-404b
Gestational diabetes, diagnostic cut-offs in, 413b Guillain-Barre syndrome (GBS), 299, 313
GHB. see Gamma hydroxybutyrate Gynaecomastia, finasteride and, 190, 199

downloaded from www.medicalbr.com


432 • INDEX

H Hereditary haemochromatosis, 215, 215t, 221


Ht N5 influenza A, 83, 94 transferrin saturation for, 252, 259
Haematocrit Heroin, 42b
in pregnancy, 411 b Herpes 8 virus infection, treatment for, 85, 95
reference range of, 409b-41 Ob Herpes simplex virus (HSV)
Haematological values, 409b-41 Ob infection
Haematology, transfusion medicine and, 261-277 diagnosis of, 349, 360--361
Haematopoietic stem cells, 264, 272 erythema multiforme and, 357, 365
Haematuria, in endocarditis due to viridans streptococci, 141, in pregnancy, 86b
151 in viral encephalitis, 300, 314
Haemochromatosis, 284 Herpesvirus 8 (HHV-8) infection, Kaposi 's sarcoma and; 81, 92
Haemoglobin HFE C282Y gene mutation, venesection for, 252, 259--260
in hookworm infection, 75, 87 HHS. see Hyperosmolar hyperglycaemic state
in pregnancy, 411b Hiatus hernia, 225, 235
reference range of, 409b-41Ob High-altitude pulmonary oedema (HAPE), 52-53
Haemoglobin-oxygen dissociation curve, 263, 2631, 271-272, High-resolution CT chest (HRCT], for respiratory pathogens,
2711 177
leftward shift of, 59, 68, 681 Hip fracture, post-test probability of, 1, 3--4
Haemolytic uraemic syndrome, 81-82,92, 117, 125 Hip protectors, in falls, 380
Haemophagocytic syndrome, juvenile idiopathic arthritis Hippei-Undau syndrome, 202
associated with, 286, 295 Histamine-2 antagonists, for gastrointestinal bleeding, 57, 66
Haemorrhage History of presenting complaint, 340
in cerebral vein thrombosis, 326, 329 HIT. see Heparin-induced thrombocytopenia
as complication of percutaneous tracheostomy, 63, 72 HIV. see Human immunodeficiency virus
intracerebral, cocaine use and, 326, 329 HIV-1 combined antigen and antibody test, 75, 87
Haemorrhagic transformation, brain-death testing and, 57-58, HIV-1 infection, in pregnancy, 86b
67 HL. see Hodgkin lymphoma
Hair loss HLA-827, for inflammatory back pain, 285, 294
tinea capitis and, 348, 3481, 360 HLA-DR1 genes, 287, 295
w ig for, 352, 362 HNF1 -beta gene mutation, 121, 128-129
Haldane effect, 66 Hodgkin lymphoma (HL), 265, 273
Hallpike manoeuvre, in falls, 378, 381 Hole, in right side of head, extradural haematoma and, 305,
Hallucination, 336--337, 341 318
Haloperidol, for Wernicke-Korsakoff syndrome, 343-344 Homocystinuria, 107, 111
Hand grip strength, in frail, 378, 380 Hookworm infection, abnormalities of, 75, 87
Hantavirus infection, 82, 93 Hormones, in venous blood, 403b-404b
HAP. see Hospital-acquired pneumonia Horner's syndrome, 181, 330, 333, 391
HAPE. see High-altitude pulmonary oedema Hospital-acquired pneumonia (HAP)
Haptoglobin, reference range of, 409b-41Ob Acinetobacter and , 167, 180
HBV surface antigen (HBsAg), needlestick injury and, 251 , 258 local antibiotic policy for, 166--1 67, 180
Headache mortality from, 167, 180
likelihood ratio for, 1, 3 Hospitalisation, risk factor for venous thromboembolism, 262,
'red ftag' symptom in, 59, 69 271
Heart beat, suspected pulmonary embolism and, 2, 4 Howell-Jolly bodies, in essential thrombocythaemia, 263, 271
Heart disease, 29, 31 HPOA. see Hypertrophic pulmonary osteoarthropathy
Heart failure HRCT. see High-resolution CT chest
due to abnormalities in thyroxine, 138, 149 Human chorionic gonadotrophin, in pregnancy, 41 1b
medical treatments for, survival and, 138, 148 Human factors, definition of, 2, 5
Heart murmur, scabies and, 80, 91 Human herpesvirus 6, 73, 85
Heart rate, ~-blockers controlling, 134, 145 Human immunodeficiency virus (HIV), 28, 30
Heat stroke, complications of, 51 , 53 associated nephropathy (HIVAN), 99, 102
Helicobacter pylori (H. pylori) infection, 227, 236--237 diagnosing, blood test for, 96, 100
eradication therapy for, 227, 237 infection and AIDS, 96--1 02
gastric cancer and, 228, 238 intramuscular injection to, 97, 101
Henoch-Schonlein purpura, 118, 126--127 primary, 96, 100, 1001
Heparin-induced thrombocytopenia (HIT], 262, 270 protease enzyme, 96--99, 991
Hepatic artery thrombosis, 247, 255 risk of acquiring, 96, 99--100
Hepatic glucose uptake, stimulation of, 215, 221 viral load in, 98, 102
Hepatic hydrothorax, 175 Human papillomavirus (HPV), 104, 106, 390
Hepatic vein thrombosis, in Budd-Chiari syndrome, 250, 258 vaccine, 32, 34
Hepatic venous thrombosis. see Budd-Chiari syndrome Human T-celllymphotropic virus type 1 (HTLV-1), 83, 94
Hepatitis A, 269, 277 Humerus, bone metastasis in, from breast cancer, 384, 389
Hepatitis A virus (HAV), 246--249, 257 Hydrochloric acid secretion, 234, 243
Hepatitis B immunoglobulin, as post-liver transplant prophylaxis, Hydrocortisone, 9, 49
249, 257 Hydrogen ion, in arterial blood, 403b
Hepatitis B virus (HBV) Hydrothorax, hepatic, 175
infection, in pregnancy, 86b Hydroxocobalamin, 42b
vaccination against, 104, 106 Hydroxychloroquine, 284, 291
Hepatitis C virus (HCV), 251-252, 259 blood monitoring in, 278, 290
Hepatitis D virus (HDV), 247-248, 256 5-hydroxyindoleacetic acid (5-HIM) concentration in urine,
Hepatitis E virus, 246, 254 408b
infection, in pregnancy, 86b carcinoid syndrome and, 192, 201
RNA in stool, 248, 257 21-hydroxylase deficiency, co~genital adrenal hyperplasia and,
Hepatology, 245-260 190--191,200
Hepatorenal syndrome, 120, 128 25-hydroxyvitamin D (25(0H)D), 282, 292
Hepcidin level, anaemia of chronic disease and, 266, Hyoscine butylbromide, for abdominal colic, 400b
273-274 Hyperalgesia, 397

downloaded from www.medicalbr.com


INDEX • 433

Hyperandrogenism, polycystic ovarian syndrome and, 189, 198 Immunological reference ranges, 41 Ob
Hyperbaric oxygen therapy, for radiation proctitis, 229, 239 Impaired renal function, in peripheral diabetic neuropathy, 393,
Hypercalcaemia, 177 397-398
Hypercholesterolaemia, 147 Implantable cardiac defibrillator, 135, 146
Hyperemesis gravidarum, 214, 220, 366, 368 Incident pain, 400b
Hyperglycaemia, hyperosmotic hyponatraemia secondary to, Incontinence, treatment of, 377, 380
109, 113 Increased intracranial pressure, 400b
Hyperinsulinaemia, 198 Indomethacin, 227-228
Hyperlactaemia, in chronic renal failure , 61, 70 In-dwelling catheter, for urinary problems, in multiple sclerosis,
Hyperosmolar hyperglycaemic state (HHS), 215-216, 222 307, 320
Hyperprolactinaemia, 197-198 Infection-related glomerulonephritis, 116, 124
Hyper-reflexia, 304, 317 Infectious disease, 73~95
Hypersensitivity adverse drug reactions, 7, 10 principles of, 32-36
Hypersensitivity pneumonitis, 171-172, 183 Infective endocarditis, 167, 180
Hypersensitivity reactions, 23, 26 aortic valve affected by, 133, 144
due to allopurinol, 79, 9D-91 lnfiammatory back pain, 285, 294
Hypertension Inflammatory bowel disorder (lBO). 230, 239, 2391
essential, 132, 143 pathogenesis of, 23D-231, 240
poorly controlled, poor adherence to medication and, 140, Inflammatory polyarthritis, 286, 294-295
151 lnfliximab, 173
secondary, renal disease and, 140, 151 lngenol mebutate, 359
Hyperthermia, cocaine intoxication causing, 342 Inhaled carcinogens, 388-389
Hypertrophic cardiomyopathy, 136, 143, 147 Inhaled corticosteroid (ICS)/LABA combination inhaler, 165
inheritance of, 141, 152 Inhibiting interleukin (IL)-13, 355, 364
ventricular arrhythmia and, 136, 147 Inhibitory amino acid neurotransmitters, 396-397
Hypertrophic pulmonary osteoarthropathy (HPOA), 168, 181, 293 Innate immune response, acute liver failure and, 246, 254
Hyperventilation, 53 Innate immunity, key feature of, 22- 25
Hypnagogic hallucination, 341-342 Inotropic agents, 45b
Hypnopompic hallucination, 337, 341-342 Insulin, 9, 369
Hypoalbuminaemia, 10 exogenous, 191-192, 201
Hypochondriacal delusion, 341 in venous blood, 403b-404b
Hypoglycaemia Insulin pump therapy, 218, 223-224
patient education on, 216, 222 'Insulin resistance syndrome', 219-220
stroke and, 326, 328-329 Insulin-like growth factor 1, in growth and puberty, 411b-412b
type 1 diabetes and, 214, 220 Integ rase inhibitors, 98, 101, 101 b
Hypokalaemia, Cushing's syndrome associated with, 191, 200 Interferon-gamma release assay?, 167, 18D-181
Hypomania, 342 Intermittent short-acting opioids, for incident pain, 400b
Hypotension, postural, 143 Internal jugular vein thrombosis, Lemierre's syndrome and, 81,
Hypothyroidism 92
due to diastolic hypertension, 186, 195 International system of units (SI units), 402, 402b
due to Graves' disease, 186, 195 'International units' (IU/L), 402
transudative pleural effusion and, 164, 179 Interscapular pain, 143
weight gain and, 203-207 Interstitial pneumonia
Hypoxaemia bronchoscopy and, 176
severity of, 70 CT appearance of, 160, 176
shunt and, 54, 64 lymphocytic, 182
Interventricular septum, rupture of, 133, 144
Intra-aortic balloon pump (IABP), use of, 63, 71
lBO. see Inflammatory bowel disorder Intracellular fluid, 108, 112
Ibuprofen, 119, 121-122,381 Intracerebral blood, in acute stroke, 326, 328
arteriolar vasoconstriction with, 121, 129 Intracerebral haemorrhage, cocaine use and, 326, 329
Idiopathic intracranial hypertension (II H), 301, 309, 315, 321 Intracranial pressure (ICP). management of, 63, 71
IIH. see Idiopathic intracranial hypertension Intramural haematoma, aortic dissection and, 150
Ileocolonic tuberculosis (TB), 229, 238 lntraretinal microvascular anomalies, in diabetic retinopathy,
Illusion, 341 334-335
lmipenem, for Nocardia infection, 84, 94-95 Intrauterine fetal death, UDCA therapy for, 247, 255-256
lmiquimod, 363 Intravenous (IV) benzylpenicillin, for meningococcal sepsis,
for basal cell carcinoma, 359, 362 30D-301' 314
for lentigo maligna, 360 Intravenous fluid
Immature teratoma, 385-386, 3861, 390 in OKA, 217, 223
Immune deficiency for hypercalcaemia, 395-396, 400-401
primary, 22, 25-26 Intravenous glucocorticoid therapy, for ulcerative colitis,
secondary, 23, 26 23D-231' 239-240
Immune reconstitution inflammatory syndrome (IRIS), 32, 34, Intravenous normal saline, for hypercalcaemia, 386, 391
98, 101 Intubation
Immunoglobulin A (lgA), 115, 124 for asthma, 54-55, 64-65
nephropathy, 115-116, 124-125 noradrenaline and, 57, 66, 66b
Immunoglobulin E (lgE) testing lpratropium, 333
for latex rubber allergy, 352, 362 IRIS. see Immune reconstitution inflammatory syndrome
for urticaria, 361 Iron, 42b-43b
Immunoglobulin G (lgG) deficiency, 275
coeliac disease and, 229, 239 reference range of, 409b-41 Ob •
testing·, for latex rubber allergy, 362 Irritant dermatitis, 355, 364
Immunoglobulins (lg), 22, 26 lschaemic pain, 400b
deficiency, 22-23, 26 lschaemic stroke, in polycythaemia rubra vera, 261, 270
levels, bronchiectasis and, 179 Isoniazid, 167
reference range of, 41 Ob Isopropyl nitrite, 37, 40, 41 I

downloaded from www.medicalbr.com


434 • INDEX

lsosorbide mononitrate, 173 LEMS. see Lambert-Eaton myasthenic syndrome


Isotope bone scan, 385, 390 Lentigo maligna, 348-349, 360
lsotretinoin, 361-362 melanoma, 363
for papulopustular acne, 359-360 Leprosy, lepromatous, 89
lvacaftor, 166, 180 Leptin, 200
lvermectin, 35 Leucopenia, 291
Leukaemia, in radiation exposure, 385, 390
J Levetiracetam, 313, 366
JIA. see Juvenile idiopathic arthritis for genetic generalised epilepsy, 300, 313-314
Joint hypermobility, 284, 293 Levofloxacin, 274
Joint replacement surgery, 291-292 Levothyroxine, 186-188, 196-197
Joint space, narrowing of, in osteoarthritis, 281, 291 dry skin and hair and, 187, 196
Juvenile idiopathic arthritis (JIA), methotrexate for, 373, 375, variable treatment adherence and, 187-188, 196
376b vitamin C and, 187, 196
Juxta-articular new bone formation, 294 Leydig cells, producing testosterone, 370-373
Lichenoid reaction, drug-induced, 356, 364-365
K Lidocaine, 9, 145
Kallmann's syndrome, 189, 198 Li-Fraumeni syndrome, 18, 21
Kaposi's sarcoma (KS) Light's criteria, 164, 179
AIDS-associated, 98, 102 Light-touch testing, for Dissociated sensory loss, 299, 313
HHV-8 infection and, 81, 92 Limb weakness, CT angiogram and, 55, 65
Keratoacanthoma, benign, excisional surgery for, 348, 3481, Lipoprotein receptor-related protein 5 (LRP5), 278, 290
360 Liquid laundry detergent capsule, 37-39, 39b
Ketamine Liraglutide, for weight loss, 216, 222
for ischaemic pain, 400b Lisinopril, 116, 118, 121. see also Angiotensin-converting
for neuropathic pain, 400b enzyme (ACE) inhibitor
Ketoconazole, 191 , 200 Listeria monocytogenes, 83, 94
Ketones, in venous blood, 405b-406b Listeriosis, in pregnancy, 86b
Ketonuria, repeated vomiting and, 213, 219 Lithium
Kidneys, CT of, 118, 127 causing lichenoid reactions, 364-365
Kinaesthetic hallucination, 341-342 salts, for bipolar disorder, 339-340, 344
Kleihauer test, 266-267, 274 Liver
Klinefelter's syndrome, 189-190, 198-199 anatomy of, 245-253
dysgenesis of the seminiferous tubules and, 190, 199 biopsy, lung cancer and, 183
small testes and, 198 capsule pain, 400b
Kussmaul's sign, 148 function of, 253
Liver function tests, for acute cholestasis of pregnancy,
L 345-346, 358
L. brasiliensis, 92 Liver transplantation
LABA. see Long-acting p2 -agonist assessment for, interferon treatment and, 248, 256
Laboratory reference ranges hepatorenal failure after, 246, 254
in adolescence, 411-414, 4131 yellow fever vaccine and, 77, 89
in adults, 403-409, 403b Lobar collapse, acute type I respiratory failure and, 164, 179
in childhood, 411-414, 4131 Long QT syndrome, 143
in pregnancy, 411 Long-acting p, -agonist (LABA), 164-165
Laburnum anagyroides (laburnum), 45b Long-acting muscarinic antagonist (LAMA), 165
Labyrinthitis, acute, 304, 309, 318, 321 Loop diuretics, 109, 112, 148, 391
Lactate, in venous blood, 405b-406b effects of, 134, 145
Lactate dehydrogenase (LDH) Loop of Henle, inadequate solute delivery to, 109, 11 3
in Cushing's syndrome, 384, 389 Loperamide, 230-231, 239- 240
in pleural fluid, 409b Lopinavir, 101 b
in venous blood, 405b- 406b Lorazepam, 40b, 45b, 400
LADA, 218, 224 Lower back pain, magnetic resonance imaging (MRI) for, 300,
lAMA. see Long-acting muscarinic antagonist 314
LAMNLABA combination inhaler, for COPD, 165, 179 Lower respiratory tract infection, H1N5 influenza A as, 83, 94
Lambert-Eaton myasthenic syndrome (LEMS), 305, 318 Lower urinary tract symptoms (LUTS), 130
Lamina densa, 361 Low-grade glioma, in progressive degenerative disorder, 311,
Lamina Iucida, 361 322-323
Lamivudine, 101 , 101b Low-molecular-weight heparin, 164- 165
Lamotrigine, 368 Lozenges, 235
for focal-onset epilepsy, 300, 313 LSD. see Lysosomal storage diseases
Langerhans' cells, 349, 361 Lung, right
Lansoprazole, 119 apex of, irregularly shaped mass at, 387, 391
Lantus, 332 collapse of, 154-1 75, 1561
Latent infection, 32-34 Lung cancer, 177, 182
LDH. see Lactate dehydrogenase staging of, 168, 181
Lead, 42b Luteinising hormone (LH)
poisoning, 41 in growth and puberty, 411 b-412b
in venous blood, 405b-406b in venous blood, 403b- 404b
Leber's hereditary optic neuropathy, 334 LUTS. see Lower urinary tract symptoms
Leeds Assessment of Neuropathic Signs and Symptoms Lyme borreliosis, 91
(s-LANSS), 398b Lyme disease, 312, 324
Left frontal lobe, abnormality in, 302- 303, 315 Lymecycline, for persistent papulopustular acne, 34 7, 3471,
Left ventricular failure, physical signs associated with, 133, 144 359- 360
Legless lizards, 50 Lymph node biopsy, supraclavicular, 172, 183
Leishmaniasis, 33, 35 Lymphangioleiomyomatosis, 170-171, 182
Lemierre's syndrome, 81, 92-93 Lymphangitis carcinomatosa, 168-169, 181

downloaded from www.medicalbr.com


- INDEX • 435

Lymphocytes, reference range of, 409b-41 Ob MERS. see Middle East respiratory syndrome
Lymphocytic interstitial pneumonia, 182 MERS-CoV. see Middle East respiratory syndrome coronavirus
Lymphogranuloma venereum, 103, 105 Mesalazine, 371
Lymphoma, 169, 181-182 Mesangiocapillary glomerulonephritis, 118-119, 127
Lymphoproliferative disorder, post-transplant, 372, 375 Mesenteric lymphadenopathy, Yersinia enterocofitica infection
Lynch's syndrome, autosomal dominant inheritance of, 16, 19 and, 82, 92-93
Lysosomal storage diseases (LSDs), 107, 111 Mesothelioma, 163, 171, 175, 178, 182-183
Metabolic acidosis, with respiratory compensation, 110, 114
M 'Metabolic syndrome', 219-220
M2 proton channel inhibitors, 89 Metacarpophalangeal joints (MCPJs), erosions at, 291
Macroadenoma, cabergoline for, 192, 201 Metadrenalines, in urine, 408b
Macroprolactin, 201 Metastatic spinal cord compression, 301, 314
Macular degeneration, age-related, smoking and, 331-332, Metformin, 369
3321, 334 estimated glomerular filtration rate and, 8, 12
Magnesium, in venous blood, 405b-406b for type 2 diabetes mellitus, 213, 219
Magnesium sulphate, 39-40, 40b, 45b Methadone, 42b
Magnetic resonance imaging (MRI) Methaemoglobinaemia, 40
loss of biceps reflex for, 300, 314 Methanol, 42b-43b
urinary incontinence for, 300, 314 ingestion, 110-111, 114
Malaria, in pregnancy, 86b Methotrexate, 24, 279, 352
Malathion, for head louse, 360 for eczema, 355, 3551, 364
Malignancy, 286, 295 increasing, 291
effusions as signs of, 137, 148 during pregnancy, 366-368
Malignant oesophageal stricture, 226, 236 for ulcerative colitis, 367-369
Malignant pleural disease, 178 in pregnancy, 281, 291
Malnutrition, in dementia, 206, 211, 211 I for rheumatoid arthritis, 279, 291
Mammogram, in breast lump, 387, 391 3,4-Methylene-dioxymethamphetamine, 42b
Mantle cell lymphoma, 265, 273 Methylthioninium chloride, 42b
MAOI. see Monoamine oxidase inhibitor Metoclopramide, 40b, 45b
Maraviroc, 101b Metolazone, 190
Marian's syndrome, 132, 143 Metronidazole, 88, 227, 237
aortic dissection and, 150 Mexican orange-kneed tarantula (Brachype/ma smithi), 50
mutations in fibrillin gene and, 141 , 152 MGUS. see Monoclonal gammopathy of uncertain
Market authorisation, granting of, 7, 11 significance
Mast cell tryptase, measurement of, 23, 26 Microalbuminuria, 214, 221
Maternal hyperglycaemia, fetal hyperglycaemia and, 213, 219 Microaneurysms, in diabetic retinopathy, 334-335
Maternal medicine, 366-369 Microangiopathic haemolytic anaemia, in thrombotic
Maturity-onset diabetes of the young (MODY), 212, 218 thrombocytopenic purpura (TTP), 263, 271
Maximum heart rate, reduced, ageing and, 378, 380 Microcephaly, investigations for, 15, 19
MCH. see Mean cell haemoglobin Microscopic polyangiitis, 115, 124
MCTD. see Mixed connective tissue disease Micturition, 122, 129
MCV. see Mean cell volume Middle East respiratory syndrome (MERS), 166, 180
MDRD. see Modification of Diet in Renal Disease Middle East respiratory syndrome coronavirus (MERS-Co\1), 80,
MOS. see Myelodysplastic syndromes 91
Mean cell haemoglobin (MCH), reference range of, 409b-41 Ob Mid-stream urine (MSU), 123, 130
Mean cell volume (MC\1), reference range of, 409b-41 Ob Migraine, 299, 304-305, 313, 318
Measles, in pregnancy, 86b with aura, 311 , 323
mecA penicillin-binding protein, 33, 35 'Million Death Study', 50
Mechanical clot retrieval, for cerebral ischaemia, 326, 329 Mineralocorticoids, 200
Median nerve compression, 284, 293 Minimal change disease, 115, 124
Medical ophthalmology, 330-335 Minocycline, papulopustular acne, 359-360
Medical psychiatry, 336-344 Minoxidil, 362
Medication reconciliation, errors and, 7, 11 20-Minute whole-blood clotting test (20WBC1), 46, 48
Medullary thyroid cancer, 197 Mitochondrial inheritance, 16, 19
Mefenamic acid, 44, 45b Mitosis (M), in cell cycle, 383, 389
Mefloquine, 35 Mitral stenosis, pre-systolic accentuation in, 140, 151
Melanoma Mitral valve prolapse, 143, 146-147
malignant Mixed connective tissue disease (MCTD), 287, 296
in men, 354, 363 Mobitz type I second-degree AV block, 145
prognosis of, 350, 3501, 361 Mobitz type II second-degree AV block, 134, 145
management of, 346-34 7, 359 Model for End-Stage Liver Disease (MELD), 250, 258
Melarsoprol, for trypanosomiasis, 84, 95 Modification of Diet in Renal Disease (MDRD), 115, 124
MELAS syndrome, 16, 19 MODY. see Maturity-onset diabetes of the young
MELD. see Model for End-Stage Liver Disease Mofetil, 371
MEN syndromes, 199 Mohs' micrographic surgery
MEN1 genes for benign keratoacanthoma, 360
acromegaly and, 1g3, 202 for nodular basal cell carcinoma, 359, 362
mutation of, 233, 243 Molluscum contagiosum, 360-361
Menaquinone. see Vitamin K Monkeypox, 82, 93
Menetrier's disease, 227, 237 Monoamine oxidase inhibitor (MAOI), 41b, 44b
Meningitis, mumps and, 77, 89 Monoclonal gammopathy of uncertain significance (MGUS),
Meningococcal meningitis, 304, 317 264, 272
Meningococcal sepsis, 300-301, 314 Monocytes, reference range of, 409b-41 Ob
sickle cell disease and, 268-269 Monoethylglycinexylidide (MEGX) test, 253
Mental state examination (MSE), 340 Mononeuritis, 293
Mephedrone, 38, 42b, 43 Montelukast, 164-165, 179,365
Meropenem, 34, 91 for asthma, 165, 179

downloaded from www.medicalbr.com


436 • INDEX

Morphine, 9-10, 42b, 72, 154 Neuroendocrine system activation, 138


brain-death testing and, 63, 72 Neurology, 299-324
for pa1n control, 394-395 Neuropathic pain, 40Gb
Motor loss, in total anterior circulation stroke, 326, 329 Neuropathy, secondary to myeloma, 312 , 323-324
Motor nerves, delayed conduction in, in chronic inflammatory Neuropeptides, in pain processing, in spinal cord, 397b
demyelinating polyneuropathy (CIDP). 309, 321 Neutrophil function tests , for recurrent lower respiratory tract
Motor neuron disease, 307, 320 infection, 24, 27
Mouth ulcers, 225-235 Neutrophil granulocytes, reference range of, 409b-41 Ob
Moxifloxacin, QT prolongation and, 7, 11 Next-generat1on sequencing (NGS), 'capture' in, 17, 20
MRI, 390 NGS. see Next-generation sequencing
MSE. see Mental state examination Niacin. see Vitamin 8 3
MSU . see Mid-stream urine Nicotinamide. see also Vitamin 8 3
Multi-organ dysfunction, following arthroplasty, 54-64 for pellagra, 205,' 208
Multiple biopsies, in ulcerative colitis , 386, 391 Nicotinic acid . see Vitamin 8 3
Multiple metastases, 384, 389 Nifurtimox, 95
Multiple myeloma, 264, 272 for Chagas disease, 79, 91
Multiple sclerosis, development of, 308, 321 Nihilistic delusion, 341
Mumps, 82-83, 93 Nijmegen questionnaire, 162, 177
meningitis and, 77, 89 Nikolsky sign, in bullous pemphigoid, 352-353, 3521, 363
Muscle electromyogram (EMG). for polymyositis, 288, 296 Nitrates, 184
Muscle MRI, for polymyositis, 288, 296 Nitrofurantoin, 170, 182
Muscle relaxants, 70 lung disease, 170, 182
Muscles, inspection of, 76, 88 Nits, 34 7-348, 360
Mycetoma, haemoptysis in, 163, 178 NNRTis. see Non-nucleoside reverse transcriptase inhibitors
Mycobacterium chelonae, 92 NNT. see Number needed to treat
Mycobacterium tuberculosis, 180 Nocardia sp., 92
Mycophenolate, 128, 371 Nociceptin, in pain processing, in spinal cord, 397b
Mycophenolate immunosuppression, 25 Nodules, 291
yellow fever vaccine and, 77, 89 Non-alcoholic fatty liver disease (NAFLD)
Mycophenolate mofetil, 120-121, 128 abnormal liver biochemistry and, 252, 260
Mydriasis, pharmacological, 330, 333 diagnosis and treatment of, 251, 258
Myelodysplastic syndromes (MDS), 264, 272 severity of, 251, 258
Myeloma, neuropathy secondary to, 312, 323-324 triple-phase CT scan for, 249, 257
'Myeloma kidney' (cast nephropathy), pathogenesis of, 120, type 2 diabetes mellitus and, 213-214, 219-220
128 Non-competitive antagonists, effects on pharmacodynamic
Myeloproliferation, in clozapine, 339, 344 actions of agon1st, 6, 9
Myocardial infarction, 133, 144, 379, 382 Non-disabling relapse, 308, 320-321
mortality following, age and, 140, 151 Non-HDL cholesterol, 107-1 08, 112
MRI for, 142, 152 Non-invasive ventilation, for type II respiratory failure,
perioperative risk factor for, during non-cardiac surgery, 142, 61' 70-71
152-153 Non-nucleoside reverse transcriptase inhibitors (NNRTis), 1 01 ,
smoking and, 138, 149 101b
cessation of, 142, 153 Non-selective ~-adrenoceptor antagonists, for variceal
ST segment elevation, tissue plasminogen activator for, 138, haemorrhage, 252, 259
149 Nonsense mutation, 14-15, 18
transthoracic echocardiography for, 141 , 152 Non-small cell cancer, in asbestosis, 391-392
Myocardial infiltration, 147 Non-steroidal anti-inflammatory drugs (NSAIDs), 40b, 44, 45b,
Myocarditis, viral, venous-arterial ECMO for, 56, 66 115, 124, 147,236-237, 290,360-361,399
Myotonic dystrophy, 143 for bone pain, 40Gb
type 1, genetic counselling for, 15, 18-19 for ischaemic pain, 400b
lichenoid reactions caused by, 364-365
N for osteoarthritis of hip, 281-282, 291-292
NAFLD. see Non-alcoholic fatty liver disease Noradrenaline (norepinephrine)
Nail, longitudinal ridging of, 351, 362 arthroplasty and, 54-64
Naloxone, 42b intubation and, 57, 66
Narcolepsy, 300, 313 Normetadrenaline, in urine, 408b
National health information systems, 29, 31 Novorapid, 332
National screening programme, Wi lson and Jungner's criteria 'Novotreat', 29, 31
for, 28, 30 NRTis. see Nucleoside reverse transcriptase inhibitors
Natrix natrix. see Grass snake NSAID-induced ulcer, 227-228, 237
Nebulised salbutamol, for acute severe asthma, during management of, 228, 237-238
pregnancy, 367, 369 Nuchal rigidity, likelihood ratio for, 1 , 3
Neck swelling, mumps and, 82-83, 93 Nucleic acid amplification testing, for respiratory pathogens,
Necrotising fasciitis, 82, 93 161, 177
Neisseria gonorrhoeae infection, in pregnancy, 86b Nucleoside reverse transcriptase inhibitors (NRTis), 101, 101b
Neonatal alloimmune thrombocytopenia, 269, 277 Number needed to treat (NNT), 29-31
Neotropical rattlesnake or cascabel (Crotalus durissus Nutrition , enteral route of, 206, 210
terrificus}, 47, 49 Nutritional factors, in disease, 203-211
Nephrology and urology, 115-13 1 Nystagmus, in acute labyrinthitis, 304, 318
Nephron Nystatin, 235
amino acids in, 108, 112
numbers, red uction of, 379, 381-382 0
Nephropathy, HIV-associated, 99, 102 Obesity
Nerium oleander (pink oleander), 45b in osteoarthritis, 278, 290
Nerve block, for incident pain, 40Gb small testes and, 198
Neuraminidase inhibitors, 89 Obsessive-compulsive disorder, 342
Neuroborreliosis, 73, 85 Obstructive sleep apnoea, sleep study for, 173, 184

downloaded from www.medicalbr.com


--- INDEX • 437

Oesophageal carcinoma, 226, 236 Oxalate, in urine, 408b


investigation and management of, 227, 236 Oxycodone, 42b
Oesophagogastroduodenoscopy, 90 for chronic abdominal pain, 394
Oesophagus, major aphthous ulceration of, 98 Oxygen saturation, in arterial blood, 403b
1 7~-0estradiol Oxytetracycline, 105
in growth and puberty, 411 b-412b
in pregnancy, 411 b p
in venous blood, 403b- 404b Pacemaker, 380
Oestrogen, hepatocellular carcinoma from, 391 permanent, for sick sinus syndrome, 135, 146
Oftoxacin, 105 Packed cell volume (PCV)
0 1. see Osteogenesis imperfecta in pregnancy, 411 b
Oligoarticular juvenile idiopathic arthritis, 286, 295 reference range of, 409b-41 Ob
Omalizumab, 165, 179 PaC02 , in arterial blood , 403b
Omeprazole, 119, 121,127, 161-162,225 Paget's disease, 283, 2831, 292
Oncology, 383-392 of bone, 283, 292
Open muscle biopsy, for polymyositis, 288, 296 Pain
Open radical nephrectomy, 122, 130 localisation of, 398b
Ophthalmological review, for candidaemia, 79, 90 palliative care and, 393- 401
Opiate medication spontaneous, 393, 397
for chronic obstructive pulmonary disease, 399 types of, in cancer, 400b
side-effects of, 395, 399 Pain Catastrophising Scale, 398b
Opioids, 42b Pain Detect, in neuropathic pain, 393, 398, 398b
for borderline cognitive impairment, 393-394, 398, 398b Pain Self-efficacy Questionnaire, 398b
for breathlessness, 400 Painful obstructive jaundice, ultrasound for, 252, 259
for chronic abdominal pain, 399 Palatoglossus, genioglossus and, 173, 184
for complex regional pain syndrome (CRPS) type 1, 394 Palliative care, 399
for pain for chronic obstructive pulmonary disease (COPD), 395, 399
in oropharyngeal tumour, 394, 399 pain and, 393-401
processing, in spinal cord, 397b Palmoplantar pustulosis, PUVA for, 349, 3491, 360
Optic discs, blind spots and, 303, 316 Palpitations, 132
Optic neuritis, 308, 320 Pancreas
demyelinating, 331, 334 arterial blood supply to, 232, 241
retrobulbar, 334 calcification, in cystic fibrosis, 233, 2331, 242
Oral 5-aminosalicylic acid (5-ASA), 231, 240 Pancreatic cancer, 245, 253
Oral antiviral therapy, for thrombocytopenia, 248, 256-257 treatment w ithdrawal and, 54, 64
Oral bisphosphonate therapy, for osteoporosis, 280, 291 Pancreatic insufficiency, 212, 218
Oral contraceptives, 39b Pancreatic lipase, deficiency of, 232, 242
cystic fibrosis and, 372, 375 Pancreatitis, 175
Oral dryness, in primary Sjogren's syndrome (PSS), 288, 297 after endoscopic retrograde cholangiopancreatography
Oral fluconazole, 235 (ERCP), 233 , 242
Oral glucose tolerance test, 192 , 201 in paracetamol overdose, 245, 253
Oral hairy leucoplakia, 97, 100 Panic disorder, 337, 342
Oral mucosal bleeding, in immune thrombocytopenia, 261, 270 Pan-retinal laser therapy, 332, 3321, 335
Oral nucleoside, as post-liver transplant prophylaxis, 249, 257 Pa02 , 64-65
Orchitis, post-infection, 198-199 in arterial blood, 403b
Organ transplantation, 24, 26-27 Papillary serous carcinoma, 390
Organic nitrites, 42b Papilloedema, causes of, 331, 334
Organophosphate insecticides, 45b Paracetamol, 32, 34, 37-38, 42b, 44, 119, 159, 360-361,
Organophosphorus compounds, 42b 377-379, 393-394, 399
Orthodeoxia, 64 for osteoarthritis, 278, 290
Orthopnoea, 132, 143 overdose, 245, 253
Oseltamivir, 369 toxicity, 38-39, 43
for influenza infection prophylaxis, 77, 89 Paraprotein assessment, for rheumatoid arthritis, 24, 27
Osmolality Parathyroid adenoma, 199-200
definition of, 407 Parathyroid hormone (PTH)
urine, 408 effects of, 190, 199
in venous blood, 405b-406b in venous blood, 403b-404b
Osmolarity, in venous blood, 405b-406b Parathyroid surgery, for primary hyperparathyroidism, 190, 199
Osteoarthritis, 278, 281, 290-291, 293 Parenteral medication, for end-stage cardiac failure, 396, 401b
development of, 281 , 291 Parenteral vitamins (Pabrinex), for Wernicke-Korsakoff
of hip, treatment of, 281 - 282, 291 - 292 syndrome, 339, 343-344
Osteoblasts, 290 Parkinsonian gait, 382
Osteoclasts, 290 Parkinson's disease, 305, 318- 319
Osteocytes, 278-290 gait and, 379, 382
Osteogenesis imperfecta (01), 283, 292 investigation of, 306, 319
type II, due to dominant negative mutation, 15, 19 treatment of, 305-306, 319
Osteomalacia, 282, 292 Paroxysmal nocturnal dyspnoea, 143
Osteomyelitis, 293 Parvovirus 819, 97, 100
Osteoporosis, 280, 291 infection
post-menopausal, 280, 291 aplasia and, 75- 76, 88
in seronegative rheumatoid arthritis, 279, 291 in pregnancy, 86b
Osteoporotic fracture, 286, 295 PASI. see Psoriasis area and severitv index
vertebral, 280, 2801, 291 Patch testing
Osteoprotegerin, 290 for latex rubber allergy, 362
Osteosarcoma, 285, 293 for urticaria, 361
Ovarian cancer, 389 Patent foramen ovale, 146-1 47
Ovulation, tests for, 189, 197, 1971 Patient-centred evidence-based medicine, 2, 4

downloaded from www.medicalbr.com


438 • INDEX

Patients, treatment of, 2, 4 Plantar reflexes , upgoing , 304 , 317


PBC. s ee Primary bi liary cirrhosis Plaque psoriasis, adalimumab for, 352, 362
PBI. see Pressu re bandage and immobilisation Plasma osmolality, 407
PCOS. see Polycystic ovarian syndrome Plasmodium knowles! infection, primates and, 81 , 92
PCR. see Polymerase chain reaction Plasmodium vivax, 105-106
PCV. see Packed cell volume Platelet count, CML and, 264, 272-273
Peak expiratory flow diary, occupational asthma and , 171, 182 Platelets, reference range of, 409b-41 Ob
Pediculus humanus capitis, 347-348 , 360 Pleural effusion
Pelvic floor right-sided, 157 , 1571, 175
damage to, management of, 23 1, 240-241 cessation of apixaban for, 157, 175
exercises , 123, 131 transudative, hypothyroidism and, 164, 179
for stress incontinence, 380 Pleural fluid, analytes in, 409b
Pelvis radiograph, for inflammatory back pain, 285, 294 Plummer-V1nson syndrome, 235-236
Pemphigus, long-term treatment for, 356, 365 PMF. see Progressive massive fibrosis
Pendred's syndrome, 194 Pneumocystis jirovecii, 180
Pendrin, 194 Pneumocystis jirovecii pneumonia, 97 , 97f, 101
Penicillamine Pneumonia, 178
causing lichenoid reactions, 364-365 drug-induced chronic eosinophilic , 170, 182
causing pemphigus, 365 falls and, 379, 382
Penicillins, 35 hospital-acquired
broad-spectrum, 11 Acinetobacter and, 167, 180
Pentoxifylline, for alcoholic hepatitis, 258- 259 local antibiotic policy for, 166-167 , 180
Perception, tactile hallucinations, 336, 340 mortality from, 167, 180
Periarticular osteoporosis, 291 interstitial
Pericardia! effusion, 137- 138, 174 bronchoscopy and, 176
Pericardiocentesis, for basal cell carcinoma, 384, 389 CT appearance of, 160, 176
Pericarditis lymphocytic, 182
acute, 137 Pneumonitis, hypersensitivity, 171- 172, 183
aspirin for, 136, 147 Pneumothorax, 17 4
viral, 136, 147 left-sided, 157, 1571, 175
chest pain and, 58, 67 therapeutic aspiration for, 158, 1581, 175-1 76
PR segment depression in, 137, 148 primary spontaneous, intercostal chest drain for, 173- 17 4,
Peri-odontoid ligaments, calcification of, 285, 294 184
Periorbital haematoma, 330-333 secondary spontaneous, 59, 68-69
Peripheral embolism, with lower limb ischaemia, 132- 133, 143 POCT. see Point -of-care test
Peripheral neuropathy, 378-379, 381 Point-of-care test (POCT), 107-111
Peripheral vascular disease, 217, 223 Poisoning, 37-44
Peritoneal dialysis, 118, 126 Poly ADP ribose polymerase (PARP) inhibitors, mechanism of,
Persecutory delusion, 341 18, 21
Persistent delusional disorder, 336, 341 Polyarticular joint involvement, in gout, in older woman, 286,
Persistent ductus arteriosus, 146-147 294-295
Persistent non-visible haematuria, 123, 130 Polycystic kidney disease, adult, 117-118, 11 71, 126
Personality disorders, prevalence of, 341 b Polycystic ovarian syndrome (PCOS), hyperandrogenism and ,
Pethidine, 42b 189, 198
Phaeochromocytoma, phenoxybenzamine for, 191, 200 Polycythaemia rubra vera (PRV), ischaemic stroke and, 261,
Phagocytes, key feature of, 22 , 25 270
Pharmacovigilance, voluntary reporting of, 7, 10 Polymerase chain reaction (PCR), for Cyclospora cayetanensis
Phencyclidine, 42b infection, 75, 87
Phenoxybenzamine, for phaeochromocytoma, 191 , 200 Polymorphic eruption of pregnancy, 350, 361
Phenytoin Polymorphic light eruption, 363- 364
in cerebellar function , 310, 322 causi ng photo -aggravation of psoriasis, 347, 3471, 359
zero-order drug kinetics in, 6, 9 diagnosis of, 345-358, 3451
Phosphate Polymyalgia, 292
in urine, 408b Population health and epidemiology, 28-31
in venous blood, 405b-406b Porphyria cutanea tarda, investigations for, 351 , 351 I, 357,
Phosphodiesterase inhibitor, 138 361, 365
Phosphodiesterase 4 (PDE4), in apremilast, 287, 296 Porphyrin plasma scan, 351, 351 I, 357, 361 , 365
Photo-ag eing, 362-363 Posaconazole, 27 4
Photocoagulation, pan-retinal, 332, 3321, 335 Positron emission tomography (PET) scan
Photodynamic therapy for inflammation site identification, 74, 86-87
for basal cell carcinoma, 353, 3531, 362-363 for pulmonary nodule, 156, 1561, 163-164, 175, 178--179
for Bowen's disease, 3461, 358- 359 Post-concussion syndrome, 310, 322
with verteporfin, 335 Post-head injury, anosmia and ageusia caused by, 307, 320
Phylloquinone. see Vitamin K Post-menopausal osteoporosis, 280, 291
Pigbel, 95 Post-partum blues, 343
Pilocarpine, 298 Post-polio syndrome, 174, 184
for Adie's pupil, 333 Post-test probability, of hip fracture, 1, 3-4
Pin-prick testing, for dissociated sensory loss, 299, 313 Post-transplant lymphoproliferative disorder, 372, 375
Pioglitazone, 219 Post-traumatic stress disorder (PTSD), 337, 342- 343
Piperacillin, 274 Postural hypotension, 143
Piperacillin/tazobactam, 91 accompanied by tachycardia, in joint hypermobility, 284,
for neutropenic fever, 266, 27 4 293
Pituitary adenoma, 232 , 242 Potassium, 108, 112
Pituitary apoplexy, 192, 202 in DKA management, 215, 222
Pityriasis rosea, 363-364 plasma concentration of, 110, 113
Pityriasis versicolor, 354, 3541, 363-364 in urine, 408b
Placebo or 'Novotreat', 29, 31 in venous blood, 403b

downloaded from www.medicalbr.com


- INDEX • 439

Potency, of drug, 6, 9 Propionibacterium acnes, 289, 297


PPCI. see Primary percutaneous coronary intervention Propofol, for intracranial pressure, 63, 71
PPis. see Proton pump inhibitors Propranolol
Pralidoxime, 42b for post-partum thyroiditis, 186, 195
Prazocin, 49 for variceal haemorrhage, 252, 259-260
Pre-diabetes, diagnostic cut-offs in, 413b Proprioception testing, for Dissociated sensory loss,
Predictive value, of test, 1 , 1t, 3 299, 313
Prednisolone, 25, 39b, 74, 87, 162-165, 170, 179, 371 Prostate cancer, 130
for alcoholic hepatitis, 258-259 Prostate ultrasound, 130
drug-induced diabetes caused by, 215, 221-222 Protease inhibitors (Pis), 101, 101 b
for EMV infection, 74, 86 Protein
for liver capsule pain, 395, 399, 400b in urine, 408b
oral, for asthma, pregnancy and, 164-165, 179 in venous blood, 4o5b-406b
pustular psoriasis and, 356, 3561, 364 Protein energy malnutrition, 30
for rheumatoid arthritis, 279, 291 Proteinase 3-positive ANCA-associated vasculitis (AA\!j, 288,
Pre-eclampsia, 118, 126 297
during pregnancy, with renal disease, 371, 374 Prothrombin complex concentrate, 275
Pre-frail, 378, 381, 381b Prothrombin time (PT), reference range of, 409b-41Ob
Pregabalin, 397- 398 Proton pump inhibitors (PPis), 226, 236
for neuropathic pain, 400b causing lichenoid reactions, 364- 365
Pregnancy for gastrointestinal bleeding, 57, 66
acitretin, 351, 361-362 Prucalopride, 232, 241
acute fatty liver of, 367, 369 PRV. see Polycythaemia rubra vera
aortic dissection and, 139, 150 Pseudoephedrine, 42b
autoimmune hepatitis in, azathioprine monotherapy for, 246, Pseudogout, 294
254-255 Pseudo-hallucination, 341
avoiding unprotected sex during, 105-106 Pseudomonas aeruginosa, cyst1c fibrosis and, 166, 180
bariatric surgery and, 204, 208 Pseudoseizures, pregnancy and, 366, 368
community-acquired pneumonia during, treatment of, 367, Psi/ocybe semilanceata ('magic mushrooms'), 45b
369 Psoas abscess, mycobacterial testing of, 167, 181
diabetes during, medication for, 367, 369 Psoralen and ultraviolet A (PUVA), for palmoplantar pustulosis,
diffuse symmetrical goitre and, 187, 196 349, 3491, 360
epilepsy during, folic acid for, 366, 368 Psoriasis, 363-364
infections in, 86b Psoriasis area and severity index (PAS!), 351, 362
w ith lupus nephritis, 367, 369 Psoriatic arthritis, 289, 2891, 298, 298b
methotrexate during, 366-368 PSP. see Primary spontaneous pneumothorax
w ith renal disease, 371, 374 PSS. see Primary Sjogren's syndrome
respiratory rate during, 366, 368 Psychiatric disorders
rheumatoid arthritis and, 281, 291 biological basis of, 339, 343
sodium valproate on, 8, 11 prevalence of, 341b
spider naevi during, 367, 369 Psychiatric history, 336-340
ulcerative colitis during, methotrexate for, 367-369 Psychiatric interview, 336-340, 340b
Pregnancy-associated dissection, 150 Ptosis, wound botulism and, 82, 93
Preload, stroke volume and, 60, 69 PTSD. see Post-traumatic stress disorder
Premature ovarian failure, karyotype for, 189, 198 Puberty, in male, 370- 373
Pressure bandage and immobilisation (PBI), 47, 49 Puerperal psychosis, 338-339, 343
Prick testing Pulmonary embolism, 4
for latex rubber allergy, 362 sinus tachycardia in, 137, 148
for urticaria, 361 suspected, physical examination for, 2, 4
Primary adrenal failure, 193 Pulmonary fibrosis, idiopathic, 159, 1591, 169, 176, 181
Primary biliary cholangitis, 246, 254 Pulmonary hypertension, 172- 173, 183
alkaline phosphatase (ALP) level in, 24 7, 255 bosentan for, 173, 184
ERCP for, 250, 257 transthoracic echocardiography for, 173, 183- 184
Primary biliary cirrhosis (PBC), 245, 253 Pulmonary infarction, 178
Primary hyperparathyroidism, parathyroid surgery for, 190, 199 Pulmonary Langerhans cell histiocytosis, 182
Primary immune deficiency, 22, 25-26 Pulmonary nodules
Primary percutaneous coronary intervention (PPCI), 139-140, PET scanning in, 156, 1561, 163-164, 175, 178- 179
150-151 ' 366-368 risk of malignancy in, upper lobe distribution and, 163, 178
Primary sclerosing cholangitis, 247, 256 Pulmonary oedema
Primary Sjogren's syndrome (PSS), 286, 288, 294, 297 acute, treatment for, 142, 152
malignancy and, 289, 297- 298 Hantavirus infection and, 82, 93
Primary spontaneous pneumothorax (PSP), intercostal chest Pulmonary thromboembolism, 162-163, 178
drain for, 173- 174, 184 CT pulmonary angiogram for, 172, 183
Primates, Plasmodium knowlesi infection and, 81, 92 thrombolysis for, 172, 183
Procainamide, intravenous bolus dose of, 39-40, 40b Pulmotrauma, 61 , 71
Procyclidine, 40b, 45b Pulseless electrical activity (PEA) cardiac arrest, 133, 144
Progesterone Pulsus paradoxus, 138, 148
in pregnancy, 411b Punched out erosions, 291
in venous blood, 403b-404b Pustular plantar foot rash, 295
Progesterone dermatitis, 364 Pustular psoriasis, prednisolone and, 356, 3561, 364
Progressive degenerative disorder, 311, 322- 323 Pyogenic liver abscess, 249, 257
Progressive massive fibrosis (PMF), 171, 182-183 Pyrexia, post-cardiac arrest and, 61, 70
Prolactin (PRL), 193, 389 Pyridoxine. see Vitamin 8 6
for pituitary macroadenoma, 193, 202
in pregnancy, 411b Q
in venous blood, 403b-404b QT prolongation, moxifloxacin and, 7, 11
Prolonged post-ictal dysphasia, 302, 315 Quadrantanopia, right inferior, 304, 318

downloaded from www.medicalbr.com


L
440 • INDEX

Quadriceps, wasting of, in spinal cord, lesions outside, Respiratory rate, as sign of clinical deterioration, 60, 69
299-300, 313 Restrictive cardiomyopathy, 138, 149
Quartz, silicosis and, 171, 183 Retention of contrast, malignancy and, 191, 200
Quinidine, 40b, 45b Reticulocytes, reference range of, 409b-41 Ob
Quinine, 271 Retinal vein occlusion, atherosclerosis causing, 332, 332f, 334
Ouinolone, 27 4 Retinoids, systemic, for papulopustular acne, 359-360
with antichlamydial efficacy, 105 Retinol. see Vitamin A
Retrobulbar haemorrhage, 330-333
R Reversibility, delirium and, 58, 68
RAAS. see Renin-angiotensin-aldosterone system Reynolds' pentad of ascending cholangitis, 205, 209
Rabies vaccine, for bat bites, 82, 93 RFTs. see Respiratory function tests
Radiation, stochastic (random) effect of, 51-52 Rhabdomyolysis, 119, 127
Radical cystectomy, 122-123, 130 tests for, 60, 70
Radioactive iodine Rheumatic fever
for Graves' disease, 187, 195-196 acute, c-reactive protein and, 140, 151
post-thyroidectomy, 197 aspirin for, 140, 151
Radioallergosorbent testing (RAS1), 352, 362 Rheumatoid arthntis, 170, 1701, 182
Radiotherapy Rheumatoid nodules, 169-170, 182
for bone pain, 400b Rheumatology, 278-298
for increased intracranial pressure, 40Gb Rhinovrrus, acute coryza and, 166, 180
Raltegravi r, 98, 101 b Riboflavin. see Vitamin B2
Ramipril. see Angiotensin-converting enzyme (ACE) inhibrtor Richmond Agitation and Sedation Score (RASS), 55, 65
Random venous blood glucose, for diagnosis, 212-218 Ricinus communis (castor oil plant), 45b
Randomisation, 29-30 Rickets, X-linked hypophosphataemic, 282- 283, 292
Ranitidine, 365 Rickettsia alricae infection, tick bites and, 78, 90
RANK. see Receptor activator of nuclear factor kappa B Rifampicin, 167
Rash on contraceptive failure, 7, 11
monkeypox and, 82, 93 cytochrome P450 induced by, 6, 9
scabies and, 80, 91 unpasteurised milk and, 76, 88
RASS. see Richmond Agitation and Sedation Score Rifamycin, 11
RAST. see Radioallergosorbent testing Right arm weakness, headache and, 59, 69
Rathke's pouch, development of, into adenohypophysis, 192, Right lung, apex of, irregularly shaped mass at, 387, 391
201 Right pleural effusion, 388, 392
Raynaud's disease, 150 Right subthalamic nucleus, lesion in, 307, 319-320
RBILD. see Respiratory bronchiolitis-interstitial lung disease Right upper lobectomy, 156, 1561, 175
Reactive arthritis, 290, 294 Rilpivirine, 101b
Receptor activator of nuclear factor kappa B (RANK), 278, 290 Rimantidine, 89
Receptor antagonists, 42b Rituximab, 24, 287, 295, 298
Recombinant tissue plasminogen activator (rt-PA), intravenous Road injury, causing adolescent deaths, 370, 374
thrombolysis with, 326-327, 329 Rodents, Francisef!a tularensis infection from , 83-84, 94
Recompression, 53 ROSIER (Rule Out Stroke In Emergency Room) clinical stroke
Recurrent oral thrush, 225, 235 tool, 326, 328, 329b
Red cell count, reference range of, 409b-41 Ob Rubella, in pregnancy, 86b
Red cell lifespan, reference range of, 409b-41 Ob Ruminations, 338, 343
Red cell transfusion Ruptured berry aneurysm, 121, 129
for sickle cell disease, 268, 275 Russell's viper (Daboia russelii and Daboia siamensis), 46, 48
for upper Gl haemorrhage, 63-64, 72
Reducing agents, 42b s
5a-Reductase inhibitor, 123, 131 SABA. see Short-acting ~,-agonist
Refeeding syndrome, chronic alcoholism and, 203-204, Sacroiliac disease, 294
207-208, 208b Sacroiliac joints (SIJs)
Reference ranges, 402-414 irregularity and fusion of, 291
immunological, 41 Ob MRI, for inflammatory back pain, 285, 294
laboratory Salbutamol, 162, 165
in adolescence, 411 - 414 inhaler, 164-165
in adults, 403-409, 403b nebulised, 333
in chi ldhood, 411-414 Salicylates, 37-38, 43b
in pregnancy, 411 Salivation, 39, 43, 44b
Regular prompted toileting, for incontinence, 378, 381 Sarcoidosis, 128, 169, 1691, 181, 183
Renal artery stenosis, 381-382 stage IV, 169, 181-182
Renal disease, secondary hypertension and, 140, 151 Saudi Arabia, coronavirus and, 33, 35
Renal failure, chronic, hyperlactaemia in, 61, 70 Saw-scaled (carpet) viper (Echis ocellatus), 47, 49
Renal function, deterioration in, 118, 126 Scabies, 80, 91
Renal replacement therapy, for hyperkalaemia, 62, 71 Scanning speech, in dysarthria, 303, 315-316, 316b
Renal tubular acidosis, type 2, 110, 114 Scheuermann's disease, 283, 292
Renin, 38g Schistosoma haematobium infection, squamous cell bladder
in venous blood, 403b-404b carcinoma and, 78, 89-90
Renin-angiotensin-aldosterone system (RAAS), chronic cardiac Schizophrenia, 28, 30, 343
failure associated with, 134, 145 clozapine for, 339, 344
Respiratory alkalosis, 111, 114 diagnosis of, 339, 344
Respiratory bronchiolitis-interstitial lung disease (RBILD), 169, 181 prevalence of, 336, 340, 341 b
Respiratory failure , acute type I, lobar collapse and, 164, 179 Scombroid poisoning, uncoo~d tuna and, 83, 94
Respiratory function tests (RFTs) Scombrotoxic fish poisoning, 40
in autoimmune connective tissue disease, long-term Scopolamine, 41b, 44b
management of, 288, 296 SCORTEN, 358, 359b
interpretation of, 161 , 177 SCRAs. see Synthetic cannabinoid receptor agonists
reversible obstructive defect revealed by, 161, 177 Secondary immune deficiency, 23, 26

downloaded from www.medicalbr.com


INDEX • 439

Potency, of drug, 6, 9 Propionibacterium acnes, 289, 297


PPCI. see Primary percutaneous coronary intervention Propofol, for intracranial pressure, 63, 71
PPis. see Proton pump inhibitors Propranolol
Pralidoxime, 42b for post-partum thyroiditis, 186, 195
Prazocin, 49 for variceal haemorrhage, 252, 259-260
Pre-diabetes, diagnostic cut-offs in, 413b Proprioception testing, for Dissociated sensory loss,
Predictive value, of test, 1, 1t, 3 299, 313
Prednisolone, 25, 39b, 74, 87, 162-165, 170, 179, 371 Prostate cancer, 130
for alcoholic hepatitis, 258-259 Prostate ultrasound, 130
drug-induced diabetes caused by, 215, 221-222 Protease inhibitors (Pis), 101, 101b
for EMV infection, 74, 86 Protein
for liver capsule pain, 395, 399, 400b in urine, 408b
oral, for asthma, pregnancy and, 164-165, 179 in venous blood, 405b- 406b
pustular psoriasis and, 356, 356f, 364 Protein energy malnutrition, 30
for rheumatoid arthritis, 279, 291 Proteinase 3-positive ANCA-associated vasculitis (AAV), 288,
Pre-eclampsia, 11 8, 126 297
during pregnancy, with renal disease, 371, 374 Prothrombin complex concentrate, 275
Pre-frail, 378, 381, 381b Prothrombin time (P1], reference range of, 409b-41 Ob
Pregabalin, 397- 398 Proton pump inhibitors (PPis), 226, 236
for neuropathic pain, 400b causing lichenoid reactions, 364- 365
Pregnancy for gastrointestinal bleeding, 57, 66
acitretin, 351, 361-362 Prucalopride, 232, 241
acute fatty liver of, 367, 369 PRV. see Polycythaemia rubra vera
aortic dissection and, 139, 150 Pseudoephedrine, 42b
autoimmune hepatitis in, azathioprine monotherapy for, 246, Pseudogout, 294
254-255 Pseudo-hallucination, 341
avoiding unprotected sex during, 105-1 06 Pseudomonas aeruginosa, cystic fibrosis and, 166, 180
bariatric surgery and, 204, 208 Pseudoseizures, pregnancy and, 366, 368
community-acquired pneumonia during, treatment of, 367, Psi/ocybe semilanceata ('magic mushrooms'), 45b
369 Psoas abscess, mycobacterial testing of, 167, 181
diabetes during, medication for, 367, 369 Psoralen and ultraviolet A (PUVA), for palmoplantar pustulosis,
diffuse symmetrical goitre and, 187, 196 349, 349(, 360
epilepsy during, folic acid for, 366, 368 Psoriasis, 363-364
infections in, 86b Psoriasis area and severity index (PASI), 351, 362
with lupus nephritis, 367, 369 Psoriatic arthritis, 289, 2891, 298, 298b
methotrexate during, 366-368 PSP. see Primary spontaneous pneumothorax
with renal disease, 371, 374 PSS. see Primary Sjogren's syndrome
respiratory rate during, 366, 368 Psychiatric disorders
rheumatoid arthritis and, 281, 291 biological basis of, 339, 343
sodium valproate on, 8, 11 prevalence of, 341b
spider naevi during, 367, 369 Psychiatric history, 336-340
ulcerative colitis during, methotrexate for, 367- 369 Psychiatric interview, 336-340, 340b
Pregnancy-associated dissection, 150 Ptosis, wound botulism and, 82, 93
Preload , stroke volume and, 60, 69 PTSD. see Post-traumatic stress disorder
Premature ovarian failure, karyotype for, 189, 198 Puberty, in male, 370-373
Pressure bandage and immobilisation (PBI), 4 7, 49 Puerperal psychosis, 338-339, 343
Prick testing Pulmonary embolism, 4
for latex rubber allergy, 362 sinus tachycardia in, 137, 148
for urticaria, 361 suspected, physical examination for, 2, 4
Primary adrenal failure, 193 Pulmonary fibrosis, idiopathic, 159, 1591, 169, 176, 181
Primary biliary cholangitis, 246, 254 Pulmonary hypertension, 172- 173, 183
alkaline phosphatase (ALP) level in, 247, 255 bosentan for, 173, 184
ERCP for, 250, 257 transthoracic echocardiography for, 173, 183-184
Primary biliary cirrhosis (PBC), 245, 253 Pulmonary infarction, 178
Primary hyperparathyroidism, parathyroid surgery for, 190, 199 Pulmonary Langerhans cell histiocytosis, 182
Primary immune deficiency, 22, 25-26 Pulmonary nodules
Primary percutaneous coronary intervention (PPCI), 139-140, PET scanning in, 156, 1561, 163-164, 175, 178-1 79
150-151 ' 366-368 risk of malignancy in, upper lobe distribution and, 163, 178
Primary sclerosing cholangitis, 247, 256 Pulmonary oedema
Primary Sjogren's syndrome (PSS), 286, 288, 294, 297 acute, treatment for, 142, 152
malignancy and, 289, 297-298 Hantavirus infection and, 82, 93
Primary spontaneous pneumothorax (PSP), intercostal chest Pulmonary thromboembolism, 162-163, 178
drain for, 173-174, 184 CT pulmonary angiogram for, 172, 183
Primates, Plasmodium know/esi infection and, 81, 92 thrombolysis for, 172, 183
Procainamide, intravenous bolus dose of, 39-40, 40b Pulmotrauma, 61, 71
Procyclidine, 40b , 45b Pulseless electrical activity (PEA) cardiac arrest, 133, 144
Progesterone Pulsus paradoxus, 138, 148
in pregnancy, 411b Punched out erosions, 291
in venous blood, 403b-404b Pustular plantar foot rash, 295
Progesterone dermatitis, 364 Pustular psoriasis, prednisolone and, 356, 3561, 364
Progressive degenerative disorder, 311 , 322-323 Pyogenic liver abscess, 249, 257
Progressive massive fibrosis (PMF), 171, 182- 183 Pyrexia, post-cardiac arrest and, 61, .JO
Prolactin (PRL), 193, 389 Pyridoxine. see Vitamin 8 6
for pituitary macroadenoma, 193, 202
in pregnancy, 411 b Q
in venous blood, 403b-404b QT prolongation, moxifloxacin and, 7, 11
Prolonged post-ictal dysphasia, 302, 315 Quadrantanopia, right interior, 304, 318

downloaded from www.medicalbr.com


440 • INDEX

Quadriceps, wasting of, in spinal cord, lesions outside, Respiratory rate, as sign of clinical deterioration, 60, 69
299-300, 313 Restrictive cardiomyopathy, 138, 149
Quartz. silicosis and, 171 , 183 Retention of contrast, malignancy and. 191, 200
Quinidine, 40b, 45b Reticulocytes, reference range of, 409b-41 Ob
Quinine, 271 Retinal vein occlusion, atherosclerosis causing. 332, 3321, 334
Quinolone, 274 Retinoids, systemic, for papulopustular acne, 359-360
with antichlamydial efficacy, 105 Retinol. see Vitamin A
Retrobulbar haemorrhage, 33D-333
R Reversibility, delirium and, 58, 68
RAAS. see Renin-angiotensin-aldosterone system Reynolds' pentad of ascending cholangitis, 205, 209
Rabies vaccine, for bat bites, 82, 93 RFTs. see Respiratory function tests
Radiation, stochastic (random) effect of, 51-52 Rhabdomyolysis, 119, 127
Radical cystectomy, 122-123, 130 tests for, 60, 70.
Radioactive iodine Rheumatic fever
for Graves' disease, 187, 195-196 acute, c-reactive protein and, 140, 151
post-thyroidectomy, 197 aspirin for, 140, 151
Radioallergosorbent testing (RAST). 352. 362 Rheumatoid arthritis, 170, 1701, 182
Radiotherapy Rheumatoid nodules, 169-170, 182
for bone pain. 400b Rheumatology, 278-298
for increased intracranial pressure, 400b Rhinovirus, acute coryza and, 166, 180
Raltegravir, 98, 101 b Riboflavin. see Vitamin 8 2
Ramipril. see Angiotensin-converting enzyme (ACE) inhibitor Richmond Agitation and Sedation Score (RASS), 55, 65
Random venous blood glucose, for diagnosis, 212-218 Ricinus communis (castor oil plant), 45b
Randomisation, 29-30 Rickets, X-linked hypophosphataemic, 282-283, 292
Ranitidine, 365 Rickettsia alricae infection, tick bites and, 78, 90
RANK. see Receptor activator of nuclear factor kappa B Rifampicin, 167
Rash on contraceptive failure, 7, 11
monkeypox and, 82, 93 cytochrome P450 induced by, 6, 9
scabies and, 80, 91 unpasteurised milk and, 76, 88
RASS. see Richmond Agitation and Sedation Score Rifamycin, 11
RAST. see Radioallergosorbent testing Right arm weakness, headache and, 59, 69
Rathke's pouch, development of, into adenohypophysis. 192, Right lung, apex of, irregularly shaped mass at, 387, 391
201 Right pleural effusion, 388, 392
Raynaud's disease, 150 Right subthalamic nucleus, lesion in, 307, 319- 320
RBILD. see Respiratory bronchiolitis-interstitial lung disease Right upper lobectomy, 156, 1561, 175
Reactive arthritis, 290, 294 Rilpivirine, 101 b
Receptor activator of nuclear factor kappa B (RANK), 278, 290 Rimantidine, 89
Receptor antagonists. 42b Rituximab, 24, 287, 295, 298
Recombinant tissue plasminogen activator (rt-PA), intravenous Road injury, causing adolescent deaths, 370, 374
thrombolysis with, 326-327, 329 Rodents, Francisel/a tularensis infection from, 83-84, 94
Recompression, 53 ROSIER (Rule Out Stroke In Emergency Room) clinical stroke
Recurrent oral thrush, 225, 235 tool, 326, 328, 329b
Red cell count, reference range of, 409b-41 Db Rubella, in pregnancy, 86b
Red cell lifespan, reference range of, 409b-410b Ruminations, 338, 343
Red cell transfusion Ruptured berry aneurysm, 121 , 129
for sickle cell disease. 268, 275 Russell's viper (Daboia russe/ii and Daboia siamensis), 46, 48
for upper Gl haemorrhage, 63-64, 72
Reducing agents, 42b s
Sa-Reductase inhibitor, 123, 131 SABA. see Short-acting ~,-agoni st
Refeeding syndrome, chronic alcoholism and, 203-204, Sacroiliac disease, 294
207-208, 208b Sacroiliac joints (SIJs)
Reference ranges, 402-414 irregularity and fusion of, 291
immunological, 41 Ob MRI, for inflammatory back pain, 285, 294
laboratory Salbutamol, 162, 165
in adolescence, 411-414 inhaler, 164-165
in adults, 403-409, 403b nebulised, 333
in childhood, 411 - 414 Salicylates, 37-38, 43b
in pregnancy, 411 Salivation, 39, 43, 44b
Regular prompted toileting, for incontinence, 378, 381 Sarcoidosis, 128, 169, 1691, 181 , 183
Renal artery stenosis, 381-382 stage IV, 169, 181-182
Renal disease. secondary hypertension and, 140, 151 Saudi Arabia, coronavirus and, 33, 35
Renal failure, chronic, hyperlactaemia in, 61, 70 Saw-scaled (carpet) viper (Echis ocel/atus), 47, 49
Renal function, deterioration in, 118, 126 Scabies, 80, 91
Renal replacement therapy, for hyperkalaemia, 62, 71 Scanning speech, in dysarthria, 303, 315-316, 316b
Renal tubular acidosis, type 2, 110, 114 Scheuermann's disease, 283, 292 ·
Renin, 389 Schistosoma haematobium infection, squamous cell bladder
in venous blood, 403b-404b carcinoma and, 78, 89-90
Renin-angiotensin-aldosterone system (RAAS), chronic cardiac Schizophrenia, 28, 30, 343
failure associated with, 134, 145 clozapine for, 339, 344
Respiratory alkalosis, 111, 114 diagnosis of, 339, 344
Respiratory bronchiolitis-interstitial lung disease (RBILD), 169, 181 prevalence of, 336, 340, 341b
Respiratory failure, acute type I, lobar collapse and, 164. 179 Scombroid poisoning, unco.oked tuna and, 83, 94
Respiratory function tests (RFTs) Scombrotoxic fish poisoning, 40
in autoimmune connective tissue disease, long-term Scopolamine, 41b, 44b
management of, 288, 296 SCORTEN, 358, 359b
interpretation of, 161, 177 SCRAs. see Synthetic cannabinoid receptor agonists
reversible obstructive defect revealed by, 161 , 177 Secondary immune deficiency, 23, 26

downloaded from www.medicalbr.com


-- INDEX • 441

Secondary spontaneous pneumothorax, 59, 68-69 SOD. see Sphincter of Oddi dysfunction
Secukinumab, 291, 295-296 Sodium
Sedation, in intensive care, 55, 65 fractional excretion of, 119, 121
Sedative agents, 40b, 45b in urine, 408b
Se-homocholic acid taurine (SeHCAT) scan, for bile acid in venous blood, 403b
malabsorption, 230, 239 Sodium bicarbonate, 40b , 45b
Seizures, 38, 43 Sodium calcium edetate, 42b
stroke and, 326, 328 Sodium channel blockers, 40b, 45b
Selective serotonin re-uptake inhibitor (SSRI), 41b, 44b Sodium valproate, 313, 368
Selenium, oral, 195 on pregnancy, 8, 11
Self-harm, 331, 341, 341b as teratogenic agent, 371, 374
Self-management strategies, for chronic abdominal pain, 394, SOFA see Sequential Organ Failure Assessment
399 Somatisation disorder, 338, 343
Self-selection bias, 28, 30 Somatoform disorders, prevalence of, 341b
Seminiferous tubules, dysgenesis of, Klinefelter's syndrome Somatostatin, 200, 242
and, 190, 199 Sotalol, 40b, 45b
Sensitivity, of test, 1, 1t, 3 Spastic quadriplegic cerebral palsy, gastro-oesophageal reflux
Sensory motor neuron signs, in brainstem stroke, 299- 313 in, 372, 374-375
Sensory nerves, delayed conduction in, in chronic inflammatory SPC. see Summary of product characteristics
demyelinating polyneuropathy (ClOP), 309, 321 Sphincter of Oddi dysfunction (SOD), management for, 250,
Sepsis, 294 258
definition of, 61, 10, lOb Spider naevi, during pregnancy, 361, 369
treatment for, 61, 10 Spinal cord, lesions outside, 299- 300, 313
typhoid and, 80, 92 Splenic atrophy, coeliac disease and, 234, 244
Septic arthritis, 219, 290 Splenomegaly, abdomen and, 263-264, 272
Sequential Organ Failure Assessment (SOFA), 61, 10, lOb Spontaneous bacterial peritonitis, diagnostic paracentesis for,
Seronegative rheumatoid arthritis, 219, 290 251' 259
Seropositive rheumatoid arthritis, 280, 291 Spontaneous intracranial hypotension, 302, 315
Serotonin syndrome, 31, 40, 4 1b-42b Spontaneous pain, 393, 391
Serum adrenocorticotrophic hormone (ACTH), 384 Sporothrix spp., 92
Serum allergen testing, reference range of, 41 Ob Sputum culture, haemoptysis and, 178
Serum cortisol, 384 SQSTMI gene, mutations of, in Paget's disease of bone, 283,
Serum iron studies, 111, 126 292
Serum osmolality, 109, 113,401-408 Squamous cell carcinoma
Severe haemophilia A (factor VIII deficiency), 262, 270 bladder, Schistosoma haematobium infection and, 78,
'Severe hypoglycaemia', 214, 2 19 89-90
Sex hormone-binding globulin (SHBG), in venous blood, genetic heterogeneity in, 353, 363
405b- 406b Sri Lanka, 47, 49
Sexually acquired reactive arthritis, 281, 289-290, 295, 298 SSRI. see Selective serotonin re-uptake inhibitor
Sexually transmitted infections, 103-106 SSSS. see Staphylococcal scalded skin syndrome
SGLT2 inhibitors, 222 St Mark's solution, 2 10
SHARP. see Study of Heart and Renal Protection ST segment elevation myocardial infarction, tissue plasminogen
Short Form 36 (SF-36), 398b activator for, 138, 149
Short-acting B2 -agonist (SABA), 164-165 Stamping gait, 382
Short-acting bronchodilator, 111 Standardised 'units' (U/ L), 402
Shortness of breath, due to pulmonary oedema, 58, 67 Staphylococcal scalded skin syndrome (SSSS), 354, 3541,
Shunt, hypoxaemia and, 54, 64 363
Sick euthyroidism, 188, 196 Staphylococcus aureus, 151
Sick sinus syndrome, 134, 145 Starling's law, 134, 145
permanent pacemaker for, 135, 146 Stalin-induced myopathy, 292
Sigmoid carcinoma, 385, 390 Statins, 150, 214
Silicosis, quartz and, 111, 183 cardiovascular events with, 116, 125
Simple coal worker's pneumoconiosis, 182- 183 Steroids, 128, 147
Simvastatin, 170 for asbestosis, 391-392
Sinoatrial disease. see Sick sinus syndrome causing drug-induced diabetes, 215, 221-222
Sinus tachycardia hepatocellular carcinoma from, 391
in pulmonary embolism, 131, 148 Stevens-Johnson syndrome, 363
vasodilation and, 55-57, 561, 66 Sticking sensation, 234, 243
Sister Joseph's nodule, 228, 238 Stochastic (random) effect, of radiation, 51-52
Six-minute walk distance, 380 Stomal losses, 206, 210
Skin ulceration, Francisel/a tularensis infection and , 83-84, 94 Stool calprotectin, for lBO, 231 - 232, 241
s-LANSS. see Leeds Assessment of Neuropathic Signs and Stool culture, in Yersinia enterocolitica infection identification,
' Symptoms 82, 92- 93
Slapping one foot, in peripheral nerve problem, 303, 316 Stop-gain mutation, other name for, 14- 15, 18
SLE. see Systemic lupus erythematosus Streptococcal infection, management of, 76, 88
Sleep, breathing during, 173, 184 Streptococcus pneumoniae, 97, 101, 166, 180
Sleep paralysis, in narcolepsy, 300, 3 13 'Stress hyperglycaemia', 215, 221
Small bowel magnetic resonance imaging (MRI) enterography, Stroke, 145-146
for Crohn's disease, 231, 240 atrial fibrillation and, 135, 146
Smoking direct oral anticoagulants for, 326, 329
cessation of medicine, 325-329
for abdominal aortic aneurysm rupture, 139, 150 prevention of, 7-8, 11
fer myocardial infarction, 142, 153 seizure and, 326, 328
in coffee drinkers, 29-30 total anterior circulation, 326, 329
Crohn's disease and, 373, 375 Stroke volume, dependence on, 60, 69
myocardial infarction and, 138, 149 Study of Heart and Renal Protection (SHARP), 125
Snakebite, 41, 50 Subacute thyroiditis, 188, 1881, 196

downloaded from www.medicalbr.com


442 • INDEX

Subarachnoid haemorrhage, 60, 69 Testicular pain, mumps and, 82--<33, 93


CT head scan for, 325, 327 Testosterone
Subchondral sclerosis, in osteoarthritis, 281, 291 in growth and puberty, 411 b-412b
Subdural haematoma, 302, 315 in venous blood, 403b-404b
chronic, 58-59, 68 Tetanus, 311, 323
Sublingual lorazepam, for breathlessness, 395, 400 Tetracyclines, 35, 39b, 237
Substance abuse, prevalence of, 341 b Tetralogy of Fallot, 135, 146
Substance P, in pain processing, in spinal cord, 397b TGA see Transient global amnesia
Sucralfate enema, for radiation proctitis, 229, 239 Thalidomide, 298
Sudden collapse, 136 Theophylline, 40b, 44, 45b
Sudden death, 136-137 Therapeutic lumbar puncture, 98-99, 102
Sugar alcohols, 204, 208 Thevelia peruviana (yellow oleander), 45b
Suicide, risk factors for, 341b Thiamin. see Vitamih 8 1
Sulfasalazine, 284 Thiazide diuretics, 109, 113, 118, 126, 381, 391
with leflunomide, 291 Thiazides, 361
Sulphonamides, causing lichenoid reactions, 364-365 causing lichenoid reactions, 364-365
Sulphonylurea (gliclazide), 213, 219 Thiazolidinedione (pioglitazone), 222
Summary of product characteristics (SPC), 11 mechanism of action, 213, 219
Sunlight exposure, 208, 292 Thinking
Superior mesenteric artery territory, infarct, treatment of, 206, confirmation bias and, 2, 4
210 type 2 (analytical), time spent in, 2, 4
Supraclavicular lymph node biopsy, 172, 183 Thiopurines, side-effects of, 240
Supraventricular tachycardia, 144 Thoracic kyphoscoliosis, tuberculosis and, 174, 184
vagal manoeuvres for, 135, 146 Thoracoscopy, 160, 176
Surgical resection, for fibrolamellar hepatocellular carcinoma, Thrombectomy, for cerebral ischaemia, 326, 329
250, 258 Thrombocytopenia, in thrombotiC thrombocytopenic purpura
Surgical synovectomy, 290 (ITP), 263, 271
Swallowing difficulty, wound botulism and, 82, 93 Thrombolysis
Sydney funnel web spider, Alrax robuslus), 50 intravenous, with recombinant tissue plasminogen activator
Synacthen test, 185-193 (rt-PA), 326-327, 329
Synchronised DC cardioversion, for tachycardia, 60-61, 70 for pulmonary thromboembolism, 172, 183
Syncope, 59-60, 69 Thyroglobulin, measurement of, after thyroidectomy and neck
Syndesmophytes, 294 dissection with lymph node clearance, 188, 197
Syneresis, 333 Thyroglossal duct cysts, 195
Synovial fluid, in acute monoarthritis, 285, 294 Thyroid function tests, subclinical hypothyroidism and, 186, 195
Synovitis, in rheumatoid arthritis (RA), 294-295 Thyroid gland, structure and function of, 1941
Synovitis-acne-pustulosis-hyperostosis- osteitis (SAPHO) Thyroid hormone synthesis pathway, carbimazole and, 186,
syndrome, 289, 297 194, 1941
Synthetic cannabinoid receptor agonists (SCRAs), 38, 43 Thyroid peroxidase, 194
Syphilis Thyroidectomy, neck dissection with lymph node clearance
early latent, 104, 1041, 106 and, thyroglobulin measurement after, 188, 197
partially treated late, 104, 1041, 106 Thyroiditis, 196
in pregnancy, 86b post-partum, 368-369
primary, 104, 1041, 106 propranolol for, 186, 195
secondary, 363-364 Thyroid-stimulating hormone (TSH)
Syringomyelia, 301, 314 in pregnancy, 411 b
Systems International (SI) units, 402 secretion, increase in, 186, 194-195
Systemic lupus erythematosus (SLE), 287-288, 296 in venous blood, 403b-404b
weight gain and, 203-207
T Thyrotoxicosis, 195
T4N2MO disease, 158, 1581, 176 amiodarone-induced, 188, 197
Tachycardia glucocorticoids for, 188, 197
acute inferior myocardial infarction and, 134, 145 type I, 197
supraventricular, vagal manoeuvres for, 135, 146 type II, 197
Tachypnoea, 69 Thyroxine, 372, 377-379
Tacrolimus, 25, 128, 367 abnormalities in, heart failure due to, 138, 149
immunosuppression, yellow fever vaccine and, 77, 89 in pregnancy, 411 b
Tactile hallucinations, perception, 336, 340 in venous blood, 403b-404b
Taenia solium, as causative agent of cysticercosis, 80, 91 Tick bites
Takotsubo (stress) cardiomyopathy, 137, 147-148 Babesia microti infection and, 84, 95
Tamoxifen, 190, 199 Rickettsia alricae infection and, 78, 90
Tampa Scale of Kinesiophobia, 398b Tietze's syndrome, 178
Taxus baccala (yew), 45b Tigecycline, 34, 91
Tazobactam, 274 Timed 'get up and go' test, 378, 380-381
Tazocin, 369 Tinea capitis, 348, 3481, 360
TCA see Tricyclic antidepressant Tinea corporis, 360-361
T-cell lymphoproliferation, HTLV-1 and, 83, 94 Tinel's sign, over fibular neck, in foot drop, 304, 317
Technetium-labelled methylene diphosphonate ("'"'Tc-MDP) Tissue plasminogen activator
radionucleotide scan, 122, 129-130 intravenous, ST segment depression and, 142, 152
Teicoplanin, 34 for ST segment elevation myocardial infarction, 138, 149
Temperature Titan mutations, 152
in haemoglobin-oxygen dissociation curve, 59, 68, 681 Tocopherol. see Vitamin E
post-cardiac arrest and, 61, 70 Total iron-binding capacity, relerence range of, 409b-41 Ob
Temporal artery biopsy, 331 , 334 Tourniquets, for snakebite, 49
Tenofovir, 101, 101b, 256 Toxic adenoma, 187, 1871, 195
Tension-free vaginal tape, in stress incontinence, 380 Toxic epidermal necrolysis, 363
Tertiary hyperparathyroidism, 117, 125-126 disease severity score for, 359b

downloaded from www.medicalbr.com


-- INDEX • 443

Toxic shock syndrome, 89 Ultraviolet B (UVB)


Toxoplasmosis, in pregnancy, 86b exposure, for cutaneous vitam1n D synthesis, 349- 350, 361
Tracheal deviation, 392 phototherapy
Tracheostomy, percutaneous, complication of, 63, 72 for alopecia areata, 362
Tracheostomy tube, obstruction or displacement of, 55-56, 65 for generalised itch, 353, 363
Tramadol, 42b narrowband, for chronic urticaria, 356, 365
Transcription, components in, 14, 18 Ultraviolet (UV) radiation, 385, 3851, 390
Transferrin, reference range of, 409b-41 Ob Uncooked tuna, scombroid poisoning due to, 83, 94
Transfusion Underlying immune deficiency, 24- 25, 27
medicine, haematology and, 261-277 Unpasteurised milk, antimicrobial regimen for, 76 , 88
safety and, 267, 274 Upper gastrointestinal upset, in osteoporosis, oral
Transfusion-related acute lung injury, 267-268, 275 bisphosphonate therapy in, 280, 291
Transient global amnesia [TGA), 306, 319 Upper motor neuron signs, in brainstem stroke, 299-313
Transient ischaemic attack, dysphasia in, 325-327 Urate
Transition policy, 371, 374 in urine, 408b
Translocation, as outcome of non-homologous end-joining in venous blood, 405b-406b
between fragments from different chromosomes, 15, Urate-lowering therapy with allopurinol, 291
19 Urea
Transplantation in urine, 408b
cardiac, 136, 147 in venous blood, 403b
liver Ureteric stent, insertion of, 122, 130
assessment for, interferon treatment and, 248, 256 Urethral catheterisation, 123, 131
hepatorenal failure after, 246, 254 Uric acid, testing of, 289, 298
yellow fever vaccine and, 77, 89 Urinalysis, 115-1 24, 408
Transthoracic echocardiography Urinary flow test, 122, 130
in autoimmune connective tissue disease, long-term Urinary incontinence
management of, 288, 296 for magnetic resonance imaging (MRI), 300 , 3 14
for myocardial infarction, 141, 152 worsening, 312, 324
for pulmonary hypertension, 173, 183-184 Urinary tract, with contrast, 11 8, 127
Transudative pleural effusion, hypothyroidism and, 164, 179 Urine
Trash foot. see Dusky toes analytes in, 408b
Trauma fracture, of right wrist, 280, 291 osmolality, 109, 113 , 408
Traumatic dissection, of extracranial carotid artery, 311- 312, sodium, 109, 11 3
323 Urticaria, thyroid function tests for, 350, 3501, 361
Treatment adherence, variable, levothyroxine and, 187-188, 196 Urticarial rash, blood transfusion and, 269, 277
Treatment threshold, factors involved in, 2, 4 Ustekinumab, 240, 295-296
Trichrome pattern, vitiligo and, 356-357, 3571, 365 Uveitis, 103, 105
Tricyclic antidepressant [TCA), 40b-41b, 44, 44b-45b, 363
for depression, 340, 344 v
Triglycerides, in venous blood, 405b-406b Vacuolar myelopathy, 96-97, 100
Triiodothyronine, in venous blood, 403b- 404b Vagal manoeuvres, for supraventricular tachycardia, 135, 146
Trimethoprim, 346, 369 Vaginal bleeding, unexpected, 103- 105
Triple-negative breast cancer Valaciclovir, 35, 105- 106
poly ADP ribose polymerase (PARP) inhibitors for, 18, 21 Valganciclovir, for cytomegalovirus infection, 74, 85
stop-gain mutation and , 14-15, 18 Valsalva manoeuvre, for supraventricular tachycardia, 135, 146
Trisomy 21, aneuploidy screening of, 17, 20 Vancomycin, 151-152
Troponins for immune-mediated thrombocytopenia, 261, 270
testing, 144 Variceal bleeding, 246, 255
in venous blood, 405b-406b Varicella zoster immunoglobulin, for varicella zoster virus
Troponin I, 138, 149 infection, 73, 85, 86b
Trypanosoma brucei gambiense in"fection, 90 VAS. see Visual analogue scale
Tryptamines, 41b, 44b Vascularity of synovitis, ultrasound for, 285, 294
Tryptase, in venous blood, 405b- 406b Vasculitis, 291
TSH. see Thyroid-stimulating hormone Vasodilation, 56-57, 561, 66
TSH receptor antibodies [TRAbs), reference range of, 41 Ob Vasodilator antihypertensives, 45b
Tuberculosis, 98, 102, 177-178 Vasopressors, 40b, 45b
Turner's syndrome, 15, 151, 19 Vedolizumab, 239-240
24-hour electrocardiogram (ECG) monitoring, 379, 381 Venereal disease research laboratory test (VORL), in yaws
Typhoid fever, 89 diagnosis, 83, 94
sepsis and, 80, 92 Venom immunotherapy, for venom hypersensitivity, 24
Typical antipsychotics, 40b, 45b Venous blood
electrolytes in, 403b
u hormones in, 403b-404b
UDCA therapy, for intrauterine fetal death, 247, 255- 256 other common analytes in, 405b-406b
Ulcerative colitis urea in, 403b
Campylobacter infection and, 81, 92 Venous thromboembolism (VTE), risk factors for, 262, 271
primary sclerosing cholangitis in, 230, 239 Ventricular arrhythmia, hypertrophic cardiomyopathy and, 136,
Ulnar entrapment neuropathy, 305, 318 147
Ultrasound Ventricular ectopic beats, 144
for hepatocellular carcinoma, 252, 260 Ventricular fibrillation, 139, 149-150
prostate, 130 Ventricular septal defect, 135-136, 146-147
renal tract, 123, 130 Verapamil, 147
in rheumatology practice, 285, 294 Vesicovaginal fistula, 122, 130 •
scrotal, 123, 131 Vestibular neuronitis, 377, 380
small echogenic kidneys on, 119, 127 Vestibular repositioning, for benign paroxysmal positional
Ultraviolet A (UVA), sensitivity, on monochromator phototesting, 'vertigo (BPPV), 308, 320
345, 3451, 358 Vibration testing , for Dissociated sensory loss, 299, 313

downloaded from www.medicalbr.com


444 • INDEX

Vinyl chloride, 390 following ischaemic stroke, 325, 328, 3281


Viral conjunctivitis, 330, 333 as prophylaxis, 9, 12-13
Viral encephalitis, 300, 314 Warthin-Starry silver stain, for bacillary angiomatosis diagnosis,
Viridans streptococci, endocarditis due to, 141 , 151 81,92
antibiotic regime for, 141, 151-152 Weakness, in Guillain-Barre syndrome (GBS), 299, 313
haematuria in, 141, 151 Weaning
Visual acuity, reduced, 304, 317-318 acute respiratory distress syndrome and, 55, 65
Visual analogue scale 0/AS), 398b critical illness polyneuropathy and, 63, 72
Vitamin A (retinol), 209b Wedge deformity, in Scheuermann's disease, 283, 292
teratogenic, 383, 389 Weight gain, 203-207, 207b
Vitamin B, (thiamin), 209b Weight loss, in osteoarthritis, 278, 290
deficiency, 205, 208-209 Weight-bearing exercise, 290
Vitamin 8 2 (riboflavin), 209b Wernicke-Korsakoff syndrome
Vitamin 8 3 (niacin, nicotinic acid , nicotinamide), 209b diagnosis of, 337, 342
Vitamin 8 6 (pyridoxine), 209b management of, 339, 343-344
Vitamin 8 12 (cobalamin), 209b Whipple's disease, management of, 229, 239 ·
deficiency, 268, 275 Whole spine MRI, for inflammatory back pain, 285, 294
pregnancy and, 263, 272 Whole-body CT scan, for polymyositis, 288, 296
reference range of, 409b-41 Ob Whole-exome sequencing, whole-genome sequencing versus,
Vitamin C (ascorbic acid), 209b 17, 20
deficiency, gingival swelling and bleeding in, 206, 210 Whole-genome sequencing, whole-exome sequencing versus,
levothyroxine and, 187, 196 17, 20
Vitamin D (cholecalciferol), 209b Withdrawal, of treatment, 54, 64
deficiency, 190, 199-200 Wnt, 290
in falls, 380 Wolff-Parkinson-White syndrome, 143
insufficiency, 282, 292 Wound botulism, 82, 93
metabolism, 282, 292
in osteoporosis, 286, 291 , 294 X
supplementation, for chronic tiredness, 204-205, 208 Xeroderma pigmentosum, vitamin D deficiency and, 349-350,
supplements for, 159, 162 361
in venous blood, 405b-406b X-linked hypophosphataemic rickets, 282-283, 292
Vitamin E (tocopherol), 209b X-linked immune deficiency, 24, 27
Vitamin K (phylloquinone, menaquinone), 209b, 275 X-linked inheritance, 16-17, 161, 20
deficiency, 263, 271
sources of, 205, 209, 209b y
Vitamin supplements, 383 Yaws, 83, 94
Vitreous detachment, posterior, 331, 3321, 333 Yellow fever, 89
Voltage-gated calcium channel 0/GCC), in Lambert-Eaton Yellow fever virus vaccine, 32, 34-35
myasthenic syndrome (LEMS), 305, 318 Yersinia enterocolitica infection, identification of, 82, 92-93
Volume of distribution, large, 38, 43
Voluntary reporting, of pharmacovigilance, 7, 10 z
Volutrauma, 71 Zanamivir, 89
von Willebrand disease, type 2a, 262, 270 Zero-order drug kinetics, in phenytoin, 6, 9
von Willebrand factor activity, reference range of, 409b-41 Ob Zidovudine, 101, 101 b
Voriconazole, 33, 35 Zika virus infection, management for, 74, 85
for aspergillosis, 167-168, 181 Zinc
in urine, 408b
w in venous blood, 405b-406b
Warfarin, 11 9, 143, 205, 359 Zollinger-EIIison syndrome, 232, 242
atrial fibrillation and, 261-262, 270
for CT scanning, 325, 327

downloaded from www.medicalbr.com

Vous aimerez peut-être aussi